Các chuyên đề bồi dưỡng học sinh giỏi Hình học 9

Tài liệu gồm 652 trang, tuyển tập các chuyên đề bồi dưỡng học sinh giỏi Hình học 9, giúp học sinh lớp 9 ôn tập để chuẩn bị cho kì thi chọn HSG môn Toán 9 cấp trường, cấp quận / huyện, cấp tỉnh / thành phố. Mời bạn đọc đón xem.

CHUYÊN ĐỀ BỒI DƯỠNG HỌC SINH GIỎI HÌNH HỌC 9
1 | THCS.TOANMATH.com
HH9-CHUYÊN ĐỀ 1. H THỨC LƯỢNG TRONG TAM GIÁC VUÔNG
I. H thc v cạnh và đường cao
KIN THỨC CƠ BN
Khi giải các bài toán liên quan đến cạnh đường cao trong tam giác vuông, ngoài vic nm vng
các kiến thc v định lý Thales, v các trường hp đồng dng ca tam giác, cn phi nm vng các
kiến thc sau:
Tam giác ABC vuông ti A, đường cao AH, hiu:

= = = = = =, , , , ,AB c BC a CA b AH h HB c HC b
. Ta có:
1)
=+
2 2 2
a b c
.
2)

==
22
. ; .b a b c a c
3)

=
2
.h b c
4)
=..a h b c
.
5)
=+
2 2 2
1 1 1
h b c
.
6)
=
2
2
bb
a
a
.
Chú ý: Din tích tam giác vuông:
Chng minh:
+ Xét các tam giác vuông AHB CHA, ta có:
=BAH HCA
(cùng ph vi
HAC
) suy ra
AHB CHA
(g.g) nên ta có:
= =
2
.
AH CH
AH BH CH
HB HA
.
+ Xét các tam giác vuông BHA BAC, ta có:
ABC
chung suy ra
BAH BAC
(g.g) nên ta có:
= =
2
.
BH BA
BA BH BC
BA BC
.
+ Tương tự ta có:
AHC BAC
(g.g) nên
= =
2
.
AC BC
CA CH CB
HC AC
.
+ Ta có:
( )
+
= = = = = +
2 2 2
2 2 2 2 2 2 2 2 2
1 1 1 1 1
. . 2
..
ABC
BC AB AC
AH BC AB AC S
AH AB AC AH AB AC AH AB AC
.
II. T s ng giác ca góc nhn
KIN THỨC CƠ BN
Cho tam giác ABC vuông ti A. Khi đó ta các góc B, C góc nhọn, đt

==,CB
thì

+ = 90
.
CHUYÊN ĐỀ BỒI DƯỠNG HỌC SINH GIỎI HÌNH HỌC 9
THCS.TOANMATH.com | 2
Xét góc:
ta thy: AB là cạnh đối ca góc
, AC gi là cnh k ca góc
.
1. Các t s ng giác ca góc nhn
(hình) được định nghĩa như sau:
= = = =sin ;cos ;tan ;cot
AB AC AB AC
BC BC AC AB
+ Nếu
là mt góc nhn thì:
0 sin 1;0 cos 1;tan 0;cot 0
2. Vi hai góc

,

+ = 90
,
Ta có:
==sin cos ;cos sin ;
==tan cot ;cot tan
Nếu hai góc nhn

=sin sin
hoc

=cos cos
thì

=
.
3.
+ = =
22
sin cos 1; .cot 1tg g
.
4. Vi mt s góc đặc bit ta có:
= = = =
12
sin30 cos60 ;sin45 cos45
22
= = = = = = = =
31
cos30 sin60 ;cot60 tan30 tan45 cot45 1;cot30 tan60 3
2
3
.
Vic chng minh các h thức khá đơn giản:
+ Ta có:

==sin ,cos
AB AB
BC BC
suy ra

= cossin
. Tương tự cho các trường hp còn li.
+ Ta có:
+
= = = = + = =
22
22
2 2 2 2
2
sin ,cos sin ,cos sin cos 1
AB AC AB AC AB AC
BC BC BC BC
BC
III. Mt s ví d tiêu biu
Ví d 1.
Cho tam giác ABC vuông ti A, dựng đường cao AH. Tính độ dài các cnh còn li ca tam giác ABC
trong mỗi trường hp sau:
a,
==
3
,
2
a
AB a AH
. b,
==
1
2,
4
BC a HB BC
.
c,
==
3
,
2
AB a CH a
. D,
==
3
3,
2
a
CA a AH
.
e,
==
3
,5
4
AB
BC a
AC
.
Gii:
CHUYÊN ĐỀ BỒI DƯỠNG HỌC SINH GIỎI HÌNH HỌC 9
3 | THCS.TOANMATH.com
a, Áp dng công thc:
=+
2 2 2
1 1 1
AH AB AC
Ta có:
= + = =
2 2 2 2
2
1 1 1 1 1
3
3
3
4
AC a
a AC AC a
a
.
b, Ta có tam giác OAB cân ti O,
= 2BC BO
= = 4BC BH BH BO OAB
cũng cân ti B. Hay
OAB là tam giác đều. Suy ra
= = = =
2 2 2 2 2 2
, 4 3AB a AC BC AB a a a
nên
= 3AC a
.
c, Áp dng công thc:
= = +
22
. ( )BA BH BC BA BH BH HC
hay
=+
22
3
.
2
a BH a BH
( )( )
+ = + = =
22
2 3 . 2 0 2 2 0
2
a
BH a BH a BH a BH a BH
. Vy
= =23BC a AC a
.
d, Áp dng công thc:
=+
2 2 2
1 1 1
AH AB AC
Ta có:
= + = = = + =
2 2 2 2
2 2 2 2
2
1 1 1 1 1
4
3
3
4
AB a BC AB AC a
AB a AB a
a
. Hay
= 2BC a
.
e, Đặt
==3 , 4AB k AC k
vi
+ = + = = =
2 2 2 2 2 2 2
0 (3 ) (4 ) 25 25k AB AC k k k BC a
suy ra
= = =3 , 4k a AB a AC a
.
Ví d 2.
Cho tam giác vuông ABC
= =90 , 2A BC a
, gi O là trung điểm ca BC. Dng
AH BC
a, Khi
=30ACB
. Tính độ dài các cnh còn li ca tam giác.
b, Khi
=30ACB
. Gi M là trung điểm ca AC. Tính độ dài BM.
c. Khi
=30ACB
. Các đoạn thng AO, BM ct nhau điểm G. Tính độ dài GC.
d. Gi s điểm A thay đổi sao cho
= =90 , 2BAC BC a
. Tam giác ABC phi tho mãn điều kin
để din tích tam giác AHO ln nht?
e. Gi s CG ct AB tại đim N. T giác AMON hình gì? Tam giác ABC phi tho mãn điều kin
gì để din tích t giác AMON ln nht?
Gii:
a, Khi
=30ACB
thì tam giác ABC tam giác nửa đều nên
==
1
2
AB BC a
,
= = = =
2 2 2 2 2 2
4 3 3AC BC AB a a a AC a
.
CHUYÊN ĐỀ BỒI DƯỠNG HỌC SINH GIỎI HÌNH HỌC 9
THCS.TOANMATH.com | 4
b, Theo câu a) ta có:
= = = + = + =
2
2 2 2 2 2
3 3 7
3
2 4 4
aa
AC a AM BM BA AM a a
=
7
2
a
BM
.
c. Do G là trng tâm ca tam giác ABC nên
=
2
3
CG CN
(vi N là trung điểm ca AB).
Áp dụng định Pitago ta có:
= + = + = =
2
2 2 2 2 2
13 13
3
4 4 2
aa
CN AN AC a a CN
. Suy ra
==
2 13
33
a
CG CN
.
d. Ta có:
( )
= + = = =
2
2 2 2 2
1 1 1
.
2 2 2 4
AHO
BC
S AH HO AH HO AO a
. Din tích tam giác AHO ln
nht khi và ch khi
=AH HO
. Tc là AHO vuông cân ti H. Suy ra

= = 22 30 , 77 30ACB ABC
e. T giác AMON là hình ch nht nên
= .
AMON
S AM AN
. Theo bất đẳng thc Côsi ta có:
+
2 2 2
2 . 2 .AM AN AM AN MN AM AN
.
==
2 2 2
MN OA a
nên
2
.
2
a
AM AN
. Vy
2
2
AMON
a
S
. Du bng xy ra khi ch khi
= =AM AN AB AC
, hay tam giác ABC vuông cân
ti A.
Ví d 3.
Cho tam giác ABC vuông ti A, k đường cao AH.
T H dng HM, HN lần lượt vuông góc vi AC,
AB.
a, Chng minh:
=
4
. . .CM CA BN BA AH
.
b, Chng minh:
=
3
..CM BN BC AH
.
c, Chng minh:
=
3
.
AH
AM AN
BC
.
d, Chng minh:
=
3
3
AB BN
CM
AC
.
e, Chng minh:
+=
2
..AN NB AM MC AH
.
f, Chng minh:
=+
3 3 3
2 2 2
BC BN CM
.
Gii:
a, Áp dng h thức lượng trong tam giác vuông AHB, AHC, ABC ta có:
CHUYÊN ĐỀ BỒI DƯỠNG HỌC SINH GIỎI HÌNH HỌC 9
5 | THCS.TOANMATH.com
= = =
2 2 2
. , . , .BN BA BH CM CA CH HB HC AH
, suy ra
( )
==
2
4
. . . .CM CA BN BA CH BH AH
.
b, Chú ý rng:
( )
==. . 2
ABC
AB AC AH BC S
t câu a) ta suy ra
= =
43
. . . . .CM BN AH BC AH CM BN BC AH
.
c, Ta có:
= = =
2 2 4
. , . . . .AM AC AH AN AB AH AM AN AB AC AH
, mt khác
( )
==. . 2
ABC
AB AC AH BC S
nên
=
3
..AM AN BC AH
.
d, Ta có:
= = =
2
22
2
.
. , .
.
BN BH CA
CM CA CH BN BA BH
CM
CH AB
(*) ta li có:
= = = =
44
2 2 2 2
22
. , .
BA CA
BH BC BA BH CH CB CA CH
BC BC
thay vào (*) ta suy ra
=
3
3
BN AB
CM
AC
.
e, Ta có:
= = + = +
2 2 2 2
. , . . .AN NB HN AM MC HM AN NC AM MC HN HM
. T giác ANHM
hình ch nht nên
+ = =
2 2 2 2
HM HN MN AH
hay
+=
2
..AN NB AM MC AH
.
f, Ta có:
= = = =
44
2 2 2 2
22
. , . ,
BH CH
CM CA CH BN BA BH BN CM
AB CA
. Li
=
2
.BA BH BC
n
suy ra
= = =
2 8 4 6
4
4 2 4
BA BA BH BA
BH BH
BC
BC BA BC
hay
=
6
2
4
BA
BN
BC
tương tự ta cũng có:
+
= + = + =
6 6 6 2 2
33
2 2 2
33
4 4 4
3
4
CA BA CA BA CA
CM BN CM
BC BC BC
BC
. Theo định lí Pitago ta có:
+=
2 2 2
BA CA BC
suy ra
+ = =
2
3 3 3
2 2 2
3
4
BC
BN CM BC
BC
.
Ví d 4.
Cho tam giác nhn ABC các đường cao AD, BE, CF ct
nhau ti H, gi O trung đim ca BC, I trung điểm ca
AH, K là giao điểm ca EF, OI biết
= 2BC a
.
a, Chng minh: Các tam giác IEO, IFO là tam giác vuông.
b, Chng minh: OI là trung trc ca EF.
c, Chng minh:
=
2
4.AH IK IO
.
d, Chng minh:
= cos
EF
A
BC
.
e, Chng minh:
=. . cos .cos .cos
EF FD ED
A B C
BC AC AB
.
f, Chng minh:
=
2
cos
AEF
ABC
S
A
S
.
CHUYÊN ĐỀ BỒI DƯỠNG HỌC SINH GIỎI HÌNH HỌC 9
THCS.TOANMATH.com | 6
g, Chng minh:
( )
= + +
2 2 2
1 cos cos cos
DEF
ABC
S
A B C
S
.
h, Chng minh:
=tan .tan
AD
BC
HD
.
i, Gi s
= = 60 , 45ABC ACB
. Tính
ABC
S
theo a.
j, Gi M là điểm trên AH sao cho
=90BMC
. Chng minh:
= .
BMC ABC BHC
S S S
.
Gii:
a, Do BE, CF là các đường cao ca tam giác ABC nên các tam giác AEH, AFH lần lượt vuông ti E,
F. Do I là trung điểm cnh huyn AH nên tam giác AIE cân ti I suy ra
=IEA IAE
(1), tam giác
OEC cân ti O nên
=OEC OCE
(2). Ly (1) + (2) theo vế ta có:
+ = + = 90IEA OEC IAE OCE
hay
=90OEI
. Tương tự ta cũng có
=90OFI
.
b, Do
= =
2
AH
IE IF I
nm trên trung trc ca EF,
==
2
BC
OE OF
nên O nm trên trung trc
ca EF suy ra OI là trung trc ca EF,
c, Do OI là trung trc ca EF nên
IO EF
ti K. Áp dng h thức lượng trong tam giác vuông
IEO ta có:

= =


2
22
. 4 .
2
AH
IK IO IE IK IO AH
.
d, Trong tam giác vuông AEB ta có:
=cos
AE
A
AB
, trong tam giác vuông AFC ta cũng có:
= =cos
AF AE AF
A
AC AB AC
, suy ra
= = cos
EF AE
AEF ABC A
BC AB
.
e, Tương tự như câu d) ta cũng có:
==cos , cos
FD ED
BC
AC AB
suy ra
=. . cos .cos .cos
EF FD ED
A B C
BC AC AB
.
f, Theo câu d) ta có:

= =


2
2
cos
AEF
ABC
S
AE
AEF ABC A
S AB
.
g, Ta có:
= = 1
ABC AEF BFD DFE
DEF AEF BFD DFE
ABC ABC ABC ABC ABC
S S S S
S S S S
S S S S S
. Tương tự như câu f) ta cũng có:
==
22
cos , cos
BFD DFE
ABC ABC
SS
BC
SS
suy ra
( )
= + +
2 2 2
1 cos cos cos
DEF
ABC
S
A B C
S
.
h, Ta có:
==tan ,tan
AD AD
BC
BD DC
suy ra
=
2
tan .tan
.
AD
BC
BD CD
, ta cn chng minh:
CHUYÊN ĐỀ BỒI DƯỠNG HỌC SINH GIỎI HÌNH HỌC 9
7 | THCS.TOANMATH.com
= =
2
..
.
AD AD
AD HD BD CD
BD CD HD
. Tht vy xét tam giác
BDH và tam giác ADC ta có:
= =180BHD DHE ACD
suy ra
BDH ADC
nên
=
DH BD
DC AD
hay
=..AD HD BD CD
đpcm.
i, Để tính din tích tam giác ABC ta cần tính đường cao AD
theo a.
Do tam giác ACD vuông cân ti D nên
=AD DC
(*). Do
=60ABC
nên:
= =tan60 . 3
AD
AD BD
BD
(**). Nhân (*) vi
3
ri cng vi (**) ta có:
( )
( )
( )
( )
+ = + = = = =
+
23
3 1 3 3 2 3 3 3 1
31
a
AD DC BD BC a AD a
.
Vy
( ) ( )
= =
2
1
. 3 3 1 .2 3 3
2
ABC
S a a a
.
j, Ta cn chng minh:
=
2
.
BMC ABC BHC
S S S
(***)
Áp dng công thc tính din tích các tam giác ta có:
=
1
.
2
BMC
S MD BC
,
=
1
.
2
ABC
S AD BC
,
=
1
.
2
BHC
S HD BC
Thay (***) thì điều cn chứng minh tương đương với
=
2
.MD AD HD
.
Áp dng h thức lượng trong tam giác vuông BMC ta có:
=
2
.MD DB DC
. Như vậy ta quy v chng
minh:
=..DB DC AD HD
. Xét tam giác BDH và tam giác ADC ta có:
= =180BHD DHE ACD
suy ra
BDH ADC
nên
=
DH BD
DC AD
hay
=..AD HD BD CD
đpcm.
Ví d 5.
Cho tam giác ABC
= = =,,BC a AC b AB c
. Chng minh rng:
a,
= +
2 2 2
2 .cosa b c bc A
.
b,
( )( )( )
= S p p a p b p c
(Công thc Heron) vi
++
=
2
a b c
p
.
c,
+ +
2 2 2
43a b c S
= = =
1 1 1
.sin .sin . sin
2 2 2
S ab C bc A ac B
.
CHUYÊN ĐỀ BỒI DƯỠNG HỌC SINH GIỎI HÌNH HỌC 9
THCS.TOANMATH.com | 8
d,
= = = 2
sin sin sin
a b c
R
A B C
(vi R là bán kính đường tròn ngoi tiếp tam giác ABC).
Gii:
a, Dựng đường cao BE ca tam giác ABC ta có:
Cách 1: Gi s E thuc cnh AC.
Ta có:
=+AC AE EC
. Áp dụng định lí Pitago cho các tam giác vuông AEB, BEC ta có:
= + = +
2 2 2 2 2 2
,AB AE EB BC BE EC
Tr hai đẳng thc trên ta có:
( )( ) ( )
= = + =
2 2 2 2
.c a EA EC EA EC EA EC b EA EC
=
22
ca
EA EC
b
ta cũng có:
+−
+ = =
2 2 2
2
b c a
EA EC b AE
b
.
Xét tam giác vuông AEB ta có:
+−
= = = +
2 2 2
2 2 2
cos 2 cos
2
AE b c a
A a b c bc A
AB bc
.
Cách 2: Xét tam giác vuông CEB ta có:
( )
= + = +
2
2 2 2 2
BC BE EC BE AC AE
= + +
2 2 2
2.BE AE AC AC AE
. Ta có:
= .cosAE AB A
suy ra
= + +
2 2 2 2
2 . .cosBC BE AE AC AC AB A
hay
= +
2 2 2
2 . .cosBC BA AC AC AB A
= +
2 2 2
2 cosa b c bc A
.
b, Ta gi s góc A là góc ln nht ca tam giác ABC
B, C là các góc nhọn. Suy ra chân đường
cao h t A lên BC là điểm D thuc cnh BC. Ta có:
=+BC BD DC
. Áp dụng định lý Pitago cho
các tam giác vuông ADB, ADC ta có:
= + = +
2 2 2 2 2 2
,AB AD DB AC AD DC
. Tr hai đẳng thc trên
ta có:
( )( ) ( )
= = + = =
22
2 2 2 2
.
cb
c b DB DC DB DC DB DC a DB DC DB DC
a
ta cũng
có:
+−
+ = =
2 2 2
2
a c b
DB DC a BD
a
. Áp dụng định lý Pitago cho tam giác vuông ADB ta có:
+ + +
= = +
2
2 2 2 2 2 2 2 2 2
22
2 2 2
a c b a c b a c b
AD c c c
a a a
( ) ( ) ( )( )( )( )
+ + + + + +
==
22
22
2
.
22
4
a c b b a c a b c a c b b a c b c a
aa
a
. Đặt
= + +2p a b c
thì
( )( )( )
( )( )( )
−−−
−−−
= =
2
2
16
2
4
p p a p b p c
p p a p b p c
AD AD
a
a
. T đó ta được
( )( )( )
= =
1
.
2
S BC AD p p a p b p c
c, T câu b) ta có:
( )( )( )
= S p p a p b p c
. Áp dng bất đẳng thc Côsi ta có:
CHUYÊN ĐỀ BỒI DƯỠNG HỌC SINH GIỎI HÌNH HỌC 9
9 | THCS.TOANMATH.com
( )( )( )

+ +
=


3
3
3 27
p a p b p c p
p a p b p c
. Suy ra
=
33
.
27
33
pp
Sp
.
Hay
( )
++
2
12 3
a b c
S
. Mt khác ta d chứng minh được:
( )
( )
+ + + +
2
2 2 2
3a b c a b c
. Suy ra
( )
++
+ +
2 2 2
2 2 2
3
43
12 3
a b c
S a b c S
. Dấu đẳng thc xy ra khi và ch khi tam giác ABC đều.
d, Ta có:
=
1
.
2
ABC
S AD BC
, trong tam giác vuông ABD ta có:
= =sin .sin
AD
B AD AB B
AB
thay
vào ta có
= = =
1 1 1
. . .sin .sin
2 2 2
ABC
S AD BC AB BC B ac B
. Tương tự cho các công thc còn li.
e, Dựng đường kính BK của đường tròn
( )
O
ngoi tiếp tam giác ABC thì
= = 90BAK BCK
= = = =OA OB OC OK R
.
Trong tam giác vuông BKC ta có:
==sin
2
BC a
BKC
BK R
.
Áp dng tính cht góc ngoài ca tam giác ta có:
= = + = +
11
2,
22
BOC BKC BAC BAO OAC AOK AOx
Hay
= = =
11
22
BAC KOx BOC BKC
. T đó suy ra:
==sin sin
2
a
ABC BKC
R
hay
= 2
sin
a
R
A
. Tương tự:
==2
sin sin
bc
R
BC
.
Chú ý: Vic dựng đường kính AK giúp ta tạo ra tam giác vuông để s dng t s ng giác góc
nhn,
=BAC BKC
là mt kết qu quen thuộc trong chương 2- Hình 9 (hai góc ni tiếp chn cùng
mt cung).
Nếu ch chng minh:
==
sin sin sin
a b c
A B C
. Ta làm đơn giản hơn như sau:
V
,,AD BC D BC DAB
=90D
nên
=sin ;
AD
B DAC
AB
=90D
nên
=sin
AD
C
AC
.
CHUYÊN ĐỀ BỒI DƯỠNG HỌC SINH GIỎI HÌNH HỌC 9
THCS.TOANMATH.com | 10
Do đó
= = =
sin
sin sin sin
B AC b b c
C AB c B C
. Chứng minh tương tự ta có
=
sin sin
ab
AB
Vy
==
sin sin sin
a b c
A B C
.
Ví d 6.
Cho tam giác ABC với các đnh A, B, C và các cạnh đối din với các đỉnh tương ứng là: a, b, c. Gi
D là chân đường phân giác trong góc A. Chng minh rng:
a, Chng minh:
=
+
BD a
AB b c
.
b, Chng minh:
+
sin
2
Aa
bc
.
c, Chng minh:
1
sin .sin .sin
2 2 2 8
A B C
d, Chng minh:



=
+
2 .cos
2
A
bc
AD
bc
.
Gii:
a, Áp dng tính chất đường phân giác trong ta có:
=
AB DB
AC DC
suy ra
= .
b
DC DB
c
nên:
( ) ( )
++
+ = + = =
b c b c
b
DB DC DB DB DB a DB
c c c
hay
= =
++
ac DB a
DB
b c AB b c
.
b, Dng
BH AD
thì
= =
+
sin
2
A BH BD a
AB AB b c
.
c, Áp dng kết qu các câu a, b ta có:
+ + +
sin .sin .sin
2 2 2
A B C a b c
b c c a a b
.
Theo bất đẳng thc
AM GM
ta có:
+ + + 2 , 2 , 2b c bc c a ac a b ab
nhân các BĐT
(có các vế dương) cùng chiều ta có:
( )( )( )
+ + + 8b c c a a b abc
suy ra
+ + +
1
8
a b c
b c c a a b
hay
1
sin .sin .sin
2 2 2 8
A B C
. Du bng xy ra khi và ch khi
==a b c
hay tam giác ABC đều.
c, Để chng minh bài toán ta cn kết qu sau:
+
=sin2 2sin .cos
.
CHUYÊN ĐỀ BỒI DƯỠNG HỌC SINH GIỎI HÌNH HỌC 9
11 | THCS.TOANMATH.com
+
=
1
sin
2
S ab C
.
*) Tht vy xét tam giác vuông ABC,
=90A
, gi O là trung
điểm ca BC, dựng đường cao AH.
Đặt

= = 2ACB AOB
.
Ta có:

= = = = = =sin sin ,cos cos
AH h AC b
CC
AC b BC a
= = = =
2
sin2 sin
2
AH h h
AMH
a
AM a
. T đó ta suy ra:
=sin2 2sin .cos
.
*)
=
1
sin
2
S ab C
(Xem ví d 5).
Tr li bài toán:
Ta có:

==


1
11
. sin . .sin
2 2 2
ABD
A
S AD AB A AD c
.

==


2
11
. sin . .sin
2 2 2
ACD
A
S AD AC A AD b
. Suy ra


= + = +



1
sin
22
ABC ACD ABD
A
S S S AD c b
. Mt khác
( )


= + = = =


+


+


2 cos
1 sin
2
sin sin sin
22
sin
2
ABC
A
bc
A bc A
S bc A AD c b bc A AD
cb
A
bc
.
Ngoài ra ta cũng có thể chng minh theo cách khác:
Dng
BH AD
, BH ct AC ti K thì tam giác ABK cân
ti A nên H là trung điểm ca BK.
Ta có:
= =cos .cos
22
A AH A
AH c
AB
,
Theo tính cht phân giác ta cũng có:
++
= =
AB DB AB AC DB DC
AC DC AC DC
=
+
.AC BC
DC
AB AC
hay
= =
++
ab DC b
DC
b c a b c
CHUYÊN ĐỀ BỒI DƯỠNG HỌC SINH GIỎI HÌNH HỌC 9
THCS.TOANMATH.com | 12
Như vậy ta cn chng minh:
=
2
.
DC
AH AD
a
Dng
//BE AD
(E nằm trên đường thng AC)
Suy ra
=2AH BE
nên ta ch cn ch ra
=..BE DC AD BC
, hay
=
BE BC
AD DC
nhưng điều này luôn đúng theo định lý Thales.
Chú ý rng: Ta chứng minh được kết qu sau:
= =
22
cos2 2cos 1 1 2sin
.
Tht vy xét tam giác vuông ABC,
=90A
, gi O là trung điểm ca BC, dựng đường cao AH. Đặt

= = 2ACB AOB
. Ta có:

= = = = = =cos cos ,sin sin
AC b AB c
CC
BC a BC a
,
+−

+
= = = = =


22
2
2
2 2 2 2 2
2
2
44
cos2 cos 1 2
2.
2.
22
aa
c
AO OB AB a c c
AOH
aa
AO OB a
a

= =


2
22
2
1 2. 2 1
a b b
a
a
. T đó suy ra
= =
22
cos2 2cos 1 1 2sin
.
Áp dng công thc:

= + = +


2 2 2 2 2 2 2
2 cos 2 2cos 1
2
A
a b c bc A a b c bc
.
( )
+−
+−
= + =
2
2
2 2 2
22
2cos 1 cos
2 2 2 4
b c a
A b c a A
ac bc
. Thay vào công thức đường phân giác ta có:
( )
( )( )
+−
+ + +
= = =
+ + +
2
2
2
2 cos
4
2
b c a
A
bc
bc
bc b c a b c a
bc
AD
c b b c b c
. Áp dng bất đẳng thc Côsi ta
có:
( )( )
( )
+ + +
+
=
22
b c a b c a
bc
bc AD p p a
, vi
= + +2p a b c
.
Áp dng công thc:
= +
2 2 2
2 cosa b c bc A
. Ta cũng chứng minh được h thc rt quan trng
trong hình hc phẳng (Định lí Stewart) đó là: “Cho điểm D nm trên cnh BC ca tam giác ABC khi
đó ta có:
( )
+ = +
2 2 2
. . .AB CD AC BD BC AB BD DC
+ Tht vây: Ta k
AH BC
không mt tính tng quát, ta gi s D nằm trong đoạn HC. Khi đó ta
có:
= +
2 2 2
2 . .cosAB AD BD AD BD ADB
= +
22
2.AD BD DB DH
(1)
CHUYÊN ĐỀ BỒI DƯỠNG HỌC SINH GIỎI HÌNH HỌC 9
13 | THCS.TOANMATH.com
Tương tự ta có:
=++
2 2 2
2.AC AD DC DH DC
(2). Nhân đẳng thc (1) vi DC đẳng thc (2) vi
BD ri cng li theo vế ta có:
( )
+ = +
2 2 2
. . .AB CD AC BD BC AB BD DC
.
Ví d 7.
Cho tam giác cân ABC,
= = = =20 , , ,A AB AC AC b BC a
. Chng minh rng:
+=
3 3 2
3a b ab
.
Gii:
V tia Bx sao cho
=20CBx
, Bx ct cnh AC ti D. V
⊥,AE Bx E Bx
. Xét
BDC
ABC
có:
20 ;CBD BAC BCD= =
chung nên
BDC ABC
do đó
BD BC DC
AB AC BC
==
BD BC a==
;
22
.;
BD a a
DC BC AD AC DC b
AB b b
= = = =
. Ta có:
ABE
vuông ti E
60ABE ABC CBD= =
nên
ABE
là nửa tam giác đều, suy ra
22
AB b
BE ==
2
b
DE BE BD a = =
.
ABE
vuông ti E, nên theo định lý Pitago ta có:
2 2 2 2 2 2 2
3
.
4
AE BE AB AE AB BE b+ = = =
ADE
vuông ti E, nên theo định lý Pitago ta có:
2
2
24
2 2 2 2 2 2 2 2 2
2
3 3 1
2
4 2 4 4
b a a
AE DE AD b a b b b ab a b a
bb


+ = + = + + = +




4
2 3 3 2
2
33
a
ab a a b ab
b
+ = + =
.
Chú ý: Nếu không dùng định lý Pi ta go ta cũng có thể áp dng công thc:
2 2 2 2 2 2 2
1
2 . .cos 2 . .
2
AD AB BD AB BD ABD AB BC AB BC c a ac= + = + = +
t đó dựa vào h
thc:
( ) ( )
( )
2
2
2 2 2
. . .BC AC DC AC AC AD AC AD AC BC= = =
ta cũng có được kết qu cn
tìm.
Ví d 8.
Tính
sin22 30 ,cos22 30 ,tan22 30
.
Gii:
Dng tam giác vuông cân ABC, không mt tính tổng quát ta đặt:
1, 90 2AB AC A BC= = = =
.
Gi AD là phân giác góc
B
, theo tính chất đường phân giác ta có:
CHUYÊN ĐỀ BỒI DƯỠNG HỌC SINH GIỎI HÌNH HỌC 9
THCS.TOANMATH.com | 14
1
21
21
AD AB AD AB
AD
DC BC AC BC AB
= = = =
+
+
. Áp dụng định lý Pitago
( ) ( )
2
2 2 2 2
1 2 1 4 2 2 2 2 2BD AB AD BD BD = + = + = =
( )
( )
( )
2
21
2 1 2 2
sin22 30
2
2 2 2 1
2 2 2
AD
BD
−−
= = = =
( )
1 2 2 sin22 30
cos22 30 ,tan22 30 2 1
2 cos22 30
2 2 2
AB
BD
+

= = = = =
Ví d 9.
Chng minh rng trong tam giác ABC,
( )
2
2A B a b b c= = +
.
Gii:
K đường phân giác AD, ta có:
,2CAD DAB ADC DAB B B A= = + = =
AB DA
ABC DAC cb aAD aBD
BC AC
= = =
.
BD AB BD AB ac
AD
CD AC BC AB AC b c
= = =
++
( )
2
ac
bc a a b b c
bc
= = +
+
.
Ví d 10.
Chng minh rng
51
sin18
4
=
.
Gii:
Dng tam giác cân ABC
( )
, 36 2AB AC A B C A= = = =
. Áp dng ví
d 2 ta có:
22
.AB BC AB BC=+
, chia hai vế cho
2
AB
, ta được
2
10
BC BC
AB AB

+ =


.
Gi I là trung điểm
18BC AI BC BAI CAI = =
2
sin18 4 2. 1 0
2 2 2
BI BC BC BC
AB AB AB AB

= = + =


2
4sin 18 2sin18 1 0 + =
. Giải phương trình ta tìm được
51
sin18
4
=
(do
sin18 0
).
CHUYÊN ĐỀ BỒI DƯỠNG HỌC SINH GIỎI HÌNH HỌC 9
15 | THCS.TOANMATH.com
Bài toán tương tự: Cho tam giác MNP cân ti M và có góc
36NMP =
. Tính t s
NM
NP
. (Trích đề
tuyn sinh vào lp 10 chuyên Toán TP Hà Nội năm 2011-2012).
Ta chng minh b đề sau:
Cho tam giác ABC
2ABC ACB=
.
Chng minh:
22
.AC AB AB BC=+
Tht vy: Dng phân giác trong BD ca tam giác ABC ta có:
BDC là tam giác cân t đó suy ra
2ADB DBC ABC==
suy ra
ABC ADB
(g.g)
2
.
AB AD
AB AD AC
AC AB
= =
(1). Theo tính chất đường phân giác ta cũng có:
.AD AB AD AB AB AC
AD
DC BC AD DC AB BC AB BC
= = =
+ + +
thay vào (1) ta có:
22
.AC AB AB BC=+
.
Có th biến đổi theo cách:
AB AC BC
ABC ADB
AD AB DB
= =
suy ra
2
. , . . .AB AC AD AB BC AC DB AC DC= = =
(do
DB DC=
).
T đó ta có:
( )
22
.AB AB BC AC AD DC AC+ = + =
Tr li bài toán: Tam giác MNP cân ti M
36NPM =
suy ra
2N P M==
. Áp dng b đề ta có:
22
.AB BC AB BC=+
. Chia hai vế cho
2
AB
suy ra:
2
2
2
1 1 0
BC BC BC BC
AB AB AB AB

= + + =


15
2
BC
AB
−+
=
Ví d 11.
Chng minh
6 2 6 2
cos15 ,sin15
44
+−
= =
.
Gii:
Dng tam giác vuông ABC:
90 , 30AB= =
. Gi s
1 2, 3AC BC AB= = =
Dng phân giác BD:
3 3 3
2 3 3
2
2 3 2 3
AD AB AD
AD
DC BC AD DC
= = = = =
+
++
( )
2 2 2 2
3 12 9 12 3 12 2 3BD AB AD BD= + = + + =
( )
( )
2 3 3 1
6 3 1
2
BD
= =
CHUYÊN ĐỀ BỒI DƯỠNG HỌC SINH GIỎI HÌNH HỌC 9
THCS.TOANMATH.com | 16
( )
3 3 1 6 2
cos15
4
22
6 3 1
AB
BD
++
= = = =
( )
( )( )
( )
3 2 3 3 1
2 3 3 6 2
sin15
4
6 3 1
6 3 1
AD
BD
−+
−+
= = = =
.
Ví d 12.
Chng minh rng
15
cos36
4
+
=
.
Gii:
Dng tam giác cân ABC
()AB AC=
108A=
, lấy điểm D trên BC sao
cho
CD CA=
. Ta có:
CAD
cân
72 108ADC ADB ABC DAB = =
( )
2 2 2
..
AB DA
AB BC DA BC BC AB AB BC BC AB
BC AB
= = = =
. Chia hai vế cho
2
AB
được:
2
1 5 1 5
1 0 cos36
2 2 4
BC BC BC BC
AB AB AB AB
++

= = = =


.
Ví d 13.
Chng minh các h thc:
1.
22
tan 36 tan 72 10 + =
2.
44
tan 36 tan 72 90+ =
.
Gii:
22
2
222
sin 36 1 cos 36 1
tan 36 1
cos 36 cos 36 cos 36
= = =

. S dng
( )
( )
22
2
8 3 5
1 5 1 8
cos 36 2 3 5 tan 36 5 2 5
4 cos 36 4
35
+
= = = = =
+
.
Tương tự,
2
2
1
cot 1
sin
=−
, thay
51
sin18
4
=
tính được
2 2 2 2
cot 18 5 2 5 tan 72 tan 36 tan 72 5 2 5 5 2 5 10 = + = + = + + =
.
( )
2
4 4 2 2 2 2
tan 36 tan 72 tan 36 tan 72 2tan 36 tan 72 90+ = + =
.
Ví d 14.
Cho tam giác ABC, có
60A=
và đường phân giác AD. Chng minh rng:
1 1 3
AB AC AD
+=
.
CHUYÊN ĐỀ BỒI DƯỠNG HỌC SINH GIỎI HÌNH HỌC 9
17 | THCS.TOANMATH.com
Gii:
Dng tam giác ABC,
60A=
, AD là phân giác trong
30A BAD CAD = =
.
K
1
,
2
DH AC DK AB AKD AHD DH DK AD = = =
1 1 1
. .sin60 . sin30 . .sin30
2 2 2
ABC ABD ADC
S S S AB AC AB AD AC AD= + = +
hay
3 1 1 1 1 3
...
2 2 2
AB AC AB AD AC AD
AB AC AD
= + + =
. Cũng có thể gii nhanh bng cách áp dng
công thức tính đường phân giác trong AD.
Ví d 15.
Chng minh rng trong tam giác ABC,
60A=
khi ch khi
2 2 2
a b c bc= +
;
120A=
khi và ch khi
2 2 2
a b c bc= + +
.
Gii:
H BH vuông góc vi AC.
1 1 3 3
60 30 ,
2 2 2 2
AB c
A ABH AH AB c BH= = = = = =
. Trong
BHC
ta có:
2
2
2 2 2 2 2 2 2
3
42
cc
a BC BH HC b b c bc

= = + + = +


. Trường hp
120A=
chứng minh tương
t.
Ví d 16.
Tính độ dài các đường trung tuyến ca tam giác, biu th qua ba cnh ca tam
giác y.
Gii:
Gi AD là trung tuyến thuc cnh
BC DB DC=
K
2 2 2
AH BC AD AH HD = +
2 2 2 2
AD AB BH HD = +
(1)
Tương tự,
2 2 2 2
AD AC CH HD= +
(2)
Cng (1) và (2) theo tng vế ta được
2 2 2 2 2 2
22AD AB AC BH CH HD= + +
( )
2 2 2 2 2 2
2 2 . 2AD AB AC BH CH BH HC HD= + + + +
( )( )
2 2 2 2 2
2 2 2AD AB AC BC BD HD DC HD HD= + + + +
( )
2 2 2 2 2 2 2
2 2 2AD AB AC BC BD HD HD= + + +
CHUYÊN ĐỀ BỒI DƯỠNG HỌC SINH GIỎI HÌNH HỌC 9
THCS.TOANMATH.com | 18
( )
2 2 2 2 2 2 2 2 2 2 2 2
1 1 1 1
2
2 2 2 4
a
AD AB AC BC BC AB AC BC m b c a= + + = + = +
.
Hoàn toàn tương tự ta tính được độ dài các đường trung tuyến còn li (dành cho bạn đọc).
T các h thức này, ta suy ra: trong hình bình hành, đ dài các cnh a, b và hai đường chéo m, n. Ta
có:
( ) ( )
2 2 2 2 2 2 2 2
11
2;
42
c
m n a b b m a b+ = + = +
.
Ví d 17.
Cho tam giác ABC. Chng minh rằng các đường trung tuyến k t B C vuông góc vi nhau khi
và ch khi
2 2 2
5b c a+=
.
Gii:
Gi BM, CN là hai đường trung tuyến
22
24
39
BG BM BG BM = =
( )
2 2 2 2 2 2 2
4 1 1 2 1
9 2 4 9 9
BG a c b a c b


= + = +




. Tương tự,
( )
2 2 2 2
21
99
CG a b c= +
. Khi đó.
( ) ( )
2 2 2 2 2 2 2 2 2 2 2 2 2
2 1 2 1
5
9 9 9 9
BM CN BG CG BC a c b a b c a b c a + = + + + = + =
.
Ví d 18.
Cho tam giác ABC
( )
,,BC a CA b AB c= = =
. Trung tuyến AD, đường
cao BH phân giác CE đồng quy. Chng minh đẳng thc:
( )
( )
2 2 2 2
2a b a b c ab+ + =
.
Gii:
Xét tam giác vuông BHC:
( )
2 2 2 2 2 2
CH BC BH BC AB AH= =
( )
2
2 2 2 2 2
BC AB AH BC AB CA CH= + = +
2 2 2
2.BC CA AB CACH + =
Tương tự,
2 2 2 2 2 2
2 2 2
2
CA AB BC CH BC CA AB
AH
CA AH CA AB BC
+ +
= =
+−
(1)
CE là phân giác ca tam giác ABC, AD, BH, CE đồng quy
CO
là đường phân giác ca
2
OD CD BC
ADC
OA CA CA
= =
(2). T D k đường thng
//
2
HC
DK AC BH DK HK =
2
OD HK CH
OA HA HA
= =
(3). T (1), (2), (3)
2 2 2
2 2 2
BC CA AB BC
CA AB BC CA
+−
=
+−
2 3 2 2 2 3
BC CA CA AB CA CA BC AB BC BC + = +
CHUYÊN ĐỀ BỒI DƯỠNG HỌC SINH GIỎI HÌNH HỌC 9
19 | THCS.TOANMATH.com
( )
3 3 2 2 2 2 2
2.BC CA BC CA CA BC AB CA AB BC BC CA + + + =
( )
( )
( )
( )
2 2 2 2 2 2 2 2
2 . 2BC CA BC CA AB BC CA a b a b c ab + + = + + =
.
Ví d 19.
Cho tam giác ABC tho mãn
24A B C==
. Chng minh rng:
1 1 1
a b c
+=
.
Gii:
Gi H là trung điểm ca BC, qua H dựng đường thng vuông góc vi BC ct AB kéo dài ti D
DBC
là tam giác cân
B BCD=
.
Theo gi thiết,
2 4 7 180A B C C= = =
Đặt
180
, 2 , 4
7
C B A
= = = =
,
33DAC B C ACD BDC
= + = = =
.
Do đó
CAD
cân
AB AB
CA CD BD
AC BD
= = =
(1).
CA là phân giác ca góc
AB AD
BCD
AC CD
=
(2). Cng (1) vi (2) tng vế được:
1 1 1 1 1 1
1
AB AB AB AD AB AD
AC BC BD CD BD AC BC AB a b c
+
+ = + = = + = + =
.
Ví d 20.
Cho tam giác vuông ABC
( )
90A=
, đường cao AH. Đ dài các cnh ca tam giác là các s nguyên
tho mãn
1 1 1
1
AB AC AH
+ + =
. Xác định các cnh ca tam giác.
Gii:
Đặt
,,AB a AC b AH h= = =
, ta có:
1 1 1
1
a b h
+ + =
bh ah ab abh + + =
. Tam giác ABC vuông
ab ch=
22
c a b=+
22
bh ah ch abh a b c ab a b a b ab + + = + + = + + + =
( )
2 2 2 2
2 2 0ab a b a b a b ab a b ab = + + + =
( )
22
2 2 2 0 2 2 2 0 2
22
ab
ab ab a b ab a b b
aa
+ = + = = = +
−−
.
ab là các s nguyên nên 2 chia hết cho
2 2 2aa =
hoc
21a−=
.
CHUYÊN ĐỀ BỒI DƯỠNG HỌC SINH GIỎI HÌNH HỌC 9
THCS.TOANMATH.com | 20
Trường hp:
2 2 2
1 1 1 25 12
2 2 4 3 5
144 5
a a b c h
h a b
= = = = = + = =
. Thay vào tha
mãn
4, 3, 5AB AC BC = = =
.
Trường hp:
2 1 3 4 5a a b c = = = =
. Vy c hai trường hp tam giác có các cnh 3,4,5.
Ví d 21.
Cho tam giác, tha mãn
2 3 180BC+ =
. Chng minh rng
22
.BC BC AC AB=+
.
Gii:
Ta viết li biu thc cn chng minh thành:
( )
2 2 2
.BC BC AC AB BC BC AC AB = =
. Trên
BC lấy điểm D sao cho
CD AC=
. Khi đó biểu thc cn chng minh tr thành:
2
.BC DB AB=
ta
nghĩ đến vic chng minh:
CBA ABD
, tht vy, ta có:
( )
180 2 3
180 180 180 180
22
C B C C
ADB ADC B C A
+
= = = = + =
đpcm.
CHUYÊN ĐỀ 2.ĐƯNG TRÒN DÂY CUNG TIP TUYN CỦA ĐƯỜNG TRÒN
I. TÓM TT LÝ THUYT
1. Tp hợp các điểm M cách điểm O cho trước mt khoảng không đổi R
là đường tròn tâm O bán kính R. Kí hiu
( )
;OR
.
2. Đưng kính và dây cung
+ Đoạn thng nối 2 điểm nằm trên đường tròn đi qua tâm của đường tròn gọi đường kính ca
đường tròn đó.
+ Đoạn thng nối 2 điểm bt k nằm trên 1 đường tròn gi là 1 dây của đường tròn đó.
Các tính cht cn nh:
a. Nếu điểm M nm trên
( )
O
đường kính AB thì
90AMB =
, đảo li: Nếu
90AMB =
(vi A, B c
định) thì điểm M nằm trên đường tròn đường kính AB.
b. Trong cácy cung ca một đường tròn, đường kính là dây cung ln nht.
c. Trong một đường tròn, đường vuông góc vi một dây thì đi qua trung điểm của dây đó.
d. Trong một đường tròn, đường kính đi qua trung điểm ca một dây không đi qua tâm thì vuông góc
vi dây đó.
e. Trong một đường tròn: Hai dây bằng nhau thì cách đều tâm; hai dây cách đều tâm thi bng nhau.
f. Trong 2 y ca một đường tròn: Dây nào lớn hơn thì gần tâm hơn; dây nào gần tâm hơn thì dây đó
lớn hơn.
3. Tiếp tuyến của đường tròn. Đường thẳng đi qua một điểm của đường tròn vuông góc vi bán kính
đi qua điểm đó thì đường thng đó là một tiếp tuyến của đường tròn.
Như vậy:
+ Nếu một đường thng một đường tròn ch một điểm chung
thì đường thẳng đó là một tiếp tuyến của đường tròn.
+ Nếu khong cách t tâm đường tròn đến đường thng bng bán
kính của đường tròn thì đường thẳng đó là tiếp tuyến của đường tròn.
+ Qua một điểm ngoài đường tròn ta luôn k đưc 2 tiếp tuyến đến
đường tròn đó.
Mt s tính cht cn nh đổi vi 2 tiếp tuyến ct nhau:
T M nm ngoài
( )
O
dng các tiếp tuyến MA , MB đến
( )
O
( A , B là các tiếp điểm ).
AB ct MO tại H , đoạn thng MO ct
( )
O
tại điểm I. Khi đó ta có:
+ Tam giác MAB cân ti M .
+ MO là phân giác ca
AMB
.
+ MO vuông góc vi AB ti trung điểm H ca AB .
+ I là tâm đường tròn ni tiếp tam giác MAB .
4. Quan h đường thẳng và đường tròn.
Để xét qua h một đường thng
( )
d
với đường tròn
( )
; OR
ta phi da vào khong cách t tâm O
của đường tròn đến đường thng.
+ Nếu khong cách t tâm O đến đường thng
( )
d
lớn hơn bán kính thì đường thng không cắt đường
tròn.
+ Nếu khong cách t m O đến đường thng
( )
d
bằng bán kính thì đường thng tiếp xúc với đường
tròn (lúc này đường thng
( )
d
gi là tiếp tuyến của đường tròn).
+ Nếu khong cách t tâm O đến đường thng
( )
d
nh hơn bán kính thì đường thng cắt đường tròn
tại 2 điểm phân bit .
+ T điểm M nm ngoài
( )
O
ta dng tiếp tuyến MA, MB đến
( )
O
và dng cát tuyến MCD. Khi đó ta có:
2 2 2 2
MA MB MO R= =
.
5. Quan h 2 đường tròn.
Để xét quan h ( v trí tương đối ) của 2 đường tròn
( )
11
; OR
( )
22
; OR
ta da vào khong cách
12
OO
, và các bán kính
12
RR
.
+ Hai đường tròn ct nhau khi và ch khi :
11 2 1 22
OO| | + R R R R
.
+ Hai đường tròn tiếp xúc nhau :
2 21 1
| | O = ORR
hoc
21 12
= OO + RR
.
+ Hai đường tròn không giao nhau :
21 12
> OO + RR
hoc
11 22
O | O | RR−
.
II. MT S BÀI TP TNG HP V TIP TUYN, CÁT TUYN, DÂY CUNG CỦA ĐƯỜNG
TRÒN.
Ví d 1.
Cho nửa đường tròn
( )
; OR
. Trên cùng mt na mt phng b AB,
dng các tiếp tuyến Ax, By ca nửa đường tròn. Ly một điểm M trên na
đường tròn
( )
O
. Tiếp tuyến ti M ca
( )
O
ct Ax, By lần lượt ti D, C tia
AM, BM kéo dài ct By, Ax lần lượt ti F, E.
a, Chứng minh: Các điểm D, M, O, A cùng nm trên một đường tròn, c điểm C, M, O, B cùng nm
trên một đường tròn.
b. Chng minh:
COD
vuông.
c. D là trung điểm AE.
d.
CBO BAE
.
e. Chng minh:
2
. AD BC R=
,
AD BC CD+=
.
f. Dng MH vuông góc vi AB. Chứng minh: AC, BD đi qua trung điểm I ca MH.
g. Chng minh:
EO AC
.
h. Tìm v trí điểm M để din tích tam giác MHO ln nht.
i, Tìm v trí điểm M để din tích tam giác MAB ln nht,
j. Tìm v trí điểm M để chu vi tam giác MAB ln nht.
k. Tìm v trí điểm M để din tích t giác ABCD nh nht.
l. Tìm v trí điểm M để chu vi t giác ABCD nh nht.
Li gii:
a. DA, DM các tiếp tuyến ca
( )
O
nên
90DMO DAO= =
, suy ra 4 điểm D, M, O, A nm
trên đường tròn đường kính DO. Hoàn toàn tương t ta các điểm C, M, O, B nằm trên đường tròn
đường kính CO.
b. Theo tính cht hai tiếp tuyến ct nhau ta có: OC, OD lần lượt phân giác ca các góc
MOA
,
MOB
nên
( )
1
90
2
COD MOC MOD BOM COM= + = + =
hay tam giác COD vuông ti O.
c. Do điểm M nằm trên đường tròn đường kính AB nên
90 90AMB EMA= =
. Cũng theo tính
cht hai tiếp tuyến ct nhau ta có:
DA DM=
nên
DAM DMA=
90 90 DAM DMA DEM DME DM DE = = =
. Vy
DE DA DM==
hay D
trung đim của AE. Cũng thể chng minh theo cách ch ra OD đưng trung bình ca tam giác
EAB.
d. Xét tam giác CBO tam giác BAE ta có:
90CBO BAE= =
. Theo tính cht hai tiếp tuyến ct
nhau ta có:
BM CO
nên
COB BEA=
cùng ph vi
( )
. EBA CBO BAE g g
.
e. Theo tính cht hai tiếp tuyến ct nhau ta có:
AD DM=
,
. .BC CM AD BC DM CM= =
. Mt
khác tam giác COD vuông ti O OM đường cao nên theo h thức lượng trong tam giác vuông ta
có:
22
. CM DM OM R==
. Vy
2
. AD BC R=
= = AD BC CM DM CD++
.
f. Gi s BD ct MH tại I. Theo đnh Thales ta có:
IM IB IH IM IH
DE DB AD DE AD
= = =
DE DA IH IM= =
hay I trung đim ca HM. Chứng minh tương tự ta cũng AC đi qua trung
điểm I ca MH tc là MH, BD, AC đồng quy ti I.
Chú ý: Ta cũng thể chng minh bng cách dùng B đề hình thang: “Cho hình thang ABCD hai
cnh bên AB, CD ct nhau tại M, hai đường chéo ct nhau ti N. Gọi E, F trung điểm ca 2 cnh
đáy BC, AD. Khi đó 4 điểm M, E, N, F cùng nm trên một đường thẳng”
Tht vy: Gi s MN ct BC, AD tại E, F theo định lý Thales ta có:
( )
= 1
BE CE
AF DF
(cùng bng
ME
MF
).
( )
= 2
BE CE
DF AF
(cùng bng
NE
NF
). Nhân hai đẳng thc
( )
1
,
( )
2
Ta có:
22
.AF .
BE CE
DF AF DF
=
suy ra
BE CE=
thay
( )
1
vào ta có:
AF DF=
( đpcm ).
g. Theo chng minh câu d) ta có:
1
2
2
AB
AE BO AE
BAE CBO
AB BC AO BC
= =
hay
AE AB
OAE CBA OE AC
AO BC
=
.
Chú ý: Nếu AC ct
( )
O
ti K t vic chng minh:
OE AC
ta suy ra
90EAO EKO EAO EKO= = =
ta cũng suy ra EK là tiếp tuyến ca
( )
O
.
h. Tam giác MOH vuông ti H nên ta có:
( )
22
22
11
.
2 2 2 4 4
MHO
MH HO
OM R
S MH HO
+
= = =
Du bng xy ra khi và ch khi
MH HO=
nên tam giác MHO vuông cân ti H. Tc là M nm trên na
đường tròn sao cho OM to vi AB mt góc
45
.
i. Ta có
11
. .2 .
22
MAB
S MH AB MH R R MH= = =
. Trong tam giác vuông MHO ta có:
MH MO R=
nên
2
MAB
SR
. Du bng xy ra khi ch khi
HO
,
MH AB
. Hay M điểm
chính gia ca cung AB.
j. Chu vi tam giác MAB kí hiu là
2 p
thì
2 2p MA MB AB MA MB R= + + = + +
.
Để ý rng
( )
( )
2
2
2 2 2
2 . 2 8MA MB MA MB AB R+ + = =
suy ra
2 2MA MB R+
. Suy
ra
( )
2 2 2 2 2 2 1 p R R R + = +
. Du bng xy ra khi ch khi
MA MB=
, hay M điểm
chính gia ca cung AB.
k. Ta có:
11
). .2 . .
22
(
ABCD
S AD BC AB R CD R CD= + = =
. Do
2CD AB R=
nên
2
2
ABCD
SR
.
Du bng xy ra khi và ch khi
CD AB=
hay
/ /CD AB
khi đó M là điểm chính gia ca cung AB.
l. Chu vi t giác ABCD bng q:
2 2 2q AD CD BC AB CD AB CD R= + + + = + = +
2 2CD AB R=
nên chu vi t giác ABCD:
2 2 6q CD R R= +
. Du bng xy ra khi ch khi
CD AB=
hay
/ / CD AB
khi đó M là điểm chính gia ca cung AB.
Ví d 2.
Xét đường thng
( )
d
c định ngoài
( )
; OR
(khong cách t O đến
( )
d
không nh n
2R
).
T một điểm M nm trên đường thng
( )
d
ta dng các tiếp tuyến MA, MB đến
( )
; OR
(A, B là các
tiếp đim) dng các tuyến MCD (tia MC nm gia hai tia MO, MA
MC MD
). Gi E trung
điểm của CD, H là giao điểm ca AB và MO.
a. Chứng minh: 5 điểm M, A, E, O, B cùng nm trên một đường tròn.
b. Chng minh:
2 2 2
. MC MD MA MO R= =
.
c. Chng minh: Các tiếp tuyến ti C, D của đường tròn
( )
; OR
ct nhau ti một điểm nằm trên đường
thng AB.
d. Chứng minh: Đường thẳng AB luôn đi qua một đim c định.
e. Chng minh: Một đường thẳng đi qua O vuông góc vi MO ct các tia MA, MB lần lượt ti P, Q.
Tìm GTNN ca
MPQ
S
.
f. Tìm v trí điểm M để AB nh nht.
Li gii:
a. MA, MB các tiếp tuyến ca
( )
O
nên
90MAO MBO= =
.
E là trung đim ca CD nên
90MEO =
. T
đó suy ra 5 điểm M, A, E, O, B cùng nm trên
đường tròn đường kính MO.
b. Ta có:
( )( )
. MC MD ME EC ME ED= +
.
ED EC=
nên ta suy ra
( )
. - )( MC MD ME EC ME EC=+
2 2 2 2 2
- - - ME EC MO EO EC==
( )
2 2 2 2 2 2 2 2 2 2
= - MO EC EO MO OC MO R MO OA MA + = = = =
đpcm.
c. Gi s các tiếp tuyến ti C, D ca
( )
O
ct nhau F. Theo a) ta đã chứng minh 5 điểm M, A, E, O, B
nằm trên đường tròn đường kính MO, suy ra 4 điểm A, E, O, B cũng nằm trên đường tròn đường kính
MO. Hoàn toàn tương tự ta có: C, H, O, D cùng nằm trên đường tròn đường kính OF suy ra
90FHO =
hay
FH HO
, mt khác ta cũng
AH HO⊥
F, A, H thng hàng. Nói cách khác: Các
tiếp tuyến ti CD ct nhau tại điểm F nằm trên đường thng AB.
d. Dng
( )
d OK
thì K đim c định OK độ dài không đổi. Gi s AB ct OK tại đim I
thì
( )
OHI OKM g g
suy ra
..
OI OM
OH OM OI OK
OH OK
= =
. Mt khác theo h thc ng
trong tam giác vuông MAO ta
22
. OH OM OA R==
suy ra
2
. OI OK R=
hay
2
R
OI
OK
=
suy ra OI
không đổi, I nằm trên đường thng OK c định, suy ra điểm I c định. Vậy đường thẳng AB luôn đi qua
điểm I.
e. Ta có:
( )
2 O .
MPQ MOP
S S A MP R MA AP= = = +
. Theo bất đẳng thc
AM GM
ta có:
2 . MA AP MA AP+
. Theo h thức ng trong tam giác vuông MOP thì
22
. MA AP OA R==
T đó suy ra
2
2
MPQ
SR
. Du bng xy ra khi ch khi
MA AP=
hay tam giác MOP vuông n.
Suy ra MAOB là hình vuông, tc là
2MO R=
.
f. Ta có
22
2 2 AB AH R OH= =
nên AB nh nht khi và ch khi OH ln nht.
Để ý rng:
2
R
OI
OK
=
2OK R
nên
2
2
R
OI
nên điểm I luôn nằm trong đường tròn
( ; )OR
.
Trong tam giác vuông OHI ta có:
OH OI
nên OH ln nht bng OI khi và ch khi
HI
hay
MK
.
Cũng thể lp lun theo cách khác:
2 2 2
2
.
2 2 2 2 1
MA OA MO R R
AB AH R R
MO MO MO
= = = =
nên AB
nh nht khi và ch khi MO nh nht. Hay M là hình chiếu vuông góc ca O lên
( )
d
.
Ví d 3.
Cho nửa đường tròn tâm
( )
O
đường kính B điểm A trên nửa đường tròn
( )
O
(A khác B, C). H
AH vuông góc vi BC (H thuc BC). I, K lần lượt đối xng vi H qua AB, AC. Đưng thng IK tia
CA ct tiếp tuyến k t B ca
( )
O
lần lượt ti M, N. Gọi E là giao điểm của IH và AB, F là giao đim
KH vi AC.
a. Chng minh: I, A, K thng hàng. IK là tiếp tuyến ca
( )
O
.
b. Chng minh:
2 2 2
1 1 1
BH AB AN
=+
.
c. Chứng minh: M là trung điểm ca BN và MC, AH, EF đồng quy.
d. Xác định v trí điểm A trên nửa đường tròn để din tích t giác BIKC ln nht.
c. Chng minh:
2
. . BE CF BC AH=
.
f. Tiếp tuyến ti C của đường tròn
( )
O
ct IK ti P.
Chng minh:
NO PB
.
g. Chng minh:
AO EF
.
h. Gi Q, R lần lượt giao điểm ca OM, OP vi AB,
AC. Xác định tâm tính bán kính đường tròn ngoi tiếp
t giác MPRO biết
30ACB =
.
Li gii:
a. Vì I, K là các điểm đối xng vi H qua AB, AC nên
2 IAH HAB=
,
2PAH HAC=
. Suy ra
2 2 180IAH PAH HAB HAC+ = + =
nên I, A, K thẳng hàng . Ngoài ra ta cũng có:
AH AI AK==
nên tam giác IHK vuông ti H. T tính chất đối xng ta có:
90AIB AHB= =
90AKC AHC= =
nên
//BI KC
suy ra t giác BCKI hình thang. Nên OA đường trung bình
ca hình thang BCKI suy ra
OA KI
, nói cách khác KI là tiếp tuyến ca
( )
O
ti A .
b. Dng
AD BN
thì
2 2 2
1 1 1
AD AN AB
=+
. Chú ý rng ADBH là hình ch nht nên
AD BH=
. T đó
suy ra
2 2 2
1 1 1
BH AN AB
=+
(Đpcm).
c. Do MA, MB là tiếp tuyến ca
( )
O
nên
MA MB=
nên
MAB MBA=
, mt khác ta có:
90 MNA ABN=
,
90 - MAN MAB=
suy ra
MNA MAN=
hay tam giác MAN cân ti M.
Suy ra
MN MA MB==
hay M là trung điểm ca NB.
d. Ta chứng minh: MC đi qua trung đim ca AH . Tht vy gi s MC ct AH tại J, theo định lý Thales
ta có:
JA JC JH
MN CM MB
==
M trung đim ca NB nên
MN MB=
suy ra
JA JH=
hay J trung
điểm ca AH. Chú ý rng: T giác AEHF hình ch nht nên AH, EF ct nhau tại trung điểm J ca
mỗi đường, nói cách khác AH, EF, MC đồng quy ti J.
c. Theo tính chất đối xng ta có:
IH AB
ti E,
IK AC
ti F. Áp dng h thức lượng trong các tam
giác vuông AHB, AHC ta có:
2
. BE BA BH=
,
2
.CF CA CH=
suy ra
22
. . . .BE CF AB AC BH CH=
.
Trong tam giác vuông ABC ta cũng có:
2
. BH CH AH=
,
( )
. . 2
ABC
AB AC AH BC S==
nên suy ra
4 3
. . . . .BE CF AH BC AH BE CF BC AH==
f. Ta d chứng minh được:
( )
. NBC OCP g g
suy ra
1
2
2
CB
NB OC NB
BC CP BO CP
= =
hay
NB CB
NBO BCP
BO CP
=
suy ra
NO BP
.
g. Do AEHF là hình ch nht nên
AFE FEH AHE ABC= = =
t đó suy ra
90AFE OAC ABC OCA+ = + =
hay
.
h. Gọi X trung điểm của QR, Z trung điểm của MP. Đường
trung trc ca QR cắt đường trung trc ca MP ti Y thì Y chính
là tâm đường tròn đi qua các điểm M, P, R, Q.
Tương tự câu g) ta thy
OZ QR
nên
// XY OZ
Li
// OX YZ
nên t giác XYZO hình bình hành suy
ra
OZ XY=
, gi thiết
30ACB =
30MPO CPO ACB = = =
,
3OA R AP R= =
.
2
.OA MA AP=
nên
2
3 4 2 3
. 3 3
3 3 3
RR
R MA R MA MP R OZ XY= = = = =
,
11
2 2 2
R
QX QR OA= = =
. Áp dụng định lý Pitago cho tam giác vuông QXY ta có:
2
2 2 2 2
2 2 2
2 3 4 19
4 3 4 3 12
R R R R R
QY QX XY

= + = + = + =



suy ra
57
6
R
QY =
. Vậy đường tròn đi qua các
điểm M, P, R, Q có bán kính là
57
6
R
QY =
.
Ví d 4.
Cho tam giác nhn ABC ni tiếp
( )
O
các đường cao AD, BE, CF ct nhau ti H. Gi M trung
điểm BC, I trung điểm AH. Dựng đường kính AK ca
( )
O
. Gọi P giao điểm th 2 ca AH vi
( )
O
(P khác A).
a. Chứng minh: 4 điểm A, E, H, F nm trên một đường tròn,
b. Chng minh: IM là đường trung trc ca EF.
c. Chng minh: H, K, M thng hàng t đó suy ra
OA EF
.
d. Chng minh: ME, MF là các tiếp tuyến của đường tròn ngoi tiếp tam giác AEF,
e. Chứng minh: P đối xng vi H qua BC. T đó suy ra đường tròn ngoi tiếp tam giác HBC đưng
tròn ngoi tiếp tam giác ABC có cùng bán kính.
f. Gọi N giao điểm th 2 của đường tròn ngoi tiếp tam giác ABC với đường tròn ngoi tiếp tam giác
AEF (N khác A). Chng minh:
2
.
4
BC
MH MN =
.
Li gii:
a. Do BE, CF là các đường cao ca tam giác ABC nên
90AEH AFH= =
suy ra 4 điểm A, E, H, F nằm trên đường
tròn tâm I đưng kính AH ta gi là
( )
I
.
b. Do
90BEC BFC= =
suy ra 4 điểm B, F, E, C cùng nm
trên đường tròn tâm M đường kính BC ta gi là
( )
M
. Vì
( )
I
,
( )
M
ct nhau theo y cung EF nên theo tính chất hai đường
tròn cắt nhau ta có: IM là đường trung trc ca EF.
c. Do AK là đường kính ca
( )
O
nên
90 AACK C KC=
mt khác
// BH AC BH KC⊥
. Tương tự
// CH BK
nên t giác BHCK
hình bình hành suy ra hai đưng chéo BC, HK ct nhau tại trung đim mỗi đường. Nói cách khác ta
có H, M, K thng hàng. T đó suy ra OM là đường trung bình ca tam giác AHK nên
1
/ / //
2
OM AH BOM AI= =
nên t giác AOMI hình bình hành. Suy ra
//AO MI
,
OAMI EF EF
.
d. Ta thy rằng đường tròn ngoi tiếp tam giác AEF cũng chính đường tròn
( )
I
ngoi tiếp t giác
AEHF. Tam giác AIE cân ti I nên
( )
1 IEA IAE=
, tam giác MEC cân ti M nên
( )
2 MEC MCE=
. Ly
( )
1
+
( )
2
theo vế ta có:
I 90IEA MEC AE MCE+ = + =
hay
90MEI =
. Nói cách khác ME tiếp tuyến ca
( )
I
, hoàn toàn tương t ta cũng MF tiếp tuyến
ca
( )
I
.
e. Do AK đường kính ca
( )
O
nên
90 // APK PK DM=
Mtrung đim HK nên MD
đường trung bình của tam giác HPK suy ra D trung đim ca HP. Hay P, H đối xng nhau qua BC.
Ba đim B, P, C nằm trên đường tròn
( )
O
nên theo tính chất đối xứng ta 3 đim B, H, C cũng nằm
trên đường tròn
( )
' O
đối xng vi
( )
O
qua BC. Nên hai đường tròn này có cùng bn kính.
f. Vì N nằm trên đường tròn
( )
I
đường kính AH nên
90ANH =
, N cũng nằm trên
( )
O
đường kính
AK nên
90ANK =
suy ra K, H, N thẳng hàng. K, M, H cũng thng hàng nên suy ra M, H, N
thng hàng. Hay M, H, N là mt cát tuyến ca
( )
I
. Theo tính cht quen thuc cát tuyến và tiếp tuyến ta
có:
2
. MH MH ME=
(xem câu b) Ví d 2). Mặt khác ta cũng có
2
.
24
BC BC
ME MF MH MN= = =
Ví d 5.
Cho hai đường tròn
( )
11
; OR
,
( )
22
; OR
( )
12
R R
ct nhau ti A, B.
a. Nêu cách dng tiếp tuyến chung ngoài của 2 đường tròn khi biết
12
OO
.
b. Gi M, N 2 tiếp điểm ca 1 tiếp tuyến chung ngoài của hai đường tròn
( )
11
; OR
,
( )
22
; OR
.
Tính MN theo
12
OO
,
12
RR
.
c. Gi s AB ct MN ti D. Chng minh:
DM DN=
.
Li gii:
a. Dng
( )
11
; OR
bán kính
1
OM
. Dng tiếp tuyến
( )
d
qua M ca
( )
O
. Lấy điểm I trên
( )
d
ni
1
IO
, trên
tia đi ca
1
O I
lấy điểm
2
O
, sao cho
12
OO
bằng độ dài đã cho. Dựng
( )
2
O Nd
ti N. V đường tròn
( )
22
; O O N
ta được
( )
d
tiếp tuyến chung ca 2 đưng tròn
( )
d
tiếp xúc vi
( )
11
; OR
,
( )
22
; OR
lần lượt ti M, N.
b. Gi s
12
RR
k
12
OC O N
ti C ta có
1
MNCO
là hình ch nht nên
( )
2
1 1 2 2 1 1 2
22
2
2
MN O C OO O C O O R R= = =
.
c. Theo tính cht cát tuyến, tiếp tuyến ta có:
22
. DM DA DB DN==
suy ra D là trung điểm ca MN.
Ví d 6.
Cho 2 đường tròn
( )
11
; OR
,
( )
22
; OR
tiếp xúc ngoài ti A. Dng tiếp tuyến chung ngoài ca
( )
11
; OR
,
( )
22
; OR
ti A là
( )
d
. Đường tròn tâm O đường kính
12
OO
, ct
( )
d
ti I. Đưng tròn
( )
; I IA
ct
( )
11
; OR
,
( )
22
; OR
lần lượt ti M , N khác A .
a. Chng minh : MN là mt tiếp tuyến chung ngoài ca
( )
11
; OR
,
( )
22
; OR
.
b. K đường kính NP ca
( )
2
O
. Chng minh: M, A, P thng hàng.
Li gii:
a. T gi thiết ta có:
IA IM IN==
dn ti
1 1 2
90IMO IAO INO= = =
(hs t cm)
Suy ra IM, IN lần lượt các tiếp tuyến ca
( )
11
; OR
,
( )
22
; OR
nên
11
OMIO IA=
22
ONIO IA=
.
Suy ra
1 2 1 2
2O 2 2 180MIA NIA IA O IA O IO+ = + = =
hay M, I, N
thng hàng. Tc là MN là tiếp tuyến chung ca
( )
11
; OR
,
( )
22
; OR
.
b. Do MN là đường kính ca
( )
; I IA
nên
90MAN =
, ta cũng có
90NAP =
suy ra
90 90 180MAN NAP+ = + =
hay M, A, P thng hàng.
Ví d 7.
Cho đoạn thng AB k tia
Bx AB
. Trên tia Bx ly O sao cho
2
BA
BO =
. Tia AO ct
( )
; O OB
ti
D và E (D nm gia A, O). Đường tròn
( )
; A AD
ct AB C.
a. Chng minh:
2
.AEDE AD=
.
b. Chng minh:
2
. AC CB AB=
.
c. Tia BD ct
( )
; A AD
ti P. Một đường thng qua D ct
( )
; A AD
ti M và ct
( )
O
ti N. Chng minh:
DPM DBN
.
Li gii:
a. Ta có:
( )
. = ( ) AD AE AO OD AO OE−+
( )
)( AO OD AO OD= +
2 2 2 2
AO OD AO OB= =
( )
2
2
2
2 AB OB DE= = =
b. Ta có:
( )
. - . CB AB AB AC AB=
2
. . .AB AC AB AD AE AD AB = =
( )
22
- AAD AE AB AD C= = =
c. Xét các tam giác:
DPM
,
DBN
ta có:
PDM BDN=
đối đnh, các cp tam giác cân ODN, ADM
và ODB, ADP đồng dng vi nhau
( )
. gg
t đó suy
( )
. . DPM DBN c g c
đpcm .
d 8. Cho tam giác nhn ABC ni tiếp (O) AB < AC. Đường phân giác ca góc BAC ct
(O) ti D khác A. Gi M trung điểm AD và E điểm đối xng vi D qua tâm O. Gi s
đưng tròn ngoi tiếp tam giác ABM cắt đoạn thng AC tại điểm F khác A.
a) Chng mình rng tam giác BDM và tam giác BFC đồng dng.
b) Chng minh
EF AC
.
(Tuyn sinh lớp 10, THPT chuyên ĐHKHTN Hà nội, năm học 2013 - 2014)
Li gii
a) Ta có
AFB AMB=
(
1
2
=
AB
của đường tròn ngoi tiếp
ABM
).
BMD BFC=
.
Li có
1
2
BDM BCF==
AB
BDM BCF
(g.g).
b) b)
=
BD DM
BDM BCF
BC CF
.
D thy
BDC
COE
là các tam giác cân.
1
2
DBC OEC==
CD
DBC OEC
.
DB OE OD
BC CE CE
= =
nên
DO DM
CE CF
=
1
2
MDO FCE==
AE
OMD EFC
(c.g.c)
90
o
EFC OMD = =
hay
EF AC
.
Ví d 9. Cho đoạn thẳng AC có đ dài bng a. Trên đoạn AC lấy điểm B sao cho AC = 4AB.
Tia Cx vuông góc vi AC tại điểm C, gi D là một điểm bt k thuc tia Cx (D không trùng
vi C). T đim B k đưng thng vuông góc vi AD cắt hai đường thng AD CD ln
t ti K, E.
a) Tính giá tr DC, CE theo a.
b) Xác định v trí điểm D để tam giác BDE có din tích nh nht.
c) Chng minh rằng khi điểm D thay đổi trên tia Cx thì đường tròn đường kính DE luôn
có mt dây cung c định.
(thi hc sinh gii lp 9, tỉnh Vĩnh Phúc, năm học 2014 2015)
Li gii
a) Ta có:
EBC ADC=
(Cùng bù vi góc
KBC
);
90ACD ECB= =
ACD ECB
(g-g)
. . .
DC AC
DC CE AC BC
BC EC
= =
Do
4
a
AB =
;
2
33
. . .
44
aa
BC DC EC AC BC= = =
b)
1
.
2
BDE BDE
S BC DE S

=
nh nht khi và ch khi DE nh nht.
Ta có:
2
3
2 . 2 3
4
a
DE DC EC DC EC a= + = =
(Theo chng minh phn a)
Du
3
" " .
2
a
DC EC= = =
( )
BDE
S
nh nht bng
2
33
8
a
khi D thuc tia Cx sao cho
3
.
2
a
CD =
c) Gi giao điểm của đường tròn đường kính DE với đường thng AC M, N (M nm gia A
và B)
M, N đối xng qua DE.
Ta có:
AKB ACD
(g-g)
..
AK AB
AK AD AC AB
AC AD
= =
(1)
AKM AND
(g-g)
..
AK AM
AK AD AM AN
AN AD
= =
(2)
T (1) và (2) suy ra
2
..
4
a
AM AN AC AB==
( )( )
2
22
4
a
AC MC AC NC AC MC = + =
(Do
MC NC=
)
2
2
33
42
aa
MC MC NC = = =
M, N là hai điểm c định.
Vậy đường tròn đường kính DE luôn có dây cung MN c định.
Ví d 11. Cho đường tròn tâm O mt dây AB của đường tròn đó. Các tiếp tuyến v t A và
B của đường tròn ct nhau ti C. Gi D một điểm trên đường tròn đưng kính OC (D
khác A B). CD ct cung AB của đường tròn (O) ti E. (E nm gia C D). Chng minh
rng:
a)
BED DAE=
.
b)
2
..DE DA DB=
Li gii
a) Ta có:
;EBC EAB DCB DAB==
nên
EBC DCB EAB DAB+ = +
.
Mt khác:
EBC DCB BED+=
,
EAB DAB DAE+=
Vy
BED DAE=
.
b) Ta có:
ADE ABC CAB EDB= = =
Mà theo câu a):
BED DAE=
, suy ra:
2
.
DE DB
BED EAD DE DA DB
DA DE
= =
.
d 12. Cho tam giác ABC ni tiếp đường tròn (O). I trung điểm của BC, M điểm trên
đoạn CI (M khác C I), đường thng AM cắt đường tròn (O) tại điểm D. Tiếp tuyến ca
đưng tròn ngoi tiếp tam giác AMI ti M cắt đường thng BD, DC lần lượt ti P và Q. Chng
minh rng
..DM IA MP IC=
và tính t s
MP
MQ
.
Li gii
DMP AMQ AIC==
ADB BCA=
nên
MDP ICA
(g.g)
..
DM MP
DM IA MP IC
CI IA
= =
.
; 180
o
ADC CBA DMQ AMQ= =
180
o
AIM BIA= =
nên
DMQ BIA
(g.g)
..
DM MQ
DM IA MQ IB
BI IA
= =
(1)
T
. . . .DM IA MP IC DM IA MP IB= =
(2)
T (1) và (2) suy ra
1
MP
MQ
=
.
CHUYÊN ĐỀ BỒI DƯỠNG HỌC SINH GIỎI HÌNH HỌC 9
1 | THCS.TOANMATH.com
CHUYÊN ĐỀ 3. GÓC VỚI ĐƯỜNG TRÒN
A.TRNG TÂM CẦN ĐẠT
I. GÓC TÂM S ĐO CUNG
KIN THỨC CƠ BN
Góc tâm: Là góc có đỉnh trùng vi tâm của đường tròn.
1. S đo cung:
+ S đo cung nhỏ bng s đo góc ở tâm chắn cung đó.
+ S đo cung lớn bng hiu gia 360° s đo cung nh (có chung 2
mút vi cung ln)
2. So sánh 2 cung:
+ Hai cung được gi là bng nhau nếu chúng có s đo bằng nhau.
+ Trong 2 cung, cung nào có s đo lớn hơn gọi là cung lớn hơn.
Trên hình 1: Góc
AOB
gi là góc tâm chn bi cung nh
AB
(hay
AmB
).
Ta có:
AmB AOB, AnB AOB= = 360
.
3. Đim thuc cung tròn:
Nếu C là một điểm nm trên cung AB thì
sñAB sñAC sñCB=+
.
II. LIÊN H GIA CUNG VÀ DÂY CUNG
1. Vi 2 cung nh trong một đường tròn hay hai đường tròn bng nhau:
+ Hai cung bằng nhau căng hai dây bằng nhau.
+ Hai dây bằng nhau căng 2 cung bằng nhau.
2. Vi 2 cung nh trong một đường tròn hay hai đường tròn bng nhau:
+ Cung lớn căng dây lớn hơn.
+ Dây lớn căng cung lớn hơn.
Trên hình 2:
AB CD AB CD= =
.
AB EF AB EF
.
III. GÓC NI TIP
1. Góc ni tiếp góc đnh nằm trên đường tròn hai cnh
cha 2 dây cung của đường tròn.
2. Trong một đường tròn, s đo góc nội tiếp bng na s đo cung
b chn.
+ Trong một đường tròn: Các góc ni tiếp bng nhau
chn các cung bng nhau.
+ Các góc ni tiếp chn cùng mt cung hoc chn cung bng nhau
CHUYÊN ĐỀ BỒI DƯỠNG HC SINH GII HÌNH HC 9
THCS.TOANMATH.com | 2
thì bng nhau.
+ Góc ni tiếp (nh hơn hoặc bng 90°) có s đo bằng na s đo góc ở tâm chn cùng mt cung.
+ Góc ni tiếp chn nửa đường tròn là góc vuông.
Trên hình 3:
+ Các góc
AMB, ANB
là góc ni tiếp cùng chn cung
AB AMB ANB AB AOB = = =
11
22
.
+
AB BC AMB BPC= =
.
IV. GÓC TO BI TIA TIP TUYN VÀ DÂY CUNG
1. Đưng thng xy là tiếp tuyến ca
( )
O
tại điểm A.
Tiếp điểm A là gc chung của 2 tia đối nhau Ax, Ay,
AB mt y cung ca
( )
O
. Khi đó góc
xAB,yAB
các góc
to bi tia tiếp tuyến và dây cung AB.
2. S đo góc tạo bi tia tiếp tuyến y cung bng na s đo
cung b chn.
3. Trong một đường tròn: Góc to bi tia tiếp tuyến dây cung góc ni tiếp chn cùng mt
cung thì bng nhau.
Trên hình 4: Ta có:
xAB AB AMB; yAM AM ABM= = = =
11
22
.
V. GÓC ĐỈNH BÊN TRONG ĐƯỜNG TRÒN, GÓC
ĐỈNH BÊN NGOÀI ĐƯỜNG TRÒN
1. Cho hai dây cung AB, CD ca
( )
O
ct nhau ti một điểm M
nằm trong đường tròn
( )
O
. Khi
BMC
gọi góc đỉnh nm
trong đường tròn
( )
O
. Ta định lý: S đo góc đỉnh nm
trong đường tròn bng na tng s đo hai cung bị chn. Tc là:
(
)
BC s DBMC ñA=+
1
2
2. Hai tia CD, BA ct nhau ti N (AB, CD là hai dây cung ca
( )
O
). Khi đó góc
BNC
gi là góc có
đỉnh nằm ngoài đường tròn
( )
O
.
Định lí: S đo góc có đỉnh nằm ngoài đường tròn bng na hiu s đo hai cung bị chn.
Tc là:
(
)
BC s DBNC ñA=−
1
2
VI. CUNG CHA GÓC
CHUYÊN ĐỀ BỒI DƯỠNG HỌC SINH GIỎI HÌNH HỌC 9
3 | THCS.TOANMATH.com
1. Cho 2 điểm c định A, B . Qu tích những điểm M nhìn đoạn thẳng AB cho trưc mt góc
AMB
=
không đổi
( )
0 180
hai cung tròn đối xng nhau qua
AB, gi cung chứa góc α dựng trên đoạn thẳng AB. Đc bit, qu tích
các điểm M nhìn đoạn AB dưới một góc vuông đường tròn đường
kính AB.
2. Cách dng cung chứa góc α.
+ Dựng đường trung trc (d) của đoạn thng AB .
+ Dng tia Ax to vi AB một góc α.
+ Dng tia
Ay Ax
ct (d) tại điểm O.
+ Ta O chính tâm của đường tròn chứa cung α dựng trên đon
thng AB.
MT S BÀI TP VN DNG
Ví d 1
Cho tam giác nhn ABC(AB < AC) 3 trc tâm H ni tiếp đường tròn
( )
O
và ngoi tiếp đường
tròn
( )
I
. Gọi E là trung điểm của AH , M là trung điểm BC. Tia phân giác ca
BAC
cắt đường tròn
( )
O
ti K (khác A), ct EM Q.
a) Chng minh:
KB KC KI==
b) Chng minh
AQH =90
.
c) Gọi D giao điểm ca phân giác góc A vi BC. Dng tiếp tuyến
AN ca
( )
K,KB
. Chng minh:
ND AK
Li gii
a) Do AK là phân giác trong góc A nên
KB KC KB KC= =
. Ta s
chng minh tam giác KIC cân ti K.
Xét tam giác KIC ta có:
KIC IAC ICA A C= + = +
11
22
(1)
Ta cũng có:
=+KCI KCB ICB
.
KCB KAB A==
1
2
(góc ni tiếp
cùng chn cung KB),
ICB C=
1
2
suy ra
11
22
=+KCI A C
(2).
T (1) và (2) ta suy ra
KIC KCI=
Hay tam giác KIC cân ti K tc là KI = KC.
Vy KI = KC = KB.
CHUYÊN ĐỀ BỒI DƯỠNG HC SINH GII HÌNH HC 9
THCS.TOANMATH.com | 4
Chú ý: Điểm K chính là tâm đường tròn ngoi tiếp tam giác IBC.
b) D thy O, M, K thng hàng. K đường kính AS ca
( )
O
thì
SC AC,SB AB⊥⊥
li
BH AC, CH AB
BH // SC, CH // SB suy
ra t giác BHCS hình bình hành nên H, M, S thng hàng M trung
điểm ca HS .
Ta
HAQ AKO OAK==
(cp góc so le tam giác AOK cân). Li
EQA QAS=
(so le). T đó suy ra
EAQ EQA=
suy ra
EA EQ EA EQ EH= = =
. Tc là tam giác AQH vuông ti Q.
c)
KBC KAC=
(cùng chn cung KC),
KAC KAB=
suy ra
KBC KAB=
suy ra

D ABKB K
nên
KD KB KN
KB KA KA
==
(do KB = KN).
Hay
KD KN
KB KA
=
kết hp vi
DKN NAK=
suy ra

DNK NAK
suy ra
NDK ANK= = 90
đpcm.
Ví d 2
Cho tam giác ABC nhn ni tiếp
( )
O; R
, đường cao AD kéo dài ct
( )
O
ti E (E khác A). Dng
đường kính AK .
a) Chng minh:
ABE CAK=
t đó suy ra BCKE là hình thang cân.
b) Gi H là trc tâm ca tam giác ABC. Chứng minh: H đối xng vi E qua BC.
c) Chng minh:
a b c
R
sin A sin B sinC
= = = 2
vi AB = c, BC =a ,CA = b.
d) Tiếp tuyến ti A của đường tròn
( )
O
ct tia CB ti
M, phân giác trong góc A ct BC ti F. Chng minh:
Tam giác AMF cân.
e) Đường thng qua A vuông góc vi MO ct
( )
O
ti
Q . Đoạn thng MO ct
( )
O
ti I. Chng minh: I
tâm đường tròn ni tiếp tam giác AMQ.
Li gii
a) Do AK là đường kính ca
( )
O
nên
AKC =90
, ta có:
BAE ABC= 90
ABC AKC=
(cùng chn cung AC)
nên
BAE ABC AKC CAK= = =90 90
.
CHUYÊN ĐỀ BỒI DƯỠNG HỌC SINH GIỎI HÌNH HỌC 9
5 | THCS.TOANMATH.com
= = BAE CAK BE CK
EK // BC hay BCKE là hình thang cân.
b) Do
BH AC
nên
EAC HBC=
(cùng ph vi góc
ACB
). Mặt khác ta cũng có:
EAC EBC=
(cùng chn cung EC) nên suy ra
HBC EBC=
∆HBD = ∆EBD (g.c.g) suy ra DH =DE, hay H đối
xng vi E qua BC .
Ta cũng có thể chng minh theo cách khác:
KC AC,KB AB⊥⊥
li có
BH AC,CH AB
BH // KC, CH // KB suy ra t giác BHCK nh bình hành nên BC ct
HK tại trung điểm N ca mỗi đường. Do
AEK =90
nên EK // DN mà N là trung điểm HK nên ND
đường trung bình ca tam giác HEK suy ra D trung điểm của HE hay H đối xng vi E qua
BC.
c) Ta có:
ABC AKC=
(cùng chn cung AC). Trong tam giác vuông AKC ta có:
AC b b b
sin AKC sin B R
AK R R sin B
= = = = 2
22
, tương tự ta có
a
R
sin A
= 2
,
c
R
sinC
= 2
hay
a b c
R
sin A sin B sinC
= = = 2
d) Ta
=+MAF MAB ABF
MAB ACB=
(tính cht góc to bi tiếp tuyến và dây cung). Suy
ra
MAF BAC ACB=+
1
2
, ta cũng
AFM FAC FCA BAC ACB= + = +
1
2
suy ra
MAF AFM=
hay tam giác AMF cân ti M .
e) Theo tính chất đối xứng ta suy ra MQ cũng là tiếp tuyến ca
( )
O
, MO trung trc ca AQ nên
IA IQ=
. Ta
MAI AQI=
(tính cht góc to bi tiếp tuyến dây cung). Mặt khác ta cũng có:
IAQ AQI=
nên suy ra
MAI IAQ=
hay AI là phân giác ca góc MAE . Theo tính cht 2 tiếp tuyến
cắt nhau ta cũng có: MI phân giác của góc AMQ. Suy ra I là tâm đường tròn ni tiếp tam giác
AMQ.
Ví d 3
Cho tam giác đều ABC ni tiếp đường tròn
( )
O
bán kính R, điểm M chuyển động trên cung nh
BC.
a) Chng minh: MA = MB + MC .
b) Tìm GTLN, GTNN ca P = MA + MB + MC.
c) Tìm giá tr nh nht ca biu thc:
Q
MB MC
=+
11
Li gii:
a) Áp dụng định lý Ptolemy (*) cho t giác ABMC ta có:
CHUYÊN ĐỀ BỒI DƯỠNG HC SINH GII HÌNH HC 9
THCS.TOANMATH.com | 6
=+MA.BC AB.MC AC.MB
.
Do AB = BC = CA
Suy ra MA = MB + MC.
Cách khác: Trên AM lấy điểm E sao cho MB = ME (1).
Thế thì tam giác BME cân ti M, mt khác ta có:
BME BCA= = 60
(cùng chn cung AB) suy ra tam giác BME đều.
Xét tam giác BEA và tam giác BMC ta có:
BE = BM do tam giác BME đều.
AB = BC (do ABC đều).
ABE EBC CBM= =60
suy ra ∆ABE = CBM
AE = MC (2).
T (1) và (2) suy ra MA = MB + MC
b) Ta có P = MA + MB + MC = 2MA . P nh nht khi và ch khi
MB
hoc
MC
.
P ln nht khi và ch khi MA là đường kính ca
( )
O
.
c) Vi mi s thực dương x, y ta có:
( )
x y xy. .
x y xy

+ +


1 1 1
22
. Nên ta suy ra
x y x y
+
+
1 1 4
, du bng
xy ra khi và ch khi x = y.
Áp dng vào bài toán ta có:
MB MC MB MC MA
+ =
+
1 1 4 4
Do
MA R 2
nên ta suy ra
R
MA MB R
+ =
1 1 4
2
2
. Du bng xy ra khi
và ch khi AM là đường
kính ca
( )
O
. Hay M là điểm chính gia cung nh BC.
Định lý Ptolemy (*):
Định lý Ptolemy cho t giác ni tiếp: Cho t giác ABCD ni tiếp trong đường tròn
( )
O
. Khi đó ta
có:
AB.CD AD.BC AC.BD+=
.
Chng minh:
Trên đường chéo BD lấy điểm E sao cho
DAE BAC=
.
Ta có
DAE BAC=
ADE ACB=
(cùng chn
AB
) nên

A ACBDE
(g.g)
AD DE
AC BC
=
AD.BC AC.DE=
(1).
Do
DAE CAB=
nên
DAC EAB=
, li có
ABE ACD=
(cùng chn
AD
)

AB ACDE
(g.g)
CHUYÊN ĐỀ BỒI DƯỠNG HỌC SINH GIỎI HÌNH HỌC 9
7 | THCS.TOANMATH.com
AB BE
AB.CD AC.BE
AC CD
= =
(2).
T (1) và (2) suy ra
( )
AB.CD AD.BC AC BE DE AC.BD+ = + =
.
Ví d 4
Cho tam giác nhn ABC ni tiếp
( )
O
, ngoi tiếp
( )
I
, các đường
thng AI, BI,CI ct
( )
O
tại giao điểm th 2 X,Y Z. Các đưng
thng XY lần lượt ct AB, BI, BC lần t M, N, P. XZ ct IC ti
Q.
a) Chng minh: Tam giác BMP cân.
b) Chng minh: I là trc tâm ca tam giác XYZ.
c) Chng minh: NQ song song vi BC .
Li gii:
a) Do AI, BI,CI các đường phân giác góc A, B, C nên X, Y, Z các điểm chính gia ca các
cung BC, CA, AB.
Ta có:
(
)
1
2
=+BMP AY BX
(
)
1
2
=+BPM BY CX
AY BY,CX BX==
Suy ra
BMP BPM=
hay tam giác BMP cân ti B.
b) Ta có:
(
)
(
)
(
)
1 1 1
90
2 2 4
= + = + + = + + = BNX BX YZ BX AY AZ BC AB AC
suy ra
XY BZ
Chứng minh tương tự ta cũng có
⊥⊥AX YZ,CY XZ
suy ra I là trc tâm ca tam giác XYZ.
c) T chng minh câu b) ta suy ra
INX IQX= = 90
nên 4 đim I, N, X, Q nằm trên đường tròn
đường kính XI. Suy ra
IQN IXN=
(cùng chn cung IN ).
Mặt khác ta cũng có:
IXN AXY YCB==
(Hai góc ni tiếp chn cung bng nhau) t đó suy ra
IQN YCB=
NQ // BC.
Ví d 5
Cho tam giác nhn ABC trc tâm H ni tiếp
( )
O
, đường cao AD. Gi M, N lần lượt trung
điểm ca BC, AB. Gi I, K lần lượt là hình chiếu vuông góc của B, C lên đường thng AO.
a) Chng minh: I, D đối xng nhau qua MN.
b) Đường thng IM ct AC ti E . Chng minh:
BE AC
.
CHUYÊN ĐỀ BỒI DƯỠNG HC SINH GII HÌNH HC 9
THCS.TOANMATH.com | 8
c) Chng minh: Khi B, C c định A di chuyển trên đường tròn
( )
O
thì tâm đường tròn ngoi tiếp
tam giác DIK là một điểm c định.
d) Chng minh: Khi B, C c định A di chuyển trên đường tròn
( )
O
hãy tìm giá tr ln nht ca
HA HB HC++
.
Li gii
a) Do N trung điểm ca AB nên
ON AB
. Các điểm N, I, M cùng nhìn BO mt góc bng 90°
nên suy ra 5 đim B, N, O, I, M nằm trên đường tròn đường kính BO. Ta cũng
AIB ADB= = 90
nên 4 điểm A, I, D, B cùng nằm trên đường tròn tâm N đường kính AB.
Ta có:
IBD IND=
1
2
(Liên h góc ni tiếp và góc tâm). Mt khác
=IBD IBM INM
(cùng chn
cung
IM
). Suy ra
INM IND=
1
2
nói cách khác NM phân giác góc N ca tam giác cân IND suy
ra
MN DI
tại trung điểm ca DI. Hay MN trung trc ca ID.
b) Dng
BE AC
. Ta chng minh: E, I, M thng hàng. Ta thy 4 điểm A, E, I, B nằm trên đường
tròn tâm N đường kính AB. Suy ra
EIA EBA=
(1) (cùng chn cung EA).
Ta
AIM AIB BIM BNM BAC= + = + = +90 90
(do
BIM BNM=
cùng chn cung BM
MN//AC) (2). T (1) (2) suy ra
AIM EIA BAC EBA+ = + + = 90 180
hay E, I, M thng hàng
(đpcm).
c) Gọi P trung điểm ca AC. Chứng minh tương tự a) ta PM
cũng trung trực ca DK . T đó suy ra tâm đường tròn ngoi tiếp
tam giác DIK chính là trung điểm M ca AC.
d) Dựng đường kính AS thì S, H, M thng hàng (tính cht quen
thuc, xem thêm d 1) suy ra
AH OM= 2
. Do BC c định nên
OM không đổi suy ra AH không đổi. Đặt
BOC
= 2
.
Trên tia đi ca HB ta lấy điểm F sao cho HF = HC khi đó HB +
HC =BF .
Để ý rng:
(
)
(
)
HFC BHC A
= = =
1 1 1
180 180
2 2 2
không
đổi. Theo bài toán cung chứa góc ta suy ra đim F thuc cung
cha góc
(
)
−
1
180
2
dựng trên đon BC nm trên na mt phng
b BC chứa điểm A. T đó suy ra BF lớn nht khi và ch khi BF là đường kính của đường tròn cha
CHUYÊN ĐỀ BỒI DƯỠNG HỌC SINH GIỎI HÌNH HỌC 9
9 | THCS.TOANMATH.com
cung cha góc
(
)
−
1
180
2
dựng trên đoạn BC. Tc
BCF =90
, hay B,O, F thng hàng. Tc
tam giác ABC cân tại B khi đó giá trị ln nht ca HA + HB + HC là:
BC
OM R R R+ = +
2
2
2 2 2 2
4
VII. T GIÁC NI TIP
1. Tiêu chun 1:
T giác ABCD ni tiếp khi ch khi tổng hai góc đối bng 180°. H qu: ABCD ni tiếp
xAD BCD=
(T giác ABCD ni tiếp khi và ch khi góc ngoài một đỉnh bằng góc trong đối din
với đỉnh đó)
Ví d 1
Cho tam giác ABC ba góc nhn ni tiếp
( )
O
, các đường cao AD, BE, CF ct nhau tại điểm H .
Gọi M là trung điểm ca BC .
a) Chng minh: AEHF là t giác ni tiếp.
b) Đường tròn ngoi tiếp tam giác AEF cắt đường thng AM ti K. Chng minh: Các t giác
EKMC, FKMB ni tiếp.
c) Chng minh: BHKC ni tiếp.
Li gii
a) Do BE, CF các đường cao ca tam giác ABC nên
AEH AFH= 90
, suy ra
AEH AFH+ = 180
nên t giác AEHF ni tiếp.
b) Do 4 đim A, E, H, F nằm trên đường tròn
( )
I
đường kính AH nên đường tròn ngoi tiếp tam
giác AEF cũng chính đưng tròn
( )
I
. Ta
AKE AFE=
(cùng chn cung AE) (1). Mt khác ta
cũng BFEC nội tiếp nên
AFE ECB=
(2). T (1) (2) suy ra
AKE ECM=
nên t giác EKMC ni tiếp.
Tương tự ta cũng chứng minh được: T giác FKMB ni tiếp hoc
th chứng minh theo cách khác như sau:
Ta có:
FKM EKM EKF EKM EKF ACB BAC ABC= = + = + = 360 180 180 180
suy ra
FKM ABC+ = 180
nên t giác FKMB ni tiếp.
c) Ta có:
HKC HKE EKC HKE EKC DAC EKC= = + = + 360 180 180 180
(3)
CHUYÊN ĐỀ BỒI DƯỠNG HC SINH GII HÌNH HC 9
THCS.TOANMATH.com | 10
Mt khác t giác EKMC ni tiếp nên:
EKC EMC=
(cùng chn cung EC). T giác BFEC ni tiếp
BC
M;



2
nên
EMC EBC= 2
(4), ngoài ra ta cũng có
EBC DAC=
cùng ph vi
BCA
(5).
T (3), (4), (5) suy ra
HKC EBC HKC HBC= + = 180 180
nên t giác BHKC ni tiếp.
Cách khác:
IEM IEA MEC IAE ECM= = = 180 180 90
suy ra ME tiếp tuyến ca
( )
I
.
T đó ta có
MCK MEK EHK==
nên suy ra t giác BHKC ni tiếp.
Chú ý: Thông qua d này ta cũng thêm mt tính chất: Đường tròn ngoi tiếp tam giác HEF,
HBC ct nhau ti một điểm nm trên AM.
Ví d 2
Cho tam giác ABC ni tiếp
( )
O
có ba đường cao AD, BE, CF ct nhau ti H, một điểm M di chuyn
trên cung nh BC (M ≠ B, C). Gọi P, Q lần lượt là các điểm đối xng vi M qua AB, AC.
a) Chng minh: AHBP ni tiếp.
b) Chng minh: P, H, N thng hàng.
c) Tìm v trí điểm M trên cung nh BC để PN ln nht.
Li gii
a) Ta có:
APB AMB=
(tính chất đối xng).
AMB ACB DHE AHB= = = 180 180
hay
AMB AHB+ = 180
. Suy ra AHBP là t giác ni tiếp.
b) Chứng minh tương t câu a ta thy t giác AHCN
ni tiếp. Theo câu a t giác AHBP ni tiếp nên ta
biến đổi góc sau:
AHP ABP ABM ACM ACN AHN= = = = = 180 180 180
, hay
AHP AHN+ = 180
tc là N, H, P thng hàng.
c) Do ba đim M, N, P nằm trên đường tròn tâm A
bán kính AM suy ra
(
)
NP AM.sinPMN AM.sin BAC AM.sinBAC= = =2 2 180 2
. Do đó NP lớn nht khi và ch khi AM ln nhất, lúc đó AM là đường kính của đường tròn
( )
O
.
Vậy độ dài đoạn NP ln nht khi và ch khi M là điểm đối xng ca A qua O.
Ví d 3
Cho hình bình hành ABCD
(
)
A 90
hai đường chéo ct nhau ti I. Gọi M trung điểm ca
BC, đường thng AM cắt đường tròn ngoi tiếp tam giác ABC ti N, gi H là hình chiếu vuông góc
ca C trên AB.
CHUYÊN ĐỀ BỒI DƯỠNG HỌC SINH GIỎI HÌNH HỌC 9
11 | THCS.TOANMATH.com
a) Chng minh: T giác ADNH ni tiếp.
b) Dựng các đường thng qua A ln t vuông góc vi BC, CD, BD ti X, Y, Z. Chng minh 4
điểm X, Y, I, Z nm trên một đường tròn.
Li gii
a) Để chng minh: t giác ADNH ni tiếp ta chng minh:
BHN ADN=
.
Ta có:
BHN BHM NHM=−
.
MB MC,CH BH=⊥
nên suy ra MB = MH = MC suy ra
BHM ABC=
Vy
BHN ABC NHM=−
.
Ta cũng có:
ADN ADC CDN ABC NDC= =
Như vậy ta cn chng minh:
NHM NDC=
. Để ý rng:

AMB CMN
(g.g) suy ra
MB AB
MN CN
=
thay AB =CD, MH = MB suy ra
MH CD
MN CN
=
(1). Ta có:
(
)
(
)
(
)
(
)
HMN HMB BMN HMB BN AC HCB BAN ANC ABC BAN ANC= + = + + = + + = + +
1
2 2 90
2
Do
ABC ANC=
suy ra
HMN ABC BAN BAD BAN BCD BCN DCN= + = + = + =180
(2).
T (1) và (2) suy ra

HMN DCN
nên
NHM NDC=
suy ra
BHN ADN=
hay ADNH ni tiếp.
b) T các t giác XAZC, ABXY ni tiếp ta có biến đổi góc sau:
IZX AZC AZI XZC BAZ ZAC CAX=BAX=BYX= =
nên t giác XYIZ ni tiếp.
2. Tiêu chun 2:
T giác ABCD ni tiếp khi và ch khi có 2 đỉnh liên tiếp cùng nhìn mt cnh nhng góc bng nhau:
DAC DBC=
ABCD là t giác ni tiếp.
Mt s ví d tiêu biu
Ví d 1
Cho tam giác ABC ba góc nhn ni tiếp
( )
O
, các đường cao AD,
BE, CF ct nhau tại điểm H. Gi M là trung điểm ca BC .
a) Chng minh: BFEC là t giác ni tiếp.
CHUYÊN ĐỀ BỒI DƯỠNG HC SINH GII HÌNH HC 9
THCS.TOANMATH.com | 12
b) Chng minh: MEFD là t giác ni tiếp.
c) Gọi I trung điểm AH , đường thng AH kéo dài cắt đường
tròn tại giao điểm th 2 N (N A). Chứng minh: BNEI là t
giác ni tiếp.
Li gii
a) Do BE, CF các đường cao ca tam giác ABC nên
BEC BFC= = 90
suy ra t giác BFEC t giác ni tiếp (Hai
đỉnh liên tiếp E, F cùng nhìn cnh BC mt góc bng 90°).
b) Ta d dàng chứng minh được: các t giác BFHD, CEHD ni tiếp nên t đó suy ra
FBH FDH=
(1),
ECH EDH=
(2), mặt khác theo a) ta cũng có:
FBE FCE=
(3). T (1), (2), (3) suy ra
FDE EBF= 2
(4). Li t giác BFEC ni tiếp đường tròn tâm M đường kính BC nên
2=EMF EBF
(5) (Tính cht góc tâm bng 2 ln s đo góc nội tiếp chn cùng mt cung). T (4),
(5) suy ra
EDF EMF=
nên t giác EMDF ni tiếp.
Chú ý: Có th chứng minh: 4 điểm E, M, D, F nằm trên đường tròn Ơle của tam giác.
c) Ta có:
EIN EAN= 2
(6) (Tính cht góc ngoài tam giác cân AIE). Mặt khác ta cũng có:
==EBC CAN CBN
suy ra
EBN EBC EAN==22
(7). T (6) và (7) suy ra
EIN EBN=
hay t
giác BNEI ni tiếp (Có hai đỉnh liên tiếp I, B cùng nhìn cnh EN các góc bng nhau).
Ví d 2
Cho đường tròn
( )
O
một điếm M nm ngoài
( )
O
, qua M k các tiếp tuyến MA, MB đến
( )
O
(
A,B là các tiếp điểm), dng cát tuyến MCD (MC < MD ) sao cho tia MD nm gia hai tia MO, MA
. Gọi E là trung điểm ca CD.
a) Chứng minh: 5 điếm M, A, E, O, B cùng nm trên một đường tròn.
b) Đường thng qua E song song vi BD ct AB ti N. Chng minh: AENC là t giác ni tiếp.
c) Đường thng AE ct
( )
O
tại giao điểm th 2 là F. Chng minh: BF // CD .
Li gii:
a) Do E là trung điểm CD nên
OE CD
.
Suy ra
MAO MEO= = 90
nên t giác MAEO ni tiếp (1). Li
MAO MBO+ = 180
nên
MAOB ni tiếp (2). T (1) (2) ta suy ra 5 điểm M, A, E, O, B cùng nằm trên đường tròn đường
kính MO.
b) Do EN // BD nên
CEN CDB=
, li có:
CDB CAB=
cùng chn cung CB nên suy ra
CEN CAN=
hay AENC là t giác ni tiếp.
CHUYÊN ĐỀ BỒI DƯỠNG HỌC SINH GIỎI HÌNH HỌC 9
13 | THCS.TOANMATH.com
c) Ta có:
AFB AOB AOM==
1
2
(3) (Tính cht góc ni tiếp góc
tâm + Tính cht 2 tiếp tuyến ct nhau). Mt khác do AEOM ni tiếp nên
AOM AEM=
(4). T (3) (4) suy ra
AEM AFB=
, hai góc này
đồng v nên EC // BF .
Ví d 3
Cho tam giác ABC ngoi tiếp đường tròn
( )
I
. Gi E, F lần lượt các tiếp đim ca
( )
I
vi AC,
AB. Đường thng BI, CI ct EF lần lượt ti M, N.
a) Chng minh: E, M, C, I cùng nm trên một đường tròn
b) Chứng minh: 4 điểm B, C, N, M cùng nm trên một đường tròn.
Li gii
a) Ta có:
B C A
MIC IBC ICB= + = + = 90
2 2 2
(1) (Tính cht góc ngoài ca tam giác BIC).
Ta cũng có:
AA
MEC AEF
−
= = =
180
90
22
(2).
T (1) và (2) ta suy ra
MEC MIC=
hay MEIC là t giác ni tiếp.
b) MEIC là t giác ni tiếp nên
IEC IMC= = 90
(3). Chứng minh tương tự câu a) ta có BFNI là t
giác ni tiếp nên
INB IFB= = 90
(4). T (3), (4) ta suy ra
BNC BMC= = 90
nên BNMC là t giác ni tiếp.
Ví d 4
Cho hình bình hành ABCD
A 90
. Đường phân giác trong
góc A ct cnh BC ti P, ct CD ti Q. Gọi O tâm đường tròn
ngoi tiếp tam giác CPQ. Chứng minh 4 điểm B, D, O, C cùng
nm trên một đường tròn.
Li gii
Do ABCD là hình bình hành và
BAQ DAQ=
suy ra
APB DQA CPQ==
hay CPQ cân ti C suy ra
ABP cân ti P hay AB = BP = CD (4).
Ta cũng có:
BPO OPC OCP OCQ OCD= = = =180 180 180
(5), chú ý rng: OP = OC (6).
T (4), (5), (6) suy ra ∆BPO = ∆DCO
OBC ODC=
hay 4 điểm B, D, O, C cùng nm trên mt
đường tròn.
Ví d 5
CHUYÊN ĐỀ BỒI DƯỠNG HC SINH GII HÌNH HC 9
THCS.TOANMATH.com | 14
Cho t giác ABCD ni tiếp
( )
O
, hai đường chéo AC, BD ct nhau ti P.
Q, R hai điểm bt k trên cung CD không cha A, B. RA, RC lần t
cắt đường tròn ngoi tiếp tam giác PQR ti L, K khác R. PK, PL lần lượt
ct BC, AD ti M, N.
a) Chng minh: P, Q, N, D cùng nm trên một đường tròn.
b) Chng minh: BC, AD cắt nhau trên đường tròn ngoi tiếp tam giác
QMN .
Li gii:
a) Các t giác PQLR, ABQR, ABQD ni tiếp nên ta có biến đổi góc sau:
NPQ QRL ABQ QDN NPQ QDN= = = + = 180 180 180
nên NPQD là t giác ni tiếp.
b) Gi S là giao điểm ca BC, AD. Ta chng minh t giác SMQN ni tiếp.
Tht vy t các t giác NPQD, BCQR RQKP ni tiếp ta biến đổi góc:
MBQ QRK QPK==
nên MBPQ t giác ni tiếp. T đó suy ra
BMQ QPD QND==
suy ra SMQN t giác ni tiếp.
Suy ra đpcm.
3. Tiêu chun 3:
Cho 2 đuờng thng ct nhau ti M
+Nếu
( ) ( )
A,B d ;C,D d
12
sao cho
MA.MB MC.MD=
thì ABCD là t giác ni tiếp (Hình 1).
Tht vy:
MA.MB MC.MD=

MAD MCB
MAD MCB=
suy ra ABCD ni tiếp.
+ Nếu
MA.MC MB.MD=
thì ABCD là t giác ni tiếp (Hình 2)
Tht vy:
MA.MC MB.MD=

MAD MBC
DAC DBC=
suy ra ABCD ni tiếp.
Ví d 1
CHUYÊN ĐỀ BỒI DƯỠNG HỌC SINH GIỎI HÌNH HỌC 9
15 | THCS.TOANMATH.com
Cho đường tròn
( )
O
và một điểm M nm ngoài
( )
O
, qua M k tiếp tuyến MA đến
( )
O
và cát tuyến
MCD sao cho MC < MD. Dng
AH MO
. Chng minh: T giác CHOD ni tiếp.
Li gii
Do MA là tiếp tuyến ca
( )
O
AH MO
Theo h thức lượng trong tam giác vuông MAO ta có:
MA MH.MO=
2
(1)
Mặt khác ta cũng có:
MAC MDA=
suy ra

MAC MDA
nên
MA MC
MA MC.MD
MD MA
= =
2
(2).
T (1) (2) suy ra
MC.MD MH.MO=
hay CHOD t
giác ni tiếp (tiêu chun 3).
Ví d 2
Cho tam giác ABC 3 góc nhn ni tiếp
( )
O
, các
đường cao AD, BE, CF ct nhau tại H, đường tròn ngoi
tiếp tam giác AEF ct
( )
O
tại điểm th 2 là N khác A, AN
ct BC ti M.
a) Chng minh: M, E, F thng hàng.
b) Đoạn ME ct
( )
O
ti X. Chng minh: AX tiếp tuyến của đường tròn ngoi tiếp tam giác
XHD.
Li gii
a) Đường tròn ngoi tiếp tam giác AEF có tâm là trung điểm I ca AH, ta kí hiu là
( )
I
Gi s MF ct
( )
I
ti E
1
thì ANFE
1
ni tiếp nên suy ra MN.MA = MF.ME
1
.
Mt khác ANBC ni tiếp nên MN.MA = MB.MC
T đó suy ra MF.ME
1
= MB.MC hay BFE
1
C ni tiếp. Suy
ra
EE
1
(hai đường tròn ct nhau tối đa tại 2 điểm) hay
M, E, F thng hàng.
b) ME ct
( )
O
tại giao đim th 2 Y thì
OA XY
(Tính cht quen thuc, dành cho hs) suy ra tam giác AXY
cân ti A nên
AYX AXY=
mt khác
AYX ABX=
nên
CHUYÊN ĐỀ BỒI DƯỠNG HC SINH GII HÌNH HC 9
THCS.TOANMATH.com | 16
AXF ABX=

AXF ABX
suy ra
AX AF.AB=
2
. Mt khác BDHF t giác ni tiếp nên
=AF.AB AH.AD
t đó suy ra
2
=AX AH.AD
hay AX tiếp tuyến của đường tròn ngoi tiếp tam
giác XHD.
Ví d 3
Cho tam giác nhọn ABC đường cao AH. Đường tròn
( )
O
đường kính AH ct AB, AC lần lượt ti
D, E. Đường thng ED ct BC tại điểm M. Đường thng MO ct AB, AC ti N và P.
a) Chng minh:
MH MB.MC=
2
.
b) Chng minh: OP = ON.
c) Đường thng DE ct HN, HP lần lượt ti R, Q. Chứng minh: BR, CQ, AH đồng quy.
Li gii
a) 4 điểm A, D, H, E cùng nằm trên đường tròn
( )
O
đường kính AH nên MH tiếp tuyến ca
( )
O
suy
ra
MH MD.ME=
2
(1)
Để chng minh:
MH MB.MC=
2
ta s chng minh:
BDEC là t giác ni tiếp.
Tht vy: Ta có:
ECH EHA=
cùng ph vi
EHC
EHA EDA ECH EDA= =
hay BDEC
t giác ni tiếp, suy ra MD.ME = MB.MC (2).
T (1) và (2) ta suy ra
MH MB.MC=
2
.
b) Dng
HK MO
thì
MH MK.MO=
2
, mt khác
MH MD.ME=
2
suy ra MK.MO = MD.ME
hay DKOE ni tiếp suy ra
OKE ODE OED MKD= = =
nên suy ra
DKH EKH=
. Li t giác
HKPE ni tiếp nên
HPE HKE=
, suy ra
HPE DKE DOE DAC= = =
11
22
suy ra HP // AB nên tam
giác
POH NOA OP ON

= =
.
c) Ta có
= = =PQE DQH QDA ECH
nên QECH t giác ni tiếp suy ra
CQH CEH= = 90
hay
CQ HP CQ AB
. Chứng minh tương tự ta cũng có:
DR AC
hay BR, CQ, AH đồng quy.
Ví d 4
Cho đưng tròn
( )
O
điểm A nm ngoài đường tròn
đó. Kẻ các tiếp tuyến AB, AC với đường tròn
( )
O
(B, C
các tiếp điểm). Gọi I giao điểm ca OA BC. K
dây cung DE của đường tròn
( )
O
qua I.
a) Chng minh rằng 4 điểm A, D, O, E cùng nm trên
CHUYÊN ĐỀ BỒI DƯỠNG HỌC SINH GIỎI HÌNH HỌC 9
17 | THCS.TOANMATH.com
một đường tròn.
b) Chng minh rng
BAD CAE=
Li gii
a) Ta thy bốn điểm B, D, C, E cùng nằm trên đường tròn
( )
O
nên
ID.IE IB.IC IB==
2
(1). S
dng h thức lượng trong tam giác vuông ABO với BI là đường cao ta có:
IB IA.IO=
2
(2). T (1)
và (2) ta thu được ID.IE = IA.IO. Chng t bốn điểm A, D, O, E cùng nm trên một đường tròn.
b) T câu a ta thy
ODE OAE=
(cùng chn
OE
);
OED OAD=
(cùng chn
OD
). Do tam giác
ODE cân ti O nên
OED ODE=
. Do đó
OAE OAD=
(3). Chú ý AO là tia phân giác ca góc
BAC
, t (3) suy ra
BAE CAD=
(4). T (3), (4) suy ra
BAD CAE=
(đpcm).
4. Đường tròn Ơ - le ca tam giác
Cho tam giác nhn ABC ni tiếp
( )
O
các đường cao AD,
BE, CF ct nhau ti H. Gi X, M, Y, I, Z, T lần t trung
điểm ca AB, BC, AC, HA, HB, HC. Khi đó 9 điểm D, E, F, X,
M, Y, I, Z, T nm trên cùng một đường tròn tâm trung
điểm ca OH (gọi là đường tròn ơle của tam giác ABC).
Chng minh: Ta IZ đường trung bình ca tam giác ABH
IZ AB=
1
2
, YM đường trung bình tam giác ABC nên
YM AB=
1
2
. T đó suy ra
IZ YM=
IZMY hình bình
hành. Lại ZM đường trung bình ca tam giác BHC nên ZM//CH
AB CH
suy ra
ZM IZ
vy t giác IZMY là
hình ch nhật nên hai đường chéo IM, ZY ct nhau tại trung điểm J ca mỗi đường.
Tương tự ta cũng chứng minh được các t giác XITM, XYTZ các hình ch nht nên suy ra các
đường chéo IM, ZY, TX đồng quy tại trung điểm J ca mỗi đường.
Chú ý rng: IDM, ZEY, TFX lần lượt vuông ti D, E, F nên tâm vòng tròn ngoi tiếp chính
trung đim J ca các cnh huyền tương ứng. Suy ra 9 đim D, E, F, X, Y, M, I, Z, T cùng nm trên
đường tròn tâm J. Ta cũng có:
OM AH IH==
1
2
nên t giác IHMO hình bình hành, suy ra
hai đường chéo IM, HO ct nhau tại trung điểm mỗi đường. Nói cách khác điểm J cũng chính
trung điểm ca HO . Nếu A' là điểm đối xng vi A qua J thì AHA'O hình bình hành ta d dàng
suy ra A' đối xng vi O qua M.
Ví d 1
CHUYÊN ĐỀ BỒI DƯỠNG HC SINH GII HÌNH HC 9
THCS.TOANMATH.com | 18
Cho tam giác ABC ni tiếp đường tròn
( )
O
AD đường
kính ca
( )
O
. M trung đim ca BC, H trc tâm ca
tam giác. Gi X, Y, Z ln lượt hình chiếu vuông góc ca
điểm D lên HB, HC, BC. Chứng minh 4 điểm X, Y, Z, M
cùng thuc một đường tròn.
Li gii
Phân tích: M là trung điểm BC
M cũng là trung điểm ca
HD (Bài toán quen thuc).
X, Y, Z lần lượt là hình chiếu vuông góc của điểm D lên HB, HC, BC kết hp tính chất điểm M làm
ta liên tưởng đến đường tròn ơle của mt tam giác: T những cơ sở đó ta có lời giải như sau:
+ Gi s HB ct DY ti I, HC ct DX tại K, J trung đim ca IK. Ta d chứng minh được BHCD
hình bình hành suy ra hai đưng chéo HD, BC ct nhau tại trung điểm M ca mỗi đường.
DX HI ,DI HC⊥⊥
suy ra K là trc tâm ca tam giác IHD nên
KDI KHI HCD==
(chú ý HI //
CD) và
CHD KID=
(cùng ph vi góc
HDI
).
T đó suy ra

KID CHD
+ Mt khác CM, DJ là hai trung tuyến tương ứng của tam giác CHD và KID, như vậy ta có

DIJ CHM
JDI HCM =
. T đó suy ra
DJ BC
ti Z hay Z thuộc đường tròn đường kính
MJ . Ta thy rằng, đường tròn đường kính MJ là đường tròn ơle của tam giác IHD .
T đó ta có: X, Y, Z, M đều cùng nằm trên đường tròn đường kính MJ. Đó điều phi chng
minh.
Ví d 2
Cho hai đim A, B c định và điểm C di động trên mt phng
sao cho
( )
ACB

= 0 180
. Đường tròn tâm I ni tiếp
tam giác ABC tiếp xúc vi các cnh AB, BC, CA lần t ti
D, E, F. Các đường thng AI, BI ct EF lần lượt ti M, N.
Chng minh:
a) Đoạn thẳng MN có độ dài không đổi.
b) Đường tròn ngoi tiếp tam giác DMN luôn đi qua điểm c
định.(VMO-2009)
Li gii
a) Xét t giác IEMB ta có:
A B C
MIB IAB IBA= + = + = 90
2 2 2
(Tính cht góc ngoài).
CHUYÊN ĐỀ BỒI DƯỠNG HỌC SINH GIỎI HÌNH HỌC 9
19 | THCS.TOANMATH.com
Ta cũng có:
CC
MEB CEF
−
= = =
180
90
22
. T đó suy ra IEMB t giác ni tiếp, do đó:
IMC IEC= = 90
. Tương t ta cũng chứng minh được 4 điểm I, N, A, F cùng nm trên một đường
tròn suy ra
INA IFA= = 90
. T đó suy ra
BMA BNA= = 90
hay 4 điểm B, C, M, N nm trên
đường tròn tâm J đường kính AB .
Ta có:
2 2 2
= = = = =MJN MBN NEI ICF ACB
.
Ta có:
MJN
MN NJ.sin AB.sin
==2
22
không đổi.
b) Gi s AN ct BM ti H thì I trc tâm ca tam giác AHB.
Đưng tròn ngoi tiếp tam giác DMN chính đường tròn ơle
của tam giác HAB . Suy ra đưng tròn ngoi tiếp tam giác DMN
luôn đi qua trung điểm J của AB. Như vậy đường tròn ngoi tiếp
tam giác DMN luôn đi qua điểm c định là J.
Ví d 3
Cho tam giác ABC ni tiếp
( )
O
điểm P di chuyn trên
( )
O
.
Đưng thng qua P song song vi BC ct CA ti P. Gi K là tâm
đường tròn ngoi tiếp tam giác PCE L tâm đưng tròn ơle
ca tam giác PBC . Chứng minh: Đường thng qua L song song với PK luôn đi qua một đim c
định khi P di chuyn trên
( )
O
.
Li gii
Gi H, N lần lượt trực tâm, tâm đường tròn ơle của tam giác ABC, M trung điểm BC. G là
điểm đối xng vi P qua L.
Theo kết qu quen thuc v tâm đường tròn ơle thì G đối xng vi O qua BC.
Gọi Q là giao điểm ca AH vi
( )
O
.
Do PE // BC nên
CEP ACB=
suy ra
(
)
CPK CKP CEP ACB CAQ CPQ= = = = =
1
180 90 90
2
nên P, K, Q thng hàng.
Đưng thng qua L song song vi PK ct GQ tại J thì J là trung điểm của QG, do H đối xng Q qua
BC nên OHQG hình thang cân NJ đưng trung bình của hình thang. Suy ra J đi xng vi
N qua BC. Suy ra đường thng qua L song song vi PK luôn đi qua một điểm c định điểm J đối
xng với tâm ơle của tam giác ABC qua BC.
5. Đường tròn Apollonius
Bài toán
CHUYÊN ĐỀ BỒI DƯỠNG HC SINH GII HÌNH HC 9
THCS.TOANMATH.com | 20
Bài toán: Cho đoạn thng AB = a, k s cho trước
( )
k ,k01
, M điểm chuyển động trong
mt phng sao cho
MA
k
MB
=
. Tìm qu tích điểm M.
Li gii
Phn thun: Gi s (MA > MB). Dng MD, ME lần lượt
các phân giác trong phân giác ngoài ca
AMB
thì
DME =90
.
Theo tính cht phân giác ta có
DA EA MA DA k ak
k DA
DB EB MB DA DB k k
= = = = =
+ + +11
, tương tự
ak
EA
k
=
1
. Suy ra các điểm E,
D c định,
DME =90
suy ra M thuộc đường tròn đường kính DE.
Phần đo: Lấy điểm M' trên đường tròn đường kính DE. Ta chng minh: M'D là phân giác ca góc
AM' B
. Qua B k các đưng thng vuông góc vi D'M ct M'D MA ti H, K
1
= = =
BK AB k BH DB
BK M' E ,
EM' AE k EM' DE
ak ak ak
DE AE AD
kk
k
= = =
−+
2
2
11
1
.
Mặt khác ta cũng có
a
DB
k
=
+1
Suy ra
BH DB a k k BH BK
. BK BH HB HK
EM DE k ak k EM EM
−−
= = = = = =
+
2
1 1 2
2
1 2 2
MBK cân
suy ra M'D phân giác ca góc
AM' B
M'E phân giác
ngoài ca góc
AM' B
.
Ví d 1
Cho tam giác ABC ni tiếp
( )
O
điểm M thay đổi trên cung
BC không cha A . Gọi I, J tâm đường tròn ni tiếp các tam
giác ABM, CAM . Chứng minh: Đường tròn ngoi tiếp tam
giác MIJ luôn đi qua một điểm c định.
Li gii
Gọi N giao điểm của đường tròn ngoi tiếp tam giác
MIJ vi
( )
O
, MI ct
( )
O
ti E, MJ ct
( )
O
ti D .
Ta có EA = EB, DA = DC.
Mặt khác ta cũng nh chất quen thuc: EA = EB = EI
và DA = DC = DJ.
CHUYÊN ĐỀ BỒI DƯỠNG HỌC SINH GIỎI HÌNH HỌC 9
21 | THCS.TOANMATH.com
Tam giác NIE và NJD
NEI NDJ=
(cùng chn cung MN )
EIN NIM NJM NJD= = =180 180
nên suy ra

NIE NJD
suy ra
NE EI AE
ND DJ AD
==
(không
đổi). Suy ra N thuộc đường tròn Apollonius dựng trên đoạn ED vi t s
AE
AD
. Hay N giao đim
của đường tròn Apollonius với đường tròn
( )
O
.
Ví d 2
Cho tam giác ABC (AB >AC) và đim M nm trong tam
giác sao cho
MB AB
MC AC
=
. Gọi N là điểm đối xng vi M
qua BC. Chng minh:
MAB NAC=
Li gii
Gi AE, AF lần lượt các đường phân giác trong, phân
giác ngoài góc A ca tam giác ABC.
Theo gi thiết và tính cht của đường phân giác ta có:
NB MB AB EB FB
NC MC AC EC FC
= = = =
Suy ra 5 điểm M, N, A, E, F cùng nằm trên đường tròn Apollonius xác định bi BC và t s
AB
AC
.
M, N đối xứng nhau qua BC nên M, N đối xứng nhau qua đưng kính EF của đường tròn nói
trên. Do đó
MAE NAE=
(1). Mt khác AE phân giác ca góc
BAC
nên
BAE NAE=
(2). T
(1), (2) suy ra
MAB NAC=
.
B.BÀI TOÁN THƯỜNG GẶP TRONG ĐỀ HSG VÀ TOÁN CHUYÊN
Bài 1. Cho hình hành ABCD đỉnh D nằm trên đường tròn đường kính AB. K BN DM cùng
vuông góc với đường chéo AC. Chng minh rng:
a) T giác CBMD là t giác ni tiếp.
b) Khi điểm D di động trên đường tròn thì
BMD BCD+
không đổi.
c)
..DB DC DN AC=
.
Bài 2. Cho hai đường tròn
( )
O
( )
O
ct nhau ti A B. Các tiếp tuyến ti A của đường tròn
( )
O
( )
O
cắt đường tròn
( )
O
( )
O
theo th t ti CD. Gi P và Q lần lượt là trung điểm
ca các dây AC và AD. Chng minh rng:
a) Hai tam giác ABD và CBA đồng dng.
b)
BQD APB=
.
CHUYÊN ĐỀ BỒI DƯỠNG HC SINH GII HÌNH HC 9
THCS.TOANMATH.com | 22
c) T giác APBQ ni tiếp.
Bài 3. Cho tam giác ABC nhn ni tiếp đường tròn
( )
O
. Đường tròn ngoi tiếp tam giác OBC ct
AB và AC th t ti D và E. Chng minh AO vuông góc vi DE.
Bài 4. Cho hai vòng tròn
( )
1
O
( )
2
O
tiếp xúc ngoài nhau tại điểm T. Hai vòng tròn này nm
trong vòng tròn
( )
3
O
tiếp xúc vi
( )
3
O
tương ng ti M N. Tiếp tuyến chung ti T ca
( )
1
O
( )
2
O
ct
( )
3
O
ti P. PM ct vòng tròn
( )
1
O
tại điểm th hai A và MN ct
( )
1
O
tại điểm th hai
B. PN ct vòng tròn
( )
2
O
tại điểm th hai D và MN ct
( )
2
O
tại điểm th hai C.
a) Chng minh rng t giác ABCD là t giác ni tiếp.
b) Chng minh rằng các đường thẳng AB, CD và PT đồng quy.
Bài 5. T điểm A nằm ngoài đường tròn tâm O k hai tiếp tuyến AB AC (B C các tiếp
điểm). Gọi M đim bt trên cung nh BC của đường tròn
( )
O
(M khác B C). Tiếp tuyến
qua M ct AB và AC tại E và F. Đường thng BC ct OE và OF P và Q. Chng minh rng:
a) T giác OBEQ, OCFP là các t giác ni tiếp.
b) T giác PQFE là t giác ni tiếp.
c) T s
PQ
FE
không đổi khi M di chuyển trên đường tròn.
Bài 6. Cho tam giác ABC, D và E là các tiếp điểm của đường tròn ni tiếp vi các cnh AB và AC.
Chứng minh đường phân giác trong của góc B, đường trung bình ca tam giác song song vi cnh
AB và đường thẳng DE đồng quy.
Bài 7. Cho đưòng tròn
( )
;OR
đường kính AB c định đường kính CD quay quanh điểm O. Các
đường thng AC và AD ct tiếp tuyến ti B của đường tròn theo th t ti E và F.
1. Chng minh rng t giác CDFE ni tiếp đường tròn.
2. Gọi I tâm đưng tròn ngoi tiếp t giác CDFE. Chng minh rằng điểm I di động trên đường
thng c định khi đường kính CD quay quanh điểm O.
(Thi Hc sinh gii lp 9, tỉnh Trà Vinh, năm học 2009 - 2010)
Bài 8. Cho tam giác ABC vuông ti A D một điểm trên cnh AC (Khác vi A C). V
đường tròn tâm D tiếp xúc vi BC ti E. T B k tiếp tuyến th hai BF với đường tròn
( )
D
. Gi M
trung điểm của BC, N giao điểm ca BF AM. Chứng minh năm điểm A, B, E, D, F cùng
nm trên một đường tròn và
AN NF=
.
(Thi Hc sinh gii lp 9, tỉnh Vĩnh Long, năm học 2009 -2010)
Bài 9. Cho hai đường tròn
( )
;OR
( )
;OR

ct nhau tại hai điểm phân bit A và B. T một đim
C thay đổi trên tia đi ca tia AB, v các tiếp tuyến CD, CE với đường tròn tâm O (D; E các tiếp
CHUYÊN ĐỀ BỒI DƯỠNG HỌC SINH GIỎI HÌNH HỌC 9
23 | THCS.TOANMATH.com
điểm E nằm trong đường tròn tâm
O
). Hai đường thng AD AE cắt đường tròn tâm
O
ln
t ti M và N (M, N khác với điểm). Đường thng DE ct MN ti 1. Chng minh rng:
a)
..MI BE BI AE=
.
b) Khi điểm C thay đổi thì đường DE luôn đi qua một điểm c định.
(Thi Hc sinh gii lp 9, tnh Ngh An, năm học 2009 - 2010)
Bài 10. Cho tam giác ABC vuông tại A. Đường tròn tâm I ni tiếp tam giác ABC, tiếp xúc vi CA
CB lần lượt tại M N. Đường thng MN cắt đường thng AI ti P. Chng minh rng góc IPB
vuông.
(Thi Hc sinh gii lp 9, tỉnh Gia Lai, năm học 2009 - 2010)
Bài 11. Cho đường tròn
( )
;OR
dây AB c định,
2AB R=
. Điểm P di đng trên dây AB (P
khác A B). Gi
( )
1
;CR
đường tròn đi qua P tiếp xúc với đường tròn
( )
;OR
ti A,
( )
2
;DR
là đường tròn đi qua P và tiếp xúc vi
( )
;OR
tại B. Hai đường tròn
( )
1
;CR
( )
2
;DR
ct nhau ti
điểm th hai M.
a) Trong trường hp P không trùng với trung điểm y AB, chng minh
//OM CD
4 điểm C, D,
O, M cùng thuc một đường tròn;
b) Chng minh khi P di động trên dây AB thì điểm M di động trên đường tròn c định đường
thẳng MP luôn đi qua một điểm c định N;
c) Tìm v trí ca P để tích PM.PN ln nht? Din tích tam giác AMB ln nht.
(Thi Hc sinh gii lp 9, tnh Phú Thọ, năm học 2009 - 2010)
Bài 12. Cho tam giác ABC
( )
AB AC
ni tiếp đường tròn
( )
O
AD phân giác góc
BAC
, tia
AD cắt đường tròn tại đim E (E khác A). K đưng kính EF của đường tròn
( )
O
. Gi P mt
điểm nm gia A và D. Tia FP cắt đường tròn
( )
O
ti Q khác F. Đường thng qua P vuông góc vi
AD ct CA, AB lần lượt ti M, N.
a) Chng minh rng các t giác PQBN, PQCM là t giác ni tiếp.
b) Gi s QN PC ct nhau ti một đim thuộc đường tròn
( )
O
. Chng minh rng QM PB
cũng cắt nhau ti một điểm thuộc đường tròn
( )
O
.
Bài 13. Cho tam giác ABC 3 góc nhn ni tiếp
( )
;OR
AB AC
. V 3 đường cao AD, BE,
CF ca tam giác ABC ct nhau ti H, AD ct
( )
O
ti K và ct EF ti I.
a) Chng minh rng: BC là trung trc ca HK và
..IF IE IH IA=
;
b) Chng minh rng : Các t giác DHEC, BFIK ni tiếp được;
c) Chng minh rng:
KC BK EF
AC BA AI
+=
;
CHUYÊN ĐỀ BỒI DƯỠNG HC SINH GII HÌNH HC 9
THCS.TOANMATH.com | 24
d) Đưng thng qua E song song vi AD ct BK ti M. Chng minh rằng: 3 điểm F, D, M thng
hàng;
Bài 14. Cho tam giác ABC nhn vi
AB AC
AD đường phân giác. Đường thng qua C
song song vi AD cắt đường trung trc ca AC tại E. Đường thng qua B song song vi AD ct
đường trung trc ca AB ti F.
a) Chng minh rng tam giác ABF đồng dng vi tam giác ACE.
b) Chng minh rằng các đường thẳng BE, CF, AD đồng quy ti một điểm, gọi điểm đó là G.
c) Đưng thng qua G song song vi AE cắt đường thng BF ti Q. Đường thng QE cắt đường
tròn ngoi tiếp tam giác GEC và P khác E. Chng minh rằng các điểm A, P, G, Q, F cùng thuc mt
đường tròn.
Bài 15. Cho tam giác ABC
BAC
góc ln nhất. Các điểm P, Q thuc cnh BC sao cho
QAB BCA=
CAP ABC=
. Gi M, N lần lượt là các điểm đối xng ca A qua P; Q. Chng minh
rng BN và CM ct nhau ti một điểm thuộc đường tròn ngoi tiếp tam giác ABC.
(Thi vô địch Toán Quc Tế IMO, năm 2014)
Bài 16. Cho 19 đim nm trong hay trên cnh ca mt lục giác đều cnh bng 4 cm. Chng minh
rng luôn tn ti 2 trong s 19 điểm đã cho mà khoảng cách gia chúng không vượt quá
43
3
cm.
(thi hc sinh gii lp 9, tnh Hưng Yên, năm học 2013 - 2014)
Bài 17. Cho hình thang ABCD vuông góc tại A và B, M là trung điểm của AB. Đường thng qua A
vuông góc vi MD cắt đường thng qua B vuông góc vi MC ti N. Chng minh
MN CD
.
(tuyn sinh lớp 10 chuyên Toán. Đại học sư phạm TP. H Chí Minh, năm học 2015 - 2016)
Bài 18. Cho tam giác ABC
( )
AB AC
có các góc nhn, ni tiếp trong đường tròn tâm O. Gi M là
trung điểm cạnh BC, E là điểm chính gia ca cung nh BC, F là điểm đối xng ca E qua M.
a) Chng minh rng
2
.EB EF EO=
;
b) Gọi D là giao điểm ca AE và BC. Chứng minh các điểm A, D, O, F cùng thuc một đường tròn.
c) Gọi I là tâm đường tròn ni tiếp tam giác ABC P điểm thay đổi trên đường tròn ngoi tiếp
tam giác IBC sao cho P, O, F không thng hàng. Chng minh rng tiếp tuyến ti P của đường tròn
ngoi tiếp tam giác POF đi qua một điểm c định.
(tuyn sinh lớp 10, trường ph thông Năng khiếu Đại hc Quc gia TP. H Chí Minh, năm học
2015-2016)
Bài 19. Cho tam giác nhn ABC
( )
AB AC
, M trung điểm cnh BC, O tâm của đưòng tròn
ngoi tiếp tam giác. Các đưng cao AD, BE, CF của tam giác ABC đng quy ti H. Các tiếp tuyến
vi
( )
O
ti B C ct nhau ti S. Gi X, Y lần ợt giao điểm của đường thng EF vi các
đường thng BS, AO. Chng minh rng:
CHUYÊN ĐỀ BỒI DƯỠNG HỌC SINH GIỎI HÌNH HỌC 9
25 | THCS.TOANMATH.com
a)
MX BF
.
b) Hai tam giác SMX và DHF đồng dng.
c)
EF BC
FY CD
=
.
(tuyn sinh lớp 10, THPT chuyên, Đại học sư phạm Hà Nội, năm học 2015 - 2016)
Bài 20. Cho tam giác đu ABC ni tiếp đường tròn
( )
O
, H trung đim ca BC. M điểm bt kì
thuộc đoạn thng BH (M khác B). Ly điểm N thuộc đoạn thng CA sao cho
CN BM=
. Gi I
trung điểm ca MN.
1) Chng minh bốn điểm O, M, H, I cùng thuc một đường tròn.
2) Gi P là giao điểm ca OI và AB. Chng minh tam giác MNP là tam giác đều.
3) Xác định v trí của điểm M để tam giác IAB có chu vi nh nht.
(tuyn sinh lp 10, THPT chuyên, TP. Hà Nội, năm học 2014 - 2015)
Bài 21. Cho tam giác ABC ni tiếp đường tròn
( )
O
điểm P nm trong tam giác tha mãn
PB PC=
. D là điểm thuc cnh BC (D khác B và D khác C) sao cho P nằm trong đưng tròn ngoi
tiếp tam giác DAB đưng tròn ngoi tiếp tam giác DAC. Đường thng PB cắt đường tròn ngoi
tiếp tam giác DAB tại E khác B. Đường thng PC cắt đường tròn ngoi tiếp tam giác DAC ti F
khác C.
1) Chng minh bốn điểm A, E, P, F cùng nm trên một đường tròn .
2) Gi s đường thng AD cắt đường tròn
( )
O
ti Q khác A, đường thng AF cắt đường thng QC
ti L. Chng minh tam giác ABE đồng dng vi tam giác CLF.
3) Gọi K là giao điểm của đường thẳng AE và đường thng QB. Chng minh:
QKL PAB QLK PAC+ = +
.
(tuyn sinh lớp 10, THPT chuyên, ĐHKHTN, Đại hc Quc Gia Hà Nội, năm học 2014 - 2015)
Bài 22. Cho đường tròn
( )
;OR
dây cung BC không đi qua tâm. Gọi A điểm chính gia ca
cung nh BC. Góc ni tiếp
EAF
quay quanh điểm A s đo bằng
không đổi sao cho E, F
khác phía với điểm A so vi BC; AFAE cắt đưng thng BC lần lượt ti MN. Ly điểm D sao
cho t giác MNED là hình bình hành.
a) Chng minh MNEF là t giác ni tiếp.
b) Gi I tâm đường tròn ngoi tiếp tam giác MDF. Chng minh rng khi góc ni tiếp
EAF
quay
quanh điểm A thì I chuyển động trên một đường thng c định.
c) Tìm độ dài nh nht của đoạn thng OI khi
60
=
BC R=
.
(thi hoc sinh gii lp 9, tnh Phú Thọ, năm học 2013 - 2014)
CHUYÊN ĐỀ BỒI DƯỠNG HC SINH GII HÌNH HC 9
THCS.TOANMATH.com | 26
Bài 23. Cho 3 điểm A, B, C c định nm trên một đường thng d (B nm gia A C). V đường
tròn tâm O thay đổi nhưng luôn đi qua B C (O không thuộc đường thng d). K AM AN
các tiếp tuyến với đường tròn tâm O ti M N. Gọi I trung đim ca BC, AO ct MN ti H
cắt đường tròn tại các điểm P và Q (P nm gia A và O), BC ct MN ti K.
1. Chứng minh 4 điểm O, M, N, I cùng nm trên một đường tròn.
2. Chứng minh điểm K c định khi đường tròn tâm O thay đổi.
3. Gọi D trung điểm ca HQ, t H k đường thng vuông góc vi MD cắt đường thng MP ti E.
Chứng minh P là trung điểm ca ME.
(thi hc sinh gii lp 9, tinh Thanh Hóa, năm học 2014 - 2015)
NG DN GII - ĐÁP SỐ
1.
a) AB đường kính đường tròn
( )
90ADBO =
ADB DBC=
(so le trong)
90DBC =
.
Mt khác
90DMC =
suy ra:
90DMC DBC= =
do đó tứ giác CBMD ni tiếp đường tròn đường
kính CD.
Nhn xét. Ngoài cách gii trên, chúng ta có th gii
theo hướng sau:
Ta có:
MDB DBN DAN MCB===
.
Suy ra điều phi chng minh.
Ta có:
DMB DNB=
;
DAB DCB=
180DAB DNB+ =
.
Suy ra điều phi chng minh.
b) Khi điểm D di động trên đường tròn
( )
O
thì t
giác CBMD luôn là t giác ni tiếp
Suy ra
180BMD BCD+ =
(điều phi chng
minh).
c) Do
90ANB =
thuc
( )
O
.
Ta có:
BDN BAN=
(góc ni tiếp) mà
ACD BAN=
(so le trong)
BDN ACD=
.
Mt khác
DAC DAN DBN==
(cùng chn cung DN)
Suy ra:
ACD BDN
(g.g)
..
AC CD
AC DN BD CD
BD DN
= =
CHUYÊN ĐỀ BỒI DƯỠNG HỌC SINH GIỎI HÌNH HỌC 9
27 | THCS.TOANMATH.com
2.
a) Áp dng h qu góc to bi tia tiếp tuvến
và dây cung, ta có:
CAB ADB=
,
ACD BAD=
Suy ra:
ABD CBA
(g.g).
b) Vì
ABD CBA
, suy ra:
AD BD
CA BA
=
2
AD
DQ =
;
2
AC
AP =
BD DQ
BA AP
=
Li có:
QDB PAB=
Suy ra:
BQD APB
(c.g.c)
BQD APB=
.
c) Ta có:
180AQB BQD+ =
, mà
180BQD APB AQB APB= + =
Suy ra t giác APBQ ni tiếp.
3. Tia AO ct DE ti H.
Vì O là tâm đường tròn ni tiếp AABC nên
2.AOC B=
.
Suy ra
( )
1
180
2
OAC OCA AOC= =
( )
1
T giác BDEC ni tiếp nên
( )
2
Kết hp
( )
1
( )
2
ta có:
90 90OAC HAE B AED= = =
.
Suy ra:
90 90HAE AEH AHE+ = =
Hay AO vuông góc vi BC.
4.
a) Gi
1
O
; T;
2
O
thng hàng.
Các tam giác cân
1
OMB
3
O MN
chung góc M suy
ra
13
OMB O MN
1
3
MO
MB
MN MO
=
CHUYÊN ĐỀ BỒI DƯỠNG HC SINH GII HÌNH HC 9
THCS.TOANMATH.com | 28
Tương tự suy ra
13
OMA O MP
1
3
MO
MA
MP MO
=
Vy
//
MB MA
AB PN
MN MP
=
Tương tự ta có
//CD PM
.
Gọi E là giao điểm AB và CD .
T giác AEDP là hình bình hành.
Tacó:
EBC PNM=
;
ECB PMN=
nên
EBC PNM
(g.g)
( )
1
EB PN
EC PM
=
Ta có:
PTA PMT=
MPT
chung, nên
PAT PTM
(g.g)
2
.
PA PT
PA PM PT
PT PM
= =
Tương tự, ta có:
2
.PD PN PT=
..PAPM PD PN=
nên
PNM PAD
(c.g.c)
( )
2
Mà APDE là hình bình hành nên
EDA PAD =
( )
3
T
( )
1
, (2),
( )
3
suy ra:
EBC EDA EBC EDA =
Do đó tứ giác ABCD ni tiếp,
b) Gọi giao điểm ca PT và AB là I. Tia IC ct
( )
2
O
ti
D
Ta có:
2
..IA IB IT IC ID
==
suy ra
IDA IIBC BC IDA

=
Do đó tứ giác
ABCD
ni tiếp mà ABCD ni tiếp nên D trùng
D
Vậy các đường thắng AB, CD và PT đồng quy.
5.
a) Ta có EB, EM là tiếp tuyến nên
1
2
EOM BOM=
;
Ta có FC, FM là tiếp tuyến nên
11
22
FOM COM EOF BOC= =
;
Mt khác
11
22
EOF BOC sdBMC

==


Suy ra
EBQ EOQ=
T đó ta có O và B là hai đỉnh liên tiếp cùng nhìn EQ dưới mt góc bng nhau
Vy OBEQ là t giác ni tiếp.
CHUYÊN ĐỀ BỒI DƯỠNG HỌC SINH GIỎI HÌNH HỌC 9
29 | THCS.TOANMATH.com
Chứng minh tương tự ta có OCFP là t giác ni tiếp.
b) OBEQ là t giác ni tiếp nên
180 90 90 .OBE OQE OQE FQE+ = = =
OCFP t giác ni tiếp nên
180 90 90OCF OPF OPF EPF+ = = =
Suy ra
90EPF EQF= =
.
Vy t giác PQFE là t giác ni tiếp.
c) K OH vuông góc vi BC.
Ta có: PQFE là t giác ni tiếp
Suy ra
OPQ EFO=
Do đó
OPQ OFE
(g.g)
PQ OH
EF OM
=
điểm A
( )
O
c định nên OH OM không đổi do đó tỉ s
PQ
FE
không đổi khi M di chuyn
trên đường tròn.
4. T giác ADOE ni tiếp
EAO EDO=
.
Gi tia BO ct tia DE ti H thì:
180 180 90
2 2 2
A B C
BHD HDB HBD= = =
Mt khác
2
C
ACO =
nên t giác EOCH ni tiếp
90OHC OEC = =
.
Hay BH vuông góc vi CH.
Gọi M là trung điểm ca BC
Suy ra
MB MC MH BHM= =
cân
HBM MHB ABH MHB = =
Suy ra BH song song vi AB.
Suy ra điều phi chng minh.
7.
1. Ta có:
ACD ABD=
;
ABD AFB=
nên
ACD AFB=
.
Do đó tứ giác CDFE ni tiếp.
2. Gi I là tâm đường tròn ngoi tiếp t giác CDFE.
CHUYÊN ĐỀ BỒI DƯỠNG HC SINH GII HÌNH HC 9
THCS.TOANMATH.com | 30
Đưng tròn
( )
I
qua CD nên I thuc trung
trc ca CD.
Đưng tròn
( )
I
qua EF nên I thuc trung
trc ca EF.
Gọi H là trung điểm ca EF.
Do đó I giao điểm hại đường trung trc
ca CD và EF
//AO HI
hoc trùng vi HI (cùng vuông
góc vi EF)
( )
1
Tam giác AEF vuông, AH trung tuyến
ng vi cnh huyn nên
HA HE HAE=
cân ti H
HAE HEA HAE ADC = =
90ADC ACD+ =
nên
90HAE ACD+ =
Suy ra
AH CD
.
OI CD
nên
//AH OI
( )
2
T
( )
1
( )
2
, suy ra t giác AOIH hình bình hành. Do đó
IH OA R==
. Suy ra I cách EF mt
khoảng không đổi bng R, nên I di động trên đưng thng d song song vi EF cách EF mt
khong bng R.
5. Ta có:
90BFD BED BAD= = =
.
Do đó B, E, D, A, F cùng thuc một đường tròn đường
kính BD.
Trong tam giác vuông ABC có AM lcà cnh huyn nên
MA MC=
MAC
cân ti M
MAC MCA=
.
Xét đường tròn đi qua năm điểm A, B, E, D, F
Ta có
DE DF=
nên
DF DE DBE DBF= =
Xét:
NAF MAC DAF MCA DBF MCA DBE BDA NFA= + = + = + = =
NAF
cân ti N
NF NA=
.
CHUYÊN ĐỀ BỒI DƯỠNG HỌC SINH GIỎI HÌNH HỌC 9
31 | THCS.TOANMATH.com
9.
a) Ta
BDE BAE=
(cùng chn cung BE ca
đường tròn tâm O)
BDE BMN=
(cùng chn cung BN của đường
tròn tâm
O
)
BDE BMN=
hay
BDI BMN=
T giác
BDMI ni tiếp
MDI MBI=
(cùng chn cung MI)
MDI ABE=
(cùng chn cung AE ca
đường tròn tâm O)
ABE MBI=
Mt khác:
BMI BAE=
MBI ABE
(g.g)
..
MI BI
MI BE BI AE
AE BE
= =
b) Gọi Q là giao điểm ca CO và DE.
Ta có
OC DE
ti Q
OCD
vuông tại D , có đường cao là DQ nên
22
.OQOC OD R==
( )
1
Gọi K là giao điểm của hai đường thng
OO
và DE, H là giao điểm ca AB và
OO
Ta có:
OO AB
ti H.
KQO CHO
(
90QH= =
;
O
chung)
..
KO OQ
OC OQ KOOH
CO OH
= =
( )
2
T
( )
1
( )
2
, suy ra:
2
2
.
R
KOOH R OK
OH
= =
Vì OH c định và R không đổi nên OK không đổi. Do đó K cố định.
10. Ta có:
45PIB IAB IBA IBC= + = +
( )
1
Mt khác
( )
1
180
2
PNB CNM ACB= =
( )
1
90 90
2
ACB

= +

( )
1
90
2
ACB= +
CHUYÊN ĐỀ BỒI DƯỠNG HC SINH GII HÌNH HC 9
THCS.TOANMATH.com | 32
1
45 45
2
ABC IBC= + = +
( )
2
T
( )
1
( )
2
, suy ra:
PIB PNB=
.
Do đó 4 điểm P, N, I, B cùng nm trên một đường tròn.
Mt khác
90INB =
nên IB là đường kính của đường tròn này
90IPB =
11.
a) Ni CP, PD .
Ta có A, C, O thng hàng; B, D, O thng hàng.
Ta có:
ACP
,
OAB
lần lượt cân ti C, O nên
CPA CAP OBP==
.
Do đó
//CP OD
( )
1
Tương tự, ta có
//OD CP
( )
2
.
T
( )
1
( )
2
suy ra t giác ODPC là hình bình hành.
Gọi H giao điểm của CD MP, K giao điểm ca CD
OP.
Do đó K là trung điểm ca OP.
Theo tính cht của hai đường tròn ct nhau thì
CD MP
H là trung điểm ca MP.
Do đó
// //HK OM CD OM
.
Gi s
AP BP
.
t giác CDOM nh bình hành nên
OC DP=
;
2
DP DM R==
nên t giác CDOM hình
thang cân.
Do đó 4 điểm C, D, O, M cùng thuc một đường tròn.
b) Ta có:
2 2 2 2
2OA OB R AB+ = =
. Do đó
AOB
vuông cân ti O.
Vì 4 điểm C, D, O, M cùng thuc một đường tròn (K c M trùng O) nên
COB CMD=
( )
1
Ta có:
MAB MCD=
(cùng bng
1
2
sd MP
của đường tròn
( )
C
)
MBP MDC=
(cùng bng
1
2
sd MP
của đường tròn
( )
D
).
Do đó
MAB MCD
(g-g)
90AMB AOB = =
CMD COD=
(t giác CDOM ni tiếp).
Do AB c định nên điểm M thuộc đường tròn tâm I đường kính AB.
Ta có:
1
90 45
2
ACP BDP AOB AMP ACP= = = = =
(Góc ni tiếp và góc tâm ca
( )
C
)
CHUYÊN ĐỀ BỒI DƯỠNG HỌC SINH GIỎI HÌNH HỌC 9
33 | THCS.TOANMATH.com
1
45
2
BMP BCP = =
(góc ni tiếp và góc tâm ca
( )
D
)
Do đó MP tia phân giác của
AMB
.
90AMB AOB= =
nên M thuc đường tròn
( )
I
ngoi
tiếp tam giác AOB. Gi s MP cắt đường tròn
( )
I
tại N N trung điểm cung AB không cha
điểm O nên N c định.
c) Ta có:
MPA BPN=
;
AMP PBN=
(góc ni tiếp cùng chn mt cung)
Do đó
MAP BNP
(g - g)
2
22
..
2 4 2
PA PM PA PB AB R
PM PN PA PB
PN PB
+

= = = =


(không đổi)
Vy PM.PN ln nht
2
2
R
khi
PA PB=
hay P trung điểm ca dây AB. Tam giác AMB vuông
ti M nên:
( )
22
22
11
.
2 4 4 2
AMB
AB R
S AM BM AM BM= + = =
Vy
ABM
S
ln nht là
2
2
R
khi
PA PB=
hay P là trung điểm ca dây AB.
12.
a) EF là đường kính nên
90EAF =
AE MN
suy ra
//AF MN QPN QFA=
.
Mà AFQB ni tiếp nên
180QFA QBA+ =
180QPN QBN + =
.
Suy ra t giác PQBN ni tiếp.
Li có
QCA QFA QPN QCM= =
Suy ra t giác PQCM ni tiếp.
b) Gi s QN và PC ct nhau ti R thuc
( )
O
.
T t giác PQBN ni tiếp suy ra
NPB NQB BCP==
.
T t giác PMCQ ni tiếp ta có:
PBC RPB PCB RPN NPB NPB RPN MPC MQC= = + = = =
T đó nếu QM ct BP tại điểm S thì SBQC ni tiếp hay S thuc
đường tròn
( )
O
.
13.
CHUYÊN ĐỀ BỒI DƯỠNG HC SINH GII HÌNH HC 9
THCS.TOANMATH.com | 34
a) Ta có:
BKA ACB=
(2 góc ni tiếp cùng chn cung AB)
ACB BHK=
(cùng ph vi góc EBC)
BKA BHK=
tam giác BHK cân
BH BK=
Lp luận tương t ta có
CH CK=
BC là trung trc ca HK.
Ta có:
90AEH AFH= =
T giác AFHE ni tiếp.
Xét tam giác AIE và tam giác FIH ta có:
AIE FIH=
(2 góc đối đỉnh),
IAE IFH=
(T giác AFHE ni tiếp)
AIE FIH
(g.g)
..
AI FI
AI HI EI FI
EI HI
= =
b) Xét t giác DHEC ta có:
90HDC BEC= =
T giác DHEC ni tiếp .
Xét t giác BFEC ta có:
90BFC BEC= =
T giác BFEC ni tiếp
AFE ACB=
ACB AKB=
(chng minh trên)
AFE AKB =
T giác KBFI ni tiếp .
c) Theo như trên ta đã có:
BKH BHK=
BHK IHE=
(2 góc đối đỉnh)
BKH IHE=
.
Xét tam giác HEI và tam giác KAB ta có:
BKH IHE=
(cmt),
IHE BAK=
(t giác AFHE ni tiếp)
HEI KAB
(g.g)
KB HI
AB EI
=
Chứng minh tương tự ta có:
KC HI
AC FI
=
T đó suy ra
1 1 .
.
..
KB KC EI FI IH EF EF
IH IH
AB AC EI FI EI FI AI HI AI
+

+ = + = = =


(theo chng minh câu a
..IF IE IH IA=
).
d) Ta có:
BME BKH=
(2 góc v trí đồng v do
//HK ME
)
BKH BHK=
;
BKH BME=
(2 góc v trí đồng v do
//HK ME
)
BME BEM=
Tam giác BEM là tam giác cân.
Ta có:
AD BC
//EM BC EM BC⊥
.
CHUYÊN ĐỀ BỒI DƯỠNG HỌC SINH GIỎI HÌNH HỌC 9
35 | THCS.TOANMATH.com
Trong tam giác cân BEM có BC là đường cao ca tam giác (do
BC ME
)
BC là trung trc ca ME.
Ta có D nm trên đường trung trc ca
ME DM DE=
Tam giác DME là tam giác cân
MDC EDC=
.
Xét t giác ABDE ta có:
90ADB AEB= =
T giác ABDE ni tiếp
EDC BAC=
.
Xét t giác AFDC ta có:
90AFC ADC= =
T giác AFDC ni tiếp
BAC BDF=
.
T đó suy ra
MDC BDF=
Ta có:
180 BDC MDC BDM BDF BDM FDM = = + = + =
Ba điểm F, D, M thng hàng.
14.
a) Ta
ABF
;
ACE
các tam giác cân ti F
E
FBA BAD DAC ECA ABF ACE= = =
.
b) Gọi G là giao điểm ca BE và CF.
Ta có:
GF BF AB DB
GC CE AC DC
= = =
//DG BF
Mt khác
//DA BF
suy ra A, D, G thng hàng.
Suy ra điều phi chng minh.
c) Ta có
BQG QGA GAE GAC= = =
GAC CAE GAB BAF GAF= + = + =
Suy ra AGQF là t giác ni tiếp.
Mt khác
QPG GCE GFQ==
nên QGPF t giác ni tiếp. Suy
ra điều phi chng minh.
15. Gọi R là giao điểm ca BN và CM.
Ta thy
ABC PAC QBA∽∽
Do đó
BQ PA QB PM
QA PC QN PC
= =
.
CHUYÊN ĐỀ BỒI DƯỠNG HC SINH GII HÌNH HC 9
THCS.TOANMATH.com | 36
Mt khác
MPC NQB=
nên
MPC BQN
BNQ PCM=
T giác QCNR ni tiếp
CRN CQN BAC = =
.
Vy t giác ABQR ni tiếp, suy ra điều phi chng minh.
16. Chia lục giác đều ABCDEF tâm O thành 6 tam giác đều
cnh 4cm (hình v).
Theo nguyên lý Điriclê có ít nhất 4 điểm trong 19 điểm nm
trong hay trên cnh một trong 6 tam giác đó. Không mất
tính tng quát gi s tam giác đó là OAB.
Chia tam giác đều OAB trng tâm G thành 3 t giác ni tiếp
(hình v) vi
GM AB
;
GN OB
;
GP OA
.
OAB
đều cnh bằng 4 đường cao
4 3 4 3
23
23
GA= =
Các t giác GMBN, GMAP, GPON ni tiếp trong đường
tròn đường kính GB, GA, GO đều bng
43
3
Theo nguyên lý Điriclê ít nhất 2 điểm trong 4 điểm đang xét nằm trong hay trên cnh mt trong
4 t giác nói trên, gi s t giác đó là GMBN
khong cách giữa hai điểm đó không vượt quá đường kính
43
3
GB =
ca đường tròn ngoi
tiếp t giác
điều phi chng minh.
14. Gọi H giao điểm cúa AN MD, K giao điểm cua BN MC, I giao đim ca MN
CD.
AMD
vuông tại A, AH là đường cao
2
.AM MH MD=
.
BMC
vuông tại B, BK là đường cao
2
.BM MK MC=
.
AM BM=
Do đó
..
MH MK
MH MD MK MC
MC MD
= =
MKH
MDC
KMH
chung,
MH MK
MC MD
=
MKH MDC
(c.g.c)
MKH IDC=
.
T giác MKNH
90 90 180MKN MHN+ = +
t giác
MKNH ni tiếp
MKH MNH=
, ta
CHUYÊN ĐỀ BỒI DƯỠNG HỌC SINH GIỎI HÌNH HỌC 9
37 | THCS.TOANMATH.com
( )
MNH IDN MKH= =
t giác HNID ni tiếp
90MIC NHD = =
. Vy
MN CD
.
18.
a) Ta E, M, O, F thng hàng,
ME MF=
(E, F đối xng
qua M)
EF BC
BEF
cân ti B
BFE FEB=
.
Mt khác
OB OE=
suy ra
OBE
cân ti O
OBE OEB=
.
Ta có
BFE FEB OBE==
BEF OBE
(g.g)
2
.
EB EF
EB EF EO
OB EB
= =
b) Không gim tính tng quát xét O nm gia M và F.
D thy
FBD EAB
(g.g)
2
.
EB ED
EB ED EA
EA EB
= =
Ta có
( )
2
..ED EA EF EO E
EO ED
EF
B
AE
= ==
.
Xét
EOD
EAF
EO ED
EA EF
=
,
OED
chung
EOD EAF
(c.g.c)
EOD EAF=
, dẫn đến t giác DAFO ni tiếp. Vy các điểm A, D, O, F cùng thuc một đường
tròn.
c) Ta có
EIB ABI BAI=+
,
ABI IBC=
,
( )
BAI CBE EB EC==
EBI IBC CBE ABI BAI EIB EBI = + = + =
cân ti E
EB EI=
EB EC=
nên
EB EI EC= =
E là tâm đường tròn ni tiếp
tam giác IBC.
Do đó
EP EB=
nên
2
.EP EF EO=
.
Xét
EPO
EFP
EP EO
EF EP
=
,
PEO
chung
EPO EFP
(c.g.c)
EPO EFP =
EP tiếp tuyến của đường tròn ngoi tiếp
tam giác POF.
Vy tiếp tuvến ca đường tròn ngoi tiếp tam giác POF đi qua
điểm E c định.
19.
CHUYÊN ĐỀ BỒI DƯỠNG HC SINH GII HÌNH HC 9
THCS.TOANMATH.com | 38
a)
BE CA
,
CF AB
nên BCEF t giác ni tiếp
XFB ACB XBF XFB = =
cân ti
X.
MFB
cân ti M, suy ra
MX BF
.
b) D thy
MX AB
,
HF AB
nên
//MX HF
;
MS BC
,
HD BC
nên
//MS HD
.
Mt khác, do CAFD là t giác ni tiếp và SB tiếp xúc vi
( )
O
ti B nên
SBD BAC BDF==
Suy ra
//SX FD
. Do đó
MXS HFD
(có cặp canh tươngng song song).
c) Ta có:
180
90 90
2
AOC
OAE ABC AEF
−
= = =
, suy ra
OA EF
.
D dàng chứng minh được
AEF ABC
;
AFY ACD
.
Suy ra
FY AF EF EF BC
CD AC BC FY CD
= = =
20.
1) H là trung điểm ca BC nên
OH BC
.
BMO CNO =
(cgc) suy ra
OM ON=
( )
1
Tam giác OMN cân ti O, I là trung điểm ca MN nên
OI MN
( )
2
.
T
( )
1
( )
2
suy ra 4 điểm O, M, H, I cùng thuộc đường tròn đường kính OM.
2) Chng minh t giác ABON ni tiếp (vì
60 120 180PAN PON+ = + =
)
Suy ra
30OPN OAN= =
Chứng minh tương lự.
30OPM =
, suy ra
60MPN =
.
Kết hp vi
MP NP=
(P thuộc đường trung trc OI cua MN )
Suy ra tam giác MNP đều.
3) T câu 1 suy ra
60IHC IOM ABC= =
nên
//IH AB
.
Suy ra đường thng IH c định. Gọi K là trung điểm ca AC suy ra H, I, K thng hàng.
Ly điểm T đối xng vi A qua HI suy ra T c định.
Ta có
AI BI AB TI IB AB BT AB const+ + = + + + =
.
Chu vi tam giác AIB nh nht bng
BT AB+
, đạt được khi ba điểm B, I,T thng hàng.
Khi đó I là trung điểm ca BT c định (theo tính chất đường trung bình
BAT
).
Suy ra t giác BMTN là hình bình hành và
//TN BC
.
Li có
BH KT=
,
BK MT=
Suy ra t giác BHTK là hình bình hành và
//TK BC
.
T đó
NK
, suy ra
MH
.
21.
1) Chng minh bốn điểm A, E, P, F cùng nm trên một đường tròn
CHUYÊN ĐỀ BỒI DƯỠNG HỌC SINH GIỎI HÌNH HỌC 9
39 | THCS.TOANMATH.com
Ta có
AEP ADB=
(chn cung AB ca
( )
ABD
):
Ta có
AFP ADC=
(chn cung AC ca
( )
ADC
);
Nên
AEP AFP ADB ADC+ = +
nên AEFP ni tiếp
2) Gi s đường thng AD cắt đường tròn
( )
O
tại Q khác A, đường thng AF cắt đường thng QC
ti I
Chng minh tam giác ABE đồng dng vi tam giác CLF
Xét
ABE
;
CLF
AEB CFL=
(cùng
AFP
)
( )
1
ta li
BAE BAD DAE=+
;
FCL BCL FCB=+
BAD BCL=
;
DAE FCB=
;
Nên
BAE FCL=
( )
2
t
( )
1
( )
2
suy ra
ABE CLF
(g.g)
3) Gọi K giao điểm của đường thẳng AE đường
thng QB chng minh
QKL PAB QLK PAC+ = +
Theo 2)
ABE CLF
nên
..LF AE BE CF=
Ta li có
..KE AF BECF=
Suy ra
..KE AF LF AE=
//
LF KE
EF LK
AF AE
=
Nên
AEF AKL=
AEF APF APF AKL= =
Nên
PAC PCA EKP QKL+ = +
PCA EKP PAC QKL= =
.
Tương tự
PAB QLK=
, suy ra
QKL PAB QLK PAC+ = +
.
22.
a) Ta có
( )
1
2
MNE sd AB sd BEF=+
( )
1
2
sd AC sdBEF=+
( )
1
2
AFE sd AB sdBE=+
Suy ra:
180MNE MFE+ =
Vy t giác MNEF ni tiếp.
CHUYÊN ĐỀ BỒI DƯỠNG HC SINH GII HÌNH HC 9
THCS.TOANMATH.com | 40
b) Gi P là giao điểm khác A ca AO với đường tròn
( )
;OR
.
Ly G đối xng vi E qua AP
D EG
,
( )
GO
Ta có
MDG NEG=
,
AEG AFG MDG MFG= =
Suy ra t giác MDGF ni tiếp
( )
1
Gọi giao điểm ca AGBCH.
Chứng minh tương tự a) có t giác MHGF ni tiếp
( )
2
T
( )
1
( )
2
suy ra các điểm M, H, D, G, F nm trên một đường tròn.
Trung trc của đoạn thng FG đi qua O cắt đường tròn
( )
O
ti J;
I OJ
,
sdJF sdJG=
sđ PG sđ PE=
nên
JOP
=
hay I nằm trên đường thng c định. Đó đường thẳng đi qua O
to vi AO mt góc
không đổi.
c) H
( )
IT BC T BC TH TM =
.
Gi Q là giao điểm ca BC và AP.
Do
QH QN=
Suy ra
QT QH MH=−
( )
11
22
NM MH MN−=
K
1
2
IS AP IS QT MN = =
Tam giác vuông OSI
IOS
=
không đổi nên OI nh nht khi và ch khi IS nh nht
MN nh
nht.
Ta chng minh MN nh nht khi và ch khi tam giác AMN cân ti A.
Tht vy, trên BC ly
MN

sao cho
MAN

=
.
Không mt tính tng quát gi s
QM QN

Suy ra
AM AN

.
Trên đoạn
AM
lấy điểm U sao cho
AU AN
=
'AUM ANN =
(c.g.c)
AMM ANN
S S MM NN MN MN

Vi
60
=
;
BC R=
suy ra
( )
23
3
22
R
R
AQ R
= =
( ) ( ) ( )
2 3 2 3 3 2 3 3
2
.
2 3 6
3
R R R
MN OI
= = =
CHUYÊN ĐỀ BỒI DƯỠNG HỌC SINH GIỎI HÌNH HỌC 9
41 | THCS.TOANMATH.com
23.
1. I là trung điểm của BC (Dây BC không đi qua O)
90OI BC OIA =
Ta có
90AMO =
,
90ANO =
Suy ra 4 điểm O, M, N, I cùng thuộc đường tròn đường kính OA.
2. AM, AN hai tiếp tuyến ca
( )
O
nên OA phân giác
MON
MON
cân O nên
OA MN
ABN ANC
(Vì
ANB ACN=
,
CAN
chung)
2
.
AB AN
AB AC AN
AN AC
= =
ANO
vuông tại N đường cao NH nên
2
.AH AO AN=
..AB AC AH AO=
AHK AIO
(g-g) nên
AH AK
AI AO
=
..AI AK AH AO=
..AI AK AB AC=
.AB AC
AK
AI
=
Ta A, B, C c định nên I c định
AK
c định
Mà A c định, K là giao điểm ca dây BC và dây MN nên K thuc tia AB
K c định
3) Ta có
90PMQ =
Ta có:
MHE QDM
(g-g)
ME MH
MQ DQ
=
Ta có:
PMH MQH
2
MP MH MH
MQ QH DQ
= =
1
.2
2
MP ME
ME MP
MQ MQ
= =
P là trung điểm ME.
C.T LUYN
Bài 1. Trên cnh huyn
BC
ca tam giác vuông
ABC
v phía ngoài ta dng hình vuông vi tâm
tại điểm
O
. Chng minh rng
AO
là tia phân giác ca góc
BAC
.
Li gii:
O
là tâm ca hình vuông nên
0
90BOC
.
N
M
O
C
B
A
CHUYÊN ĐỀ BỒI DƯỠNG HC SINH GII HÌNH HC 9
THCS.TOANMATH.com | 42
Li có
0
90BAC
suy ra bốn điểm
, , ,A B O C
cùng nằm trên đường tròn đường kính BC.
Đối với đường tròn y ta thy
BAO BCO
(cùng chn
BO
). Mà
00
45 45BCO BAO
.
Do
0
90BAC
, nên
0
45CAO BAC BAO
. Vy
BAO CAO
, nghĩa
AO
tia phân
giác ca góc vuông
BAC
(đpcm).
Bài 2. Cho tam giác nhn
ABC
ni tiếp đường tròn
O
. T đỉnh
A
ta k đường cao
AH
(
H
thuc
BC
). Chng minh rng
BAH OAC
.
Li gii:
K đường kính
AE
của đường tròn
O
. Ta thy
0
90ACE
(góc ni tiếp chn nửa đường tròn).
T đó
0
90OAC AEC
(1).
Theo gi thiết bài ra, ta có:
0
90BAH ABC
(2). Li vì
AEC ABC
(cùng chn
AC
) (3).
T (1),(2) và (3) suy ra
BAH OAC
(đpcm).
E
H
O
D
C
B
A
CHUYÊN ĐỀ BỒI DƯỠNG HỌC SINH GIỎI HÌNH HỌC 9
43 | THCS.TOANMATH.com
Lưu ý: Cũng th giải bài toán theo hướng sau: Gi
D
giao điểm ca tia
AH
với đường tròn
O
, chng t t giác
BDEC
hình thang cân. T đó suy ra
đđssBD CE
, dẫn đến
BAD CAE
, hay
BAH OAC
.
Bài 3. Cho tam giác đều
ABC
ni tiếp đường tròn
O
. Trên cung
BC
không cha
A
ta ly điểm
P
bt k (
P
khác
B
P
khác
C
). Các đoạn
PA
BC
ct nhau ti
Q
.
a) Gi s
D
là một điểm trên đoạn
PA
sao cho
PD PB
. Chng minh rng
PDB
đều.
b) Chng minh rng
PA PB PC
.
c) Chng minh h thc
1 1 1
PQ PB PC
.
Li gii:
a) Trước tiên ta nhn thy rng tam giác
PBD
cân ti
P
. Mt khác,
0
60BPD BPA BCA
(hai góc ni tiếp cùng chn
AB
của đường tròn
O
). Vy nên tam giác
PDB
đều.
b) Ta đã
PB PD
, vy đ chng minh
PA PB PC
ta s chng minh
DA PC
. Tht
vy, xét hai tam giác
BPC
BDA
có:
BA BC
(gi thiết),
BD BP
(do tam giác
BPD
đều). Li
0
60ABD DBC
,
0
60PBC DBC
nên
ABD PBC
. T đó
BPC BDA
(c.g.c), dẫn đến
DA PC
(đpcm).
c) Xét hai tam giác
PBQ
PAC
ta thy
0
60BPQ
,
0
60APC ABC
(hai góc ni tiếp
cùng chn cung
AC
) suy ra
,BPQ APC PBQ PBC PAC
(hai góc ni tiếp cùng chn
PC
). T đó
PBQ PAC
(g.g)
PQ PC
PB PA
, hay
..PQ PA PB PC
. Theo kết qu câu
P
O
Q
D
C
B
A
CHUYÊN ĐỀ BỒI DƯỠNG HC SINH GII HÌNH HC 9
THCS.TOANMATH.com | 44
b
, ta
PA PB PC
nên
.PQ PB PC PB PC
. H thức này tương đương với
1 1 1
PQ PB PC
(đpcm).
Ghi chú:
- T giác
ABCD
có tính cht
..ABCD BC AD
(*) nói ví d trên được gi là t giác điều hòa.
Loi t giác đặc bit này có nhiu ng dng trong vic gii các bài toán hình hc phng khác.
- Nếu h thức (*) dưới dng
AB BC
AD CD
và nh li tính chất đường phân giác trong tam giác ta
th nêu thêm mt tính cht ca t giác điều hòa.
- T giác
ABCD
mt t giác điều hòa khi ch khi các đường phân giác ca góc
BAD
BCD
ct nhau ti một điểm trên đường chéo
BD
.
- T giác
ABCD
t giác điu hòa khi ch khi đường phân giác ca góc
ABC
ADC
ct
nhau trên đường chéo
AC
.
Bài 4. Cho tam giác
ABC
ni tiếp trong đường tròn
()O
. Đường phân giác trong góc
A
cắt đường
tròn ngoi tiếp tam giác ti
D
. Gi
I
tâm vòng tròn ni tiếp tam giác
ABC
. Chng minh
DB DC DI
Li gii:
Ta luôn
DB DC
do
AD
phân giác trong góc
A
. Ta s chng minh tam giác
DIB
cân ti
D
.
Tht vy ta có:
IBD IBC CBD
.
Mt khác
CBD CAD
(Góc ni tiếp chn cung
CD
) mà
BAD CAD
,
IBC IBA
(Tính cht phân giác) suy ra
IBD ABI BAI
. Nhưng
BID ABI BAI
(Tính chất góc ngoài). Như vậy tam giác
BDI
cân ti
D DB DI DC
Nhn xét: Thông qua bài toán này ta có thêm nh chất: Tâm đường tròn ngoi tiếp tam giác
IBC
là giao điểm ca phân giác trong góc
A
vi
()O
Bài 5. Cho tam giác nhn
ABC
ni tiếp đường tròn
()O
AB AC
. Lấy đim
M
thuc cung
BC
không chứa điểm
A
. V
,,MH MK MI
lần lượt vuông góc vi
BC AC AB
MH MK MI
O
I
D
C
B
A
CHUYÊN ĐỀ BỒI DƯỠNG HỌC SINH GIỎI HÌNH HỌC 9
45 | THCS.TOANMATH.com
Li gii:
Trong bài toán có các t s độ dài
ta ngh đến các tam giác đồng dng
và định lý Thales.
Cách 1: Dựng đường thng qua
A
song song vi
BC
ct
()O
ti
N
. Gi
E
là giao điểm ca
BC
MN
Ta có:
AB NC
.
Ta
đđ
11
22
BME BMN s AB AN s NC AN AMC
,
MBC MAC BME AMC
,MH MK
hai đường cao tương ng nên:
AC BE
MK MH
, chứng minh tương tự ta cũng có:
AB CE
MI MH
. Cộng hai đng thc trên ta có:
BC AC AB
MH MK MI
Cách 2: Ta thy
,MH MI
các đường cao ca tam giác
,MBC MAB
nhưng hai tam giác y
không đng dng với nhau. Điều y giúp ta ngh đến vic ly một điểm
E
trên cnh
BC
sao cho
BMA DMC
để tạo ra tam giác đồng dạng nhưng vẫn gi được hai đường cao tương ng. (Phn
li gii xin dành cho bạn đọc).
Bài 6. Gi s
A
B
hai điểm phân biệt trên đường tròn
O
. Các tiếp tuyến của đường tròn
O
. Các tiếp tuyến của đường tròn
O
ti
A
B
ct nhau tại điểm
M
. T
A
k đường thng
song song vi
MB
cắt đường tròn
O
ti
C
.
MC
cắt đường tròn
O
ti
E
. Các tia
AE
MB
ct nhau ti
K
.
Chng minh rng
2
.MK AK EK
MK KB
.
Li gii:
Do
//MB AC
nên
BMC ACM
(1), ta li có
ACM ACE MAE
(cùng chn
AE
) (2). T (1) và (2)
E
N
O
I
K
M
H
C
B
A
O
K
E
M
C
B
A
CHUYÊN ĐỀ BỒI DƯỠNG HC SINH GII HÌNH HC 9
THCS.TOANMATH.com | 46
suy ra
KME KAM
(g.g)
MK EK
AK MK
hay
2
.MK AK EK
(3). Ta thy
EAB EBK
(cùng chn
BE
). T đó
EBK BAK
(g.g)
BK EK
AK BK
hay
2
.BK AK EK
(4). T (3) và (4) suy ra
22
MK KB
nghĩa là
MK MB
(đpcm).
Bài 7. Cho đường tròn
C
tâm
O
,
AB
mt y cung ca
C
không đi qua
O
I
trung
điểm ca
AB
. Một đường thẳng thay đổi đi qua
A
cắt đường tròn
1
C
tâm
O
bán kính
OI
ti
P
Q
. Chng minh rng tích
.AP AQ
không đổi đường tròn ngoi tiếp tam giác
BPQ
luôn đi
qua một điểm c định khác
B
.
Li gii:
Ta
PQI PIA
(cùng chn
PI
), nên
API AIQ
(g.g). Suy ra
2
.
AP AI
AP AQ AI
AI AQ
(không đổi). Gi s đưng tròn ngoi tiếp tam giác
BPQ
ct
AB
ti
D
DB
.
Khi đó
ADP AQB
, suy ra
AD AP
AQ AB
hay
2
..AD AB AP AQ AI
(không đổi). Do đó điểm
D
là điểm
c định (đpcm).
Bài 8. Cho tam giác nhn
ABC
trc tâm
H
0
60BAC
. Gi
,,M N P
theo th t chân
các đường cao k t
,,A B C
ca tam giác
ABC
I
trung điểm ca
BC
.
a) Chng minh rng tam giác
INP
đều.
b) Gi
E
K
lần lượt trung đim ca
PB
NC
. Chng minh rằng các điểm
, , ,I M E K
cùng thuc một đường tròn.
c) Gi s
IA
là phân giác ca
NIP
. Tìm s đo
BCP
.
Li gii:
a). T gi thiết ta có
D
Q
P
I
B
A
O
CHUYÊN ĐỀ BỒI DƯỠNG HỌC SINH GIỎI HÌNH HỌC 9
47 | THCS.TOANMATH.com
1
2
IN IP BC
nên tam giác
E
H
I
P
N
M
C
B
A
INP
cân ti
I
. Li vì
, , ,B P N C
nằm trên đường tròn tâm
I
, đường kính
BC
nên theo mi liên h gia góc ni tiếp góc tâm
cùng chn mt cung, ta thy
0
2 60PIN PBN
. Vy tam giác
INP
đều.
b) Rõ ràng bốn điểm
,,I M E
K
cùng nằm trên đường tròn đường kính
AI
.
c) T điều kin ca bài toán ta thy
AI
tia phân giác ca
0
60BAC
,
I
là trung điểm ca
BC
nên tam giác
ABC
đều. T đó suy ra
0
30BCP
.
Bài 9. Cho tam giác cân
,( )ABC AB AC
. Gi
O
trung đim ca
BC
. Dựng đường tròn
()O
tiếp xúc vi các cnh
,AB AC
ti
,DE
.
M
điểm chuyn động trên cung nh
DE
tiếp tuyến
với đường tròn
()O
ti
M
ct
,AB AC
ti
,PQ
. Chng minh
2
4.BC BPCQ
tìm v trí điểm
M
để din tích tam giác
APQ
ln nht.
Li gii:
Ta thy
ABC
S
không đổi nên
APQ
S
ln nht khi và ch khi
BPQC
S
nh nhất, đây là cơ sở để ta làm
xut hin các biu thc có liên quan
Q
P
O
C
B
A
M
E
D
đến
,BP CQ
. Ta có
,,AB PQ AC
CHUYÊN ĐỀ BỒI DƯỠNG HC SINH GII HÌNH HC 9
THCS.TOANMATH.com | 48
lần lượt là các tiếp tuyến tại các điểm
,,D M E
ca
()O
nên ta có:
, , ,AB OD PQ OM AC OE BD CE
. T đó ta tính được:
11
22
22
BPQC
S R BP PQ CQ R BD DP EQ CE
.R BD DP EQ R BP CQ BD
.
Mặt khác ta cũng có:
0
11
180
22
POQ DOE A B C
nên suy ra
00
180 180BOP POQ QOC QCO QOC CQO
BPO COQ
2
..
4
BP BO BC
BP CQ BOCO
CO CQ
. Theo bất đẳng thc si ta :
2.BP CQ BPCQ BC
.
BPQC
S R BC BD
. Vy
BPQC
S
nh nht khi
BP CQ M
là trung điểm ca cung
DE
.
Bài 10. Trên đường tròn
O
cho các đim
,,,A B C D
theo th t đó. Gọi
1 1 1 1
,,,A B C D
lần lượt
điểm chính gia ca các cung
,,AB BC CD
DA
. Chứng minh các đường thng
11
AC
11
BD
vuông góc vi nhau
Li gii:
I
D
1
C
1
B
1
A
1
O
D
C
B
A
Gi
I
giao điểm ca
11
AC
11
BD
;
, , ,
theo th t s đo của các cung
, , ,AB BC CD DA
. Khi đó
0
360
. Xét góc
11
A IB
là góc có đỉnh nm
trong đường tròn
O
. Ta có
đđ
1 1 1 1 1 1
1
ss
2
AIB ABB C DD
đ đ đ đ
1 1 1 1
1
s s s s
2
AB BB C D DD
0
1
90
4
.
Nghĩa là
1 1 1 1
AC B D
(đpcm).
CHUYÊN ĐỀ BỒI DƯỠNG HỌC SINH GIỎI HÌNH HỌC 9
49 | THCS.TOANMATH.com
Bài 11. Cho bốn điểm
, , ,A D C B
theo th t đó nằm trên đường tròn tâm
O
đường kính
2AB R
(
C
D
nm v cùng mt phía so vi
AB
). Gi
E
F
theo th t là hình chiếu vuông góc ca
,AB
trên đường thng
CD
. Tia
AD
ct tia
BC
ti
I
. Biết rng
3AE BF R
.
a) Tính s đo
AIB
.
b) Trên cung nh
CD
lấy điểm
K
. Gọi giao điểm ca
,KA KB
vi
DC
lần lượt
M
N
.
Tìm giá tr ln nht ca
MN
khi
K
di động trên cung nh
CD
.
Li gii:
a). K
OH CD H CD
,
ta thy
OH
là đường trung bình
ca hình thang
ABFE
,
suy ra
13
22
R
OH AE BF
.
T đó tam giác
OCD
đều,
suy ra
đđ
0
s s 60COD KCD
.Ta thy
AIB
đỉnh nằm ngoài đường tròn
O
n
đ đ đ
0 0 0
11
s s s 180 60 60
22
AIB AmB KCD
. b)
Ta thy
AEM NFB
suy ra
..EM NF AE BF
(không đổi) do đó
MN
ln nht khi ch
khi
EM NF
nh nht. Theo trên,
.EM NF
không đổi nên
EM NF
nh nht khi
.EM FN AE BF
. Vy
giá tr ln nht ca
MN
bng
2.EF AE BF
.
Bài 12. Trong tam giác
ABC
, đường phân giác ca
BAC
ct cnh
BC
ti
D
. Gi s
T
đường tròn tiếp xúc vi
BC
ti
D
đi qua điểm
A
. Gi
M
giao điểm th hai ca
T
AC
,
P
là giao điểm th hai ca
T
BM
,
E
là giao điểm ca
AP
BC
.
a) Chng minh rng
EAB MBC
.
b) Chng minh h thc
2
.BE EP EA
.
Li gii:
a). Gi
N
là giao điểm th hai
ca
AB
với đường tròn
T
.
H
N
M
K
O
F
E
I
D
C
B
A
T
P
D
E
N
M
C
B
A
CHUYÊN ĐỀ BỒI DƯỠNG HC SINH GII HÌNH HC 9
THCS.TOANMATH.com | 50
Do
AD
là phân giác ca
BAC
nên
đđssDM DN
. Ta có
đ đ đ đ
11
s s s s
22
MBC MBD DM DP DN DP
đ
1
s
2
NP NAP EAB
(đpcm).
b) T kết qu câu a, ta thy
EBP EAB
. T đó
EBP EAB
(g.g), suy ra
BE EA
EP BE
hay
2
.BE EP EA
(đpcm).
Bài 13. Trên đường tròn
O
ta lấy các điểm
1 1 1
, , , , ,AC B A C B
theo th t đó.
a) Chng minh rng nếu các đường thng
1 1 1
,,AA BB CC
là các đường phân giác trong ca tam giác
ABC
thì chúng là các đường cao ca
111
ABC
.
b) CHng minh rng nếu các đường thng
1 1 1
,,AA BB CC
các đường cao ca tam giác
ABC
thì
chúng là đường phân giác trong ca tam giác
111
ABC
.
c) Gi s
1
T
2
T
hai tam giác ni tiếp đường tròn
O
, đồng thời các đỉnh ca tam giác
2
T
là các điểm chính gia ca các cung đường tròn b chia bi các đỉnh ca tam giác
1
T
. Chng
minh rng trong hình lc giác giao ca các tam giác
1
T
2
T
các đường chéo nối các đỉnh
đối nhau song song vi các cnh ca tam giác
1
T
và đồng quy ti một điểm.
Li gii:
a) Ta chng minh
1 1 1
AA BC
. Tht vy, gi
M
giao điểm ca
1
AA
11
BC
, khi đó:
đ đ đ đ đ
1 1 1 1 1 1 1
11
s s s s s
22
AMB AB ABC AB AB BC
0
1 1 1
1
90
2
ABB AAB BCC ABC CAB BCA
(đpcm).
Chứng minh tương tự ta cũng có
1 1 1 1 1 1
;BB AC CC AB
.
I
K
M
C
1
B
1
A
1
C
B
A
CHUYÊN ĐỀ BỒI DƯỠNG HỌC SINH GIỎI HÌNH HỌC 9
51 | THCS.TOANMATH.com
b)
Gi
1
M
giao đim ca
1
BB
AC
. Ta
đđ
1 1 1 1 1
1
ss
2
BM A AC B AC BCA AC C
(1) Li
đ
2 1 1 1 1
1
s
2
BM A AC B BC BCA B C C
(2).
0
12
90BM A BM A
, nên t
(1) (2) suy ra
1 1 1 1
AC A B C C
. Tc
1
CC
chứa đường phân giác ca
111
AC B
. Chng minh
tương tự, ta cũng thu được
1
AA
chứa đường phân giác ca
1 1 1
B AC
,
1
BB
chứa đường phân giác
ca
111
A B C
.
c) hiệu các đỉnh ca tam giác
1
T
,AB
C
;
11
,AB
1
C
điểm chính gia các cung
,BC CA
AB
tương ứng. Khi đó
2
T
tam giác
111
AB C
. Các đường
1 1 1
,,AA BB CC
cha các
đường phân giác ca tam giác
1
T
nên chúng đng quy tại điểm
I
. Gi s
K
giao điểm ca
AB
11
BC
. Ta ch cn chng minh rng
//IK AC
.
Tht vy, ta thy tam giác
1
AB I
cân ti
1
B
n tam giác
AKI
cân ti
K
. T đó
KIA KAI IAC
, dẫn đến
//IK AC
(đpcm).
Bài 14. Cho tam giác cân
ABC
AB AC
D
một điểm trên cnh
BC
. K
//DM AB
(
M AC
),
//DN AC N AB
. Gi
'D
là điểm đối xng ca
D
qua
MN
. Tìm qu tích điểm
'D
khi điểm
D
di động trên cnh
BC
.
Li gii:
M
2
M
1
A
B
C
A
1
B
1
C
1
D
N
M
D'
C
B
A
CHUYÊN ĐỀ BỒI DƯỠNG HC SINH GII HÌNH HC 9
THCS.TOANMATH.com | 52
Phn thun: T gi thiết đề ra ta thy
'NB ND ND
,(1) do đó ba điểm
, , 'B D D
nm trên
đường tròn tâm
N
. T đó
1
'
2
BD D DMC
(2). Li có
BND DMC BAC
, nên t (1) và (2)
suy ra
'BD C BAC
(không đổi).
BC
c định,
'D
nhìn
BC
dưới mt góc
BAC
không đổi,
'D
khác phía vi
D
(tc cùng phía vi
A
so vi
MN
) nên
'D
nm trên cung cha góc
BAC
v trên đoạn
BC
(mt phn của đường tròn ngoi tiếp tam giác
ABC
).
Phần đo: Bạn đọc t gii.
Kết lun: Qu tích của đim
'D
cung cha góc
BAC
trên đoạn
BC
. Đó chính cung
BAC
của đường tròn ngoi tiếp tam giác
ABC
.
Lưu ý: Quy trình để gii mt bài toán qu tích như sau:
Để tìm qu tích các điểm
M
tha mãn mt tính cht
T
nào đó ta tiến hành các bước
*Phn thun: Ch ra mọi điểm có tính cht
T
đều thuc hình
H
.
*Phần đảo: Chng t rng mọi điểm thuc hình
H
đều có tính cht
T
.
*Kết lun: Qu tích các điểm
M
có tính cht
T
là hình
H
.
Chú ý rng trong mt s bài toán, sau phn thun, trước phần đảo ta th thêm phn gii hn qu
tích.
Bài 15. Cho đường tròn
O
dây cung
BC
c định. Gi
A
điểm di động trên cung ln
BC
của đường tròn
O
(
A
khác
B
,
A
khác
C
). Tia phân giác ca
ACB
cắt đường tròn
O
ti
điểm
D
khác điểm
C
. Ly điểm
I
thuộc đoạn
CD
sao cho
DI DB
. Đường thng
BI
ct
đường tròn
O
tại điểm
K
khác điểm
B
.
a) Chng minh rng tam giác
KAC
cân.
b) Chứng minh đường thng
AI
luôn đi qua một điểm
J
c định.
c) Trên tia đối ca tia
AB
lấy điểm
M
sao cho
AM AC
. Tìm qu tích các điểm
M
khi
A
di
động trên cung ln
BC
của đường tròn
O
.
CHUYÊN ĐỀ BỒI DƯỠNG HỌC SINH GIỎI HÌNH HỌC 9
53 | THCS.TOANMATH.com
Li gii:
O
x
J
K
M
D
C
B
A
a). Ta có
đđ
1
s s ;
2
DBK DA AK
đ đ đ
1
s s s
2
DIB BD KC
đđssBD DA
DBI
cân ti
D
nên
đđssKC AK
. Suy ra
AK CK
hay
KAC
cân ti
K
(đpcm).
b) T kết qu câu a, ta thy
I
là tâm đường tròn ni tiếp
ABC
nên đường thng
AI
luôn đi qua
điểm
J
(điểm chính gia ca cung
BC
không cha
A
). Rõ ràng
J
là điểm c định.
c) Phn thun: Do
AMC
cân ti
A
, nên
1
2
BMC BAC
. Gi s s đo
BAC
2
(không
đổi) thì khi
A
di động trên cung ln
BC
thì
M
thuc cung cha góc dựng trên đoạn
BC
v
phía điểm
O
.
Phần đảo: Tiếp tuyến
Bx
với đường tròn
O
ct cung cha góc v trên đon
BC
tại điểm
X
.
Lấy điểm
M
bt k trên
Cx
(mt phn ca cung cha góc v trên đoạn
;BC M X M C
. Nếu
MB
cắt đường tròn
O
ti
A
thì ràng
A
thuc cung ln
BC
ca
đường tròn
O
.
2;BAC AMC
suy ra
AMC
cân ti
A
hay
AC AM
.
Kết lun: Qu tích các đim
M
cung
Cx
, mt phn ca cung cha góc v trên đoạn
BC
v
phía
O
tr hai điểm
C
Bài 16. Gi s
AD
đường phân giác trong góc
A
ca tam giác
ABC
(
D BC
). Trên
AD
ly
hai điểm
M
N
sao cho
ABN CBM
.
BM
cắt đường tròn ngoi tiếp tam giác
ACM
ti
điểm th hai
E
CN
cắt đường tròn ngoi tiếp tam giác
ABM
tại điểm th hai
F
.
a) Chng minh rng bốn điểm
, , ,B C E F
cùng nm trên một đường tròn.
b) Chứng minh ba điểm
,,A E F
thng hàng.
c) Chng minh
BCF ACM
, t đó suy ra
ACN BCM
.
CHUYÊN ĐỀ BỒI DƯỠNG HC SINH GII HÌNH HC 9
THCS.TOANMATH.com | 54
Li gii:
a) Ta
BFC BAN
(cùng chn cung
BN
);
BEC CAN
(cùng chn
CM
),
BAN CAN
, suy ra
BFC BEC
.
T đó bốn điểm
, , ,B C E F
cùng nm trên một đường tròn (đpcm).
b) T kết qu trên, ta
CFE NFA
. Do đó hai tia
FA
FE
trùng nhau nghĩa ba đim
,,A E F
thẳng hàng (đpcm).
c)
BCF BEF
do
ACM BEF
nên
BEF ACM
. T đó suy ra
ACM BCF
, dn
đến
ACN BCM
(đpcm).
-----------Toán Học Sơ Đồ----------
F
E
N
M
D
C
B
A
HH9-CHUYÊN Đ 4. CHÙM BÀI TOÁN LIÊN QUAN ĐIỂM, ĐƯỜNG ĐC
BIT TRONG TAM GIÁC, TIP TUYN CÁT TUYN CỦA ĐƯỜNG TRÒN
I. CHÙM BÀI TOÁN LIÊN QUAN ĐẾN ĐƯỜNG CAO
VÀ ĐƯỜNG TRÒN NGOI TIP TAM GIÁC
Cho tam giác nhn
ABC
ni tiếp
( )
O
có các đường cao
,,AD BE CF
ct nhau ti
,H
các đường thng
,BH CH
kéo dài ct
( )
O
tại giao điểm th 2
,,P Q R
(
P
khác
,B
Q
khác
,C
R
khác
).A
Gi
,MI
lần lượt là trung điểm ca
đường thng
EF
ct
AH
ti
.K
1. Các t giác
,BFHD
,CEHD
BFEC
ni tiếp.
Chng minh: Do
,,AD BE CF
là các đường cao ca tam giác
ABC
nên
0
90HDB BFD==
0
180HDB BFD + =
suy ra t giác
BFHD
ni tiếp (tổng hai góc đối nhau bng
0
180 ).
Tương tự ta
cũng có tứ giác
CEHD
ni tiếp.
Ta có:
BFC BEC=
nên
BFEC
t giác ni tiếp (Hai đỉnh liên tiếp
,FE
cùng nhìn cnh
BC
mt
góc bng nhau).
2. Các đường thng
,,AD BE CF
chứa các đường phân giác ca góc
,,EDF DEF EFD
t đó suy ra trực
tâm
H
là tâm đường tròn ni tiếp tam giác
.DEF
Chng minh:
BFHD
ni tiếp nên:
FBH FDH=
(cùng chn
)FH
(1),
CEHD
ni tiếp nên
HDE HCE=
(cùng chn
)EH
(2), t giác
BFEC
ni tiếp nên
FBE FCE=
(cùng chn
)EF
(3). T
(1), (2), (3) ta suy ra
FDH EDH=
hay
AD
phân giác ca góc
.EDF
Chứng minh tương tự ta ng
,BE CF
cha các đường phân giác ca góc
,DEF EFD
t đó suy ra trực tâm
H
tâm đường tròn
ni tiếp tam giác
.DEF
3. Dựng đường kính ca
( )
.O
Khi đó tứ giác
BHCN
hình bình hành. Suy ra
,,H M N
thng hàng.
,,H G O
thng hàng và
3.HO GO=
Chng minh:
AN
đường kính ca
( )
O
nên
,NC AC
do
. // BH AC BH NC⊥
Chng minh
tương t ta cũng
// CH NB
nên t giác
BHCN
hình bình hành, suy ra 2 đường chéo
,NH BC
ct
nhau tại trung điểm ca mỗi đường, mà
M
là trung điểm ca
BC
nên
,,N H M
thng hàng.
Ta
MO
đường trung nh ca tam giác
AHN
nên
1
/ / .
2
MO AH=
Gi
G
giao điểm ca
AM
,HO
do
//MO AH
(cùng vuông góc vi
).BC
Theo định lý Thales ta có:
1
2
AG MO
G
GM AH
= =
trng tâm ca tam giác
ABC
,,H G O
thng hàng. Do
1
3.
2
GO OM
HO GO
GH AH
= = =
(Đường thng
qua
,,H G O
gọi là đường thẳng Ơle của tam giác
).ABC
4. Các đường thng
,,AH BH CH
kéo dài ct
( )
O
tại giao điểm th 2 là
,,P Q R
khi đó:
,,P Q R
là các điểm đối xng vi
H
qua
, , .BC CA AB
Chng minh:
AN
đường kính ca
( )
O
nên
0
90 / / .APN PN DM=
Li
M
trung đim
HN
(chng minh 3). Suy ra
DM
là đường trung bình ca tam giác
HPN
suy ra
D
là trung điểm ca
.HP
Nói cách khác
P
điểm đối xng vi
H
qua
.BC
Chứng minh tương tự ta cũng có:
,QR
các
điểm đối xng vi
H
lần lượt qua
,.CA AB
5.
,OA EF
tam giác
ARQ
cân.
Chng minh: Dng tiếp tuyến
Ax
của đường tròn ngoi tiếp tam giác
ABC
ta
Ax OA
(4). Ta
cũng
xAC ABC=
(tính cht góc to bi tiếp tuyến dây cung). Mt khác t giác
BFEC
ni tiếp
nên
AEF ABC=
t đó suy ra
xAC AEF=
hay
//EF Ax
(5). T (4) (5) suy ra
.EF OA
Chú ý
rng
EF
đường trung bình ca tam giác
HQR
nên
//QR EF QR OA⊥
suy ra
OA
vuông góc vi
QR
tại trung điểm ca
QR
nên tam giác
AQR
cân ti
.A
6. Đưng thng
EF
kéo dài cắt đường tròn
( )
O
lần lượt ti
11
,EF
(
E
nm gia
1
E
).F
Khi đó:
11
,,AE AF
lần lượt là tiếp tuyến của đường tròn ngoi tiếp các tam giác
11
,.CEE BFF
Chng minh:
Theo chng minh câu 5 ta
11
/ / / /EF QR E F QR
suy ra tam giác
11
AFE
cân ti
A
nên
1 1 1 1
.AFE AE F=
Mt khác
1 1 1 1 1 1 1
.AE F AB E AFE ABF= =
Suy ra
1
AF
tiếp tuyến của đường tròn
ngoi tiếp tam giác
1
.BFF
Chứng minh tương tự ta cũng có:
1
AE
là tiếp tuyến của đường tròn ngoi tiếp
tam giác
1
.CEE
Chú ý rng: T chứng minh trên ta cũng có các hệ thức đẹp:
2
1
. . .AF AF AB AH AD==
2
1
. . .AE AE AC AH AD==
7. Gi
, , ,X Y Z T
lần lượt trung đim ca
, , , .AB AC HC HB
Khi đó 9 đim
, , , , , , , ,D E F X Y M I Z T
cùng nm trên một đường tròn tâm trung điểm ca
OH
(gọi đường tròn Ơle của tam giác
).ABC
Chng minh: Ta
IZ
đường trung bình ca tam giác
1
/ / ,
2
ABH IZ AB=
YM
đường trung
bình tam giác
ABC
nên
1
/ / .
2
YM AB=
T đó suy ra
//IZ YM IZMY=
hình bình hành. Li
ZM
đường trung bình ca tam giác
BHC
nên
//ZM CH
AB CH
suy ra
ZM IZ
vy t
giác
IZMY
hình ch nhật nên hai đường chéo
,IM ZY
ct nhau tại trung đim
J
ca mỗi đường.
Tương tự ta cũng chứng minh được các t giác
,XITM XYTZ
là các hình ch nhật nên suy ra các đưng
chéo
,,IM ZY TX
đồng quy tại trung điểm
J
ca mỗi đường.
Chú ý rng:
,,IDM ZEY TFX
lần lượt vuông ti
,,D E F
nên tâm vòng tròn ngoi tiếp chính trung
điểm
J
ca các cnh huyền tương ứng. Suy ra 9 điểm
, , , , , , , ,D E F X Y M I Z T
cùng nằm trên đường
tròn tâm
.J
Ta cũng có:
1
/ / / /
2
OM AH IH==
nên t giác
IHMO
hình bình hành, suy ra hai đưng chéo
,IM HO
ct nhau tại trung điểm mỗi đường. Nói cách khác điểm
J
cũng chính là trung điểm ca
.HO
Chú ý rng: Nếu chng minh mi b 4 điểm trong s 9 điểm nêu trên nm trên một đường tròn thì ta có
th làm đơn giản hơn như sau:
Ví d: Chng minh t giác
DMEF
ni tiếp.
Ta có:
M
tâm đường tròn ngoi tiếp t giác
BFEC
nên
2EMF EBF=
(tính cht góc tâm góc
ni tiếp cùng chn mt cung). Mặt khác theo 2) ta đã chỉ ra
22EDF HDF EBF==
suy ra
EMF EDF=
hay t giác
DMEF
ni tiếp (các đỉnh liên tiếp
,DM
cùng nhìn cnh
EF
mt góc bng
nhau).
8.
K
là trc tâm ca tam giác
.IBC
Chng minh: Để chng minh
K
là trc tâm ca tam giác
IBC
ta chng minh:
.CK IB
Ta s chng minh:
0
90 .IBC KCB+=
Ta có:
0
90IBC IBE EBC IBE ACB= + = +
(5)
Để ý rằng: 4 điểm
, , ,A E F H
nằm trên đường tròn tâm
I
trung điểm ca
AH
nên
22EIH EAH PBC PBE= = =
(do
H
đối xng vi
P
qua
).D
Suy ra t giác
EIBP
ni tiếp nên
IBE IPE=
(6). Ta cũng có:
AEF ABC APC==
nên t giác
EKPC
ni tiếp suy ra
IPE KPE KCE ACB KCB= = =
(7). T (6) (7) suy ra
IBE ACB KCB=−
thay vào (5) ta có:
0
90 .IBC KCB=−
T đó suy ra
0
90IBC KCB+=
vy
KC IB
kết hp vi
IK BC
suy ra
K
là trc
tâm tam giác
.IBC
Ngoài ra ta cũng có thể làm theo cách sau:
Gi s đường tròn ngoi tiếp t giác
BFEC
ct
IC
ti
1
K
theo
tính cht
Quen thuc v tiếp tuyến cát tuyến ta có:
2
1
.IK IC IE=
(1’) mặt khác ta cũng 5 điểm
, , , ,I F D M E
nm
trên cùng một đường tròn nên
IEF IDF IDE==
suy ra
2
.IEK IDE IE IK ID =
(2’).
T (1’), (2’) suy ra
1
..IK IC IK ID=
suy ra t giác
1
DKK C
ni
tiếp. Nên
0
1
90 .KK C =
Li có
0
1
90BK C =
(Do
1
K
nằm trên đường tròn đường kính
),BC
t đó suy ra
1
,,B K K
thng hàng hay
KB IC
kết hp vi
IK BC⊥
đpcm.
9.
,ME MF
là các tiếp tuyến của đường tròn ngoi tiếp tam giác
.AEF
Chng minh: Đưng tròn ngoi tiếp tam giác
AEF
tâm là trung điểm
I
ca
.AH
Ta có
IAE IEA=
(8), tam giác
MEC
cân ti
M
nên
MEC MCE=
(9),
0
90IAE MCE+=
(10).
T (8), (9), (10) suy ra
0
90IEA MEC+=
hay
0
90IEM =
tc là
ME
tiếp tuyến của đường tròn ngoi
tiếp tam giác
.AEF
Chứng minh tương tự ta cũng
MF
tiếp tuyến của đường tròn ngoi tiếp tam
giác
.AEF
10. Đưng tròn ngoi tiếp tam giác
AEF
ct
( )
O
ti
T
(
T
khác
)A
thì
,,M H T
thng hàng
Chng minh: Ta thy đường tròn ngoi tiếp tam giác
AEF
cũng đường tròn ngoi tiếp t giác
AEHF
chính
( )
I
đường kính
.AH
đường tròn
( )
O
ct
( )
I
tại giao điểm th 2
T
nên
0
90HTA=
0
90 , ,NTA N H T=
thng hàng.
Mặt khác theo 3) ta đã chứng minh:
,,N M H
thng hàng.
T đó suy ra
,,M H T
thng hàng.
11. Đưng thng
RD
kéo dài cắt đường tròn
( )
O
ti
1
D
thì
1
AD
đi qua trung điểm ca
.DE
Chng minh: Xét tam giác
FHD
tam giác
AED
ta có:
,DFH DAE=
FDH ADE=
nên
FDH AED
suy ra
,
FH AE
HD ED
=
gi
1
A
trung đim ca
1
AD
thì
FH AE
HD ED
=
1
1
2
2
RH
AE
HD EA
=
1
1
RH AE
RHD AEA
HD EA
=
nên
1
,DRH A AE=
mt khác
1
DRH D AC=
suy ra
11
D AC A AE=
hay
11
AA AD
tc là
11
,,A A D
thng hàng.
12. Đưng thng
1
KE
ct
BC
ti
1
K
thì
11
.AK HE
Chng minh: Gi s đưng tròn ngoi tiếp tam giác
AEF
ct
1
AK
tại giao điểm th 2
1
H
suy ra
0
1
90AH H =
nên
11
HH K D
t giác ni tiếp. Theo 6) cũng chứng minh được:
2
1 1 1
. . . .AH AK AH AD AE AC AE= = =
Tam giác
11
AE K
vuông ti
1
E
2
1 1 1
.AE AH AK=
nên theo h
thức lượng trong tam giác vuông ta suy ra
1 1 1
E H AK
(đpcm).
13. Đưng thng
AT
ct
BC
ti
1
B
thì
1
,,E F B
thng hàng.
Chng minh:
Cách 1: Ta t giác
ATBC
ni tiếp nên
1 1 1 1
. . ,BT B A B B BC=
gi s
1
BF
ct
( )
I
ti
'E
thì t giác
'ATFA
ni tiếp nên
1 1 1 1
. . 'BT B A B F B E=
t đó suy ra
1 1 1 1
. . 'B B BC B F B E=
hay t giác
'BFE C
ni tiếp,
suy ra
'.EE
Suy ra
1
,,E F B
thng hàng.
Cách 2: Ta
0 0 0 0
1
360 180 180 180TFB TFE EFB TFE EFB TAC ACB AB B= = + = + =
nên
0
1
180TFB AB B+=
nên t giác
1
BFTB
ni tiếp.
Ta có:
0
11
180TFB TFE TBB TFE TAE TFE+ = + = + =
suy ra
1
,,E F B
thng hàng.
14. Đưng tròn ngoi tiếp tam giác
,HEF HBC
ct nhau ti một điểm nằm trên đường thng
.AM
Chng minh: Gi s đường tròn ngoi tiếp tam giác
HEF
ct đường thng
AM
ti
1
.M
D thy
đường tròn ngoi tiếp tam giác
HEF
cũng chính là đường tròn ngoi tiếp t giác
AEHF
nên ta có:
0
1 1 1
90HM A HM M HDMM= =
t giác ni tiếp. Ta biến đổi góc sau:
0 0 0 0
1 1 1 1 1
360 180 180 180EM M EM H HM M EM H HM M HAE HDC ECM= = + = + =
nên t
giác
1
EM MC
ni tiếp.
Ta có:
0 0 0 0
1 1 1 1
360 180 180 180HM C HM E EM C HM E EMC HAC EMC= = + = +
00
180 2 180HAC HBC HBC= + =
suy ra
1
HM CB
ni tiếp. Tức là: Đường tròn ngoi tiếp tam giác
,HEF HBC
ct nhau ti một điểm nằm trên đường thng
.AM
Cách 2: Gi s đường tròn ngoi tiếp tam giác
HBC
và
HEF
ct nhau ti
1
.M
Làm tương tự như trên
ta chứng minh được các t giác
11
,EM MC FM MB
ni tiếp. T đó ta có:
00
11
180 180AM E EM M AFE ACB+ = + =
suy ra
1
,,A M M
thng hàng.
15. Ba điểm
11
,,B H M
thng hàng.
Chng minh: Theo 10 ta :
1
,MH AB
ta cũng
1
AH B M
suy ra
H
trc tâm ca tam giác
1
ABM
suy ra
1
B H AM
theo 14 ta có
1
HM AM
t đây suy ra
11
,,B H M
thng hàng.
Chú ý: Các tính cht 13, 14, 15 có th d dàng suy ra nh định lý Brocard sau đây:
Định Brocad: Cho t giác
ABCD
ni tiếp đường
tròn
( )
.O
Gi
M
là giao điểm ca
AB
;CD
N
giao điểm ca
AD
;BC
I
giao điểm ca
AC
.BD
Chng minh rng
I
trc tâm ca tam giác
.OMN
Chng minh:
Gi
E
giao điểm th 2 của đường tròn ngoi tiếp
tam giác
ABI
.CDI
Trước tiên ta chng minh:
,,E I M
thng hàng:
Tht vy gi s
MI
cắt đường tròn ngoi tiếp tam
giác
AIB
ti
'E
thì
. ' . .MI ME MB MA MC MD==
suy ra 4 điểm
, ', ,D E I C
nm trên một đường tròn suy ra
'EE
suy ra
,,M I E
thng hàng.
Ta có:
0 0 0
360 180 180AED DEI AEI DEI AEI ABD ACD AOD= = + = + =
nên t giác
AEOD
ni tiếp.
Ta cũng có:
BEC BEI IEC BAC BDC BOC= + = + =
nên t giác
BEOC
ni tiếp. T đó ta cũng suy ra
,,N E O
thng hàng.
Ta có:
0
0
180
90
2
BOC
OEM OEC CEM OBC BDC BDC
= + = + = + =
suy ra
.ME ON
Chng minh
tương tự ta cũng có:
NI OM
suy ra
I
là trc tâm ca tam giác
.OMN
Tr li bài toán: Ta thy t giác
BFEC
ni tiếp đường tròn tâm
M
đường kính
,BC
giao điểm
hai đường chéo là
H
nên suy ra
H
là trc tâm tam giác
1
.AMB
Cũng thể chng minh:
1
B H AM
không dùng đến định lý Brocard như sau: Gọi
1
I
trung điểm
HM
thì
1
I
tâm đường tròn
ngoi tiếp tam giác
,HDM
các điểm
,EF
nm
trên đường tròn tâm
I
đường kính
.AH
Ta có:
T giác
DEFM
ni tiếp trong đường tròn Ơle
ca tam giác
ABC
nên
1 1 1 1
..BF B E B B BM=
t
đó suy ra
1
BH
trục đẳng phương của hai
đường tròn
1
; , ;
22
AH HM
II
suy ra
11
,B H II
1
II
đường trung bình ca tam giác
HAM
suy
ra
1
B H AM
16. Gi
1
T
một điểm nm trên
( )
,O
234
,,T T T
lần lượt hình chiếu vuông góc của đim
1
T
lên các
cnh
, , .AB BC CA
Khi đó các điểm
234
,,T T T
nm trên một đường thng gọi đường thng Simson ca
điểm
1
T
đối với đường tròn
( )
.O
T đó suy ra các điểm đi xng vi
1
T
qua
,,AB BC CA
nm trên mt
đường thng gọi đường thng Steiner của điểm
1
T
đối vi tam giác
.ABC
Hãy chứng minh: Đường
thng Steiner của điểm
1
T
thì đi qua trực tâm
H
ca tam giác
.ABC
Chng minh:
+ T giác
1 2 3
TT BT
0 0 0
2 1 3 1
90 90 180BT T BTT+ = + =
nên t giác ni tiếp
2 1 2 3 1
T BT T T T=
(cùng
chn cung
21
),TT
mà t giác
1
ABTC
ni tiếp nên
2 1 1
,T BT ACT=
do đó
2 3 1 1
.T T T ACT=
Mt khác t giác
1 3 4
TTT C
ni tiếp nên
00
1 3 4 4 1 2 3 1 1 3 4
180 180 .TT T T CT T TT TTT+ = + =
Vy
234
,,T T T
thng
hàng.
+ Ta
234
,,T T T
thng hàng. D thy
23
TT
đường trung bình ca tam giác
1 5 6
TT T
nên
2 3 5 6
/ / .T T TT
Tương tự
3 4 6 7
/ / .TT T T
Theo tiên đề Ơ-clit và do
234
,,T T T
thng hàng nên suy ra
5 6 7
,,T T T
thng hàng.
+ Đường thẳng Steiner đi qua trực tâm ca tam giác
.ABC
Theo 4) ta chứng minh được
R
đối xng vi
H
qua
,AB
Q
đối xng vi
H
qua
AC
nên
51
HRTT
là
hình thang cân các t giác
1 2 3 1 3 4
,TT BT TTT C
ni tiếp nên ta có:
5 1 1 3 1 3 2 1
HT T HRT T BT TT T= = =
do đó
5 2 3
/ / .HT T T
Tương tự ta cũng có:
7 3 4
//HT TT
234
,,T T T
thng hàng nên
57
,,T H T
thng hàng. Nói
cách khác: Đường thng Steiner của điểm
1
T
đi qua trực tâm ca tam giác
.ABC
17. Gi s
BC
c định, điểm
A
chuyển động trên cung ln
.BC
Chng minh: Trc tâm
H
ca tam
giác
ABC
thuc một đường tròn c định.
Chng minh: Do
,BC
c định nên độ dài
BC
không đổi. Gi
'M
điểm đối xng vi
O
qua
BC
theo 3) ta suy ra
'/ /OM AH=
nên t giác
'AHM O
hình bình hành
'M
điểm c định
' ' 'M H OA OB OC M B M C R= = = = = =
(Do
BC
trung trc ca
').MM
Như vậy
'M
chính
tâm đường tròn ngoi tiếp tam giác
.HBC
Hay điểm
H
thuộc đường tròn c định
( )
'; .MR
18. Gi s
BC
c định, điểm
A
chuyển động trên cung ln
.BC
Tìm v trí điểm
A
để
HA HB HC++
ln nht.
Chng minh:
Theo 17) ta
H
thuộc đường tròn c định
( )
';MR
BC
y cung c định ca
( )
';MR
nên
BHC
không đổi suy ra
0
180EHC BHC
= =
không đổi. Theo 3) ta cũng có:
2AH OM=
suy ra
HA
có độ dài không đổi.
Như vậy để tìm GTLN ca
HA HB HC++
ta quy v tìm GTLN ca
.HB HC+
Trên tia đi ca tia
HB
ta ly một điểm
'E
sao cho
''HE HC HE C=
cân ti
H
00
' 180 2 ' 180 2HE C CHE
= =
không đổi. Suy ra
'E
chuyển động trên cung cha góc
0
180 2
dựng trên đoạn
.BC
Ta có:
'HB HC BE+=
nên
HB HC+
ln nht khi ch khi
'BE
đường kính của đường tròn cha
cung cha góc
0
180 2
dựng trên đoạn
BC
tc
'.BC E C
T đó suy ra cách dựng điểm
A
như
sau:
Dng cung cha góc
0
180 2
trên đoạn
,BC
dng tia
Cx BC
ct cung cha góc ti
'E
. Dng
đường thng qua
B
vuông góc vi
'BE
ct
( )
O
ti
A
là v trí cn tìm.
19. Gi s
BC
c định, điểm
A
chuyển động trên cung ln
.BC
Tìm v trí điểm
A
để
EF FD DE++
ln nht.
Chng minh: Chng minh:
OA EF
suy ra
1
.,
2
AEOF
S OA EF=
tương tự ta có:
1
.,
2
BDOF
S OB DF=
1
.
2
CDOE
S OC DE=
t đó suy ra
( )
2.
ABC
S R EF FD DE= + +
Do
R
không đổi nên
EF FD DE++
ln
nht khi ch khi
ABC
S
ln nht
khong cách t điểm
A
đến
BC
ln nht. Ta
AH AM AO OM R OM + = +
nên
AH
ln nht khi và ch khi
,,A O M
thng hàng. Hay
A
điểm
chính gia cung ln
.BC
Ngoài ra ta cũng có thể gii theo cách khác:
Ta thấy các điểm
, , ,B F E C
nằm trên đường tròn tâm
M
đường kính
.BC
Do
BC
không đổi nên
BAC
không đổi.
Ta có
,ME MF
là các tiếp tuyến của đường tròn ngoi tiếp t
giác
AEHF
nên
MFE BAC=
không đổi. Ta
cos
EF AE
BAC
BC AB
==
không đổi suy ra độ dài
EF
không
đổi.
Gi
'K
là điểm đối xng vi
E
qua
BC
thì
'.EDC K DC=
Ta li
.EDC EHC FHB FDB= = =
Suy ra
EDB KDM=
nên
, , 'F D K
thng hàng.
Ta chu vi
DEF
ln nht khi ch khi
DE DF+
ln
nht. T các chng minh trên ta có:
' 2 ' .DE DF FK R BC+ = =
Suy ra
DE DF+
ln nht khi ch
khi
DM
hay
A
là điểm chính gia cung ln
.BC
Cũng th tiếp cận bài toán theo hướng khác: Gi
', 'PQ
lần lượt hình chiếu vuông góc ca
,BC
trên
.EF
Ta có các tính cht quen thuc
,FH DH
lần lượt là phân giác trong các góc
,EFD FDE
suy ra
,FB FD
lần lượt các đường phân giác ngoài ca
,EFD FDE
. H
', 'BX BY
lần lượt vuông góc vi
,DE DF
thì suy ra
' ', ' ', 'FP FY DX DY EP EX= = =
(tính cht một điểm nằm trên phân giác cách đu
2 cnh). Ta có: Chu vi tam giác
DEF
2 ' ' 'p DF EF DE Y F DX EF DE FP EF= + + = + + + = + +
' ' ' 2 '.EX EP EX EP= + =
Hoàn toàn tương tự ta cũng có:
2 2 '.p FQ=
Suy ra
( ) ( )
4 2 ' ' 2 ' ' 2 2 ' ' 2p EP FQ P Q EF p P Q EF= + = + = +
suy ra
''DE DF P Q+=
. Mt khác
''P Q BC
nên chu vi tam giác
DEF
ln nht bng
EF BC+
khi ch khi
' '/ /P Q BC
hay
A
điểm chính gia cung ln
.BC
20. Gi s
BC
c định, điểm
A
chuyển động trên cung ln
.BC
Tìm v trí điểm
A
để
.DH DA
ln
nht.
Chng minh: Ta d chứng minh được:
. . .
BD AD
BHD ACD DH DA BDCD
HD CD
= =
Theo bất đẳng thức Côsi ta cũng
2 . 2 .BD CD BDCD BC BDCD+
suy ra
2
..
4
BC
BDCD
Vy
2
.,
4
BC
DH DA
du bng xy ra khi ch khi
BD CD D=
trung điểm ca
,BC
suy ra
A
là điểm chính gia cung ln
.BC
21. Chng minh:
. . 8 . .HAHB HC HD HE HF
3.R HA HB HC++
Để gii quyết câu hi này ta cn kiến thc b tr là bài toán ni tiếng sau:
Bất đẳng thc Erdoss-Mordell.
Cho tam giác
ABC
M
một điểm bt k nằm trong tam giác đó. Gi
,,
a b c
R R R
theo th t
khong cách t
M
đến các đnh
,,.A B C
Còn
,,
a b c
d d d
lần lượt khong cách t
M
đến các cnh
, , .BC CA AB
Khi đó ta có bất đẳng thc:
a,
( )
2.
a b c a b c
R R R d d d+ + + +
b,
. . 8 .
a b c a b c
R R R d d d
Đẳng thc xy ra khi và ch khi tam giác
ABC
đều và
M
là tâm ca tam giác.
ng dn gii:
Đặt
, , .BC a CA b AB c= = =
a, Ly điểm
1
M
đối xng với điểm
M
qua
đường phân giác trong ca
.BAC
Dng
1
BH AM
1
.CK AM
Gi s
1
AM
ct
BC
ti
.D
Khi đó
,.BD BH DC CK
Đẳng thc xy ra khi và ch
khi
AD BC
hay
1
.AM BC
T đó ta có:
11
22
a ABM ACM
a BH CK aR S S + +
(chú ý rng
1
)
a
AM AM R==
hay
.
a b c
aR cd bd+
T đó
a b c
cb
R d d
aa
+
(1). Tương t ta
b c a
ac
R d d
bb
+
(2);
c b a
ab
R d d
cc
+
(3). Cng theo vế các bt
đẳng thức (1), (2), (3) ta thu được:
( )
2.
a b c a b c a b c
b c a c a b
R R R d d d d d d
c b c a b a
+ + + + + + + + +
(S dng bất đẳng thc
2x y xy+
cho các biu thc trong ngoặc). Đẳng thc xy ra khi ch khi
abc==
đồng thi
1
M
là trc tâm ca tam giác
.ABC
Nói cách khác,
1
M
(và do đó cả
)M
là tâm ca
tam giác đều
.ABC
b, T cách chng minh bất đẳng thc Erdos Mordell câu a) ta có:
a b c
cb
R d d
aa
+
(1);
b c a
ac
R d d
bb
+
(2);
c b a
ab
R d d
cc
+
(3). Nhân theo vế ba bất đẳng thc
trên ta được:
. . .
a b c b c c a b a
c b a c a b
R R R d d d d d d
a a b b c c
+ + +
Áp dng bất đẳng thc:
2x y xy+
ta có:
2 . .2 . .2 . 8
b c c a b a a b c
c b a c a b
d d d d d d d d d
a a b b c c
=
(đpcm).
Tr li bài toán:
+ Áp dng bất đẳng thc Erdor- Mordell dng tích cho trc
tâm
H
ca tam giác
ABC
ta có ngay:
8 . . .HA HB HC HD HE HF+ +
+ Áp dng bất đẳng thc Erdor- Mordell dng tng cho tâm
O
của đường tròn ngoi tiếp tam giác
ABC
ta có:
( )
2.OA OB OC OM OX OY+ + + +
Theo 3) ta
2,HA OM=
tương tự
2 , 2 .HB OY HC OX==
Suy ra
3.R HA HB HC++
II. CHÙM BÀI TOÁN LIÊN QUAN ĐẾN TÂM
ĐƯNG TRÒN NI TIP
1. Cho tam giác nhn
ABC
ngoi tiếp đường tròn
( )
I
ni tiếp
( )
,O
gi
,,D E F
lần lượt tiếp
điểm ca
( )
I
vi
, , .BC CA AB
Đưng thng
AI
kéo dài cắt đường tròn
( )
O
tại giao điểm th 2
K
(
K
khác
).A
Ta có các tính cht, bài toán sau:
a,
KB KC KI==
(
K
là tâm đường tròn ngoi tiếp tam giác
).IBC
Chng minh:
AI
đường phân giác trong ca góc
A
nên
K
điểm chính gia ca cung
BC
suy ra
.KB KC=
Bây gi ta chng minh: Tam giác
KBI
cân ti
:I
Ta
BIK IBA ABI IBC IAC IBC CBK IBK= + = + = + =
nên tam giác
KBI
cân ti
.K
T đó suy ra
.KB KI KC==
b, Đưng thng
,BI CI
ct
EF
lần lượt ti
,.MN
Khi đó 4 điểm
, , ,I E M C
cùng nm trên một đường
tròn, 4 điểm
, , ,I N F B
cùng nm trên một đường tròn.
Chng minh:
Xét t giác
IEMC
ta có:
0
90
2 2 2
B C A
MIC IBC ICB= + = + =
(Tính cht góc ngoài).
Ta cũng có:
0
0
180
90 .
22
AA
MEC AEF
= = =
T đó suy ra
IEMC
t giác ni tiếp, do đó:
0
90 .IMC IEC==
Tương tự ta cũng chứng minh được: 4 điểm
, , ,I N B F
cùng nm trên một đường
tròn suy ra
0
90 .INB IFB==
T đó suy ra
0
90BMC BNC==
hay 4 điểm
, , ,B C M N
nằm trên đường tròn tâm
Q
đường kính
.BC
Chú ý rng: Tam giác
QBM
cân ti
Q
nên
,QMB QBM=
li
QBM IBA=
(Tính cht phân giác)
nên
QMB IBA=
suy ra
//QM AB
hay đường thng
QM
chứa đường trung bình song song vi cnh
AB
ca tam giác
.ABC
c, hiu
,Rr
lần lượt bán kính đường tròn ngoi tiếp, ni tiếp tam giác
.ABC
Khi đó ta có:
22
2.R R r OI−=
(công thức Ơle).
Chng minh:
Dựng đường kính
PK
ca
( )
O
đường kính
''IO
qua
,OI
ca
( )
O
(
'I
thuc cung nh
).AB
Ta
có:
( )( )
22
'. ' . . .II IO IAIK R OI R OI IAIK R OI IAIK= + = =
(*). Để ý rng:
IFA KCP
nên
. .2IA KP IF KP r R
IA
IF KC KC KC
= = =
thay vào (*) ta có:
22
.2
.
rR
R OI IK
KC
−=
theo 1) ta đã chứng minh:
KI KC KB==
nên suy ra
22
2.R OI R r−=
(đpcm).
Chú ý: Do
22
0 2 0 2 .OI R Rr R r
Du bng xy ra khi và ch khi
OI
hay tam giác
ABC
đều.
d, Xét đường tròn tâm
'K
tâm đường tròn bàng tiếp ng vi góc
A
(Đường tròn tiếp xúc vi
BC
tiếp xúc vi phn kéo dài các cnh
, ).AB AC
Khi đó ta có
'.KK KI KB KC= = =
Chng minh:
'K
là tâm đường tròn bàng tiếp góc
A
nên
'K
nm trên phân giác tròn góc
,A
mt khác
( )
'K
tiếp xúc
vi
BC
phn kéo dài ca cnh
AB
nên
'K
nm trên phân giác ngoài góc
,B
suy ra
'K B IB
hay
tam giác
'IBK
vuông ti
.B
Mặt khác theo 1) ta đã chứng minh:
KI KB KC==
nên suy ra
K
trung
điểm ca
'IK
hay
'.KK KI KB KC= = =
2. Các đường thng
,BI CI
kéo dài ct
( )
O
tại giao đim
th 2
,XY
(
X
khác
,A
Y
khác
).B
Đưng thng
KY
ct
,,AB BC BI
lần lượt ti
1 2 3
,,B B B
.
Đưng thng
KX
ct
,,AC BC CI
lần lượt ti
1 2 3
, , .C C C
Khi đó ta có:
+
1 2 1 2
,BB B CCC
là các tam giác cân
+
11
,,B I C
thng hàng.
+
.
KI BC
KA AB AC
=
+
+
33
/ / .B C BC
+
I
là trc tâm tam giác
.KXY
+ Gi
J
trung điểm
IK
thì 4 điểm
33
, , ,A B C J
cùng
nm trên một đường tròn.
Chng minh:
+ Ta có:
( ) ( )
12
11
,
22
KC C AX KC CC X CX BK= + = +
,AX XC KC BK==
12
KC C CC X=
hay tam giác
12
CCC
cân ti
.C
Chứng minh tương tự cho trường hp tam giác
12
.BB B
+ Ta có:
11
1
2
IAC IXC BC==
suy ra t giác
1
IAXC
ni tiếp. Suy ra
1
XIC XAC XBC==
1
/ / ,IC BC
chng minh tương tự ta cũng có:
1 1 1
/ / , ,IB BC B I C
thng hàng
11
/ / .BC BC
+ Theo 1) ta có:
.KI KB KC==
Áp dụng định lý Ptolemy cho t giác ni tiếp:
ABKC
ta có:
. . .AB KC AC BK AK BC+=
suy ra
( )
..
KI BC
KI AB AC KA BC
KA AB AC
+ = =
+
+ Theo tính chất đường tròn ni tiếp tam giác ta có:
0 0 0 0 0
3 3 3 3
11
180 90 90 90 180 .
2 2 2 2
BC
BIC B KC BIC FDE B KC BAC BKC+ = + = + = + + = + =
Suy ra t giác
33
KB IC
ni tiếp. Suy ra
3 3 3 3 3
/ / .IB C IKC ABX XBC B C BC= = =
+ Ta có:
( ) ( ) ( )
0
3
1 1 1 1
90
2 4 2 2
KC C KC XY KC AX AY AB BC CA= + = + + = + + =
nên
,KX CY
chứng minh tương tự ta cũng có:
KY BX
suy ra
I
là trc tâm tam giác
.KXY
+ Theo trên ta đã chứng minh:
33
,IC K IB K=
nên 4 điểm
3 .3
, , ,I B K C
nm trên đường tròn tâm
J
đường kính
,IK
chứng minh tương t phn
33
//B C BC
ta suy ra
3 3 3 3
/ / , / / .A B AB AC AC
T đó ta có:
0
3 3 3 3 3 3 3
2 180B A C B JC BAC B KC BAC BKC+ = + = + =
suy ra 4 đim
3 3 3
, , ,B A C J
cùng nm trên mt
đường tròn.
3. Đưng thng qua
E
song song vi
BC
ct
,AD DF
lần lượt ti
12
,EE
thì
1
E
trung đim ca
2
.EE
Chng minh:
Dựng đường thng qua
A
song song vi
BC
ct
DF
ti
'.A
Ta d chứng minh được các tam giác
,'AEF AFA
cân
ti
A
nên
'.AE AF AA==
Áp dụng định lý Thales ta có:
1 2 1 1 1
.
''
E E DE EE EE
BE BD
AA DA BA BA EA AA
= = = = =
Suy ra
1 2 1
E E EE=
hay
1
E
là trung điểm ca
2
.EE
4.
DI
là phân giác ca góc
.MDN
Chng minh:
Theo 1) ta đã chng minh:
,,BENI CMFI BMNC
các t giác ni tiếp nên
0
90BNI CMI==
suy ra
các t giác
,CMID BNID
ni tiếp nên
IDN IBN ICM IDM= = =
hay
ID
là phân giác ca góc
MDN
.
5. Đưng tròn tâm
1
K
tiếp xúc trong vi
( )
O
ti
2
K
tiếp xúc vi các cnh
,AB AC
lần lượt ti
34
,.KK
Khi đó tâm đường tròn ni tiếp tam giác
ABC
là trung điểm ca
34
,KK
(Đường tròn
( )
1
K
gọi là đường tròn Mixtilinear ng vi góc
A
ca tam giác
).ABC
Chng minh:
Ta
12
,,O K K
thng hàng, dng tiếp tuyến chung
2
Kx
ca
( )
O
( )
1
.K
Gi s
24
KK
ct
( )
O
ti
5
.K
Các tam
giác
1 2 4 2 5
,K K K OK K
cân ti
1
,KO
nên
1 4 5
/ / ,K K OK
14
K K AC
5
OK AC⊥
suy ra
5
K
điểm chính gia
ca cung
AC
suy ra
5
,,B I K
thng hàng.
Gi s
5
BK
ct
34
KK
ti
.I
Ta chng minh:
CI
phân
giác ca góc
.ACB
Ta có:
5 2 4 3 2 5 2
K BK K K K K K x==
(Tính cht góc to bi tiếp tuyến dây cung). Suy ra t giác
32
IK BK
ni tiếp. Ta cũng có:
( )
0 0 0
2 2 2 3 4 4 3
180 180 180 2IK C BK C BK I BAC AK K AK K= = =
4 3 4 3
AK K AK K−=
nên t giác
42
IK CK
ni tiếp. T đó ta tính được:
0
2 4 2 2 5 2 3 4 5
180ACI IK K BK C BK I K K C BAC AK K ABK= = =
0
00
180
180 90 .
2 2 2 2 2
BAC ABC ABC BAC ACB
BAC
= = =
Điều đó chứng t
CI
phân giác
ca góc
.ACB
Hay tâm đường tròn ni tiếp tam giác
ABC
là trung điểm ca
34
.KK
6. Tam giác
ABC
ni tiếp
( )
O
điểm
M
trên cnh
.BC
Đưng tròn
( )
J
tiếp xúc vi
,MA MC
lần lượt
ti
,EF
đồng thi tiếp xúc vi
( )
O
ti
.P
Khi đó tâm
I
đường tròn ni tiếp tam giác
ABC
nm trên
đường thng
.EF
(B đề Sawayama- Thebault)
Chng minh:
Đưng thng
PF
ct
( )
O
ti
.D
Đưng tròn
( )
J
tiếp xúc vi
( )
O
ti
P
suy ra
,,P J O
thng hàng.
Tam giác
,PJF POD
tam giác cân nên
ODP JFP=
suy ra
/ / ,OD JF
JF BC OD BC DC DB =
hay
AD
phân
giác ca góc
.BAC
Gi
I
giao dim ca
AD
vi
EF
thì
,IAP FPx=
FEP FPx=
IEP IAP=
nên t giác
IEAP
ni
tiếp. Suy ra
,AEP AIP EFP AEP AIP EFP= = =
suy ra
DIP DFI DIF DIP=
suy ra
2
..DI DP DF=
2
..DC DB CDF PDC DC DF DP DI DC= = =
Theo chng minh: 1) ta suy ra
I
tâm đường tròn ni tiếp tam
giác
.ABC
Áp dng b đề ta có bài toán sau:
Đưng tròn tâm
'K
tiếp xúc trong vi
( )
O
ti
2
.K
Xét hai điểm
', 'PQ
nằm trên đường tròn
( )
',K
qua
'P
k tiếp tuyến vi
( )
'K
ct
( )
O
ti
', ',BD
qua
'Q
k tiếp tuyến vi
( )
'K
ct
( )
O
ti
', '.AC
Khi đó tâm đường tròn ni tiếp các tam giác
''AC D
' ' 'B C D
nm trên
' '.PQ
7. Cho tam giác
ABC
ni tiếp
( )
,O
ngoi tiếp đường tròn
( )
,I
D
đim bt k trên cnh
,BC
gi
( )
1
O
đường tròn tiếp xúc vi
,AB BC
( )
,O
( )
2
O
đường
tròn tiếp xúc vi
,AD BC
( )
.O
Khi đó
12
,,I O O
thng hàng.
Chng minh:
Gi
,EF
lần lượt các tiếp điểm ca
( )
1
O
vi
,,BC AD
,PQ
tiếp điểm ca
( )
2
O
vi
,.AD BC
Theo 6) ta
EF PQ I=
tâm vòng tròn ni tiếp tam giác
.ABC
Gi
H
giao đim
ca
1
DO
vi
,EF
K
giao đim ca
2
DO
vi
PQ
thì
1 2 1 2
, / / ,DO EF DO DO DO EF
tương tự ta có
1
//DO PQ
suy ra
IHDK
là hình ch nht.
Theo định lý Thales
1
12
2 2 1
,,
OH
IH KD
O I O
O D O D O D
= =
thng hàng.
Chú ý: Khi
D
chân đường phân giác trong góc
A
thì
( ) ( )
12
,OO
tiếp xúc nhau ti
.I
8. Tam giác
ABC
ni tiếp đường tròn
( )
O
ngoi tiếp đường tròn
( )
.I
Đưng tròn
( )
K
tiếp xúc vi
( )
O
ti
P
và tiếp xúc vi
,AB AC
ti
,.DE
Khi đó
PI
đi qua điểm chính gia cung
.BAC
Chng minh: Gi
F
giao điểm ca
CI
vi
( )
,O
Q
giao
điểm
PI
vi
( )
O
theo câu 5) ta
I
trung điểm ca
.DE
Ta
cũng có:
0
1
; 90
2
FPB DPB ACB DIB BIA= = =
( )
00
11
180 90
22
ABC BAC ACB= + =
nên
.DPB DIB=
Suy ra t
giác
DBPI
ni tiếp suy ra
.BPQ ADE=
Tương tự t giác
IPCE
ni tiếp nên
IPC AED BPQ CPQ= =
suy ra
Q
đim chính
gia cung
.BAC
9. Tam giác
ABC
ngoi tiếp đường tròn
( )
.I
AI
ct
BC
ti
.D
Gi
,EF
là điểm đối xng ca
D
qua
,.CI BI
Gi
,MN
lần lượt trung đim ca
,.DE DF
Đưng tròn ngoi tiếp tam giác
,AEM AFN
ct
nhau ti
P
khác
.A
Khi đó
AP
chia đôi
.BC
Chng minh:
Theo tính chất đối xng ca phân giác
Ta có
,EF
lần lượt thuc
,.AC AB
Ta có:
/ / .
BF BD CD CE
EF BC
BA BA CA CA
= = =
Gi
J
trung điểm ca
EF
ta có:
MPN MPA NPA MEC NFP= + = +
00
180 180 .MDC NDB MDN MJN= + = =
Suy ra
MJPN
ni tiếp. T đó ta có:
MPJ MNJ MEJ EDC DEC MPA= = = = =
suy ra
,,A J P
thng hàng, hay
AP
chia đôi
,EF
suy ra
AP
chia đôi
.BC
10. Cho đường tròn tâm
I
ni tiếp tam giác
ABC
tiếp xúc vi
,,BC CA AB
lần lượt ti
, , .D E F
Đưng
thng qua
A
song song vi
BC
ct
EF
ti
,K
gi
M
là trung điểm
.BC
Chng minh:
.MI DK
Gi
P
là giao điểm ca
ID
EF
,AE AF
các tiếp tuyến ca
( )
I
ti
,EF
nên
.IA EF
IP BC
suy ra
.IP AK
T
đó suy ra
P
trc tâm ca tam giác
AIK AP IK⊥
ti
H
suy ra t giác
ANHK
ni tiếp.
K đường thng qua
P
song song vi
BC
ct
,AB AC
ti
,XY
t các t giác
,IPXF IPEY
ni tiếp ta
suy ra
PXI PFI PEI PYI===
dn ti tam giác
IXY
cân ti
I
nên
P
trung đim
.XY
Theo định
thales ta d suy ra
,,A P M
thng hàng. Li có:
22
..
IH ID
IH IK IN IA IF ID
ID IK
= = = =
suy ra
IHD IDK
(c.g.c). T giác
IDMH
ni tiếp suy ra
,IDH IMH=
IHD IDK IDH IKD =
IMH IKD=
IMH
ph vi
MIH
nên
IKD
ph vi
.MIH IM DK⊥
11. Dựng đường kính
DK
của đường tròn
( )
,I
đường thng
AK
ct
BC
ti
.H
Chng minh:
.BD HC=
Chng minh
Dựng đường thng qua
K
song song vi
BC
ct
,AB AC
ln
t ti
,ST
suy ra
ST
tiếp tuyến ca
( )
,I
theo tính cht 2
tiếp tuyến ct nhau ta có:
,SK SE=
,BD BE=
li có:
( )
0
1
90
2
SIE BIE KIE DIE+ = + =
nên tam giác
SIB
vuông ti
,I
suy ra
22
. . .SK BD SE EB IE r= = =
Tương tự ta cũng có:
22
.KT CD IF r==
suy ra
..KT CD SK BD=
SK TK
CD DB
=
áp dng tính cht y t s bng nhau ta suy ra:
,
SK TK SK TK ST
DC DB CD BD BC
+
= = =
+
mt khác
theo định lý Thales ta cũng có:
SK SA ST
BH AB BC
==
t đó suy ra
SK SK
DC BH
DC BH
= =
đpcm.
III. CHÙM BÀI TOÁN CÁT TUYN, TIP TUYN
VỚI ĐƯỜNG TRÒN
Cho đường tròn
( )
;OR
và một điểm
M
nm ngoài đường tròn
( )
,O
qua
M
k các tiếp tuyến
,MA MB
đến đường tròn
( )
O
(
,AB
các tiếp đim) dng cát tuyến
MCD
sao cho
.MC MD
Gi
E
trung điểm ca
,CD
đoạn thng
MO
ct
( )
O
AB
lần lượt ti
,.IH
Khi đó các tính cht hình hc
sau có liên quan đến nhau:
1. 5 điểm
, , , ,M A O E B
nm trên một đường tròn.
2.
ME
là tia phân giác ca góc
.AEB
3.
2
..MA MC MD=
4.
.
AC BC
AD BD
=
5.
I
là tâm đường tròn ni tiếp tam giác
.MAB
6. T giác
CHOD
ni tiếp.
7.
AB
chứa đường phân giác ca góc
.CHD
8.
.CAD BHD=
9.
OE
kéo dài ct
AB
ti
K
thì
,KC KD
là tiếp tuyến ca
( )
.O
10.
AE
ct
( )
O
tại giao điểm th 2 là
.F
(
F
khác vi
).A
Khi đó
/ / .BF CD
11. Tia
CH
cắt đường tròn
( )
O
ti giao điểm th 2 là
P
(khác
)C
thì
/ / .DP AB
12. Đưng thng qua
E
song song vi
BD
ct
AB
ti
.N
Khi đó
.CN OB
13. V đường kính
,AQ
các đường thng
,QC QD
cắt đường thng
MO
lần lượt ti
,XY
thì
O
trung điểm ca
.XY
14. Đưng thẳng qua trung đim
E
ca
CD
song song vi
AD
ct
AB
ti
,F
DF
ct
AM
ti
1
N
thì
1
N
là trung điểm
.AM
15. Qua
M
dng cát tuyến th 2 ca
( )
O
11
.MC D
Chng minh:
11
,CD C D
ct nhau tại 1 điểm nm
trên
.AB
16. Gi s
MC
ct
AB
ti
.K
Gi
L
là trung điểm
MK
ta có các h thức sau tương đương nhau:
+
KC MC
KD MD
=
(*);
22
.LM LK LC LD==
(H thức Niu tơn) (**);
..ME MK MC MD=
(H thc
Maclaurin) (***)
17. K đường kính
'CC
ca
( )
O
đường thẳng đi qua trung điểm ca
BD
song song vi
'BC
ct
'CD
ti
'J
thì
'J
nm trên một đường tròn có bán kính không đổi.
18. Gi s
M
c định, chng minh: Khi cát tuyến
MCD
thay đổi, trng tâm
G
ca tam giác
BCD
thuc một đường tròn c định.
19. Gi s
M
c định, chng minh: Tia
BO
ct
( )
O
tại giao điểm th 2 là
L
(
L
khác
).B
Đưng thng
ML
ct
( )
O
tại giao điểm th 2 là
T
(khác
).L
Chng minh rằng: đường tròn ngoi tiếp
tam giác
ATM
luôn tiếp xúc với đường thng c định.
20. Gi s
M
thuộc đường thng c định
( )
.d
MO
ct
AB
ti
.H
Chng minh:
H
thuộc đường tròn
c định.
21. Gi s
M
c định nm ngoài
( )
O
cát tuyến
MCD
thay đổi quanh điểm
.M
Đưng thng
BE
ct
đường tròn
( )
O
tại giao điểm th 2
1
B
(khác
).B
Tìm v trí cát tuyến
MCD
để din tích tam giác
1
MBD
ln nht.
22. Gi s
M
thay đổi ngoài
( )
.O
Đưng thng qua
O
vuông góc vi
MO
ct các tia
,MA MB
ti
,.SW
Khi nào thì din tích tam giác
MSW
nh nht.
23. Gi s
2,MO R=
cát tuyến
MCD
thay đổi quanh
.M
Tìm v trí ca cát tuyến để
EA EB EM++
ln nht.
24. Gi s
2,MO R=
cát tuyến
MCD
thay đổi quanh
.M
Tìm v trí ca cát tuyến để
11
EA EB
+
nh
nht.
CHNG MINH CÁC BÀI TOÁN
1.
,MA MB
các tiếp tuyến ca
( )
O
nên
0
90 ,MAO MBO==
E
trung đim ca
CD
nên
0
90EO CD MEO =
(Tính chất đường kính đi qua trung đim mt dây cung). T đó suy ra 5
điểm
, , , ,M A O E B
nằm trên đường tròn đường kính
.MO
2. t giác
MAEB
ni tiếp và
MA MB MA MB AEM BEM= = =
(Góc ni tiếp chn 2 cung
bng nhau thì có s bng nhau). T đó suy ra
ME
là phân giác ca góc
.AEB
3.
MA
tiếp tuyến ca
( )
O
nên
MAC ADC=
(Tính cht góc to bi tiếp tuyến dây cung). Xét
hai tam giác
MAC
MDA
ta có:
,MAC ADC=
AMD
chung nên
MAC MDA
(g.g) suy ra
2
..
MA MD
MA MC MD
MC MA
= =
4. T chng minh
MAC MDA
(g.g) suy ra
CA MC
AD MA
=
(1), chứng minh tương t ta cũng có:
CB MC
MCB MBD
BD MB
=
(2), t (1) và (2) chú ý rng
.
MC MC CA CB
MA MB
MA MB AD BD
= = =
5. Do
I
nm trên
MO
nên
MI
phân giác trong ca góc
AMB
(3). Li có
MO
là trung trc ca
AB
I
nm trên
MO
nên
IA IB=
suy ra
.IAB IBA=
Mặt khác ta cũng
MAI ABI=
(tính cht góc
to bi tiếp tuyến y cung) t đó suy ra
MAI IAB=
hay chính phân giác ca góc
MAB
(4).
T (3) và (4) suy ra
I
là tâm đường tròn ni tiếp tam giác
.MAB
6. Theo chng minh câu 3) ta
2
.MA MC MD=
(5), áp dng h thức lượng trong tam giác vuông
,MAO AH MO
ta
2
.MA MH MO=
(6). T (5) (6) suy ra
..MH MO MC MD=
hay
.
MH MD
MC MO
=
Xét tam giác
,MCH MOD
ta có:
MH MD
MC MO
=
OMD
chung nên
MCH MOD
(g.g) suy ra
MHC MDO=
t giác
CHOD
ni tiếp (góc ngoài đỉnh
H
bng góc trong đối din
đỉnh
).H
7. T 6) ta
MHC MDO=
(7)
MDO OCD=
(8) (do tam giác
COD
cân ti
).O
Mt khác ta
OCD OHD=
(9) (góc ni tiếp cùng chn
).OD
T (7), (8), (9) suy ra
.MHC OHD=
Các
góc
,CHA DHA
ph vi các góc
,MHC OHD
tương ng nên suy ra
CHA DHA=
hay
AH
phân
giác ca góc
.CHD
Chú ý: T vic chng minh:
AH
là phân giác ca
CHD
nếu gi ta
Z
là giao điểm ca
AH
CD
ta có:
,
HC ZC
HD ZD
=
do
MH ZH
nên
MH
phân giác ngoài ca góc
CHD
t đó ta cũng có:
HC MC
HD MD
=
suy ra
.1
MC ZC MC ZD
MD ZD MD ZC
= =
ây là h thc rt hay gặp trong các đ thi TS vào 10
chuyên).
8. Ta
0 0 0
11
180 180 180
22
BHD DHA CHD COD= = =
(10) (Do
AH
phân giác ca góc
CHD
t giác
CHOD
ni tiếp đã chứng minh trên).
1
2
COD CBD=
(11) (tính cht góc ni tiếp
bng
1
2
s đo góc ở tâm chn cùng mt cung)
0
180CBD CAD=−
(12). T (10), (11), (12) suy ra
.BHD CAD=
Chú ý rng:
ACD
HBD
BHD CAD=
ACD HBD=
cùng chn cung
AD
nên
ACD HBD
(g.g) suy ra
.ADC HDB=
T đó ta cũng suy ra
...ADH CDB=
9. Gi s tiếp tuyến ti
,CD
ct nhau
K
thì
0
90KCO KDO==
nên t giác
KCOD
ni tiếp. Mt
khác theo 6) ta t giác
CHOD
ni tiếp. T đó suy ra 5 điểm
, , , ,K C H O D
cùng nằm trên đường
tròn đường kính
KO
suy ra
0
90 ,KHO =
mặt khác ta cũng
0
90 , ,AHO K A H=
thng hàng,
hay
K
nằm trên đường thng
.AB
10. Theo chng minh 1) thì 4 đim
, , ,A E O M
nằm trên đường tròn đường kính
MO
nên ta
1
2
AEM AOM AOB AFB= = =
(Tính cht góc ni tiếp góc tâm). hai góc
,AEM AFB
đồng
v nên suy ra
/ / .EM BF
11. Theo chng minh 6) ta t giác
CHOD
ni tiếp kết hp vi 7)
AH
chứa đường phân giác ca
góc
CHD
nên
11
22
CHA CHD COD CPD===
mà hai góc
,CHA CPD
đồng v nên
/ / .HA DP
12. Để chng minh:
CN OB
ta chng minh
/ / .CN MB
Tht vy theo ch dựng điểm
N
ta có:
CEN CDB=
ng v). Mt khác
CDB CAB=
cùng chn
cung
BC
suy ra
CEN CAB CAN= =
t giác
CEAN
ni tiếp (có 2 đỉnh liên tiếp
,AE
cùng nhìn
cnh
CN
mt góc bng nhau). T đó suy ra
ECN EAN EAB==
(13) mt khác t giác
EAMB
ni
tiếp (chng minh 1) nên
EAB EMB=
(14). T (13), (14) suy ra
ECN EMB=
suy ra
//CN MB
.MB OB CN OB
13. Do
AQ
đường kính ca
( )
O
nên
0
90ADQ ADYH=
t giác ni tiếp. Suy ra
.AYD AHD=
Mt khác theo 6, 7) ta
CHOD
ni tiếp
AH
phân giác ca góc
CHD
suy ra
11
22
AHD CHD COD CQD AYD CQD= = = =
suy ra
/ / .AY CQ
Xét hai tam giác
,AOY QOX
ta
có:
,OA OQ=
,AOY QOX=
YAO XQO=
nên
AOY QOX =
(g.c.g) suy ra
.OX OY=
Chú ý: Ta cũng có thể chứng minh theo cách khác như sau: Tứ giác
AEOM
ni tiếp nên:
,AEC AEM AOM QOY = =
YQO DQO DCA ECA =
nên
QY CA
OYQ EAC
QO CE
=
hay
11
22
QY CA QY CA
QYA CAD
QA CD
QA CD
= =
(c.g.c) nên
YAQ ADC ECA AQC= = =
//AY QX
nên t giác
AXQY
là hình bình hành suy ra
.OX OY=
Tính chất này là 1 trường hợp đc bit ca bài toán:
Cho tam giác nhn
ABC
ni tiếp
( )
O
điểm
P
nằm trong tam giác, đường thng
AP
ct
,BC
( )
O
lần lượt ti
,.QD
Đưng tròn ngoi tiếp tam giác
PQD
ct
,S
SO
ct
,AC AB
lần lượt ti
,.EF
Khi đó ta có:
.
PE QE
PF QF
=
Ta có th chứng minh bài toán này như sau:
Ta có:
DQC DPS APF==
PAF QCD=
nên
QCD PAF
Tương tự ta cũng có:
QBD PAE
Nên
..
PE PE AP QD QC QC
PF AP PF QB QD QB
= = =
đpcm. Khi
PO
ta có bài toán trên.
14. Đưng thẳng qua trung đim
E
ca
CD
song song vi
AD
ct
AB
ti
,F
DF
ct
AM
ti
1
N
thì
1
N
là trung điểm
.AM
Ta có biến đổi góc:
FEC ADC ABC==
Suy ra
EFCB
ni tiếp. Gi s
CF
ct
AD
ti
1
K
thì
EF
đường trung bình
ca tam giác
1
CDK
hay
F
trung đim
ca
1
.CK
Theo b đề hình thang ta suy ra
1
N
là trung điểm ca
.AM
15. Ta
CHOD
t giác ni tiếp,
AH
phân giác
ca góc
.CHD
Tương tự ta cũng có:
11
C HOD
cũng tứ giác ni tiếp
HB
là phân giác ca góc
11
.C HD
Ta có:
( )
1 1 1 1 1 1 1 1
1
2
CI C CC DD CHC CHM MHC ODC OD C= + = + = +
( )
( ) ( )
0 0 0
1 1 1 1 1 1
1 1 1
180 180 360
2 2 2
COD C OD CD C D CC DD= + = = +
hay
1 1 1
CI C CHC=
tc t giác
11
CI HC
ni tiếp, tương tự ta cũng
11
DD HI
ni tiếp. T đó ta
biến đổi góc
1 1 1 1 1
CHI CC D CD D CC D DHI= = = =
hay
1
IH
cũng phân giác của
.CHD
Suy ra
1
,,A I H
thng hàng.
16. Chng minh các h thc:
+
KC MC
KD MD
=
(*)
+
22
.LM LK LC LD==
+
..ME MK MC MD=
Gii:
+ Trước tiên ta chng minh các h thc trên
tương đương với nhau:
Tht vy ta có:
( )( )
22
.LM LC LD LM MC ML MD ML= =
( ) ( )
22
..LM MC MD ML MC MD ML MC MD ML MC MD= + + = +
( )
. .2 .
22
MK MK
MC MD MC MD ME MK ME = + = =
như vậy (**) và (***) tương đương nhau.
+ Ta cũng có:
( ) ( ) ( )
. . 2 .
KC MC
MD KC MC KD MD MK MC MC MD MK MC MD MK MC MD
KD MD
= = = = +
2 . .2 . .MC MD MK ME MC MD MK ME= =
vậy (*) và (***) tương đương nhau.
Bây gi ta chng minh (***). Gi
H
giao điểm ca
,AB MO
thì
CHOD
KHOE
các t giác
ni tiếp nên
. . .MC MD MH MO MK ME==
(đpcm)
17. Gi
'E
là trung điểm ca
.BD
Do
''EJ
song song vi
'BC
nên
''EJ
đường trung bình ca
'DBC
dn ti
'J
trung
điểm ca
'CD
suy ra
0
' 90OJ D =
nên 4 điểm
''OE J D
nằm trên đường tròn đường kính
.OD R=
18. Qua
G
k các đường thng song song vi
,EO MD
ct
,OB MB
lần lượt ti
,.UV
Ta các đim
,,B O M
c định
2
,
3
BG BU BV UV
BE BO BM MO
= = = =
suy ra
,UV
c định và
0
90 .UGV =
Suy ra
G
thuộc đường tròn đường kính
2
.
3
PQ MO=
19. Do
0
90MTB MHB==
nên t giác
MTHB
ni tiếp nên
TBH TMH=
mt khác
TBH TBA TAM=
suy ra
TMH TAM=
hay
HM
tiếp tuyến của đường tròn ngoi tiếp tam giác
.ATM
Vậy đường tròn ngoi tiếp tam giác
ATM
luôn tiếp xúc với đường thng c định
.MO
Một bài toán tương tự ca tính cht này.
Cho đường tròn tâm
,O
đường kính
.BC
Một điểm
A
di chuyn trên
( )
O
sao cho
.AB AC
Tiếp
tuyến ti
A
ca
( )
O
cắt đường thng
BC
ti
.D
Gi
E
là điểm đối xng vi
A
qua
,BC
AE
ct
BC
ti
.M
K đường cao
AH
ca
.ABE
Gi
I
trung đim ca
,AH BI
ct
( )
O
ti
.K
Gi
N
giao
điểm ca
AK
vi
.BD
Chng minh:
N
trung điểm ca
.MD
ng dn:
Ta
M
trung điểm ca
AE
nên
IM
đường
trung bình ca tam giác
AHE
suy ra
/ / ,IF HE
kết
hp vi
BAKC
t giác ni tiếp. Ta biến đổi
góc:
AMI AEB ACB AKI= = =
suy ra t giác
AIFK
ni tiếp. Chú ý rng:
IM AH
(do
)AH BE
nên t giác
AIMK
ni tiếp đường tròn
đường kính
.AM
Gi
S
giao đim ca
DK
vi
( )
.O
Ta
KMD KAE=
(cùng ph vi
)KMA
KAE KED KMD KED= =
hay
KMED
ni tiếp. Dẫn đến
KDM KEM KEA KSA= = =
suy ra
/ / .AS BD
Ta có:
.KDN KSA KAD==
Suy ra
2
..KND DNA NK NA ND =
Trong tam giác
vuông
NMA
ta có:
2
.MN NK NA=
t đó suy ra
.MN ND=
20. Dng
( )
1
,OA d
do
( )
,dO
c định nên
1
OA
không đổi, gi s
1
OA
ct
AB
ti
2
.A
Ta có:
1
1 2 1 2
2
. . .
OA
OM
MAO A HO OA OA OH OM
OH OA
= =
Trong tam giác vuông
MOA
ta cũng có:
22
.OH OM OA R==
suy ra
2
2
1 2 2
1
.
R
OA OA R OA
OA
= =
không đổi, suy ra
2
A
điểm c định. Mt khác
0
2
90A HO =
suy ra
H
thuộc đường tròn đường kính
2
.OA
21. Theo tính cht 10) ta có
1
//AB CD
nên
1
.
MB D MAD
SS=
Dng
1
DD AM
thì
1
1 1 1
. . 2 . . .
2 2 2
MAD
S DD MA DAMA R MA R MA= =
Du bng xy ra khi và ch khi
1
, 2 .D A DA R=
Hay
D
điểm đối xng vi
A
qua
.O
T đó ta xác định đưc v trí cát tuyến
MCD
như sau: Dựng đường
kính
,AD
ni
MD
được cát tuyến
MCD
cn tìm.
22. Ta có:
( )
2 . .
MWS MOS
S S OAMS OA MA AS
= = = +
Áp dng bất đẳng thc Côsi dng
2x y xy+
ta
có:
2 . .MA AS MA AS+
Mt khác theo h thức lượng trong tam giác vuông
MOS
ta
22
.MA AS OA R==
suy ra
2.MA AS R+
Do đó
2
2.
MWS
SR
Du bng xy ra khi ch khi
AM AS MOS=
vuông cân ti
O
hay
2.MO R=
Để gii các câu hi 19, 20 ta cn biết bài toán ni tiếng sau:
Định lý Ptolemy cho t giác ni tiếp: Cho t giác
ABCD
ni tiếp trong đường tròn
( )
.O
Khi đó ta có:
. . . .ABCD AD BC AC BD+=
Chng minh:
Trên đường chéo
BD
lấy điểm
E
sao cho
.DAE BAC=
Ta
DAE BAC=
ADE ACB=
(cùng chn
)AB
nên
ADE ACB
(g.g)
AD DE
AC BC
=
..AD BC AC DE=
(1). Do
DAE CAB=
nên
,DAC EAB=
li
ABE ACD=
(cùng chn
)AD
ABE ACD
(g.g)
AB BE
AC CD
=
..ABCD AC BE=
(2).
T (1) và (2) suy ra
( )
. . . .ABCD AD BC AC BE DE AC BD+ = + =
23. Khi
00
2 2 30 60 ,MO R MO OA AMO AMB MAB= = = =
đều nên
.MA MB AB==
Mt khác t giác
MEOB
ni tiếp (chng minh
1). Áp dụng định Potolemy cho t giác
MEOB
ta có:
. . . .EAMB EB MA EM AB+=
MA MB AB==
suy ra
.EA EB EM+=
Do đó
2.EA EB EM EM+ + =
vy tng
EA EB EM++
ln nht khi ch
khi
EM
ln nht, mt khác
2.EM MO R=
Vy
EM
ln nht
2.EM MO R E O = =
Nói
cách khác cát tuyến
MCD
đi qua tâm
.O
24. Vi mi s thực dương
,xy
ta có:
( )
1 1 1
2 .2 4.x y xy
x y xy

+ + =


Nên ta suy ra
1 1 4
.
x y x y
+
+
Áp dng vào bài toán ta có:
1 1 4 4
.
EB EC EB EC EM
+ =
+
Do
2EM MO R=
nên ta
suy ra
1 1 4
2.
2
R
MA MB R
+ =
Du bng xy ra khi ch khi
EO
nói cách khác cát tuyến
MCD
đi
qua tâm
.O
CHUYÊN ĐỀ BỒI DƯỠNG HỌC SINH GIỎI HÌNH HỌC 9
1 | THCS.TOANMATH.com
CHUYÊN ĐỀ 5. THNG HÀNG, ĐỒNG QUY, ĐIỂM C ĐỊNH, ĐƯỜNG C ĐỊNH
I.THNG HÀNG, ĐỒNG QUY:Những điểm thẳng hàng đặc bit :
1. B đề hình thang
Cho hình thnag ABCD có hai đáy AB, CD khi đó trung điểm các cạnh đáy, giao điểm 2 đường
chéo và giao điểm ca 2 cnh bên nm trên một đường thng.
Chng minh:
Gi s các đường thng AD, BC ct nhau ti M,
AC, BD ct nhau ti P, đường thng MP ct AB, CD
lần lượt ti N, Q .Ta chng minh: N,Q lần lượt là trung điểm
ca AB, CD.
Tht vy: Do
/ / ,AB CD
theo định lý Thales ta có:
( ) ( )
1 , 2
AN NB AN BN
QD QC QC QD
==
. Ly (1) nhân vi (2) ta có:
22
..
AN NB
AN NB
QC QD QC QD
= =
thay vào (1) ta
QD QC=
. Hay N, Q lần lượt trung điểm ca
AB, CD.
Ví d 1.
Cho tam giác nhn ABC đường cao AH, phân giác trong góc
BAC
ct BC ti O, qua O dng các
đường thng OM vuông góc vi AB, ON vuông góc vi AC.
a. Chứng minh: 5 điểm A, M, H, O, N cùng nm trên một đường tròn.
b. Chng minh: AH là phân giác ca
MHN
.
c. Đưng thng qua O vuông góc vi BC ct MN ti K. Chng minh:
..KN AC KM AB=
.
d. Gi I là trung điểm BC. Chng minh: A, K, I thng hàng .
Gii:
Do
AO MN
nên ta có:
ONK NAO=
(cùng ph vi
NOA
), ta cũng có:
90NOK NOC OCA= =
.
T đó suy ra
OKN COA
(g.g) dẫn đến
,
KN ON
OA CA
=
tương
t ta cũng có:
KM OM
OA BA
=
Do
OM ON=
suy ra
KM AC
KN AB
=
hay
..KM AB KN AC=
.
CHUYÊN ĐỀ BỒI DƯỠNG HC SINH GII HÌNH HC 9
THCS.TOANMATH.com | 2
Dng đường thng qua K song song vi BC ct AB, AC lần lượt ti E, F, ta d chứng minh được:
KEMO, KNFO ni tiếp, kết hp vi
OMK ONK=
ta biến đổi góc:
OEK OMK ONK OFK= = =
suy ra tam giác OEF cân ti O, dn ti
,KE KF=
theo b đề hình
thang ta có A, K, I thng hàng.
2. Đưng thẳng Ơle:
Trong mt tam giác: Trc tâm H, trng tâm G, tâm đường tròn ngoi tiếp O nm trên một đường
thng gọi là đường thẳng Ơle của tam giác. Đồng thi ta có:
3HO GO=
.
Chng minh :
Dựng đường kính AN ca (O) .Vì AN là đường
kính ca (O) nên
,NC AC
do
//BH AC BH NC⊥
.
Chng minh tương tự ta cũng có
//CH NB
nên t giác
BHCN là hình bình hành, suy ra 2 đường chéo NH, BC
ct nhau tại trung điểm M ca mỗi đường nên N, H, M
thng hàng.
Ta MO đường trung bình ca tam giác AHN nên
1
//
2
MO AH=
. Gi G giao đim ca AM
HO, do
//MO AH
(cùng vuông góc vi BC). Theo định lý Thales ta có:
1
2
AG MO
GM AH
==
G
trng tâm ca tam giác ABC H, G, O thng hàng. Do
1
3
2
GO OM
HO GO
GH AH
= = =
. (Đường
thng qua H, G, O gọi là đường thẳng Ơle của tam giác ABC).
3. Đưng thng Simson , đường thng Steiner.
Đưng thng Simson: Cho tam giác ABC ni tiếp đường tròn (O) M là một đim bt k trên đường
tròn. K MH, MI , MK lần lưt vuông góc vi AB, BC, AC. Chng minh rằng ba điểm H, I, K thng
hàng.
Chng minh :
T giác MIBH
90 90 180BHM BIM+ = + =
nên là t giác ni tiếp
MIH MBH=
(cùng chn
cung HM), mà t giác ABMC ni tiếp nên
MBH KCM=
, do đó
MIH KCM=
.
Mt khác t giác KCMI ni tiếp
(vì
90MIC MKC= =
) nên
180 180 180KCM MIK MIH MIK HIK+ = + = =
. Vy H, I,
K thng hàng.
CHUYÊN ĐỀ BỒI DƯỠNG HỌC SINH GIỎI HÌNH HỌC 9
3 | THCS.TOANMATH.com
Đưng thẳng đi qua H, I , K được gọi là đường thng Simson của điểm M.
Chú ý: Ta bài toán đảo v bài toán Simson như sau: Cho tam giác ABC một điểm M nm
ngoài tam giác. Chng minh rng nếu hình chiếu ca M lên ba cnh ca tam giác ABC ba đim
thng hàng thì M nằm trên đường tròn ngoi tiếp tam giác ABC.
Ví d 1.
Cho tam giác nhn ABC ni tiếp (O) , tiếp tuyến ti A ca (O) ct CB ti K, k tiếp tuyến KD vi
(O). Gi G,E,F lần lượt là hình chiếu vuông góc ca D trên AB, BC, CA.
a. Chng minh:
2
.KA KB KC=
.
b. Chng minh:
AB DB
AC DC
=
c. Chng minh:
2 sinBC R BAC=
.
d. Chng minh: G là trung điểm ca EF.
Gii:
a. Hc sinh t chng minh
b. T chng minh câu a ta suy ra:
KBA KAC
suy ra
,
KA AB
KC AC
=
tương t
KB BD
KC BC
=
suy ra
.
AB BD
AC BC
=
c. K đường kính BK ca (O) ta có:
90 ,BCK =
li có
,BAC BKC=
suy ra
sin sin .sin ,hay BC=2RsinA
BC
BAC BKC BC BK BAC
BK
= = =
.
d. Áp dng câu c ta có: T giác BGDE ni tiếp đường tròn đường kính BD nên
.sin .sin ,GE BD GDE BD ABC==
tương tự ta cũng có:
.sin .sinGF CD FCG CD ACB==
T đó suy ra
.sin .sin
,
.sin .sin
GE BD ABC AB ABC
GF
CD ACB AC ACB
==
dựng đường cao AH ca tam giác ABC thì ta có:
.sin .sin
1
.sin .sin
GE BD ABC AB ABC AH
GF AH
CD ACB AC ACB
= = = =
suy ra
GE GF=
. Mt khác t các t giác BGDE,
CFGD, ABCD ni tiếp ta biến đổi góc:
180EGD EBD ACD DGF= = =
180EGD DGF + =
hay E, G, F thng hàng. Nói cách khác G là trung đim ca EF.
(Đường thng qua E, G, F chính là đường thng Simson ca D vi tam giác ABC)
Đưng thng Steiner
Cho tam giác ABC ni tiếp đường tròn (O), M điểm bt k thuộc đường tròn. Gi N, P, Q theo
th t là các điểm đối xng vi M qua AB, BC, CA. Chng minh rng N, P, Q thng hàng.
CHUYÊN ĐỀ BỒI DƯỠNG HC SINH GII HÌNH HC 9
THCS.TOANMATH.com | 4
Chng minh:
Gi H, I, K theo th t là hình chiếu ca M lên AB, BC, AC; thế thì H, I, K thng hàng (đường thng
Simson ). D thy IH là đường trung bình ca tam giác MNP nên
//IH NP
.
Tương t
//IK PQ
.
Theo tiên đề Ơ-clit và do H, I, K thng hàng
nên suy ra N, P, Q thng hàng. Đưng thẳng đi qua N, P, Q được gi
là đường thng Steiner của điểm M.
Chú ý :
a) Ta có th chứng minh ba đim N, P, Q thng hàng bng cách dùng phép v t: Các điểm N, P, Q
lần lượt nh ca H, I, K trong phép v t tâm M t s 2, H, I, K thng hàng nên N, P,Q cũng
thng hàng. Như vậy đường thng Steiner là nh của đường thng Simson trong phép v t tâm M t
s 2.
Ngoài ra liên quan đến đường thng Simson, Steiner ta cũng có kết qu sau:
“ Đường thẳng Steiner đi qua trực tâm ca tam giác ABC.
Tht vy, gi D là trc tâm ca tam giác ABC; BD, CD ct (O) lần lượt E, F. D dàng chng minh
được E đối xng vi D qua AC, F đối xng vi D qua AB. Ta FDMN hình thang cân và các t
giác IBHM , MBFC ni
tiếp nên ta có:
DFM DNM MBC IHM= = =
do đó
//ND IH
. Tương tự ta cũng có:
//DQ IK
H, I, K thng hàng nên N, P, Q
thng hàng. Nói cách khác: Đường thng Steiner
của điểm M đi qua trực tâm ca tam giác ABC.
Cách khác:
Gi AS, BJ, CR các đường cao ca tam giác ABC, D trc tâm. Ta
ANB AMB=
(tính cht
đối xng ). Li có
AMB ADJ=
(cùng bù vi
SDJ
).
Suy ra
ANB ADJ=
nên ADBN t giác ni tiếp, do đó
NAB NDB=
.
NAB MAB NDB MAB= =
.
Chng minh tương tự
CDQ CAM=
. Ta có
NDB CDQ MAB CAM BAC+ = + =
180NDQ NDB BDC CDQ BAC BDC = + + = + =
. Vy N, D, Q thẳng hàng hay đường thng
Steiner đi qua trực tâm ca tam giác ABC.
Ví d 1.
CHUYÊN ĐỀ BỒI DƯỠNG HỌC SINH GIỎI HÌNH HỌC 9
5 | THCS.TOANMATH.com
Cho tam giác nhn ABC ni tiếp (O) có trực tâm là điểm H. Một điểm D nm trên cung nh BC, gi
E điểm đối xng vi D qua BC, đường tròn ngoi tiếp tam giác ODE ct AD ti G. Gi J giao
điểm th 2 của đường tròn ngoi tiếp tam giác AGO vi AH.
Chng minh: J, O, E thng hàng.
a. Chng minh: G là tâm đường tròn ngoi tiếp tam giác JHE.
b. Chng minh: Trc tâm tam giác AGO nằm trên đường thng HE.
Gii:
a, Ta có:
180GOE GDE GAJ GOJ= = =
Suy ra
180GOE GOJ+ =
hay J, O, E thng hàng.
b, T các t giác AJOG, ODEG ni tiếp
Ta có biến đổi góc:
GJO GAO GDO GEO= = =
suy ra tam giác GJE cân ti G.
Gi F giao điểm th 2 ca AH vi (O) thì F đối xng vi H qua BC nên t giác HEDF hình
thang cân. Gi s đường tròn ngoi tiếp tam giác JHE cắt đường tròn ngoi tiếp tam giác AGO ti
giao điểm th 2 K. Suy ra
( )
2 2 2 180 2JGE AOD ABD AFD JHE JKE= = = = =
suy ra G
tâm đường tròn ngoi tiếp tam giác JHE.
c, Ta có:
GOE GDE GAJ GKJ GJK GOK= = = = =
li
GKO GJO GEO==
suy ra
GOK GOE =
hay OG là trung trc ca KE hay E là điểm đối xng vi K qua OG. Do
GK GJ=
t giác AJGK ni tiếp nên AG phân giác ca
HAK
GKA GJH GHJ==
suy ra H, K đối
xng nhau qua AG. Suy ra HE đường thng Steiner ca K trong tam giác AGO. Theo tính cht
va chng minh thì HK đi qua trực tâm ca tam giác AGO.
4. Đưng thng Pascal
Cho 6 điểm A, B, C, D, E, F cùng thuc một đường tròn (Có th hoán đổi th t ). Gi P, Q , R ln
ợt giao điểm ca các cặp đường thng
( ) ( ) ( )
, , , , ,AB DE BC EF CD FA
. Khi đó 3 điểm P, Q, R
cùng nm trên một đường thng gọi là đường thng Pascal.
Chng minh:
Gi s DE ct BC ti M ct AF
ti N, BC ct AF ti S.
Áp dụng định lý Menelaus cho
SMN
và cát tuyến: ABP ta có :
( )
. . 1 hay . 1
PM AN BS PM AS BM
PN AS BM PN AN BS
==
.
CHUYÊN ĐỀ BỒI DƯỠNG HC SINH GII HÌNH HC 9
THCS.TOANMATH.com | 6
Áp dụng định lý Menelaus cho
SMN
cát tuyến : CDR ta có:
( )
. . 1 suy ra . 2
RN CS DM RN CM DN
RS CM DN RS CS DM
==
. Áp dng định lý
Menelaus cho
SMN
và cát tuyến: QEF ta có:
( )
. . 1 suy ra . 3
QM FS EN QS FS EN
QS FN EM QM FN EM
==
.
Mt khác các t giác ABCF, BCDE, AFED ni tiếp nên:
( )
. . ; . . , . . 4SB SC SASF MC MB MD ME NF NA ND NE= = =
. T
( ) ( ) ( ) ( )
1 , 2 , 3 , 4
ta suy ra
. . 1
PM RN QS
PN RS QM
=
.Theo định lý đảo Menelaus ta suy ra P, Q, R thng hàng
Đưng thng PRQ trên được gọi là đường thng Pascal ng vi b điểm
A,B,C, D,E,F. Bng cách hoán v các điểm A,B,C, D,E,F
ta thu được rt nhiều các đường thng Pascal khác nhau, c th
ta có ti 60 đường thng Pascal. Chng hn v hình bên minh ha
trường hợp các điểm ACEBFD.
Ngoài ra khi cho các điểm trùng nhau (khi đó lục giác suy biến thành tam giác,
t giác, ngiác), dụ
EF
thì cnh EF tr thành tiếp tuyến của đường ti E, ta còn thu thêm
được rt nhiều các đường thng Pascal khác na. Hình v dưới đây minh họa trường hợp các điểm
ABCDEE, ABCCDD.
5. Đưng thng Gauss
Cho t giác ABCD AB, CD ct nhau ti M, AD, BC ct nhau ti N. Khi đó trung điểm các đoạn
thng AC, BD, MN nm trên một đường thng gọi là đường thng Gauss ca t giác ABCD.
Chng minh:
Gi I, E, F lần lượt là trung điểm ca BD, AC ,MN
K, G, H lần lượt là trung điểm ca các
đoạn thng CN, CD, DN. D thy các điểm F, H, K thng hàng.
CHUYÊN ĐỀ BỒI DƯỠNG HỌC SINH GIỎI HÌNH HỌC 9
7 | THCS.TOANMATH.com
E, G, K thng hàng. I, G, H thng hàng.
Ta có:
/ / , / / , / /FK MC IH BC EK DN
nên
,,
IG BC FH MD EK AN
IH BN FK MC EG AD
= = =
nhân 3 đẳng thc ta có:
. . . .
IG FH EK BC MD AN
IH FK EG BN MC AD
=
. Áp dụng định lý Menelaus cho tam giác CDN đường thẳng đi
qua B, A, M ta có:
. . 1
BC MD AN
BN MC AD
=
, suy ra
. . 1
IG FH EK
IH FK EG
=
. Theo định lý Menelaus đảo ta suy
ra I, E, F thng hàng.
6. Đưng thẳng Niutơn
Cho t giác ABCD ngoi tiếp (I). Gi M, N lần lượt là trung điểm ca BD, AC . Khi đó 3 điểm I, M,
N thng hàng. Đưng thẳng đi qua I, M, N gọi là đường thẳng Niutơn của t giác ABCD.
Chng minh: (Ta ch xét trường hp AB không song song vi CD)
Gi các tiếp điểm ca (I) vi AB, BC, CD, DA lần lượt
X, Y, Z, T thì
IX IY IZ IT r= = = =
.
Gi s AD, BC ct nhau ti P, trên PD ly E sao cho
,PE AD=
trên PC ly F sao cho
PF BC=
thế thì:
1 1 1
2 2 2
MAD MBC DAB DBC ABCD
S S S S S+ = + =
.
1 1 1
2 2 2
NAD NBC CAD ABC ABCD
S S S S S+ = + =
. T đó suy ra :
MAD MBC NAD NBC
S S S S+ = +
. Theo cách xác định
E, F ta có:
, , ,
MAD MPE MBC MPF NAD NPE NBC NPF
S S S S S S S S= = = =
suy ra:
( )
hay S 1
MPE MPF NPE NPF MEPF NEPE
S S S S S+ = + =
.
Li có:
,
IAD IPE IBC IPF IAD IBC IPE IPF IEPF
S S S S S S S S S= = + = + =
nhưng
1 1 1 1 1
2 2 2 2 2
IAD IBC AXYT DZYT IZCY XIYB ABCD
S S S S S S S+ = + + + =
suy ra
( )
2
MEPF IEPF
SS=
. T (1), (2) ta suy ra
hay S / / , / / suy ra
MEPF NEPF IEPF MEF NEF IEF
S S S S S MN EF MI EF M,N,I= = = =
thng hàng.
7. Trục đẳng phương của hai đường tròn
a. Cho đường tròn (O;R) và một đim M, đường thẳng thay đổi qua M ct (O) ti A, B.
Khi đó đại lượng:
( )
/
.
MO
P MAMB=
gọi là phương tích của điểm M với đường tròn (O)
+ Nếu M nm ngoài (O) thì
( )
22
/
.
MO
P MAMB MO R= =
.
CHUYÊN ĐỀ BỒI DƯỠNG HC SINH GII HÌNH HC 9
THCS.TOANMATH.com | 8
+ Nếu M nm trong (O) thì
( )
22
/
.
MO
P MAMB R MO= =
.
+ Nếu M nm trên (O) thì
( )
/
0
MO
P =
.
T đó ta cũng có kết qu: Khi M nm ngoài (O)MC
tiếp tuyến ca (O) ti C thì
( )
2
/
.
MO
P MAMB MC==
.
(Đây là những kết qu quen thuộc đã được chng minh trong phn cát tuyến, tiếp tuyến).
b. Các điểm cùng phương tích với 2 đường tròn phân bit
( )
11
;OR
( )
22
;OR
nm trên mt
đường thng gi là trục đẳng thc của 2 đường tròn đó.
Trong phm vi THCS ta cần chú ý đến các trường hp là:
+ Nếu
( ) ( )
12
,OO
ct nhau theo dây AB, lấy điểm M trên đường thng AB (M nằm ngoài 2 đường
tròn) cát tuyến qua M ct
( )
1
O
ti C, D , cát tuyến qua M ct
( )
2
O
ti E, F thì
. . .MC MD ME MF MAMB==
(Hình 1).
+ Nếu
( ) ( )
12
,OO
tiếp xúc nhau ti N thì trục đẳng phương đường thng qua N vuông góc vi
12
OO
(Hình 2,3)
II. PHƯƠNG PHÁP GIẢI BÀI TOÁN CHNG MINH THẲNG HÀNG , ĐỒNG QUY
1. Mt s tiêu chuẩn để chứng minh ba đim thng hàng.
Để chứng minh 3 điểm A, B, C thng hàng theo th t, thc cht của các phương pháp bn
chứng minh hai đường thng ABAC trùng nhau.
Trong phần này chúng ta đưa ra một s tiêu chuẩn để chứng minh 3 điểm A, B, C thng hàng.
Tiêu chun 1. Ba điểm phân bit A, B, C theo th t nm trên một đưng thng khi ch khi
180 .ABC =
Xét một đường thng
xx
qua B
Để chng minh A, B, C thng hàng ta quy
v chng minh:
CHUYÊN ĐỀ BỒI DƯỠNG HỌC SINH GIỎI HÌNH HỌC 9
9 | THCS.TOANMATH.com
+
180ABx xBC+ =
hoc
ABx x BC
=
Ví d 1.
Cho nửa đường tròn đường kính AB. Ly điểm C thuc AB sao cho
CA CB
điểm M thuc na
đường tròn đó. Đưng thng qua M vuông góc vi MC ct tiếp tuyến qua A ti
1
M
. Đưng thng C
vuông góc vi
1
MC
ct tiếp tuyến qua M ti
2
M
. Chng minh
12
,,M M M
thng hàng.
Gii:
Ta có:
1
CAM CM M=
(T giác
1
ACMM
ni tiếp ).
2
CAM MBM=
(góc gia hai tiếp tuyến và dây cung).
12
CM M MCM=
(cùng ph
1
MCM
)
22
MCM MBM=
t giác
2
BCMM
ni tiếp
2
90CMM =
.
đó ta có:
1 2 1 2
180M MM M MC M MC= + =
(đpcm).
Ví d 2.
Cho tam giác ABC ni tiếp trong đường tròn (O). Các tiếp tuyến qua A, C ct nhau M. V hình
bình hành ACMN. Đưng tròn ngoi tiếp tam giác AMN ct (O) D. Chng minh N, C, D thng
hàng.
Gii:
AMN ADN=
(t giác ADMN ni tiếp) .
( )
so le trongAMN CAM=
CAM ABC=
(góc gia tiếp tuyến và dây cung).
( )
1ADN ABC=
.
Mt khác, t giác ABCD ni tiếp nên
( )
180 2ABC ADC+ =
.
T (1) và (2) ta có :
180ADN ADC+ =
suy ra (đpcm).
Ví d 3.
Cho t giác ABCD ni tiếp đưng tròn (O) các tia AB , CD ct nhau ti E .AD ct BC ti F. Gi M
giao đim th hai khác C của hai đường tròn ngoi tiếp các tam giác BCE, CDF. Chng minh E,
M, F thng hàng.
CHUYÊN ĐỀ BỒI DƯỠNG HC SINH GII HÌNH HC 9
THCS.TOANMATH.com | 10
(Trích đề thi vào lp 10 -Trường chuyên Lê Quý Đôn -Đà Nẵng năm 2012)
Gii:
Để chng minh E, M, F thng hàng ta chng minh:
180 .CME CMF+ =
vy ta cn quy hai góc này v hai góc đối trong mt t giác ni tiếp.
Tht vy ta có:
180EMC EBC=−
do EBCM
ni tiếp.
180FMC CDF=−
do FMCD
ni tiếp. T đó ta có:
360EMC FMC CDF EBC+ =
.
Để ý rng: ABCD là t giác ni tiếp nên:
,CDF ABC EBC CDA==
180ABC CDA+ =
nên
180 .CME CMF+ =
Do đó 3 điểm E, M, F thng hàng.
Ví d 4.
Cho 3 điểm thng hàng theo th t A, M, B v cùng mt phía của đường thng AB v hai hình
vuông AMCD BMEF. Hai đường tròn
( )
1
O
( )
2
O
ngoi tiếp hai hình vuông đó cắt nhau ti M
N. Chng minh rng:
a. B, C, N thng hàng.
b. A, E, N thng hàng.
Gii:
a. Ta có:
90ANC =
(góc chn nửa đường tròn)
45MNE MEF= =
(góc ni tiếp).
45MNB MEB= =
(góc ni tiếp).
90ANB =
.
Vy
180CNA ANB CNB+ = =
(đpcm).
b. T kết qu câu trên, ta có :
MBC MEA =
( )
. . .c g c MBC MEA=
Mt khác t giác BMEN ni tiếp được nên
180 180MBN MEN MEN MEA+ = + =
(đpcm).
Ví d 5.
Cho tam giác nhn ABC ni tiếp đường tròn (O) trc tâm H. Gi P điểm nằm trên đường tròn
ngoi tiếp tam giác
( )
,,HBC P B C H
nm trong tam giác ABC. PB cắt đường tròn (O) ti M
khác B. PC ct (O) ti N khác C. BM ct AC ti E, CN ct AB ti F. Đưng tròn ngoi tiếp tam giác
CHUYÊN ĐỀ BỒI DƯỠNG HỌC SINH GIỎI HÌNH HỌC 9
11 | THCS.TOANMATH.com
AME đường tròn ngoi tiếp tam giác ANF ct nhau ti Q khác A. Chng minh M, N, Q thng
hàng.
(Trích đề tuyn sinh vào lp 10 chuyên Trường THPT chuyên ĐHQG Hà Nội - năm 2013)
Gii:
Để chng minh: M, N, Q thng hàng ta chng minh:
180MQA NQA+ =
.
Ta
180BPC BHC BAC= =
nên t giác AEPF ni tiếp,
suy ra
BFC BEA=
suy ra
180BFC BEC+ =
T các t giác AQFN, AQEM ni tiếp
ta có
MQN MQA NQA=+
180MEA NFA BFC BEC= + = + =
.
Vy 3 điểm M, N, Q thng hàng.
Ví d 6.
Cho tam giác ABC điểm O nằm trong tam giác đó (O không nm trên các cnh tam giác). Đim
M nm trên tia OA (M khác O, A) sao cho đường tròn ngoi tiếp tam giác ABM ct tia OB ti giao
điểm th haiN, đường tròn ngoi tiếp tam giác ACM ct tia OC tại giao điểm th hai P. Gi I,
J lần lượt là tâm đường tròn ngoi tiếp các tam giác ABC, MNP. Chng minh: O, I, J thng hàng.
(Trích đề tuyn sinh vào lp 10 Trường chuyên tnh Bà Rịa Vũng Tàu – Năm 2012).
Phân tích định hướng gii
Gi s đường tròn (J) ct OA, OB, OC lần lượt ti D, E, F. Để chng minh O, I, J thng hàng ta
chng minh:
180AOJ DOJ+ =
. Điều này cũng
tương đương với
AOI DOJ=
. Ta cn
tìm liên h ca các góc này vi các góc
ca t giác ni tiếp. Tht vy ta có:
,JDO JDE ODE IAO IAB OAB= =
li có:
180 180
,,
22
EJD AIB
JDE EIB
==
mt khác
2 , 2EJD DFE AIB ACB==
.
Bây gi ta quan tâm đến hai tam giác
,ABC DEF
.
Nếu hai tam giác này đồng dng vi nhau thì bài toán s được gii quyết.
T định hướng trên ta có th giải bài toán như sau:
CHUYÊN ĐỀ BỒI DƯỠNG HC SINH GII HÌNH HC 9
THCS.TOANMATH.com | 12
+ Để ý rng do các t giác AMNB, AMPC là t giác ni tiếp nên ta d chứng minh được:
. . .OM OA ON OB OPOC==
suy ra tam giác
OBC OPN
,
BC OB OC
OPN OEF OBC OEF
BF OE OF
= = =
. Hoàn toàn tương tự ta có :
OAB ODE
,
AB OA OB AC OA OC
OAC ODF
DE OD OE DF OD OF
= = = =
.
T đó suy ra
BC AB AC
EF DE DF
==
do đó
ABC DEF
suy ra
DEF ACB=
dẫn đến:
AIB EJD JDE EIB= =
mt khác
ODE OAB=
nên
JDO IAO=
.
+ Tam giác IAB cân ti I ,
JDE
cân ti J
AIB EJD=
nên hai tam giác này đồng dng vi
nhau, dẫn đến
IA AB
JD DE
=
nhưng
AB OA
DE OD
=
suy ra
IA OA
JD OD
=
t đó ta có:
AOI DOJ
suy ra
AOI DOJ=
.
Tiêu chun 2. A, B, C thng hàng
đường thng ABAC cùng song song hoc cùng vuông góc
vi một đường thng.
Ngoài ra ta cũng có thể chng minh
theo hướng:
DAB DAC=
thì A, B, C thng hàng.
Ví d 1.
Cho nửa đường tròn tâm (O) đưng kính AB. Gi C trung điểm cung
,AB K
trung điểm đoạn
BC, AK ct (O) ti M . K CH vuông góc vi AM; OH ct BC ti N, MN ct (O) ti D. Chng minh
B, H, D thng hàng.
Gii:
Ta có:
90AMB =
(góc chn nửa đường tròn)
//HC BM
(cùng vuông góc vi AM).
K là trung điểm ca BC nên K là trung điểm ca
HM BM HC=
BHCM
là hình bình hành
( )
/ / 1BH MC
.
Mt khác,
45AMC =
(góc chn cung
AC
)
HCM
vuông cân ti
H HC HM=
.
Li có
OC OM=
(bán kính đường tròn (O))
OH
là phân giác ca
COM NC NM=
NCM
cân ti
N NCM NMC=
CMD CBD=
(góc ni tiếp cùng chn cung CD), do đó
( )
/ / 2BCM CBD BD MC=
.T (1) và (2) ta có đpcm.
Ví d 2.
CHUYÊN ĐỀ BỒI DƯỠNG HỌC SINH GIỎI HÌNH HỌC 9
13 | THCS.TOANMATH.com
Cho đường tròn tâm O đường kính AB. Trên đường tròn lấy điểm D khác A sao cho
60DAB 
.
Trên đường kính AB ly điểm C khác A, Bdng
CH AD
ti H. Phân giác trong góc
DAB
ct
đường tròn ti E ct CH ti F. Đưng thng DF cắt đường tròn tại giao đim th 2 N. Chng
minh: T giác AFCN ni tiếp và N, C, E thng hàng.
(Trích đề tuyn sinh vào lp 10 Trường chuyên Phan Bi Châu Ngh An năm 2013)
Gii:
Ta có:
ACH ABD=
ng v).
ABD AND=
(cùng chn cung AD) suy ra
ACH ANF=
Hay t giác AFCN ni tiếp.
Ta có:
CND BAE=
BAE DAE DNE==
vy
CND DNE=
hay 3 điểm N, C, E thng hàng
Ví d 3.
Cho đường tròn (O) đường kính
T điểm A nằm ngoài đường tròn k hai tiếp tuyến AM,
AN đến đường tròn (O) (M, Nhai tiếp đim). Gi H là trc tâm tam giác ABC, F là giao điểm ca
BC AH. Chng minh M, H, N thng hàng.
(Trích đề tuyn sinh vào lp 10 Trường chuyên Bắc Ninh năm 2014)
Gii:
Để chng minh M, H, N thng hàng ta chng minh:
ANH ANM=
.
Ta có:
ANM AFM AFN==
(Do 5 điểm
A, M, N, O, F cùng nm trên một đường tròn và
AM AN=
).
Như vậy ta cn chng minh:
.AFN ANH=
Xét tam giác
,.ANH AFN
Ta có:
NAH
chung,
2
..AH AF AI AO AN==
(Tính cht quen thuc ca tiếp tuyến, cát tuyến)
suy ra
hay ANH AFN suy ra .
AH AN
AFN ANH
AN AF
= =
Ví d 4.
Cho tam giác ABC ni tiếp trong đường tròn (O . Gi
1 1 1
,,A B C
trung đim các cung
,,BC CA AB
ca (O) I tâm đường tròn ni tiếp ca tam giác ABC .
11
AC
ct AB M;
11
AB
ct AC N.
Chng minh M, I, N thng hàng.
Gii:
CHUYÊN ĐỀ BỒI DƯỠNG HC SINH GII HÌNH HC 9
THCS.TOANMATH.com | 14
+
1 1 1
,,A B C
trung đim các cung
,,BC CA AB
nên
1 1 1
,,AA BB CC
các tia phân giác trong ca
tam giác ABC do đó chúng đồng qui ti I.
Ta có:
1 1 1
1
,
2
IBA ACB=
( )
1 1 1 1 1
11
sd
22
A IB AB BA sd ACB= + =
1 1 1
ABI AIB A BI =
cân ti
1
A
+ Tương tự ta có :
1
ACI
cân ti
1
A
Xét t giác
1
ABMI
có:
11
AB AI=
11
MA B MAI=
(góc ni tiếp)
1
AM BI MB MI MBI MIB = =
.
Mt khác,
1
BB
là phân giác góc
ABC
nên
( )
so le trong / /IBM IBC MIB IBC MI BC= =
. Tương tự
//NI BC
,,M N I
thng hàng.
Ví d 5.
Hai đường tròn
( ) ( )
12
,OO
tiếp xúc ngoài ti A .V hai đường kính
12
,AA AA
tiếp tuyến chung
ngoài tương ứng
12
BB
. Gi (d) là đường thng
2
B
và vuông góc vi
12
AB
. Gi
( )
d
là đường thng
qua
2
A
vuông góc vi
2
,AA d
d
giao nhau ti M. Chng minh
1
,,B A M
thng hang.
Phân tích định hướng gii
1 1 1
AB AB
(góc ni tiếp chn nửa đường
tròn), để chng minh
1
,,B A M
thng hàng ta
s chng minh
1 1 1
MB AB
nghĩa là chứng minh
t giác
1 1 2
AB B M
ni tiếp được. Do
1 1 2 2
//O B O B
nên
1 1 2 2
AO B AO B=
(2 góc đồng
v)
1 1 2 2 2 1 2 2
//B AO B A O B A B A =
. Tương tự,
1 1 2 1 1
/ / ,AB AB AB
ct
22
AB
ti N thì
12
AB NB
hình ch nht
1 2 1 2
NB B B B A=
. Mt khác,
1 2 2 2
B B A B A A=
(góc gia tiếp tuyến dây cung )
1 2 2 2 1
NB B B A A =
t giác
1 1 2 2
AB B A
ni tiếp
được. Ta
1 2 2
AB A M
ni tiếp được trong đường tròn đường kính
1
AM
. Vy t giác
1 1 2
AB B M
ni
tiếp được. T đó ta có đpcm.
CHUYÊN ĐỀ BỒI DƯỠNG HỌC SINH GIỎI HÌNH HỌC 9
15 | THCS.TOANMATH.com
Ví d 6.
Cho tam giác nhn ABCtrực tâm điểm H. Gi M, Nchân các đường cao h t B, C ca tam
giác A, B, C. Gi D là điểm trên cnh BC. Gi
( )
1
w
là đường tròn đi qua các điểm B, N, D gi
( )
2
w
là đường tròn đi qua các điểm C, D, M. Dng DP, DQ lần lượt là đường kính ca
( ) ( )
12
,ww
.Chng
minh P, Q, H thng hàng.
( )
2013IMO
Phân tích định hướng gii
Gi S là giao điểm th 2 của hai đường tròn
( ) ( )
12
,ww
.
Ta d chứng minh được ANSM
t giác ni tiếp (Đây là bài toán rất quen
thuc) t đó suy ra 5 điểm A, N, S, H, M
cùng nm trên một đường tròn.
+ Trước hết ta chng minh:
A, S, D thng hàng: Ta có:
ASN AHN=
cùng chn cung
, 180AN NSD NBD NHK= =
do các
t giác NSDB, NHKB ni tiếp. Suy ra
180ASN NSD AHN NHK+ = + =
do đó A, S, D thng hàng.
+ Vì 5 điểm A, N, S, H, M cùng nm trên một đường tròn nên:
90ASH =
. DP đường kính ca
( )
1
w
suy ra
90 ,PSD DQ=
đường kính ca
( )
2
w
nên
90DSQ =
điều đó chứng t các tia PS, HS, QS trùng nhau. Hay P, S, Q thng hàng
Tiêu chun 3: Xét đường thng
( )
đi qua B hai đường thng
( )
1
,
( )
2
song song tương ng
qua A, C to thành hai tam giác BMABNC như một trong hai hình v dưới đây:
Khi đó nếu
MA NC
MB NB
=
thì A, B, C thng hàng.
Ta chng minh tiêu chuẩn này như sau:
12
//
nên ta có:
AMB CNB=
do đó
AMB CNB°
CHUYÊN ĐỀ BỒI DƯỠNG HC SINH GII HÌNH HC 9
THCS.TOANMATH.com | 16
suy ra
MBA NBC=
hay A, B, C thng hàng.
Ta còn gọi đây là phương pháp: “Chứng minh 2 tia trùng nhau”
Ví d 1.
Cho hình thang
( )
//ABCD AB CD
. Hai đường chéo ct nhau ti M , kéo dài hai cnh bên ct nhau
ti N. Chứng minh đường MN đi qua trung điểm ca 2 cạnh đáy. (B đềnh thang)
Gii:
Gi E, F lần lượt là trung điểm ca AB, CD.
Ta chng minh M, E, F thng hàng.
Tht vy, do
//AB CD
nên:
2
2
MC CD FC MC FC
MA AB EA MA EA
= = =
,,M E F
thng hàng.
Ta còn phi chng minh M, E, F thng hàng.
Xét
NAB
//CD AB
nên
,,
CD ND ND DF
N E F
AB NA NA AE
= =
thng hàng. T đó ta có đpcm.
Ngoài ra ta cũng có thể chng minh theo cách khác:
Áp dụng định Thales vi
//AB CD
ta có:
DF CF
AE BE
=
(cùng bng
NF
NE
). Ta cũng có:
DF CF
EB AE
=
(cùng bng
MF
ME
) . Nhân hai đẳng thc trên ta có:
..
DF DF CF CF
AE EB BE AE
=
22
DF CF DF CF = =
suy ra F là trung điểm ca CD.
T đó ta cũng suy ra E là trung điểm ca AB.
Ví d 2.
Cho đường tròn (I) ni tiếp tam giác ABC, (I) tiếp xúc vi BC, AB, AC lần lượt ti D, E, F. V AM
đường trung tuyến ca tam giác ABC. Gi K giao điểm ca EF và DI. Chng minh A, K, M
thng hàng.
Gii:
Qua K v đường thng song song vi BC ct AB, AC lần lượt X, Y
, / /DI BC XY BC DI XY
Ta
90XEI XKI= =
T giác EXKI ni tiếp.
IEK IXK=
.
Tương t t giác IKFY ni tiếp
IFK IYK=
.
( )
IE IF r IEF= =
cân ti
I IEK IFK=
CHUYÊN ĐỀ BỒI DƯỠNG HỌC SINH GIỎI HÌNH HỌC 9
17 | THCS.TOANMATH.com
nên
,IXK YIK IXY=
cân ti I. Mà IK là đường cao
( )
ID XY
.
Do đó IK đường trung tuyến
2
XY
XK KY = =
. Do đó
2
,
2
XK XK XY
ABC
BM BM BC
= =
//
AX XY
XY BC
AB BC
=
. Xét
AXK
ABM
AXK ABM=
ng v
//XY BC
)
AX XK XY
AB BM BC

==


. Do đó
( )
.AXK ABM g g XAK BAM =
Hai tia AK, AM trùng nhau.
Vậy ba điểm A, K, M thng hàng.
Ví d 3.
Cho đường tròn (O) đường kính AB. Trên tiếp tuyến ti A của đường tròn (O) lấy điểm C. V cát
tuyến CDE (tia CD nm gia hai tia CA, CO; D, E thuộc đường tròn (O), D nm gia C E). Gi
M giao điểm ca CO BD. Gi F giao điểm ca AM đường tròn (O) (F khác A). Chng
minh rằng ba điểm E, O, F thng hàng.
Gii:
V
AH OC
ti H. Xét
CAD
CEA
( )
,ACD chung CAD CEA=
(h qu góc to
bi tia tiếp tuyến và dây cung).
Do đó
( )
.CAD CEA g g
2
.
CA CD
CA CD CE
CE CA
= =
,
+
ACO
vuông ti A , AH là đường cao
2
.CA CH CO=
. Do đó
( )
2
..CDCE CH CO CA==
.
Xét
CDH
COE
( )
,
CD DH
DCH chung
CO CE
=
(vì
..CDCE CH CO=
).
Do đó
( )
. . .CDH COE c g c CHD CEO =
Mt khác
90ADB =
(góc ni tiếp chn nửa đường tròn).
Ta
90 90 180ADM AHM+ = + =
t giác ADMH ni tiếp
DAM CHD=
.
CEF DAM=
(hai góc ni tiếp cùng chn cung DF). Do đó
CEF CEO=
hai tia EF, EO trùng
nhau. Vy E, O, F thng hàng.
Ví d 4.
T điểm A nằm ngoài đường tròn (O) v hai tiếp tuyến AB, AC (B, C hai tiếp điểm ) mt cát
tuyến ADE đến (O) sao cho ADE nm gia hai tia AO, AB. Đưng thng qua D song song vi BE
CHUYÊN ĐỀ BỒI DƯỠNG HC SINH GII HÌNH HC 9
THCS.TOANMATH.com | 18
ct BC, AB lần lượt P, Q. Gọi K là điểm đối xng ca B qua E. Chng minh rằng ba điểm A, P, K
thng hàng.
Gii:
+ Gi H, I lần lượt là giao điểm
ca BC vi OA, DE. Ta có AB, AC
là các tiếp tuyến của đường tròn (O)
,AB AC AO=
là tia phân giác ca
BAC
, ABC cân ti
O AO
đường cao ca tam giác ABC.
+ Xét
ABD
AEB
có:
( )
,BAD chung ABD AEB=
(h qu góc to bi tia tiếp tuyến dây
cung).
Do đó
( )
2
. . ,
AB AD
ABD AEB g g AB AD AE ABO
AE AB
= =
vuông ti B, BH đường cao
2
.AB AH AO=
. Do đó
( )
2
..AD AE AH AO AB==
+ Xét
AHD
AEO
( )
,
AH AD
HAD chung
AE AO
=
(vì
..AD AE AH AO=
).
Do đó
( )
..AHD AEO c g c AHD AEO =
t giác OEDH ni tiếp
,OHE ODE ODE =
( )
OD OE R ODE= =
cân ti
O ODE OED=
. Do đó:
OHE AHD=
. Ta
( )
90OHE EHI AHD IHD+ = + =
nên
EHI IHD HI=
là tia phân giác
DHE
.
+ Gi Hx tia đối ca tia HE,
( )
xHA AHD OHE HA= =
đường phân giác ngoài ca góc
HED
nên
ID AD
DE AE
=
.
ABE
/ / ,
DQ AD
DQ BE IBE
BE AE
=
//BE PD DQ DP=
.
ABE
//
AQ QD
QD BE
AB BE
=
. Do đó
2
2
AQ QD PQ
AD BE BK
==
+ Xét
APQ
AKB
AQP ABK=
( đồng v
//PQ BE
),
AQ PQ
AB BK
=
do đó
( )
..APQ AKB c g c
QAP BAK =
hai tia AP, AK trùng nhau hay A, P, K thng hàng.
Ví d 5.
Cho tam giác nhn
( )
ABC AB AC
ni tiếp đường tròn (O) các đường cao AD, BE, CF ct nhau
ti H. V CI vuông góc vi OA ti I. Gi M trung điểm BC. Chng minh rằng ba điểm M, I, F
thng hàng.
Gii:
CHUYÊN ĐỀ BỒI DƯỠNG HỌC SINH GIỎI HÌNH HỌC 9
19 | THCS.TOANMATH.com
+
( )
OB OC R OBC= =
cân ti O,
OM là đường trung tuyến
OM
là đường
cao, đường phân giác nên
1
2
BAC MOC BOC

==


.
Ta có:
90BAC ACF MOC OCM+ = + =
nên
ACF OCM=
. T giác OMIC có:
90OMC OIC= =
nên ni tiếp
AIF ACF=
do vy
AIF OIM=
hai tia IM, IF trùng nhau. Vậy ba điểm M, I, F thng hàng.
Ví d 6.
Cho tam giác ABC nhn ni tiếp đường tròn (O). Đường phân giác AD ca tam giác ct cung BC
E. Đưng tròn (I) tiếp xúc trong vi (O) ti S và tiếp xúc vi BC ti T ct AD M, N (N nm gia A
M), CM cắt đường tròn (O) K. V dây KL song song vi AB. Chng minh rng C, N, L thng
hàng.
Gii:
//OE IT
( cùng vuông góc vi BC)
IST OSE OES==
do các tam giác IST, OSE cân suy ra
S, T, E thng hàng.
+ Xét
ECT
ECS
( )
CET chung
,
ECT ESC=
do đó
( )
..ECT ESC g g
suy ra
2
.
EC ET
ET ES EC
ES EC
= =
+ Xét
EMT
ESA
( )
,MET chung EMT ESA=
.
Do đó
( )
. . .
EM ET
EMT ESN g g ET ES EM EN
ES EN
= =
nên
( ) ( )
2
. . . . .
EC EM
EC EM EN ET ES ECM ENC c g c ECM ENC
EN EC
= = = =
ECD DCM NAC NCA + = +
.
ECD NAC=
nên
DCM NCA=
do
//KL AB BK AL DCM LCA = =
. Ta có
( )
NCA LCA DCM= =
Hai tia CN, CL trùng nhau. Vy C, N , L thng hàng.
Tiêu chun 4: S dng các tính cht của đường tròn đ chng minh A, B, C thng hàng. Ví d: B
tâm của đường tròn đường kính AC hoặc các đường tròn tâm A tâm C tiếp xúc ti B, hai đường
tròn cắt nhau thì đường ni tâm của hai đường tròn vuông góc dây cung chung của hai đường tròn
đó …
CHUYÊN ĐỀ BỒI DƯỠNG HC SINH GII HÌNH HC 9
THCS.TOANMATH.com | 20
Ví d 1.
Cho đường tròn (O) và y cung AB. Ly I thuộc đoạn AB sao cho
IA IB
. Gi D trung điểm
cung nh AB. DI ct (O) tại giao điểm th hai C. Tiếp tuyến vi (O) ti C ct AB ti K, EC ct (O)
tại giao điểm th hai F. Chng minh D, O, F thng hàng.
Gii:
Ta có:
CAD KCD=
(góc gia tia tiếp tuyến
và dây cung).
( )
11
22
CIK sd AD sdBC sdCBD CAD= + = =
CIK ICK KI KC = =
.
Do K là trung điểm ca IE nên
CIE
1
2
CK IE=
CIE
vuông ti
90C DCF O =
trung điểm đường kính DF.
Ví d 2.
Cho đường tròn (O;R) đưng kính AB C nm gia O A. V đưng tròn (I) đường kính BC .
V AD tiếp tuyến ACF cát tuyến của đường tròn
( )( )
I AE AF
sao cho tia AO nm gia hai
tia AD, AE. Đưng thng vuông góc vi AB v t C cắt đường tròn (O) N, P (D, P cùng thuc mt
na mt phng b AB). DI ct NB S. Gi J trung điểm ca SD; L, T lần lượt tâm đưng tròn
ngoi tiếp các tam giác SBC, SEF. Chng minh rằng ba điểm J, L, T thng hàng.
Gii:
Xét
ADC
ABD
DAC
(chung),
ADC ABD=
(h qu góc to bi tia tiếp tuyến dây
cung). Do đó
( )
.ADC ABD g g
2
.,
AD AC
AD AB AC
AB AD
= =
90ANB =
(góc ni tiếp chn nửa đường tròn).
NAB
vuông ti N, NC là đường cao
2
.AN AB AC=
. Do đó
( )
22
.AD AN AB AC==
AD AN=
. Gọi R là giao điểm ca DNAS.
Xét
( )
90DAS ADS =
( )
90DAS ANS =
AS chung,
AD AN=
. Do đó
DAS NAS =
(cnh huyn cnh góc vuông)
CHUYÊN ĐỀ BỒI DƯỠNG HỌC SINH GIỎI HÌNH HỌC 9
21 | THCS.TOANMATH.com
,DAS NAS AND =
cân ti
( )
A AD AN=
AR đường phân giác
( )
DAS NAS=
cũng đường
cao,
ADS
vuông ti D, DR là đường cao
2
.AD AR AS=
nên
( )
2
..AR AS AB AC AD==
+ Xét
ACR
ASB
( )
,
AC AR
CAR chung
AS AB
=
(vì
..AR AS AB AC=
) do đó
( )
..ACR ASB c g c
ACR ASB =
T giác CBSR ni tiếp
L
thuộc đường trung trc ca
RS (1)
Chứng minh tương tự có t giác RSFE ni tiếp
T
thuc đường trung trưc của RS (2).
Mt khác
RDS
vuông ti R , RJ là đường trung tuyến
RJ SJ DJ J = =
thuộc đường trung trc ca RS (3).
T (1),(2) và (3) có J, L, T thng hàng.
Ví d 3.
Cho điểm A nằm ngoài đường tròn (O;R). V các tiếp tuyến AB, AC của đường tròn
( ) ( )
( )
, , ,O B C O
v cát tuyến ADE của đường tròn
( ) ( )
( )
,O D E O
AD AE
tia AD nm
gia hai tia AO, AB). Gi F điểm đối xng ca D qua AO, H giao đim ca EF BC. Chng
minh rng A, O, H thng hàng.
Gii:
Vì tiếp tuyến AB, AC của đường tròn
( )
O AB AC=
AO là tia phân giác
ca
BAC
. Do D,F đối xng vi AO
;OF OD R DF OA = =
Ta
//DF BC
T giác DBCF là hình thang.
Hình thang DBCF ni tiếp đường tròn (O)
t giác DBCF là hình thang cân
,DB CF BF CD = =
.
+ Xét
ABD
AEB
( )
,ABD AEB BAD chung=
Do đó
BD AB
ABD AEB
BE AE
=
. Tương tự
AC CD
ACD AEC
AE CE
=
+ Xét
HBF
HEC
,BHF EHC BFH HCE==
. Do đó
HBF
HEC
,BHF EHC BFH HEC==
.
Do đó
HB BF
HBF HEC
HE CE
=
. Tương tự
.
CF HC
HFC HBE
BE HE
=
Ta có:
HB BF CD AC AB BD CF HC
HB HC
HE CE CE AE AE BE BE HE
= = = = = = = =
, , , ,HB HC AB AC OB OC H A O= = =
cùng thuộc đường trung trc của đoạn thng BC.
CHUYÊN ĐỀ BỒI DƯỠNG HC SINH GII HÌNH HC 9
THCS.TOANMATH.com | 22
Vy A, O, H thng hàng.
Ví d 4.
T A ngoài đường tròn (O) v tiếp tuyến AB v BH vuông góc vi OA ti H, v cát tuyến ADE.
Đưng thng qua B vuông góc vi OE cắt đường tròn (O) I. Đường thng qua D song song vi
BE ct BH G đường thng qua D song song vi EI ct AI K. V hình thoi
( )
KMGN DKG MKG
chng minh rằng ba điểm M, N, D thng hàng.
Gii:
Xét
ABD
AEB
( )
,BAD chung
ABD AEB=
. Do đó
( )
.ABD AEB g g
2
.
AB AD
AB AD AE
AE AB
= =
.
ABO
vuông ti B, BH là đường cao
2
.AB AH AO=
. Xét
ADH
AOE
( )
,
AD AH
DAH chung
AO AE
=
(vì
2
..AD AE AH AO R==
).
Do đó
( )
..ADH AOE c g c
AHD AOE =
t giác DHOE ni tiếp do đó
ODE OHE=
ODE OED=
(vì
OD OE=
) nên
AHD OHE=
. Gi S giao điểm ca BH AE ; HS, HA các
đường phân giác trong ngoài ca tam giác
AD SD
HDE
AE SE
=
.
SBE
/ / ,
SD DG
DG BE AEI
SE BE
=
//
AD DK
DK IE OE BI BE IE BE IE
AE IE
= = =
. Do đó
DK DG=
.
,MK MG NK NG==
(t giác KMGN hình thoi). Ta D, M, N cùng thuộc đường trung
trc của đoạn thng KG. Vy M, N, D thng hàng.
Ví d 5.
Cho đường tròn (O) đường kính AB. Trên tiếp tuyến ti A của đường tròn (O) ly điểm C. V cát
tuyến CDE (tia CD nm gia hai tia CA, CO ; D, E thuộc đường tròn (O), D nm gia CE ). Gi
M giao điểm ca CO BD. Gi F giao điểm ca AM đường tròn (O) (F khác A). Chng
minh rng E, O, F thng hàng.
Gii:
V CN là tiếp tuyến của đường tròn (O) (N khác A).
Ta có
,CA CN CO=
là tia phân giác góc
CAN
CHUYÊN ĐỀ BỒI DƯỠNG HỌC SINH GIỎI HÌNH HỌC 9
23 | THCS.TOANMATH.com
MAC MNC=
(tính cht trục đối xng ) mà
90ANB =
(góc ni tiếp chn nửa đường tròn)
Ta có:
, / / .CO AN BN AN CO BN CMD NBD =
NBD CND=
(h qu góc to bi tiếp tuyến và dây cung)
Do đó
( )
CMD CND NBD==
t giác CMND ni tiếp
BDE MNC=
.
Ta có:
( )
,BDE MAC MNC BDE BAE= = =
nên
MAC BAE=
.
Ta
90FAE BAF BAE BAF MAC EF= + = + =
đường kính của đường tròn (O)
Vậy ba điểm A, E, F thng hang.
Tiêu chun 5: S dụng các điểm ph:
+ Để chứng minh 3 đim A, B, C thng hàng, nếu ta xác định được điểm D khác A, B, C 2 trong
3 b ba điểm A, B, D hoc A, C, D hoc B, C , D thng hàng thì suy ra A, B, C, D thng hàng, dn
đến A, B, C thng hàng.
+ Một cách khác ta cũng thường xuyên dùng đó là: Giả s điểm C thuc hình (H). Gi s AB ct
hình (H) ti
C
. Chng minh
C
các tính chất như đim C t đó suy ra
CC
dẫn đến A, B, C
thng hàng.
Ví d 1.
Cho điểm A nằm ngoài đường tròn (O;R). V các tiếp tuyến AB, AC của đường tròn
( ) ( )
( )
,,O B C O
, v cát tuyến ADE của đường tròn
( ) ( )
( )
,O D E O
AD AE
tia AD nm
gia hai tia AO, AB). Gi F điểm đối xng ca D qua AO, H là giao điểm ca EF BC. Chng
minh rng A, O, H thng hàng.
Gii:
+ Gi
H
giao đim ca AO BC, D, F đối xng qua
OA OF OD R F = =
thuộc đường
tròn (O) và có
AH D AH F

=
AB, AC là các tiếp tuyến ca (O) nên
AO BC
.
+ Xét
ABD
AEB
có:
ABD AEB=
(h qu góc to bi tia tiếp tuyến và dây cung),
BAD
chung do đó
( )
.ABD AEB g g
2
.,
AB AD
AB AD AE ABO
AE AB
= =
vuông
ti
,B BH
là đường cao
2
.AB AH AO
=
do đó
( )
2
..AD AE AH AO AB
==
.
CHUYÊN ĐỀ BỒI DƯỠNG HC SINH GII HÌNH HC 9
THCS.TOANMATH.com | 24
+ Xét
ADH
AOE
( )
,
AD AH
DAH chung
AO AE
=
(vì
..AD AE AH AO=
)
do đó
( )
..ADH AOE c g c ADH AOE

=
t giác
DH OE
ni tiếp
,AH D OED OH E ODE

= =
.
OD OE R ODE= =
cân ti
O OED ODE=
do đó
AH D OH E

=
vì vy
AH F OH E

=
. Ta có
180EH F AH F AH E OH E AH E
= + = = =
,,E H F
thng hàng. Vy
HH
. Do đó A, H, O thng hàng.
Ví d 2.
Cho điểm A nằm ngoài đường tròn (O;R). V các tiếp tuyến AB, AC của đường tròn
( ) ( )
( )
,,O B C O
, v cát tuyến AEF của đường tròn
( ) ( )
( )
,O E F O
AE AF
tia AE nm
gia hai tia AO, AB). V đường thng qua E vuông góc vi OB ct BC, BF lần t ti M, N. Gi I
là trung điểm ca AB. Chng minh rằng ba điểm F, M, I thng hàng.
Gii:
+ V
OK EF
ti
KK
trung điểm
ca
, 90 , , , ,EF ABO AKO ACO A B K O C= = =
cùng thuc một đường tròn
BAK MCK=
. Mà
,AB OB EN OB⊥⊥
/ / .AB EN BAK MEK =
+ Ta có
( )
MCK MEK BAK= =
t giác
EMKC ni tiếp
ECM EKM=
ECM EFB=
nên
/ / ,EKM EFB MK BN=
EFN
/ / , .KM NF EK KF EM MN= =
Gi
I
là giao điểm ca FM và
,.
EM MN FM
AB AI I B
AI I B FI


= = =


Vy
II
do đó F,M,I thng hàng.
Ví d 3.
Cho đường tròn (O) AB CD các đường kính. Tiếp tuyến ca đường tròn (O) ti B ct AC
E. DE cắt đường tròn (O) F (F khác D). Gi M giao điểm ca AF BE, N giao điểm ca
AMBC. Chng minh rằng ba điểm E, N, O thng hàng.
Gii:
Ta có:
90 , 90CFD AFB= =
(góc ni tiếp chn nửa đường tròn)
90 ,CFE BMF ABF = =
ECF ABF=
(t giác ABFC ni tiếp)
CHUYÊN ĐỀ BỒI DƯỠNG HỌC SINH GIỎI HÌNH HỌC 9
25 | THCS.TOANMATH.com
nên
( )
BMF ECF ABF==
T giác CEMF ni tiếp
90 .CME CFE = =
Ta có:
90 / / .CME ABE CM AB= =
+ Gi I là giao điểm ca ONCM,
K là giao điểm ca OECM. Do
//CM AB
. Áp dng h qu của định lý Talet.
Ta có
CI NI IM
CI CM
OB NO OA
= = =
CK EK KM
CK KM
OA EO OB
= = =
. Do vy
.IK
Vy ba điểm E, N, O thng hàng.
Chú ý rng: Bài toán này thc cht là b đề hình thang.
Ví d 4.
Cho tam giác nhn ABC (AB, AC) ni tiếp đường tròn (O) các đường cao BD, CE. Gi M giao
điểm ca các tiếp tuyến v t B, C của đường tròn, N trung điểm của đoạn thng DE. Chng
minh rng A, M, N thng hàng.
Gii:
+ Qua M v đường thng song song vi DE ct
AB, AC lần lượt K, S. Gi Cx là tia đối ca tia CM.
+ Ta có
90BEC BDC= =
T giác BEDC ni tiếp
,ADE ABC AED ACB = =
. Mà
,ADE MSC=
AED BKM=
ng v
//DE KS
). Ta có
ACx ABC=
(h qu góc to bi tia tiếp tuyến và dây cung)
ACx MCS=
ối đỉnh) do đó
MCS MSC MCS=
cân ti
M MC MS=
. Tương tự
MB MK=
MB MC=
do đó
MK MS=
. Gi
M
là giao điểm ca
ANKS,
AKM
/ / ,
EN AN
EN KM
KM AM
=

AM S
//
ND AN
ND M S
SM AM
=

nên
EN ND AN
KM SM AM

==


. Mà
.EN ND=
Do đó
KM SM

=
.Ta có
MM
.Vậy ba điểm A, M, N thng hàng.
Ví d 3.
Cho tam giác
( )
ABC AB AC
ni tiếp đường tròn (O) ,I tâm đường tròn ni tiếp tam giác ABC.
Đưng thng AO, AI ct đường tròn (O) lần lượt ti D, E, DI cắt đường tròn (O) M. V IK vuông
góc vi BC ti K. Chng minh rng M, K, E thng hàng.
Gii:
CHUYÊN ĐỀ BỒI DƯỠNG HC SINH GII HÌNH HC 9
THCS.TOANMATH.com | 26
Gi s: ED ct BC N , ME ct BC
T,
EIB BAI ABI CBE IBC EBI= + = + =
EBI
cân ti
E EB EI=
EBT EMB
(
BET
chung,
EBT EMB=
)
;
EB ET EI ET
EM EB EM EI
= =
EIT EMI EIT EMI =
2 2 2
sdEB sdCD sdEC sdCD sdED
ENT
−−
= = =
EMI EIT EITN= =
ni tiếp
90 ,ITN IEN IT BC = =
do đó
TK
. Vy M, K, E thng
hàng.
Tiêu chun 6. S dng tính cht ca các phép biến hình như đối xng trục, đối xng tâm , quay, v
tự… biến một đường thng thành một đường thng. T đó nếu tìm được mt trong các phép biến
hình đó biến ba điểm thng hàng
1 1 1
,,A B C
thành ba điểm A, B, C thì ta ba điểm A, B, C thng
hàng.
Ví d 1.
T giác ABCD
90BD= =
. K
AH BD
. Ly K thuc BD sao cho
BH DK=
. Dng hình
bình hành ABED. Chng minh rng E, K, C cùng nm trên một đường thẳng đường thng này
vuông góc vi BD.
Gii:
Gi M là trc tâm
ABD
. D thy t giác
BMDC là hình bình hành (vì các cp cnh
đối tương ứng song song vi nhau). Gi N
là giao điểm ca BD, MC suy ra N là trung điểm
ca BD, MC. Ta thy phép đối xng tâm
N biến A thành E, M thành CH thành
K (theo gi thiết). Mt khác A, M, H cùng nm
trên một đường thẳng và đường thng này vuông góc vi BD, t đó ta có đpcm.
Ví d 2.
Cho tam giác ABC ni tiếp đường tròn (O), M điểm bt k thuộc đường tròn. Gi N, P, Q theo
th t là các điểm đối xng vi M qua AB, BC, CA. Chng minh rng N, P, Q thng hàng.
Gii:
Gi H, I, K theo th t là hình chiếu ca M lên AB, BC, CA
thế thì H, I, K thng hàng (đường thng Simson).
CHUYÊN ĐỀ BỒI DƯỠNG HỌC SINH GIỎI HÌNH HỌC 9
27 | THCS.TOANMATH.com
Ta thy rng: Phép v t tâm M t s bng
2 biến các điểm H, I, K thành N, P, Q.
H, I, K thng hàng nên suy ra N, P, Q thng hàng.
Đưng thẳng đi qua N, P, Q được gọi là đường thng Steiner của điểm M.
Ví d 3.
Cho tam giác ABC H, G ,O lần lượt trc tâm, trọng tâm, tâm đường tròn ngoi tiếp tam giác,
AH cắt đường tròn (O) ti
1
A
. Ly
2
A
điểm đối xng ca G qua BC. V hình bình hành
3
AHAO
.
Chng minh
1 2 3
,,A A A
thng hàng.
Gii:
Ta có các kết qu quen thuc
(Xem thêm phần các định lý hình hc ni tiếng),
1
A
đối xng vi H qua BC
/ / 2AH OM=
(M
trung điểm BC).
Theo gi thiết
3
AHAO
là hình bình hành
nên
3
A
đối xng vi O qua đường thng BC.
Vậy phép đối xng trc BC biến H thành
1
;AG
thành
2
;AO
thành
3
A
.
Mt khác do tính cht của đường thẳng Ơ-le thì H, G, O
thng hàng. T đó ta có
1 2 3
,,A A A
thng hàng.
II. MT S BÀI TOÁN LIÊN QUAN ĐẾN ĐƯỜNG, ĐIỂM C ĐỊNH
Bài 1.
T điểm M thuộc đường thng (d) ngoài đường tròn (O;R) sao cho khong cách t điểm O đến (d)
bng h không đổi ta k 2 tiếp tuyến MA, MB đến (O) (Vi A, B là các tiếp điểm).
a, AB luôn đi qua điểm I c định
b, Gi K giao điểm ca OM, AB. Chng minh khi M di chuyn trên (d) thì K thuộc đường c
định.
Gii
a, Dng
( )
OH d
thì
OH h=
. Gi I
giao điểm ca OHAB thì ta có:
.
.
OI OK OK OM
OKI OHM OI
OM OH OH
= =
Li có
22
.OK OM OA R==
CHUYÊN ĐỀ BỒI DƯỠNG HC SINH GII HÌNH HC 9
THCS.TOANMATH.com | 28
Suy ra
2
R
OI
h
=
. Như vậy điểm I luôn c định.
b, Theo câu a) thì OI c định, mt khác
90IKO =
suy ra điểm K thuộc đường tròn đường kính OI.
Bài 2.
Cho tam giác ABC điểm M chuyển động trong đon BC. Lấy hai điểm N, P trên AC, AB đ
ANMP là hình bình hành. Khi đó đường tròn ngoi tiếp tam giác AMN luôn đi qua điểm c định.
Gii
Dng
( )
1
O
qua B tiếp xúc vi AC ti A .
Dng
( )
2
O
qua B tiếp xúc vi AB ti A.
Gi s
( )
1
O
ct nhau tại giao điểm th hai là K thì điểm K
là điểm c định.
Ta có:
( )
,1KAB KCA KCA KBA==
suy ra
( )
2
KA AB
KAB KCA
KB AC
=
ANMP là hình bình hành nên theo định lý Thales ta có:
( )
3
BP BM AN BP AN BC AN BP AN BP AB
PA MC NC PA NC PA BP AN NC AB AC AN AC
= = = = = =
++
. T
(1),(2),(3) suy ra
KBP KAB BPK ANK ANPK =
ni tiếp suy ra đưng tròn ngoi tiếp
tam giác AMN luôn đi qua điểm c định K.
Bài 3.
T đim M ngoài đường tròn (O;R) ta k 2 tiếp tuyến MA, MB đến (O) (Vi A,B là các tiếp điểm)
cát tuyến MCD đến (O) sao cho
MC MD
tia MC nm gia hai tia MO, MA. Gi s M c
định, khi cát tuyến MCD thay đổi thì trng tâm G ca tam giác BCD luôn nằm trên đường tròn c
định.
Gii
Gi I là trung điểm ca CD, ta có
2
3
BG
BI
=
Qua G k các đường thng song song vi
IO, MD ct OB, MB lần lượt ti Q, R
Ta có các điểm B, O, M c định, suy ra
P, Q c định và
90PGQ =
. Suy ra G thuc
đường tròn đường kính
2
3
PQ MO=
.
Bài 4.
CHUYÊN ĐỀ BỒI DƯỠNG HỌC SINH GIỎI HÌNH HỌC 9
29 | THCS.TOANMATH.com
Cho 3 đim A, B, C thng hàng theo th t đó. Đưng tròn (O) thay đổi luôn đi qua B, C. V đường
kính MN vuông góc vi BC ti H. Tia AN ct (O) tại giao điểm th 2 D. Khi đó MD luôn đi qua
một điểm c định nm trên BC.
Gii
Ta có:
. . .ADK AHN AK AH AD AN =
Mt khác ta cũng dễ chứng minh được:
..AB AC AD AN=
. T đó suy ra
..AB AC AK AH=
Hay
.AB AC
AK
AH
=
không đổi. Suy ra điểm K c định.
Hay MD đi qua điểm c định K.
Bài 5.
Cho tam giác ABC ni tiếp (O). Trên cnh BC lấy điểm M. Dng
( )
1
O
qua M tiếp xúc vi AB ti A.
Dng
( )
2
O
qua M tiếp xúc vi AC ti A. Hai đường tròn này ct nhau tại giao đim th 2 N. Khi
đó:
a. Đim N nm trên (O).
b. Đưng thng MN luôn đi qua một điểm c định khi M di chuyn trên cnh BC.
Gii
a, Ta có:
,NBM ABC MNC ACB==
. T giác ABNC
180BAC BNC BAC ABC ACB+ = + + =
.
Suy ra t giác ABNC ni tiếp. Nói cách khác
điểm N thuộc đường tròn (O) c định.
b, Ta có:
DAC DNC=
(cùng chn cung DC).
Mặt khác ta cũng có:
DNC ACB=
suy ra
DAC ACB=
. Suy ra
/ / ,AD BC
do đó điểm D là điểm c định.
Vậy đường thng MN luôn đi qua điểm c định là D.
Bài 6.
Cho tam giác ABC 3 góc nhn ni tiếp (O;R). Dựng đường cao AD của tam giác đường kính
AK ca (O). H BE, CF lần lượt vuông góc vi AK. Cho BC c định, điểm A di chuyn trên cung
ln BC. Chứng minh: Tâm đường tròn ngoi tiếp tam giác DEF là một điểm c định.
Gii
90ABD AEB= =
suy ra 4 điểm A, B, D, E nằm trên đường tròn
đường kính AB có tâm là trung điểm N ca AB.
90ADC AFC= =
nên 4 điểm A, D, F, C nằm trên đường tròn
CHUYÊN ĐỀ BỒI DƯỠNG HC SINH GII HÌNH HC 9
THCS.TOANMATH.com | 30
đường kính AC có tâm là trung điểm P ca AC.
Gi M là trung điểm ca BC, I là trung điểm ca BO thì
,ON AB OM BC⊥⊥
suy ra 5 điểm N, O, E, M,B nằm trên đường tròn
đường kính BO.
Ta có:
1
2
MNE MBE DBE DAE DNE= = = =
suy ra MN phân giác ca góc
DNE
.Tam giác
DNE cân ti N suy ra MN cũng là trung trực ca DE, tương t ta cũng MP trung trc ca DF.
Suy ra M tâm đường tròn ngoi tiếp tam giác DEF. Vậy tâm đường tròn ngoi tiếp tam giác DEF
là điểm M c định.
Bài 7.
Cho đường tròn (O) đưng kính AB c định, M một điểm thuc (O) (M khác A, B). Các tiếp
tuyến ca (O) ti A và M ct nhau C. Đường tròn (I) đi qua M và tiếp xúc với đường thng AC ti
C. CD là đường kính ca (I).
a, Chng minh: O, M, D thng hàng.
b, Chng minh: Tam giác COD cân
c, Đưng thẳng đi qua D vuông góc vi BC luôn đi qua một điểm c định khi M di động trên
đường tròn (O).
Phân tích, định hướng gii
a. Ta có MC là tiếp tuyến của đường tròn (O)
( )
1MC MO⊥
Xét đường tròn (I): Ta có
90CMD =
( )
2MC MD⊥
. T (1) và (2)
//MO MD MO
MD trùng nhau
,,O M D
thng hàng.
b. CA là tiếp tuyếm của đường tròn (O)
( )
3CA AB⊥
. Đưng tròn (I) tiếp xúc vi AC ti
( )
4.C CA CD⊥
T (3) và
( ) ( )
4 / / *CD AB DCO COA =
(hai góc so le trong)
CA, CM là hai tiếp tuyến ct nhau ca
( ) ( )
**O COA COD=
.
T (*) và
( )
** DOC DCO COD =
cân ti D.
CHUYÊN ĐỀ BỒI DƯỠNG HỌC SINH GIỎI HÌNH HỌC 9
31 | THCS.TOANMATH.com
**Thc nghim hình v cho thy đường thẳng đi qua D vuông góc vi BC luôn đi qua trung
điểm K ca AO. Ta chứng minh điều này như sau:
c. Gọi chân đường vuông góc h t D ti BC H, N giao điểm ca CO (I). Ta
( )
90CHD H I=
(bài toán qu tích)
90DH AB K CND = =
COD
cân ti
,D NC NO=
. Ta có t giác NHOK ni tiếp.
+
21
H O DCO==
(cùng vi
DHN
)
( )
180 5NHO NKO + =
. Ta
NDH NCH=
(cùng
chn cung NH của đường tròn (I))
( )
.
HN OB
gg
HD OC
=
. Tương t ta có:
;
OB OA OA CN ON HN ON
OC OC OC CD CD HD CD
= = = =
. Mà
( )
..ONH CDH NHO DHC c g c=
90NHO =
( )
180 5 90 / /NHO NKO NKO NK AB NK AC+ = =
K
trung điểm ca OA c định
đpcm.
Bài 8.
Cho đường tròn (O;R) và một đường thng d ct (O) ti C,D. Một điểm M di động trên tia đối ca
tia DC. Qua M k hai tiếp tuyến MA, MB với đường tròn (O) (A, B tiếp điểm ). Chng minh
đường thng AB luôn đi qua một điểm c định.
Gii
Gi H là trung điểm ca CD và giao điểm ca AB
vi OM, OH lần lượt là E, F .Tam giác OBM vuông
ti B, đường cao BE. Suy ra
( )
22
.1OE OM OB R==
.
Do
90FHM FEM= =
nên t giác MEHF ni tiếp.
T hai tam giác vuông OMH, OEF đồng dng, suy ra
( )
.
2
OH OM OE OM
OF
OE OF OH
= =
. T (1) và (2) suy ra
2
R
OF
OH
=
. Do đường tròn (O), đường thng d cho
trước nên OH không đổi. Suy ra OF không đổi, điểm F c định. Do đó đưng thng AB đi qua điểm
F c định.
i 9.
Cho đường tròn tâm (O). T điểm A c định ngoài (O) k tiếp tuyến AB, AC ti (O) (B, C tiếp
điểm). Lấy điểm M trên cung nh BC. Gi D, E, F th t hình chiếu t M đến BC, AC, AB. Gi
MB ct DF ti P, MC ct DE ti Q. Chứng minh đưng thng ni giao đim của hai đường tròn
ngoi tiếp tam giác MPFMQE luôn đi qua một điểm c định.
CHUYÊN ĐỀ BỒI DƯỠNG HC SINH GII HÌNH HC 9
THCS.TOANMATH.com | 32
Gii
Gi s đường tròn ngoi tiếp tam giác MPF MQE ct nhau ti M, N. Đưng thng MN ct PQ,
BC theo th t ti KI. Ta có các t giác MDCE, MDBF ni tiếp nên
;MCE MDE MBC MBF MDF MCB= = = =
. Suy ra
PMQ PDQ PMQ PDM QDM+ = + +
180PMQ MCB MBC= + + =
. Do đó tứ giác MPDQ là t giác ni tiếp.
Suy ra
,MQP MCB MEQ==
suy ra KQ là tiếp tuyến của đường tròn
ngoi tiếp
MQE
. Tương t KP là tiếp
tuyến của đường tròn ngoi tiếp tam giác
MFP. Ta có
2
.,KM KN KQ=
2
.KM KN KP=
. Suy ra
KP KQ=
. Xét
tam giác
, / / ,MBC PQ BC KP KQ=
.
Theo định lý Thales suy ra I là trung điểm BC. Vy MN đi qua điểm c định I là trung điểm BC
Bài 10.
Cho đường tròn (O) y cung AB. Lấy đim E trên y cung AB (E khác A B). Qua E v dây
cung CD ca (O). Trên hai tia DA, DB lấy hai điểm P, Q đối xng qua E. Chng minh rằng đường
tròn (I) đi qua C tiếp xúc vi PQ ti E luôn đi qua một điểm c định khi E di động trên dây cung AB
Gii
Gi M là giao điểm ca AB và đường tròn (I)
EP là tiếp tuyến ca (I) nên
CMA PEC QED==
.
Mt khác
BAC BDC=
. Suy ra
( )
.CMA QED g g
( )
1
AM DE
CM QE
=
. Tương tự
;DEP BMC ADC ABC==
nên
( ) ( )
. 2 .
BM DE DE
BMC DEP g g
CM PE QE
= =
T (1) và (2) suy ra
AM BM
AM BM
CM CM
= =
.
Do đó đường tròn (I) luôn đi qua trung điểm M ca AB là điểm c định.
Bài 11.
Cho đường tròn (O) đưng kính BC. Gi A đim chính gia cung BC, điểm M thuộc đon BC.
K ME, MF lần lượt vuông góc vi AB, AC, MN vuông góc vi EF ti N.
a. Khi M di chuyn trên BC, hãy chng minh: MN luôn đi qua một đim c định.
CHUYÊN ĐỀ BỒI DƯỠNG HỌC SINH GIỎI HÌNH HỌC 9
33 | THCS.TOANMATH.com
b. Lấy điểm D thuc đoạn OA, tia BD cắt đường tròn tại giao điểm th 2 P. Chng minh:
Khi D di chuyn trên OA thì tâm I của đường tròn ngoi tiếp tam giác APD luôn thuộc đường thng
c định.
Gii
a, Dựng đường kính AK ca (O). Ta chng
minh MN đi qua K. Gi s FM ct BK ti J,
EM ct KC ti H. Do ABKC là hình vuông
nên BEMJ là hình vuông. Suy ra
,EMF KHM =
gi s KM ct EF ti
N
thì
JMK N MF
=
li có:
MFE HMK=
suy ra
90NMF MFE JMK HMK
+ = + =
hay
KM EF
ti
N
suy ra
NN
tc là:
N,M,K thng hàng. Suy ra MN luôn đi qua điểm K c định.
b, Ta
45 ,APB ACB= =
1
2
APB AID=
suy ra
90 ,AID =
hơn na ta
45DAI ADI= =
nên suy ra I nm trên BC c định.
Bài 12.
Cho đường tròn (O;R) một đường thng d nằm ngoài đường tròn. I một điểm di động trên d.
Đường tròn đường kính IO cắt đường tròn (O;R) tại hai đim M,N. Chứng minh đường thng MN
luôn đi qua một điểm c định.
Gii:
Đường tròn đường kính IO, cắt đường tròn (O;R)
ti
, 90 ,M N IMO INO IM IN = =
là tiếp
tuyến của đường tròn (O;R). Gi E là giao điểm
ca IOMN thì IE cũng là đường cao ca
tam giác cân
MIN IE MN⊥
. Xét
OMI
ta có
90 ;OMI ME OI=
nên
2
.OM OE OI=
.
K
OA d
ti A, gi K giao điểm ca OA MN thì
( )
.OEK OAI g g
( )
. . 2
OE OK
OAOK OE OI
OA OI
= =
. T (1) (2) suy ra
22
.OK OA OM R==
. T đó suy ra
2
R
OK
OA
=
OA không đổi, do đó K là điểm c định phi tìm.
Bài 13.
Cho ba điểm c định A,B,C thng hàng theo t t đó. Một đường tròn (O) thay đổi luôn qua BC.
CHUYÊN ĐỀ BỒI DƯỠNG HC SINH GII HÌNH HC 9
THCS.TOANMATH.com | 34
T điểm A k các tiếp tuyến AM, AN đến đường tròn. Đưng thng MN ct AO AC lần lượt ti H
K
a, Chng minh M, N di động trên một đường tròn c định.
b, Gi I là trung điểm ca BC. NI cắt đường tròn (O) ti P. Chng minh
//MP BC
c, Chng minh đường tròn ngoi tiếp tam giác OHK luôn đi qua hai điểm c định.
Gii:
a, Vì AN là tiếp tuyến của đường tròn
(O) nên
ANB BCN=
CAN
chung nên
( )
.ANB ACN g g
suy ra
AN AB
AC AN
=
2
.AN AB AC=
AM AN=
(tính cht
tiếp tuyến). Suy ra M, N nằm trên đường tròn
tâm A bán kính
c định.
b,
90 ; 90ANO AIO= =
nên t giác AOIN ni tiếp đường tròn đường kính AO, suy ra
AON AIN=
. Mt khác ,
11
;
22
AON MON MPN MON==
nên
AIN MPN=
suy ra
//MP BC
.
c, Theo câu a), ta
( )
2
.1AN AB AC=
. Mt khác
ANO
vuông ti N
NH AO
nên
( )
2
.2AN AH AO=
.
AHK AIO
(vì
OAI
chung,
90AHK AIO= =
)
( )
. . 3
AH AK
AH AO AI AK
AI AO
= =
. T (1),(2),(3) suy ra
.
..
AB AC
AB AC AI AK AK
AI
= =
AB, AC, AI c định
AK
không đổi
K
c định. T giác OIKH ni tiếp nên đường tròn
ngoi tiếp
OHK
luôn đi qua IK c định.
Bài 14.
Cho tam giác ABC vuông ti C
. Gi I đim trên AB
IB IA
. K đường thng d
đi qua I vuông góc vi AB. Gọi giao điểm ca d vi AC, BC lần lượt F E. Gọi M đim
đối xng vi B qua I.
a, Chng minh rng
IME
đồng dng vi
IFA
..IE IF IAIB=
.
b, Đường tròn ngoi tiếp tam giác CEF ct AE ti N. Chng minh rng E, N, B thng hàng.
c, Cho AB c định, C thay đổi sao cho
90BCA=
. Chng minh rằng đường tròn ngoi tiếp
AEF
luôn đi qua hai điểm c định và tâm đường tròn này trên đường thng c định.
Gii:
a, Ta có IE là đường trung trc ca BM
EBM
cân ti
M EBM EMB=
.
CHUYÊN ĐỀ BỒI DƯỠNG HỌC SINH GIỎI HÌNH HỌC 9
35 | THCS.TOANMATH.com
EBM EFA=
(cùng ph vi
FAB
)
( )
.
IM IE
IME IFA g g
IF IA
=
. . . .IE IF IAIM IE IF IAIB = =
.
b, Ta
90 90ECF ENF= =
. Xét
BAE
EI, AC các đường cao ct nhau ti F nên
BF EA
,,FN EA B F N⊥
thng hàng.
c, Ta
EMB EFA=
suy ra t giác AMFE ni tiếp. T đó suy ra đường tròn ngoi tiếp
AEF
luôn đi qua hai đim A, M c định. Vy tâm đường tròn ngoi tiếp
AEF
luôn nằm trên đường
trung trc ca AM c định.
Bài 15.
Cho đường tròn (O;R) và điểm A c định vi
2OA R=
. Một đường kính BC quay quanh O sao cho
ba điểm A, B, C không thẳng hàng. Đường tròn ngoi tiếp tam giác ABC cắt đường thng OA ti
điểm th hai I. Đường thng AB, AC cắt đường tròn (O;R) lần lượt ti D E. Ni DE cắt đường
thng OA ti K.
a, Chng minh rng
..OI OA OBOC=
..AK AI AE AC=
.
b, Tính độ dài đoạn thng OIAK theo R.
c, Chng minh rằng đường tròn ngoi tiếp tam giác ADE luôn đi qua một điểm c định khác A khi
BC quay quanh O.
Gii:
a,
( )
.
OI OC
OIC OBA g g
OB OA
=
..OBOC OI OA=
. Xét các góc ni tiếp
ta có:
;ICB IAB BCE EDB==
ICB BCE IAB EDB ICE EKA + = + =
( ) ( )
. . . 1 .AKE ACI g g AK AI AE AC =
b, T
. . . .2OBOC OI OA R R OI R= =
1
2
OI R=
.Gi M, N là giao điểm ca
đường thng AO với đường tròn (O)
(M nm gia A
( ) ( )
. . . 2N AME ACN AME ACN g g AM AN AE AC = =
. T (1)
(2) suy ra
56
. . . .3
25
RR
AK AI AM AN AK R R AK= = =
.
c, Gi H là giao điểm th hai của đường tròn ngoi tiếp
ADE
vi AO. Ta có:
CHUYÊN ĐỀ BỒI DƯỠNG HC SINH GII HÌNH HC 9
THCS.TOANMATH.com | 36
( ) ( )
. . . , . . .KHE KDA g g KH KA KD KE KDN KME g g KN KM KD KE = =°∽∽
6 6 6 3
. . . . 3
5 5 5 10
R R R R
KH KA KM KN KH R R KH H
= = =
c định
điều phi chng minh.
Bài 16.
Cho đường tròn (O;R), mt dây cung CD trung điểm H. Trên tia đi ca tia DC ly một điểm S
qua S k các tiếp tuyến SA, SB với đường tròn. Đường thng AB cắt đường thng SO, OH ln
t ti E, F. Gi I là giao điểm ca ABCD.
a, Chng minh t giác SEHF ni tiếp được.
b, Chng minh
2
.OH OF R=
c, Chng minh
..SI SH SC SD=
.
d, Khi S di động trên tia đối ca tia DC, chứng minh đường thng AB luôn đi qua một điểm c định.
Gii
a, T giác SEHF ni tiếp vì HE
cùng nhìn FS i mt góc vuông.
b, Ta có
( )
.OHS OEF g g
suy
ra
..OH OF OE OS=
. S dng h thức lượng
trong tam giác vuông AOS ta có:
2
.OE OS OA=
nên
22
.OH OF OA R==
.
c, Chng minh các cặp tam giác đồng dng:
( ) ( ) ( )
22
. . , . . , .SAD SCA g g SA SC SD SAE SOA g g SA SE SO SEI SHO g g = = °
..SI SH SE SO=
.
T đó suy ra
..SI SH SC SD=
d, T đề bài O, C, D c định nên H c định.
T câu b) suy ra
2
R
OF
OH
=
không đổi. Vậy đường thng AB luôn đi qua điểm c định là F.
Bài 17.
Cho nửa đường tròn tâm O đường kính
2AB R=
. Gi C trung đim của đoạn thng AO. Đưng
thng Cx vuông góc với đường thng AB, Cx ct nửa đường tròn trên ti I. Gi K điểm bt k
nằm trên đoạn thng CI (K khác C I), tia AK ct nửa đường tròn tâm O tại điểm M, tiếp tuyến ti
M ca (O) ct Cx ti N. Gi D là giao điểm ca BM ct Cx.
a. Chng minh rằng 4 điểm A, C, M, D cùng nm trên một đường tròn.
b. Chng minh tam giác MNK cân.
c. Tính din tích
ABD
khi K là trung điểm của đoạn thng CI.
CHUYÊN ĐỀ BỒI DƯỠNG HỌC SINH GIỎI HÌNH HỌC 9
37 | THCS.TOANMATH.com
d. Chng minh rng khi K di động trên đoạn thng CI thì đường tròn ngoi tiếp tam giác AKD
đi qua một điểm c định khác A.
Gii:
a.
90 ; 90 , , ,ACD AMD A C M D= =
cùng thuộc đường tròn đường kính AD.
MN là tiếp tuyến ca (O)
NMK ABM=
(góc gia tiếp tuyến và
dây cung). Ta li có
NKM MBA=
(cùng ph
MAB
) suy ra
NKM NMK MNK=
cân ti N.
b. Ta có
COI
vuông ti
2
2 2 2 2
3 3 3
4 2 4
R R R
C IC CO OI IC IC CK + = = = =
.
Ta
CAK CDB
. Vì
90 ;ACD BCD KAC CDB= = =
(cùng ph
DBA
)
33
. . . . 3
4 2 2
CA CK R R R
CDCK CACB CD CD R
CD CB
= = = =
. Do đó diện tích
ABD
là:
2
11
. .2 . 3 3
22
S AB CD R R R= = =
.
c. Gi s đường tròn ngoi tiếp tam giác AKD ct BA ti E. Suy ra AKDE ni tiếp nên ta có:
AEK ADK KBA==
nên tam giác KEB cân ti K suy ra
CE CB=
. Như vậy E là điểm c định.
Hay đường tròn ngoi tiếp tam giác AKD luôn đi qua điểm c định E.
Bài 18.
Cho đường tròn (O) đim A khác O nằm trong đường tròn. Một đường thẳng thay đổi đi qua A
nhưng không đi qua O cắt đưng tròn tại hai đim M, N. Chng t rằng đường tròn ngoi tiếp tam
giác OMN luôn đi qua một điểm c định khác O.
Gii:
Gọi giao điểm của đường thng OA với đường
tròn (O) là
,,I K I K
c định.
Gọi giao điểm của đường thng OA với đường
tròn ngoi tiếp
OMN
P.
T
( )
.AMI AKN g g
suy ra
( )
. . 1AM AN AK AI=
.
( )
.AON AMP g g
suy ra
( )
. . 2AO AP AM AN=
.
CHUYÊN ĐỀ BỒI DƯỠNG HC SINH GII HÌNH HC 9
THCS.TOANMATH.com | 38
T (1) (2) ta có:
..AO AP AI AK=
hay
.AI AK
AP
AO
=
AI, AK, AO c định
AP
không đổi
P
là điểm c định.
Bài 19.
Cho điểm M bt k nm trên nửa đường tròn (O) đường kính
2,AB R=
qua điểm H c định trên
đoạn OB, v đường thng d vuông góc vi AB. Gọi giao điểm ca MA (d) D, giao điểm ca
tiếp tuyến ti M ca (O) vi (d) I (d) ct MB ti C. Gi E giao đim ca AC đường tròn
(O). Gi K là giao điểm OIME.
a. Chng minh rng IE là tiếp tuyến của đường tròn (O).
b. Cho M di chuyển trên đường tròn (M không trùng vi A;B). Chng minh rng tích OI.Ok
không đổi và ME luôn đi qua một điểm c định.
Gii:
a. Xét
ABD
;BM AD DH AB⊥⊥
C
trc tâm
AC DB⊥
AE BE
( )
90 , ,AEB B E D=
thng hàng.
T giác AMCH
90AMC AHC= =
AMCH ni tiếp đường tròn đường kính
AC MAH MCI=
(cùng bù vi
ABM
)
MI tiếp tuyến
IMC MAH IMC MCI MCI = =
cân ti
( )
1I IM IC=
Ta
90MDI DCM+ =
DCM IMC DMI MDI IMD= =
cân ti
( )
2I IM ID=
. T (1) và
( )
2 ID IC IM = =
.
CDE
vuông ti E
IC ID IC ID IE= = =
. T đó ta
;,IM IE OM OE OI==
cnh chung suy ra
( )
..OEI OMI c c c OEI OMI IE = =
tiếp
tuyến của đường tròn (O).
b. IM, IE tiếp tuyến ca
O IO ME⊥
ti K.
OMI
90 ;OMI MK OI=
nên
22
..OM OK OI OK OI R= =
không đổi. Gi S giao điểm ca ME AB. Xét
OKS
OHI
( )
90 ; chung . . .OHI OKS IOS OKS OHI g g OK OI OH OS= = =
2
2
.
R
OH OS R OS
OH
= =
OH c định
OS
không đổi
S
c định hay ME luôn đi qua
điểm S c định.
Bài 20.
CHUYÊN ĐỀ BỒI DƯỠNG HỌC SINH GIỎI HÌNH HỌC 9
39 | THCS.TOANMATH.com
Cho đường tròn (O;R) dây BC c định, A điểm chuyển đng trên cung ln BC ca
( )( )
,O A B C
, gi M trung điểm ca AC, H hình chiếu vuông góc ca M trên AB. Chng
minh khi điểm A thay đổi trên cung ln BC thì H nm trên một đường tròn c định.
Gii:
Ta có
OM AC
nên M nằm trên đường tròn
đường kính OC. Gi I là trung điểm ca OC
thì I c định. Gi s (I) ct CB ti N thì
90ONC =
nên suy ra N là trung điểm ca BC.
Suy ra MN là đường trung bình ca tam giác
ABC kết hp gi thiết
MH AB
suy ra
,NH HM
gi K là giao điểm ca (I) vi HM
thì
90NMK =
nên NK đường kính ca (I), N, I c định suy ra K c định, NK đường kính
ca (O) nên
90 hay 90 ,KCN KCB= =
t giác BCKH ni tiếp nên H nằm trên đường tròn đường
kính KB c định.
Bài 21.
Cho 3 điểm B, M, C c định theo th t nm trên một đường thng. Dng (O;R) qua B, C
( )
2,BC R
qua M k cát tuyến vuông góc vi OB ct (O) ti A, P. Chng minh: A,P thuc mt
đường tròn c định.
Gii:
Gi H là giao điểm ca AP vi OB thì H là trung
điểm ca AP, k đưng kính BD ca (O) thì
DA DP=
. Trong tam giác vuông BAD ta có:
2
.,BA BH BD=
li có t giác MHDC ni tiếp nên
..BH BD BM BC=
(h thc quen thuc, hs t chng minh)
T đó suy ra
2
..BM BC BA BA BM BC= =
mà điểm
B,M,C c định nên BM.BC không đổi và
BA BP=
suy ra
A, P nằm trên đường tròn tâm B bán kính bng
.BM BC
.
Bài 22.
Cho đường tròn (O) dây
2BC a=
không đổi
,a R A
điểm chuyn động trên cung ln BC
ca
( )( )
,,O A B C
các tiếp tuyến ca (O) ti B, C ct nhau M, gi D, E lần lượt hình chiếu
vuông góc ca M trên AB, AC. Chng minh rằng: Khi điểm A thay đổi trên cung ln BC ca (O) thì
trc tâm ca tam giác ADE luôn nm trên một đường tròn c định.
CHUYÊN ĐỀ BỒI DƯỠNG HC SINH GII HÌNH HC 9
THCS.TOANMATH.com | 40
Gii:
Gi H là trung điểm ca BC thì
OM BC
ti H. Theo gi thiết MD, ME lần lượt vuông
góc vi AB, AC nên MHCE là t giác ni tiếp
dn ti
HEM HCM=
, li có
BAC BCM=
suy ra
BAC HEM=
nên
90 90BAC HEA BAC HEM+ = + =
suy ra
EH AB
, tương tự
DH AC
suy ra
H là trc tâm ca tam giác ADC . Ta có
2
2 2 2
4
BC
OH OA R a= =
không đổi, O
c định suy ra điểm H nằm trên đường tròn
(
)
22
;O R a
.
Bài 23.
Cho đường tròn (O;R) và một điểm A c định nằm trên đường tròn. Trên tiếp tuyến với đường tròn
(O) ti A ly một điểm K c định, một đường thng (d) thay đổi đi qua K không đi qua O ct (O)
B,C (B nm gia K C). Gi M trung điểm ca BC. V đường kính AN ca (O). Đưng thng
qua A vuông góc vi CB ct MN ti H. Khi đường thng (d) thay đổi quanh K và thỏa mãn điều kin
đề bài thì điểm H di động trên đường nào.
Gii:
Do AN là đường kính ca (O) nên
O là trung điểm ca AN . Li có
AH, OM cùng vuông góc vi BC
nên
//AH MO
suy ra MO là đường
trung bình ca tam giác AHN suy ra
2AH OM=
. T giác BHMC có 2
đường chéo BC, HM ct nhau ti
trung điểm mỗi đường nên BHMC hình bình hành. Dn ti
/ / ,BH NC
NC AC
(do AN
đường kính ca (O) suy ra
,BH AC
kết hp vi gi thiết
AH BC
ta suy ra H trc tâm ca
tam giác ABC.
Gi I điểm đi xng vi O qua M , t chng minh câu c) ta suy ra AHIO là hình bình hành nên
//HI AO
, ta cũng có tam giác KOI cân ti K suy ra
KI KO=
không đi. Dng hình
bình hành HIKJ thì
//IK HI
,IK HI=
kết hp vi
//HI AO
HI AO=
thì
//KJ AO
ta suy
ra AOKJ hình bình hành A, O, K c định ta suy ra đim J c định. Do
JH KI KO==
không
đổi suy ra điểm H nằm trên đường tròn tâm J bán kính KO.
CHUYÊN ĐỀ BỒI DƯỠNG HỌC SINH GIỎI HÌNH HỌC 9
1
HH9-CHUYÊN ĐỀ 6.CC TR HÌNH VÀ ĐẲNG THC
Bài toán 1. S dụng định lí pythagore để chứng minh đẳng thc, bất đẳng thc
Định lý Pythagore là một định lý rất đẹp ca hình học sơ cấp th hin mi quan h v độ dài gia
các cnh ca mt tam giác vuông. Ta có th ng dụng định lý Pythagore vào vic chng minh các
quan h hình học, đặc bit là chứng minh các đẳng thc, bất đẳng thc hình hc.
I. KIN THC CN NH
1. Định lý Pythagore. Trong tam giác vuông, bình phương cạnh huyn
bng tổng bình phương hai cạnh góc vuông.
ABC
vuông ti A
2 2 2
BC AB AC = +
.
Chú ý: Nếu đặt
BC a=
;
AC b=
;
AB c=
thì ta có
2 2 2
a b c=+
.
2. Định lý Pythagore đảo
Nếu tam giác ABC có độ dài ba cnh tha mãn
2 2 2
BC AB AC=+
thì
tam giác ABC vuông tại đỉnh A.
3. Chú ý
Để vn dng có hiu qu định lý Pythagore, chúng ta cn trang b mt s kiến thức cơ bản sau:
a) Các đẳng thức được học trong đại s:
( )
2
22
2a b a ab b+ = + +
( )
2
22
2a b a ab b = +
( )( )
22
a b a b a b = +
b) Tính cht hình học: Hai đoạn thng song song chn giữa hai đường thng song song thì chúng
bng nhau.
c) Tính cht hình hc: Nếu
ABC
vuông ti A và
60B =
thì
2BC AC=
.
II. CÁC VÍ D
Ví d 1. Cho tam giác ABC vuông ti A. Gọi M là trung điểm ca AB. K MH vuông góc vi BC
( )
H BC
. Chng minh
2 2 2
CH BH AC−=
.
NG DN GII
Áp dụng định lý Pythagore vào các tam giác
vuông MCH và MBH ta được:
2 2 2
CH CM MH=−
( )
1
2 2 2
BH BM MH=−
( )
2
Tr
( )
1
cho
( )
2
:
CHUYÊN ĐỀ BỒI DƯỠNG HỌC SINH GIỎI HÌNH HỌC 9
2
2 2 2 2
CH BH CM BM =
Áp dụng định lý Pythagore vào tam giác vuông ACM và chú ý
AM BM=
ta được điều phi chng
minh.
Ví d 2. Cho tam giác ABC vuông ti A có
12AB cm=
;
18AC cm=
. Trên cnh AC lấy điểm M
sao cho
5AM cm=
. Chng minh rng:
2AMB C=
.
NG DN GII
Áp dụng định lý Pythagore vào ta
2 2 2 2 2
12 5 169BM AB AM= + = + =
.
13BM cm=
Mt khác
18AC cm=
;
5AM cm=
nên
13MC cm=
.
Vy tam giác BMC cân ti M.
T đó
MBC C=
.
Theo tính cht góc ngoài ca tam giác ta có
2AMB MBC C C= + =
Ví d 3. Cho tam giác ABC, D là điểm bt kì trong trong tam giác. Gi H, I, K lần lượt là hình
chiếu ca D lên BC, CA, AB. Chng minh rng:
2 2 2 2 2 2
BH CI AK CH AI BK+ + = + +
.
NG DN GII
Ni DA, DB, DC. Áp dụng định lý Pythagore vào các tam
giác vuông BDH và CDH ta được:
2 2 2 2 2
DH BD BH CD CH= =
.
Suy ra:
2 2 2 2
BH CH BD CD =
( )
1
.
Tương tự ta có:
2 2 2 2
CI AI CD AD =
( )
2
;
2 2 2 2
AK BK AD BD =
( )
3
.
Cộng các đẳng thc
( )
1
,
( )
2
( )
3
ta được:
2 2 2 2 2 2
0BH CH CI AI AK BK + + =
. T đó:
2 2 2 2 2 2
BH CI AK CH AI BK+ + = + +
.
Ví d 4. Cho tam giác ABC. Gọi M là trung điểm ca cnh BC. Chng minh rng
2 2 2
2
24
AB AC BC
AM
+
=−
.
( )
*
NG DN GII
K
( )
AH BC H BC⊥
.
CHUYÊN ĐỀ BỒI DƯỠNG HỌC SINH GIỎI HÌNH HỌC 9
3
Áp dụng định lý Pythagore vào các tam giác vuông ABH, ACH và AHM ta được:
2 2 2
AB BH AH=+
( )
1
2 2 2
AC CH AH=+
( )
2
Cng các vế của đẳng thc
( )
1
( )
2
:
( ) ( )
22
2 2 2 2 2 2
22AB AC BH CH AH BM HM BM HM AH+ = + + = + + +
22
2 2 2
2 2 2
22
BC BC
HM AH AM= + + = +
T đó:
2 2 2
2
24
AB AC BC
AM
+
=−
Chú ý: 1) H thc
( )
*
cho phép tính độ dài đường trung tuyến ca một tam giác thông qua độ dài
các cnh của tam giác đó. Người ta gi
( )
*
công thc trung tuyến.
2) Nếu tam giác ABC vuông tại A, khi đó AM là đường trung tuyến ng vi cnh huyền BC. Để ý
rng
2 2 2
AB AC BC+=
, thay vào h thc
( )
*
ta được:
2
2
4
BC
AM =
. T đó
1
2
AM BC=
.
Ta có tính cht quen thuc: Trong tam giác vuông, đường trung tuyến ng vi cnh huyn dài bng
na cnh huyn.
Ví d 5. Cho tam giác ABC cân a ti A, có
AB AC b==
BC a=
. K hai đường cao AH và BK.
Chng minh:
a)
2
2
4
a
AH b=−
; b)
4
2
2
4
a
BK a
b
=−
NG DN GII
a) Theo tính cht tam giác cân:
4
a
BH CH==
;
Áp dụng định lý Pythagore cho tam giác ABH vuông ti H:
2
2 2 2 2 2 2 2
4
a
AB AH BH AH AB BH b= + = =
Vy
2
2
4
a
AH b=−
b) Đặt
KC x AK b x= =
. Áp dụng định lý Pythagore cho hai tam giác
AKB và tam giác CKB ta có:
( )
2 2 2 2 2
BA AK BC KC BK = =
CHUYÊN ĐỀ BỒI DƯỠNG HỌC SINH GIỎI HÌNH HỌC 9
4
( )
2
2
2 2 2
2
a
b b x a x x
b
= =
.
Áp dụng định lý Pythagore cho tam giác BCK vuông ti K, ta có
4
2 2 2 2 2 2 2
2
4
a
BC BK KC BK BC CK a
b
= + = =
Vy
4
2
2
4
a
BK a
b
=−
Ví d 6. Chi hình v
2AB CD cm==
,
3DE cm=
,
1BC cm=
. Chng minh rng
32AE cm=
.
NG DN GII
T B k đưng thng song song vi CD, t D k đường
thng song song vi BC, chúng ct nhau ti M.
Áp dng tính cht v hai đoạn thng song song b chn bi
các đường thng song song
Ta có:
2BM CD cm==
1MD BC cm==
Suy ra:
4AM EM cm==
.
Áp dụng định lý Pythagore cho tam giác AME vuông ti M, ta có
2 2 2 2 2 2
4 4 32AM BM AE AE+ = = + =
.
Vy
32AE cm=
.
Ví d 7. Cho tam giác ABC nhn có ba cnh AB, BC, CA lần lượt là 3 s t nhiên liên tiếp. K
đường cao AH ca tam giác ABC. Chng minh
4HC HB−=
.
NG DN GII
Theo đề bài ta có
12AC BC AB= + = +
.
Suy ra
2AB AC BC+=
.
Áp dụng định lý Pythagore vào hai tam giác vuông ABH và ACH ta có
( )
2 2 2 2 2
HC HB AC AB AH = =
( )( ) ( )( )
HC HB HC HB AC AB AC AB + = +
( )
2.2HC HB BC BC =
4HC HB =
CHUYÊN ĐỀ BỒI DƯỠNG HỌC SINH GIỎI HÌNH HỌC 9
5
Ví d 8. Cho tam giác ABC vuông tại A, đường cao AH. Chng minh:
a)
2
.AH BH CH=
; b)
2
.AB BH BC=
NG DN GII
a) Áp dụng định lý Pythagore cho ba tam giác vuông
ABH, AHC và ABC, ta có:
2 2 2
AB AH BH=+
( )
1
2 2 2
AC AH HC=+
( )
2
2 2 2
BC AB AC=+
( )
3
Cng vế vi vế của ba đẳng thc trên:
2 2 2 2
2BC AH BH HC= + +
( )
2
2 2 2
2BH CH AH BH HC + = + +
2 2 2 2 2
2 . 2BH BH CH HC AH BH HC + + = + +
2
.BH CH AH=
( )
4
b) Kết hợp đẳng thc
( )
4
và đẳng thc
( )
1
ta được
( )
22
. . .AB BH CH HB BH CH HB BH BC= + = + =
.
Ví d 9. Cho
ABC
vuông tại A, đường cao AH. Chng minh
2 2 2
1 1 1
AB AC AH
+=
NG DN GII
S dng kết qu ví d 8, ta có:
2
.AB BH BC=
2
.AC CH BC=
Khi đó:
22
1 1 1 1
. . . .
CH BH
AB AC BH BC CH BC BC BH CH
+
+ = + =
2 2 2
1 1 1 1
. . .
BC
AB AC BC BH CH BH CH AH
+ = = =
III. BÀI TP
Bài 1. Cho tam giác ABC vuông tại A, đường cao AH
( )
H BC
. Chng minh rng
2 2 2 2
2AH BH CH BC+ + =
.
CHUYÊN ĐỀ BỒI DƯỠNG HỌC SINH GIỎI HÌNH HỌC 9
6
Bài 2. Cho hai điểm
( )
;
AA
A x y
( )
;
BB
B xy
trong mt phng tọa độ. Chng minh:
( ) ( )
22
A B A B
AB x x y y= +
.
Bài 3. Cho tam giác ABC vuông ti A
( )
AB AC
, đường cao AH, trung tuyến AM. Biết rng
40AH cm=
;
41AM cm=
. Chng minh rng
54AB AC=
.
Bài 4. Cho tam giác ABC vuông ti A,
30C =
. Chng minh rng
2BC AB=
.
Bài 5. Cho tam giác ABC có
135A=
. Biết
2BC =
;
2AB =
. Chng minh rng
2CB=
.
Bài 6. Cho tam giác ABC vuông ti A. Một đường thng bt k ct cnh AB, AC theo th t ti D
và E. Chng minh rng
2 2 2 2
BC CD BE DE =
.
Bài 7. Cho tam giác ABC có
60A=
. Chng minh rng
2 2 2
.BC AB AC AB AC= +
.
Bài 8. Cho tam giác ABC vuông ti A, k AH vuông góc vi BC
( )
H BC
. Trên tia đối ca tia
HA lấy điểm D, trên cnh AC lấy điểm E sao cho
90BDE =
. Đường thng qua E song song vi
BC ct AH ti F. Chng minh
AF HD=
.
Bài 9. Cho tam giác ABC vuông tại A, các đường trung tuyến BM và CN. Chng minh rng:
2
22
5
4
BC
BM CN+=
.
Bài 10. Cho tam giác ABC có hai đường trung tuyến BM và CN vuông góc vi nhau. Chng minh
rng
2 2 2
5BC AB AC=+
.
Bài 11*. Cho tam giác ABC vuông tại A. I là giao điểm của các đường phân giác trong. E và F ln
t là hình chiếu vuông góc ca A xung BI và CI. Chng minh
22
2AI EF=
.
CHUYÊN ĐỀ BỒI DƯỠNG HỌC SINH GIỎI HÌNH HỌC 9
7
Bài 12. Cho tam giác ABC. Gi H là trc tâm ca tam giác. Chng minh
rng
2 2 2 2
AH BC BH AC+ = +
.
Bài 13*. Cho tam giác ABC nhn có trc tâm H. Gọi M là trung điểm của BC. Đường thng qua A
song song vi MH và đường thng qua H song song vi MA ct nhau ti N. Chng minh rng
2 2 2
AH BC MN+=
.
Bài 14*. Cho tam giác ABC tho mãn
AC AB
( )
2BC AC AB=−
. D là một điểm trên cnh
BC. Chng minh rng
2ABD ADB=
khi và ch khi
3BD CD=
.
Bài 15*. Cho tam giác ABC nhn có
60A=
. Chng minh rng:
1 1 3
BC AC BC AB AB BC CA
+=
+ + + +
Bài 16. Cho tam giác ABC vuông cân ti A, gọi M là điểm nm trên cnh BC. Chng minh rng
2 2 2
2MB MC MA+=
.
Bài 17. Cho tam giác ABC, t điểm M nm trong tam giác, ta h các đường vuông góc
MD BC
,
ME AB
,
MF AC
. Chng minh rng
2 2 2 2 2 2
AE BD CF AF BE CD+ + = + +
.
IV. NG DN GII
Bài 1. (Bạn đọc t v hình)
Áp dụng định lý Pythagore vào các tam giác vuông AHB và AHC ta được:
2 2 2
AB AH BH=+
( )
1
;
2 2 2
AC AH CH=+
( )
2
.
Cộng các đẳng thc
( )
1
( )
2
và chú ý
2 2 2
BC AB AC=+
ta được điều phi chng minh.
Bài 2. Thy rng tam giác ABH vuông ti H và
;
A B A B
HA y y HB x x= =
.
Áp dụng định lý Pythagore vào tam giác ABH cho ta điều phi chng minh.
Bài 3. Vì AM là đường trung tuyến ng vi cnh huyn ca
tam giác vuông ABC nên theo nhn xét ví d 3 ta có
41MA MB MC cm= = =
. Áp dụng định lý Pythagore vào
tam giác vuông AHM ta tính được
9HM cm=
.
T đó tính được
32HB cm=
;
50HC cm=
.
Áp dụng định lý Pythagore vào các tam giác vuông ABH và
ACH ta có:
2 2 2 2 2
40 32 2624AB AH BH= + = + =
;
CHUYÊN ĐỀ BỒI DƯỠNG HỌC SINH GIỎI HÌNH HỌC 9
8
2 2 2 2 2
40 50 4100AC AH CH= + = + =
Suy ra
2
2
2624 16
4100 25
AB
AC
==
Vy
4
5
AB
AC
=
hay
54AB AC=
Bài 4. Vì tam giác ABC vuông ti A,
30C =
nên
60B =
.
Lại có AM là đường trung tuyến ng vi cnh
huyn ca tam giác vuông ABC nên
MA MB MC==
.
T đó tam giác MAB đều.
Vy
1
2
AB MB BC==
hay
2BC AB=
.
Chú ý: Có th chứng minh được rng: Mt tam giác vuông có mt cnh góc vuông dài bng mt
na cnh huyền thì góc đối din vi cạnh góc vuông đó bằng 30°.
Bài 5. V đường cao CH ca tam giác ABC.
Ta có:
180 135 45CHA= =
.
ACH
có:
90H =
;
45CAH =
.
Vy
ACH
vuông cân tại đỉnh H.
Áp dụng định lý Pythagore cho
ACH
ta có:
1HC HA==
.
Tam giác CHB vuông ti H ta có
1
2
HC BC=
nên
30CBH =
t
đó ta có điều phi chng minh.
Bài 6. Ni B vi E; C vi D.
Áp dụng định lý Pythagore vào các tam giác vuông ABC
và ADC ta có:
2 2 2
BC AB AC=+
( )
1
;
2 2 2
CD AD AC=+
( )
2
.
Tr
( )
1
cho
( )
2
ta được
2 2 2 2
BC CD AB AD =
Tương tự áp dụng định lý Pythagore vào các tam giác vuông ADE và
ABE ta được
2 2 2 2
BE DE AB AD =
. Vy
2 2 2 2
BC CD BE DE =
.
CHUYÊN ĐỀ BỒI DƯỠNG HỌC SINH GIỎI HÌNH HỌC 9
9
Bài 7. Không mt tính tng quát gi s
BC
.
K đường cao BH vi H nm trên cnh AC.
Tam giác AHB vuông ti H có
30ABH =
nên
1
2
AH AB=
.
Theo định lý Pythagore ta có:
2 2 2 2
BC BH HC BC=+=
( )
2 2 2 2
2
BH HC AB AH AC AH= + = +
2 2 2 2
2 . .AB AC AC AH AB AC AB AC= + = +
.
Bài 8. Áp dụng định lý Pythagore vào các tam giác vuông
ABE, ABH, AEF, BDE, BHD, BHA, BAE, EAF ta được
2 2 2
BE AB AE=+
( ) ( )
2 2 2 2
BH AH AF EF= + + +
( )
1
Áp dụng định lý Pythagore vào các tam giác BDE, BDH,
DFE ta được
( ) ( )
2 2 2 2 2 2 2
BE BD DE BH HD DF EF= + = + + +
( )
2
T
( )
1
( )
2
suy ra:
2 2 2 2
AH AF DF HD+ = +
( )
3
* Nếu
AF HD
thì
AH DF
, khi đó
2 2 2 2
AH AF DF HD+ +
.
* Nếu
AF HD
thì
AH DF
, khi đó
2 2 2 2
AH AF DF HD+ +
.
Vậy đẳng thc
( )
3
ch xy ra khi
AF HD=
, t đó ta có điều phi chng minh.
Bài 9. Cách 1: S dng công thc trung tuyến.
Cách 2: Áp dụng định lý Pythagore vào các tam giác ABM
và CAN ta được:
2
22
4
AC
BM AB=+
;
2
22
4
AB
CN AC=+
Cộng các đẳng thc trên li và để ý rng
2 2 2
AB AC BC+=
,
ta có điều phi chng minh.
Bài 10. (Bạn đọc t v hình)
Gọi G là giao điểm của BM và CN, khi đó G là trọng tâm
ca tam giác. Áp dng công thc trung tuyến ta được:
2 2 2
2
24
AB BC AC
BM
+
=−
;
2 2 2
2
24
AC BC AB
CN
+
=−
.
Li có
3
2
BM BG=
;
3
2
CN CG=
, thay vào công thức trên ta được:
CHUYÊN ĐỀ BỒI DƯỠNG HỌC SINH GIỎI HÌNH HỌC 9
10
2 2 2
2
9
4 2 4
AB BC AC
BG
+
=−
( )
1
2 2 2
2
9
4 2 4
AC BC AB
CG
+
=−
( )
2
Cộng các đẳng thc
( )
1
,
( )
2
và chú ý tam giác BGC vuông tại G, ta có điều phi chng minh.
Bài 11. Ni AI. Gi O là trung điểm ca AI.
Các tam giác vuông AFI và AEI có FO và EO lần lượt là đường trung tuyến ng vi cnh huyn AI
nên ta có
2
AI
OF OE==
.
Vy tam giác FOE cân ti O.
Li có
180 135
2
BC
BIC
+
= =
Hay
135FIA EIA+ =
Do đó:
FAI EAI+
( ) ( )
90 90FIA EIA= +
( )
180 FIA EIA= +
180 135 45= =
.
Có các tam giác OAF và OAE cân ti O, theo tính cht góc ngoài ca tam giác, ta có
( )
2 90FOE FOI EOI FAI EAI= + = + =
.
Vy tam giác FOE vuông cân ti O. T đó áp dụng định lý Pythagore vào tam giác vuông cân FOE
ta được:
( )
( )
2
2 2 2 2 2
2. 4. 2 2AI OE OE OE OF EF= = = + =
.
Bài 12. Gọi I là giao điểm ca CH và AB. Áp dụng định lý Pythagore vào các tam giác vuông AHI,
BHI, ACI, BCI ta suy ra:
2 2 2 2
AH AI BH BI =
( )
1
2 2 2 2
AC AI BC BI =
( )
2
Tr
( )
2
cho
( )
1
ta được
2 2 2 2
AC AH BC BH =
T đó:
2 2 2 2
AH BC BH AC+ = +
.
Chú ý:
CHUYÊN ĐỀ BỒI DƯỠNG HỌC SINH GIỎI HÌNH HỌC 9
11
+ Chng minh trên vẫn đúng trong trường hợp tam giác ABC là tam giác tù. Trong trường hp tam
giác ABC vuông thì mt s điểm trùng nhau nhưng kết qu vẫn đúng.
+ Bng cách chứng minh tương tự có th suy ra:
2 2 2 2 2 2
AH BC BH AC CH AB+ = + = +
Bài 13. Lấy D là điểm đối xng vi H qua M.
D dàng chứng minh được
//BH DC
,
BH DC=
t đó suy ra
DC AC
.
Áp dụng định lý Pythagore vào tam giác ADC vuông ti C, ta được:
2 2 2 2 2
AD AC CD AC BH= + = +
(vì
BH CD=
).
Theo kết qu bài tp 12 ta có:
2 2 2 2
AH BC BH AC+ = +
.
Như vậy:
2 2 2
AD AH BC=+
.
Cui cùng, d dàng chứng minh được
MN AD=
.
Do đó
2 2 2
AH BC MN+=
.
Bài 14. Ta xét ba trường hp:
+ Trường hp
90B 
(Hình 1.17a)
H
AH BC
. Lấy điểm E thuộc đoạn thng CH sao cho
AE AB=
.
Theo định lý Pythagore ta có:
( )( )
2 2 2 2
AC AB CH BH CH BH CH BH = = +
( )
. .2CE BC CE AC AB= =
Do vy
( )( ) ( )
2AC AB AC AB CE AC AB + =
.
Suy ra
2AC AB CE+=
.
Theo bài ra
( )
1
2 2 2 .
2
BC AC AB AB BC CE= + =
( )
1
Vì vy
22ABD ADB AEB ADB= =
Tam giác AED cân ti E
AB AE DE = =
1
22
2
DE BC CE + =
. (theo
( )
1
)
( )
44BC CE DE CD = =
3BD CD=
.
+ Trường hp
90B =
CHUYÊN ĐỀ BỒI DƯỠNG HỌC SINH GIỎI HÌNH HỌC 9
12
Theo định lý Pythagore ta được:
( )( )
22
AC AB AC AB AC AB = +
( )
2
2.BC AC AB BC= =
2AC AB BC + =
1
2
2
BC AB AB BC + + =
3
4
AB BC=
.
Do đó
2 45ABD ADB ADB BAD AB BD= = = =
3
3
4
BD BC BD BC = =
.
+ Trường hp
90B 
.
H
AH BC
. Lấy điểm E thuc đon thng CH sao cho
AE AB=
.
Theo định lý Pythagore ta có:
2 2 2 2
AC AB CH BH =
( )( )
CH BH CH BH= +
( )
. .2.CE BC CE AC AB= =
Do vy
( )( ) ( )
2AC AB AC AB CE AC AB + =
.
Suy ra
2AC AB CE+=
.
Theo bài ra
( )
1
2 2 2
2
BC AC AB AB BC CE = + =
.
Vì vy:
2 180 2ABD ADB ABE ADB= =
2 2 180AEB ADB ABE ADB+ = + =
( )
2
180AEB EAD ADE+ + =
nên
( )
2 EAD ADE=
Tam giác AED cân ti E
AB AE DE = =
1
22
2
DE BC CE + =
( )
44BC CE DE CD = =
3BD CD=
.
CHUYÊN ĐỀ BỒI DƯỠNG HỌC SINH GIỎI HÌNH HỌC 9
13
Bài 15. Đặt
BC a=
,
CA b=
,
AB c=
.
K BH vuông góc vi AC (H thuc AC)
Theo bài ra ta có
60A=
nên
30ABH =
.
Theo bài 1.4 ta có
1
2
AH AB=
.
Đẳng thc cn chng minh
1 1 3
BC AC BC AB AB BC CA
++
+ + + +
tr thành
1 1 3
a b a c a b c
++
+ + + +
( )( ) ( )( ) ( )( )
3a b c a c a b c a b a b a c+ + + + + + + = + +
( )
1
Ta s chứng minh đẳng thc
( )
1
.
Tht vy: Theo định lý Pythagore ta có:
2 2 2
BC BH HC=+
;
2 2 2
BH AB AH=−
( )
2
2
HC AC AH=−
.
Do đó:
2 2 2 2 2
2 . .BC AB AC AH AC AB AC AB AC= + = +
.
Hay
2 2 2
a b c bc= +
. Ta có:
2 2 2
a b c bc= +
2 2 2 2
3 2 3 3 3 3 3 2a a ab ab ac ac b c bc bc + + = + +
( )
2 2 2 2
3 2 3 3 2a ab ac bc a b c ab ac bc + + + = + + + + +
( )( ) ( )( ) ( )( )
3 a b a c a b c a c a b c a b+ + = + + + + + + +
.
Bài 16. Gọi điểm E và điểm F lần lượt là hình chiếu ca
điểm M trên các đường thng AB và AC.
Do tam giác ABC vuông cân ti A nên các tam giác
BEM và tam giác CFM lần lượt cân ti E và F.
Áp dụng định lý Pythagore cho tam giác BME vuông
ti E:
2 2 2 2
2MB EB EM EM= + =
( )
1
Áp dụng định lý Pythagore cho tam giác CMF vuông ti F.
2 2 2 2
2MC FM FC FM= + =
( )
2
T
( )
1
( )
2
suy ra:
( )
2 2 2 2
2MB MC EM FM+ = +
Vy
2 2 2
2MB MC MA+=
.
CHUYÊN ĐỀ BỒI DƯỠNG HỌC SINH GIỎI HÌNH HỌC 9
14
Bài toán 2.s dng tam giác bằng nhau để chứng minh đẳng thc hình hc
I. KIN THC CN NH
1. Định nghĩa: Hai tam giác bng nhau là hai tam giác có các cạnh tương ứng bng nhau và các
góc tương ứng bng nhau.
Như vậy:
,,
,,
AB A B AC A C BC B C
ABC A B C
A A B B C C
= = =
=
= = =
2. Các trường hp bng nhau ca tam giác
a) Trường hp bng nhau th nht cnh cnh cnh (c.c.c)
* Nếu ba cnh ca tam giác này bng ba cnh của tam giác kia thì hai tam giác đó bằng nhau.
* C th: Xét
ABC
ABC
Nếu có:
AB AB

=
'CA C A
=
BC B C

=
Thì
( )
c.c.cABC AB C
=
.
b) Trường hp bng nhau th hai cnh góc cnh (c.g.c)
* Nếu hai cnh và góc xen gia ca tam giác này bng hai
cnh và góc xen gia của tam giác kia thì hai tam giác đó bằng
nhau.
* C th: Xét
ABC
ABC
Nếu có:
AB AB

=
BB
=
BC B C

=
Thì
( )
c.g.cABC A B C
=
.
* Chú ý: T trường hp bng nhau cnh góc cnh nói trên ta suy ra: Nếu hai cnh góc vuông
ca tam giác vuông này lần lượt bng hai cnh góc vuông ca tam giác kia thì hai tam giác vuông
đó bằng nhau.
c) Trường hp bng nhau th ba góc cnh góc (g.c.g)
CHUYÊN ĐỀ BỒI DƯỠNG HỌC SINH GIỎI HÌNH HỌC 9
15
* Nếu mt cnh và hai góc k ca tam giác này bng mt cnh và hai góc k ca tam giác kia thì
hai tam giác đó bằng nhau.
* C th: Xét
ABC
ABC
Nếu có:
BB
=
BC B C

=
CC
=
Thì
( )
c.c.cABC AB C
=
.
* Chú ý: T trường hp bng nhau góc cnh góc nói trên
ta suy ra:
- Nếu cnh huyn và mt góc nhn ca tam giác vuông này bng cnh huyn và mt góc nhn ca
tam giác vuông kia thì hai tam giác vuông đó bằng nhau.
- Nếu mt cnh góc vuông và mt góc nhn k cnh y ca tam giác vuông này bng mt cnh góc
vuông và mt góc nhn k cnh y ca tam giác vuông kia thì hai tam giác vuông đó bằng nhau.
d) Trường hợp đặc bit ca tam giác vuông
* Nếu cnh huyn và mt cnh góc vuông ca tam giác vuông này
bng cnh huyn và mt cnh góc vuông ca tam giác vuông kia
thì hai tam giác vuông đó bằng nhau.
* C th: Xét
ABC
ABC
Nếu có:
90AA
= =
AB AB

=
BC B C

=
Thì
ABC A B C
=
(cnh huyn cnh góc vuông)
II. CÁC VÍ D
Ví d 1. Cho tam giác ABC vuông cân ti A. Qua A k đường thng d bt k sao cho d không ct
đoạn thng BC. T BC k BHCK vuông góc vi
( )
,d H K d
. Chng minh rng
BH CK HK+=
.
NG DN GII
Ta có
90HAB KAC+ =
;
90KCA KAC+ =
.
T đó
HAB KCA=
.
CHUYÊN ĐỀ BỒI DƯỠNG HỌC SINH GIỎI HÌNH HỌC 9
16
Hai tam giác vuông BHAAKC
AB AC=
(vì tam giác ABC cân ti A);
HAB KCA=
(chng
minh trên) nên bng nhau (cnh huyn góc nhn).
Suy ra
; BH AK CK AH==
(các cp cạnh tương ứng).
T đó
BH CK AK AH HK+ = + =
.
Ví d 2. Cho tam giác ABC. Gi MN lần lượt là trung điểm ca ABAC. Chng minh rng
1
2
MN BC=
.
NG DN GII
Trên tia MN ly P sao cho N là trung điểm ca MP.
Ta có
( )
c.g.cANM CNP =
. Suy ra:
; PC MA AMN CPN==
.
Vì hai góc
AMN
CPN
v trí so le trong nên
//AB CP
.
T đó
BMC PCM=
.
Hai tam giác MPCCBM
( )
MB PC MA==
; MC chung;
BMC PCM=
nên bng nhau (c.g.c),
T đó
MP BC=
. Vy
1
2
MN BC=
.
Chú ý:
Theo NG DN GII ca ví d trên, vì
MPC CBM =
nên
BCM PMC=
.
T đó
//MN BC
. Người ta gọi đoạn thng nối trung điểm hai cnh ca mt tam giác là đường trung
bình của tam giác đó, ta có tính chất: Đưng trung bình ca mt tam giác song song vi cnh còn
li và dài bng na cnh y.
Ví d 3. Cho tam giác ABC
90A
, v v phía ngoài tam giác ABC các tam giác ABDACE
vuông cân ti A.
a) Chng minh
BE CD=
.
b) Gi M là trung điểm ca BC. Chng minh
1
2
AM DE=
.
NG DN GII
CHUYÊN ĐỀ BỒI DƯỠNG HỌC SINH GIỎI HÌNH HỌC 9
17
a) Ta có
( )
90DAC BAE BAC= = +
.
Hai tam giác DACBAE
AD AB=
;
AC AE=
;
DAC BAE=
nên bng nhau (c.g.c), suy ra
BE CD=
.
b) Trên tia AM lấy điểm N sao cho M là trung điểm ca AN.
Thy rng,
180DAE BAC ABC BAC= = +
.
Mt khác
( )
c.g.cCAM BNM =
Nên
ACB CBN=
,
BN AC=
.
Ta có
ABN ABC CBN ABC ACB DAE= + = + =
Vy
( )
c.g.cDAE ABN =
.
T đó suy ra,
2DE AN AM==
hay
1
2
AM DE=
.
Ví d 4. Cho tam giác ABC, đường phân giác AD. Đường thng vuông góc vi AD ti A ct BC ti
E. Biết C nm gia B, E
BE AB AC=+
. Chng minh rng:
3 360BAC ACB+ =
.
NG DN GII
Trên tia đối ca tia AB lấy điểm F sao cho
AF AC=
.
Ta có:
BE AB AC AB AF BF= + = + =
nên
BEF
cân ti B.
Do đó
( )
1F BEF=
.
Li có
AE AD
, mà AD là đường phân giác trong đỉnh A ca
ABC
nên AE là đường phân giác ngoài đỉnh A ca
ABC
.
Hay
CAE FAE=
.
Do vy,
( )
c.g.cCAE FAE =
.
T đó suy ra
ACE F=
,
( )
2AEC AEF=
.
T (1) và (2) suy ra
2ACE F CEF AEC= = =
.
Ta có
180ACB ACE CAE AEC= = +
.
Suy ra
( )
33ACB CAE AEC=+
.
Do đó:
( )
33BAC ACB BAC CAE AEC+ = + +
( ) ( )
BAC CAF AEC ACE CAE= + + + +
CHUYÊN ĐỀ BỒI DƯỠNG HỌC SINH GIỎI HÌNH HỌC 9
18
180 180 360= + =
.
III. BÀI TP
Bài 1. Cho tam giác ABC, trên cnh AB ly DE sao cho
AD BE=
. Qua DE k các đường
thng song song vi BC ct AC theo th t ti MN. Chng minh
BC DM EN=+
.
Bài 2. Cho tam giác ABC cân ti A
30A=
. Bên ngoài tam giác ABC, dựng tam giác đều BDC.
Chng minh rng
2 2 2
AD AB AC=+
.
Bài 3. Cho tam giác ABC cân ti A
100A=
. Tia phân giác trong góc B ct AC D. Chng minh
BC BD AD=+
.
Bài 4. Cho tam giác ABC cân ti A
80A=
. Lấy điểm M min trong của tam giác và điểm N
trên cnh AC sao cho
150 , 10 , 160BMC MBC BMN= = =
. Chng minh rng
BM MN NA=+
.
Bài 5. Cho tam giác ABC vuông cân ti A. Trên na mt phng b AC chứa điểm B, lấy điểm D sao
cho CD vuông góc vi AC
CD AC=
. M là điểm trên đoạn thng CD sao cho
2MD MC=
. N
trung điểm của đoạn thng BD. Chng minh
AMC AMN=
.
Bài 6. Cho tam giác ABC vuông ti A
3AC AB=
. Trên cnh AC lấy hai điểm DE sao cho
AD DE EC==
(D nm gia AE). Chng minh
45AEB ACB+ =
.
Bài 7. Cho tam giác ABC. V đường phân giác trong AD ca tam giác. Trên AD lấy hai điểm EF
sao cho
ABE CBF=
. Chng minh
ACE BCF=
.
Bài 8. Cho tam giác ABC vuông ti A. E là một điểm nm trên cnh BC sao cho
2EC EB=
. Chng
minh rng
( )
2 2 2
3AC EC EA=−
.
Bài 9. Cho tam giác ABC, đường trung tuyến AM. Trên na mt phng b AB chứa đỉnh C v đoạn
thng AE vuông góc vi AB
AE AB=
. Trên na mt phng b AC chứa đỉnh B v đoạn thng
AF vuông góc vi AC
AF AC=
. Chng minh rng
2EF AM=
.
Bài 10. Cho tam giác ABC vuông cân ti A. Gi E là trung điểm ca AC. Qua A k đường thng
vuông góc vi BE ct BC ti D. Chng minh
2AD ED=
.
Bài 11. Cho tam giác ABC vuông cân ti A. M là điểm nm trong tam giác sao cho
15ABM =
;
30BAM =
. Chng minh rng:
2BC AM=
.
Bài 12. Cho tam giác ABC vuông cân ti A. Lấy các điểm DE lần lượt thuc các cnh ABAC
sao cho
AD AE=
. Đường thẳng đi qua D và vuông góc vi BE ct CA K. Chng minh rng
AK AC=
.
CHUYÊN ĐỀ BỒI DƯỠNG HỌC SINH GIỎI HÌNH HỌC 9
19
Bài 13. Cho tam giác ABC. Các tia phân giác trong ca góc BC ct nhau ti I. Qua I k mt
đường thng song song vi BC, đường thng này ct AB, AC theo th t DE. Chng minh
rng
DE BD CE=+
.
Bài 14. Cho tam giác ABC cân ti A. K đường phân giác trong CD. Qua D k đường thng vuông
góc vi CD ct BC ti F. Đường thng k qua D song song vi BC ct AC ti E. Tia phân giác góc
BAC ct DE ti M. Chng minh rng:
a)
2CF BD=
b)
4CF MD=
Bài 15. Gi I là giao điểm ba đường phân giác trong ca tam giác ABC. Chng minh rng
AB BI AC+=
khi và ch khi
2.ABC ACB=
.
Bài 16. Cho tam giác ABC cân ti A
20A=
. Trên cnh AB lấy điểm D sao cho
30BDC =
.
Chng minh rng
AD BC=
.
Bài 17. Cho tam giác ABC
60 , 70AB= =
. Lấy điểm D trên cnh AB sao cho
20ACD =
.
Chng minh rng
AC AD BD BC+ = +
.
Bài 18. Cho tam giác ABC nhn, gi H là trc tâm ca tam giác. Gi M là trung điểm của đoạn
thng BC, I là giao điểm các đường phân giác ca các góc
ABH
ACH
. Đường thng MI ct AH
ti N. Chng minh rng
NA NH=
.
IV. NG DN GII
Bài 1.
Qua N k
( )
//NF AB F BC
. Ni
EF
.
Ta có
( )
g.c.gBEF NEF =
nên
;BE AD NF EN BF= = =
.
Li có
NFC ABC ADM==
ng v);
NCF AMD=
ng v).
Do vy
DAM FNC=
.
Ta có
( )
g.c.gADM NFC =
nên
DM FC=
.
T đó suy ra
BC BF FC DM EN= + = +
.
Bài 2.
Dng phía ngoài tam giác ABC tam giác AEB đều, ni EC.
Ta có:
90 ,EAC EBC ACD= =
.
Hai tam giác EBCACD bng nhau
( )
c.g.c
, suy ra
EC AD=
.
Li có tam giác EAC vuông ti A nên theo định lý Pythagore, ta có:
2 2 2
EC EA AC=+
.
CHUYÊN ĐỀ BỒI DƯỠNG HỌC SINH GIỎI HÌNH HỌC 9
20
Để ý rng,
,EA AC EC BD==
nên
2 2 2
AD AB AC=+
.
Bài 3.
Trên cnh BC lấy hai điểm DE sao cho
80BDE =
,
60BDK =
.
Ta có
( )
g.c.gBAD BKD =
nên
AD DK=
.
Li có
20KDE =
,
100DKE A= =
suy ra:
1
80E =
,
100DEC =
,
40EDC C= =
.
Vy
DEC
cân ti E, t đó
DE EC=
.
D dàng chng minh
BDE
cân ti B,
KDE
cân ti D nên
BD BE=
,
DE DK AD==
.
Cui cùng,
BC BE EC BD AD= + = +
.
Bài 4.
Ni AM. Đường cao AH ca
ABC
ct BM ti P. K
AK PM
. CM ct AK ti Q.
Ta có:
10PAK PBH= =
và các tam giác APBBPC cân ti P.
Tính được s đo các góc:
20 , 10 , 20 , 30MCB PCB MPC QAC= = = =
,
80 , 100ANM APM APC= = =
.
D dàng chng minh
PAC
cân ti P,
QAC
cân
ti Q nên PQ là đường trung trc ca AC và do đó tia
PQ là tia phân giác ca góc APC.
Như vậy,
50
2
APC
QPC = =
30QPM QPC MPC= =
.
Mt khác,
30QMP =
nên
QPM
cân ti Q.
T đó,
APM
cân ti A.
Vy
80AMP APM= =
,
80AMN PMN AMP= =
.
Chứng minh được hai tam giác cân APMAMN bng nhau nên
AP AN=
,
PM MN=
.
Cui cùng,
BM BP PM AP PM MN AN= + = + = +
.
CHUYÊN ĐỀ BỒI DƯỠNG HỌC SINH GIỎI HÌNH HỌC 9
21
Bài 5.
Trên tia đối tia BD lấy điểm E sao cho
BE MC=
.
Ta có
//BA CD
(cùng vuông góc vi AC) nên
ACB DBC=
(hai
góc so le trong). Mt khác
( )
AB CD AC==
.
Vy
( )
c.g.cABC DCB =
Suy ra,
BD AC AB DC= = =
.
Ta có
( )
c.c.cABD ACD =
nên
90ABD ACD= =
.
D dàng chng minh
( )
c.c.cABE ACM =
, suy ra
, AE AM AEB AMC==
.
Chng minh được
( )
c.g.cAEN AMN =
nên ta có
AMN AEN=
.
Vy
AMN AMC=
.
Bài 6.
Trên na mt phng b AC không cha B v hình vuông ADKH.
Ta có
( )
c.g.cBHK CDK =
nên
, BK CK BKH CKD==
.
Tam giác BKC
BK KC=
nên
BKC
cân ti K.
Hơn nữa,
11
90BKC K CKD K BKH+ + = + =
.
Vy
BKC
vuông cân ti K.
T đó,
45KCB =
.
Li có
( )
c.g.cAEB DCK =
nên
1
AEB C=
.
Cui cùng,
1
45AEB ACB C ACB KCB+ = + = =
.
Bài 7.
Dựng các điểm H, I, K sao cho AB là đường trung trc của đoạn
thng EI, AC là trung trc của đoạn thng EH, BC là đường
trung trc của đoạn thng FK.
Theo tính cht của điểm nằm trên đường trung trc ta có
AI AE AH==
. Vy
IAH
cân ti H.
Mt khác d dàng chứng minh được AD là tia phân giác ca góc
IAH. Như vậy, AD là đường trung trc của đoạn thng IH. Do F
nm trên AD nên ta có
( )
1FI FH=
.
CHUYÊN ĐỀ BỒI DƯỠNG HỌC SINH GIỎI HÌNH HỌC 9
22
Li có
( )
c.g.cFIB KEB =
nên
( )
2FI KE=
.
T (1) và (2) suy ra
FH KE=
.
Xét
FHC
KEC
, , FH KE FC KC HC EC= = =
.
T đó
( )
c.c.cFHC KEC =
.
Vy
HCF ECK=
. Suy ra
HCE KCF=
.
Cui cùng, vì
;
22
KCF HCE
BCF ACE==
nên
ACE BCF=
.
Bài 8.
Gi G là giao điểm của hai đường trung tuyến ADBF
ca tam giác ABC.
Theo tính chất đường trung tuyến ng vi cnh huyn ca
mt tam giác vuông, suy ra
2
BC
DA DB DC= = =
.
G là trng tâm ca tam giác ABC nên
1
3
DG
AD
=
.
Mt khác, do D là trung điểm ca BC nên theo đề bài ra ta
1
3
DE
BD
=
.
Như vậy
1
3
DG DE
AD BD

==


.
Kết hp vi
AD BD=
ta được
DG DE=
.
( )
c.g.cDGB DEA =
nên
AE BG=
.
Áp dụng định lý Pythagore vào các tam giác ABFABC ta được:
( )
( )
2 2 2 2 2 2
*BF AB AF BC AC AF= + = +
.
Thay
3 3 1
, ,
2 2 2
BF BG BC EC AF AC= = =
vào h thc (*) ta được:
2 2 2
2
3 3 1
2 2 2
BG EC AC AC
= +
Thu gn h thức này ta được điều phi chng minh.
Bài 9.
Trường hp
90BAC =
, kết qu là hin nhiên.
Ta chứng minh bài toán cho trường hp
90BAC 
(trường hp
90BAC 
chứng minh tương
t).
CHUYÊN ĐỀ BỒI DƯỠNG HỌC SINH GIỎI HÌNH HỌC 9
23
Trên tia đối ca tia MA lấy điểm D sao cho
MA MD=
. Đường
thng vuông góc vi AB ti B ct AD ti G. ta có
( )
c.g.cAMC DMB =
suy ra
, D CAM AC BD==
.
Li có
//BG AE
(cùng vuông góc vi AB) nên
BGA EAG=
(Hai góc so le trong).
, BGA D DBG EAG EAC CAG EAC D= + = + = +
.
Vy
DBG EAC=
.
90DBA DBG GBA DBG= + = +
,
90FAE EAC FAC EAC= + = +
.
Vy
DBA FAE=
. Hai tam giác DBAFAE
( )
, , AE AB AF BD AC DBA FAE= = = =
nên
bng nhau
( )
c.g.c
Do đó
FE AD=
. Mà
2AD AM=
nên
2FE AM=
.
Bài 10.
Qua C v đường thng vuông góc vi AC, ct AD ti F.
Do
ABE CAF=
(cùng ph vi
AEB
) nên
( )
g.c.gBAE ACF =
.
T đó suy ra
CF AE EC==
.
Vy
( )
c.g.cCDE CDF =
suy ra
CDE CDF=
.
Trên tia DE lấy điểm G sao cho
ED EG=
.
Ta có
( )
c.g.cAEG CED =
nên
CDE AGE=
//AG DC
.
DAG FDC=
(hai góc đồng v) suy ra
DAG DGA=
.
Vy
DAG
cân ti D, t đó
2DA DG DE==
.
Bài 11.
K đường trung tuyến AD ca
ABC
.
Khi đó AD đồng thời là đường cao và đường phân giác ca
ABC
.
Trên na mt phng b AC không cha D, ly điểm E sao cho
ADE
đều.
D dàng tính được s đo các góc:
30 ; 15ACE CAE= =
.
CHUYÊN ĐỀ BỒI DƯỠNG HỌC SINH GIỎI HÌNH HỌC 9
24
Như vậy:
( )
g.c.gAEC AMB =
.
T đó suy ra:
2
BC
AM AE AD= = =
.
Bài 12.
Ta có
11
DE=
(cùng ph vi góc
K
).
Do đó
( )
g.c.gKAD BAE =
.
T đó suy ra
AB AK=
.
AB AC=
(
ABC
vuông cân ti A).
Nên
AK AC=
.
Bài 13.
//DE BC
nên
, DIB IBC EIC ICB==
.
Mt khác BICI lần lượt là tia phân giác góc B và góc C ca
tam giác ABC nên
, IBC IBD ICB ICE==
.
Do đó
, DIB IBD EIC ICE==
.
T đó suy ra các tam giác BDICEI là các tam giác cân ln
t tại các đỉnh DE.
Vy
DE DI IE DB EC= + = +
.
Bài 14.
a) Gi N là trung điểm ca CF. Ni EN. Ta có DN
đường trung tuyến ng vi cánh huyn ca tam giác
vuông CDF nên:
2
CF
ND NF NC= = =
.
Do đó
DNC
cân ti N.
CD là phân giác góc C ca
ABC
nên
ECD NCD=
.
Mt khác
NDC NCD=
(vì
DNC
cân ti N) nên
NDC ECD=
.
T đó
//DN AC
. Vì
//DN AC
nên
ACB DNB=
(hai góc đồng v),
Li có
ACB B=
( vì
ABC
cân ti A) nên
B DNB=
.
CHUYÊN ĐỀ BỒI DƯỠNG HỌC SINH GIỎI HÌNH HỌC 9
25
T đó suy ra:
DBN
cân ti D.
DBN
cân ti D nên
2
CF
BD DN==
hay
2CF BD=
.
b) Chứng minh được ADE cân ti DAM vừa là đường phân giác vừa là đường trung tuyến ca
ADE
.
Ta có:
( )
g.c.gDEN CNE =
nên
2
CF
DE CN==
hay
2CF DE=
.
Do đó M là trung điểm ca DE nên
4CF MD=
.
Bài 15.
Trên tia AB lấy điểm D sao cho
BD BI=
. Vì
BDI
cân ti
B nên
11
24
BID BDI ABI ABC= = =
.
* Nếu
AB BI AB+=
thì
AD AC=
.
( )
c.g.cADI ACI =
nên
ADI ACI=
.
Do đó
11
42
ABC ACB=
hay
2.ABC ACB=
.
* Nếu
2.ABC ACB=
thì
11
42
ABC ACB=
, suy ra
ADI ACI=
.
Do đó
( )
g.c.gADI ACI =
. Nên
AD AC=
.
Mt khác
AD AB BD AB BI= + = +
. Do vy
AC AB BI=+
.
Bài 16.
ABC
cân ti A
20BAC =
nên ta có
80ABC ACB= =
.
Li có
30BDC =
nên
10ACD =
.
Trên na mt phng b AB cha C dng tam giác đều ABE. Theo đó
ta có
40 ; CAE AB BE AE= = =
. Do
ACE
cân ti A nên:
70ACE AEC= =
.
70 60 10BEC AEC AEB= = =
.
80 60 20EBC ABC ABE= = =
.
Ta có
( )
g.c.gADC BCE =
nên
AD BC=
.
CHUYÊN ĐỀ BỒI DƯỠNG HỌC SINH GIỎI HÌNH HỌC 9
26
Nhn xét: Ta có th đưa ra bài toán ngược lại như sau: Cho tam giác ABC
80B =
. Trên cnh
AB lấy điểm D. Biết rng
30BDC =
. Chng minh rng
AB AC=
.
Bài 17.
Trên tia đối ca tia DC lấy điểm E, trên tia đối ca tia CD ly
điểm I, trên tia CA lấy điểm F sao cho:
, , DE DB DI CE CE CF= = =
.
Ta tính được:
50 , 30 , 80ACB DCB CDB= = =
.
Lai có
CEF
cân ti C
20C =
nên
80CFE CEF= =
.
Theo phn nhn xét bài 15, ta có
IDB
vi
80IDB =
, C
điểm trên cnh ID tha mãn
30DCB =
nên
IB ID=
. Hơn
na ta có
BID
cân ti I vi
20I =
.
Ta có
( )
c.g.cCEF IDB =
nên
EF BD ED==
.
Do đó
EFD
cân ti E. Ta tính được
50 , 30 , 30EFD EDF ADF AFD= = = =
Ta có
AFD
cân ti A nên
AF AD=
.
Cui cùng,
AC AD AC AF CF CE CD DE CD DB+ = + = = = + = +
.
Bài 18.
Gi H là giao điểm các đường cao BDCE. Ni EM, EN, DM, DN.
Theo tính chất đường trung tuyến ng vi canh huyn ca tam giác
vuông, ta có
2
BC
EM DM==
.
Li có
90ABD ACE BAC= =
nên
IBA IBD ICA ICE= = =
1 1 1
45
2 2 2
ABD ACE BAC= = =
.
Do vy
( ) ( ) ( ) ( )
IBC ICB ABC ABI ACB ACI ABC ACB ABI ACI+ = + = + +
( )
1
180 2 45 90
2
BAC BAC

= =


.
T đó
BIC
vuông ti I.
CHUYÊN ĐỀ BỒI DƯỠNG HỌC SINH GIỎI HÌNH HỌC 9
27
Ta có IM là đường trung tuyến ng vi cnh huyn ca
BIC
nên
2
BC
IM EM DM= = =
. T đó,
M nằm trên đường trung trc ca DE (1).
Mt khác vì
IMB
EMB
cân ti M nên
( ) ( ) ( )
180 2 180 2 2IME IMB EMB IBC EBC EBC IBC= = =
2 90EBI ABD BAC= = =
.
Chứng minh tương tự ta được
90IMD BAC=
. Do vy
IME IMD=
.
Ta có
( )
c.g.cEMI DMI =
nên
ID IE=
. T đó I nằm trên đường trung trc ca DE (2).
T (1) và (2) suy ra MI là đường trung trc ca DE.
Li có N nm trên MI nên
NE ND=
.
Gi
N
là trung đim ca AH, theo tính chất đường trung tuyến ng vi cnh huyn ca tam giác
vuông ta cũng có
N D N E

=
.
Do vy
N
cũng nằm trên đường trung trc ca DE.
T đó N
N
trùng nhau suy ra Ntrung điểm ca AH hay
NA NH=
.
CHUYÊN ĐỀ BỒI DƯỠNG HỌC SINH GIỎI HÌNH HỌC 9
28
Bài toán 3.s dng quan h góc và cạnh đối din, quan h đường vuông góc và đường xiên,
quan h đưng xiên và hình chiếu, bất đẳng thc tam giác
I. KIN THC CN NH
1. Quan h gia góc và cạnh đối din:
Trong một tam giác, góc đối din vi cnh lớn hơn là góc lớn hơn, cạnh đối din vi góc lớn hơn là
cnh lớn hơn. Cho tam giác ABC, ta có:
B C AC AB
B C AC AB
Chú ý:
* Nếu
BC=
thì
AC AB=
và ngược li.
* Trong tam giác vuông cnh huyn là cạnh có độ dài ln nht.
2. Quan h giữa đường vuông góc và đường xiên, đường xiên và hình chiếu:
Xét tt c các đường vuông góc và đường xiên k t một điểm nm ngoài một đường thng ti
đường thẳng đó, ta có các kết lun sau:
* Đường vuông góc ngắn hơn mọi đường xiên.
* Đường xiên nào lớn hơn thì có hình chiếu lớn hơn và ngược lại, đường xiên nào có hình chiếu ln
hơn thì lớn hơn.
* Nếu hai đường xiên bng nhau thì hình chiếu bằng nhau. Ngược li, nếu hai đường chiếu bng
nhau thì hai đường xiên bng nhau.
Cho một điểm M nằm ngoài đường thng d. Qua M k đường
vuông góc MH và các đường xiên MA, MB xuống đường thng
d.
Khi đó ta có:
;MH MA MH MB
MB MA HB HA

3. Bất đẳng thc tam giác
Trong một tam giác, độ dài mt cnh luôn lớn hơn hiệu và nh hơn tổng độ dài hai cnh còn li.
Cho tam giác ABC, đặt
;;BC a CA b AB c= = =
Khi đó
b c a b c
a c b a c
a b c a b
+
+
+
B sung: Với ba điểm A, B, C bt k, ta luôn có:
AB AC CB+
. Du
bng xy ra
A, B, C thng hàng và C nm gia A và B.
CHUYÊN ĐỀ BỒI DƯỠNG HỌC SINH GIỎI HÌNH HỌC 9
29
II. CÁC VÍ D
Ví d 1. Cho tam giác ABC không cân, M là trung điểm ca BC.
a) Chng minh
2AB AC AM+
b) Biết
AC AB
, chng minh
MAB MAC
c) K AH vuông góc vi BC (H BC). Tia phân giác góc A ct BC D. Chng minh
MH MD
NG DN GII
Trên tia đối ca tia MA lấy điểm E sao cho
MA ME=
. Ta có AMB = EMC (c.g.c) nên suy ra
,AB CE MAB E==
Áp dng bất đẳng thc tam giác vào ACE ta được
AC CE AE+
,2AB CE AE AM==
nên
2AB AC AM+
b) Xét ACE có
CE AB AC=
suy ra
EAC E
hay
MAC E
MAB E=
nên ta có
MAB MAC
c) Không mt tính tng quát gi s
( )
AB AC
(Trường hp
AB AC
, đổi vai trò ca B và C ri chng minh
tương tự).
Thy rng nếu H nằm ngoài đoạn thng BC. Khi y B nm gia H
và C. Mà D và M nằm trên đoạn thng BC nên hin nhiên
MH MD
Xét trường hp H nằm trên đoạn thng BC.
AB AC
nên ta có
CB
Li có
90 ; 90BAH B CAH C= =
T đó suy ra
BAH CAH
Theo chng minh câu a, vì
AB AC
nên
BAM CAM
Do vy
BAH BAD BAM
Mt khác H, D, M nằm trên đoạn thng BC nên
BH BD BM
hay
MH MD
.
Ví d 2. Cho tam giác ABC, v AH vuông góc vi BC (H thuc đoạn thẳng BC). D là điểm nm
giữa A và H, E là điểm nm giữa B và H, F là điểm nm gia C và H. Chng minh rng chu vi tam
giác DEF nh hơn chu vi tam giác ABC. Tìm một v trí của các điểm D, E, F để chu vi tam giác
CHUYÊN ĐỀ BỒI DƯỠNG HỌC SINH GIỎI HÌNH HỌC 9
30
DEF bng
1
2
chu vi tam giác ABC.
NG DN GII
Ni CD. Vì F nm gia C và H nên
HF HC
. Theo quan h giữa đường xiên và hình chiếu, vì
HF HC
nên
DF DC
.
Li có D nm gia A và H nên
HD HA
. Áp dng quan h giữa đường xiên và hình chiếu, ta được
CD CA
T đó
( )
1DF DC CA
Chứng minh tương tự, ta được
( )
2DE DB AB
Ta có
( )
3EF HE HF HB HC BC= + + =
T(1) (2) và (3) suy ra
+ + + +DE EF FC AB AC BC
. Hay chu
vi tam giác DEF nh hơn chu vi tam giác ABC.
Vì D, E, F lần lượt là trung điểm ca AH, BH và CH, theo Ví d 2
(Chuyên đề 2) ta chứng minh được
;DF ;
2 2 2
AB AC BC
DE EF= = =
Khi đó chu vi tam giác DEF bằng
1
2
chu vi tam giác ABC.
Ví d 3. Cho tam giác ABC vuông cân ti A. Gọi D và E là hai điểm theo th t nm trên hai cnh
AB và AC sao cho
AD AE=
. Gọi K là điểm thuc cnh BC. Trên na mt phng b AC không
chứa điểm B, v điểm I sao cho
EAI DAK=
AI AK=
. Chng minh rng
KE KD AB+
NG DN GII
Ta có
;,AD AE AK AI EAI DAK= = =
nên ADK = AEI (c.g.c).
T đó suy ra
DK EI=
Ta có
( )
1KE KD KE KI KI+ = +
Li có
EAI DAK=
nên
90KAI BAC= =
Như vậy KAI vuông cân ti A.
Theo định lý Pythagore ta được
2 2 2 2
2KI AK AI AK= + =
T đó suy ra
( )
2. 2. 2KI AK AH=
Li có AHB vuông cân ti H nên
2 2 2 2
2AB AH BH AH= + =
, suy ra
( )
23AB AH=
CHUYÊN ĐỀ BỒI DƯỠNG HỌC SINH GIỎI HÌNH HỌC 9
31
T (1), (2) và (3) suy ra:
KE KD AB+
. Dấu “=” xảy ra khi và ch khi K trùng vi H.
Ví d 4. Cho hai tam giác ABC và A'B'C' có
' '; ' 'AB A B AC A C==
. Chng
minh
' ' ' A A BC B C
.
NG DN GII
Gi s tam giác ABC và tam giác A'B'C' có
' '; ' '; 'AB A B AC A C A A= =
. Dựng tam giác A'B”C'
bng tam giác ABC (hình v). Tia phân giác ca góc B'A'B" ct B'C' D. Ta có B'A'D = B”A'D
(c.g.c) nên
'"DB DB=
. Trong tam giác DB"C' ta có
" ' " ' ' ' " 'DB DC B C B C B C BC+ =
.
Ngược li, nếu tam giác ABC và tam giác A'B'C' có:
' '; ' '; 'C'AB A B AC A C BC B= =
.
Ta s chng minh
'AA
. Tht vy:
* Nếu
'AA=
thì ABC = A'B'C'
''BC B C=
(loi)
* Nếu
'AA
, áp dng phn thun ta suy ra
''BC B C
(loi)
Vy
'AA
.
Chú ý: Quan h gia góc và cạnh đối din ch đúng trong một tam giác hoc hai tam giác bng
nhau, còn đối vi hai tam giác không bng nhau thì không áp dụng được. Tuy vy, qua ví d trên ta
thy với hai tam giác có thêm điều kin hai cp cnh bng nhau, ta có mt kết qu đáng chú ý trong
vic chng bất đẳng thc hình hc.
III. BÀI TP
Bài 1. Cho tam giác ABC đều. Gọi M là trung điểm ca BC. Trên cnh AB lấy điểm D, tia DM ct
AC ti E. Chng minh rng
MD ME
.
Bài 2. Cho tam giác ABC cân ti A. Trên cnh BC lấy hai điểm D và E sao cho
BD DE EC==
.
Chng minh rng
BAD EAC DAE=
.
Bài 3*. Cho tam giác ABC nhn,
BC
. K hai đường cao BD và CE. Chng minh
rng:
AC AB CE BD
CHUYÊN ĐỀ BỒI DƯỠNG HỌC SINH GIỎI HÌNH HỌC 9
32
Bài 4*. Cho tam giác ABC có
AB AC
. P là một điểm nm trong tam giác sao cho
PBA PCA=
.
Gọi H và K là chân đường vuông góc h t P xung AB và AC. Gọi I là trung điểm ca BC. Chng
minh
HIB KIC
.
T bài 5 đến bài 11 áp dng kết qu ví d 4.
Bài 5. Cho tam giác ABC có
AB AC
. Trên các cnh AB và AC lần lượt lấy các điểm D và điểm
E sao cho
BD CE=
. Gọi M là trung điểm ca BC. Chng minh
MD ME
.
Bài 6. Cho tam giác ABC có
AB AC
. Gọi M là trung điểm của BC. Trên đoạn thng AM ly
điểm O bt k
( )
OM
. Chng minh rng
OB OC
.
Bài 7. Cho tam giác ABC cân ti A. Trên tia BA lấy điểm E, trên tia đối ca tia CA lấy điểm F sao
cho
.BE CF=
EF ct BC tại điểm D. Chng minh
BD DC
.
Bài 8. Cho tam giác ABC cân ti A. Ly M là một điểm trên cnh BC sao cho
MB MC
. Trên
đoạn thng AM lấy điểm O. Chng minh rng
BOA COA
.
Bài 9. Cho tam giác ABC có
; 90AB AC A
. V v phía ngoài tam giác này các tam giác vuông
cân ABD và ACE. Gọi M là trung điểm ca BC. Chng minh
MD ME
.
Bài 10. Cho hai đoạn thng AC và BD ct nhau tại trung điểm O của đoạn thng AC. Biết
AB BC
, chng minh
CD DA
.
Bài 11. Cho tam giác ABC có
AB AC
. Gọi BD và CE là hai đường cao ca tam giác. Chng
minh
BD CE
.
Bài 12. Cho tam giác ABC tha mãn có
AB AC
, tia phân giác góc A ct BC D. Gọi M là điểm
bt k trên đoạn thng AD (M khác A). Chng minh rng
AB AC MB MC
.
Bài 13. Cho tam giác ABC, M là điểm nm trong tam giác. Chng minh rng:
)a MB MC AB AC+ +
) ++b MA MB MC
lớn hơn nửa chu vi và nh hơn chu vi tam giác ABC.
Bài 14. Chng minh rng tổng độ dài ba đường trung tuyến lớn hơn
3
4
chu vi và nh hơn chu vi
tam giác đó.
Bài 15. Cho tam giác ABC có
1
90 ;
2
B AB AC =
. Chng minh rng
2
A
C
.
Bài 16. Cho tam giác ABC có
90C =
, đường cao CH. Chng minh rng
AC BC AB CH+ +
.
Bài 17. Cho tam giác ABC vuông tại B, đường phân giác AD. Qua C k đường thng vuông góc
vi BC ct AD ti E. Chng minh chu vi tam giác ECD lớn hơn chu vi tam giác ABD.
CHUYÊN ĐỀ BỒI DƯỠNG HỌC SINH GIỎI HÌNH HỌC 9
33
Bài 18. Cho tam giác ABC có
AB AC
. Trên các cnh AC và AB lấy tương ứng các điểm M và N
sao cho
AM AN=
. Gọi O là giao điểm ca BM và CN. Chng minh rng
OB OC
.
Bài 19. Cho tam giác ABC có
60BAC 
. Chng minh rng
2AB AC BC+
Bài 20.Cho tam giác ABC nhn, gi H là trc tâm của tam giác đó. Chứng minh
( )
2
3
HA HB HC AB AC BC+ + + +
.
IV. NG DN GII
Bài 1.
Vì tam giác ABC đều nên
60B ACB A= = =
Theo tính cht góc ngoài
60BDM A E= +
Áp dng quan h gia góc và cạnh đối din trong tam giác BDM ta
được
( )
1DM BM CM=
Tam giác MCE có
120MCE =
và là góc ln nht ca tam giác MCE
nên
( )
2MC ME
T (1) và (2) suy ra
MD ME
.
Bài 2.
Ta có ABD = ACE (c.g.c) nên
( )
1BAD CAE=
Trên tia đối ca tia DA lấy điểm F sao cho
DA DF=
.
AED = FBD (c.g.c) nên
;AE BFDAE F==
.
AEC ABC ACB=
nên trong tam giác AEC ta có
AE AC
T đó suy ra
BF AB
(vì
;)AE BF AB AC==
Trong tam giác ABF có
BF AB
nên
BAD F
DAE F=
nên
( )
2BAD DAE
T (1) và (2) suy ra
BAD EAC DAE=
.
Bài 3.
Trong ABC, vì
ABC ACB
nên
AC AB
.
Trên cnh AC lấy điểm M sao cho
AM AB=
.
Gi N và F lần lượt là hình chiếu ca M trên AB và CE.
Ta có ABD = AMN (cnh huyn - góc nhn) nên
MN BD=
.
Mặt khác vì ∆MNE = ∆EFM (cạnh huyn- góc nhn) nên MN = EF.
CHUYÊN ĐỀ BỒI DƯỠNG HỌC SINH GIỎI HÌNH HỌC 9
34
Do vy
MN BD EF==
.
FCM vuông ti F nên
CM CF
hay
AC AM CE EF
.
T đó suy ra
AC AB CE BD
.
Bài 4.
Gi E và F lần lượt là trung điểm ca BP và CP. Áp dng tính chất đường trung bình ca tam giác
và tính chất đường trung tuyến ng vi cnh huyn của tam giác vuông, ta được:
;;
22
BP CP
HE IF KF IE HEI IFK= = = = =
T đó HEI = IFK (c.g.c), suy ra
( )
1EIH FKI=
AB AC
nên
ABC ACB
Kết hp vi
PBA PCA=
suy ra
PBC PCB
, mà
PBC CIF=
PCB BIE=
(hai góc đồng v) nên
( )
2CIF BIE
T
CIF ICF BIE=
suy ra
FC FI
. Mà
2
PC
FC FK

==


nên
IF FK
T đó
KIF IKF
hay
( )
3KIF EIH
T (1), (2) và (3) suy ra
KIF CIF BIE EIH hay HIB KIC+ +
Bài 5.
Xét tam giác ABC. Vì
AC AB
nên
ABC ACB
hay
DBM ECM
Vi hai tam giác BDM và CEM có
;CE BD CM BM==
DBM ECM
nên theo kết qu ví d 4 ta có
MD ME
.
Bài 6.
Xét hai tam giác AMB và AMC có
MB MC=
; MA chung, mà
AB AC
nên theo kết qu ví d 4, ta có
AMB AMChayOMB OMC
Bây gi ta xét hai tam giác OMB và OMC có
MB MC=
, OM
chung,
OMB OMC
. Theo kết qu ví d 4 ta được
OB OC
Bài 7.
Trước hết ta s chứng minh D là trung điểm ca EF. Tht vy, K EH và
FK vuông góc với đường thng BC (H, K BC).
CHUYÊN ĐỀ BỒI DƯỠNG HỌC SINH GIỎI HÌNH HỌC 9
35
Ta có BEH = CFK (cnh huyn- góc nhn) nên
EH FK=
Xét hai tam giác EHD và FKD có
90 ; ;EHD FKD EH FK DEH DFK= = = =
(hai góc so le trong)
Suy ra: DEH = DFK (g.c.g)
DE DF=
Cui cùng vi hai tam giác BED và FCD ta thy
;DE DF BE CF==
BED CFD
(góc BED
là góc ngoài ca tam giác AFE) nên
BD CD
(theo kết qu ví d 4).
Bài 8.
Thy rng, vi một điểm M bt k thuc min trong ca tam giác ABC
cân tại A (điểm M có th nm trên các cnh ca tam giác, M không
trùng vi B hoc C) ta luôn có
AM AB
(Bạn đọc t chng minh).
Quay li bài toán.
Xét BAM và CAM có
AB AC=
, AM chung.
BM CM
nên
BAM CAM
. Áp dng kết qu ví d 4 mt ln
na vi ABO và ACO ta được
OB OC
Để chng minh
BOA COA
ta s dng hai tam giác có hai cp cnh
tương ứng bng nhau và có các góc xen gia là
BOA
COA
như sau: Trên cạnh OC lấy điểm N
sao cho
OB ON=
. Rõ ràng N nm min trong tam giác cân ABC nên
AN AB
. Hai tam giác
AOB và AON có
OB ON=
, OA chung,
AB AN
nên
BOA COA
.
Bài 9. Do
AB AC
nên
BD CE
(Tam giác vuông cân có cnh góc
vuông lớn hơn thì cạnh huyn lớn hơn). Ta có EAB = CAD (c.g.c)
nên
BE CD=
. Hai tam giác DCB và EBC có
BE CD=
, BC chung. Vì
BD CE
nên
DCB EBC
hay
DCM EBM
.
Xét hai tam giác DCM và BEM có
;,BE CD MB MC DCM EBM= =
nên
MD ME
.
Nhn xét: Trong bài toán trên, ta v ra phía ngoài tam giác ABC các tam
giác vuông cân ti A. Nếu v ra phía ngoài tam giác ABC các tam giác
đều thì sao? Chúng ta có bài toán tương tự như sau: Cho tam giác ABC có
, 120AB AC A
. V v phía ngoài tam giác ABC các tam giác đều
ABD và ACE. Gọi M là trung điểm ca cnh BC. Chng minh
MD ME
.
Bài 10.
Gọi M là trung điểm ca AC.
CHUYÊN ĐỀ BỒI DƯỠNG HỌC SINH GIỎI HÌNH HỌC 9
36
Ta có
AB BC
, theo kết qu ca ví d 1 áp dụng vào tam giác ABC, đường trung tuyến BM ta
được
( )
1ABM CBM
Cũng từ
AB BC
nên
( )
2BAM BCM
T (1) và (2) suy ra
ABM BAM CBM BCM++
hay
AMD CMD
. Hai tam giác AMD và
CMD có
MA MC=
; cnh chung MD;
AMD CMD
nên theo kết qu ví d 4, ta được
AD CD
.
Bài 11.
AB AC
nên ta có
ABC ACBhay EBC DCB
Trên tia đối ca tia DB lấy điểm F sao cho
DF DB=
; trên tia đối
ca tia EC lấy điểm G sao cho
EG EC=
. Ta có EGB = ECB
(c.g.c) và DFC = DBC (c.g.c) nên
GB BC CF==
.
Mt khác
22GBC EBC DCB FCB= =
.
Áp dng kết qu ví d 4 vi hai tam giác GBC và FCB ta suy ra
BF CG
. T đó
BD CE
.
Nhn xét: T kết qu ca bài toán ta có: Trong mt tam giác đưng
cao ng vi cnh lớn hơn thì nhỏ hơn.
Bài 12.
Trên cnh AB lấy điểm E sao cho
.AE AC=
Ta có AME = AMC
(c.g.c) nên
ME MC=
.
Ta có
AB AC AB AE BE MB ME = =
(Áp dng bất đẳng thc
tam giác vào EMB).
T đó
AB AC MB MC
.
Bài 13.
a) Gi s tia BM ct cnh AC ti D.
Ta có
MC MD CD+
t đó
MB MC MB MD CD BD CD+ + + = +
.
Li có
BD AB AD+
. T đó
MB MC BD CD AB AD CD AB AC+ + + + = +
b) Theo kết qu phn a ta có:
( )
( )
( )
1
2
3
MB MC AB AC
MC MA BC BA
MA MB CA CB
+ +
+ +
+ +
Cộng (1), (2) và (3) ta được
( ) ( )
22MA MB MC AB BC CA+ + + +
. T đó ta có
CHUYÊN ĐỀ BỒI DƯỠNG HỌC SINH GIỎI HÌNH HỌC 9
37
( )
*MA MB MC AB BC CA+ + + +
Mt khác, áp dng bất đẳng thức tam giác vào các tam giác MAB, MAC, MBC ta được
;;MA MB AB MB MC BC MC MA CA+ + +
Do vy
( ) ( )( )
1
2 **
2
+ + + + + + + +MA MB MC AB BC CA hay MA MB MC AB BC CA
T (*) và (**) suy ra
MA MB MC++
lớn hơn nửa chu vi nhưng nhỏ hơn chu vi ABC.
Bài 14.
Gi s ABC các đường trung tuyến AM, BN, CP ct nhau ti
trng tâm G. Theo ví d 1, ta có
2
2
2
AB AC
AM
BA BC
BN
CA CB
CP
+
+
+
Cng các bất đẳng thức trên ta được
( )
1AM BN CP AB BC CA+ + + +
Mt khác, theo bài tp 13 ta có
( )
1
2
GA GB GC AB AC BC+ + + +
hay
( ) ( )
21
.
32
+ + + +AM BN CP AB AC BC
T đó
( )( )
3
2
4
AM BN CP AB AC BC+ + + +
T (1) và (2) suy ra tổng độ dài ba đường trung tuyến ca mt tam giác lớn hơn
3
4
chu vi và nh
hơn chu vi của tam giác đó.
Bài 15.
Gọi M là trung điểm ca AC, phân giác góc A ct cnh BC ti D.
Ta có ABD = AMD (c.g.c) nên
ABD AMD=
.
Li có
90ABD 
nên
90AMD 
.
T đó
AMD CMD
Hai tam giác AMD và CMD có MD là cnh
chung;
;MA MC AMD CMD=
nên
AD DC
.
Xét trong ADC có
AD DC
nên
2

BAC
C DAC hay C
.
CHUYÊN ĐỀ BỒI DƯỠNG HỌC SINH GIỎI HÌNH HỌC 9
38
Bài 16.
ABC vuông ti C và CBH vuông ti H nên
AB CB BH
. Tn tại điểm D thuộc đoạn
thng AH sao cho
BC BD=
. Gi E là hình chiếu ca D trên
AC.
Mt khác vì CBD cân ti B nên
BCD BDC=
.
Li có
90 ; 90BCD ECD BDC HCD+ = + =
.T đó
ECD HCD=
CED = CHD (cnh huyn-góc nhn) nên
;CE CH DE DH==
.
Ta có
AB CH BD DA CH AE BC CE CA CB+ = + + + + = +
.
Nhn xét: T kết qu ca bài toán ta có: Trong mt tam giác vuông, tng hai cnh góc vuông nh
hơn tổng ca cnh huyền và đường cao ng vi cnh huyn y.
Bài 17.
Ta có AB // CE (vì cùng vuông góc vi BC) nên
ABD E=
(hai góc so le trong).
Mt khác do AD là phân giác ca góc BAC nên suy ra
E CAD=
T đó CAE cân ti C.
Ta có
CE CA AB=
nên
( )
22
1CE AB
.
Gi F là hình chiếu ca D trên AC.
Ta có ABD = AFD (cnh huyn - góc nhn) nên
.DB DF=
Ta có
CD DF DB=
nên
( )
22
2CD BD
.
T (1) và (2) suy ra
2 2 2 2 2 2
CE CD AB DB DE DA hayDE DA+ +
.
;;CE AB CD DB DE DA
nên chu vi ECD lớn hơn chu vi ABD.
Bài 18.
AB AC
nên có điểm P trên cnh AB sao cho
AP AC=
. Gọi D là giao điểm ca CN và PM.
AN AM AC AP= =
nên P nm gia N và B.
T đó
BMN PMN
D dàng chứng minh được ∆DMN cân tại D nên
;PMN CNM=
t đó
BMN CNM hayOMN ONM
.
Trong OMN có
OMN ONM
nên
( )
1ON OM
.
CHUYÊN ĐỀ BỒI DƯỠNG HỌC SINH GIỎI HÌNH HỌC 9
39
Ta có APM = CAN (c.g.c) nên
PM CN=
.
APC cân ti A nên
90APC 
T đó
90 90APM hayBPM
.
Trong PBM có
90BPM 
là góc ln nht nên
( )
2BM PM CN=
T (1) và (2) suy ra
BM OM CN ON hayOB OC
.
Bài 19.
Ta xét hai trường hp:
+ Trường hp 1: ABC có
60A=
.
Gi D và E lần lượt là hình chiếu ca B và C lên phân giác góc A ca
ABC.
Các tam giác vuông ABD và AEC lần lượt có các góc nhn
30 ; 30BAD CAE= =
nên
11
;
22
BD AB CE AC==
.
T đó
( )
22AB AC BD CE BC+ = +
.
Dấu “=” xảy ra khi và ch khi ABC đều.
+ Trường hp 2: ABC có
60A=
.
Trên cnh BC lấy điểm B' sao cho
' 60CAB =
.
Theo kết qu trường hp 1 ta có
( )
' 2 ' 1AB AC B C+
Trong ABB' ta có
( )
' ' 2BB' 2AB AB BB
T (1) và (2) suy ra
2AB AC BC+
.
Trường hp này không xy ra dấu “=”.
Tóm li, ta luôn có
2AB AC BC+
.
Dấu “=” xảy ra khi và ch khi ABC đều.
Bài 20.
Qua H k các đường thng song song vi AB và AC, ct AC và AB lần lượt ti D và E.
Ta có ADH = HEA (g.c.g) nên
;AE DH AD EH==
.
Xét AHD có
AH AD DH+
, suy ra
( )
1AH AD AE+
Li có
//EH AC
AC BH
nên
EH BH
.
T đó EBH vuông ti H, suy ra
( )
2BH BE
. Chứng minh tương tự ta được
( )
3CH CD
T (1), (2) và (3) suy ra
( )
4AH BH CH AB AC+ + +
CHUYÊN ĐỀ BỒI DƯỠNG HỌC SINH GIỎI HÌNH HỌC 9
40
Chứng minh tương tự ta được:
( )
( )
5
6
AH BH CH AB BC
AH BH CH AC BC
+ + +
+ + +
T (4), (5) và (6) suy ra
( )
2
3
HA HB HC AB AC BC+ + + +
.
CHUYÊN ĐỀ BỒI DƯỠNG HỌC SINH GIỎI HÌNH HỌC 9
41
Bài toán 4. S dụng định lí thales (ta-lét) và tính chất đường phân giác của tam giác để chng
minh đẳng thc
I. KIN THC CN NH
1. Đon thng t l
Hai đoạn thng
AB
CD
gi là t l với hai đoạn thng
AB


CD
nếu có t l thc:
=
AB CD
A B C D
2. Định lý Thales trong tam giác
Nếu
//MN BC
thì:
;==
AM AN AM AN
MB NC AB AC
.
* M rộng định lý Thales:
Cho
ABCD
là hình thang (
//AB CD
) nếu
//MN AB
thì:
;==
AM BN AM BN
MB NC AD BC
.
3. Định lý Thales đảo
Nếu
,MN
là hai điểm lần lượt trên hai cnh
, ACAB
sao cho
=
AM AN
MB NC
thì
//MN BC
.
4. H qu định lý Thales
Nếu một đường thng ct hai cnh ca mt tam giác và song song
vi mt cnh còn li thì nó to thành mt tam giác mi có ba cnh
tương ứng t l vi ba cnh ca một tam giác đã cho.
(Chú ý: H qu này vẫn đúng trong trường hợp đường thẳng đã cho cắt phn kéo dài ca hai cnh
kia).
5. Tính chất đường phân giác ca tam giác
Trong tam giác, đường phân giác ca mt góc chia
cạnh đối diện thành hai đoạn thng t l vi hai cnh
k hai đoạn y.
Chú ý: Tính cht vẫn đúng với tia phân giác góc
ngoài ca tam giác. Cho
ABC
,
AD
AE
ln
ợt là các đường phân giác trong và phân giác ngoài
tại đỉnh
A
ca tam giác.
Khi đó:
==
DB EB AB
DC EC AC
.
B sung: Mt s tính cht ca t l thc
CHUYÊN ĐỀ BỒI DƯỠNG HỌC SINH GIỎI HÌNH HỌC 9
42
Vi
, , ,a b c d
khác
0
. Nếu
=
ac
bd
thì ta có các h thc sau:
=ad bc
=
ab
cd

=
a b c d
bd
,
=

ac
a b c d
==
a c a c
b d b d
(Gi s các t s đều có nghĩa)
II. CÁC VÍ D
Ví d 1. Cho hình bình hành
ABCD
. T điểm
C
k đường thng cắt tia đối ca tia
DA
và tia đối
ca tia
BA
lần lượt tại điểm
E
và điểm
F
. Trên cnh
DC
lấy điểm
K
sao cho
=DK BF
. Gi
giao điểm ca
AK
EF
M
. Chng minh
=EM MF
.
NG DN GII
T điểm
M
k đường thng song song với đường thng
AF
và cắt đường thng
AE
tại điểm
N
.
Áp dụng định lý Thales ta có:
.==
EN NM NM DK
AE AF DK AF
(1)
=
NM AN
DK AD
(2) do
//DK MN
= = =
DK BF BC AD
AF AF AE AE
(3)
T (1), (2) và (3) ta có:
.= = =
EN AN AD AN
EN AN
AE AD AE AE
.
Theo định lý đường trung bình ca tam giác thì
=EM MF
.
Ví d 2. Cho tam giác
ABC
, lấy các điểm
,MN
lần lượt trên hai cnh
,AB AC
sao cho
//MN BC
,
BN
ct
CM
tại điểm
I
,
AI
ct
BC
tại điểm D. Chng minh
=BD DC
.
NG DN GII
T điểm
A
, k đường thng
d
song song vi
BC
ct
tia
BI
và tia
CI
lần lượt tại điểm
E
và điểm
F
(hình v).
Áp dng h qu định lý Thales ta có:
= = = =
AF AM AN AE
AF AE
BC MB NC BC
(1)
CHUYÊN ĐỀ BỒI DƯỠNG HỌC SINH GIỎI HÌNH HỌC 9
43
//EF BC
, theo định lý Thales ta có:
==
AF AI AE
DC ID BD
(2)
T (1) và (2) suy ra
=BD DC
.
Ví d 3. (B đề hình thang) Cho hình thang
ABCD
( )
//AB CD
. Hai cnh bên
AD
BC
ct
nhau điểm
E
. Hai đường chéo
AC
BD
ct nhau tại điểm
O
. Gi
F
là giao điểm ca
AB
EO
. Chng minh
=AF BF
.
NG DN GII
Kéo dài tia
EO
ct cnh
DC
tại điểm
M
.
Áp dng h qu định lý Thales ta có:

==


AF BF EF
DM CM EM
(1)

==


AF BF FO
CM DM OM
(2)
Nhân vế vi vế của đẳng thức (1) và (2) ta được:
22
..
= =
AF BF
AF BF
DM CM DM CM
.
Ví d 4. Cho tam giác
ABC
có trng tâm là
G
, t
G
k đường thng cắt tia đối tia
CB
tại điểm
D
và ct hai cnh
AC
,
AB
lần lượt tại điểm
E
và điểm
F
. Chng minh
1 1 1
+=
GD GE GF
.
NG DN GII
K trung tuyến
AM
đi qua
G
. T điểm
B
, điểm
C
k các đường thng song song vi
DF
ct tia
AM
lần lượt tại điểm
K
và điểm
H
.
Do
= BMK CMH
(g.c.g) nên
=BK HC
=HM MK
.
S dng tính cht trng tâm ca tam giác và định lý Thales
ta có:
2. 2.= = =
GA GM HM HK
GD GD HC BK
(1)
Do
//GE HC
nên
==
GA AH AH
GE HC BK
(2)
Do
/ /BGF K
nên
=
GA AK
GF BK
(3)
T (1), (2) và (3) suy ra
1 1 1
= + = +
GA GA GA
GF GD GE GF GD GE
.
Ví d 5. Cho hình thang vuông
ABCD
( )
//AB CD
90=A
. Gi
M
là trung điểm ca
AB
, tia
CHUYÊN ĐỀ BỒI DƯỠNG HỌC SINH GIỎI HÌNH HỌC 9
44
CM
ct
AD
ti
K
sao cho
90=DBK
. Chng minh
=CB CD
.
NG DN GII
Lấy điểm
N
là trung điểm của đoạn
BD
.
Kéo dài
MN
ct
BK
tại điểm
H
.
Áp dụng định lý Thales vi
//AB CD
=
KM KA
MC AD
(1)
Áp dụng định lý Thales vi
//HN KD
=
HM KA
MN AD
(2)

=


BM
BA
T (1) và (2) suy ra
//=
KM HM
KH CN
MC MN
nh lý Thales
đảo).
Do đó
CN BD CBD
cân ti
=C CB CD
.
Ví d 6. Cho tam giác
ABC
AB AC
, đường phân giác
AD
, đường trung tuyến
AM
. Trên
cnh
AC
lấy điểm
E
sao cho
=AE AB
. Đường thng
BE
ct
AD
AM
lần lượt tại điểm
H
và điểm
F
, đường thng
HM
ct
DF
tại điểm
I
. Chng minh
=DI IF
.
NG DN GII
T điểm
M
k đường thng song song vi
AB
ct
AD
,
BE
lần lượt tại điểm
K
và điểm
P
. Tia
MH
ct
AB
tại điểm
N
.
Do tam giác
ABE
cân ti
A
AH
là phân giác nên
=BH HE
S dng tính chất đường trung bình ca tam giác: do
//MH EC
nên
=NA NB
.
Xét hình thang
ABKP
(
//AB KP
).
Ta thy
=NA NB
nên
=MK MP
(theo b đề hình thang).
Do
//PK AB
, s dng h qu của định lý Thales ta có:
=
MD MK
DB AB
=
MP MF
AB FA
= =
MD MF
MK MP
DB FA
.
Áp dụng định lý Thales đảo suy ra
//AB DF
.
Xét hình thang
ABDF
(
//AB DF
) ta có
N
là trung điểm ca
AB
nên
I
là trung điểm ca
DF
. Vy
=DI IF
.
Ví d 7. Cho tam giác
ABC
vuông cân ti
A
. Trên cnh
AB
lấy điểm
E
. Tia
CE
cắt đường
thng vuông góc vi
AB
ti
B
K
. T
E
k đường thng song song vi
BC
ct
BK
tại điểm
F
. Đường thng
AF
cắt đường thng
CK
tại điểm
G
, đường thng
BG
ct
AC
tại điểm
D
.
CHUYÊN ĐỀ BỒI DƯỠNG HỌC SINH GIỎI HÌNH HỌC 9
45
Gi
DE
ct
BK
tại điểm
H
. Chng minh
=AC BH
.
NG DN GII
//BK AC
, theo h qu định lý Thales ta có:
=
AD FK
AC FB
.
//DC KH
, theo h qu định lý Thales ta có:
=
AD KB
AC BH
.
T đó suy ra
=
FK KB
FB BH
. (1)
Do
//EF BC
nên ta có
==
KF KE KB
FB EC AC
(2)
T (1) và (2) suy ra
= =
KB KB
AC BH
BH AC
.
Ví d 8. Cho hình thang
ABCD
( )
//AB CD
. K đường thng song song với đường thng
AB
ct
các cnh
, , ,AD BD AC BC
lần lượt tại các điểm
, , ,I E F K
sao cho
==IE EF FK
. Gi s đường
thng
BF
cắt đáy
DC
tại điểm
M
. Chng minh rng
=DM MC
và ba điểm
,,A E M
thng
hàng.
NG DN GII
Áp dng h qu định lý Thales khi
//EK DC
:

==


EF FK BF
DM MC BM
Do
=EF FK
nên
=DM MC
hay
M
là trung điểm ca
DC
.
Gi s
AE
ct
DC
ti
M
tương tự ta chứng minh được
M
là trung điểm ca
DC
.
Suy ra
MM
.
Vậy, ba điểm
,,A E M
thng hàng.
Ví d 9. Cho hình bình hành
ABCD
. Gi
K
là điểm thuc cnh
BC
. Đường thng
AK
cắt đường
chéo
BD
, cắt đường thng
DC
ti
G
. Chng minh
2
.=AE EK EG
NG DN GII
//AD BK
, áp dng h qu của định lý Thales ta
được:
=
AE DE
EK EB
(1)
Vì
//AB DG
, áp dng h qu định lý Thales ta có:
=
EG DE
AE EB
(2)
CHUYÊN ĐỀ BỒI DƯỠNG HỌC SINH GIỎI HÌNH HỌC 9
46
T (1) và (2) suy ra
2
.= =
AE EG
AE EK EG
EK AE
.
Ví d 10. Cho tam giác
ABC
AB AC
, đường phân giác
AD
. Lấy điểm
I
thuc cnh
BC
sao cho
2=BI IC
. T điểm
I
k đường thng song song vi
AD
ct
,AB AC
lần lượt tại điểm
K
và điểm
E
. Chng minh
2=BK CE
.
NG DN GII
Áp dng tính chất đường phân giác ca tam giác ta có:
=
AC AB
CD BD
(1)
//EI AD
, áp dụng định lý Thales ta có:
=
AC CE
CD CI
(2)
T (1) và (2) suy ra
=
AB CE
BD CI
(3)
//AD KI
, theo định lý Thales ta có:
=
AB BK
BD BI
(4)
T (3) và (4) suy ra
=
CE BK
CI BI
2=BI IC
nên
2=BK CE
.
Ví d 11. Cho tam giác
ABC
, trên cnh
AB
lấy điểm
M
, trên tia đối ca tia
CA
lấy điểm
N
sao
cho
=BM CN
. Đường thng
MN
cắt đường thng
BC
tại điểm
D
. Chng minh
=
AB DN
AC DM
.
NG DN GII
//ME AC
, áp dng h qu định lý Thales, ta có:
=
AB BM
AC ME
(1)
Do
//ME CN
, áp dng h qu của định lý Thales ta
có:
=
DN CN
DM ME
(2)
Mà t gi thiết ta có
=BM CN
(3)
T (1), (2) và (3) ta có:
=
AB DN
AC DM
.
Ví d 12. Cho tam giác
ABC
cân ti
A
. V các đường phân giác
BD
CE
. Chng minh rng
1 1 1
=+
DE BC AC
.
NG DN GII
Theo tính chất đường phân giác ca tam giác ta có:
;==
AD BA AE CA
DC BC EB BC
.
CHUYÊN ĐỀ BỒI DƯỠNG HỌC SINH GIỎI HÌNH HỌC 9
47
=AB AC
(
ABC
cân ti
A
) nên
=
AD AE
DC EB
. Vy
//DE BC
Theo h qu định lý Thalès ta có:
1
+
= = = +
BC AC AD DC DC
DE AD AD AD
Vy
11
.
=+
DC
DE BC AD BC
1 1 1
.
= + = +
BC
BC AB BC BC AB
.
III. BÀI TP
Bài 1. Cho hình thoi
ABCD
có cnh là
a
. Một đường thẳng qua điểm
C
ct
,AB AD
lần lượt ti
điểm
E
và điểm
F
. Chng minh
1 1 1
+=
AE AF a
Bài 2. Cho hình thang
ABCD
( )
//AB CD
. Trên tia đối ca tia
BA
lấy điểm
E
sao cho
=BE CD
. Gọi giao điểm ca
AC
vi
BD
DE
theo th t là điểm
I
và điểm
K
. Chng minh
=
AK AC
KC CI
.
Bài 3. Cho tam giác
ABC
AC AB
. Trên hai cnh
AB
AC
lấy hai điểm
D
E
sao
cho
=BD CE
. Gi
K
là giao điểm
DE
BC
. Chng minh
=
AB KE
AC KD
.
Bài 4. Gi
O
là giao điểm của các đường thng cha hai cnh bên ca hình thang
ABCD
, đường
thẳng đi qua
O
song song với đáy
AB
ct
,AC BD
theo th t
M
N
. Chng minh
=OM ON
.
Bài 5. Cho hình thang
ABCD
( )
//AB CD
. Một đường thng
d
song song với hai đáy cắt hai cnh
bên
AD
BC
theo th t các điểm
,MN
và cắt hai đường chéo
BD
AC
điểm
H
điểm
K
. Chng minh
=MH KN
.
Bài 6. Cho tam giác
ABC
, đường trung tuyến
AM
. Gi
I
là điểm bt kì trên cnh
BC
. Đường
thẳng đi qua
I
và song song vi
AC
ct
AB
tại điểm
K
. Đường thẳng đi qua
I
và song song vi
AB
ct
,AM AC
theo th t
D
E
. Chng minh rng
=DE BK
.
Bài 7. Cho tam giác
ABC
vuông ti
A
, v ra ngoài tam giác đó tam giác
ABD
vuông cân ti
B
,
tam giác
ACF
vuông cân ti
C
. Gi
H
là giao điểm ca
AB
CD
. Gi
K
là giao điểm ca
AC
BE
. Chng minh
=AH AK
.
CHUYÊN ĐỀ BỒI DƯỠNG HỌC SINH GIỎI HÌNH HỌC 9
48
Bài 8. Cho tam giác
ABC
. K một đường thng ct các cnh
,BC AC
theo th t điểm
D
, điểm
E
và cắt đường thng
BA
điểm
F
. V hình bình hành
BDEH
. Đường thẳng đi qua điểm
F
song song vi
BC
ct tia
AH
tại điểm
I
. Chng minh
=FI DC
.
Bài 9. Cho hình bình hành
ABCD
, gi
M
là điểm thuc cnh
BC
,
N
là điểm thuộc tia đối ca tia
BC
sao cho
=BN CM
. Các đường thng
DN
DM
cắt đường thng
AB
lần lượt tại điểm
E
và điểm
F
. Chng minh
2
.E=AE EB F
.
Bài 10. Cho hình thang
ABCD
( )
//AB CD
. Gi
I
là giao điểm của hai đường chéo
AC
BD
.
K
là giao điểm ca
AD
BC
. Đường thng
IK
ct
AB
CD
theo th t
N
M
.
a) Chng minh
=NA NB
,
=MC MD
.
b) Đường thng qua
I
song song với hai đáy của hình thang
ABCD
ct
AD
BC
theo th t
E
F
. Chng minh rng
2 1 1
=+
EF AB CD
Bài 11. Cho tam giác
ABC
cân ti
A
. V các đường phân giác
BD
CE
. Chng minh rng
1 1 1
=+
DE BC AC
.
Bài 12. Cho tam giác
ABC
vuông ti
A
, đường phân giác
AD
. Gi
M
N
theo th t là hình
chiếu ca
B
C
trên
AD
. Chng minh rng
2 +AD BM CN
.
Bài 13. Cho tam giác
ABC
( )
AB AC
. Gi
D
là trung điểm ca
BC
.
AE
AF
lần lượt là
phân giác trong và phân giác ngoài đỉnh
A
ca tam giác
ABC
. Gi
H
là hình chiếu vuông góc
ca
A
trên
BC
. Chng minh rng
..=EF DH AB AC
Bài 15. Cho hai điểm
A
B
nm cùng na mt phng b là đường thng
xy
. Gi
H
K
ln
t là hình chiếu ca
A
B
trên đường thng
xy
. Gọi giao điểm ca
AK
BH
O
, gi
I
là hình chiếu của điểm
O
trên đường thng
xy
. Chng minh rng
( )
.+=OI AH BK AH BK
.
Bài 16. Cho ba điểm
,,
A B C
lần lượt nằm trên ba đường thng cha ba cnh
,,BC CA AB
ca tam
giác
ABC
sao cho chúng không có điểm nào hoặc có đúng hai điểm nm trên hai cnh ca tam
giác. Khi đó
,,
A B C
thng
hàng khi và ch khi
. . 1
=
A B B C C A
A C B A C B
(Định lý Melelaus)
Bài 17. Cho ba điểm
,,
A B C
thuc ba cnh
,,BC CA AB
ca tam giác
ABC
. Khi đó
,,
AA BB CC
đồng quy khi và ch khi
. . 1
=
A B B C C A
A C B A C B
(Định lý Ceva)
CHUYÊN ĐỀ BỒI DƯỠNG HỌC SINH GIỎI HÌNH HỌC 9
49
Bài 18. Cho tam giác đều
ABC
. Trên các cnh
,AC AB
lần lượt lấy hai điểm
D
E
sao cho
15
2
+
==
CD AE
DA EB
. Các đường thng
BD
CE
ct nhau ti
O
. Trên đoạn thng
BD
CE
lần lượt lấy hai điểm
M
N
sao cho
MN
song song vi
AC
. Chng minh
2BN OM
.
IV. HƯỚNG DN GII
Bài 1.
Do
//DC AE
, áp dng h qu định lý Thales ta có:
=
DC CF
AE EF
(1)
Do
/ /ABC F
, áp dng h qu định lý Thales ta có:
=
BC CE
AF EF
(2)
T (1) và (2) suy ra
1 1 1
1+ = + = + = + =
DC BC a a CF CE
AE AF AE AF EF EF AE AF a
.
Bài 2.
Áp dng h qu định lý Thales:
=
AK AE
KC DC
(1)
Áp dng h qu định lý Thales ta có:
++
= = =
AI AB AC AI CI AB DC
CI DC CI CI DC
+
==
AB BE AE
DC DC
(2)
T (1) và (2) suy ra
=
AK AC
KC CI
.
Bài 3.
T điểm
D
k đường thng song song vi
AC
cắt đường thng
BC
tại điểm
H
. Ta có
//DH AC
, áp dng h qu định lý Thales ta
có:
==
AB DB EC
AC DH DH
(1)
Do
//EC DH
, áp dng h qu định lý Thales
ta có:
=
KE EC
KD DH
(2)
CHUYÊN ĐỀ BỒI DƯỠNG HỌC SINH GIỎI HÌNH HỌC 9
50
T (1) và (2) suy ra
=
AB KE
AC KD
.
Bài 4.
Áp dng h qu định lý Thales vi
( )
//OM DC
ta có:
=
OM OA
DC AD
(1)
Áp dng h qu định lý Thales vi
( )
//ON DC
ta có:
=
ON OB
DC BC
(2)
Áp dụng định lý Thales cho
ODC
( )
//AB DC
ta có:
=
OA OB
AD BC
(3)
T (1), (2) và (3) ta có:
= =
OM ON
OM ON
DC DC
.
Bài 5.
S dng h qu ca định lý Thales ta có:
=
MH MD
AB AD
(1)
=
KN NC
AB BC
(2)
Áp dụng định lý Thales m rng cho hình thang ABCD
ta được:
=
MD NC
AD CB
(3)
T (1), (2) và (3) suy ra:
= =
MH KN
MH KN
AB AB
.
Bài 6.
Lấy điểm
N
trên tia đối ca tia
MA
sao cho
=MA MN
. Suy ra
ABNC
là hình bình hành.
=AB CN
(*).
Áp dng h qu của định lý Thales ta có các h thc sau:
=
BK KI
AB AC
(1)
=
DE AE
CN AC
(2)
=KI AE
, kết hp vi (1) và (2) suy ra
=
BK DE
AB CN
T (*) ta có
=AB CN
nên
=BK DE
.
Bài 7.
Đặt
==AB BD c
CHUYÊN ĐỀ BỒI DƯỠNG HỌC SINH GIỎI HÌNH HỌC 9
51
==AC CE b
Áp dng h qu định lý Thales ta có
= = =
++
AH AC b AH b
HB BD c AH HB c b
= =
++
AH b bc
AH
c c b c b
(1)
Áp dng h qu định lý Thales ta có
= = =
++
AK AB c AK c
CK CE b AK CK c b
= =
++
AK c bc
AK
b c b c b
(2)
T (1) và (2) suy ra:
=AH AK
.
Bài 8.
Kéo dài đường thng FI cắt đường thng AC tại điểm K. Kéo dài đường thng EH cắt đường thng
AB tại điểm M. Áp dụng định lý Thales vi
//ME FK
ta được:
=
FI MH
FK ME
(1)
//BH EF
nên
MH BH ED
ME EF EF
==
(2)
Do
//DC FK
, áp dụng định lý Thales ta được:
ED DC
EF FK
=
(3)
Kết hợp (1), (2) và (3) ta được:
FI DC
FI DC
FK FK
= =
.
Bài 9.
Nối điểm
A
với điểm
N
.
Ta có
//AD MN
AD MN=
nên t giác
ADMN
là hình bình hành.
Do
//AN DM
, áp dng h qu định lý Thales ta có:
AE EN
EF ED
=
(1)
Do
//AD BN
, áp dng h qu ca định lý Thales ta
có:
EN EB
ED AE
=
(2)
T (1) và (2) suy ra
2
.
AE EB
AE EB EF
EF AE
= =
.
Bài 10.
a) Vì
/ / , / / , / / , / /AN DM NB DM AN MC NB MC
nên theo h qu định lý Thalès ta có:
CHUYÊN ĐỀ BỒI DƯỠNG HỌC SINH GIỎI HÌNH HỌC 9
52
==
AN KN BN
DM KM CM
(1)
AN IN BN
CM IM DM
==
(2)
Do đó
..
AN AN BN BN
DM CM DM CM
=
hay
22
AN BN=
Vy
AN BN=
. Kết hp vi (1) ta có
DM CM=
.
b) Vì
/ / , / / , / /EI DC IF DC AB CD
nên theo h qu định lý Thalès
ta có:
EI AI BI IF
DC AC BD DC
= = =
(*). Vy
IE EF=
Li có
//EI AB
nên
EI DI
AB DB
=
. Kết hp với (*) ta được:
1
EI EI BI CI BD
AB CD BD BD BD
+ = + = =
. Hay
1 1 1
EI AB CD
=+
Để ý rng vì
,IE IF IE IF EF= + =
nên
12
EI EF
=
T đó
2 1 1
=+
EF AB CD
Ví d trên cho ta một phương pháp chứng minh hai đoạn thng bng nhau.
Để chng minh
xz=
, ta chng minh t l thc
xz
yt
=
.
Nếu có
yt=
, ta s có điều phi chng minh.
Bài 11.
Theo tính chất đường phân giác ca tam giác ta có:
;
AD BA AE CA
DC BC EB BC
==
.
AB AC=
(
ABC
cân ti
A
) nên
AD AE
DC EB
=
. Vy
//DE BC
.
Theo h qu định lý Thalès ta có:
1
BC AC AD DC DC
DE AD AD AD
+
= = = +
.
Vy
1 1 1 1 1
..
DC BC
DE BC AD BC BC AB BC BC AB
= + = + = +
.
Bài 12.
Không mt tính tng quát, gi s
AB AC
.
CHUYÊN ĐỀ BỒI DƯỠNG HỌC SINH GIỎI HÌNH HỌC 9
53
AD
là tia phân giác ca
ABC
nên ta có
1
DB AB
DC AC
=
T đó
DB DC
.
Li có các tam giác
AMB
ANC
vuông cân nên
;BM MA CN NA==
.
Ta có
BM CN AM AN+ = +
( )
2AD DN DM= +
(1)
//BM CN
(cùng vuông góc vi
AD
) nên theo định lý Thales
ta có
1
DM DB
DN DC
=
, t đó suy ra
DM DN
hay
0DN DM−
(2).
T (1) và (2) suy ra
2BM CN AD+
.
Dấu “=” xảy ra khi và ch khi
ABC
vuông cân ti
A
.
Bài 13.
Không mt tính tng quát, gi s
AB AC
. Vì
AE
AF
lần lượt là phân giác trong và ngoài
ca tam giác
ABC
nên ta có:
EB FB AB
EC FC AC
==
.
Suy ra
EB AB
EB EC AB AC
=
++
hay
EB AB
BC AB AC
=
+
.
T đó
.
=
+
AB BC
EB
AB AC
.
Mt khác
FB AB
FC FB AC AB
=
−−
suy ra
.FB AB AB BC
FB
BC AC AB AC AB
= =
−−
Ta có
22
. . 2 . .AB BC AB BC AB BC AC
EF EB FB
AB AC AC AB AC AB
= + = + =
+
(1)
Áp dụng định lý Pythagore vào các tam giác vuông ABH và ACH ta được
2
2 2 2 2
2

= + = + +


BC
AC AH CH AH DH
2
2 2 2 2
2
BC
AB AH BH AH DH

= + = +


CHUYÊN ĐỀ BỒI DƯỠNG HỌC SINH GIỎI HÌNH HỌC 9
54
T đó
22
2.AC AB BC DH−=
hay
22
2
AC AB
DH
BC
=
(2)
T (1) và (2) suy ra
22
22
2 . .
. . .
2
AB BC AC AC AB
EF DH AB AC
AC AB BC
==
.
Bài 15.
Ta có
/ / / /AH OI BK
(vì cùng vuông góc vi
xy
). Theo
h h qu định lý Thalès ta có
;
OI IK OI HI
AH KH BK KH
==
.
Vy
1
OI OI IK HI KH
AH BK KH KH KH
+ = + = =
Hay
11
.1OI
AH BK

+=


T đó suy ra
( )
.OI AH BK AH BK+=
.
Bài 16.
Hình v cho 1 điểm nm trên phần kéo dài, trường hp còn lại làm tương tự.
* Điều kin cn:
Gi s ba điểm
;AB

C
thng hàng, t điểm
B
k
//BM AC
(
M AB

).
Áp dụng định lý Thales ta có:
AB AB
B C B C

=
,
CA B C
A B BM

=
,
BC BM
C A AB
=

.
. . . . 1
AB CA BC AB B C BM
B C A B C A B C BM AB
= =
* Điều kiện đủ:
Gi s:
. . 1
AB CA BC
B C A B C A
=
(1)
A B AB C
=
Ta có:
. . 1
AB CA BC
B C A B C A
=
(2)
T (1) và (2) suy ra:
BC BC
C A C A
=
CHUYÊN ĐỀ BỒI DƯỠNG HỌC SINH GIỎI HÌNH HỌC 9
55
,,
C C A B C
là ba điểm thng hàng.
Bài 17.
* Điều kin cn: K qua điểm A đường thng song song vi BC cắt BB’ tại điểm M cắt CC’ tại
điểm N.
Theo h qu và định lý Thales:
AC AN
C B BC
=
(1);
BA A I A C BA AM
AM AI AN A C AN
= = =
(2);
CB BC
B A AM
=
(3)
T (1), (2), (3) ta có:
. . . . 1
AC BA CB AN AM BC
C B A C B A BC AN AM
==
* Điều kiện đủ:
Gi s 3 điểm
;AB

C
được ly lần lượt trên ba cnh
,,BC CA AB
như hình thỏa mãn
. . 1
AC BA CB
C B A C B A
=
(1)

=AA BB I
. K CI ct AB ti
C

.
Khi đó
. . 1
AC BA CB
C B A C B A
=
(2)
T (1) và (2) suy ra
AC AC
CC
C B C B
=
.
Vậy ba đường thng
;AA BB

CC
đồng quy.
Bài 18.
Áp dụng định lý Menalaus vi
ABD
và ba điểm thng hàng
,,E O C
ta được:
. . 1=
EA OB CD
EB OD CA
(*)
Theo bài ra
15
2
CD EA
DA EB
+
==
15
35
CD
CA
+
=
+
.
Thay các kết qu trên vào h thức (*) ta được
1 5 1 5
. . 1
2
35
OB
OD
++
=
+
CHUYÊN ĐỀ BỒI DƯỠNG HỌC SINH GIỎI HÌNH HỌC 9
56
1
OB
OB
=
hay
OB OD=
(1)
T N k đường thng song song vi BC, ct BD ti P.
//MN BC
nên ta có
OP ON OM
OB OC OD
==
(2).
T (1) và (2) suy ra
OM OP=
.
Ta có
60 ; 60= = = PMN PDC PNM BCA
nên
PMN PNM
.
T đó
2PN PM OM=
.
Mt khác
180 180 180 120BPN MPN DBC ABC= = =
nên
2BN PN OM
.
CHUYÊN ĐỀ BỒI DƯỠNG HỌC SINH GIỎI HÌNH HỌC 9
57
Bài toán 5.s dụng phương pháp diện tích để chứng minh đng thc và bất đẳng thc
I. MT S KIN THỨC CƠ BẢN
a) Công thc tính din tích tam giác:
Trong đó S là diện tích tam giác ABC, a là độ dài cạnh tương ứng đỉnh A,
h là chiều cao tương ứng k t đỉnh A.
b) Công thc khác tính din tích tam giác:
Công thc 1:
Trong đó: a, c là hai cạnh k ca góc B.
Tht vy: Ta có:
.sinh c B=
thay vào công thc
1
2
S ha=
Ta được
1
sin
2
S ac B=
.
Công thc 2:
Trong đó p là nửa chu vi tam giác ABC, r là bán kính đường tròn ni
tiếp tam giác ABC.
Tht vy:
OBC OAC OAB
S S S S= + +
1 1 1
2 2 2
ra rb rc= + +
( )
1
2
S r a b c pr= + + =
.
Công thc 3:
Trong đó p là nửa chu vi tam giác ABC,
a
r
là bán kính đường tròn
bàng tiếp đỉnh A ca tam giác ABC.
Tht vy,
AEIF BDIE DCFI
S S S S=
1
2
S ha=
1
sin (1)
2
S ac B=
(2)S pr=
( )
(3)
a
S p a r=−
CHUYÊN ĐỀ BỒI DƯỠNG HỌC SINH GIỎI HÌNH HỌC 9
58
( ) ( )
11
22
22
aa
S r c BE b CF r BE CF= + + + +
22
aa
b c a a b c
S r a r
+ + +
= =
( )
a
p a r=−
Công thc 4:
Trong đó a, b, c là ba cạnh của tam giác, R là bán kính đường tròn ngoi tiếp tam giác ABC.
Tht vy, Áp dụng định lý hàm sin ta có:
2 sin
sin sin sin 2
a b c b
RB
A B C R
= = = =
Thay vào công thc tính din tích tam giác
1
sin
2
S ac B=
ta được:
1
.
2 2 4
b abc
S ac
RR
==
Công thc 5:
(công thc Hê-rông)
Trong đó a, b, c là ba cạnh ca tam giác, p là na chu vi tam giác
ABC.
Tht vy: Trong tam giác ABC luôn tn ti một đỉnh mà chân đường
cao h t đỉnh đó thuộc cạnh đối din. Không mt tính tng quát, gi
s đỉnh đó là A. Gọi
AH h=
,
BH x=
, khi đó
CH a x=−
. (vi
0 xa
). Áp dụng định lý Pythagore cho hai tam giác vuông:
( )
2 2 2
2 2 2
2
22
(*)
2
h x c
a b c
x
a
h a x b
+=
−+
=
+ =
.
Thay vào (*) ta được:
( )( )( )( )
2
2 2 2
2 2 2
2
24
a b c a c b b a c b a c
a b c
h c h
aa
+ + + + +

−+
+ = =


.
2
abc
p
++
=
nên
2 ( )( )( )p p a p b p c
h
a
=
thay vào công thc
1
2
S ha=
. Ta được z.
c) Tính chất cơ bản v din tích ca tam giác.
Tính cht 1: Nếu hai tam giác có cùng đáy thì tỉ s din tích bng t s hai đường cao.
Chng minh:
(4)
4
abc
S
R
=
( )( )( ) (5)S p p a p b p c=
CHUYÊN ĐỀ BỒI DƯỠNG HỌC SINH GIỎI HÌNH HỌC 9
59
1
.
2
1
.
2
ABC
DBC
AE BC
S
AE
S DF
DF BC
==
.
Tính cht 2: Nếu hai tam giác có cùng đường cao thì t s din tích bng t s hai
đáy.
Chng minh:
1
.
2
1
.
2
ADC
ABD
AE DC
S
DC
S BD
AE BD
==
.
H qu: Đưng trung tuyến ca tam giác chia tam giác thành hai phn có din tích bng nhau.
Tính cht 3: Cho tam giác ABC, gọi điểm D và điểm E là các điểm
thuộc đường thng AB và AC.
Khi đó
.
ABC
ADE
S
AD AE
S AB AC
=
.
Chng minh:
1
.
2
1
.
2
ABC
ADE
AD EH
S
S
AB CK
=
EH AE
CK AC
=
nh lý Thales)
Vì thế
.
ABC
ADE
S
AD AE
S AB AC
=
Tính cht 4: T s diện tích hai tam giác đồng dng bằng bình phương tỉ s đồng dng.
Chng minh:
Gi s
ABC DEF
theo t s k
Nghĩa là
BC
k
EF
=
AH
k
DK
=
Ta có:
2
1
.
2
1
.
2
ABC
DEF
AH BC
S
k
S
DK EF
==
Tính cht 5: Cho t giác ABCD và điểm O là giao điểm của hai đường chéo AC và BD. Khi đó,
//
ADO BCO
AB CD S S=
Chng minh:
CHUYÊN ĐỀ BỒI DƯỠNG HỌC SINH GIỎI HÌNH HỌC 9
60
• Nếu
//AB CD
, v
;AH DC BK DC⊥⊥
Thì
ADC BDC
AH BK S S= =
ADO DOC BCO DOC
S S S S + = +
ADO BCO
SS=
.
• Nếu
11
.DC .
22
ADO BCO
S S AH BK DC AH BK= = =
//AH BK
nên ABHK là hình bình
hành. Vy
//AB CD
.
Tính cht 6: Nếu hai cnh bên ca hình thang ABCD (
//AB CD
) ct
nhau tại E, hai đường chéo ct nhau ti O thì
AOE BOE
SS=
.
Chng minh:
Gọi F là giao điểm ca EO vi AB. S dng b đề hình thang thì F là
trung đim ca AB.
S dng tính cht 2 thì:
;
AEF BEF AOF BOF AEO BOE
S S S S S S= = =
Công thc tính din tích ca t giác.
- Din tích t giác bng nửa tích hai đường chéo nhân vi góc to bởi hai đường chéo.
- Nếu t giác có hai đường chéo vuông góc thì din tích t giác bng nửa tích hai đường chéo.
1. ng dng ca din tích trong vic gii toán
a) Chứng minh các đẳng thc hình hc
Ví d 1. (Tính chất đường phân giác). Cho tam giác ABC, AD là đường phân giác trong. Chng
minh rng
DB AB
DC AC
=
.
NG DN GII
Ta có
1
.
2
1
.
2
ABD
ACD
DE AB
S
AB
S AC
DF AC
==
(1)
(vì
DE DF=
).
Li có
ABD
ACD
S
DB
S DC
=
(2)
(vì hai tam giác ABD và ACD có chung đường cao).
T (1) và (2) suy ra
DB AB
DC AC
=
.■
Ví d 2. Cho tam giác ABC nhn ni tiếp
()O
. Các đường cao AD, BE, CF ct nhau ti H. Chng
CHUYÊN ĐỀ BỒI DƯỠNG HỌC SINH GIỎI HÌNH HỌC 9
61
minh rng
DEF
ABC
P
r
PR
=
, trong đó r, R lần lượt là bán kính đường tròn ni tiếp và ngoi tiếp tam giác
ABC.
NG DN GII
Ta chứng minh được
OA EF
,
OB DF
,
OC DE
.
Vì tam giác ABC nhn nên O nm bên trong tam giác, t đó ta có:
ABC OEAF ODBF ODCE
S S S S= + +
1 1 1 1
. . . .
2 2 2 2
ABC
r P DE OA DF OB CE OC = + +
..
DEF
ABC DEF
ABC
P
r
r P R P
PR
= =
.■
Ví d 3. Cho tam giác ABC, M là điểm nm trong tam giác AM, BM và CM ct các cnh BC, AC
và AB lần lượt ti D, E, F. Chng minh rng
1
MD ME MF
AD BE CF
+ + =
.
NG DN GII
CHUYÊN ĐỀ BỒI DƯỠNG HỌC SINH GIỎI HÌNH HỌC 9
62
Ta có
MD AH
AD MI
=
.
Áp dng tính cht v t s din tích ta có:
CMD BMD CMD MBC
BMD
BAD CAD BAD CAD ABC
S S S S
S
MD
AD S S S S S
+
= = = =
+
(1)
Chứng minh tương tự ta cũng có
MAC
ABC
S
ME
BE S
=
(2),
MAB
ABC
S
MF
CF S
=
(3).
Cộng (1), (2) và (3) ta có điều cn chứng minh.■
Ví d 4. (Định lý Carnot) Cho tam giác ABC nhn có
,
,
a b c
d d d
lần lượt là khong cách t tâm
đường tròn ngoi tiếp O đến các cnh BC, CA, AB. Gi R, r lần lượt là bán kính đường tròn ngoi
tiếp và ni tiếp tam giác. Khi đó ta có hệ thc:
a b c
d d d R r+ + = +
NG DN GII
(Ph thuc hình v)
Ta thy t giác ONAP ni tiếp trong đường tròn đường kính AO
nên theo định lí Ptoleme:
. . .APON AN OP AO PN+=
..
2 2 2
b c b c
c b a
d d R cd bd R a + = + =
.
Hoàn toàn tương tự, ta có:
. . . , . . .
a b c a
bd ad Rc ad cd Rb+ = + =
Ta cũng có:
. . 2
a OBC
d a OM BC S==
.
Tương tự:
. , .
b OCA c OAB
d b S d c S==
.
Cng tt c các đẳng thc trên li, ta có:
( )( ) ( ) (S )
a b c OAB OBC OCA
a b c d d d R a b c S S+ + + + = + + + + +
2 ( ) .2 2
a b c a b c
p d d d R p S d d d R r + + = + + + = +
.
CHUYÊN ĐỀ BỒI DƯỠNG HỌC SINH GIỎI HÌNH HỌC 9
63
Ví d 5. Cho hình bình hành ABCD, trên DC ly điểm E sao cho
AD AE=
. Gọi F là điểm bt kì
trên cnh BC. Gi H và K lần lượt là hình chiếu ca B và F trên AE và AD. Chng minh rng
BH FK=
.
NG DN GII
//DC AB
nên đường cao k t E đến AB bằng đường cao k t A và B đến DC.
Ta li thy
AB DE CE=+
nên
ABE ADE BCE
S S S=+
1
2
ABE ABCD
SS=
(1)
Tương tự
1
2
ADF ABCD
SS=
(2)
T (1) và (2) suy ra
11
..
22
ABE ADF
S S AE BH AD FK BH FK = = =
.■
Ví d 6. Cho hình bình hành ABCD, trên cnh AB và CB lần lượt ly điểm E và điểm F sao cho
AF CE=
. Gọi điểm O là giao điểm CE và AF. Chng minh
AOD COD=
.
NG DN GII
K
DH AF
DI CE
Theo ví d 5 thì:
11
..
22
ADF DCE
S S DH AF DI CE= =
Theo gi thiết thì
AF CE=
DH DI OD =
là phân giác
AOC
Vy,
AOD COD=
.
Ví d 7. Cho tam giác ABC, M là một điểm nằm trên đoạn BC. Trên tia đối ca tia MA lấy điểm N,
trên đoạn AC lấy điểm E. Đường thng NE ct BC ti P sao cho
ABM MNP CPE
S S S==
. Chng minh
rng:
PB PC=
.
NG DN GII
Do
//
MNP CPE
S S ME NC=
(tính cht 5)
Áp dng tính cht 6 ta có:
AMP AEP
SS=
. Mà theo gi thiết
ABM MNP CPE ABP ACP
S S S S S= = =
T tính cht 2 suy ra:
PB PC=
.■
II. BÀI TP ÁP DNG
CHUYÊN ĐỀ BỒI DƯỠNG HỌC SINH GIỎI HÌNH HỌC 9
64
Bài 1. Cho tam giác ABC, trên cnh AB lấy điểm E sao cho
3BE AE=
. Trên cnh BC lấy điểm F
sao cho
4BF FC=
. Gọi D là giao điểm ca AF và CE. Chng minh rng
CD DE=
.
Bài 2. Cho tam giác ABC, trên tia đối ca tia AC lấy điểm D sao cho A là trung điểm ca DC, trên
tia đối ca BA lấy điểm E sao cho B là trung điểm của AE, trên tia đối ca tia CB ly điểm F sao
cho
7
DEF ABC
SS=
. Chng minh rng
CB CF=
.
Bài 3. Cho hình thang ABCD có đáy lớn AB, gi M, N lần lượt là trung điểm ca AB và CD. Mt
đường thng song song với hai đáy cắt các đoạn AD, BC, MN lần lượt ti E, I, F. Chng minh
EI FI=
III. NG DN GII
Bài 1. Do
1
3
4
BE EA AE AB= =
.
Theo tính cht 2 thì
1
4
AEF ABF
SS=
(1).
Do
11
44
ACF ABF
CF BF S S= =
(2)
T (1) và (2) suy ra:
AEF ACF
SS=
.
K hai đường cao EH và CK ca hai tam giác AEF và tam giác ACF.
Suy ra
EH CK EHD CKD= =
(g.c.g)
CD DE=
.■
Bài 2. Vì A là trung điểm ca DC, theo tính cht 2 thì:
2
ABD ABC ADE ABC
S S S S= =
(1)
Tương tự:
2
DCF ABC
SS=
(2)
Theo đề bài
7
DEF ABC
SS=
(3)
T (1), (2) và (3) ta có:
22
BEF ABC BEF BCE BCE DEF
S S S S S S= = =
.
Theo tính cht 2:
BC CF=
.■
Bài 3. K đường cao EH và FK ca hai tam giác EMN và
FMN.
AM BM=
DN CN=
nên
AMND BMNC
SS=
(1)
// , //EF AB EF DC
nên
AME BMF
SS=
(2) và
DNE CNF
SS=
(3)
T (1), (2) và (3) ta suy ra:
CHUYÊN ĐỀ BỒI DƯỠNG HỌC SINH GIỎI HÌNH HỌC 9
65
EMN FMN
S S EH FK= =
EHI FKI EI FI = =
.■
CHUYÊN ĐỀ BỒI DƯỠNG HỌC SINH GIỎI HÌNH HỌC 9
66
Bài toán 6.S dụng phương pháp về hình bình hành
Để chứng minh đẳng thc và bất đẳng thc
I. KIN THC CN NH
1. Định nghĩa: Hình bình hành là t giác có các cạnh đối song song.
2. Tính cht:
Cnh: Các cạnh đối song song và bng nhau.
Góc: Các góc đối bng nhau.
Đường chéo: Các đường chéo ct nhau tại trung điểm ca mỗi đường.
II. CÁC VÍ D
Ví d 1. Chng minh rằng trong tam giác vuông, đường trung tuyến ng vi cnh huyn bng na
cnh huyn.
NG DN GII
Lấy D sao cho D đối xng vi A qua M. Suy ra ABDC là hình bình hành.
Li có
90BAC =
.
Nên ABDC là hình ch nht.
Vy nên:
1
2
AM BC=
.■
Ví d 2. Chng minh rng: Trong một tam giác, ba đường trung tuyến đồng quy.
NG DN GII
Xét tam giác ABC, trung tuyến AM. Ly G trên AM sao cho
2AG GM=
.
V hình bình hành BGCD.
CHUYÊN ĐỀ BỒI DƯỠNG HỌC SINH GIỎI HÌNH HỌC 9
67
Tia CG ct AB ti E.
//GE BD
và G là trung điểm AD. Nên GE là đường trung bình
ABD
.
Suy ra E là trung điểm AB.
Tương tự BG kéo dài ct AC F thì F là trung điểm AC.
Vậy ba đường trung tuyến của tam giác đồng quy.■
Ví d 3. V ra ngoài tam giác ABC hai tam giác vuông cân ti A là:
,ABD ACE
. Gi M là trung
điểm BC. Chng minh rng:
1
2
AM DE=
.
NG DN GII
V hình bình hành ABNC
Khi đó
180DAE ACN A

= =


.
Do đó,
DAE NCA =
(c.g.c)
1
2
AN DE AM DE = =
.■
Ví d 4. Dng bên ngoài tam giác ABC hai tam giác vuông cân ti A là: ABD và ACE. Chng
minh:
ABC ADE
SS=
NG DN GII
Dng hình bình hành ABFC
1
2
ABC ABF ABFC
S S S

= =


(1)
Li có:
ADE ABF
ADE ABF S S = =
(2)
T (1) và (2) suy ra:
ABC ADE
SS=
.
Ví d 5. Bên ngoài tam giác ABC v hai tam giác cân ti A lần lượt là:
ABH
ACE
sao cho
120H =
60E =
. Gọi M là trung điểm ca BC. Chng minh
90HME =
.
NG DN GII
CHUYÊN ĐỀ BỒI DƯỠNG HỌC SINH GIỎI HÌNH HỌC 9
68
V hình bình hành BNCH.
HA CN=
90HAE NCE A

= = +


Vì vy
HAE NCE =
(c.g.c)
HE NE=
(1)
12
60E E HEN= =
(2)
T (1) và (2) suy ra
HNE
đều
90HME =
.■
Ví d 6. V bên ngoài tam giác ABC hai tam giác vuông cân ti M và N là
ABM
ACN
. Gi
E là trung điểm ca BC. Chng minh rng
MNE
vuông cân.
NG DN GII
Dng hình bình hành CMBD
MA CD=
90MAN DCN A

= = +


MAN DCN =
(c.g.c)
12
NN=
MND
vuông cân ti N
Mà E là trung điểm ca MD nên
MNE
vuông cân.
Ví d 7. Hai ngôi làng A, B nm hai bên sông, cn xây cầu DE như thế nào để đường đi ADEB
ngn nht (biết 2 b sông song song vi nhau, cu vuông góc vi b).
NG DN GII
Dựng hình bình hành ADEK, F là giao điểm ca b sông vi BK (như hình vẽ) khi đó:
AD ED EB AK KE BE AK BK+ + = + + +
Do DE không đổi nên AK không đổi
K
c định
F
c định.
AD ED EB AK BK + + +
(không đổi)
Vậy đoạn thng ngn nht khi
EF
Ví d 8. Cho lc giác ABCDEF có các cnh thỏa mãn điều kin
0BC EF ED AB AF CD = =
và cạnh đối din song song vi nhau. Chng minh lc giác
ABCDEF có các góc bng nhau.
NG DN GII
CHUYÊN ĐỀ BỒI DƯỠNG HỌC SINH GIỎI HÌNH HỌC 9
69
V các hình bình hành APEF, CDEN. K AP ct CN ti M. Suy ra t
giác ABCM là hình bình hành.
Theo bài ra ta có:
0BC EF ED AB AF CD = =
.
Thế nên, tam giác PMN đều
Ta thy:
120 ; 120 ; 120B AMC F APE D CNE= = = = = =
.
Ta cũng thấy:
60 60 120BAF AMN EPM= + = + =
.
Tương tự:
120FED DCB= =
Kết lun:
120A B C D E F= = = = = =
.
Ví d 9. Cho t giác li ABCD vi
AD BC=
, gi E, F lần lượt là trung điểm ca CD và AB. Gi
giao điểm của AD và FE là H và giao điểm ca BC và FE là G. Chng minh rng:
AHF BGF=
.
NG DN GII
Lấy điểm I sao cho F là trung điểm ca CI.
Lúc y, ACBI là hình bình hành.
Vì thế EF là đường trung bình ca tam giác CDI.
11
GI=
(1) (góc to bi hai cạnh tương ứng song song).
11
DH=
(2) (hai góc đồng v
//DI HF
).
Do
AD AI=
nên tam giác ADI là tam giác cân ti A
11
DI=
(3)
T (1) (2) và (3) ta suy ra:
AHF BGF=
.
III. BÀI TP
Bài 1. Cho t giác ABCD có M, N lần lượt là trung đim ca
AD và BC. Chng minh rng, nếu
2AB CD MN+=
thì
//AB CD
.
Bài 2. Cho t giác ABCD có hai đường chéo
,AC x BD y==
(không đổi). Góc to bởi AC và BD là a (không đổi). Xác định
hình dng ca t giác ABCD để chu vi ABCD nh nht?
Bài 3. Cho tam giác ABC nhn, có
2BC=
. K đường cao AH. Trên tia đối ca tia BA lấy điểm E
sao cho
BE BH=
. Chng minh
EA HC=
.
Bài 4. Chng minh rằng độ dài đường trung tuyến luôn nh hơn nửa tng hai cnh bên.
CHUYÊN ĐỀ BỒI DƯỠNG HỌC SINH GIỎI HÌNH HỌC 9
70
Bài 5. Chng minh rằng độ dài ba đường trung tuyến ca
tam giác luôn nh hơn chu vi và lớn hơn
3
4
chu vi ca
tam giác.
Bài 6. Cho tam giác ABC vuông cân ti A, gi K là trung
điểm của BC. Trên tia đối ca tia KA lấy điểm D sao cho
2KD KA=
. Gọi M là trung điểm ca AK. Chng minh
rng:
BM BD
IV. HƯỚNG DN GII
Bài 1. V hai hình bình hành CDME và ABFM ( như hình vẽ bên)
T đó suy ra: CEBF là hình bình hành vì có mt cp cạnh đối va song song và bng nhau.
Suy ra, E, N, F thng hàng.
Lấy điểm K sao cho N là trung điểm ca MK thì MFKE là hình bình
hành.
Ta có:
2AB CD MN MK+ = =
.
Suy ra:
MF ME MK+=
Suy ra:
MF FK MK+=
.
T đó E và F cùng nằm trên MK. Vy
//AB CD
.
Bài 2. Dng hai hình bình hành là ADBF và BDCE.
Suy ra
//AF CE
AF CE=
(do cùng bng và song song vi BD).
Suy ra ACEF là hình bình hành
Li có
AF y=
EF x=
,
FAC
=
(không đổi)
Do đó
ABCD
P AB BC CD DA= + + +
( ) ( )
ABCD
P AB BE BC BF= + + +
ABCD
P AE CF+
(không đổi)
Nên chu vi ca ABCD nh nht khi A, B, E và C, B, F thng
hàng.
Vậy lúc đó ABCD là hình bình hành.■
Bài 3. Dng hình bình hành AHCD mà
90H =
nên AHCD
là hình ch nht.
1 1 1
C H D = =
(1)
CHUYÊN ĐỀ BỒI DƯỠNG HỌC SINH GIỎI HÌNH HỌC 9
71
1
11
1
2
2
BH
HC
BC
=
=
=
(2)
T (1) và (2) suy ra
12
HH=
suy ra E, H, D thng hàng.
T đó suy ra
1
ED=
AED
cân ti A
AE AD=
AE HC=
.
Bài 4. V hình bình hành ABDC (như hình bên)
Khi đó
2
AD
AM=
(tính cht hình bình hành)
AD AC AB+
(bất đẳng thc tam giác)
T đó:
2
AB AC
AM
+

.
Bài 5. Gọi 3 đường trung tuyến ca tam giác lần lượt là AM, BN, CE.
Theo kết qu ca bài 12 thì:
;;
2 2 2
AB AC AB BC BC AC
AM BN CE
+ + +
Cng hai vế ca các bất đẳng thc cùng chiều ta được:
AM BN CE AB BC CA+ + + +
(1)
Gi G là trng tâm ca tam giác ABC, ta có
2
()
3
GA GB AB AM BN AB+ +
Tương tự ta có:
22
( ) ; ( )
33
CE BN BC CE AM AC+ +
T đó suy ra:
3
()
4
AM BN CE AB BC CA+ + + +
(2)
T (1) và (2) suy ra điều phi chng minh.
Bài 6. Lấy E là trung điểm ca DK, v hình bình hành
ABKN như hình vẽ bên.
Khi đó:
ABN EBD =
(c.g.c)
ABN EBD=
90 90ABE NBD = =
Vy
BM BD
.■
CHUYÊN ĐỀ BỒI DƯỠNG HỌC SINH GIỎI HÌNH HỌC 9
72
Bài toán 7.s dụng tam giác đồng dạng để chứng minh đẳng
thc, bất đẳng thc
I. KIN THC CN NH
1. Định nghĩa
Tam giác ABC đồng dng vi tam giác DEF khi
và ch khi
;;A D B E C F
AB BC CA
DE EF FD
= = =
==
Kí hiu:
ABC DEF
.
2. Các trường hợp đồng dng ca tam giác
a) Trường hợp đồng dng cnh - cnh - cnh:
* Nếu ba cnh ca tam giác này t l vi ba cnh. của tam giác kia thì hai tam giác đó đồng dng.
* C thể: Xét ∆ABC và ∆DEF
Ta có
AB BC CA
DE EF FD
==
Thì
ABC DEF
(c.c.c)
b) Trường hợp đồng dng cnh - góc - cnh
* Nếu hai cnh ca tam giác này t l vi hai cnh
ca tam giác kia và hai góc to bi các cp cạnh đó
bng nhau thì hai tam giác đó đồng dng.
* C thể: Xét ∆ABC và ∆DEF
Ta có:
AB CA
DE FD
=
AD=
Thì
ABC DEF
(c.g.c)
* Chú ý: Hai góc đang xét phải là hai góc xen gia các cp cnh t l đang xét.
CHUYÊN ĐỀ BỒI DƯỠNG HỌC SINH GIỎI HÌNH HỌC 9
73
c) Trường hợp đồng dng góc - góc
* Nếu hai góc ca tam giác này lần lượt bng hai góc ca tam giác kia thì hai tam giác đó đồng
dng.
* C thể: Xét ∆ABC và ∆DEF
Ta có
AD=
BE=
Thì
ABC DEF
(g.g)
* Chú ý: Nếu mt góc nhn ca tam giác vuông này bng
mt góc nhn ca tam giác vuông kia thì hai tam giác
vuông đó đồng dng.
d) Trường hợp đồng dng cnh huyn - cnh góc vuông:
* Nếu cnh huyn và mt cnh góc vuông ca tam giác vuông này t l vi cnh huyn và mt cnh
góc vuông của tam giác vuông kia thì hai tam giác vuông đó đồng dng.
* C thể: Xét ∆ABC và ∆DEF
Ta có
AB AC
DE DF
=
90
o
BE==
thì
ABC DEF
(cnh huyn, cnh góc vuông)
Chú ý: Trường hợp này đặc bit ch hai góc bng nhau không
phi là hai góc xen gia ca hai cp cnh t l đang xét.
II. CÁC VÍ D
Ví d 1. Cho tam giác ABC có hai đường cao BD và CE. Chng minh rng:
. . .AE AB AD AC=
NG DN GII
Xét ∆AEC và ∆ADB có
A
là góc chung
AEC ADB AEC ADB=
(g.g)
. . .
AE AC
AE AD AD AC
AD AB
= =
CHUYÊN ĐỀ BỒI DƯỠNG HỌC SINH GIỎI HÌNH HỌC 9
74
Ví d 2. Cho tam giác ABC cân ti A,
( )
90
o
A =
có đường cao AD, trc tâm H. Chng minh rng:
2
.CD DH DA=
NG DN GII
Xét ∆DHB∆DCA
90
o
BDH ADC==
DBH DAC=
(cùng phụ với
C
)
Suy ra
DHB DCA
(g.g)
2
..
..
DH DB
DH DA DC DB
DC DA
CD DH DA
= =
=
Ví d 3. Cho tam giác ABC nhn, trc tâm H. Gi M là trung điểm ca BC. Đường thng vuông
góc vi MH ti H ct ABAC theo th t ti IK. Chng minh rng: HI = HK.
NG DN GII
Ta có:
HAI MCH=
(cùng ph vi
ABC
)
AHI CHM=
(cùng ph vi
DHI
)
T đó suy ra:
AIH CHM
(g.g)
(1)
IH AH
HM CM
=
Ta có:
HAK MBH=
(cùng ph vi
C
)
AHK BMH=
(s dng tính cht góc ngoài ca tam giác)
T đó suy ra:
AHK BMH
(g.g)
HK AH
MH BM
=
(2). T (1) và (2) suy ra
CHUYÊN ĐỀ BỒI DƯỠNG HỌC SINH GIỎI HÌNH HỌC 9
75
.
IH HK
IH HK
HM HM
= =
Ví d 4. Cho hình bình hành ABCD. Một đường thẳng đi qua điểm A cắt đoạn BC tại điểm F, ct
đường thng DC tại điểm G. Chng minh rng:
..BF DG AB AD=
NG DN GII
Gọi giao điểm ca AFBD là điểm E.
Do
DEG BEA
(g.g)
nên
(1)
DG ED
AB EB
=
Do
AED FEB
(g.g)
Nên
(2)
AD ED
BF EB
=
T (1) và (2) suy ra:
..
AD DG
BF DG AD AB
BF AB
= =
Ví d 5. Cho tam giác ABC có
2; 3; 4AB AC BC= = =
. Chng minh rng
2BAC ABC ACB=+
NG DN GII
Trên đoạn thng BC lấy điểm D sao cho
1BD =
.
Ta có
3.CD BC BD= =
ACD
cân ti C nên ta suy ra
(1)ADC DAC=
Li có
ABD CBA
(c.g.c) nên ta có
(2).BAD C=
T (1) và (2) suy ra
BAC BAD DAC ACB ADC= + = +
2ACB ABC BAD BAC ABC ACB= + + = = +
III. BÀI TP
Bài 1. Cho hình thang ABCD (AB//CD) sao cho
BAD DBC=
. Chng minh
2
.BD AB DC=
.
Bài 2. Cho tam giác ABCAD là phân giác ca tam giác ABC. Chng minh
2
.AD AB AC
.
Bài 3. Cho tam giác ABC, v hai đường cao BDCE ca tam giác ABC. Gi EHDK là hai
đường cao ca tam giác ADE. Chng minh
.
AK AH
AB AC
=
CHUYÊN ĐỀ BỒI DƯỠNG HỌC SINH GIỎI HÌNH HỌC 9
76
Bài 4. Cho tam giác ABC có ba góc nhn, v hai đường cao BDCE ct nhau tại điểm H. Chng
minh
2
..BH BD CH CE BC+=
.
Bài 5. Cho hình vuông ABCD, trên cnh BC lấy điểm M sao cho
3
BC
BM =
. Trên tia đối ca tia CD
lấy điểm N sao cho
2
BC
CN =
. Gi I là giao điểm ca AMBN. Chng minh
90
o
CIM =
.
Bài 6. Cho AABC có
3 2 180 .AB+ =
Chng minh
( )
2
..BC AB AB AC=−
Bài 7. Cho hình thoi ABCD
60A =
. Gi M là một điểm thuc cnh AD. Đường thng CM ct
đường thng AB ti N. Gọi giao điểm ca BMDN là điểm P. Chng minh
60
o
BPD =
.
Bài 8. Cho hình bình hành ABCD (AB > AD). T điểm C k CE và CF lần lượt vuông góc vi
đường thẳng AB và đường thng
( )
, .AD E AB F AD
Chng minh
2
AB.AE AD.AF AC+=
.
Bài 9. Cho tam giác ABC nhn, k các đường cao BDCE ca tam giác ABC. Chng
minh
AED ACB=
.
Bài 10. Cho tam giác ABC, v phân giác AD ca tam giác ABC. Chng minh
2
. . .AD AB AC DB DC=−
Bài 11. Cho hình vuông ABCD. Trên cnh AB lấy điểm M. K đường thng BH vuông góc
vi
( )
CM H CM
. V HN vuông góc vi DH ct BC tại điểm N. Chng minh
rng
..AM NB NC MB=
.
Bài 12. Cho hình vuông ABCD, trên cnh AB, BC lần lượt lấy hai điểm E và điểm F sao cho BE =
BF. Gi H là hình chiếu ca B trên CE. Chng minh
90 .
o
DHF =
Bài 13. Cho tam giác ABC nhọn, ba đường cao ca tam giác là AD, BE, CF ct nhau tại điểm H.
Chng minh
DEH FEH=
.
Bài 14. Cho tam giác ABC vuông ti A, đường phân giác BD cắt đường cao AH tại điểm I. Chng
minh
..AD BD BI DC=
.
Bài 15. Cho tam giác ABC vuông ti A, trên cnh AC lấy điểm D. Gọi điểm E là hình chiếu ca
điểm C trên BD. Chng minh
. . . .AB CE AE BC AC BE+=
Bài 16. Cho tam giác ABC đều, gi O là trng tâm ca tam giác. Ly M là một điểm bt kì thuc
cnh BC, (M không trùng với trung điểm ca cnh BC). K MPMQ lần lượt vuông góc vi AB
AC sau đó cắt OB, OC th t ti IK. Gi D là giao điểm ca PQOM. Chng
minh
PD QD=
.
Bài 17. Cho t giác ABCDM, N lần lượt là trung điểm của hai đường chéo BDAC sao cho
điểm M không trùng với điểm N. Đường thng MN ct hai cnh bên ADBC lần lượt tại điểm P
Q. Chng minh
.
PA QC
PD QB
=
CHUYÊN ĐỀ BỒI DƯỠNG HỌC SINH GIỎI HÌNH HỌC 9
77
Bài 18. Cho tam giác ABC
, , AB c AC b BC a= = =
tha mãn
2AB=
. Chng minh
22
a b bc=+
.
Bài 19. Cho tam giác ABC. Các đường phân giác góc ngoài tại các đỉnh BC ca tam giác ct
nhau K. Đường thng vuông góc vi AK ti K cắt các đường thng AB, AC theo th t DE.
Chng minh
2
4 . .DE BD CE=
Bài 20. Cho tam giác ABC nhọn. Các đường cao AD, BE, CF ct nhau tại điểm H. Chng minh
rng:
a)
..AB AF AC AE=
b)
2
..BH BE CH CF BC+=
.
Bài 21. Cho tam giác ABC cân ti C
15
o
C =
. V điểm D nm min trong ca tam giác sao cho
105
o
ADB =
2AD BD=
.
Chng minh rng:
5 . 2 .AD BC CD AB=
.
Bài 22. Cho tam giác ABD, đường phân giác AD. Chng minh rng
2
.AD AB AC
Bài 23. Cho t giác ABCD
BAC DAC=
ABC ACD=
, các đường thng ADBC ct nhau
E, các đường thng ABCD ct nhau F. Chng minh rng
. . AB DE BC CE=
2
2 . . AC AD AF AB AE+
.
Bài 24. Cho tam giác cân ABC
( )
AB AC=
, I là trung điểm ca BC. Trên các cnh AB, AC ly ln
ợt các điểm K, H sao cho
2
.BK CH BI=
. Chng minh
. . . .IH KB HC IK HK BI+
Bài 25. Cho tam giác ABC vuông ti A. Lấy các điểm MN lần lượt nm trên các cnh ABAC;
các điểm PQ nm trên cnh BC sao cho MNPQ là hình vuông. Chng minh rng
3BC PQ
.
Bài 26. Cho t giác ABCD. Chng minh rng
. . .AB CD AD BC AC BD+
.
Bài 27. Cho lc giác li ABCDEF
, , AB BC CD DE EF FA= = =
.Chng minh rng
3
2
BC DE FA
BE DA FC
++
IV. NG DN GII
Bài 1.
Xét ABDBDC
Có :
BAD DBC=
(gt)
ADB BDC=
(so le trong)
Suy ra
ABD BDC
(g.g)
2
..
AB BD
BD AB DC
BD DC
= =
Bài 2.
Gi s AB < AC, trên đoạn AC lấy điểm M sao cho
ADM ABD=
CHUYÊN ĐỀ BỒI DƯỠNG HỌC SINH GIỎI HÌNH HỌC 9
78
Xét
ABD
ADM
ABD ADM=
Suy ra
ABD ADM
(g.g)
22
. . .
AD AB
AD AM AB AD AB AC
AM AD
= =
Bài 3. Xét trường hp tam giác ABC nhọn, các trường hp
khác tương tự.
Do
ABD ACE
(g.g)
(1)
AD AB
AE AC
=
Do
AHE AKD
(g.g)
(2)
AD AK
AE AH
=
T (1) và (2) suy ra
AB AK AK AH
AC AH AB AC
= =
Bài 4. K HK vuông góc vi BC
( )
K BC
Xét
BHK
BCD
B
chung
90
o
KD==
.
Suy ra
BHK BCD
(g.g)
nên
. . (1)
BH BK
BH BD BC BK
BC BD
= =
Tương tự
CKH CEB
(g.g)
. . (2)CH CE CK BC=
T (1) và (2) suy ra
( )
2
..BH BD CH CE BC BK CK BC+ = + =
Bài 5. Gi E là giao
điểm ca AIDC.
Gi F giao điểm ca
CI AB .
Đặt cnh ca hình
vuông ABCDa
Do
ABM EMC
(g.g)
2 2 .
CE CM
CE a
AB BM
= = =
3
2.
2 2 2
a a a
CN NE a= = =
Do
31
/ / : .
2 2 3 3
BF CN BF a a a
AF CE BF
AB NE a
= = = =
CHUYÊN ĐỀ BỒI DƯỠNG HỌC SINH GIỎI HÌNH HỌC 9
79
Do đó
BF BM ABM CBF= =
(c.g.c)
BAM FCB BAM ICM = =
Xét
ABM
CIM
BAM ICM=
(cmt),
AMB CMI=
ối đỉnh)
Suy ra
ABM CIM
(g.g).
Vy
90
o
CIM ABM==
.■
Bài 6. Do
3 2 180
o
AB+=
nên
3 2 2A B A B C A B+ = + + +
.
C A AB BC
AB AC
CB



Trên cnh AB lấy điểm D sao cho
AD AC=
.
180
(1)
2
o
A
ADC
=
Theo bài ra ta có:
180
3 2 180 (2)
2
o
o
A
A B A B
+ = + =
T (1) và (2) suy ra
ADC A B = +
.
1
ADC C B=+
(tính cht góc ngoài ca tam giác).
Suy ra
1
AC=
.
Xét
ABC
CBD
có:
B
là góc chung
1
AC=
(chng minh trên).
Suy ra
ABC CBD
(g.g).
( )
2
.
AB BC
BC AB BD AB AB AC
BC BD
= = =
Bài 7.
(1) (do )
NA AM NA NB
NAM NBC BC BD
NB BC AM BD
= = =
Do
(2) (do )
NA CD NA BD
NAM CDM CD BD
AM DM AM DM
= = =
Do
T (1) và (2) suy ra:
NB BD
BD DM
=
BND
Xét
60 (gt)
o
DMN MDB==
BDM
NB BD
BD DM
=
(chng minh
trên)
CHUYÊN ĐỀ BỒI DƯỠNG HỌC SINH GIỎI HÌNH HỌC 9
80
Suy ra
(c.g.c)BND DBM BND DBM =
BPD BND NBP=+
(tính cht góc ngoài)
60
o
BPD DBM NBP DBN = + = =
Bài 8.
Gọi các điểm M và điểm N lần lượt là hình chiếu của điểm D điểm B trên đường thng AC.
Do
ANB AEC
(g.g)
. . (1)
AB AN
AB AE AC AN
AC AE
= =
Do
AMD AFC
(g.g)
. . (2)
AD AM
AD AF AC AM
AC AF
= =
T (1) và (2) suy ra
( )
2
..AB AE AD AC AN AM AC+ = + =
.
Bài 9. Do
ADB AEC
(g.g)
AD AB AD AE
AE AC AB AC
= =
Xét
AED
ACB
A
là góc chung
AD AE
AB AC
=
(cmt)
AED ACB
(c.g.c)
AED ACB=
.
Bài 10. Theo tính cht tam giác ngoài ca tam giác, ta có
ADC ABC
. Trên tia đối ca ta DA lấy điểm E sao cho
ADC ABE=
( )
2
(*)
( . )
. . .
. . (1)
CE
ADC ABE g g
AD AC
AB AE
AD AE AB AC AD AD DE AB AC
AD AB AC AD DE
=
=
= + =
=
Xét
ADC
ABE
12
DD=
ối đỉnh)
CE=
(chng minh (*))
Suy ra
ADC ABE
(g.g)
. . (2)
AD DC
AD DE BD DC
BD DE
= =
T (1) và (2) suy ra
2
..AD AB AC BD DC=−
.
CHUYÊN ĐỀ BỒI DƯỠNG HỌC SINH GIỎI HÌNH HỌC 9
81
Bài 11. Xét
DHC
NHB
DCH NBH=
(cùng ph
BCH
)
DHC NHB=
(cùng ph
CHN
)
DHC NHB
(g.g)
(1)
NB HB
DC HC
=
Do
MBH BCH
(g.g)
(2)
BH MB
HC BC
=
T (1) và (2) suy ra
..
NB MB
BC NB DC MB NB MB
DC DC
= = =
(do
BC DC=
).
Suy ra
AM NC=
.
Vy
..AM NB NC MB=
Bài 12. Tha thy
HBC HEB HCD==
EHB BHC
(g.g)
HB BE BF
HC BC CD
= =
(do
;BE BF BC CD==
)
Xét
HBF
HCD
HBF HCD=
(chng minh trên)
HB BF
HC CD
=
(chng minh trên)
Suy ra
HBF HCD
(c.g.c)
90
o
BHF CHD DHF = =
.
Bài 13.
BHF CHE
(g.g)
BH CH
HF HE
=
Xét
BHC
FHE
BHC FHE=
ối đỉnh)
BH CH
HF HE
=
(chng minh trên)
BHC FHE
(c.g.c)
11
(1)CE=
Tương tự, chứng minh được
AHB EHD
(c.g.c)
22
(2)AE=
12
(3)CA=
(cùng ph vi
1
A
)
T (1), (2) và (3) ta có:
12
EE=
. Vy
DEH FEH=
.
Bài 14. Ta có
ABH CBA
(g.g)
CHUYÊN ĐỀ BỒI DƯỠNG HỌC SINH GIỎI HÌNH HỌC 9
82
BH BA
BA BC
=
BA AD
BC DC
=
(tính chất đường phân giác)
(1)
BH AD
BA DC
=
Do
BHI BAD
(g.g)
(2)
BH BI
BA BD
=
T (1) và (2) suy ra
. . .
AD BI
AD BD DC BI
DC BD
= =
Bài 15. Gi sa ABCE ct nhau ti điểm M. Trên
BE lấy điểm F sao cho
90
o
BAF EAC EAF= =
Do
MEB MAC
(g.g)
ME MB MA ME
MA MC MC MB
= =
.
Xét
MAE
MCB
M
là góc chung
MA ME
MC MB
=
(chng minh trên)
MAE MCB
(c.g.c)
11
MEA MBC E C = =
(do cùng ph vi
MBC
)
Ta có
21
2 2 2 2
12
90
o
BD
C D B C
DD
=
= = =
=
Do
22
BAF CAE
BAF CAE
BC
=
=
(g.g)
. .BF (1)
AB BF
AB CE AC
AC CE
= =
Do
0
1
1
EAF CAB 90
EAF CAB
EC
==
=
. . (2)
AE EF
AE BC AC EF
AC BC
= =
T (1) và (2) suy ra
( )
. . . .AB CE AE BC AC BF EF AC BE+ = + =
Bài 16. Do tam giác ABC đều nên trng tâm O là giao điểm của các đường cao ca tam giác y. Suy
ra IMKO là hình bình hành.
Ta thy
MIB MKC
(g.g)
CHUYÊN ĐỀ BỒI DƯỠNG HỌC SINH GIỎI HÌNH HỌC 9
83
(1)
MI MB
MK MC
=
Ta thy
MPB MQC
(g.g)
(2)
MB MP
MC MQ
=
T (1) và (2) suy ra
/ / .
MI MP MI MK
IK PQ
MK MQ MP MQ
= =
IMKO là hình hình hành nên
NI NK=
//IK PQ
.
Vy
PD QD=
nh lý Thales).
Bài 17.
T điểm A và điểm C k các đường thng song song với đường thng BD cắt đường thng MN ln
t tại điểm E và điểm F.
ANE CNF =
(g.c.g)
AE CF=
APE DPM
(1)
PA AE
PD DM
=
CQF BQM
(g.g)
(2)
CQ CF
QB BM
=
(3)
AE CF
DM BM
=
do
AE CF;DM BM.==
T (1), (2) và (3) suy ra
.
PA QC
PD QB
=
Bài 18. V phân giác AD của ∆ABC. Áp dụng tính chất đường phân giác ca tam giác ta có:
.
c BD c b BD DC ab
DC
b DC b DC c d
++
= = =
+
Xét
ABC
DAC
có:
C
chung và
1
.
2
ABC DAC A

==


ABC DAC
(g.g)
( )
2 2 2
. . .
BC AC ab
AC BC DC b a b b c a
AC DC b c
= = = + =
+
Vy
22
a b bc=+
Chú ý: M rng bài toán.
Nếu gi thiết bài toán tr thành
24A B C+=
thì khi đó
1 1 1
.
c a b
=+
CHUYÊN ĐỀ BỒI DƯỠNG HỌC SINH GIỎI HÌNH HỌC 9
84
Tht vy, áp dng bài 18 ta có:
Do
( )
22
22
22
2
.
2
A B a b bc
a c ac bc c a b c
B C b c ac
= = +
= + + = + +
= = +
( )
22
1 1 1 1 1
.
2
a b c b c b c
c a a a b b c a b
+ + + +
= = + = + = +
+
Bài 19.
Theo tính chất điểm nm trên tia phân giác ca góc, khong cách
t K đến AD bng khong cách t K đến AE (cùng bng khong
cách t K đến BC).
Do đó K nm trên tia phân giác trong góc A ca tam giác ABC.
Tam giác ADE có đường cao đồng thi là đường phân giác
nên cân ti A. T đó
DE=
.
Có tng 4 góc ca t giác BCED bng
360
o
.
1 2 1 2
;;B B C C D E= = =
nên
11
360
180
2
o
o
B C E+ + = =
Cũng có
1
180
o
CKE C E= + =
nên
1
CKE B=
Hai tam giác BDK và KEC có
1
,CKE B D E==
nên chúng đồng dng
(g.g).
T đó
BD DK
KE EC
=
hay
. . (*)BD EC DK KE=
.
Để ý rng
2
DE
DK KE==
, thay vào (*) ta được
2
.
4
DE
BD CE =
hay
2
4.DE BD CE=
Bài 20.
a) Xét
ABE
ACF
A
,
90
o
AEB AFC==
Do đó
ABE ACF
(g.g)
T đó suy ra
AB AE
AC AF
=
hay
. . .AB AF AC AE=
b) D dàng chứng minh được:
+
BHD BCE
(g.g) suy ra
BH BC
BD B E
=
Hay
. . (1)BH BE BD BC=
+
CHD CBF
(g.g) suy ra
CH CD
BC CF
=
Hay
. . (2)CH CF CD BC=
CHUYÊN ĐỀ BỒI DƯỠNG HỌC SINH GIỎI HÌNH HỌC 9
85
Cộng (1) và (2) ta được:
( )
2
. . . . . .BH BE CH CF BD BC CD BC BC BD CD BC+ = + = =
Chú ý: Qua bài tập trên, để chứng mình được h thc dng
2
..x y st r==
, ta có th chng minh hai
h thc
. . ; . .x y r a s t r b==
vi
a b r+=
. Cng hai h thức này cho ta điều phi chng minh.
Bài 21.
Trên na mt phng b BC không cha A v tia Cx vuông góc vi BC. Lấy điểm E trên tia Cx sao
cho BC = 2CE.
Thy rng:
105 ; 2
o
DA CA
ACE
DB CE
= = =
(vì
;2CA CB CB CE==
).
Nên ∆ADB và ∆ACE đồng dng (c.g.c). T đó:
;.
AD AB
CAE DAB
AC AE
==
Hai tam giác DAC và BAE đồng dng (c.g.c) nên
AD CD
AB BE
=
hay
. . (*)AD BE CD AB=
Mt khác tam giác BCE vuông ti C. Nếu đặt
CE x=
thì
2BC x=
.
Theo định lý Pythagore:
( )
2
2 2 2 2 2
2 5 5.BE BC CE x x x BE x= + = + = =
.
T đó
5
2
BE BC=
. Thay vào h thức (*) ta được
5 . 2 . .AD BC CD AB=
Bài 22. Ta có
C ADB=
. Trong góc ADB v tia DE (E
thuc AB) sao cho
ADE C=
, khi đó điểm E nm trên
đoạn thng AB. T đó suy ra
AE AB
Mt khác
AED ADC
(g.g) nên
AE AD
AD AC
=
hay
2
.AE AC AD=
.
T đó
2
..AD AE AC AB AC=
CHUYÊN ĐỀ BỒI DƯỠNG HỌC SINH GIỎI HÌNH HỌC 9
86
Bài 23. Ta có
,DAC BAC ABC ACD==
nên
ACB ADC=
T đó
ACE EDC=
Hai tam giác ACECDE đồng dng (g.g) nên ta có
(1)
EC EA
ED EC
=
Li có
DAC BAC=
nên AC là đường phân giác
của ∆ABE, từ đó
(2)
EA BA
EC BC
=
T (1) và (2) suy ra
EC BA
ED BC
=
hay
. . .AB DE BC CE=
Áp dng kết qu bài 22, ta có AC là đường phân giác ca tam giác ABEACF nên
2
.AE AB AC
2
..AD AF AC
T đó
2
2 . .AC AB AE AD AF+
.
Bài 24. Ta có
2
.
BK BI CI
BK CH BI
BI CH CH
= = =
(vì
BI CI=
)
T đó
BKI CIH
(c.g.c)
Suy ra
BK BI KI
CI CH IH
==
. . . ; . . .IH KB IK CI IK BI HC IK BI IH = = =
Như vậy
. . . .IH KB IK HC IK BI IH BI+ = +
( )
..BI IH IK BI HK=
Bài 25
Vì MNPQ là hình vuông nên
/ / ; ;MN BC MQ BC NP BC⊥⊥
Ta có tam giác
,,ABC QBM PNC
đồng dng vi nhau nên
BQ AB
QM AC
=
PC AC
PN AB
=
.
T đó suy ra
1
BQ PC
BC BQ QP PC QP
QP QP

= + + = + +


1 1 3 .
BQ PC AB AC
QP QP QP
QM PN AC AB


= + + = + +




Dầu “=” xảy ra khi và ch khi
AB AC=
.
CHUYÊN ĐỀ BỒI DƯỠNG HỌC SINH GIỎI HÌNH HỌC 9
87
Chú ý: Bài toàn trên s dng bất đẳng thức đại s đơn giản sau:
Vi hau s dương ab, ta luôn có
2.
ab
ba
+
Bài 26. Trong góc ABC lấy điểm E sao cho
;ABD EBC ADB ECB==
Khi đó ta có
ABD EBC
(g.g)
Suy ra
.
AB BD AD
BE BC EC
==
T đó
. . (1)AD BC BD EC=
Xét
ABE
DBC
;ABE DBC
AB BD
BE BC
==
Do vy
ABE DBC
(c.g.c),
Suy ra
AB AE
DB DC
=
hay
. . (2)AB DC AE DB=
T (1) và (2) suy ra
( )
. . . . . .AB CD AD BC AE DB EC DB DB AE EC AC BD+ = + = +
Chú ý: Kết qu ca bài tp 18 là ni dung định lý Ptolemy.
Dấu “=” ở bất đẳng thc Ptolemy xy ra khi và ch khi t
giác ABCD ni tiếp (trong trường hợp này có đẳng thc
Ptolemy).
Bài 27. Áp dng bất đẳng thc Ptolemy (bài 15) vào t
giác ACEF
ta được
. . . .AC EF AF CE AE FC+
Theo bài ra
AF FE=
nên ta có
. . .FCAC AF AF CE AE+
( )
AF .
.
AC CE AE FC
FA AE
FC AC CE
+

+
Tương tự ta có
DE AC
DA AE AC
+
.
BC AC
BE EC AE
+
Cng các bất đẳng thức trên ta được
3
2
FA DE BC AE CE AC
FC DA BE AC CE AE AC CE AE
+ + + +
+ + +
.
Chú ý: Bài toán trên s dng bất đẳng thức đại s quen thuc sau: Vi ba s a,b và c dương, ta có
3
.
2
a b c
b c c a a b
+ +
+ + +
CHUYÊN ĐỀ BỒI DƯỠNG HỌC SINH GIỎI HÌNH HỌC 9
88
Bài toán 8. S dng h thc gia cạnh và đường cao trong tam giác vuông để chng minh
đẳng thc và bất đẳng thc
I. KIN THC CN NH
Cho tam giác ABC vuông ti A (hình bên). Gi
; ; ; ; '; 'BC a AC b AB c AH h BH c HC b= = = = = =
Khi đó ta có các hệ thc:
22
2 2 2
2
2 2 2
1) '; '
2)
3) ' '
4)
1 1 1
5)
b ab c ac
a b c
h b c
ah bc
h b c
==
=+
=
=
=+
II. CÁC VÍ D
Ví d 1. Cho tam giác ABC vuông tại A, đường cao AH. Chng minh
2
2
AB BH
BC BC
=
NG DN GII
Áp dng h thc trong tam giác ABC vuông ta có:
( )
2
.1AB BH BC=
Áp dng h thc trong tam giác ABC vuông ta có:
( )
2
.2AC CH BC=
T (1) và (2) suy ra:
2
2
.
.
AB BH BC BH
BC CH BC CH
==
Áp dng tính cht ca t l thc ta có:
22
2 2 2
AB BH AB BH
AB BC BH CH BC BC
= =
++
Ví d 2. Cho tam giác ABC vuông tại A đường cao AH, gọi D và E là điểm đối xng ca H qua
AB và AC. Chng minh
22
4 .HCHD HE BH+=
NG DN GII
Do D đối xng vi H qua AB nên:
( )
2. 1DAH BAH=
Do E đối xng vi H qua AC nên:
( )
2. 2EAH CAH=
T (1) và (2) ta suy ra:
180DAH EAH+ =
D, A, E thng hàng.
Vì t giác AMHN có
90 90A M N DHE= = = =
Áp dng h thức lượng trong tam giác ABC vuông ti A:
CHUYÊN ĐỀ BỒI DƯỠNG HỌC SINH GIỎI HÌNH HỌC 9
89
( )
2
2
. 2 4 .AH BH CH AH BH CH= =
Do D và H đối xứng qua AB, E và H đối xng vi nhau qua AC nên
2.AH DE=
Vì thế:
2
4. .DE BH CH=
Áp dụng định lý Pythagore cho tam giác DHE vuông ti H, ta có:
2 2 2
DE HD HE=+
T đó:
22
4.HD HE BH HC+=
Ví d 3. Cho tam giác ABC vuông tại A, đường cao AH. T H k
( )
HM AC M AC⊥
, gi N là
hình chiếu ca M trên BC. Chng minh
2 2 2 2 2
1 1 1 1 1
AB AC HC MC MN
+ + + =
NG DN GII
Áp dng h thức lượng cho tam giác HMC vuông ti M:
( )
2 2 2
1 1 1
1
MN HM MC
=+
Áp dng h thức lượng cho tam giác AHC vuông ti H:
( )
2 2 2
1 1 1
2
HM HA HC
=+
Áp dng h thức lượng cho tam giác ABC vuông ti A:
( )
2 2 2
1 1 1
3
HA AB AC
=+
T (1) (2) và (3) ta suy ra:
2 2 2 2 2
1 1 1 1 1
AB AC HC MC MN
+ + + =
Ví d 4. Cho tam giác ABC vuông tại A, đường cao AH, gi D và E lần lượt là hình chiếu ca H
trên AB và AC. Chng minh
. . .AE CE AD BD BH HC+=
NG DN GII
Xét t giác ADHE có
90A D E= = =
nên ADHE là hình ch nht.
Suy ra:
( )
2 2 2 2 2
*DE AH DE AH HE HD AH= = + =
(áp dụng định lý Pythagore cho tam giác DHE vuông ti H).
Áp dng h thức lượng trong tam giác AHC vuông ti H:
( )
2
.1HE AECE=
Áp dng h thức lượng trong tam giác AHC vuông:
( )
2
.2HD AD BD=
Áp dng h thức lượng trong tam giác ABC vuông:
( )
2
.3AH BH HC=
Lấy (1), (2) và (3) thay vào (*) ta được:
. . .AE CE AD BD BH HC+=
Ví d 5. Cho tam giác ABC vuông tại A, đường cao AH, gi E và K lần lượt là hình chiếu ca H
CHUYÊN ĐỀ BỒI DƯỠNG HỌC SINH GIỎI HÌNH HỌC 9
90
trên AB và AC. Chng minh
2 2 2 2
1 1 1 1
AK AB AC HB
= + +
NG DN GII
Áp dng h thc trong tam giác AHB vuông ta có:
( )
2 2 2
1 1 1
1
HE HB AH
=+
Áp dng h thc trong tam giác ABC vuông ta có:
( )
2 2 2
1 1 1
2
AH AB AC
=+
Thay (2) vào (1) ta được:
2 2 2 2
1 1 1 1
HE AB AC HB
= + +
T giác AEHK là hình ch nht do có 3 góc vuông nên
AK HE=
.
Vy
2 2 2 2
1 1 1 1
AK AB AC HB
= + +
Ví d 6. Cho tam giác ABC vuông ti A, gi E, K lần lượt là hình chiếu ca H trên AB và AC còn I
là hình chiếu ca A trên EK. Chng minh
2 2 2 2 2
2 2 1 1 1
AB AC AI HB HC
+ =
NG DN GII
Ta thy t giác AEHK là hình ch nht vì có 3 góc vuông nên
;HK AE HE AK==
Áp dng h thc trong tam giác AKE vuông:
222
1 1 1
AI AE AK
=+
Suy ra
( )
2 2 2
1 1 1
1
AI HK HE
=+
Áp dng h thc trong tam giác AHC vuông:
( )
2 2 2
1 1 1
2
HK AH HC
=+
Áp dng h thc trong tam giác AHB vuông:
( )
2 2 2
1 1 1
3
HE AH HB
=+
Áp dng h thc trong tam giác ABC vuông:
( )
2 2 2
1 1 1
4
AH AB AC
=+
T (1) (2) (3) và (4) ta
có:
2 2 2 2 2 2 2 2 2 2
1 1 1 1 1 2 2 1 1 1
2
AI AB AC HA HB AB AC AI HB HC

= + + + + =


Ví d 7. Cho tam giác ABC vuông tại A đường cao AH. Gi E, F là hình chiếu ca H trên AB và
AC. Chng minh rng:
3
3
AB BE
AC CF
=
NG DN GII
Áp dng h thức lượng cho các tam giác vuông:
CHUYÊN ĐỀ BỒI DƯỠNG HỌC SINH GIỎI HÌNH HỌC 9
91
2
2
42
42
3
3
.
.
.
.
AB BH BC BH
AC CH BC CH
AB BH BE AB
AC CH CF AC
AB BE
AC CF
==
= =
=
Ví d 8. Cho tam giác ABC vuông tại A, đường cao AH. K HE vuông góc vi AB, k HF vuông
góc vi AC
( )
;E AB F AC
Gi M, N lần lượt là hình chiếu E, F trên BC. Chng minh:
4
4
AB BM
AC CN
=
NG DN GII
S dng kết qu ca ví d 7 ta được:
3 6 2
3 6 2
AB BE AB BE
AC CF AC CF
= =
Áp dng h thức lượng trong tam giác vuông, ta có:
( )
62
62
.
1
.
AB BE BM BH
AC CF CN CH
==
Mà ta có:
( )
2
2
.
2
.
AB BH BC BH
AC CH BC CH
==
T (1) và (2) ta có:
6 2 4
6 2 4
.
.
AB BM AB AB BM
AC CN AC AC CN
= =
III. BÀI TP VN DNG
Bài 1. Cho hình vuông ABCD. Gi I là một điểm nm gia A và B. Tia DI và tia CB ct nhau K.
K đường thng qua D, vuông góc với DI. Đường thng này cắt đường thng BC ti L. Chng minh
rng:
a) Tam giác DIL là mt tam giác cân.
b) Tng
22
11
DI DK
+
không đổi khi I thay đổi trên cnh AB.
Bài 2. Cho tam giác ABC vuông ti A
( )
AC AB
. Trên cnh AC lấy điểm B’ sao cho
'AB AB=
.
T B’ kẻ đường thng song song vi AC ct AD tại điểm E. Chng minh rng:
2 2 2 2
1 1 1 1
AD AE BD BC
+ = +
Bài 3. Cho hình ch nht ABCD có
.AB k AD=
(k > 0, k không đổi). Qua điểm A k đưng thng
bt kì ct BC ti M ct DC tại điểm I. Chng minh rng:
( )
2
2
11
.
AM
k AI
+
không đổi.
CHUYÊN ĐỀ BỒI DƯỠNG HỌC SINH GIỎI HÌNH HỌC 9
92
Bài 4. Cho hình vuông ABCD cạnh a không đổi. Trên đoạn AD lấy điểm F sao cho
.FDAD k=
(k
> 0, k không đổi). T điểm F k đường thng bt kì ct BC tại điểm M, ct DC tại điểm I. Chng
minh
( )
2
2
11
.FI
FM
k
+
không đổi khi đường thẳng qua F thay đổi.
Bài 5. Cho hình vuông ABCD cạnh a không đổi, ly E và F lần lượt trên DC và AD sao cho
;
aa
EC FD
mn
==
(m, n > 0, m và n không đổi). T F k đưng thng bt kì ct BC ti M, ct DC
ti I. T E k đường thng bt kì ct AB ti N ct BC ti K. Chng minh:
( ) ( )
22
22
1 1 1 1
..
FM EC
n FI m EK
+ = +
Bài 6. Cho hình vuông ABCD cnh a ly E, F lần lượt trên cnh DC và DA sao cho
( )
; , 0
aa
EC FD m n
mn
= =
. T F k đưng thng bt kì ct BC ti M, ct DC ti I. T E k đường
thng vuông góc vi FI ct AB ti N, ct BC ti K. Chng minh:
EK m
FI n
=
Bài 7. Cho tam giác ABC vuông tại A, đường cao AH. Gi E và F lần lượt là hình chiếu ca H trên
AB và AC. Gọi I là giao điểm ca AH và EF. Chng minh rng:
. 4. .BH CH IE IF=
Bài 8. Cho tam giác ABC nhọn, hai đường cao BD và CE ct nhau ti H. Trên đoạn HC và HB ln
t ly M và N sao cho
90AMB ANC= =
. Chng minh
AM AN=
Bài 9. Cho tam giác ABC vuông ti A, cnh huyền BC không đổi. Gọi AH là đường cao ca tam
giác ABC. Gi D và E lần lượt là hình chiếu của H trên AB và AC. Xác định hình dng ca tam
giác ABC để diện tích ADHE đạt giá tr ln nht.
Bài 10. Cho tam giác ABC vuông cân ti A. Gọi M là trung điểm của AC. Đường thng qua A
vuông góc vi BM ct BC ti D. Tính
DC
DB
Bài 11. Cho ABCD là hình thang có AB // CD sao cho tn tại điểm O cách đều bn cnh ca hình
thang. Một đường thng qua O vuông góc vi AB ct AB và CD lần lượt tai E và F. Chng minh
BE DF
AE CF
=
Bài 12. Cho tam giác ABC vuông tại A, đường cao AH, k HD vuông góc vi AB, HE vuông góc
vi AC (D AB; E AC). Chng minh rng:
. .AC HB.HCAD AB AE==
IV. HƯỚNG DN GII
Bài 1.
a) ADI = CDL(g.c.g)
DI DL=
DIL cân.
CHUYÊN ĐỀ BỒI DƯỠNG HỌC SINH GIỎI HÌNH HỌC 9
93
b) Áp dng h thức lượng trong tam giác DLK vuông ti D:
2 2 2 2 2 2
1 1 1 1 1 1
DL DK DC DI DK DC
+ = + =
không đổi
Bài 2.
Gọi C’ là giao điểm của B’D với BE như hình vẽ:
ABC’B’ là hình vuông cạnh a.
Theo bài trên thì:
2 2 2
1 1 1
AD AE a
+=
2 2 2
1 1 1
BD BC a
+=
T đó suy ra đpcm.
Bài 3.
Qua A k đường thng vuông góc vi AM ct ct DC ti E. Ta thy:
ABM ~ ADE(g.g)
.
AM AB
k AM k AE
AE AD
= = =
Vy
( ) ( ) ( )
2 2 2
2 2 2 2 2 2
1 1 1 1 1 1 1 1
. . .
AM k AE AI k AD
k AI k AE k AI

+ = + = + =


không đổi
Bài 4.
Theo bài 3 thì
( )
2
2 2 2
1 1 1 1
.
.
FM k FD
k FI
+=
Cơ mà
a
FD
k
=
nên
( )
2
22
1 1 1
.
FM a
k FI
+=
không đổi
Bài 5.
Theo bài 4 thì:
( )
2
22
1 1 1
.
FM a
n FI
+=
( )
2
22
1 1 1
.
EN a
m EK
+=
đpcm
Bài 6.
Theo bài 4 thì:
( ) ( )
22
22
1 1 1 1
..
FM EN
n FI m EK
+ = +
( ) ( )
22
11
..
EN FM EN FM
n FI m EK
= =
EK n
FI m
=
(đpcm).
Bài 7.
Ta thy EHF là hình ch nht nên:
22IE IF AH==
Áp dng h thức lượng trong tam giác vuông ABC, ta có
2
.AH BH CH=
Vì thế
( )
2
2
2 . . 4 4 .IE BH CH BH CH IE IE IF= = =
CHUYÊN ĐỀ BỒI DƯỠNG HỌC SINH GIỎI HÌNH HỌC 9
94
Bài 8.
Áp dng h thc cho tam giác ABM vuông ti M
( )
2
.1AM AE AB=
Áp dng h thc cho tam giác ACN vuông ti N
( )
2
.2AN AD AC=
Do ABD ~ ACE
( )
. . 3
AD AB
AD AC AE AB
AE AC
= =
T (1), (2), (3) suy ra:
22
AM AN=
Vy
AM AN=
Bài 9.
Gọi AM là đường trung tuyến ca tam giác ABC
2
BC
AM=
không đổi
Áp dng h thc trong tam giác AHB
2
2
.
AH
AE AB AH AE
AB
= =
Áp dng h thc trong tam giác AHC
2
2
.
AH
AD AC AH AD
AC
= =
Khi đó
4 4 3 3 2
.
. . 8
ADHE
AH AH AH AM BC
S AD AE
AB AC AH BC BC BC
= = = = =
(không đổi)
Vy S
ADHE
đạt giá tr ln nht khi H trùng vi M hay ABC là tam giác vuông cân ti A
Bài 10.
K thêm
;CK AD K AD⊥
Gọi H là giao điểm ca AD vi BM.
Vì BH // CK nên:
( )
1
DC CK
DB BH
=
2CK HM=
nên
( )
2
2
DC HM
DB BH
=
Áp dng h thc v cạnh và đường cao trong tam giác ta có:
2
2
2
.1
.4
AM HM BM HM AM
AB BH BM BH AB

= = =


Thay vào (2) ta được:
1
2
DC
DB
=
Bài 11.
Do AB // CD nên
180AD+ =
CHUYÊN ĐỀ BỒI DƯỠNG HỌC SINH GIỎI HÌNH HỌC 9
95
90A ADO + =
AOD vuông ti O.
Áp dng h thức lượng trong tam giác vuông. Ta có
2
2
.
..
.
OM AM DM
AM DM BN CN
ON BN CN
=
=
=
Vì theo tính cht tam giác bng nhau nên:
; ; ;AM AE DM DF BN BE CN CF= = = =
T đó suy ra:
.DF BE.CF
BE DF
AE
AE CF
= =
Bài 12.
Áp dng h thc gia cạnh và đường cao trong tam giác vuông ta có:
2
2
2
.
.
.
AD AB AH
AE AC AH
BH CH AH
=
=
=
T đó suy ra:
. . .AD AB AE AC HB HC==
BÀI TOÁN 9.
S DỤNG ĐỊNH LÍ VAN AUBEL ĐỂ CHỨNG MINH ĐẲNG THC VÀ BẤT ĐẲNG
THC
I. KIN THC CN NH
1. Định lý Van Aubel:
Trên ba cnh BC, CA, AB ca tam giác ABC lần lượt ly ba điểm M, N, K sao cho AM, BN, CK
đồng quy tại E. Khi đó:
AE AK AN
EM KB NC
=+
Chng minh:
Cách 1:
Áp dụng định lý Menelaus cho ABM đối với ba điểm K, E, C
thng hàng, ta có:
CHUYÊN ĐỀ BỒI DƯỠNG HỌC SINH GIỎI HÌNH HỌC 9
96
( )
. . 1 . 1
AK BC ME AK CM AE
KB CM AE KB BC ME
= =
Áp dụng định lý Menelaus cho ACM đối với ba điểm B, E, N thng hàng, ta có:
( )
. . 1 . 2
AN BC ME AN BM AE
NC BM AE NC BC ME
= =
T (1) và (2) suy ra:
.
AN AK AE CM BM AE
NC KB ME BC BC ME

+ = + =


Cách 2: T A k đường thng song song vi BC ct tia CK, tia BN lần lượt ti D và F.
Áp dng h qu định lý Thales, ta có:
AE AD AF
EM CM BM
==
Áp dng tính cht dãy t s bằng nhau, ta được
( )
1
AE AD AF AD AF AD AF
EM CM BM BC BC BC
++
= = = +
+
Áp dng h qu định lý Thales, ta có:
( )
2
AD AK
BC KB
=
( )
3
AF AN
BC NC
=
Thay (2) và (3) vào (1) ta được
AE AK AN
EM KB NC
=+
Cách 3.
Ta có
AEC
AKE
BKE BEC
ANE
AEB
CEN BEC
S
S
AK
KB S S
S
S
AN
NC S S
==
==
Suy ra
( )
1
AEC AEB
BEC
SS
AK AN
KB NC S
+
+=
Li có
( )
2
AEC AEB ABC AEB AEC
AEB
BEM CEM BEM CEM BEC
S S S S S
S
AE
ME S S S S S
++
= = = =
+
T (1) và (2) suy ra
AK AN AE
KB NC ME
+=
2. Nhn xét:
* Trường hợp: Điểm E nằm ngoài tam giác ABC, điểm M
nm trên cạnh BC còn điểm K, N là hai điểm nm trên tia AB,
tia AC, (hình 4) khi đó:
CHUYÊN ĐỀ BỒI DƯỠNG HỌC SINH GIỎI HÌNH HỌC 9
97
Ta có
ACE ABE ACE ABEC
ABE
BEM CEM BEM CEM BEC
S S S S
S
AE
EM S S S S S
+
= = = =
+
Ta có
AKC AKC AKE AEC
AKE
BKC BKE BKC BKE BEC
S S S S
S
AK
BK S S S S S
= = = =
Tương tự
AEB
BEC
S
AN
NC S
=
Suy ra
AEC ABEC
AEB
BEC BEC BEC
SS
S
AK AN
BK NC S S S
+ = + =
Suy ra
AK AN AE
BK NC ME
+=
Như vậy, h thc của định lý Van Auhel không b thay đổi do việc điểm E nm bên trong hay bên
ngoài tam giác ABC.
II. CÁC VÍ D
Ví d 1. (Chng minh tính cht trng tâm ca tam giác)
Cho ABC có ba đường trung tuyến AM, BN, CK ct nhau tại E. Khi đó
2
3
AE
AM
=
NG DN GII
(Bạn đọc t v hình)
Áp dụng định lý Van Aubel ta có:
1 1 2 2
AE AK AN
AE EM
AM KB NC
= + = + = =
Vy
2
3
AE
AM
=
Nhn xét: Bài này có th chng minh bng s dụng đường trung bình ca tam giác.
Ví d 2. Cho ABC có
;;BC a AC b AB c= = =
. Gọi I là tâm đường tròn ni tiếp ABC, tia AI ct
BC tại A’. Chứng minh
'
AI c b
IA a
+
=
NG DN GII
Gi tia CI ct AB tại C’, tia BI cắt AC tại B’
Áp dụng định lý Van Aubel cho ABC ta có:
''
' ' '
AI AB AC
IA B C C B
=+
Theo tính chất đường phân giác ca tam giác thì:
''
;
''
AB c AC b
B C a C B a
==
T đó
'
AI c b
IA a
+
=
CHUYÊN ĐỀ BỒI DƯỠNG HỌC SINH GIỎI HÌNH HỌC 9
98
Ví d 3. Trên ba cnh BC, CA, AB ca tam giác ABC lấy 3 điểm H, M, N sao cho AH, BM, CN
đồng quy ti G. Gi P, Q lần lượt là giao điểm ca HN vi BM và HM vi CN. Tia AP, AQ ct
cnh BC lần lượt ti E và F. Chng minh rng:
3.
AP AQ AN AM
PE QF NB MC

+ = +


NG DN GII
Áp dụng định lý Van Aubel cho tam giác ABH với AE, BG, HN đồng quy:
( )
1
AP AN AG
PE NB GH
=+
Áp dụng định lý Van Aubel cho tam giác AHC vi AF, CG,
HM đồng quy:
( )
2
AQ AM AG
QF MC GH
=+
Áp dụng định lý Van Aubel cho tam giác ABC vi AH, CN,
BM đồng quy
( )
3
AG AN AM
GH NB MC
=+
T (1) (2) (3) suy ra:
3.
AP AQ AN AM
PE QF NB MC

+ = +


Nhn xét:
- Trường hp H là trung điểm BC thì MN // BC hay
6.
AN AM AP AQ AN
NB MC PE QF NB
= + =
- Trường hợp G là trung điểm AH thì
13
AN AM AP AQ
NB MC PE QF
+ = + =
Ví d 4. Cho tam giác ABC, trên BC, CA, AB lần lượt lấy A’, B’, C’ sao cho AA’, BB’, CC’ đồng
quy ti K. Gọi giao điểm của A’B’ với CC’ là N, giao điểm của A’C’ với BB’ là M. Tia AM, AN
lần lượt ct BC ti E và F. Chng minh:
a) EN, FM, AA’ đồng quy ti I.
) . ' 3. '.b AI KA IA AK=
NG DN GII
a) Áp dụng định lý Menelaus cho ABE
''
. . 1
''
''
.
''
AM EA BC
ME A B C A
AM C A A B
ME BC EA
=
=
Áp dụng định lý Menelaus cho AFC
CHUYÊN ĐỀ BỒI DƯỠNG HỌC SINH GIỎI HÌNH HỌC 9
99
''
. . 1
''
''
.
''
FN AB CA
NA B C A F
FN A F B C
NA CA AB
=
=
Khi đó
' ' ' ' ' '
. . . . . .
' ' ' ' ' '
AM EA FN C A A B EA A F B C
ME A F NA BC EA A F CA AB
=
'''
. . 1
' ' '
C A A B B C
BC CA AB
==
(do AA’,BB’,CC’ đồng quy ti K)
'
. . 1
''
AM EA FN
ME A F NA
=
theo định lý Ceva thì AA’, EN và FM đồng quy tại điểm I.
b) Áp dụng định lý Van Aubel cho ABA', ACA', AEF ta được:
( ) ( ) ( )
''
1 ; 2 ; 3
' ' ' ' '
AM AK AC AN AK AB AM AN AI
ME KA C B NF KA B C ME NF IA
= + = + + =
Thay (1), (2) vào (3) ta được:
( )
''
2. 4
' ' ' '
AK AC AB AI
KA C B B C IA
+ + =
Áp dng tiếp định lý Van Aubel cho tam giác ABC:
''
' ' '
AC AB AK
C B B C KA
+=
Thay vào (4) ta được
3.
''
AK AI
KA IA
=
3. . ' '.AK IA KA AI=
Ví d 5. Cho K là điểm nm trong tam giác ABC. Gọi D là giao đim ca AK vi BC, E là giao
điểm ca BK với AC còn F là giao điểm ca CK vi AB. Chng minh rng
6
AK BK CK
KD KE KF
+ +
NG DN GII
Áp dụng định lý Van Aubel cho tam giác ABC:
;;
AK AE AF BK FB BD KC EC DC
KD EC FB KE AF DC FK AE BD
= + = + = +
Cng hai vế của các đẳng thức ta được:
6
AK BK KC AE AF FB BD EC DC AE EC AF FB BD DC
KD KE FK EC FB AF DC AE BD EC AE FB AF DC BD
+ + = + + + + + = + + + + +
(theo bất đẳng thc AM - GM)
Dấu “=” xảy ra khi K là trng tâm ca tam giác ABC.
Nhn xét: NG DN GII s dụng định lý Van Aubel rt t nhiên
và đơn giản hơn phương pháp sử dng diện tích để chng minh bài
này!!
Ví d 6. Cho tam giác ABC có đường trung tuyến AD, gọi M là trung điểm AD, giao điểm ca BM
CHUYÊN ĐỀ BỒI DƯỠNG HỌC SINH GIỎI HÌNH HỌC 9
100
với AC là E, giao điểm ca CM vi AB là F. Chng minh rng:
1
MF ME
CM MF BM ME
+=
−−
NG DN GII
Áp dụng định lý Van Aubel và do D là trung điểm ca BC nên:
( )
( )
11
12
BM BF BD BF BM BM ME
ME AF DC AF ME ME
CM CE CD EC CM CM MF
MF AE BD AE MF MF
= + = =
= + = =
Áp dụng định lý Van Aubel và do M là trung điểm ca AD nên:
( )
13
AE AF AM AE AF
EC FB MD EC BF
+ = + =
T (1) (2) và (3) ta có:
1
MF ME
CM MF BM ME
+=
−−
III. MT S BÀI TP NG DNG CỦA ĐỊNH LÝ VAN AUBEL
Bài 1. Cho tam giác ABC nhn ni tiếp (O), các đường thng AO, BO, CO ct các cnh BC, CA,
AB lần lượt tại D, E, F. Đặt
;;
BD CE AF
x y z
DC EA FB
= = =
. Chng minh rng
1 yx 1 1
a b c
h h h
Rr
xz z x zy y
+ + = +
+ + + + + +
Bài 2. Cho P là điểm bên trong ABC có tha mãn
PBC PAC
PBA
SS
S
a b c
==
Gi D, E, F lần lượt là giao
điểm ca AP vi BC, BP vi AC và CP vi AB. Tính giá tr biu thc
AP PB CP
PD PE PF
++
theo a, b, c
Bài 3. Cho tam giác ABC, trên ba cnh BC, CA, AB lần lượt ly 3 điểm D, E, F sao cho AD, BE,
CF đồng quy ti P. Chng minh rng:
( )
. . 2 *
AP PB CP AP PB CP
PD PE PF PD PE PF
= + + +
Bài 4. Cho tam giác ABC có
;;AB c BC a CA b= = =
, và I là tâm đường tròn ni tiếp tam giác
ABC, các tia AI, BI, CI lần lượt ct BC, AC, AB tại A’ , B’ , C’ . Chứng minh rng
8
..
' ' ' 27
AI BI CI
AA BB CC
IV. HƯỚNG DN GII
Bài 1. S dụng định lý Ceva:
. . 1 1
BD CE AF
xyz
DC EA BF
= =
S dụng định lý Van Aubel:
1.
1
a
a
AO AF AE AO
z xz
OD FB EC OD
h
AD OD
xz z h
OD xz z AD
= + = +
+ + = =
++
CHUYÊN ĐỀ BỒI DƯỠNG HỌC SINH GIỎI HÌNH HỌC 9
101
Gọi A’ là trung điểm BC, s dng Thales:
'
'
1
a
a
h
OA OD
OA
h AD xz z
= =
++
Tương tự gọi B’ và C’ lần lượt là trung điểm AC và AB ta có:
' ' '
1 yx 1 1
a b c
h h h
OA OB OC
xz z x zy y
+ + = + +
+ + + + + +
S dng h thc Carnot thì:
' ' 'OA OB OC R r+ + = +
T đó suy ra điều phi chng minh.
Bài 2. S dng công thc tính din tích tam giác:
PAB
PAC
S
BH BD c
S CK DC b
= = =
Tương tự
;
CE a AF b
EA c FB a
==
Áp dụng định lý Van Aubel:
AP AE AF c a
PD EC FB a
PB BF BD a c
PE FA DC b
CP CE CD a b
PF AE DB c
+
= + =
+
= + =
+
= + =
T đó sẽ tính được giá tr ca biu thc
AP PB CP
PD PE PF
++
Bài 3. Kí hiu
;;
PBC APC BPA
S a S b S c= = =
S dng công thc tính din tích tam giác ta có:
;;
BD c CE a AF b
DC b EA c FB a
= = =
S dụng định lý Van Aubel ta có:
;;
AP AE AF b c BP BF BD a c CP CD CE a b
PD EC FC a PE FA DC b PF DB EA c
+ + +
= + = = + = = + =
H thc (*) cn chứng minh tương đương với:
( )( )( )
( )( )( ) ( ) ( ) ( )
2
2
a b b c c a
a b b c c a
abc c a b
a b b c c a ab a b bc b c ca c a abc
+ + +
+ + +
= + + +
+ + + = + + + + + +
Đẳng thc cuối cùng luôn đúng, vậy nên đẳng thức (*) đã được chng minh.
Bài 4.
CHUYÊN ĐỀ BỒI DƯỠNG HỌC SINH GIỎI HÌNH HỌC 9
102
Ta có
1 1 1
. . . .
' ' '
' ' '
1 1 1
AI BI CI
IA IB IC
AA BB CC
AI BI CI
=
+ + +
S dụng định lý Van Aubel có kết qu ví d 2 ta được:
1 1 1
. . . . . .
' ' '
1 1 1
AI BI CI b c a c a b
a b c
AA BB CC a b c a b c a b c
b c a c a b
+ + +
==
+ + + + + +
+ + +
+++
Đặt
;;a x y b y z c z x= + = + = +
ta có
1
. . 1 1 1
' ' ' 8
AI BI CI y z x
AA BB CC x y z x y z x y z
= + + +
+ + + + + +
Áp dng bất đẳng thức “Trung bình cộng - Trung bình nhân” ta có:
3
111
18
..
' ' ' 8 3 27
y z x
AI BI CI
x y z x y z x y z
AA BB CC

+ + + + +

+ + + + + +

=



Du = xảy ra khi ABC là tam giác đều.
CHUYÊN ĐỀ BỒI DƯỠNG HỌC SINH GIỎI HÌNH HỌC 9
103
MT S BÀI TOÁN TRÍCH TRONG CÁC ĐỀ THI CHUYÊN VÀ HSG
Bài 1. Cho đường tròn tâm O, bán kính R. T một điểm M ngoài đường tròn, k hai tiếp tuyến MA
MB với đường tròn (A, B là các tiếp điểm). Qua A, k đường thng song song vi MO cắt đường
tròn ti E (E khác A), đường thng ME cắt đường tròn ti F (F khác E), đường thng AF ct MO ti
N, H là giao điểm ca MOAB.
1) Chng minh: T giác MAOB ni tiếp đường tròn.
2) Chng minh:
2
.MN NF NA=
MN NH=
.
3) Chng minh:
2
2
1
HB EF
HF MF
−=
(Trích đề thi vào 10, Hải Dương, Năm học 2017 - 2018)
NG DN GII
1)
90 180MAO MBO MAO MBO= = + =
Mà hai góc đối nhau nên t giác MAOB ni tiếp.
2) Ch ra
MNF ANM
(g.g)
suy ra
2
.MN NF NA=
Ch ra
NFH AFH
(g.g)
suy ra
2
.NH NF NA=
Vy
22
MN NH=
suy ra
MN NH=
.
MA MB=
(tính cht 2 tiếp tuyến ct nhau) và
OA OB R==
.
Suy ra MO là đường trung trc ca AB, nên
AH MO
HA HB=
.
Xét hai tam giác
MAF
MEA
có:
AME
chung,
MAF AEF=
nên
MAF MEA
(g.g), suy ra
2
.
MA MF
MA MF ME
ME MA
= =
Áp dng h thức lượng vào
vuông MAO, có:
2
.MA MH MO=
Do đó:
..ME MF MH MO=
hay
ME MO
MH MF
=
Suy ra
MFH MOE
, do đó
MHF MEO=
.
Vì
BAE
là góc vuông ni tiếp
( )
O
nên E, O, B thng hàng.
Suy ra
1
2
FEB FAB sdEB MHF FAB
Nên
90ANH NHF ANH FAB HF NA
Áp dng h thức lượng vào tam giác vuông NHA, có:
CHUYÊN ĐỀ BỒI DƯỠNG HỌC SINH GIỎI HÌNH HỌC 9
104
2 2 2
.NH NF NA NM NH NM NH
3) Áp dng h thức lượng vào tam giác vuông NHA, có:
2
.HA FA NA
2
.HF FA FN
HA HB
nên
22
22
.
.
HB HA FA NA NA
HF HF FA FN NF
,
suy ra
2
.HB AF AN
(vì
HA HB
)
//AE MN
nên
EF FA
MF NF
(h qu của định lí Thales)
Nên suy ra
2
2
1
HB EF NA FA NF
HF MF NF NF NF
(đpcm)
Bài 2. Cho tam giác ABC, M là điểm bt kì nm trong tam giác. Kéo dài AM ct BC ti P, BM ct
AC ti Q, CM ct AB ti K. Chng minh rng
. . 8 . .MA MB MC MP MQ MK
(Trích đề thi vào 10 , Tỉnh Thái Bình, Năm học 2017 - 2018)
NG DN GII
Đặt
;;
MBC MAC MAB
a S b S c S
. Ta có:
22
,,
MA b c bc MB a b ab
MP a a MQ c c
2MC a c ac
MK b b
.
Suy ra
. . 8
MA MB MC
MP MQ MK
. . 8 . .MA MB MC MP MQ MK
Dấu “=” xảy ra khi M là trng tâm ca tam giác ABC.
Bài 3. Cho t giác ABCD ni tiếp đường tròn
;OR
. Gi I là giao điểm ACBD. K IH vuông
góc vi AB; IK vuông góc vi AD (
;H AB K AD
).
1) Chng minh t giác AHIK ni tiếp đường tròn.
2) Chng minh rng
..IA IC IB ID
.
3) Chng minh rng tam giác HIK và tam giác BCD đồng dng.
4) Gi S là din tích tam giác ABD, S’ là din tích tam giác HIK. Chng minh rng:
2
2
4.
S HK
S AI
(Trích đề thi vào 10, Tnh Phú Thọ, năm học 2017 - 2018)
NG DN GII
CHUYÊN ĐỀ BỒI DƯỠNG HỌC SINH GIỎI HÌNH HỌC 9
105
1) T giác AHIK có:
90AHI IH AB
,
90AKI IK AD
180AHI AKI
Do đó tứ giác AHIK ni tiếp.
2) Xét
IAD
IBC
có:
11
AB
(2 góc ni tiếp cùng chn cung DC ca
O
)
AID BIC
(2 góc đối đỉnh),
suy ra
IAD IBC
(g.g)
do đó
..
IA ID
IA IC IB ID
IB IC
3) Xét đường tròn ngoi tiếp t giác AHIK
11
AH
(2 góc ni tiếp cùng chn cung IK)
1 1 1 1
A B H B
Chứng minh tương tự, ta được
11
KD
Xét
HIK
BCD
có:
1 1 1 1
;H B K D
nên
HIK BCD
(g.g)
4) Gi
1
S
là din tích ca
BCD
. Vì
HIK BCD
nên:
2 2 2 2
2
2
1
4 . 4 .
S HK HK HK HK
S BD IB ID IA IC
IB ID
(1)
V
, / /
CF IC
AE BD CF BD AE CF
AE IA
ABD
BCD
có chung cạnh đáy BDn:
11
SS
CF IC
S AE S IA
(2)
T (1) và (2) suy ra
22
1
2
1
..
4 . 4
S
S HK IC S HK
S S IA IC IA S IA
(đpcm).
Bài 4. Cho tam giác ABC vuông tai A đường cao AH đường tròn tâm E đường kính BH ct AB ti M
(M khác B), đường tròn tâm F đường kính HC ct AC ti N (N khác C)
1) Chng minh
..AM AB AN AC
2
.AN AC MN
.
2) Gi I là trung điểm ca EF, O là giao điểm ca AHMN. Chng minh IO vuông góc với đường
thng MN.
3) Chng minh
2 2 2 2
46EN FM BC AH
.
CHUYÊN ĐỀ BỒI DƯỠNG HỌC SINH GIỎI HÌNH HỌC 9
106
(Trích đề thi vào 10, Tỉnh Nam Định, Năm hc 2017 - 2018)
NG DN GII
1) Ta có
90BMH HNC
(các góc ni tiếp chn nửa đường tròn), suy ra
,HM AB HN AC
.
Áp dng h thức lượng vào tam giác vuông AHBAHC, có
2
.AH AM AB
2
. . .AH AN AC AM AB AN AC
Mt khác, t giác AMHN có ba góc vuông nên là hình ch nhật khi đó
2
.AH MN AN AC MN
2) T giác AMHN là hình ch
nhật, có O là giao điểm ca AH
và MN. Suy ra O là trung điểm
của AH và MN. Khi đó
EMO EHO
(c.c.c)
90EMO EHO
EM MN
Chứng minh tương tự ta có
FN MN
.
Suy ra
//ME NF MEFN
hình thang vuông.
OI là đường trung bình ca
hình thang vuông MEFN nên
OI MN
.
3) Đặt
,,MN AH h x y
lần lượt là bán kính của đường tròn
E
F
.
Ta có
2 2 2 2 2 2
44EN FM ME MN ME MN
2 2 2
42x y h
2
2 2 2 2 2 2
6 6 2 . 6BC AH HB HC h HB HC HB HC h
2 2 2 2 2 2 2
4 4 2 6 4 2x y h h x y h
Vy
2 2 2 2
46EN FM BC AH
Bài 5. Cho nửa đường tròn tâm O đường kính AB C là một điểm trên nửa đường tròn (C khác A
B). Trên cung AC lấy điểm D (D khác AC). Gi H là hình chiếu vuông góc ca C trên AB
E là giao điểm ca BD CH.
1) Chng minh ADEH là t giác ni tiếp.
CHUYÊN ĐỀ BỒI DƯỠNG HỌC SINH GIỎI HÌNH HỌC 9
107
2) Chng minh rng
ACO HCB
. . .AB AC AC AH CB CH
3) Trên đoạn OC lấy điểm M sao cho
OM CH
. Chng minh rng khi C chy trên nửa đường
tròn đã cho thì M chy trên một đường c định.
(Trích đề thi vào 10 TP Đà Nẵng, năm học 2017 - 2018)
NG DN GII
1) Ta có:
90ADE
(góc ni tiếp chn na đưng tròn)
90AHE
(do
CH AB
)
Suy ra
180ADE AHE
suy ra, t giác ADEH ni tiếp.
2) Ta có:
ACO CAO
(
OAC
cân ti O).
ACO HCB
(cùng ph
CBH
).
Suy ra:
ACO HCB
.
Xét
ACB
CHB
có:
90ACB CHB
,
ABC
chung.
Suy ra
..
AC BC
ACB CHB AC BH CB CH
CH BH
(*)
BH AB AH
thay vào (*) ta được:
. . .AC AB AC AH CB CH
(đpcm).
3) Gi K là điểm chính gia cung AB (chứa điểm C).
Suy ra
//OK AB OK HC
.
Xét
OMK
CHO
có:
MOK HCO
(so le trong),
OM CH
(gi thiết),
OK CO
(cùng bng bán kính). Suy ra
OMK CHO
(c.g.c).
Suy ra
OMK CHO
(hai góc tương ứng bng nhau)
90 90CHO OMK
Vy M chạy trên đường tròn đường kính OK c định. (đpcm).
CHUYÊN ĐỀ BỒI DƯỠNG HỌC SINH GIỎI HÌNH HỌC 9
108
Bài 6. Cho hình vuông ABCD ni tiếp đường tròn
O
. Điểm M thuc cung nh CD ca
O
, M
khác CD. MA ct DB, DC theo th t X, Z; MB ct CA, CD ti Y, T; CX ct DY ti K.
1) Chng minh rng:
,MXT TXC MYZ ZYD
135CKD
.
2) Chng minh rng:
1
KX KY ZT
MX MY CD
.
3) Gi I là giao điểm ca MKCD. Chng minh rng XT, YZ, OI cùng đi qua tâm đường tròn
ngoi tiếp tam giác KZT.
(Trích đề thi vào 10 Chuyên Toán, ĐHSP HN, năm học 2012 - 2013)
NG DN GII
a) Ta có
1
2
DXM sdDM sd AB DTM
nên t giác DXTM ni tiếp.
90 90DMT DXT
Suy ra
TX BD
, mà
D / /AC B TX AC
Do đó
MXT ZAC XCA TXC
Tương tự, t giác MCYZ ni tiếp
Suy ra
//ZY BD
nên
MYZ MBD BDY ZYD
Ta có t giác ADZY ni tiếp nên
YDC YAZ MDC
Tương tự t giác BCTX ni tiếp nên
XCD XBM MCD
Nên
. . 135DMC DKC g c g DKC DMC
Ta có
180 45XKD DKC DMX
nên t giác DXKM ni tiếp.
DXTM ni tiếp nên 5 điểm D, X, K, T, M cùng nm trên một đường tròn tâm E đường kính DT.
Tương tự 5 điểm Y, K, Z, M, C nằm trên đường tròn tâm F đường kính ZC suy ra
..XK XC XZ XM
Suy ra
XK XZ XZ DZ DZ
XM XC XA BA DC
Tương tự
YK CT
YM CD
nên
1
XK KY ZT DZ CT ZT
XM YM CD CD
CHUYÊN ĐỀ BỒI DƯỠNG HỌC SINH GIỎI HÌNH HỌC 9
109
c) Gi H là giao điểm XTYZ. Ta chng minh H là tâm đường tròn ngoi tiếp tam giác KZT.
Ta có
45HZT HTZ HT HZ
(1)
T giác KHZX ni tiếp, nên:
HKZ HXZ HXK HZH HK HZ
(2)
T (1) và (2) suy ra H là tâm đường tròn ngoi tiếp tam giác KZT.
Gọi giao điểm ca XCYZF.
Do
90FKZ
nên F, H, Z thng hàng.
Tương tự, gi XT giao ID ti E. Ta có E, H, T thng hàng
HEF COD
suy ra
HF EF HT FT
OC CD OC CB
//FT BC
nên
FT IT HT IT
BC IC OC IC
, mà
45HTI OCI
Nên suy ra
ITH IOC
do đó
HTI OCT
, hay O, I, H thng hàng.
Bài 7. Cho tam giác ABC vuông ti A, ni tiếp trong đường tròn
O
. Trên cung BC không cha A,
lấy điểm M tu ý (M khác C). P là điểm trên cnh BC sao cho
BAM PAC
. Trên các tia AB, AC
ly lần lượt các điểm E, F sao cho
BE CF BC
.
1) Chng minh:
ABP AMC
. . .MC AB MB AC MA BC
.
2) Chng minh
..MB AE MC AF
MA MB MC
BC
3) Xác định v tri điểm N trên đường tròn
O
để tng
NA NB NC
ln nht.
(Trích đề thi vào 10 Chuyên Toán, Quảng Bình, năm học 2012 - 2013)
NG DN GII
Ta có:
ABP AMC
(cùng chn cung AC)
BAM PAC BAP MAC
CHUYÊN ĐỀ BỒI DƯỠNG HỌC SINH GIỎI HÌNH HỌC 9
110
Nên:
ABP AMC
Suy ra:
..
AB BP
MC AB MA BP
MA MC
(1)
Mt khác:
,BMA BCA BAM PAC ABM APC
..
MB MA
MB AC MA PC
PC AC
(2)
T (1) và (2) suy ra:
. . .MC AB MB AC MA BC
T kết qu câu a) ta có:
..
AC AB
MA MB MC
BC BC
Do đó:
. 1 1
AC AB
MA MB MC MB MC
BC BC
..
AC BC AB BC
MB MC
BC BC
..
AC CE AB BF
MB MC
BC BC
..MB AE MC AF
BC
Xét trường hp N thuc cung BC không cha A.
- Nếu N khác C theo kết qu câu b) ta có
..NB AE NC AF
NA NB NC
BC
(3)
- Nếu N trùng C, ta thy (3) vẫn đúng.
Mt khác
2 2 2 2
2 . . . .NB AF NC AE NB AF NC AE
2
2 2 2 2 2 2
. . .NB AE NB AF NB NC AE AF BC EF
(4)
T (3) và (4) suy ra
NA NB NC EF
Du bng xy ra khi và ch khi
..NB AF NC AE
hay
NBC AEF
.
Xét trường hp N thuc cung BC cha A. Lấy N’ đối xứng N qua BC, khi đó N’ thuộc cung BC
không cha A,
,,N A NA N B NB N C NC
.
Áp dụng trường hp trên ta có:
NA NB NC N A N B N C EF
Vy trong mọi trường hp thì
NA NB NC
có giá tr ln nhất là EF, đạt được khi
NBC AEF
Bài 8. Cho tam giác OAB vuông cân ti O vi
2OA OB a
. Gi
O
là đường tròn tâm O bán
kính a. Tìm điểm M thuc
O
sao cho
2MA MB
đạt giá tr nh nht.
CHUYÊN ĐỀ BỒI DƯỠNG HỌC SINH GIỎI HÌNH HỌC 9
111
(Trích đề thi vào 10 Chuyên Toán, TP HCM, năm học 2010 - 2011)
NG DN GII
Đưng thng OA ct
O
ti C và D, B
với C là trung điểm ca OA. Gi E là
trung điểm ca OC.
* Trường hp M không trùng vi C và D.
Hai tam giác OEM và OMA đồng dng
(do
1
,
2
OM OE
MOE AOM
OA OM
)
Suy ra
1
2.
2
ME OM
MA EM
AM OA
* Trường hp M trùng vi C:
2. 2.MA CA EC EM
* Trường hp M trùng vi D:
2. 2.MA DA ED EM
Vy ta luôn có
2.MA EM
. Do đó
2. 2 2.MA MB EM MB EB
Dấu “=” xảy ra khi M là giao điểm của đoạn BE với đường tròn
O
.
Vy
2.MA MB
nh nhất khi M là giao điểm của đoạn BE với đường tròn
O
.
Bài 9. Cho ba điểm A, M, B phân bit, thng hàng và M nm gia A, B. Trên cùng mt na mt
phng b là đường thng AB, dựng hai tam giác đều AMC và BMD. Gọi P là giao điểm ca AD và
BC.
1) Chng minh AMPC và BMPD là các t giác ni tiếp.
2) Chng minh
..CP CB DP DA AB
.
3) Đường thng ni tâm của hai đường tròn ngoi tiếp hai t giác AMPC và BMPD ct PA, PB
tương ứng ti E, F. Chng minh CDFE là hình thang.
(Trích đề thi vào 10 Chuyên Toán (Vòng 1), ĐHSP Hà Nội, năm hc 2016-2017)
NG DN GII
1) Vì
60 120CMA DMB CMB DMA
- Xét
CMB
AMD
(c.g.c)
CM AM
CMB DMA CMB AMD
MB MD
MCB MAD
MBC MDA
CHUYÊN ĐỀ BỒI DƯỠNG HỌC SINH GIỎI HÌNH HỌC 9
112
Suy ra AMPC và BMPD là các t giác ni tiếp.
2) Vì AMPC là t giác ni tiếp nên
180 120CPM CAM CMB
(g.g)
CP CM
CPM CMB
CM CB
2
..CP CB CM CP CB CM
Tương tự
.DP DA DM
Vy
..CPCB DP DA CM DM AM BM AB
3) Ta có EF là đường trung trc ca PM
EB EM
, suy ra
EPM
cân ti E
Mt khác
60EPM ACM
(do AMPC là t giác ni tiếp)
nên
EPM
đều, nên
PM PE
.
Tương tự
PF PM
.
Ta có
//CM DB
nên
PCM PBD
Mà BMPD là t giác ni tiếp nên
PBD PMD
.
Suy ra
PCM PMD
Ta li có
120CPM DPM
(g.g)
CP PM CP PE
CPM MPD
MP PD PF PD
Theo định lý Thales đảo ta có
//CE DF
, suy ra CDFE là hình thang.
Bài 10. Cho tam giác ABC. Trên các cnh BC, CA, AB lần lượt lấy các điểm D, E, F. Gi
1
d
đường thng qua D và vuông góc vi BC,
2
d
là đường thng qua E và vuông góc vi CA,
3
d
đường thng qua F và vuông góc vi AB. Chng minh rng
12
,dd
3
d
đồng quy khi và ch
khi có đẳng thc sau:
2 2 2 2 2 2
0DB DC EC EA FA FB
.
(Trích đề thi vào lớp 10 Chuyên Toán THTH, ĐHSP TP HCM năm học 2014 - 2015)
NG DN GII
Gọi M là giao điểm ca
1
d
2
d
F
là hình chiếu ca M lên AB.
Ta có
2 2 2 2 2 2
0DB DC EC EA FA FB
2 2 2 2 2 2 2
0BM MD DC EC EA FA FB
CHUYÊN ĐỀ BỒI DƯỠNG HỌC SINH GIỎI HÌNH HỌC 9
113
2 2 2 2 2 2
0BM MC EC EA FA FB
2 2 2 2 2
0BM ME EA FA FB
2 2 2 2
0BM MA FA FB
2 2 2 2
0F B F A FA FB
0F B F A FA FB
(1)
F A F B FA FB AB
nên
1 2 3
1 , ,F B FB F F d d d
đồng quy.
Bài 11. Cho hai đường tròn
11
;OR
22
;OR
vi
12
RR
tiếp xúc trong vi nhau ti A. Đường
thng ct
11
;OR
22
;OR
lần lượt ti BC khác A . Đường thẳng đi qua trung điểm D ca BC
vuông góc vi BC ct
11
;OR
ti PQ.
1) Chng minh C là trc tâm tam giác APQ.
2) Chng minh
2 2 2
12
DP R R
.
3) Gi s
1 2 3 4
; ; ;D D D D
lần lượt là hình chiếu vuông góc ca D xuống các đường thng
; ; ;BP PA AQ QB
. Chng minh:
1 2 3 4
1
2
DD DD DD DD BP PA AQ QB
(Trích đề thi vào 10 Chuyên Toán (Vòng 1) Lê Hồng Phong, Nam Định, năm 2014 - 2015)
NG DN GII
1) Gi M là giao điểm ca AP với đường tròn
2
O
.
Ta có PBQC là hình thoi, nên
//QC BP
, mà
//CM BP
(cùng vuông góc vi AP) nên ta có
Q, C, M thng hàng. Tam giác APQ có hai
đường cao ADQM ct nhau ti C nên C
trc tâm tam giác APQ.
2) Ta chứng minh được DM là tiếp tuyến ca
đường tròn
2
O
.
Ta có
2 2 2 2
22
..PD BD DA DC DA DM O D R
Mt khác
2 2 1
2 2 2
AC BC AB
O D O C CD R
CHUYÊN ĐỀ BỒI DƯỠNG HỌC SINH GIỎI HÌNH HỌC 9
114
Nên ta có
2 2 2
12
PD R R
3) Ta Chứng minh được
1 4 2 3
, , ,DD DD DD DD BP BQ PA PB
nên
1 2 3 4
1
2
DD DD DD DD BP PA AQ QB
12
2 DD DD PA PB
Ta có
2 2 2
1
2.
2 . 2
DB DP
PB BD DP DB DP PB DD
PB
.
Tương tự:
2
2.
2
DA DP
AP DD
PA
. T đó, ta có điều phi chng minh.
Đẳng thc xy ra khi
DA DB DP
.
Bài 12. Cho
ABC
ni tiếp đường tròn
O
. Đường cao
1 1 1
,,AA BB CC
ca tam giác ABC ct nhau
ti H. Đường thng
1
AA
ct
O
ti
KA
.
1) Chng minh rng:
1
A
là trung điểm HK.
2) Tính
1 1 1
HA HB HC
AA BB CC
.
3) Gi M là hình chiếu vuông góc ca O lên BC. Đường thng
1
BB
ct
O
tại giao điểm th hai là
E, đường thng
1
MB
ct AE ti N.
Chng minh rng:
2
1
1
AB
AN
NE EB
.
(Trích đề thi vào 10 Chuyên Toán (Vòng 2) Lê Hồng Phong, Nam Định, năm 2014 - 2015)
NG DN GII
1)Ta có
1 2 1
A C C
nên
CHK
cân ti C.
1
CA
là đường cao nên
1
CA
là đường trung trc, suy ra
1
A
là trung điểm ca HK.
2) Ta có:
1 1 1
HA HB HC
AA BB CC
1 1 1
1 1 1
1 1 1
HA HB HC
AA BB CC
1 1 1
1 1 1
3
HA HB HC
AA BB CC
CHUYÊN ĐỀ BỒI DƯỠNG HỌC SINH GIỎI HÌNH HỌC 9
115
3 3 1 2
HBC HAC
HBA
ABC ABC ABC
SS
S
S S S
3) T gi thiết ta có M trung điểm BC, suy ra
1
B MC
cân ti M, do đó
1 1 1 1 1 1
(g.g) BMBC MCB AB N CBB B AN N AE
Áp dng h thức lượng trong tam giác vuông ta có:
2
1
1
.
.
AB
AN AE AN
EB EN EA EN
(đpcm).
Bài 13. Cho tam giác ABC ni tiếp đường tròn
O
. Lấy điểm D trên cung BC không chứa điểm A
(D khác B, C). Gi H, I, K lần lượt là hình chiếu vuông góc ca D trên các đường thng BC, CA
AB. Chng minh:
BC AC AB
DH DI DK
(Trích đề thi vào 10 Chuyên Nguyễn Du, Đăk Lắk, năm học 2014 - 2015)
NG DN GII
Ta có các t giác BHDK, DHIC, ABDC ni tiếp
KHD KBD ACD
180DHI ACD
180 180DHI KHD KHI
,,K H I
thng hàng.
Mt khác
HCD KAD
(g.g)
HC KA
HCD KAD
DH DK
HB IA
HBD IAD HBD IAD
DH AD
BK IC
BDK BHK CHI CDI BDK CDI
KD DI
Do đó, ta có:
BC BH HC AI AK AI AB BK
DH DH DH DI KD DI KD KD
AI AB CI AC AB
DI KD DI DI DK
.
Bài 14. Cho hai đường tròn
,OO
ct nhau ti AB. T điểm C thuc tia đối ca tia AB k hai
tiếp tuyến đến
O
ti DE, E nm trong
O
. Các đường thng AD, AE ct
O
tại điểm th hai
tương ứng là M, N. Gi I là giao điểm ca DEMN.
1) Chng minh rng t giác BEIN ni tiếp và
BIN BDA
.
CHUYÊN ĐỀ BỒI DƯỠNG HỌC SINH GIỎI HÌNH HỌC 9
116
2) Chng minh rng
22
CA CD DA
CB CB DB
.
3) Chng minh rng I là trung điểm ca MN.
(Trích đề thi vào Chuyên Toán (Vòng 2), ĐHV, năm học 2015-2016)
NG DN GII
1) Vì t giác ABNM ni tiếp nên
BNI BAD
. (1)
Vì t giác DAEB ni tiếp nên
BAD BED
. (2)
T (1) và (2) suy ra t giác BEIN ni tiếp.
Theo chng minh trên,
BNI BAD
.
Ta li có
BIN BEN BDA
(do các t giác BEIN, AEBD ni tiếp).
Suy ra
BIN BDA
(g.g).
2) Vì CD là tiếp tuyến ca
O
nên
2
.CA CB CD
. T đó suy ra
2
2
22
.CA CA CB CD CD
CB CB CB CB
(3)
Li có, t
.
CD DA
CAD CDB g g
CB DB
(4)
T (3) và (4) suy ra
22
CA CD DA
CB CB DB
.
3) Tương tự câu b) ta có
22
CA CE EA
CB CB EB
Suy ra
EA DA
EB DB
(5)
T câu 1),
IN DA
BIN BDA
IB DB
(6)
CHUYÊN ĐỀ BỒI DƯỠNG HỌC SINH GIỎI HÌNH HỌC 9
117
Tương tự ta có
IM EA
BIM BEA
IB EB
(7)
T (5), (6), (7) suy ra
IM IN
, hay I là trung điểm ca MN.
Bài 15.
1) Cho tam giác ABC ni tiếp đường tròn
O
, có góc A nhn và
AB AC
. Tia phân giác ca góc
BAC
cắt đường tròn
O
ti D (D khác A) và ct tiếp tuyến ti B của đường tròn
O
ti E. Gi F
là giao điểm ca BD và AC
a) Chng minh EF song song vi BC.
b) Gọi M là giao điểm ca AD và BC. Các tiếp tuyến ti B, D của đường tròn
O
ct nhau ti N.
Chng minh rng:
1 1 1
BN BE BM
2) Cho tam giác nhn ABC ni tiếp đường tròn
O
, đường cao AH. Gi M là giao điểm ca AO
và BC. Chng minh
2
HB MB AB
HC MC AC
. Dấu đẳng thc xy ra khi nào?
(Trích đề thi vào 10 Chuyên Hưng Yên, năm học 2015 - 2016)
NG DN GII
1) a) Do
EBF BAD EAF
nên BEAF ni tiếp được.
Suy ra
180BEF BAF
BAF CBE
(góc to bi tiếp tuyến và dây cung)
Nên
180BEF BCE
, mà đây là hai góc trong cùng phía nên
//BC EF
.
b)
NDB BAD CAD CBD
,
suy ra
//ND BC
.
Theo Thales:
1
NB NE
BE BE
1
ND
BM
1
NB ND
BE BM
ND NB
. Do đó
1
NB NB
BE BM
1 1 1
BN BE BM
2) K đường kính AP
CHUYÊN ĐỀ BỒI DƯỠNG HỌC SINH GIỎI HÌNH HỌC 9
118
D dàng chứng minh được:
AHB ACP
nên:
HB AH
PC AC
Tương tự:
HC AH
PB AB
Suy ra:
.
HB PB AB
HC PC AC
,
Suy ra:
.
HB PC AB
HC PB AC
Li có:
AMC BMP
nên:
MC MP
AC PB
Tương tự:
MB MP
AB PC
Suy ra:
.
MB AC PB
MC AB PC
nên:
.
MB PB AB
MC PC AC
Cng li, ta có:
. 2.
HB MB PB PC AB AB
HC MC PC PB AC AC
Bài 16. Cho tam giác ABC có ba góc nhn, ni tiếp đường tròn
O
. Các đường cao AM, BN, CP
ca tam giác ABC cùng đi qua điểm H. Gi Q là điểm bt kì trên cung nh BC (Q khác BC). Gi
E, F theo th t là điểm đối xng vi Q qua ABAC.
1) Chng minh
..MH MA MP MN
.
2) Chứng minh ba điểm E, H ,F thng hàng.
3) Gi J là giao điểm ca QEAB, I là giao điểm ca QFAC. Tìm v trí của điểm Q trên cung
nh BC để
AB AC
QJ QI
nh nht.
(Chuyên Toán TP Hà Nội, năm hc 2015 - 2016)
NG DN GII
CHUYÊN ĐỀ BỒI DƯỠNG HỌC SINH GIỎI HÌNH HỌC 9
119
1) D dàng thy rng các t giác CNHM, BMHP ni tiếp. Cho nên
NCH NMH
NMP HBP
,
kết hp vi
ACH ABH
(cùng ph vi
BAC
) ta suy ra
NMH HMP
(1)
Mt khác t giác ANMB ni tiếp nên
MNH MAB
(2)
T (1) và (2) ta suy ra
HMN PMA
dẫn đến
HM MN
MP MA
..MH MA MN MP
2) Trước hết d thy
ACQ ACF
(c.c.c)
nên
AFC AQC ABC CHM
dẫn đến t giác AFCH ni tiếp và
90ACH AFH BAC
(3)
Mt khác do tính chất đối xng ta có
AF AQ AE
.
hay tam giác AEF cân tại A để
11
90 90
22
AFE AEF EAF FAQ EAQ
90 90CAQ BAQ BAC
Do đó ta được
AFH AFE
hay ba điểm E, H, F thng hàng.
3) Trước hết thy rng
. 2 , . 2
ABQ AQC
ABQJ S ACQI S
Và đặt
AB AC
P
QJ QI
Khi đó áp dụng BĐT Cauchy-Shwarz ta có:
2 2 2 2
. . 2 2
ABQ ACQ
AB AC AB AC
P
AB QJ AC QI S S
22
22
ABQ ACQ ABC QBC
AB AC AB AC
S S S S
Đẳng thc xy ra khi và ch khi
QI QJ
.
Mt khác nếu gọi G là điểm chính gia ca cung nh BC thì luôn có
QBC GBC
SS
, do đó
2
2
ABC GBC
AB AC
P
SS
Vy
AB AC
P
QJ QI
nh nht khi và ch khi Q là điểm chính gia ca cung nh BC.
Bài 17. Cho tam giác ABC nhn, có trc tâm H và ni tiếp đường tròn tâm O. Gọi D, E, F tương
ứng là các chân đường cao ca tam giác ABC k t A, B, C. Gọi M là giao điểm ca tia AO và cnh
BC. Gọi N, P tương ứng là hình chiếu vuông góc ca M trên các cnh CA, AB.
CHUYÊN ĐỀ BỒI DƯỠNG HỌC SINH GIỎI HÌNH HỌC 9
120
1) Chng minh:
..HE MN HF MP
.
2) Chng minh t giác FENP ni tiếp.
3) Chng minh rng:
2
.
.
BD BM AB
CD CM AC
(Trích đề thi vào 10 Chuyên Toán, Vĩnh Phúc, năm học 2015 - 2016)
NG DN GII
1) Ta có
180FHE PMN A
,
FEH FAH MAN NPM
(do t giác HFAE, PMNA ni tiếp).
Do đó
PMN EHF
..HE MN HF MP
2) T phn 1) thì
90FEN FEH
90NPM BPN
Nên t giác FENP ni tiếp.
3) Ta có
BAD CAM BAM DAC
Suy ra
sin . . .
sin . . .
BAD
CAM
S BD BAD AB AD AB AD
S CM CAM AC AM AC AM
sin . . .
sin . . .
BMA
CAD
S BM BAM AB AM AB AM
S CD CAD AC AD AC AD
Do đó
2
.
.
BD BM AB
CD CM AC
Bài 18. Cho đường tròn
;OR
và dây BC c định không đi qua tâm. Trên tia đối ca tia BC ly
điểm A (A khác B). T A k hai tiếp tuyến AM và AN với đường tròn
O
(M và N là các tiếp
điểm). Gọi I là trung điểm ca BC.
1) Chng minh A, O, M, N, I cùng thuc một đường tròn và IA là tia phân giác ca góc
MIN
.
2) Gọi K là giao điểm ca MN và BC. Chng minh
2 1 1
AK AB AC
.
3) Đường thng qua M và vuông góc với đường thng ON ct
O
tại điểm th hai là P. Xác định
v trí của điểm A trên tia đối của tia BC để AMPN là hình binh hành.
(Chuyên Nguyn Trãi, Hải Dương, năm học 2015 - 2016)
NG DN GII
CHUYÊN ĐỀ BỒI DƯỠNG HỌC SINH GIỎI HÌNH HỌC 9
121
1) Theo gi thiết
90AMO ANO AIO
5 điểm A, O, M, N, I thuộc đường tròn đường kính AO
,AIN AMN AIM ANM
(Góc ni tiếp cùng chn mt cung)
AM AN AMN
cân ti
A AMN ANM
AIN AIM
(đpcm)
2)
2 1 1
2.AB AC AK AB AC
AK AB AC
..AB AC AK AI
(Do
2AB AC AI
)
ABN
đồng dng vi
2
.ANC AB AC AN
AHK
đồng dng vi
..AIO AK AI AH AO
Tam giác
AMO
vuông tại M có đường cao
2
.MH AH AO AM
2
.AK AI AM
. Do
..AN AM AB AC AK AI
3) Ta có
, , / /AN NO MP NO M AN AN MP
Do đó AMPN là hình bình hành
2AN MP x
Tam giác ANO đồng dng vi
2
2AN NO x
NEM NE
NE EM R
TH1.
2
2 2 2 2 2 2
2
2
x
NE NO OE R R x x R R R x
R
Đặt
2 2 2 2 2
,0R x t t x R t
PTTT
2 2 2 2 2
2
2 2 0
tR
R t R Rt t Rt R
tR
Do
22
00t t R R x R x A B
(Loi)
CHUYÊN ĐỀ BỒI DƯỠNG HỌC SINH GIỎI HÌNH HỌC 9
122
TH2.
2
2 2 2 2 2 2
2
2
x
NE NO OE R R x x R R R x
R
Đặt
2 2 2 2 2
,0R x t t x R t
PTTT
2 2 2 2 2
2
2 2 0
tR
R t R Rt t Rt R
tR
Do
22
3
0 2 2 2
2
R
t t R R x R x AO R
Vy A thuc BC, cách O một đoạn bng 2R thì AMPN là hình bình hành.
Bài 19. Cho đường tròn tâm O đường kính BC, A là điểm di chuyển trên đường tròn
O
(A khác
B và C). K AH vuông góc vi BC ti H. M là điểm đối xng của điểm A qua điểm B.
1) Chứng minh điểm M luôn nm trên một đường tròn c định.
2) Đường thng MH ct
O
ti E và F (E nm gia M và F). Gọi I là trung điểm của HC, đường
thng AI ct
O
ti G (G khác A). Chng minh:
2 2 2 2 2
2AF FG GE EA BC
.
3) Gi P là hình chiếu vuông góc ca H lên AB. Tìm v trí của điểm A sao cho bán kính đường tròn
ngoi tiếp tam giác BCP đạt giá tr ln nht.
(Trích đề thi vào 10 Chuyên Toán, Nguyn Trãi, Hải Dương, năm học 2016-2017)
NG DN GII
1) Lấy K là điểm đối xng ca O qua B, vì B và O c định nên K c định. T giác OAKM là hình
bình hành nên
KM OA
.
Do
2
BC
OA
không đổi.
M nằm trên đường tròn tâm K, bán kính
2
BC
.
2) Xét
AHB
CHA
CHUYÊN ĐỀ BỒI DƯỠNG HỌC SINH GIỎI HÌNH HỌC 9
123
90 ,BHC BHA BAH ACB
(cùng ph vi
ABC
)
AHB CHA
.
Gọi S là trung điểm của AH, I là trung điểm ca HC nên
ABS CAI ABS CAI
Ta lại có BS là đường trung bình ca
AMH
//BS MH ABS AMH AMH CAI
90 90CAI MAI AMH MAI AI MF
Xét t giác AEGF ni tiếp
O
, có
AG EF
K đường kính AD, do
GD AG
EF AG
nên
//EF GD
, do đó tứ giác ni tiếp EFGD là
hình thang cân
2 2 2 2 2 2
FG ED AE FG AE ED AD BC
Tương tự ta chứng minh được:
2 2 2
AF EG BC
Vy
2 2 2 2 2
2AE FG AF EG BC
.
3) Gi Q là hình chiếu ca H trên
AC
T giác APHQ là hình ch nht (S là tâm)
AQP AHP ABC
nên t giác BPQC ni tiếp.
Đưng trung trc của các đoạn thng PQ, BC, QC ct nhau tại O’ thì O’ là tâm đường tròn ngoi
tiếp tam giác BCP.
Có OO’ // AH vì cùng vuông góc với BC.
OA PQ
O S PQ
//O S OA
nên t giác ASO’O là hình hình hành
2
AH
OO AS
Trong trường hp A nm chính gia cung BC thì ta vn có:
2
AH
OO AS
Tam giác OO’C vuông tại O nên
2
2
4
AH
O C OC
Do OC không đổi nên O’C lớn nht khi AH ln nht
A chính gia cung BC.
Bài 20. Cho đường tròn tâm O, bán kính R. T một điểm M ngoài đường tròn k hai tiếp tuyến
MA và MB với đường tròn (A, B là các tiếp điểm). Qua A k đường thng song song vi MO ct
CHUYÊN ĐỀ BỒI DƯỠNG HỌC SINH GIỎI HÌNH HỌC 9
124
đường tròn tại E (E khác A), đường thng ME cắt đường tròn tại F (F khác E), đường thng AF ct
MO tại N, H là giao điểm ca MO và AB.
1) Chng minh: T giác MAOB ni tiếp đường tròn.
2) Chng minh:
2
.MN NF NA
MN NH
.
3) Chng minh:
2
2
1
HB EF
HF MF
.
(Trích đề thi vào 10, Chuyên Nguyn Trãi, Hải Dương (Vòng 1), năm học 2017 - 2018)
NG DN GII
1) Ta có
90 , 90MAO MBO
(theo tính cht ca tiếp tuyến và bán kính)
Suy ra:
180MAO MBO
Vy t giác MAOB ni tiếp đường tròn.
2) Ta có
//AE MO AEM EMN
,
AEM MAF
, suy ra
EMN MAF
NMF
NAM
MNA
chung;
EMN MAF
nên
NMF
đồng dng vi
NAM
2
.
NM NA
NM NF NA
NF NM
(1)
Mt khác có:
ABF AEF ABF EMN
hay
HBF FMH
MFHB là t giác ni tiếp
FHM FBM FAB
hay
FHN NAH
.
Xét
NHF
NAH
có:
ANH
chung;
NHF NAH NHF
đồng dng
NAH
2
.
NH NA
NH NF NA
NF NH
(2)
T (1) và (2) ta có
NH HM
.
3) Xét
MAF
MEA
có:
AME
chung,
MAF MEA
suy ra
MAF
đồng dng vi
MEA
2
2
ME MA AE ME AE
MA MF AF MF AF
(3)
Vì MFHB là t giác ni tiếp
90 90MFB MHB BFE
90AFH AHN AFE BFH
.
CHUYÊN ĐỀ BỒI DƯỠNG HỌC SINH GIỎI HÌNH HỌC 9
125
AEF
HBF
có:
;EFA BFH FEA FBA
suy ra
AEF
đồng dng vi
HBF
22
22
AE HB AE HB
A F HF AF HF
(4)
T (3) và (4) ta có:
22
22
ME HB MF FE HB
MF HF MF HF
22
22
11
FE HB HB FE
MF HF HF MF
Bài 21. Cho hai đường tròn
;OR
;OR
ct nhau tại hai điểm phân biệt A và B. Trên tia đối
ca tia AB lấy điểm C, k tiếp tuyến CD, CE vi
O
, trong đó D, E là các tiếp điểm và E nm
trong
O
. Đường thng AD, AE ct
O
lần lượt ti M và N (M, N khác A). Tia DE ct MN ti I,
OO
ct AB và DI lần lượt ti H và F.
1) Chng minh:
..FE HD FD HE
.
2) Chng minh:
. . . .MB EB DI IB AN BD
.
3) Chng minh:
OI
vuông góc vi MN.
(Trích đề thi vào 10, Chuyên Nguyn Trãi (vòng 2), Hải Dương 2017 - 2018)
NG DN GII
1)
O
ct
O
ti A, B
90OO AB CHO
(1)
CD, CE là tiếp tuyến ca
O
ti D, E
90CDO CEO
(2)
T (1) và (2)
, , , ,C D O H E
cùng thuộc đường tròn đường kính
CHE CDE
CO
CHD CED
CD CE CDE CED CHE CHD
HC
là đường phân giác ca
DHE
.
Mt khác
OO AB
ti H hay
FH HC
ti H
HF
là phân giác ngoài ti H ca
DHE
..
FE HE
FE HD FD HE
FD HD
CHUYÊN ĐỀ BỒI DƯỠNG HỌC SINH GIỎI HÌNH HỌC 9
126
2) Trong
O
có:
BMN BAN
Trong
O
có:
BAN BDE BMN BDE
BDMI là t giác ni tiếp
MBI MDI ABE
Xét
MIB
AEB
có:
;MBI ABE BMI BAE
MB IB
MIB AEB
AB EB
(3)
Xét
ABN
DBI
có:
;BAN BDI BNA BID
AB AN
ABN DBI
DB DI
(4)
T (3) và (4)
. . . . . .
MB AB IB AN
MB EB DI IB AN DB
AB DB EB DI
3) Xét
IBN
DBA
có:
IBN DBA
(vì
DEA IEN
),
BIN BDA
(vì BDMI ni tiếp)
IN DA
IBN DBA
IB DB
(5)
Xét
CDA
CBD
có:
DCB
chung;
CDA CBD
DA CD
CDA CBD
DB CB
DA CE
CD CE
DB CB
(6)
Xét
CEA
CBE
có:
BCE
chung;
CEA CBE
CE EA
CEA CBE
CB EB
(7)
Mt khác
MIB AEB
(theo phn b)
EA IM
EB IB
(8)
T (5), (6), (7), (8)
IN IM
IN IM O I MN
IB IB
Bài 22. Cho t giác ABCD có
60 , 90BAD BCD
. Đường phân giác trong ca BAD ct BD ti
E. Đường phân giác trong ca BCD ct BD ti F. Chng minh:
3 2 1 1 1 1
AE CF AB BC CD DA
(Trích đề thi vào 10 Chuyên Long An, năm học 2016 - 2017)
NG DN GII
Gi K là hình chiếu vuông góc ca E lên AB.
CHUYÊN ĐỀ BỒI DƯỠNG HỌC SINH GIỎI HÌNH HỌC 9
127
Din tích tam giác ABE là:
. .sin30 . .
2 2 4
KE AB AE AB AE AB
Din tích tam giác ADE là:
.
4
AE AD
Din tích tam giác ABD là:
.sin60 . 3 .
24
AB AD AB AD
Ta có: Din tích tam giác ABE + Din tích tam giác ADE
= Din tích tam giác ABD.
Suy ra:
3 1 1
AE AB AD
(1)
Tương tự như trên ta tìm được
3 1 1
CF CB CD
(2)
T (1) và (2) ta có:
3 2 1 1 1 1
AE CF AB BC CD DA
Bài 23. Cho đường tròn
;OR
và điểm A c định trên
;OR
. Gọi M, N là các giao điểm ca hai
đường tròn
;OR
;AR
; H là điểm thay đổi trên cung nh
MN
của đường tròn
;AR
. Đường
thng qua H và vuông góc vi AH ct
;OR
ti B, C. K
,HI AB I AB HK AC K AC
.
1) Chng minh rng IK luôn vuông góc vi một đường thng c định và
2
.2AB AC R
.
2) Tìm giá tr ln nht của điện tích
AIK
khi H thay đổi.
(Trích đề thi HSG Lp 9, Tnh Phú Thọ, năm học 2016 - 2017)
NG DN GII
1) Ta có
90 ; 90AIH AKH
180AIH AKH
nên t giác AJHK ni tiếp.
K tiếp tuyến At của đường tròn
;OR
ti A.
Ta có:
90
90
ACB HAC
AHK HAC
ACB AHK
(1)
Ta li có:
AHK AIK
CHUYÊN ĐỀ BỒI DƯỠNG HỌC SINH GIỎI HÌNH HỌC 9
128
(do t giác AIHK ni tiếp) (2)
BAt ACB
(cùng bng
1
2
AB
) (3)
T (1), (2), (3) suy ra: .
//BAt AIK At IK
Mt khác
OA At IK OA
. Vy IK luôn vuông góc vi đường thng c định OA.
Gọi J là giao điểm của AO và IK; A’ là điểm đối xng vi A qua O.
Ta có
90 ;ACH AA B AHC ABA ACH AA B
.
2
. 2 . 2
AC AH
AB AC R AH R
AA AB
2) Ta có
2
..
AK AH
AKH AHC AK AC AH
AH AC
= =
Gi
,SS
lần lượt là din tích các tam giác ABC và AIK.
Ta có
AI AK IK AJ
AIK ACB
AC AB BC AH
, suy ra:
22
1
.
.
2
.
1
.
.
2
AJ IK
S AJ IK AK AK AC
S AH BC AB AB AC
AH BC
42
2
2
1
44
.2
AH AH
R
AH R
Suy ra
2
11
. . . .2
4 8 8 8 4
R R R
S S AH BC BC R
Vy giá tr ln nht ca tam giác AJK bng
2
4
R
, đạt khi
HO
.
Bài 24. T điểm M nằm ngoài đường tròn tâm
O
.V hai tiếp tuyến MA, MB với đường tròn (A,
B là các tiếp điểm). Cát tuyến MPQ không đi qua O (P nằm gia M, Q). Gọi H là giao điểm ca
OM và AB.
1) Chng minh
HPO HQO
.
2) Tìm điểm E thuc cung ln AB sao cho tng
11
EA EB
có giá tr nh nht.
(Trích đề thi HSG Lp 9, Tnh Ngh an, năm học 2015 - 2016)
NG DN GII
1)
MPA
đồng dng
MAQ
(g.g),
suy ra
2
.MA MP MQ
(1)
CHUYÊN ĐỀ BỒI DƯỠNG HỌC SINH GIỎI HÌNH HỌC 9
129
MAO
vuông tại A, có đường cao
AH nên
2
.MA MH MO
(2)
T (1) và (2) suy ra
..P MQ MH MO
hay
MP MO
MH MQ
(*)
MPH
MOQ
có góc M chung, kết hp (*) ta suy ra
MPH
đồng dng
MOQ
(c.g.c).
Suy ra
MHP MQO
Do đó tứ giác PQOH là t giác ni tiếp
1
2
HPO HQO sdOH
(đpcm)
2) Trên tia đối ca tia EA lấy điểm F
sao cho
EB EF
hay
EBF
cân ti E, suy ra
1
2
BFA BEA
Đặt
AEB
khi đó
2
AFB
nên F di chuyn trên cung cha góc
2
dng trên BC.
Ta có:
1 1 4
EA EB EA EB
.
Như vy
11
EA EB
nh nht khi
EA EB
ln nht
hay
EA EF
ln nht
AF
ln nht (**)
Gọi O’ là điểm chính gia ca cung ln AB
suy ra
O AB
cân tại O’ suy ra
O A O B
(3)
O EB
O EF
có EB = EF, O’E chung
FEO BEO
(cùng bù vi
BAO
)
O EB O EF
(c.g.c) suy ra
O B O F
(4)
T (3) và (4) suy ra O’ là tâm cung chứa góc
2
dựng trên đoạn thẳng BC (cung đó và cung lớn AB
cùng thuc mt na mt phng b AB).
Do đó AF lớn nhất khi nó là đường kính ca
O
khi
EO
(***).
T (**) và (***) suy ra E là điểm chính gia cung ln AB thì
11
EA EB
giá tr nh nht.
CHUYÊN ĐỀ BỒI DƯỠNG HỌC SINH GIỎI HÌNH HỌC 9
130
Bài 25. Trong các hình bình hành ngoi tiếp đường tròn
;Or
, hãy tìm hình bình hành có din tích
nh nht.
(Trích đề thi HSG Lp 9, Tnh Hải Dương, năm học 2011 - 2012)
NG DN GII
Theo bài ta suy ra các cnh ca hình hành là
tiếp tuyến của đường tròn
;Or
. Gi M, N, P, Q
lần lượt là tiếp điểm của đường tròn vi các
cạnh như hình vẽ.
; , ;CM CN AP AQ BM BQ PD DN
CM BM AP PD CN DN AQ BQ
22BC AB BC AB
K
AH BC
. Ta có
AB AH
, dấu “=” xảy ra khi
90ABC
Ta có:
, , / /OM BC OP AD AD BC
P, O, M thẳng hàng, do đó
2AH PM r
.
. 2 . 2 .2
ABCD
S AH BC r AB r r
2
4
ABCD
Sr
, dấu “=” xảy ra khi
90ABC
.
Vy trong các hình bình hành ngoi tiếp đường tròn
;Or
thì hình vuông có din tích nh nht và
bng
2
4r
.
Bài 26. Cho tam giác ABC, điểm M trong tam giác, các đường thng AM, BM, CM, lần lượt ct
các cnh BC, CA, AB ti P, R, Q. Kí hiu
ABC
S
là din tích tam giác ABC.
1) Chng minh rng:
. . . 4
ABC
MA BC MB CA MC AB S
2) Xác định v trí của M để din tích tam giác PQR ln nht.
(Trích đề thi HSG Lp 9, Tỉnh Bình Định, năm học 2010 - 2011)
NG DN GII
a) Ta có:
BMC
ABC
S
PM MI
PA AH S
2
.
..
BMC BMC
ABC BMC
SS
PM MI MI BC
S S MA MA MA BC MA BC
.2
ABC BMC
MA BC S S
Tương tự ta cũng có:
CHUYÊN ĐỀ BỒI DƯỠNG HỌC SINH GIỎI HÌNH HỌC 9
131
.2
ABC AMC
MB AC S S
;
.2
ABC AMB
MC AB S S
Cng theo vế ta s được điều cn chng minh.
b) Đặt
;;
PMQ QMR RMP PQR
S x S y S z S x y z
Ta có
..
;
.
PMQ
RMP
MCP PMB MCP PMB BMC
S
S
MR MQ z x MR MQ y
S MC S MB S S MC MB S
2
.
44
BMP PMC BMC BMC
BMC BMC
S S S S
zx
y S S
(1)
Tương tự ta cũng có:
(2); (3)
44
CMA
AMB
S
S
xy yz
zx
Cng theo vế các BĐT (1), (2) và (3) ta được:
4
ABC
S
xy yz zx
z x y
(4)
Mặt khác dùng BĐT Cô-si ta s chứng minh được
xy yz zx
x y z
z x y
nên t (4) suy ra:
44
ABC ABC
PQR
SS
x y z S
Đẳng thc xy ra khi:
;;
PMB PMC CMQ QMA AMR RMB
S S S S S S
x y z
M là trng tâm ca tam giác ABC.
Vy khi M là trng tâm ca tam giác ABC thì max
4
ABC
PQR
S
S
Bài 27. Cho tam giác đều ABC ni tiếp đường tròn tâm O bán kính R. M là một điểm di động trên
cung nh BC của đường tròn đó.
1) Chng minh
MB MC MA
2) Gi H, I, K lần lượt là chân đường vuông góc h t M xung AB, BC, CA. Gi S, S' lần lượt là
din tích ca tam giác ABC, MBC. Chng minh rằng: Khi M di động ta luôn có đẳng thc:
2 3 2
3
SS
MH MI MK
R
(Trích đề thi HSG Lp 9, Tỉnh Bình Định, năm học 2016 - 2017)
NG DN GII
CHUYÊN ĐỀ BỒI DƯỠNG HỌC SINH GIỎI HÌNH HỌC 9
132
a) Cách 1: Trên tia đối ca tia MC lấy điểm E sao cho
ME MB
.
Ta có:
BEM
là tam giác đều
BE BM EM
BMA BEC MA EC
Do đó:
MB MC MA
Cách 2: Trên AM lấy điểm E sao cho
ME MB
Ta có:
BEM
là tam giác đều
BE BM EM
(c.g.c) MC AEMBC EBA
. Do đó:
MB MC MA
b) K AN vuông góc vi BC ti N
ABC
là tam giác đều nên O là
trng tâm ca tam giác
,,A O N
thng hàng
3
2
AN R
Ta có:
.sinAN AB ABN
33
:3
22
sin
AN
AB R R
ABN
Ta có:
1
.
2
ABM
MH AB S
22
3
ABM ABM
SS
MH
AB
R
22
1
.
2
3
ACM ACM
ACM
SS
MK AC S MK
AC
R
22
12
.
2
33
BCM BCM
BCM
SS
S
MI BC S MI
BC
RR
Do đó:
22
33
ABM ACM
S
MH MK MI S S
RR
22
.
33
ABMC
S
S
RR
CHUYÊN ĐỀ BỒI DƯỠNG HỌC SINH GIỎI HÌNH HỌC 9
133
2 3 2
22
.
3
33
SS
S
SS
R
RR
Bài 28. Cho tam giác ABC có bán kính đường tròn ni tiếp là r. Gi M, N, P lần lượt là trung điểm
ca các cnh BC, CA, AB . Biết rng:
1 1 1 1
AM BN CP r
Chứng minh tam giác ABC là tam giác đều.
(Trích đề thi HSG Lp 9, Tỉnh Thái Bình, năm học 2015 - 2016)
NG DN GII
Gọi đường cao tương ứng vi cnh BC là AH. Gi S là din tích tam giác ABC.
Ta d thy:
2
S S BC
AM AH
Tương tự ta có:
,
22
S AC S AB
BN CP
Cng vế theo vế ta được:
S S S
p
AM BN CP
, trong đó p là na chu vi.
1 1 1 1p
AM BN CP S r
Đẳng thc xy ra khi
ABC
đều.
Bài 29. Cho tam giác ABC và điểm M nm trong tam giác. Chng minh rng:
a.
( )
1
2
MB MC AB AC AB BC CA MA MB MC AB BC CA+ + + + + + + +
b.
BM MN NC AB AC+ + +
trong đó điểm N nm trong tam giác sao cho MN ct hai cnh AB,
AC
NG DN GII:
a. Đường thng BM ct AC P
Áp dụng BĐT (1) ta có:
MB MC MB MP PC
BP PC AB AP PC AB AC
+ + +
= + + + = +
b. Theo trên ta có:
;;BC MB MC AB AC CA MC MA AB BC + + + +
.AB MA MB AC BC + +
Cng theo tng vế các
BĐT trên ta có điều phi chng minh.
c. Áp dng câu 1) ta có:
BM MN NC BE EM MN NF FC+ + + + + +
CHUYÊN ĐỀ BỒI DƯỠNG HỌC SINH GIỎI HÌNH HỌC 9
134
.BE EF FC BE EA AF FC AB AC= + + + + + = +
Bài 30. Cho tam giác ABC và 3 trung tuyến AM, BN, CP. Chng minh rng:
a.
22
AB AC BC AB AC
AM
+ +

b.
( )
3
4
AB BC CA
AM BN CP AB BC CA
++
+ + + +
c. Gi s
.AB AC
Gi AD, AM theo th t là đường phân giác, đường trung tuyến ca tam giác
ABC. Chng minh rng:
22
AB AC BC AB AC
AD AM
+ +
NG DN GII:
+ Xét các tam giác MAB, MAC ta có:
,.AM AB BM AM AC MC
Suy ra
( )
22AM AB AC MC MC AM AB AC BC + + +
+ Gi D là điểm đối xng vi A qua M thì ABDC là hình
bình hành nên
2.AD AM=
Trong tam giác
ACD ta có:
2AD AC CD AM AB AC + +
Như vậy:
.
22
AB AC BC AB AC
AM
+ +

b, Áp dng bất đẳng thc câu a)
Cho 3 đường trung tuyến AM, BN, CP
ta có:
,
22
AB AC BC AB AC
AM
+ +

;.
2 2 2 2
BC AB AC AC BC BC AC AB AC BC
BN CP
+ + + +
Cng ba bất đẳng thc cùng
chiu ta có:
( )
3
4
AB BC CA
AM BN CP AB BC CA
++
+ + + +
c, Trong tam giác ABD, ADC có
; .AB AD BD AC AD DC + +
Cng theo tng vế hai BĐT trên
được:
2 .
2
AB AC BC
AB AC AD BC AD
+−
+ +
Kết qu này vẫn đúng với D là điểm bt k
nằm bên trong đoạn BC.
Dng
.AH BC
Vi
AB AC=
thì
.AM AD=
Vi
AB AC
thì
BH CH BM BH M
thuộc đoạn BH. Hơn nữa
ADB ADC ADB
tù. Do đó D thuộc đoạn BH. Lấy điểm P trên AB
sao cho
AP AC ADP ADC= =
(c.g.c)
,DP DC=
.APD ACD=
+ Nếu
90ACB 
(hình) thì
90 90APD ACB BPD ACB PBD=
BD PD CD=
CHUYÊN ĐỀ BỒI DƯỠNG HỌC SINH GIỎI HÌNH HỌC 9
135
BM BD MH DH AM AD
+ Nếu
90ACB 
(hình) thì
BPD ACH ADC ABC=
.BD PD CD BM BD MH DH AM AD =
Bài 31. Cho tam giác nhn ABC có trực tâm là điểm H. Chng minh rng:
( )
2
3
HA HB HC AB BC CA+ + + +
NG DN GII:
Dựng đường thng qua H song song vi AB ct AC ti D.
Dựng đường thng qua H song song AC ct AB ti E.
T giác AEHD là hình bình hành nên
,AD HE AE HD==
Xét tam giác AHD ta có:
HA HD AD HA AE AD + +
(1). Vì
//HE AC
.AC BH HE BH
Trong tam
giác vuông HBE ta có:
(2)
Tương tự ta có:
HC DC
(3). Cng các bất đẳng thc cùng
chiu (1), (2), (3) ta suy ra
( ) ( )
.HA HB HC AE EB AD DC AB AC+ + + + + = +
Tương tự ta cũng
có:
( )
2
,
3
HA HB HC AC BC HA HB HC AB BC HA HB HC AB BC CA+ + + + + + + + + +
Bài 32. Cho tam giác đều ABC ni tiếp đường tròn (O), H là trung điểm ca BC. M là điểm bt k
thuộc đoạn thng
( )
.BH M B
Lấy điểm N thuộc đoạn thng CA sao cho
.CN BM=
Gi I
trung điểm ca MN.
a, Chứng minh 4 điểm O, M, H, I cùng thuc một đường tròn.
b, Gi P là giao điểm ca OIAB. Chng minh tam giác MNP là tam giác đều.
c, Xác định v trí của điểm M để tam giác IAB có chu vi nh nht.
NG DN GII:
a, Xét tam giác BOM và tam giác CON ta có:
BM CN=
gi thiết,
,OB OC R==
30OBM OCN= =
(do tam giác ABC đều). Suy ra
BOM CON =
(c.g.c)
Suy ra
OM ON=
hay tam giác OMN cân ti O, do I là trung điểm
ca MN suy ra
OI MN⊥
90OIM OHM= =
nên t giác OMHI
ni tiếp (Có hai đỉnh liên tiếp I, H cùng nhìn OM góc bng
90 )
b, Do điểm P nm trên trung trc cnh MN nên
CHUYÊN ĐỀ BỒI DƯỠNG HỌC SINH GIỎI HÌNH HỌC 9
136
PM PN=
(1). Ta có
180 OMB OMC OMB ONC = + = +
suy ra t giác OMNC ni tiếp (tng hai
góc đối bng
180 )
nên
180 120 120MON NCM POM PON= = = =
suy ra
180POM PBM+ =
t giác PBMO ni tiếp nên
30 .OPM OBM= =
Chứng minh tương tự ta
cũng có:
30OPN OAN= =
60MPN =
(2). T (1) và (2) suy ra tam giác PMN là tam giác
đều.
c, T chng minh câu a, b suy ra
30 .OMN OHI OCN= = =
Suy ra
/ / ,HI AB
gi K là trung
điểm ca AC thì H, I, K thng hàng.
Tam giác IABAB không đổi nên chu vi tam giác nh nht khi
IA IB+
nh nhất. Đường thng HI
c định. Gi D là điểm đối xng vi B qua HI thì điểm D c định, suy ra độ dài AD không đổi. Ta
.IB ID IA IB IA ID AD= + = +
Du bng xy ra khi và ch khi A, D, I thng hàng. Tức điểm
I chính là giao điểm ca ADHK. Mt khác ta d chng
minh được AHDK là hình bình hành. Nên du bng xy ra
khi I là trung điểm ca HK, khi đó điểm
.MH
2. Trong một đường tròn, đường kính là dây cung ln nht.
3. Cho đường tròn
( )
;OR
và dây cung BC c định. Điểm A
di chuyn trên cung BC khi đó diện tích tam giác ABC ln
nht khi và ch khi A là điểm chính gia cung BC.
Chng minh:
Trường hợp 1: Điểm A thuc cung ln BC
Ta dựng đường cao AD ca tam giác ABC thì
1
..
2
ABC
S AD BC=
Do BC không đổi nên
ABC
S
ln nht khi và ch
khi AD ln nht. Gi M là trung điểm ca BC thì
2
2
4
BC
AD AM AO OM R R + = +
Suy ra
2
2
1
..
24
ABC
BC
S BC R R

+



Dấu đẳng thc xy ra khi và
ch khi
DM
O, M, A thng hàng. Hay
0
AA
là điểm chính gia cung ln BC.
Trường hợp 2: Điểm A thuc cung nh BC.
Gi I là giao điểm AO vi dây cung BC.
Ta có:
AD OM AI IO R AD R OM+ + =
CHUYÊN ĐỀ BỒI DƯỠNG HỌC SINH GIỎI HÌNH HỌC 9
137
Vy
2
2
1
..
24
ABC
BC
S BC R R




Dấu đẳng thc xy ra khi và ch khi
.D I M
Hay
1
AA
là điểm chính gia cung nh BC.
4. Cho đường tròn
( )
;OR
và dây cung BC c định. Điểm A
di chuyn trên cung BC khi đó chu vi tam giác ABC ln
nht khi và ch khi A là điểm chính gia cung BC.
Trường hợp 1: Điểm A thuc cung ln BC.
Ta có chu vi tam giác ABC bng:
.AB AC BC++
Do BC không đổi nên chu vi tam giác ln nht khi và ch
khi
AB AC+
ln nhất. Để to ra
AB AC+
ta làm như sau:
Trên tia đối ca tia AB lấy điểm N sao cho
AN AC=
khi đó
ta có: Tam giác NAC cân ti
A
1
2
ANC BAC=
không đổi. Suy ra đim N thuc cung
cha góc
1
2
BAC
dựng trên đoạn BC (phn na mt phng
b BC có chứa điểm A). Ta có:
.AB AC AB AN BN+ = + =
Nên
AB AC+
ln nht khi và ch khi
BN ln nht, tc là BN là đường kính của đường tròn cha cung cha góc
1
2
BAC
dựng trên đoạn
BC (phn na mt phng b BC có chứa điểm A) hay
1
90 .BCN N N=
Do
1 1 1
,AN AC=
tam
giác
1
BCN
vuông ti C suy ra
1 1 1 1
AN AC AB==
hay
1
AA
là điểm chính gia cung ln BC.
Trường hp A thuc cung nh BC ta cũng làm tương tự.
Bài 33. Cho nửa đường tròn tâm O đường kính
2,AB R=
điểm M nm trên nửa đường tròn sao cho
, AM R N=
là điểm nm trên cung MB (N khác M, B). Gi I là giao điểm ca AN, MB, H là hình
chiếu vuông góc ca I trên AB.
a, Chng minh: T giác HINB ni tiếp.
b, Chng minh: IH là phân giác ca góc
MHN
c, Khi N di chuyn trên cung MB. Chng minh rằng: Đường tròn ngoi tiếp tam giác MHN luôn đi
qua 2 điểm c định.
d, Tìm v trí điểm N trên cung MB để chu vi t giác AMNB ln nht.
NG DN GII:
a, Điểm N nm trên
( )
O
nên
90 .ANB =
CHUYÊN ĐỀ BỒI DƯỠNG HỌC SINH GIỎI HÌNH HỌC 9
138
Xét t giác IHBN
180IHB INB+ =
nên t giác HINB
ni tiếp (tổng 2 góc đối bng
180 .)
Suy ra
IHN IBN=
(1) (cùng chn cung IN).
b, Chứng minh tương tự câu a) ta cũng có:
AMIH ni tiếp suy ra
IHM IAM=
(cùng chn cung IM)
(2). Mt khác t giác AMNB ni tiếp nên:
MAN MBN=
(3). T (1), (2), (3) suy ra
IHM IHN=
hay
IH là tia phân giác ca góc MHN.
c, Vì
AM R AM OM OA= = =
nên tam giác AMO đều
suy ra điểm M là đim c định. T chng minh câu b ta
suy ra
2,MHN MAN=
mặt khác ta cũng có:
2,MON MAN=
(tính cht góc tâm). Suy ra
,MHN MON=
hay t giác MOHN ni tiếp (2 đỉnh liên tiếp O, H cùng nhìn cnh MN nhng góc
bng nhau). T đó suy ra đường tròn ngoi tiếp tam giác MHN luôn đi qua 2 điểm c định O, M.
d) Vì
2 2 2 2
60 120 , 4 3 .MAB MNB MB AB AM R R R= = = = =
Do MB không đổi,
120 ,MNB =
trên tia đối ca NB lấy điểm K sao cho
NK NM=
thế thì tam giác MNK cân ti N,
180 60MNK MNB= =
nên tam giác MNK đều suy ra
60 .MKB =
Suy ra điểm K nm trên
cung cha góc
60
dựng trên đoạn MB (phn na mt phng b MB không cha A). Tia AM ct
cung cha góc ti
0
K
thì
0
90 .MK B =
Ta tính được:
0
3
2.
sin60
3
2
MB R
K B R= = =
Chu vi t giác
AMNB bng
3 .AM MN NB BA R NM NB+ + + = + +
Suy ra chu vi t giác AMNB ln nht khi và
ch khi
NM NB+
ln nht. Theo cách dng trên ta có:
0
2NM MB KB K B R+ = =
chu vi t
giác
3 5 ,R NM NB R+ +
dấu đẳng thc xy ra khi và ch khi
0
.KK
Khi đó điểm N chính là
giao điểm ca
0
KB
vi
( )
O
hay
0
NN
là điểm chính gia cung nh MB.
Bài 34. Cho đường tròn
( )
;OR
và dây cung
( )
2BC BC R
c định. Điểm A di động trên cung ln
BC sao cho tam giác ABC nhọn. Các đường cao AD, BE, CF ct nhau tại điểm H. Gi N là trung
điểm ca EF.
a, Chng minh:
..R AN AM OM=
vi M là trung điểm ca BC.
b, Chng minh:
( )
2.
ABC
R EF FD DE S+ + =
Tìm v trí điểm A để
EF FD DE++
ln nht.
NG DN GII:
CHUYÊN ĐỀ BỒI DƯỠNG HỌC SINH GIỎI HÌNH HỌC 9
139
Gi
'R
bán kính đường tròn ngoi tiếp tam giác AEF thì
'R
cũng là bán kính đường tròn ngoi tiếp t giác AEHF
Suy ra
'
2
AH
R OM==
(hc sinh t chng minh tính cht
quen thuc này).
Ta có
AEF
ABC
nên
'AN R
AM R
=
hay
. . 'R AN AM R=
chú ý rng
' . . .R OM R AN AM OM= =
Gi P, Q lần lượt là trung điểm ca AC, AB. Ta có:
( )
2 2 . . . .
ABC OBC OCA OAB
S S S S OM BC OP AC OQ AB= + + = + +
T câu a ta có:
. . . . ,
AN EF
R AN AM OM OM R R
AM BC
= = =
tương tự ta cũng có:
. , . .
DF DE
OP R OQ R
AC AB
==
Thay vào ta có:
( )
2.
ABC
S R EF FD DE= + +
Do R không đổi nên
EF FD DE++
ln nht khi và
ch khi
ABC
S
ln nht. Hay A là điểm chính gia cung ln BC.
Có th lp lun theo cách khác:
Chng minh:
OA EF
suy ra
1
.,
2
AEOF
S OA EF=
tương tự ta
có:
11
. , .
22
BDOF CDOE
S OB DF S OC DE==
t đó suy ra
( )
2.
ABC
S R EF FD DE= + +
Do R không đổi nên
EF FD DE++
ln nht khi và ch khi
ABC
S
ln nht
khong cách t điểm A đến BC ln nht. Hay A là điểm chính
gia cung ln BC.
Ngoài ra ta cũng có thể gii theo cách khác:
Ta thấy các điểm B, F, E, C nằm trên đường tròn tâm M
đường kính BC.
Do BC không đổi nên
BAC
không đổi.
Ta có ME, MF là các tiếp tuyến của đường tròn ngoi tiếp t giác AEHF nên
MFE BAC=
không
đổi. Suy ra
90
2
BAC
EMF =
không đổi
suy ra độ dài EF không đổi.
Gi K là điểm đối xng vi E qua BC thì
.EDC KDC=
Ta li có
.EDC EHC FHB FDB= = =
Suy
ra
EDB KDM=
nên F, D, K thng hàng.
CHUYÊN ĐỀ BỒI DƯỠNG HỌC SINH GIỎI HÌNH HỌC 9
140
Ta có chu vi
DEF
ln nht khi và ch khi
DE DF+
ln nht. T các chng minh trên ta có:
2 ' .DE DF FK R BC+ = =
Suy ra
DE DF+
ln nht khi và ch khi
DI
hay A là điểm chính
gia cung ln BC.
Cũng có thể tiếp cận bài toán theo hướng khác:
Gi P, Q lần lượt là hình chiếu vuông góc ca B, C trên EF. Ta có các
tính cht quen thuc FH, DH lần lượt là phân giác trong các góc
, EFD FDE
suy ra FB, FD lần lượt là các đường phân giác ngoài ca
, EFD FDE
. H BX, BY lần lượt vuông góc vi DE, DF thì suy ra
, , FP FY DX DY EP EX= = =
(tính cht một điểm nm trên phân
giác cách đều 2 cnh).
Ta có: Chu vi tam giác DEF
2 2 .p DF EF DE YF DX EF DE FP EF EX EP EX EP= + + = + + + = + + = + =
Hoàn toàn tương t ta cũng có:
2 2 .p FQ=
Suy ra
( ) ( )
4 2 2 2 2 2p EP FQ PQ EF p PQ EF= + = + = +
suy ra
.DE DF PQ+=
Mt khác
PQ BC
nên chu vi tam giác DEF ln nht bng
EF BC+
khi và ch khi
//PQ BC
hay
A là điểm chính gia cung ln BC.
Bài 35. Cho nửa đường tròn tâm O đường kính
2.AB R=
EF là dây cung di động trên nửa đường
tròn sao cho E thuc cung AF
.
2
AB
EF =
Gi H là giao điểm ca
, , AF BE C
là giao điểm ca
AE, BF, I là giao điểm ca CH, AB.
a, Chứng minh 4 điểm A, C, F, I ng nm trên một đường tròn.
b, Chng minh:
..AE AC BF BC+
có giá tr không đổi khi EF di chuyn trên nửa đường tròn
( )
.O
Đưng thng AF ct tiếp tuyến ti B N, các tiếp tuyến ti A, F ca
( )
O
ct nhau M. Chng
minh:
.ON MB
c, Xác định v trí EF trên nửa đường tròn để t giác ABEF có din tích ln nht.
NG DN GII:
a, Vì các điểm E, F nm trên nửa đường tròn đường kính AB
tiếp nên
90AEB AFB= =
(Góc ni chn nửa đường tròn).
Do C là giao điểm ca AE, BF suy ra
,BE AC AF BC⊥⊥
suy
ra BE, AF ct nhau tại điểm H là trc tâm tam giác CAB suy ra
.CI AB
CHUYÊN ĐỀ BỒI DƯỠNG HỌC SINH GIỎI HÌNH HỌC 9
141
T giác ACFI
90AFC AIC= =
suy ra t giác ACFI là t giác ni tiếp (Hai đỉnh liên tiếp F, I
cùng nhìn AC góc
90 .)
b, Xét tam giác vuông ACI và tam giác vuông ABE ta có
90 , AIC AEB CAB= =
chung. Suy ra
ACI
ABE
do đó:
..
AC AB
AC AE AI AB
AI AE
= =
Tương tự ta cũng có:
. . .BC BF BI AB=
Cộng hai đẳng thc ta có:
( )
22
. . 4 .AE AC BF BC AB AI BI AB R+ = + = =
c, Xét tam giác MAO và tam giác ABN. Ta có:
90 ,OAM NBA OMA BAN= = =
(cùng ph vi
).NAM
T đó suy ra
MAO
ABN
(g.g) suy ra
2
1
2
MA AB MA OB
AO BN BN
AB
= =
hay
MA OB
AB BN
=
MAB
OBN
(c.g.c), suy ra
90NOB MBA BMA MBA+ = + =
hay
.ON MB
c, D thy: Tam giác OMN là tam giác đều có cnh
.MN R=
Gi K là trung điểm ca EF thì
.OK EF
T đó ta tính được:
22
2 2 2 2
33
.
4 4 2
R R R
OK OE KE R OK= = = =
Tam giác OMN
có din tích là:
2
1
13
..
24
R
S OK EF==
Dng
, EX FY
lần lượt vuông góc vi AB ti E, F thì t
giác EFYX là hình thang vuông, dng
KP AB
suy ra P là trung điểm XY nên KP là đường trung
bình hình thang EFYX. Kí hiu
23
, SS
lần lượt là din tích ca các tam giác AOE, BOF thì
12
1 1 1 1
. . ; . . .
2 2 2 2
S OA EX R EX S OB FY R FY= = = =
Ta có:
1 2 3AEFB
S S S S= + +
1
S
không đổi nên
AEFB
S
ln nht khi và ch khi
23
SS+
ln nht. Ta có:
( )
23
11
.2 . .
22
S S R EX FY R KP R KP+ = + = =
Trong tam giác vuông OKP ta có:
3
.
2
KP KO R=
Suy ra
2
23
3
.
2
S S R+
Du bng xy ra khi
và ch khi
/ / .P O PK EF EF AB
Vy GTLN ca
AEFB
S
2
33
4
R
khi và ch khi
/ / .EF AB
Bài 36. Cho đường tròn
( )
;OR
có đường kính AB, C là trung điểm ca OA và dây
MN OA
ti C.
Gi K là điểm tùy ý trên cung nh BM, H là giao điểm ca AK, MN.
a, Chng minh: BCHK là t giác ni tiếp.
b, Tính
.AH AK
theo R.
Xác định v trí K để
KM KN KB++
ln nhất. Tính GTLN đó.
CHUYÊN ĐỀ BỒI DƯỠNG HỌC SINH GIỎI HÌNH HỌC 9
142
NG DN GII:
a, Do K nm trên
;
2
AB
O



90AKB =
Li có
90HCB =
(gi thiết) suy ra
180AKB HCB+ =
nên
t giác BCHK ni tiếp. (Tổng hai góc đối bng
180 .)
b, Ta có
ACH
AKB
(g.g)
nên
2
. . .2 .
2
AH AB R
AH AK AC AB R R
AC AK
= = = =
c, Đây là một câu hi khá hay. Nếu biết định lý Ptolemy hoc
định lý Shooten thì bài toán được gii quyết.
Cách tiếp cn th nht:
Nhn thy tam giác BMN cân ti B và tam giác AMO đều (do
AMO cân ti O và ti M) suy ra tam giác BMN đều nên
1
sđ 60 .
2
NKB NB= =
Trên dây KN ly
điểm P sao cho
thì tam giác KPB đều. Xét tam giác MKBNPB ta có:
, , 120KB BP MB NB MKB NPB= = = =
suy ra
MKB NPB =
(c.g.c)
.KM PN KM KB KN = + =
Vy
2KM KN KB KN+ + =
D thy
2KN R
nên
4,KM KN KB R+ +
du bng xy ra khi và ch khi K, O, N thng hàng.
T đó suy ra điểm K là giao điểm ca NO vi
( )
O
(K khác N).
Cách 2. Dùng định lý Ptolemy.
5. Dùng các định lý hình hc, kết hp vi bất đẳng thc c
điển AM-GM, Cauchy-Schwarz.
Để gii quyết tt các bài toán dạng này người gii cn nm
chc các kết qu.
a, Định lý Ptolemy cho t giác ni tiếp: Cho t giác ABCD
ni tiếp trong đường tròn
( )
.O
Khi đó ta có:
. . . .ABCD AD BC AC BD+=
Chng minh:
Trên đường chéo BD lấy điểm E sao cho
.DAE BAC=
Ta
DAE BAC=
ADE ACB=
(cùng chn
)AB
nên
ADE
ACB
(g.g)
AD DE
AC BC
=
..AD BC AC DE=
(1). Do
DAE CAB=
nên
,DAC EAB=
li có
ABE ACD=
(cùng chn
)AD
CHUYÊN ĐỀ BỒI DƯỠNG HỌC SINH GIỎI HÌNH HỌC 9
143
ABE
ACD
(g.g)
..
AB BE
AB CD AC BE
AC CD
= =
(2).
T (1) và (2) suy ra
( )
. . . .ABCD AD BC AC BE DE AC BD+ = + =
Bài 37. Cho tam giác ABC đều ni tiếp đường tròn
( )
O
bán kính R, điểm M chuyển động trên cung
nh BC.
a, Chng minh:
.MA MB MC=+
b, Tìm GTLN, GTNN ca
.P MA MB MC=++
(Trích đề tuyn sinh vào lớp 10 Trường chuyên Đại hc Vinh 2006).
Tìm giá tr nh nht ca biu thc:
11
Q
MB MC
=+
NG DN GII:
a, Áp dụng định lý Ptolemy cho t giác ABMC ta có:
. . . .MABC AB MC AC MB=+
Do
.AB BC CA==
Suy ra
.MA MB MC=+
b, Ta có
2.P MA MB MC MA= + + =
P nh nht khi và ch khi
MB
hoc
.MC
P ln nht khi và ch khi MA là đường kính ca
( )
O
b, Vi mi s thực dương x, y ta có:
( )
1 1 1
2 .2 4.x y xy
x y xy

+ + =


Nên ta suy ra
1 1 4
.
x y x y
+
+
Áp dng vào bài toán ta có:
1 1 4 4
.
MB MC MB MC MA
+ =
+
Do
2MA R
nên ta suy ra
1 1 4
2.
2
R
MA MB R
+ =
Du bng xy ra
khi và ch khi AM là đường kính ca
( )
.O
Hay M là điểm chính gia cung nh BC.
Bài 38. Cho đường tròn tâm O và điểm A nằm ngoài đường tròn. K hai tiếp tuyến AB, AC vi (O)
vi B, C là các tiếp điểm. Trên cung nh BC ly một điểm M ri k các đường vuông góc MI, MH,
MK xung BC, CA, AB. Gi P, Q lần lượt là giao điểm ca các cặp đường thng BMIK, CM
IH. Chng minh:
2
..MI MH MK=
a, Tìm v trí điểm M để
..MI MH MK
đạt GTLN.
Tìm v trí điểm M để
11
MH MK
+
nh nht.
NG DN GII:
CHUYÊN ĐỀ BỒI DƯỠNG HỌC SINH GIỎI HÌNH HỌC 9
144
a, T gi thiết ta có:
90BKM BIM= =
suy ra t giác BKMI ni
tiếp. Tương tự cho t giác CIMH, AKMH.
Vì t giác BKMI ni tiếp nên:
MKI MBI=
(cùng chn cung MI).
Mt khác ta có:
MBI MCH=
(tính cht góc to bi tiếp tuyến và
dây cung). Nhưng
MCH MIH=
(cùng chn cung MH ca t giác
ni tiếp MHCI). Suy ra
MKI MIH=
Hoàn toàn tương tự ta có:
MIK MHI=
nên
MIK
MHI
(g.g)
2
..
MI MH
MI MH MK
MK MI
= =
T đó ta có
3
. . .MI MI MH MK=
Suy ra
..MI MH MK
ln nht khi và ch khi MI
ln nht. Hay M là điểm chính gia cung nh BC.
b, Áp dng bất đẳng thc AM-GM ta có:
1 1 2 2
.
.
MH MK MI
MH MK
+ =
Nên
11
MH MK
+
nh
nht khi và ch khi MI ln nht. Hay M là điểm chính gia cung nh BC.
Bài 39. Cho đường tròn tâm
( )
;OR
và một điểm M nm ngoài (O). Qua M k các tiếp tuyến MA,
MB đến (O), (A, B la các tiếp đim).
a, Đường thng qua O song song vi AB ct tia MA, MB lần lượt ti C, D. Tìm giá tr lón nht ca
din tích tam giác MCD.
b, Gi s
2OM R=
va điểm M c định. Trên đường thng (d) qua M vuông góc vi MO ta ly mt
điểm P ri k 2 tiếp tuyến PE, PF đến (O), (A, B là các tiếp điểm). Chứng minh đường thng EF
luôn đi qua một điểm c định và tìm v trí điểm P trên đường thng (d) để độ dài EF nh nht.
NG DN GII:
a, Ta có:
( )
2 . .
MCD MOC
S S OAMC OA MA AC= = = +
Theo
bất đẳng thc Cosi ta có:
..MA AC MA AC+
Mt khác áp dng h thức lượng
trong tam giác vuông MOC ta có:
22
..AM AC OA R==
Suy
ra
2
2.
MCD
SR
Du bng xy ra khi và ch khi
AM AC MOC=
vuông
cân ti O. Hay
2.MO R=
b, Gi I là giao điểm ca EFMO, H là giao điểm ca EF
NO thì MIHN là t giác ni tiếp nên
. . .OH ON OI OM=
Mt khác trong tam giác vuông NEO ta
có:
22
.OH ON OE R==
suy ra
2
.2OI R R=
.
2
R
OI=
Đim I nm trên
,
2
R
OM OI =
không đổi.
CHUYÊN ĐỀ BỒI DƯỠNG HỌC SINH GIỎI HÌNH HỌC 9
145
Suy ra I là điểm c định, nên đường thng EF luôn đi qua I c định nằm trong đường tròn
( )
.O
Ta
thy:
22
22EF HE R OH= =
nên EF nh nht khi OH ln nht. Mt khác
2
R
OH OI=
nên
EF ln nht bng
2
2
23
4
R
EF R R= =
khi và ch khi
.H I N M
Cũng có thể lý lun theo cách khác: Ta có
2 2 2
2
2 . 2
2 2 1
OE EN R ON R R
EF HF
ON ON ON
= = = =
suy ra EF nh nht khi và ch khi
2
2
R
ON
ln nht
ON
nh nht. Mà
2.ON OM R=
Suy ra
3.EF R
Du bng xy ra khi và ch khi
.MN
Bài 40. Cho đường tròn tâm O đường kính
2.AB R=
Các đường thng a, b lần lượt là tiếp tuyến
ca
( )
O
ti A, B. Gi I là trung điểm ca OA. Lấy hai điểm P, Q nm trên a, b sao cho P, Q cùng
mt phía so vi AB
90 .PIQ =
Gi H là hình chiếu vuông góc ca I lên PQ chng minh H nm
trên
( )
O
và tìm v trí P, Q trên a, b sao cho din tích tam giác IPQ ln nht.
NG DN GII:
Hc sinh t chng minh
90AHB =
để suy ra H nm trên
( )
.O
Chú ý rng:
, OH R OH=
không vuông góc vi PQ
nên PQ ct
( )
.O
Ta có
( )
3
.
2
IPQ ABQP AIP BIQ
R AP BQ
S S S S
+
= =
Mt khác
ta cũng dễ chứng minh được:
IAP
QBI
2
3
. . .
4
IA QB
APQB IA IB R
AP BI
= = =
Áp dng bất đẳng thc AM-GM ta có:
2
3 2 3 . 3AP BQ AP BQ R+ =
suy ra
2
3
.
2
IPQ
SR
Dấu đẳng thc xy ra khi và ch khi
3
,AQ
22
RR
AP ==
Bài 41. Cho tam giác ABC vuông ti A
AB AC
ngoi tiếp đường tròn tâm O. Gi D, E, F ln
t là tiếp điểm ca
( )
O
vi các cnh AB, AC, BC; M là điểm di chuyển trên đoạn CE. Gi N
giao điểm ca BM vi cung nh EF ca (O), PQ lần lượt là hình chiếu ca N trên các đường
thng DE, DF. Xác định v trí của điểm M để PQ ln nht.
CHUYÊN ĐỀ BỒI DƯỠNG HỌC SINH GIỎI HÌNH HỌC 9
146
NG DN GII:
Ta có t giác PNQD, EDFN ni tiếp
.QPN QDN FEN = =
Tương tự có ta có:
. NEFNQP NDP NFE= =
NPQ
Suy ra
.
PQ NQ
EF NF
=
Trong tam giác vuông NQF ta có:
NQ NF
do đó
1.
PQ
EF
Như vy PQ ln nht bng EF khi và ch khi
QF
khi đó
,PE
do P
Q lần lượt là hình chiếu ca N trên các đường thng DE, DF nên khi
,QF
PE
thì DN
đường kính ca
( )
.O
T đó suy ra cách xác định M như sau: Dựng đường kính DN ca (O), M
giao điểm ca BNAC
Bài 42. Cho tam giác ABC cân đỉnh A. Gi O là trung điểm ca BC. Đường tròn (O) tiếp xúc vi
AB E tiếp xúc vi AC F. Điểm H chy trên cung nh
EF
tiếp tuyến của đường tròn ti H ct
AB, AC lần lượt ti M, N. Xác định v trí của điểm H để din tích tam giác AMN đạt giá tr ln nht.
NG DN GII:
D thy OM, ON lần lượt là phân giác
,EOM FOH
T đó ta có:
0
180
2
BAC
MON ABC MBO
= =
OCN
(g.g)
2
..
4
MB BO BC
BM CN OBOC const
OC CN
= = = =
(1)
Ta li có
AMN ABC BMNC
S S S=−
nên
AMN
S
đạt giá tr ln nht khi và
ch khi
BMNC
S
đạt giá tr nh nht.
Gi R là bán kính của đường tròn
( )
,O
ta có:
( )
( )
1
2
1
2
2
BMNC BOM MON NOC
S S S S R BM MN NC
R BE CF EM FN
= + + = + +
= + + +


( ) ( )
MN EM FN R BE EM FN= + = + +
( )
BE CF=
( ) ( )
2R BE BM CN BE R BM CN BE= + + = +
(2).
Áp dng bất đẳng thc AM-GM, t (1) và (2) suy ra
( )
( )
..
BMNC
S R BM CN BE R BC BE =
CHUYÊN ĐỀ BỒI DƯỠNG HỌC SINH GIỎI HÌNH HỌC 9
147
Dấu “=” xảy ra khi và ch khi
//BM CN MN BC=
khi và ch khi H là giao điểm của đường
trung trc ca BC với đường tròn (O). Vy din tích tam giác AMN đạt giá tr ln nht khi H là giao
của đường trung trc ca BC với đường tròn
( )
O
Bài 43. Cho đường tròn
( )
;OR
và điểm A nm ngoài (O), k các tiếp tuyến AB, AC đến (O) (B, C
là các tiếp điểm).
a, Chng minh: ABOC là t giác ni tiếp.
b, Gi E là giao điểm ca BCOA. Chng minh:
BE OA
2
..OE OA R=
c, Trên cung nh BC ca (O) ly điểm K bt k (K khác B, C). Tiếp tuyến ti K ca
( )
O
ct AB, AC
lần lượt ti P, Q. Chng minh: Chu vi tam giác APQ không đổi khi K chuyn động trên cung nh
BC.
d, Đường thng qua O vuông góc vi OA cắt các đường thng AB, AC lần lượt ti M, N. Chng
minh:
.PM QN MN+
NG DN GII:
a, Vì AB, AC là các tiếp tuyến ca (O) nên
90 .ABO ACO= =
T giác ABOC
180ABO ACO+ =
nên ABOC là t giác ni tiếp
(tổng 2 góc đối bng
180 .)
b, Theo tính cht 2 tiếp tuyến ct nhau ta có
BC AO
ti E. Áp dng h thức lượng trong
tam giác vuông ABO ta có:
22
..OE OA OB R==
c. Vì điểm A c định nm ngoài (O) nên AB, AC
c định suy ra
AB AC+
không đổi.
Theo tính cht 2 tiếp tuyến ct nhau ta có:
, PK PB QK QC==
suy ra chu vi tam giác APQ là:
AP AQ PQ AP AQ PB QC AB AC+ + = + + + = +
không đổi.
d. Gi s BK ct PO ti I, CK ct OQ ti J thì
90KIO KJO= =
nên t giác KIOJ ni tiếp nên
//KOJ KIJ QKC KBC IJ BC= = =
hay
.IJ AO
T đó ta có:
90 .BPO BKO IJO JAO QON= = = =
Xét
MOP
NQO
có:
PMO ONQ=
MPO QON=
suy ra
MOP
NQO
(g.g)
2
. . .
4
PM ON MN
PM QN OM ON
OM QN
= = =
Ta cũng có:
( ) ( )
22
2
..
4 4 4
PM QN PM QN
MN
PM QN PM QN MN
++
+
CHUYÊN ĐỀ BỒI DƯỠNG HỌC SINH GIỎI HÌNH HỌC 9
148
Dấu đẳng thc xy ra khi và ch khi
//PM QN PQ MN=
hay K là điểm chính gia cung nh
BC.
Bài 44. Cho đường tròn
( )
,OR
đường thng (d) không ct
( )
,O
điểm M nằm trên đường thng
(d), qua M dng các tiếp tuyến MA, MB đến
( )
,OR
(A, B là các tiếp điểm. Gi H là hình chiếu
vuông góc ca
( )
O
lên (d). Các đường thng ct AB lần lượt ti I, K. Tìm v trí điểm M trên (d) để
din tích tam giác OIK ln nht.
NG DN GII:
Ta có:
2
..OA OI OM OK OH==
suy ra
22
OA R
OK
OH OH
==
không đổi (do OH
không đổi). Din tích tam giác OIK là:
( )
2 2 2
11
.
2 4 4
OIK
S IK IO IK IO OK
= + =
Tc là:
4
2
.
4
OIK
R
S
OH
Dấu đẳng thc xy ra
khi và ch khi
IK KO=
suy ra tam giác KIO
vuông cân. Suy ra
45 45 .KOI OMH= =
Tức là điểm M cách H 1 đoạn bng OH.
Bài 45. Cho đường tròn tâm O bán kính R, hai đường kính AB, CD vuông góc vi nhau.
Lấy điểm M bt k thuộc đoạn
( )
, .OA M O A
Tia DM cắt đường tròn
( )
O
ti N.
a, Chng minh: T giác OMNC ni tiếp.
b, Chng minh:
2
. . 2 .DM DN DO DC R==
c, Tiếp tuyến ti C ca
( )
O
ct tia DM ti E, đường tròn ngoi tiếp tam giác CDE ct BC ti F.
Chng minh:
/ / .DF AN
d, Ni B vi N ct OC ti P.
Tìm v trí điểm M để
OM OP
AM CP
+
nh nht.
NG DN GII:
a, T giác OMNC
90MNC COM= =
nên
180MNC COM+ =
suy ra OMNC là t giác ni tiếp (tng 2
góc đối bng
180 .)
CHUYÊN ĐỀ BỒI DƯỠNG HỌC SINH GIỎI HÌNH HỌC 9
149
b, Xét tam giác vuông DOM và tam giác vuông DNC ta có:
90 ,DOM DNC= =
NDC
chung. Suy
ra
DNC
DOM
(g.g)
2
. . .2 2 .
DN DC
DM DN DO DC R R R
DO DM
= = = =
c, Tam giác ECD vuông ti
, / / , C MO EC O
là trung điểm CD nên MO là đường trung bình ca
tam giác ECD suy ra M là trung điểm ca ED. Đường tròn ngoi tiếp tam giác CDE chính là đường
tròn tâm M bán kính MC.
T giác DFCE ni tiếp nên
DEF DCF=
(cùng chn cung DF), t giác DNCB ni tiếp nên
DNB DCB=
(cùng chn cung BD). T đó suy ra
/ / .DNB DEF NB EF=
, NB AN EF DF
//AN DF
(t vuông góc đến song song).
d, Ta có:
2
OM OP OA AM OC CP OA OC
AM CP AM CP AM CP
−−
+ = + = +
Ta có
( )
( )
11
, 45 ,
22
BPD sđ NC BD NC BC MDB PDB MBD= + = + = = =
Suy ra
MDB
22
. 2 .
MB DB
BPD MB DP BD R
BD DP
= = =
Ta có:
( )( ) ( )
2
, . 2 2 4 . 2AM AB MB CP CD PC AM CP R MB R DP R MB PD R MB PD= = = = + +
( )
2
6 2 .R R MB PD= +
Theo bất đẳng thc AM - GM ta có:
2 . 2 2MB DP MB DP R+ =
suy
ra
( ) ( )
2 2 2
2
2
. 6 4 2 2 3 2 2 2 2 1 .AM CP R R R R = =
Theo bất đẳng thc AM-GM ta có:
( )
( )
2
2
. 2 2 2
2 2 2
.
.
21
2 2 1
OA OC OAOC R R
AM CP AM CP
AM CP
R
+ = = = +
Dấu đẳng thc xy ra khi và ch khi
( )
2 2 2 2
22
2
2 2 2 1
OA OC OA R
AM R
AM CP
+
= = = = =
++
Suy ra điểm M nằm trên đoạn OA sao cho
( )
2 2 .AM R=−
Vy GTNN ca
OM OP
AM CP
+
2
Bài 46. Cho tam giác ABC ni tiếp
( )
;OR
và ngai tiếp
( )
;.Ir
Chng minh rng:
2
R
r
NG DN GII:
CHUYÊN ĐỀ BỒI DƯỠNG HỌC SINH GIỎI HÌNH HỌC 9
150
Ta cn tìm mi liên h gia hai bán kính R, r.
Gi s AI kéo dài ct (O) ti P. OI ct (O) ti M, N.
H
,IK AB
dựng đường kính PQ ca (O).
Ta có I nm trong (O)
( )( )
22
. . .IP IA IM IN R OI R OI R OI= = + =
Mặt khác ta cũng có
AKI
QCP
nên
. 2 . .
IK PC
IAPC r R
IA PQ
= =
Nhưng ta dễ chng minh
được:
PC PI=
suy ra
. 2 .IAIP Rr=
hay
2 2 2 2
2 . 2 . 0 2.
R
R r R OI R R r OI
r
= =
Du
bng xy ra khi và ch khi
OI
hay tam giác ABC đều.
Bài 47. Cho hai đường tròn
( )
O
( )
'O
ct nhau ti A, B. Mt cát tuyến qua A ct
( ) ( )
,'OO
ln
t ti M, N (M nm ngoài
( )
' , ON
nm ngoài
( )
).O
Các tiếp tuyến ca
( ) ( )
,'OO
ti M, N ct
nhau P. Tìm v trí ca cát tuyến MAN để bán kính đường tròn ngoi tiếp tam giác MNP ln nht.
NG DN GII:
Do MP, NP lần lượt là các tiếp tuyến ca
( )
O
( )
'O
nên ta
có:
PMN PNM MBA NBA+ = +
Hay
180 180MPN MBN MBN MPN = + =
Suy ra PMBN là t giác ni tiếp.
T đó ta có:
11
'
22
MPN NPB MNB NMB AOB AO B+ = + = +
không đổi.
Suy ra
MPN
không đổi. Gi R là bán kính đường tròn ngoi
tiếp tam giác MNP ta có:
2 .sin .MN R MPN=
Suy ra R ln nht khi MN ln nht. Gi H, K lần lượt là trung điểm ca AM,
AN ta suy ra
2.MN HK=
Dng
'OE O K
thì
.OE HK=
Ta có:
2 2 2 '.MN HK OE OO= =
Du bng xy ra khi và ch khi
'EO
hay
.MN AB
Bài 48. Cho tam giác ABC có ba góc nhn, ni tiếp đường tròn
( )
.O
Các đường cao AM, BN, CP
ca tam giác ABC cùng đi qua điểm H. Gi Q là điểm bt kì trên cung nh BC (Q khác B, Q khác
C). Gi E, F theo th t là điểm đối xng ca Q qua các đường thng ABAC.
a, Chng minh
. . .MH MA MP MN=
b, Chứng minh ba điểm E, H, F thng hàng.
CHUYÊN ĐỀ BỒI DƯỠNG HỌC SINH GIỎI HÌNH HỌC 9
151
b, Gi J là giao điểm ca QEAB, I là giao điểm ca QFAC. Tìm v trí của điểm Q trên cung
nh BC để
AB AC
QJ QI

+


nh nht
(Trích đề tuyn sinh vào lớp 10 chuyên Toán trường THPT chuyên Hà Ni Amsterdam 2016)
NG DN GII:
a, Xét tam giác MHP và tam giác MNA, ta có: APMC, MHCN ni tiếp nên
, AMP ACP NMH MPH HAN= = =
nên
MHP
MNA
MH MP
MN MA
=
hay
. . .MH MA MN MP=
b, Dng
QK BC
ta có các t giác JBKQ,
QKIC, ABQC ni tiếp nên ta có biến đổi
góc sau:
180JKQ JBQ ACQ QKI= = =
hay
180JKQ QKI+ =
nên J, K, I thng
hàng.
Gi s tia CH, BH ct
( )
O
tại giao điểm
th 2 là R, S thì
,RAB RCB HAB==
tương
t
RBA HBA=
nên suy ra H đối xng vi R qua AB, tương tự H đối xng vi S qua AC.
Ta có t giác ERHQ hình thang cân và t giác CRBQ, KBJQ ni tiếp nên
HEQ HRQ CBQ KJQ= = =
nên suy ra
/ / ,HE KJ
chứng minh tương tự ta có:
//HF KI
I, K, J
thng hàng nên suy ra E, H, F thng hàng.
c, Trên BC lấy điểm T sao cho
QCT QBA=
suy ra
QBA
,QCT
QJ, QK là các đường cao
tương ứng nên suy ra
AB TC
QJ QK
=
(1) tương tự
BQT
AC TB
AQC
QI QK
=
(2).
T (1) và (2) ta suy ra
AB AC TC TB BC
QJ QI QK QK QK
+ = + =
suy ra
AB AC
QJ QI
+
nh nht khi và ch khi
BC
QK
nh nht, hay QK ln nht suy ra Q là điểm chính gia cung nh BC.
Bất đẳng thc Erdos Mordell
Cho tam giác ABCM là một điểm bt k nằm trong tam giác đó. Gọi
, ,
a b c
R R R
theo th t
khong cách t M đến các đỉnh A, B, C. Còn
, ,
a b c
d d d
làn lượt là khong cách t M đến các cnh
BC, CA, AB. Khi đó ta có bất đẳng thc:
a,
( )
2.
a b c a b c
R R R d d d+ + + +
b,
. . 8 . .
a b c a b c
R R R d d d
Đẳng thc xy ra khi và ch khi tam giác ABC đều và M là tâm ca tam giác.
CHUYÊN ĐỀ BỒI DƯỠNG HỌC SINH GIỎI HÌNH HỌC 9
152
NG DN GII:
a, Đặt
, , .BC a CA b AB c= = =
Lấy điểm
1
M
đối xng với điểm M qua đường phân giác trong ca
.BAC
Dng
1
,BH AM
1
CK AM
Gi s
1
AM
ct BC ti D.
Khi đó
, .BD BH DC CK
Đẳng thc xy ra khi và ch khi
AD BC
hay
1
.AM BC
T đó ta có:
11
22
a ABM ACM
a BH CK aR S S + +
(chú ý
rng
1
)
a
AMAM R==
hay
.
a b c
aR cd bd+
T đó
a b c
cb
R d d
aa
+
(1). Tương tự ta có
b c a
ac
R d d
bd
+
(2);
c b a
ab
R d d
cc
+
(3).
Cng theo vế các bất đẳng thc (1), (2), (3) ta
thu được:
( )
2.
a b c a b c a b c
b c a c a b
R R R d d d d d d
c b c a b a
+ + + + + + + + +
(S dng bất đẳng thc AM-
GM dng
2x y xy+
cho các biu thc trong ngoặc). Đẳng thc xy ra khi và ch khi
abc==
đồng thi
1
M
là trc tâm ca tam giác ABC. Nói cách khác,
1
M
(và do đó cả M) là tâm ca tam
giác đều ABC.
b, T cách chng minh bất đẳng thc Erdos-Mordell câu a) ta có:
a b c
cb
R d d
aa
+
(1);
b c a
ac
R d d
bb
+
(2);
c b a
ab
R d d
cc
+
(3).
Nhân theo vế ba bất đẳng thức trên ta được:
..
ca b c c b ab a
a c a b
Rd
cb
ddR R d d d
ba ba cc
+

+
+
Áp dng bất đẳng thc:
2x y xy+
ta có:
2 .2. . .2. 8. .
c a b a a b cbc
a c a b
dd
b
d d d d d
bba cc
c
dd
a
=
(đpcm).
Bài 49. Gi I là tâm, r là bán kính đường tròn ni tiếp tam giác ABC. Chng minh rằng điều kin
cần và đủ để tam giác ABC đều và
6.IA IB IC r+ + =
NG DN GII:
K IH, IJ, IK theo th t vuông góc vi các cnh BC, CA,
AB. Ta có
.IH IJ IK r= = =
Áp dng bất đẳng thc Erdos-
Mordell cho điểm I trong tam giác ABC, ta thy
CHUYÊN ĐỀ BỒI DƯỠNG HỌC SINH GIỎI HÌNH HỌC 9
153
( )
2 6 .IA IB IC IH IJ IK r+ + + + =
Đẳng thc xy ra khi và ch khi tam giác ABC đều. Nói cách
khác, điều kin cần và đủ để tam giác ABC đều là
6IA IB IC r+ + =
(đpcm)
Bài 50. Gi s M là một điểm bt k nm trong tam giác ABC. Gi r là bán kính đường tròn ni tiếp
tam giác. Chng minh rng
6.MA MB MC r+ +
Đẳng thc xy ra khi nào?
NG DN GII:
Gi x, y, z lần lượt là khong cách t M đến các cnh BC, CA, AB.
K AH vuông góc vi BC,
1
MA
vuông góc vi BC. Khi đó
ta có
1
.AM MA AH+
T đó
2
x.
ABC
S
AM
BC
−
Tương tự,
22
; .
ABC ABC
SS
BM y CM z
CA AB
Cng theo vế ba bất đẳng thức này ta được:
( )
1 1 1
2
ABC
MA MB MC S x y z
BC CA AB

+ + + + + +


( ) ( )
1 1 1
x y z
BC CA
r BC CA
A
AB
B

+ + + +

=+

+
(1)
Áp dng bất đẳng thc AM-GM ta được:
( )
1 1 1
9
BC CA AB
BC CA AB++

+ +


(2).
Áp dng bất đẳng thc Erdos-Mordell cho điểm M đối vi tam giác ABC ta có:
( )
2MA MB MC x y z+ + + +
(3).
T (1), (2), (3) suy ra
9
2
MA MB MC
MA MB MC r
++

+ +


hay
6MA MB MC r+ +
Đẳng thc xy ra khi và ch khi tam giác ABC đều (đpcm).
Bài 51. Gi s H là trc tâm ca tam giác nhn ABC. Gi D, E, F lần lượt là trung điểm ca BC,
CA, AB; R là bán kính đường tròn ngoi tiếp tam giác ABC. Chng minh bất đẳng thc
3
.
2
HD HE HF R+ +
NG DN GII:
Gi O là tâm đường tròn ngoi tiếp tam giác ABC, K
điểm đối xng vi H qua BC suy ra K nm trên
( )
O
.HD DK=
Áp dng bất đẳng thức cho 3 điểm bt k ta có:
CHUYÊN ĐỀ BỒI DƯỠNG HỌC SINH GIỎI HÌNH HỌC 9
154
.HD OD DK OD OK R+ = + =
Suy ra
( )
3HD HE HF R OD OE OF+ + = + +
(1)
Áp dng bất đẳng thc Erdos-Mordell cho điểm
O nm trong tam giác ABC ta có:
3
22
OA OB OC R
OD OE OF
++
+ + =
(2).
T (1) và (2) suy ra
3
2
HD HE HF R+ +
(đpcm). Đẳng thc xy ra khi và ch khi tam giác ABC
đều.
Bài 52. Cho tam giác nhn ABC ni tiếp đường tròn
( )
;OR
các đường cao
1 1 1
,,AA BB CC
ct nhau
ti H.
a, Gi s BC c định, điểm A chuyển động trên cung ln BC. Tìm v trí điểm A để
HA HB HC++
ln nht.
b, K
1
OO
vuông góc vi
2
, BC OO
vuông góc vi
3
,AC OO
vuông góc vi AB. Chng minh rng:
11 1 1 2 3
3
2
R
HA HB HC OO OO OO+ + + +
NG DN GII:
a, Ta có tính cht H thuộc đường tròn c định
( )
('; 'M R M
là điểm đối xng vi O qua BC) mà BC
dây cung c định ca
( )
';MR
nên
BHC
không đổi suy
ra
180EHC BHC
= =
không đổi. Ta cũng có:
2AH OM=
suy ra HA có độ dài không đổi.
Như vậy để tìm GTLN ca
HA HB HC++
ta quy v tìm
GTLN ca
.HB HC+
Trên tia đối ca tia HB ta ly một điểm
'E
sao cho
''HE HC HE C=
cân ti
H
' 180 2 ' 180 2HE C CHE
= =
không đổi.
Suy ra
'E
chuyển động trên cung cha góc
180 2
−
dựng trên đoạn BC.
Ta có:
'HB HC BE+=
nên
HB HC+
ln nht khi và ch
khi
'BE
là đường kính của đường tròn cha cung cha góc
180 2
−
dựng trên đoạn BC tc là
'.BC E C
T đó suy ra cách dựng điểm A như sau:
CHUYÊN ĐỀ BỒI DƯỠNG HỌC SINH GIỎI HÌNH HỌC 9
155
Dng cung cha góc
180 2
−
trên đoạn BC, dng tia
Cx BC
ct cung cha góc ti
'.E
Dng
đường thng qua B vuông góc vi
'BE
ct
( )
O
tại điểm A nm chính gia cung BC là v trí cn
tìm.
b, Ta có tính cht quen thuc:
1 2 3
2 , 2 ; 2 .HA OO HB OO HC OO= = =
Áp dụng đẳng thc Erdos- Mordell cho điểm H trong tam giác ABC, ta có:
1 1 11 23
.
2
HA HB HC
HA HB HC OO OO OO
++
+ + = + +
Áp dng bất đẳng thc Erdos- Mordell cho
điểm O trong tam giác ABC ta có:
1 2 3
3
22
OA OB OC R
OO OO OO
++
+ + =
(đpcm)
Câu b) thc cht là dng tng quát ca câu a).
Bài 53. Cho đường tròn
( )
O
đường kính AB c định. Lấy điểm I nm gia AO sao cho
2
.
3
AI AO=
K dây MN vuông góc vi AB ti I, gi C là điểm tùy ý thuc cung ln MN sao cho C
không trùng vi M, NB. Ni AC cắt đoạn thng MN ti E.
a, Chng minh rng t giác IECB ni tiếp.
b, Chng minh rng
2
..AM AE AC=
c, Xác định v trí điểm C sao cho khong cách t N đến tâm đường tròn ngoi tiếp tam giác CME
nh nht.
NG DN GII:
a, Ta có
90 ; 90EIB ECB= =
t giác IECB ni tiếp đường tròn đường kính BE.
b, Ta có
AB MN AM AN =
nên
.AMN ACM=
Li có
CAM
chung nên
AME
ACM
(g.g)
2
.
AM AE
AM AE AC
AC AM
= =
c, Trên na mt phng b ME có chứa điểm A, ta v tiếp
tuyến
Mx
của đường tròn ngoi tiếp
MCE
EMx ECM=
EMA ECM=
(chng minh câu b)
EMx EMA MA =
và tia Mx trùng nhau
MA
là tiếp
tuyến của đường tròn ngoi tiếp
MCE
. Gi
1
O
là tâm
đường tròn ngoi tiếp tam giác
1
.MCE OM MA⊥
Mt
khác ta có
1
,,BM MA M O B⊥
thng hàng
1
NO
nh
nht khi
1
.NO BM
Khi đó ta xác định được điểm C như sau: Kẻ
1
NO MB
ti
1
.O
V đường
tròn tâm
1
O
bán kính
1
OM
cắt đường tròn
( )
O
tại điểm th hai là C.
CHUYÊN ĐỀ BỒI DƯỠNG HỌC SINH GIỎI HÌNH HỌC 9
156
Bài 54. Cho đường tròn
( )
;OR
và hai đường kính
,MN PQ
vuông góc vi nhau. Lấy điểm A trên
cung nh PN, PA ct MN ti B, AQ ct MN ti E.
a, Chng minh: OABQ là t giác ni tiếp.
b, Ni AM ct PQPN lần lượt ti C, I. Chng minh: Tích
.MC MA
không đổi khi A di chuyn
trên cung nh PN.
c, Chng minh:
2.IN EN=
d, Tìm v trí điểm A để din tích tam giác ACE ln nht.
NG DN GII:
a, Vì điểm A nằm trên đường tròn đường kính PQ nên
90PAQ =
suy ra
90 .QAB =
Ta có
90QOB =
(gi thiết
.)PQ MN
T đó suy ra
90QAB QOB OABQ= =
là t giác ni tiếp (hai đỉnh liên tiếp trên 1 cnh OA cùng
nhìn cnh BQ mt góc
90 .)
b, Vì điểm A nằm trên đường tròn đường kính MN nên
90 .MAN =
Xét các tam giác MANMOC có:
AMN
chung
90MAN MOC= =
suy ra
MAN
MOC
(g.g) suy ra
MA MN
MO MC
=
(t s đồng dng) hay
..MAMC MO MN=
,2MO R MN R==
nên
2
.2MAMC R=
(không đổi).
c, Vì PQ, MN là 2 đường kính ca
( )
O
PQ MN
nên PMQN là hình vuông.
Xét t giác IANE: Ta có
1
45
2
IAE MAQ MOQ= = =
(quan h góc ni tiếp
vi góc tâm chn cùng 1 cung)
45INE PNO= =
suy ra
IAE INE=
do đó IANE là t giác ni
tiếp (Hai đỉnh liên tiếp trên cùng mt cnh AN cùng nhình cnh IE góc bng nhau). Suy ra
180IAN IEN+ =
180 180 90 90 .IEN IAN = = =
Tam giác IEN vuông ti E và có
45INE IEN=
là tam giác vuông cân, dẫn đến
2.IN EN=
d, Ta có
.
ACE AMQ MCEQ
S S S=−
Xét tam giác MQE và tam giác QCQ ta có:
45 ,MQC QME= =
CHUYÊN ĐỀ BỒI DƯỠNG HỌC SINH GIỎI HÌNH HỌC 9
157
( ) ( )
1 1 1
2 2 2
MCQ sđ PA MQ PA MP MA MQE= + = + = =
suy ra
MQE
QCM
(g.g) suy
ra
22
.2
MQ ME
QC ME MQ R
QC MQ
= = =
suy ra
2
1
.
2
MCEQ
S QC ME R==
Như vậy ta có:
2
ACE AMQ
S S R=−
nên
ACE
S
ln nht khi và ch khi
AMQ
S
ln nht.
K
,AH MQ
gi K là trung điểm MQ thì ta có
OK MQ
2
2
2 2 2 2
22
2 2 2
R R R
OK OM KM R OK

= = = =



Ta có:
11
. 2. ,
22
AMQ
S AH MQ R AH==
trong tam giác AHK ta có:
,AH AK
mặt khác ta cũng
2 2 2
,
22
R
AK AO OK R R

+
+ = + =



suy ra
2
1 2 2 1 2
2
2 2 2
AMQ
S R R R

++
=



Vy
2 2 2 2
1 2 2 1
.
22
ACE AMQ
S S R R R R
+−
= =
Dấu đẳng thc xy ra khi và ch khi
HK
A, O, K thẳng hàng. Suy ra điểm A là điểm chính gia cung nh PN.
Vậy để din tích tam giác ACE ln nhất thì điểm A phải là điểm chính gia cung nh PN.
Bài 55. Cho đường tròn
( )
O
đường kính AB và một điểm C nm trên
( )
,O
lấy điểm M thuc cung
nh BC (M khác B, C). Các đường thng AC, BM ct nhau ti D. Phân giác trong góc BAM ct AB
ti P. Tìm giá tr nh nht ca biu thc
MA MB
MP
+
NG DN GII:
Kéo dài MP ct
( )
O
ti Q.
Ta có:
MPB AQB=
đối đỉnh.
BMP QAP=
cùng chn cung BQ ca
( )
O
Suy ra
MBP
AQP
.MB MP MP AQ
MB
AQ AP AP
= =
Tương tự ta có:
.MP BQ
MA
BP
=
Suy ra
. . .
+ 2 .MP
.
MP BQ MP AQ BQ AQ
MA MB
BP AP BP AP
+ =
Li có
( )
2
2
2
. , 2
44
BP AP
AB
BP AP R BQ AQ R
+
= = = =
CHUYÊN ĐỀ BỒI DƯỠNG HỌC SINH GIỎI HÌNH HỌC 9
158
Suy ra
2 2.MA MB MP+
hay
22
MA MB
MP
+
Dấu đẳng thc xy ra khi và ch khi
,MB MA BP PA==
hay M
điểm chính gia cung BC.
Cách khác:
Áp dụng định lý Ptolemy cho t giác ni tiếp MBQA ta có:
. . .MBQA MAQB MQ BA+=
do Q là điểm chính gia cung BC nên
tam giác ABQ vuông cân ti Q suy ra
2,QB QA R==
2AB R=
dẫn đến
2.MA MB MQ+=
Ta suy ra
,
2
MP MP
MA MB
MQ
=
+
dng
ME BC
thì
1 1 2
MQ PQ OQ
MP MP ME
= + = +
suy ra
22
MA MB
MP
+
dấu đẳng
thc xy ra khi và ch khi
ME OQ=
hay M là điểm chính gia
cung BC
Bài 46. Cho hình vuông ABCD ni tiếp đường tròn
O
. Điểm M thuc cung nh CD ca
O
, M
khác CD. MA ct DB, DC theo th t X, Z; MB ct CA, CD ti Y, T; CX ct DY ti K.
1) Chng minh rng:
,MXT TXC MYZ ZYD
135CKD
.
2) Chng minh rng:
1
KX KY ZT
MX MY CD
.
3) Gi I là giao điểm ca MKCD. Chng minh rng XT, YZ, OI cùng đi qua tâm đường tròn
ngoi tiếp tam giác KZT.
(Trích đề thi vào 10 Chuyên Toán, ĐHSP HN, năm học 2012 - 2013)
NG DN GII
a) Ta có
1
2
DXM sd DM sd AB DTM
nên t giác DXTM ni tiếp.
90 90DMT DXT
Suy ra
TX BD
, mà
D / /AC B TX AC
CHUYÊN ĐỀ BỒI DƯỠNG HỌC SINH GIỎI HÌNH HỌC 9
159
Do đó
MXT ZAC XCA TXC
ơng tự, t giác MCYZ ni tiếp
Suy ra
//ZY BD
nên
MYZ MBD BDY ZYD
Ta có t giác ADZY ni tiếp nên
YDC YAZ MDC
Tương tự t giác BCTX ni tiếp nên
XCD XBM MCD
Nên
. . 135DMC DKC g c g DKC DMC
Ta có
180 45XKD DKC DMX
nên t giác DXKM ni tiếp.
DXTM ni tiếp nên 5 điểm D, X, K, T, M cùng nm trên một đường tròn tâm E đường kính DT.
Tương tự 5 điểm Y, K, Z, M, C nằm trên đường tròn tâm F đường kính ZC suy ra
..XK XC XZ XM
Suy ra
XK XZ XZ DZ DZ
XM XC XA BA DC
Tương tự
YK CT
YM CD
nên
1
XK KY ZT DZ CT ZT
XM YM CD CD
c) Gi H là giao điểm XTYZ. Ta chng minh H là tâm đường tròn ngoi tiếp tam giác KZT.
Ta có
45HZT HTZ HT HZ
(1)
T giác KHZX ni tiếp, nên:
HKZ HXZ HXK HZH HK HZ
(2)
T (1) và (2) suy ra H là tâm đường tròn ngoi tiếp tam giác KZT.
Gọi giao điểm ca XCYZF.
Do
90FKZ
nên F, H, Z thng hàng.
Tương tự, gi XT giao ID ti E. Ta có E, H, T thng hàng
HEF COD
suy ra
HF EF HT FT
OC CD OC CB
//FT BC
nên
FT IT HT IT
BC IC OC IC
, mà
45HTI OCI
Nên suy ra
ITH IOC
do đó
HTI OCT
, hay O, I, H thng hàng.
Bài 47. Cho tam giác ABC vuông ti A, ni tiếp trong đường tròn
O
. Trên cung BC không cha A,
lấy điểm M tu ý (M khác C). P là điểm trên cnh BC sao cho
BAM PAC
. Trên các tia AB, AC
ly lần lượt các điểm E, F sao cho
BE CF BC
.
1) Chng minh:
ABP AMC
. . .MC AB MB AC MA BC
.
2) Chng minh
..MB AE MC AF
MA MB MC
BC
CHUYÊN ĐỀ BỒI DƯỠNG HỌC SINH GIỎI HÌNH HỌC 9
160
3) Xác định v tri điểm N trên đường tròn
O
để tng
NA NB NC
ln nht.
(Trích đề thi vào 10 Chuyên Toán, Quảng Bình, năm học 2012 - 2013)
NG DN GII
Ta có:
ABP AMC
(cùng chn cung AC)
BAM PAC BAP MAC
Nên:
ABP AMC
Suy ra:
..
AB BP
MC AB MA BP
MA MC
(1)
Mt khác:
,BMA BCA BAM PAC ABM APC
..
MB MA
MB AC MA PC
PC AC
(2)
T (1) và (2) suy ra:
. . .MC AB MB AC MA BC
T kết qu câu a) ta có:
..
AC AB
MA MB MC
BC BC
Do đó:
. 1 1
AC AB
MA MB MC MB MC
BC BC
..
AC BC AB BC
MB MC
BC BC
..
AC CE AB BF
MB MC
BC BC
..MB AE MC AF
BC
Xét trường hp N thuc cung BC không cha A.
- Nếu N khác C theo kết qu câu b) ta có
..NB AE NC AF
NA NB NC
BC
(3)
- Nếu N trùng C, ta thy (3) vẫn đúng.
CHUYÊN ĐỀ BỒI DƯỠNG HỌC SINH GIỎI HÌNH HỌC 9
161
Mt khác
2 2 2 2
2 . . . .NB AF NC AE NB AF NC AE
2
2 2 2 2 2 2
. . .NB AE NB AF NB NC AE AF BC EF
(4)
T (3) và (4) suy ra
NA NB NC EF
Du bng xy ra khi và ch khi
..NB AF NC AE
hay
NBC AEF
.
Xét trường hp N thuc cung BC cha A. Lấy N’ đối xứng N qua BC, khi đó N’ thuộc cung BC
không cha A,
,,N A NA N B NB N C NC
.
Áp dụng trường hp trên ta có:
NA NB NC N A N B N C EF
Vy trong mọi trường hp thì
NA NB NC
có giá tr ln nhất là EF, đạt được khi
NBC AEF
Bài 48. Cho tam giác OAB vuông cân ti O vi
2OA OB a
. Gi
O
là đường tròn tâm O bán
kính a. Tìm điểm M thuc
O
sao cho
2MA MB
đạt giá tr nh nht.
(Trích đề thi vào 10 Chuyên Toán, TP HCM, năm học 2010 - 2011)
NG DN GII
Đưng thng OA ct
O
ti C và D, B
với C là trung điểm ca OA. Gi E là
trung điểm ca OC.
* Trường hp M không trùng vi C và D.
Hai tam giác OEM và OMA đồng dng
(do
1
,
2
OM OE
MOE AOM
OA OM
)
Suy ra
1
2.
2
ME OM
MA EM
AM OA
* Trường hp M trùng vi C:
2. 2.MA CA EC EM
* Trường hp M trùng vi D:
2. 2.MA DA ED EM
Vy ta luôn có
2.MA EM
. Do đó
2. 2 2.MA MB EM MB EB
Dấu “=” xảy ra khi M là giao điểm của đoạn BE với đường tròn
O
.
Vy
2.MA MB
nh nhất khi M là giao điểm của đoạn BE với đường tròn
O
.
Bài 49. Cho ba điểm A, M, B phân bit, thng hàng và M nm gia A, B. Trên cùng mt na mt
phng b là đường thng AB, dựng hai tam giác đều AMC và BMD. Gọi P là giao điểm ca AD và
BC.
1) Chng minh AMPC và BMPD là các t giác ni tiếp.
CHUYÊN ĐỀ BỒI DƯỠNG HỌC SINH GIỎI HÌNH HỌC 9
162
2) Chng minh
..CP CB DP DA AB
.
3) Đường thng ni tâm của hai đường tròn ngoi tiếp hai t giác AMPC và BMPD ct PA, PB
tương ứng ti E, F. Chng minh CDFE là hình thang.
(Trích đề thi vào 10 Chuyên Toán (Vòng 1), ĐHSP Hà Nội, năm hc 2016-2017)
NG DN GII
1) Vì
60 120CMA DMB CMB DMA
- Xét
CMB
AMD
(c.g.c)
CM AM
CMB DMA CMB AMD
MB MD
MCB MAD
MBC MDA
Suy ra AMPC và BMPD là các t giác ni tiếp.
2) Vì AMPC là t giác ni tiếp nên
180 120CPM CAM CMB
(g.g)
CP CM
CPM CMB
CM CB
2
..CP CB CM CP CB CM
ơng tự
.DP DA DM
Vy
..CPCB DP DA CM DM AM BM AB
3) Ta có EF là đường trung trc ca PM
EB EM
, suy ra
EPM
cân ti E
Mt khác
60EPM ACM
(do AMPC là t giác ni tiếp)
nên
EPM
đều, nên
PM PE
.
Tương tự
PF PM
.
Ta có
//CM DB
nên
PCM PBD
Mà BMPD là t giác ni tiếp nên
PBD PMD
.
Suy ra
PCM PMD
Ta li có
120CPM DPM
(g.g)
CP PM CP PE
CPM MPD
MP PD PF PD
Theo định lý Thales đảo ta có
//CE DF
, suy ra CDFE là hình thang.
CHUYÊN ĐỀ BỒI DƯỠNG HỌC SINH GIỎI HÌNH HỌC 9
163
Bài 50. Cho tam giác ABC. Trên các cnh BC, CA, AB lần lượt lấy các điểm D, E, F. Gi
1
d
đường thng qua D và vuông góc vi BC,
2
d
là đường thng qua E và vuông góc vi CA,
3
d
đường thng qua F và vuông góc vi AB. Chng minh rng
12
,dd
3
d
đồng quy khi và ch
khi có đẳng thc sau:
2 2 2 2 2 2
0DB DC EC EA FA FB
.
(Trích đề thi vào lớp 10 Chuyên Toán THTH, ĐHSP TP HCM năm học 2014 - 2015)
NG DN GII
Gọi M là giao điểm ca
1
d
2
d
F
là hình chiếu ca M lên AB.
Ta có
2 2 2 2 2 2
0DB DC EC EA FA FB
2 2 2 2 2 2 2
0BM MD DC EC EA FA FB
2 2 2 2 2 2
0BM MC EC EA FA FB
2 2 2 2 2
0BM ME EA FA FB
2 2 2 2
0BM MA FA FB
2 2 2 2
0F B F A FA FB
0F B F A FA FB
(1)
F A F B FA FB AB
nên
1 2 3
1 , ,F B FB F F d d d
đồng quy.
Bài 51. Cho hai đường tròn
11
;OR
22
;OR
vi
12
RR
tiếp xúc trong vi nhau ti A. Đường
thng ct
11
;OR
22
;OR
lần lượt ti BC khác A . Đường thẳng đi qua trung điểm D ca BC
vuông góc vi BC ct
11
;OR
ti PQ.
1) Chng minh C là trc tâm tam giác APQ.
2) Chng minh
2 2 2
12
DP R R
.
3) Gi s
1 2 3 4
; ; ;D D D D
lần lượt là hình chiếu vuông góc ca D xuống các đưng thng
; ; ;BP PA AQ QB
. Chng minh:
1 2 3 4
1
2
DD DD DD DD BP PA AQ QB
(Trích đề thi vào 10 Chuyên Toán (Vòng 1) Lê Hồng Phong, Nam Định, năm 2014 - 2015)
NG DN GII
1) Gi M là giao điểm ca AP với đường tròn
2
O
.
CHUYÊN ĐỀ BỒI DƯỠNG HỌC SINH GIỎI HÌNH HỌC 9
164
Ta có PBQC là hình thoi, nên
, mà
//CM BP
(cùng vuông góc vi AP) nên ta có
Q, C, M thng hàng. Tam giác APQ có hai
đường cao ADQM ct nhau ti C nên C
trc tâm tam giác APQ.
2) Ta chứng minh được DM là tiếp tuyến ca
đường tròn
2
O
.
Ta có
2 2 2 2
22
..PD BD DA DC DA DM O D R
Mt khác
2 2 1
2 2 2
AC BC AB
O D O C CD R
Nên ta có
2 2 2
12
PD R R
3) Ta Chứng minh được
1 4 2 3
, , ,DD DD DD DD BP BQ PA PB
nên
1 2 3 4
1
2
DD DD DD DD BP PA AQ QB
12
2 DD DD PA PB
Ta có
2 2 2
1
2.
2 . 2
DB DP
PB BD DP DB DP PB DD
PB
.
Tương tự:
2
2.
2
DA DP
AP DD
PA
. T đó, ta có điều phi chng minh.
Đẳng thc xy ra khi
DA DB DP
.
Bài 52. Cho
ABC
ni tiếp đường tròn
O
. Đường cao
1 1 1
,,AA BB CC
ca tam giác ABC ct nhau
ti H. Đường thng
1
AA
ct
O
ti
KA
.
1) Chng minh rng:
1
A
là trung điểm HK.
2) Tính
1 1 1
HA HB HC
AA BB CC
.
3) Gi M là hình chiếu vuông góc ca O lên BC. Đường thng
1
BB
ct
O
tại giao điểm th hai là
E, đường thng
1
MB
ct AE ti N.
Chng minh rng:
2
1
1
AB
AN
NE EB
.
(Trích đề thi vào 10 Chuyên Toán (Vòng 2) Lê Hồng Phong, Nam Định, năm 2014 - 2015)
NG DN GII
CHUYÊN ĐỀ BỒI DƯỠNG HỌC SINH GIỎI HÌNH HỌC 9
165
1)Ta có
1 2 1
A C C
nên
CHK
cân ti C.
1
CA
là đường cao nên
1
CA
là đường trung trc, suy ra
1
A
là trung điểm ca HK.
2) Ta có:
1 1 1
HA HB HC
AA BB CC
1 1 1
1 1 1
1 1 1
HA HB HC
AA BB CC
1 1 1
1 1 1
3
HA HB HC
AA BB CC
3 3 1 2
HBC HAC
HBA
ABC ABC ABC
SS
S
S S S
3) T gi thiết ta có M trung điểm BC, suy ra
1
B MC
cân ti M, do đó
1 1 1 1 1 1
(g.g) BMBC MCB AB N CBB B AN N AE
Áp dng h thức lượng trong tam giác vuông ta có:
2
1
1
.
.
AB
AN AE AN
EB EN EA EN
(đpcm).
Bài 52. Cho tam giác ABC ni tiếp đường tròn
O
. Lấy điểm D trên cung BC không chứa điểm A
(D khác B, C). Gi H, I, K lần lượt là hình chiếu vuông góc ca D trên các đường thng BC, CA
AB. Chng minh:
BC AC AB
DH DI DK
(Trích đề thi vào 10 Chuyên Nguyễn Du, Đăk Lắk, năm học 2014 - 2015)
NG DN GII
Ta có các t giác BHDK, DHIC, ABDC ni tiếp
KHD KBD ACD
180DHI ACD
180 180DHI KHD KHI
,,K H I
thng hàng.
Mt khác
HCD KAD
(g.g)
HC KA
HCD KAD
DH DK
HB IA
HBD IAD HBD IAD
DH AD
BK IC
BDK BHK CHI CDI BDK CDI
KD DI
CHUYÊN ĐỀ BỒI DƯỠNG HỌC SINH GIỎI HÌNH HỌC 9
166
Do đó, ta có:
BC BH HC AI AK AI AB BK
DH DH DH DI KD DI KD KD
AI AB CI AC AB
DI KD DI DI DK
.
Bài 54. Cho hai đường tròn
,OO
ct nhau ti AB. T điểm C thuộc tia đối ca tia AB k hai
tiếp tuyến đến
O
ti DE, E nm trong
O
. Các đường thng AD, AE ct
O
tại điểm th
hai tương ứng là M, N. Gi I là giao điểm ca DEMN.
1) Chng minh rng t giác BEIN ni tiếp và
BIN BDA
.
2) Chng minh rng
22
CA CD DA
CB CB DB
.
3) Chng minh rng I là trung điểm ca MN.
(Trích đề thi vào Chuyên Toán (Vòng 2), ĐHV, năm học 2015-2016)
NG DN GII
1) Vì t giác ABNM ni tiếp nên
BNI BAD
. (1)
Vì t giác DAEB ni tiếp nên
BAD BED
. (2)
T (1) và (2) suy ra t giác BEIN ni tiếp.
Theo chng minh trên,
BNI BAD
.
Ta li có
BIN BEN BDA
(do các t giác BEIN, AEBD ni tiếp).
Suy ra
BIN BDA
(g.g).
2) Vì CD là tiếp tuyến ca
O
nên
2
.CA CB CD
. T đó suy ra
2
2
22
.CA CA CB CD CD
CB CB CB CB
(3)
CHUYÊN ĐỀ BỒI DƯỠNG HỌC SINH GIỎI HÌNH HỌC 9
167
Li có, t
.
CD DA
CAD CDB g g
CB DB
(4)
T (3) và (4) suy ra
22
CA CD DA
CB CB DB
.
3) Tương tự câu b) ta có
22
CA CE EA
CB CB EB
Suy ra
EA DA
EB DB
(5)
T câu 1),
IN DA
BIN BDA
IB DB
(6)
Tương tự ta có
IM EA
BIM BEA
IB EB
(7)
T (5), (6), (7) suy ra
IM IN
, hay I là trung điểm ca MN.
Bài 55.
1) Cho tam giác ABC ni tiếp đường tròn
O
, có góc A nhn và
AB AC
. Tia phân giác ca góc
BAC
cắt đường tròn
O
ti D (D khác A) và ct tiếp tuyến ti B của đường tròn
O
ti E. Gi F
là giao điểm ca BD và AC
a) Chng minh EF song song vi BC.
b) Gọi M là giao điểm ca AD và BC. Các tiếp tuyến ti B, D của đường tròn
O
ct nhau ti N.
Chng minh rng:
1 1 1
BN BE BM
2) Cho tam giác nhn ABC ni tiếp đường tròn
O
, đường cao AH. Gọi M là giao điểm ca AO và
BC. Chng minh
2
HB MB AB
HC MC AC
. Dấu đẳng thc xy ra khi nào?
(Trích đề thi vào 10 Chuyên Hưng Yên, năm học 2015 - 2016)
NG DN GII
1) a) Do
EBF BAD EAF
nên BEAF ni tiếp được.
Suy ra
180BEF BAF
BAF CBE
(góc to bi tiếp tuyến và dây cung)
Nên
180BEF BCE
, mà đây là hai góc trong cùng phía nên
//BC EF
.
b)
NDB BAD CAD CBD
,
suy ra
//ND BC
.
CHUYÊN ĐỀ BỒI DƯỠNG HỌC SINH GIỎI HÌNH HỌC 9
168
Theo Thales:
1
NB NE
BE BE
1
ND
BM
1
NB ND
BE BM
ND NB
. Do đó
1
NB NB
BE BM
1 1 1
BN BE BM
2) K đường kính AP
D dàng chứng minh được:
AHB ACP
nên:
HB AH
PC AC
Tương tự:
HC AH
PB AB
Suy ra:
.
HB PB AB
HC PC AC
,
Suy ra:
.
HB PC AB
HC PB AC
Li có:
AMC BMP
nên:
MC MP
AC PB
Tương tự:
MB MP
AB PC
Suy ra:
.
MB AC PB
MC AB PC
nên:
.
MB PB AB
MC PC AC
Cng li, ta có:
. 2.
HB MB PB PC AB AB
HC MC PC PB AC AC
Bài 56. Cho tam giác ABC có ba góc nhn, ni tiếp đường tròn
O
. Các đường cao AM, BN, CP
ca tam giác ABC cùng đi qua điểm H. Gi Q là điểm bt kì trên cung nh BC (Q khác BC). Gi
E, F theo th t là điểm đối xng vi Q qua ABAC.
1) Chng minh
..MH MA MP MN
.
2) Chứng minh ba điểm E, H ,F thng hàng.
3) Gi J là giao điểm ca QEAB, I là giao điểm ca QFAC. Tìm v trí của điểm Q trên cung
nh BC để
AB AC
QJ QI
nh nht.
(Chuyên Toán TP Hà Nội, năm hc 2015 - 2016)
CHUYÊN ĐỀ BỒI DƯỠNG HỌC SINH GIỎI HÌNH HỌC 9
169
NG DN GII
1) D dàng thy rng các t giác CNHM, BMHP ni tiếp. Cho nên
NCH NMH
NMP HBP
, kết hp vi
ACH ABH
(cùng ph vi
BAC
) ta suy ra
NMH HMP
(1)
Mt khác t giác ANMB ni tiếp nên
MNH MAB
(2)
T (1) và (2) ta suy ra
HMN PMA
dẫn đến
HM MN
MP MA
..MH MA MN MP
2) Trước hết d thy
ACQ ACF
(c.c.c)
nên
AFC AQC ABC CHM
dẫn đến t giác AFCH ni tiếp và
90ACH AFH BAC
(3)
Mt khác do tính chất đối xng ta có
AF AQ AE
.
hay tam giác AEF cân tại A để
11
90 90
22
AFE AEF EAF FAQ EAQ
90 90CAQ BAQ BAC
Do đó ta được
AFH AFE
hay ba điểm E, H, F thng hàng.
3) Trước hết thy rng
. 2 , . 2
ABQ AQC
ABQJ S AC QI S
Và đặt
AB AC
P
QJ QI
Khi đó áp dụng BĐT Cauchy-Shwarz ta có:
2 2 2 2
. . 2 2
ABQ ACQ
AB AC AB AC
P
AB QJ AC QI S S
CHUYÊN ĐỀ BỒI DƯỠNG HỌC SINH GIỎI HÌNH HỌC 9
170
22
22
ABQ ACQ ABC QBC
AB AC AB AC
S S S S
Đẳng thc xy ra khi và ch khi
QI QJ
.
Mt khác nếu gọi G là điểm chính gia ca cung nh BC thì luôn có
QBC GBC
SS
, do đó
2
2
ABC GBC
AB AC
P
SS
Vy
AB AC
P
QJ QI
nh nht khi và ch khi Q là điểm chính gia ca cung nh BC.
Bài 57. Cho tam giác ABC nhn, có trc tâm H và ni tiếp đường tròn tâm O. Gi D, E, F tương
ứng là các chân đường cao ca tam giác ABC k t A, B, C. Gọi M là giao điểm ca tia AO và cnh
BC. Gọi N, P tương ứng là hình chiếu vuông góc ca M trên các cnh CA, AB.
1) Chng minh:
..HE MN HF MP
.
2) Chng minh t giác FENP ni tiếp.
3) Chng minh rng:
2
.
.
BD BM AB
CD CM AC
(Trích đề thi vào 10 Chuyên Toán, Vĩnh Phúc, năm học 2015 - 2016)
NG DN GII
1) Ta có
180FHE PMN A
,
FEH FAH MAN NPM
(do t giác HFAE, PMNA ni tiếp).
Do đó
PMN EHF
..HE MN HF MP
2) T phn 1) thì
90FEN FEH
90NPM BPN
Nên t giác FENP ni tiếp.
3) Ta có
BAD CAM BAM DAC
Suy ra
sin . . .
sin . . .
BAD
CAM
S BD BAD AB AD AB AD
S CM CAM AC AM AC AM
sin . . .
sin . . .
BMA
CAD
S BM BAM AB AM AB AM
S CD CAD AC AD AC AD
Do đó
2
.
.
BD BM AB
CD CM AC
CHUYÊN ĐỀ BỒI DƯỠNG HỌC SINH GIỎI HÌNH HỌC 9
171
Bài 58. Cho đường tròn
;OR
và dây BC c định không đi qua tâm. Trên tia đối ca tia BC ly
điểm A (A khác B). T A k hai tiếp tuyến AM và AN với đường tròn
O
(M và N là các tiếp
điểm). Gọi I là trung điểm ca BC.
1) Chng minh A, O, M, N, I cùng thuc một đường tròn và IA là tia phân giác ca góc
MIN
.
2) Gọi K là giao điểm ca MN và BC. Chng minh
2 1 1
AK AB AC
.
3) Đường thng qua M và vuông góc với đường thng ON ct
O
tại điểm th hai là P. Xác định
v trí của điểm A trên tia đối của tia BC để AMPN là hình binh hành.
(Chuyên Nguyn Trãi, Hải Dương, năm học 2015 - 2016)
NG DN GII
1) Theo gi thiết
90AMO ANO AIO
5 điểm A, O, M, N, I thuộc đường tròn đường kính AO
,AIN AMN AIM ANM
(Góc ni tiếp cùng chn mt cung)
AM AN AMN
cân ti
A AMN ANM
AIN AIM
pcm)
2)
2 1 1
2.AB AC AK AB AC
AK AB AC
..AB AC AK AI
(Do
2AB AC AI
)
ABN
đồng dng vi
2
.ANC AB AC AN
AHK
đồng dng vi
..AIO AK AI AH AO
Tam giác
AMO
vuông tại M có đường cao
2
.MH AH AO AM
2
.AK AI AM
. Do
..AN AM AB AC AK AI
3) Ta có
, , / /AN NO MP NO M AN AN MP
Do đó AMPN là hình bình hành
2AN MP x
CHUYÊN ĐỀ BỒI DƯỠNG HỌC SINH GIỎI HÌNH HỌC 9
172
Tam giác ANO đồng dng vi
2
2AN NO x
NEM NE
NE EM R
TH1.
2
2 2 2 2 2 2
2
2
x
NE NO OE R R x x R R R x
R
Đặt
2 2 2 2 2
,0R x t t x R t
PTTT
2 2 2 2 2
2
2 2 0
tR
R t R Rt t Rt R
tR
Do
22
00t t R R x R x A B
(Loi)
TH2.
2
2 2 2 2 2 2
2
2
x
NE NO OE R R x x R R R x
R
Đặt
2 2 2 2 2
,0R x t t x R t
PTTT
2 2 2 2 2
2
2 2 0
tR
R t R Rt t Rt R
tR
Do
22
3
0 2 2 2
2
R
t t R R x R x AO R
Vy A thuc BC, cách O một đoạn bng 2R thì AMPN là hình bình hành.
Bài 59. Cho đường tròn tâm O đường kính BC, A là điểm di chuyển trên đường tròn
O
(A khác B
và C). K AH vuông góc vi BC ti H. M là điểm đối xng của điểm A qua điểm B.
1) Chứng minh điểm M luôn nm trên một đường tròn c định.
2) Đường thng MH ct
O
ti E và F (E nm gia M và F). Gọi I là trung điểm của HC, đường
thng AI ct
O
ti G (G khác A). Chng minh:
2 2 2 2 2
2AF FG GE EA BC
.
3) Gi P là hình chiếu vuông góc ca H lên AB. Tìm v trí của điểm A sao cho bán kính đường tròn
ngoi tiếp tam giác BCP đạt giá tr ln nht.
(Trích đề thi vào 10 Chuyên Toán, Nguyn Trãi, Hi Dương, năm học 2016-2017)
NG DN GII
CHUYÊN ĐỀ BỒI DƯỠNG HỌC SINH GIỎI HÌNH HỌC 9
173
1) Lấy K là điểm đối xng ca O qua B, vì B và O c định nên K c định. T giác OAKM là hình
bình hành nên
KM OA
.
Do
2
BC
OA
không đổi.
M nằm trên đường tròn tâm K, bán kính
2
BC
.
2) Xét
AHB
CHA
90 ,BHC BHA BAH ACB
(cùng ph vi
ABC
)
AHB CHA
.
Gọi S là trung điểm của AH, I là trung điểm ca HC nên
ABS CAI ABS CAI
Ta lại có BS là đường trung bình ca
AMH
//BS MH ABS AMH AMH CAI
90 90CAI MAI AMH MAI AI MF
Xét t giác AEGF ni tiếp
O
, có
AG EF
K đường kính AD, do
GD AG
EF AG
nên
//EF GD
, do đó tứ giác ni tiếp EFGD là
hình thang cân
2 2 2 2 2 2
FG ED AE FG AE ED AD BC
Tương tự ta chứng minh được:
2 2 2
AF EG BC
Vy
2 2 2 2 2
2AE FG AF EG BC
.
3) Gi Q là hình chiếu ca H trên
AC
T giác APHQ là hình ch nht (S là tâm)
AQP AHP ABC
nên t giác BPQC ni tiếp.
Đưng trung trc của các đoạn thng PQ, BC, QC ct nhau ti O’ thì O’ là tâm đường tròn ngoi
tiếp tam giác BCP.
Có OO’ // AH vì cùng vuông góc với BC.
OA PQ
O S PQ
//O S OA
nên t giác ASO’O là hình hình hành
2
AH
OO AS
Trong trường hp A nm chính gia cung BC thì ta vn có:
CHUYÊN ĐỀ BỒI DƯỠNG HỌC SINH GIỎI HÌNH HỌC 9
174
2
AH
OO AS
Tam giác OO’C vuông tại O nên
2
2
4
AH
O C OC
Do OC không đổi nên O’C lớn nht khi AH ln nht
A chính gia cung BC.
Bài 60. Cho đường tròn tâm O, bán kính R. T một điểm M ngoài đường tròn k hai tiếp tuyến
MA và MB với đường tròn (A, B là các tiếp điểm). Qua A k đường thng song song vi MO ct
đường tròn tại E (E khác A), đường thng ME cắt đường tròn tại F (F khác E), đường thng AF ct
MO tại N, H là giao điểm ca MO và AB.
1) Chng minh: T giác MAOB ni tiếp đường tròn.
2) Chng minh:
2
.MN NF NA
MN NH
.
3) Chng minh:
2
2
1
HB EF
HF MF
.
(Trích đề thi vào 10, Chuyên Nguyn Trãi, Hải Dương (Vòng 1), năm học 2017 - 2018)
NG DN GII
1) Ta có
90 , 90MAO MBO
(theo tính cht ca tiếp tuyến và bán kính)
Suy ra:
180MAO MBO
Vy t giác MAOB ni tiếp đường tròn.
2) Ta có
//AE MO AEM EMN
,
AEM MAF
, suy ra
EMN MAF
NMF
NAM
MNA
chung;
EMN MAF
nên
NMF
đồng dng vi
NAM
2
.
NM NA
NM NF NA
NF NM
(1)
Mt khác có:
ABF AEF ABF EMN
hay
HBF FMH
MFHB là t giác ni tiếp
FHM FBM FAB
hay
FHN NAH
.
Xét
NHF
NAH
có:
ANH
chung;
NHF NAH NHF
đồng dng
NAH
2
.
NH NA
NH NF NA
NF NH
(2)
CHUYÊN ĐỀ BỒI DƯỠNG HỌC SINH GIỎI HÌNH HỌC 9
175
T (1) và (2) ta có
NH HM
.
3) Xét
MAF
MEA
có:
AME
chung,
MAF MEA
suy ra
MAF
đồng dng vi
MEA
2
2
ME MA AE ME AE
MA MF AF MF AF
(3)
Vì MFHB là t giác ni tiếp
90 90MFB MHB BFE
90AFH AHN AFE BFH
.
AEF
HBF
có:
;EFA BFH FEA FBA
suy ra
AEF
đồng dng vi
HBF
22
22
AE HB AE HB
A F HF AF HF
(4)
T (3) và (4) ta có:
22
22
ME HB MF FE HB
MF HF MF HF
22
22
11
FE HB HB FE
MF HF HF MF
Bài 61. Cho hai đường tròn
;OR
;OR
ct nhau tại hai điểm phân biệt A và B. Trên tia đối
ca tia AB lấy điểm C, k tiếp tuyến CD, CE vi
O
, trong đó D, E là các tiếp điểm và E nm
trong
O
. Đường thng AD, AE ct
O
lần lượt ti M và N (M, N khác A). Tia DE ct MN ti I,
OO
ct AB và DI lần lượt ti H và F.
1) Chng minh:
..FE HD FD HE
.
2) Chng minh:
. . . .MB EB DI IB AN BD
.
3) Chng minh:
OI
vuông góc vi MN.
(Trích đề thi vào 10, Chuyên Nguyn Trãi (vòng 2), Hải Dương 2017 - 2018)
NG DN GII
1)
O
ct
O
ti A, B
90OO AB CHO
(1)
CD, CE là tiếp tuyến ca
O
ti D, E
90CDO CEO
(2)
T (1) và (2)
, , , ,C D O H E
cùng thuộc đường tròn đường kính
CHE CDE
CO
CHD CED
CD CE CDE CED CHE CHD
CHUYÊN ĐỀ BỒI DƯỠNG HỌC SINH GIỎI HÌNH HỌC 9
176
HC
là đường phân giác ca
DHE
.
Mt khác
OO AB
ti H hay
FH HC
ti H
HF
là phân giác ngoài ti H ca
DHE
..
FE HE
FE HD FD HE
FD HD
2) Trong
O
có:
BMN BAN
Trong
O
có:
BAN BDE BMN BDE
BDMI là t giác ni tiếp
MBI MDI ABE
Xét
MIB
AEB
có:
;MBI ABE BMI BAE
MB IB
MIB AEB
AB EB
(3)
Xét
ABN
DBI
có:
;BAN BDI BNA BID
AB AN
ABN DBI
DB DI
(4)
T (3) và (4)
. . . . . .
MB AB IB AN
MB EB DI IB AN DB
AB DB EB DI
3) Xét
IBN
DBA
có:
IBN DBA
(vì
DEA IEN
),
BIN BDA
(vì BDMI ni tiếp)
IN DA
IBN DBA
IB DB
(5)
Xét
CDA
CBD
có:
DCB
chung;
CDA CBD
DA CD
CDA CBD
DB CB
DA CE
CD CE
DB CB
(6)
Xét
CEA
CBE
có:
BCE
chung;
CEA CBE
CE EA
CEA CBE
CB EB
(7)
Mt khác
MIB AEB
(theo phn b)
EA IM
EB IB
(8)
T (5), (6), (7), (8)
IN IM
IN IM O I MN
IB IB
CHUYÊN ĐỀ BỒI DƯỠNG HỌC SINH GIỎI HÌNH HỌC 9
177
Bài 62. Cho t giác ABCD có
60 , 90BAD BCD
. Đường phân giác trong ca BAD ct BD ti
E. Đường phân giác trong ca BCD ct BD ti F. Chng minh:
3 2 1 1 1 1
AE CF AB BC CD DA
(Trích đề thi vào 10 Chuyên Long An, năm học 2016 - 2017)
NG DN GII
Gi K là hình chiếu vuông góc ca E lên AB.
Din tích tam giác ABE là:
. .sin 30 . .
2 2 4
KE AB AE AB AE AB
Din tích tam giác ADE là:
.
4
AE AD
Din tích tam giác ABD là:
.sin60 . 3 .
24
AB AD AB AD
Ta có: Din tích tam giác ABE + Din tích tam giác ADE
= Din tích tam giác ABD.
Suy ra:
3 1 1
AE AB AD
(1)
Tương tự như trên ta tìm được
3 1 1
CF CB CD
(2)
T (1) và (2) ta có:
3 2 1 1 1 1
AE CF AB BC CD DA
Bài 63. Cho đường tròn
;OR
và điểm A c định trên
;OR
. Gọi M, N là các giao điểm ca hai
đường tròn
;OR
;AR
; H là điểm thay đổi trên cung nh
MN
của đường tròn
;AR
. Đường
thng qua H và vuông góc vi AH ct
;OR
ti B, C. K
,HI AB I AB HK AC K AC
.
1) Chng minh rng IK luôn vuông góc vi một đường thng c định và
2
.2AB AC R
.
2) Tìm giá tr ln nht của điện tích
AIK
khi H thay đổi.
(Trích đề thi HSG Lp 9, Tnh Phú Thọ, năm học 2016 - 2017)
NG DN GII
1) Ta có
90 ; 90AIH AKH
180AIH AKH
nên t giác AJHK ni tiếp.
K tiếp tuyến At của đường tròn
CHUYÊN ĐỀ BỒI DƯỠNG HỌC SINH GIỎI HÌNH HỌC 9
178
;OR
ti A.
Ta có:
90
90
ACB HAC
AHK HAC
ACB AHK
(1)
Ta li có:
AHK AIK
(do t giác AIHK ni tiếp) (2)
BAt ACB
(cùng bng
1
2
AB
) (3)
T (1), (2), (3) suy ra: .
//BAt AIK At IK
Mt khác
OA At IK OA
. Vy IK luôn vuông góc với đường thng c định OA.
Gọi J là giao điểm của AO và IK; A’ là điểm đối xng vi A qua O.
Ta có
90 ;ACH AA B AHC ABA ACH AA B
.
2
. 2 . 2
AC AH
AB AC R AH R
AA AB
2) Ta có
2
..
AK AH
AKH AHC AK AC AH
AH AC
= =
Gi
,SS
lần lượt là din tích các tam giác ABC và AIK.
Ta có
AI AK IK AJ
AIK ACB
AC AB BC AH
, suy ra:
22
1
.
.
2
.
1
.
.
2
AJ IK
S AJ IK AK AK AC
S AH BC AB AB AC
AH BC
42
2
2
1
44
.2
AH AH
R
AH R
Suy ra
2
11
. . . .2
4 8 8 8 4
R R R
S S AH BC BC R
Vy giá tr ln nht ca tam giác AJK bng
2
4
R
, đạt khi
HO
.
Bài 64. T điểm M nằm ngoài đường tròn tâm
O
.V hai tiếp tuyến MA, MB với đường tròn (A,
B là các tiếp điểm). Cát tuyến MPQ không đi qua O (P nằm gia M, Q). Gọi H là giao điểm ca
OM và AB.
1) Chng minh
HPO HQO
.
CHUYÊN ĐỀ BỒI DƯỠNG HỌC SINH GIỎI HÌNH HỌC 9
179
2) Tìm điểm E thuc cung ln AB sao cho tng
11
EA EB
có giá tr nh nht.
(Trích đề thi HSG Lp 9, Tnh Ngh an, năm học 2015 - 2016)
NG DN GII
1)
MPA
đồng dng
MAQ
(g.g),
suy ra
2
.MA MP MQ
(1)
MAO
vuông tại A, có đường cao
AH nên
2
.MA MH MO
(2)
T (1) và (2) suy ra
..P MQ MH MO
hay
MP MO
MH MQ
(*)
MPH
MOQ
có góc M chung, kết hp (*) ta suy ra
MPH
đồng dng
MOQ
(c.g.c). Suy
ra
MHP MQO
Do đó tứ giác PQOH là t giác ni tiếp
1
2
HPO HQO sdOH
(đpcm)
2) Trên tia đối ca tia EA lấy điểm F
sao cho
EB EF
hay
EBF
cân ti E, suy ra
1
2
BFA BEA
Đặt
AEB
khi đó
2
AFB
nên F di chuyn trên cung cha góc
2
dng trên BC.
Ta có:
1 1 4
EA EB EA EB
.
Như vậy
11
EA EB
nh nht khi
EA EB
ln nht
hay
EA EF
ln nht
AF
ln nht (**)
Gọi O’ là điểm chính gia ca cung ln AB
suy ra
O AB
cân tại O’ suy ra
O A O B
(3)
O EB
O EF
có EB = EF, O’E chung
FEO BEO
(cùng bù vi
BAO
)
O EB O EF
(c.g.c) suy ra
O B O F
(4)
CHUYÊN ĐỀ BỒI DƯỠNG HỌC SINH GIỎI HÌNH HỌC 9
180
T (3) và (4) suy ra O’ là tâm cung chứa góc
2
dựng trên đoạn thẳng BC (cung đó và cung lớn AB
cùng thuc mt na mt phng b AB).
Do đó AF lớn nhất khi nó là đường kính ca
O
khi
EO
(***).
T (**) và (***) suy ra E là điểm chính gia cung ln AB thì
11
EA EB
giá tr nh nht.
Bài 65. Trong các hình bình hành ngoi tiếp đường tròn
;Or
, hãy tìm hình bình hành có din tích
nh nht.
(Trích đề thi HSG Lp 9, Tnh Hải Dương, năm học 2011 - 2012)
NG DN GII
Theo bài ta suy ra các cnh ca hình hành là
tiếp tuyến của đường tròn
;Or
. Gi M, N, P, Q
lần lượt là tiếp điểm của đường tròn vi các
cạnh như hình vẽ.
; , ;CM CN AP AQ BM BQ PD DN
CM BM AP PD CN DN AQ BQ
22BC AB BC AB
K
AH BC
. Ta có
AB AH
, dấu “=” xảy ra khi
90ABC
Ta có:
, , / /OM BC OP AD AD BC
P, O, M thẳng hàng, do đó
2AH PM r
.
. 2 . 2 .2
ABCD
S AH BC r AB r r
2
4
ABCD
Sr
, dấu “=” xảy ra khi
90ABC
.
Vy trong các hình bình hành ngoi tiếp đường tròn
;Or
thì hình vuông có din tích nh nht và
bng
2
4r
.
Bài 66. Cho tam giác ABC, điểm M trong tam giác, các đường thng AM, BM, CM, lần lượt ct
các cnh BC, CA, AB ti P, R, Q. Kí hiu
ABC
S
là din tích tam giác ABC.
1) Chng minh rng:
. . . 4
ABC
MA BC MB CA MC AB S
2) Xác định v trí của M để din tích tam giác PQR ln nht.
(Trích đề thi HSG Lp 9, Tỉnh Bình Định, năm học 2010 - 2011)
NG DN GII
CHUYÊN ĐỀ BỒI DƯỠNG HỌC SINH GIỎI HÌNH HỌC 9
181
a) Ta có:
BMC
ABC
S
PM MI
PA AH S
2
.
..
BMC BMC
ABC BMC
SS
PM MI MI BC
S S MA MA MA BC MA BC
.2
ABC BMC
MA BC S S
Tương tự ta cũng có:
.2
ABC AMC
MB AC S S
;
.2
ABC AMB
MC AB S S
Cng theo vế ta s được điều cn chng minh.
b) Đặt
;;
PMQ QMR RMP PQR
S x S y S z S x y z
Ta có
..
;
.
PMQ
RMP
MCP PMB MCP PMB BMC
S
S
MR MQ z x MR MQ y
S MC S MB S S MC MB S
2
.
44
BMP PMC BMC BMC
BMC BMC
S S S S
zx
y S S
(1)
Tương tự ta cũng có:
(2); (3)
44
CMA
AMB
S
S
xy yz
zx
Cng theo vế các BĐT (1), (2) và (3) ta được:
4
ABC
S
xy yz zx
z x y
(4)
Mặt khác dùng BĐT Cô-si ta s chứng minh được
xy yz zx
x y z
z x y
nên t (4) suy ra:
44
ABC ABC
PQR
SS
x y z S
Đẳng thc xy ra khi:
;;
PMB PMC CMQ QMA AMR RMB
S S S S S S
x y z
M là trng tâm ca tam giác ABC.
Vy khi M là trng tâm ca tam giác ABC thì max
4
ABC
PQR
S
S
Bài 67. Cho tam giác đều ABC ni tiếp đường tròn tâm O bán kính R. M là một điểm di động trên
cung nh BC của đường tròn đó.
1) Chng minh
MB MC MA
2) Gi H, I, K lần lượt là chân đường vuông góc h t M xung AB, BC, CA. Gi S, S' lần lượt là
din tích ca tam giác ABC, MBC. Chng minh rằng: Khi M di động ta luôn có đẳng thc:
CHUYÊN ĐỀ BỒI DƯỠNG HỌC SINH GIỎI HÌNH HỌC 9
182
2 3 2
3
SS
MH MI MK
R
(Trích đề thi HSG Lp 9, Tỉnh Bình Định, năm học 2016 - 2017)
NG DN GII
a) Cách 1: Trên tia đối ca tia MC lấy điểm E sao cho
ME MB
.
Ta có:
BEM
là tam giác đều
BE BM EM
BMA BEC MA EC
Do đó:
MB MC MA
Cách 2: Trên AM lấy điểm E sao cho
ME MB
Ta có:
BEM
là tam giác đều
BE BM EM
(c.g.c) MC AEMBC EBA
. Do đó:
MB MC MA
b) K AN vuông góc vi BC ti N
ABC
là tam giác đều nên O là
trng tâm ca tam giác
,,A O N
thng hàng
3
2
AN R
Ta có:
.sinAN AB ABN
33
:3
22
sin
AN
AB R R
ABN
Ta có:
1
.
2
ABM
MH AB S
22
3
ABM ABM
SS
MH
AB
R
22
1
.
2
3
ACM ACM
ACM
SS
MK AC S MK
AC
R
CHUYÊN ĐỀ BỒI DƯỠNG HỌC SINH GIỎI HÌNH HỌC 9
183
22
12
.
2
33
BCM BCM
BCM
SS
S
MI BC S MI
BC
RR
Do đó:
22
33
ABM ACM
S
MH MK MI S S
RR
22
.
33
ABMC
S
S
RR
2 3 2
22
.
3
33
SS
S
SS
R
RR
Bài 68. Cho tam giác ABC có bán kính đường tròn ni tiếp là r. Gi M, N, P lần lượt là trung điểm
ca các cnh BC, CA, AB . Biết rng:
1 1 1 1
AM BN CP r
Chứng minh tam giác ABC là tam giác đều.
(Trích đề thi HSG Lp 9, Tnh Thái Bình, năm học 2015 - 2016)
NG DN GII
Gọi đường cao tương ứng vi cnh BC là AH. Gi S là din tích tam giác ABC.
Ta d thy:
2
S S BC
AM AH
Tương tự ta có:
,
22
S AC S AB
BN CP
Cng vế theo vế ta được:
S S S
p
AM BN CP
, trong đó p là na chu vi.
1 1 1 1p
AM BN CP S r
Đẳng thc xy ra khi
ABC
đều.
CHUYÊN ĐỀ BỒI DƯỠNG HỌC SINH GIỎI HÌNH HỌC 9
1 | THCS.TOANMATH.com
CHUYÊN ĐỀ 7.NHỮNG ĐỊNH LÝ HÌNH HC NI TING
I. ĐỊNH LÝ THALES
1. Định lý Thales thun:
Nếu có một đường thng song song vi mt cnh ca tam giác và ct hai cnh còn lại thìđịnh ra
trên hai cạnh đó những đoạn thẳng tương ứng t l.
2. Định lý Thales đảo:
Nếu một đường thng ct 2 cnh của tam giác định ra trên 2 cạnh đó những đoạn thẳng tương
ng t l thì đường thẳng đó song sng song với cnh còn li ca tam giác.
(Chú ý nếu đường thng song song vi mt cnh ct hai cnh còn li phn kéo dài thì định lý vn
đúng)
3. H qu:
Nếu một đường thng ct 2 cnh ca tam giác và song song vi cnh th 3 thì nó to thành mt tam
giác mi có 3 cạnh tương ứng t l vi ba cạnh tam giác đã cho.
AM AN
MB NC
=
AM AN MN
AB AC BC
==
Chú ý: Định lý Thales cũng đúng cho hình thang:
/ / / /AB EF CD
thì
,
AE BF AE BF
AD BC ED FC
==
vi
,.E AD F BC
4. Mt s kết quả, định lý quan trng:
a, Cho tam giác
ABC
O
trung điểm
BC
.
Một đường thng bt k ct cnh
,,AB AO AC
ti
,,M N P
khi đó ta có:
2
.
AB AC AO
AM AP AN
+=
Chng minh:
Dựng các đường thng qua
,BC
song song vi
MP
ct
AO
lần lượt ti
,.HK
Áp dụng định lý Thales ta có:
,
AB AH AC AK
AM AN AP AN
==
cộng hai đẳng thc ta có:
AB AC AH AK
AM AN AN
+
+=
. Để ý rng:
,
AH AO OH AK AO OK
AN AN AN AN
−+
==
. Mt khác
BOH COK =
(g.c.g) suy ra
OH OK=
nên
2AB AC AH AK AO OK AO OK AO
AM AN AN AN AN
+ + +
+ = = =
đpcm.
CHUYÊN ĐỀ BỒI DƯỠNG HC SINH GII HÌNH HC 9
THCS.TOANMATH.com | 2
b, Định lý: Van-Aubel:
Đim
M
nm trong tam giác
ABC
, các đường
thng
,,AM BM CM
ct cạnh đối din ti
,,D E F
thì
=+.
AM AF AE
MD FB EC
Gii:
Dựng đường thng qua
A
song song vi
BC
ct
,BE CF
lần lượt ti
,HK
. Áp dụng định lý
Thales ta có:
,
AF AK AE AH
FB BC EC BC
==
cộng hai đẳng thc ta có:
AF AE KH MK MA
FB EC BC MC MD
+ = = =
đpcm.
CHUYÊN ĐỀ BỒI DƯỠNG HỌC SINH GIỎI HÌNH HỌC 9
3 | THCS.TOANMATH.com
II. ĐỊNH LÝ MENELAUS
1. Định lý thun
Cho tam giác
ABC
3 điểm
,,M N P
nằm trên đường thng cha 3 cnh
,,AB BC CA
. Khi đó
,,M N P
thng hàng khi và ch khi:
. . 1.
MA NB PC
MB NC PA
=
Chng minh
Dựng đường thng qua
A
song song vi
BC
cắt đường thng cha
,,M N P
ti
.Q
Áp
dụng định lý Thales ta có:
,
MA QA PC NC
MB BN PA QA
==
Suy ra
. . . . 1
MA NB PC QA NB NC
MB NC PA NB NC QA
==
đpcm.
2. Định lý Menelaus đảo
Nếu 3 điểm
,,M N P
nằm trên đường thng cha 3 cnh
,,AB BC CA
. . 1
MA NB PC
MB NC PA
=
thì
,,M N P
thng hàng.
Chng minh:
Gi s đường thng
NP
ct
AB
ti
M
. Theo định lý Thales thun ta có:
. . 1.
M A NB PC
M B NC PA
=
Kết hp vi
. . 1
MA NB PC
MB NC PA
=
ta suy ra
M A MA
M B MB
=
suy ra
MM
hay
,,M N P
thng hàng.
3. Định lý Ceva:
a, Định lý Ceva thun:
Ba đường thẳng đi qua 3 đỉnh ca tam giác
ABC
đồng quy hoc cùng song song vi nhau ct
các cnh
,,AB BC CA
(hay đường kéo dài) lần lượt ti
,,M N P
thì ta luôn có:
. . 1.
AM NB PC
MB NC PA
=
b, Định lý Ceva đảo:
Nếu
,,M N P
lần lượt nm trên các cnh
,,AB BC CA
(hay đường kéo dài) tha mãn:
=. . 1
AM NB PC
MB NC PA
thì các đường thng
,,AN BP CM
đồng quy hoc song song vi nhau.
CHUYÊN ĐỀ BỒI DƯỠNG HC SINH GII HÌNH HC 9
THCS.TOANMATH.com | 4
Chứng minh: Định lý Ceva:
Gi s đường thng qua
A
song song vi
BC
ct
,CM BP
lần lượt ti
,.QR
Theo định lý Thales ta
có:
,
MA QA PC CB
MB BC PA AR
==
. Suy ra
. . . . .
MA NB PC QA NB CB QA NB
MB NC PA BC NC AR NC AR
==
mặt khác ta cũng có:
QA IA AR
NC IN BN
==
nên
. . . 1
MA NB PC AR NB
MB NC PA BN AR
==
đpcm.
Trường hp:
/ / / / .AN BP CN
Ta có:
,
MA CN PC BC
MB BC PA BN
==
nên:
. . . . 1
MA NB PC CN NB BC
MB NC PA BC NC BN
==
đpcm.
Chứng minh: Định lý Ceva đảo:
Gi s
BP
ct
AN
ti
,I CI
ct
AB
ti
M
. Theo định lý Ceva thun:
. . 1
M A NB PC
M B NC PA
=
kết hp
vi gi thiết:
. . 1
MA NB PC
MB NC PA
=
ta suy ra
MA M A
MB M B
=
suy ra
.MM
Trường hp:
//AN BP
ta d dàng có được kết qu
/ / / / .AN BP CN
4. B đề hình thang
Cho hình thang
ABCD
hai đáy
,AB CD
khi đó trung điểm các cạnh đáy, giao điểm 2 đường
chéo và giao điểm ca 2 cnh bên nm trên một đường thng.
Chng minh:
CHUYÊN ĐỀ BỒI DƯỠNG HỌC SINH GIỎI HÌNH HỌC 9
5 | THCS.TOANMATH.com
Gi s các đường thng
,AD BC
ct nhau ti
,M
,AC BD
ct nhau ti
,P
đường thng
MP
ct
,AB CD
lần lượt ti
,.NQ
Ta chng minh:
,NQ
lần lượt là trung điểm ca
,.AB CD
Tht vy: Do
//AB CD
theo định lý Thales ta
có:
AN NB
QD QC
=
(1),
AN BN
QC QD
=
(2). Ly (1) nhân
vi (2) ta có:
22
..
AN NB
AN NB
QC QD QC QD
= =
thay vào (1) ta
.QD QC=
Hay
,NQ
lần lượt
là trung điểm ca
,.AB CD
5. Tính cht phân giác:
Cho tam giác
ABC
đường phân giác trong
góc
A
AD
phân giác ngoài góc
A
.AE
Khi đó ta có:
.
AB DB EB
AC DC EC
==
MT S BÀI TOÁN VN DNG
Ví d 1.
Cho tam giác
ABC
M
trung đim
,BC
điểm
N
nm trên cnh
AB
sao cho
1
3
AN AB=
,
điểm
Q
nm trên cnh
AC
sao cho
2
3
AQ AC=
, đường thng
QN
cắt đường thng
AM
BC
lần lượt tại điểm
,.PR
a, Tính
,.
RB PA
RC PM
b, Một đường thẳng thay đổi ct các cnh
,AB AC
ti
,EF
sao cho
4.
AB AC
AE AF
+=
Chng minh
đường thẳng này luôn đi qua điểm c định.
c, Đường thng qua
N
song song vi
BC
ct
AC
ti
T
. Chng minh:
,CN BT
ct nhau ti trung
điểm của đoạn thng
.AM
Gii:
CHUYÊN ĐỀ BỒI DƯỠNG HC SINH GII HÌNH HC 9
THCS.TOANMATH.com | 6
a, Để tính t s ta dựng đường thng qua
C
song
song vi
AB
cắt đường thng
NQ
ti
.H
Áp dng
định lý Thales ta có:
1 1 1 1 1
.
2 2 2 2 2 4
RC HC HC HC QC
RB NB NA NA QA
= = = = = =
Cách khác: Áp dụng định lý Menelaus cho tam giác
ABC
và cát tuyến
NQR
ta có:
1 1 1
. . 1 . . 1 .
2 2 4
NA RB QC RB RC
NB RC QA RC RB
= = =
b, Gi s đường thng
EF
cắt đoạn
AM
.I
Áp dng kết qu 4a) ta có:
2
.
AB AC AM
AE AF AI
+=
T
đó suy ra
2
4 2 ,
AM
AM AI
AI
= =
hay
I
là trung điểm ca
.AM
Vy đường thng
EF
luôn đi qua
một điểm c định là trung điểm
I
ca
.AM
c, Theo định lý Thales ta có:
1
.
2
AN AT
AB AC
==
Gi s
,CN BT
ct nhau tại điểm
I
AI
ct
BC
ti
M
. Theo định Van-Aubel ta có:
11
1
22
AI AN AT
I
I M NB TB
= + = + =

trung điểm ca
AM
.
Áp dụng định lý Ceva cho 3 đường thẳng đồng quy
,,AM CN BT
ta có:
1
. . 1 . .2 1 1
2
AN M B TC M B M B
M
NB M C TA M C M C
= = =
trung điểm ca
BC
. Như vậy ta có:
,M M I I


. Hay
,,BT CN AM
đồng quy ti
.I
Ví d 2.
Cho tam giác nhn
ABC
45A =
, các đường cao
BD
CE
ct nhau
.H
Đưng vuông góc
vi
AB
ti
B
, ct
AC
.I
Đưng vuông góc vi
AC
ti
,C
ct
AB
.K
Gi
F
giao ca
BI
CK
,
G
giao điểm ca
FH
EI
. Chng minh rng
G
trng tâm
ca tam giác
.AIK
Gii:
CHUYÊN ĐỀ BỒI DƯỠNG HỌC SINH GIỎI HÌNH HỌC 9
7 | THCS.TOANMATH.com
Tam giác vuông
ACK
45A =
nên tam giác
vuông cân,
CE
đường cao nên
,AE EK IE=
đường trung tuyến ca
AIK
. Ta s chng minh
2IG GE=
(bng cách chng minh
2FI EH=
).
Ta
2FI CF=
(vì
CIF
vuông n),
CF BH=
(vì
BFCH
là hình bình hành),
2BH EH=
(vì
BEH
vuông cân) nên
2FI EH=
.
Do
//EH FI
nên theo định lí Thales, ta có
2
IG FI
GE EH
==
, suy ra
2.IG GE=
Vy
G
là trng tâm tam giác
AIK
.
Ví d 3.
Cho tam giác
ABC
( )
AB AC
, đường phân giác
AD
, đường trung tuyến
.AM
Trên cnh
AC
ly
điểm
E
sao cho
AE AB=
. Gi
,OG
theo th t là giao điểm ca
BE
vi
,.AD AM
a, Chng minh rng
/ / .DG AB
b, Gi
I
là giao điểm ca
MO
DG
. Chng minh rng
.DI IG=
Gii:
a, Tam giác
ABE
cân ti
A
,
AO
đường phân giác nên
. Tam
giác
BEC
OM
đường trung bình
nên
/ / .OM EC
Tam giác
ABC
BM MC=
//OM EC
nên
MO
đi qua trung điểm
N
ca
.AB
Qua
M
k đưng thng song song vi
AB
, ct
BO
AO
theo th t
H
K
.
Ta có
//HK AB
AN NB=
nên
=MK MH
(theo b đề hình thang).
Do
//HK AB
, theo định lý Thales ta
MD MK MH
DB AB AB
==
(vì
MK MH=
)
MG
GA
=
T đó ta
//
MD MG
DG AB
DB GA
=
ịnh lí Thales đảo).
b, Ta có
//DG AB
AN NB=
nên
DI IG=
(theo b đề hình thang).
Ví d 4.
CHUYÊN ĐỀ BỒI DƯỠNG HC SINH GII HÌNH HC 9
THCS.TOANMATH.com | 8
Cho tam giác
ABC
vuông ti
A
, đường cao
AH
. Gi
I
là trung điểm ca
AH
. Đường vuông góc
vi
BC
ti
C
cắt đường thng
BI
ti
D
. Chng minh rng
.DA DC=
Gii:
Gi
M
là trung điểm ca
AC
,
N
là giao điểm ca
MI
.AB
Tam giác
AHC
MI
là đường trung bình nên
//MI HC
, tc là
/ / .MN BC
Theo định lí Thales:
Do
//AH CD
nên
.
IB HB
ID HC
=
(1)
Do
//MN BC
nên
,
IN AI IM
HB AH HC
==
tc là
.
IN HB
IM HC
=
(2)
T (1) và (2) suy ra
,
IB IN
ID IM
=
do đó
//BN DM
nh Thales
đảo).
Ta li có
BN AC
nên
DM AC
. Vy
DM
là đường trung trc ca
AC
, suy ra
.DA DC=
Ví d 5.
Cho hình bình hành
ABCD
giao 2 đường chéo là
O
, một đường thẳng đi qua đỉnh
D
ct cnh
AB
ti
M
, ct
BC
N
cắt đường chéo
AC
ti
I
, đường thng
NO
ct
CD
ti
F
. Dng
//BE AC
, đường thng
AE
ct
BC
ti
.P
a, Chng minh:
.AM CN
không đổi.
b, Chng minh:
2
..ID IM IN=
c, Chng minh:
.
EM DM
EN DN
=
d, Chng minh:
/ / .OP DN
e, Chng minh:
=+
1 1 1
.
DI DM DN
Gii:
a, Do
//AM CD
nên ta có:
CHUYÊN ĐỀ BỒI DƯỠNG HỌC SINH GIỎI HÌNH HỌC 9
9 | THCS.TOANMATH.com
. . .
AM AI AD
AM CN AD CD
CD IC CN
= = =
b, Do
//AM CD
nên:
2
..
IM IA ID
ID IM IN
ID IC IN
= = =
c, Do
//BE AC
nên
,
MI MA MI ME MA MB EI AB CD CN IN
ME MB ME MB ME MB MB BN EN
++
= = = = = =
hay
EI IN ME EI CB DM
ME EN NE NI NC DN
= = = =
đpcm.
d, T giác
ACBE
hình thang, theo b đề hình thang
OP
đi qua trung điểm ca
EB
nên
/ / .OP DN
e, Ta cn chng minh:
1.
DI DI
DM DN
+=
Ta có:
,
DI CI DI AI
DM CA DN AC
==
suy ra
+ = + = =1.
DI DI CI AI AC
DM DN AC AC AC
Ví d 6.
Cho tam giác
ABC
trọng tâm điểm
G
. Gi
P
điểm nm trên cnh
.BC
Các đường thng
qua
P
song song vi
,CG BG
ct
,AB AC
ti
,.EF
Gi s
EF
ct
,BG CG
ti
,.IJ
a, Chng minh:
1
.
3
EI EF=
b, Chng minh:
.IE IJ=
c, Chng minh:
PG
đi qua trung điểm ca
.EF
Gii:
a, Gi
,BM CN
các đường trung tuyến ca tam giác
ABC
,
BG
ct
EP
ti
R
,
CG
ct
FP
ti
.S
//PF RI
nên
EI ER
EF EP
=
(1),
//PE NC
nên
ER NG
EP NC
=
(2). T (1) (2) ta suy ra
11
.
33
EI NG
EI EF
EF NC
= = =
b, Chứng minh tương tự ta có:
S 1 1
33
JF F MG
JF EF
EF FP MB
= = = =
t đó suy ra:
.EI IJ JF==
c, T chng minh trên ta có:
S
/ / S
ER F
EF R
EP FP
=
.
RGSP
hình bình hành nên
GP
đi qua
trung điểm ca
RS
nên
GP
ng đi qua trung điểm ca
.IJ
CHUYÊN ĐỀ BỒI DƯỠNG HC SINH GII HÌNH HC 9
THCS.TOANMATH.com | 10
Chú ý: S dng b đề hình thang ta cũng suy ra
GP
đi qua trung điểm ca
.IJ
Ví d 7.
Cho tam giác nhn
ABC
và một điểm
D
bt k thuc cnh
BC
, ly một điểm
E
thuộc đoạn thng
,AD F
thuộc đoạn thng
DE
. Một đường thng qua
F
song song vi cnh
BC
ct
, , ,AB EB EC AC
theo th t ti
, , ,M P Q N
, đường thng
,MD EB
ct nhau ti
,R ND
EC
ct
nhau ti
,S DP
AB
ct nhau ti
G
,
DQ
ct
AC
ti
.H
a, Chng minh:
.
MP NQ
BD DC
=
b, Chng minh:
S/ / .R BC
c, Chng minh:
/ / S.GH R
Gi
K
chân đường cao h t
A
lên cnh
BC
,
I
một đim nm trên
AK
, các đường thng
,BI CI
ct
,AC AB
lần lượt ti
,XY
. Đường thng qua
//I BC
ct
,KX KY
ti
,.ZT
Chng minh:
KZT
cân.
Gii:
a, Do
//MF BD
theo định lý Thales
ta có:
, / / ,
MF AF NF AF
NF CD
BD FD CD FD
= =
suy ra
MF NF
BD CD
=
(1), do
//
PF EF
PF BD
BD CD
=
,
Do
/ / D suy ra (2)
D
= =
QF EF PF QF
FQ C
C FD BD CD
T (1), (2) ta có:
.
MF PF NF QF MP NQ
BD BD CD CD BD CD
= =
b, Ta có
/ / , / / / / .
MP MR NQ NS MR NS
MP BD NQ CD RS MN
BD RD CD SD RD SD
= = =
Do
/ / / / .MN BC RS BC
c, Do
/ / , / /
MP GP QN HQ
MP BD FQ CD
BD GD CD HD
= =
theo câu a thì
MP NQ
BD CD
=
nên
//
GP HQ DP DQ
GH PQ
GD HD DG DH
= =
hay
/ / / .GH BC RS
d, Gi s đường thng qua
I
song song vi
BC
ct
,AB AC
ti
,.LU
CHUYÊN ĐỀ BỒI DƯỠNG HỌC SINH GIỎI HÌNH HỌC 9
11 | THCS.TOANMATH.com
Do
//IZ KC
nên
, / /
IZ XZ
IL BC
KC XK
=
nên
IL XZ
BC XK
=
suy ra
IZ IL IZ KC
KC BC IL BC
= =
(1)
Hay
= .
KC
IZ IL
BC
, tương tự ta có:
.
BK
IT IU
BC
=
(2). T (1),(2) ta có:
.
IZ KC IL
IT BK IU
=
nhưng
KC KB
IU IL
=
nên
.1
IZ KC IL
IT BK IU
==
hay
IZ IT=
. Tam giác
KZT
có đường cao
IK
đồng thời cũng là đường trung tuyến nên
KZT
cân ti
.K
Chú ý rng: Với điểm
K
tùy ý trên
BC
ta luôn có:
.IZ IT=
Ví d 8.
Cho tam giác vuông ti
ABC
. K đường phân giác
AD
,
D BC
h
,,DH AB DK AC BK⊥⊥
ct
DH
ti
M
,CH
ct
DK
ti
.N
a, Chng minh:
.
HM BH
MD HA
=
b, Chng minh:
/ / .MN BC
c, Gi
I
là giao điểm ca
BK
CH
. Chng minh:
ABK KAN
.AI BC
Gii:
a,
//MH AC
//BH DK
theo định
Thales ta có:
BH BM MH
HA MK MD
==
(đpcm).
b, Để chng minh:
//MN BC
ta chng minh:
.
MH NH
MD NC
=
Ta d chứng minh được t giác
AHDK
hình vuông nên
.HD AK HA DK= = =
Do
/ / , / /HM AK DN BH
. Theo định Thales
ta có:
/ / .
HM MH BM BD HN MH HN
MN BC
HD AK BK BC HC HD HC
= = = = =
c, Xét tam giác vuông
,AKN ABK
ta có:
KN KN NC CD DK AK KN AK
KA AH CH CB AB AB KA AB
= = = = = =
hay
ABK KAN
(c.g.c) suy ra
=ABK KAN
nên
90AKB KAN AKB ABK+ = + =
hay
.AN BK
Chứng minh tương tự ta có:
AM CH
suy ra
,BK CH
chứa các đường cao xut phát t
,MN
ca
CHUYÊN ĐỀ BỒI DƯỠNG HC SINH GII HÌNH HC 9
THCS.TOANMATH.com | 12
tam giác
AMN
nên
CH
ct
BK
ti
I
thì
I
trc tâm ca tam giác
AMN
suy ra
.AI MN AI BC
Ví d 9.
Cho tam giác
ABC
vuông ti
A
.
D
điểm trên tia đối ca tia
BC
, k tiếp tuyến
DE
với đường
tròn tâm
C
bán kính
CA
(
,AE
khác phía nhau so vi
BC
). Đường thng qua
A
vuông góc vi
BC
cắt đường thng
CE
ti
,F
đường thng
BF
ct
DE
ti
M
, qua
B
k đường thng song
song vi
CM
ct
DE
ti
N
. Gi
J
là giao điểm th 2 của đường tròn
( )
;C CA
vi
.EC
a, Đường tròn đường kính
DC
ct
AC
ti
.I
Chng minh:
AIFE
ni tiếp.
b, Tam giác
CIF
cân ti
.C
c, Chng minh:
M
là trung điểm ca
.NE
Gii:
a, Đường tròn đường kính
DC
ct
AC
ti
I
nên
90DIC DECI=
ni tiếp. Suy ra
90EIC EDC ECD CFA AIE AFE= = = =
Hay t giác
AIFE
ni tiếp.
b, Do
CA CE=
,CEA CIF CAE CFI AIF AFI= = =
suy ra tam
giác
CIF
cân ti
C
/ / .AE IF
c, Áp dụng định lý Menelaus trong tam giác
BFC
vi cát tuyến
DME
ta có:
. . 1
MB EF DC
MF EC DB
=
(*)
Mt khác do
//AE IF
nên ta
=
EF AI
EC AC
, do
//AB DI
nên
=
DC IC
DB IA
thay vào (*)
ta có:
. . 1 / /
MB AI IC MB AC CJ
BJ CM
MF AC IA MF IC CF
= = =
hay
M
là trung điểm ca
.NE
Ví d 10.
Cho tam giác
ABC
ngoi tiếp đường tròn
( )
I
, đường thng
AI
cắt đường tròn ngoi tiếp tam giác
ABC
tại điểm
,DE
điểm trên cung
BDC
, điểm
F
trên cnh
BC
tha mãn
CHUYÊN ĐỀ BỒI DƯỠNG HỌC SINH GIỎI HÌNH HỌC 9
13 | THCS.TOANMATH.com
1
2
BAF CAE BAC=
. Gi
G
trung điểm ca
IF
. Đường thng
EI
cắt đường tròn ngoi tiếp
tam giác
ABC
ti
P
, đường thng
AI
ct
BC
ti
,J AF
cắt đường tròn ngoi tiếp tam giác
ABC
ti
K
ct
DP
ti
.Q
a, Chng minh:
APQI
là t giác ni tiếp.
b, Chng minh:
.DCJ DAC
c, Chng minh:
,DG EI
ct nhau ti một điểm nằm trên đường tròn ngoi tiếp tam giác
.ABC
Li gii:
a, Do
AD
phân giác trong góc
A
nên
BAD CAD=
,
BAK CDE KAD EAD= =
,
mt khác
EAD EPD EAD EPD= =
suy ra
t giác
PQIA
ni tiếp.
b, Xét tam giác
,DCJ DAC
ta có:
,DCJ DAC=
ADC
chung nên
DCJ DAC
(g.g)
c, Gi s
PD
ct
FI
ti
G
. Ta chng minh
.GG
Tht vy Áp dụng định lý Menelaus cho tam giác
AIF
và cát tuyến
QG D
ta có
. . 1
QA G F DI
QF G I DA
=
(*). T chng minh câu a ta có:
AQI API AKE==
suy ra
/ / .KE QI
Mt khác
cũng do
BAK CDE=
suy ra
,,BK CE KAD EAD DK DE BKEC= = =
hình thang cân
//EK BC
. Như vậy
/ / / /QI BC KE
Theo định Thales ta có:
,
QA IA
QF IJ
=
mt khác
I
tâm đường tròn ni tiếp tam giác
ABC
nên
theo tính chất phân giác trong ta cũng có:
IA CA
IJ CJ
=
, do
DCJ DAC
n
CA DA
CJ DC
=
DI DC DB==
(Tính cht quen thuc). Suy ra
CA DA
CJ DI
=
như vậy:
QA IA CA DA DA
QF IJ CJ DC DI
= = = =
suy ra
.1
QA DI
QF DA
=
thay vào (*) ta có:
1
GF
G
GI
=
là trung điểm ca
IF
hay
.GG
Ví d 11.
Cho tam giác
ABC
vuông ti
A
, đường tròn tâm
B
bán kính
BA
đường tròn tâm
C
bán kính
CA
ct nhau ti
D
khác
A
,
BC
ct
( )
B
ti
,EF
(
F
nm trong
( )
C
) ct
( )
C
ti
,MN
(
M
CHUYÊN ĐỀ BỒI DƯỠNG HC SINH GII HÌNH HC 9
THCS.TOANMATH.com | 14
nm trong
( )
B
). Đường thng
DM
ct
AE
ti
P
,
DF
ct
AN
ti
.Q
Kéo dài
DM
ct
( )
B
ti
,I DF
ct
( )
C
ti
.H
a, Chng minh:
.IB EF
b, Chng minh: T giác
APDQ
ni tiếp và
/ / .PQ EN
c, Chng minh:
..
IP HF AB
IM HQ AC
=
Gii:
a, Ta có:
(
)
1
45 .
2
AEN ANE ABF ACM+ = + =
Li có:
,AEF ADF ANM ADM==
Suy ra
45 ,IDH MDA FDA= + =
(
)
2 90IBF IBA ABF ADF ADM= + = + =
Do đó
.IB EF
b, T a suy ra tam giác
IBF
vuông cân ti
B
suy ra
45IAE =
suy ra
,,I A N
thng hàng.
Tương tự ta cũng có:
,,E A H
thng hàng suy ra
135EAN =
t giác
APDQ
ni tiếp suy ra
APQ ADQ AEF==
nên
/ / .PQ EF
Áp dụng định lý Menelaus vi tam giác
PEM
cát tuyến
IAN
ta có:
. . 1
IP NM AE
IM NE AP
=
, tương tự
vi tam giác
QFN
và cát tuyến
HAE
ta cũng có:
. . 1
HF AQ EN
HQ AN EF
=
. Nhân hai đẳng thc vi chú ý:
AP AE
AQ AN
=
suy ra:
..
IP HF EF
IM HQ MN
=
6. Đường thẳng Ơle
Trong mt tam giác: Trc tâm
H
, trng tâm
G
, tâm đường tròn ngoi tiếp
O
nm trên một đường
thng gọi là đường thẳng Ơle của tam giác. Đồng thi ta có:
3.HO GO=
CHUYÊN ĐỀ BỒI DƯỠNG HỌC SINH GIỎI HÌNH HỌC 9
15 | THCS.TOANMATH.com
Chng minh:
Dựng đường kính
AN
ca
( )
O
.
AN
đường kính
ca
( )
O
nên
NC AC
, do
/ / .BH AC BH NC⊥
Chứng minh tương tự ta cũng
//CH NB
nên t giác
BHCN
hình bình hành, suy ra 2 đường chéo
,NH BC
ct nhau tại trung điểm
M
ca mỗi đường
nên
,,N H M
thng hàng.
Ta
MO
đường trung nh ca tam giác
AHN
nên
1
//
2
MO AH=
. Gi
G
giao đim ca
AM
HO
, do
//MO AH
(cùng vuông góc vi
BC
). Theo định lý Thales ta có:
1
2
AG MO
G
GM AH
= =
trng tâm ca tam giác
ABC
,,H G O
thng hàng. Do
1
3.
2
GO OM
HO GO
GH AH
= = =
(Đường thng qua
,,H G O
gọi là đường thẳng Ơle của tam giác
ABC
).
7. Đường thng Simson Đưng thng Steiner
a, Đường thng Simson:
Cho tam giác
ABC
ni tiếp đường tròn
( )
O
M
một điểm bt k trên đường tròn. K
,,MH MI MK
lần lượt vuông góc vi
, , .AB BC AC
Chng minh rằng ba điểm
,,H I K
thng hàng.
Chng minh:
T giác
MIBH
90 90 180BHM BIM+ = + =
nên t giác ni tiếp
MIH MBH=
(cùng chn
cung
HM
), t giác
ABMC
ni tiếp nên
MBH KCM=
, do đó
.MIH KCM=
Mt khác t giác
KCMI
ni tiếp
(vì
90MIC MKC= =
) nên
180KCM MIK+ =
180 180 .MIH MIK HIK + = =
Vy
,,H I K
thng hàng.
Đưng thẳng đi qua
,,H I K
được gọi là đường thng Simson của điểm
.M
Chú ý: Ta bài toán đảo v bài toán Simson như sau: Cho tam giác
ABC
một điểm
M
nm
ngoài tam giác. Chng minh rng nếu hình chiếu ca
M
lên ba cnh ca tam giác
ABC
bao
điểm thng hàng thì
M
nằm trên đường tròn ngoi tiếp tam giác
.ABC
BÀI TP T LUYN
CHUYÊN ĐỀ BỒI DƯỠNG HC SINH GII HÌNH HC 9
THCS.TOANMATH.com | 16
1. Cho
ABC
nhn ni tiếp đường tròn
(O )
. H trc tâm
ABC
, M một điểm nm
trên cung nh AC. Gi E, F lần lượt hình chiếu vuông góc của M trên các đường thng
AB, BC.
a) Chng minh rằng EF đi qua trung điểm ca MH.
b) Xác định v trí của M để EF ln nht.
2. Cho hai đường tròn
(O )
(O')
ct nhau ti hai điểm phân bit A B. Mt cát tuyến
thay đổi qua A ct
(O )
(O')
tương ng ti C D. Gi E F th t hình chiếu
vuông góc ca B lên tiếp tuyến ca
(O )
ti C tiếp tuyến ca
(O')
ti D. Chng minh
rng EF tiếp xúc một đường tròn c định.
3. Cho ngũ giác ABCDE nội tiếp một đường tròn. Gi M, N, P, Q, R, S, T U lần lượt
hình chiếu vuông góc của E trên các đường thng AB, BC, CD, DA, MN, NP, PQ QM.
Chng minh R, S, T, U thng hàng.
4. Cho đường tròn
(O )
và một điểm A c định nm ngoài
(O )
. M một điểm thay đổi
trên đường thng qua A vuông góc vi AO. Gi MB, MC là các tiếp tuyến ca
(O )
(B, C
các tiếp điểm). K
AE MB, AF MC ( E MB; F MC )
. Chng minh rằng EF đi qua mt
đim c định.
5. Cho
xOy
, lấy điểm A c định thuc tia phân giác ca
xOy
. V đưng tròn
( I )
qua O
A ct Ox, Oy lần lượt ti B C v hình bình hành OBMC. Chng minh rng M thuc
một đường thng c định.
6. Cho tam giác nhn ABC ni tiếp đường tròn
(O )
trực tâm H. D điểm trên cung
nh BC. Lấy điểm E sao cho ADCE hình nh hành K trc tâm ca tam giác ACE.
Gi P, Q lần lượt hình chiếu ca K trên BC và AB. Chng minh rằng PQ đi qua trung
đim ca HK.
7. Cho
ABC
nhn ni tiếp đường tròn
(O )
. M điểm thay đổi trên cung nh BC. K
ME AB, MF AC (E AB,F AC )
a) Xác định v trí của M để trung điểm ca EF nằm trên đon thng BC
b) K
AP MB, AQ MC (P MB, Q MC )
. Chng minh rằng PQ đi qua một đim c
định.
CHUYÊN ĐỀ BỒI DƯỠNG HỌC SINH GIỎI HÌNH HỌC 9
17 | THCS.TOANMATH.com
8. Cho t giác ABCD ni tiếp đường tròn
(O )
. Gi H, K th t hình chiếu vuông góc
ca B trên AC, CD. Gi M, N th t trung điểm ca AD, HK. Chng minh rng tam
giác BMN là tam giác vuông.
NG DN GII - ĐÁP SỐ
1.
a) Xét trường hp
BAM BCM
(các trường hp khác chứng minh tương tự)
Khi đó E thuộc tia đối ca tia AB và F nm trên cnh BC
H
MI AC (I AC)⊥
Ta được EF đường thng Simson ng với điểm M ca
ABC
Hơn nữa, I nm gia E và F.
Gi P, Q theo th t điểm đối xng ca M qua các
đưng thng AB, AC
APB AMB ACB 180 AHB= = =
nên t giác AHBP ni
tiếp
AHP ABP ABMT = =
. Tương tự
CHQ CBM=
Suy ra
AHP AHC CHQ ABM AHC CBM 180+ + = + + =
Do đó P, H, Q thẳng hàng.
IE đường trung bình ca
MPQ
nên IE đi qua trung điểm của MH hay EF đi qua
trung điểm ca MH (đpcm).
Nhn xét. Có th dùng đường thẳng Steiner để chng minh.
b) Vì
MEI MAI=
MFI MCI=
nên
MEF MAC ( g.g )
Suy ra
EF EM
1
AC AM
=
Do đó EF lớn nht bng AC và xy ra khi và ch khi BM là đưng kính ca
(O)
2.
Đặt
T CE DF=
CHUYÊN ĐỀ BỒI DƯỠNG HC SINH GII HÌNH HC 9
THCS.TOANMATH.com | 18
Gi s
TCB TDB
H
BI CD ( I CD )⊥
CBD CBA DBA TCD TDC= + = +
nên
CBD CTD 180+ =
Do đó tứ giác TCBD ni tiếp.
Suy ra EF là đường thng Simson ng với điểm B ca
TCD
Do đó I, E, F thẳng hàng I thuộc đường tròn đường kính
AB.
Mà t giác BICE ni tiếp nên
BIE BCE BAE==
Vy EF tiếp xúc với đường tròn đường kính AB c định (đpcm).
3.
H
EI AC ( I AC )⊥
Đưng thng Simson ng vi điểm E vi tam giác ABC cho
ta ba điểm M, N, I thng hàng.
Đưng thng Simson ng vi điểm E vi tam giác ACD cho
ta các ba điểm P, Q, I thng hàng.
Ta A, I, Q, E, M cùng thuc đường tròn đường kính AE,
nên vn dụng đường thng Simson ng với đim E vi tam
giác IMQ cho ta các cặp điểm R, U, T thng hàng.
Tương tự R, S, T thẳng hàng, ta có đpcm.
4
Ta có
MAO MBO MCO 90= = =
Suy ra năm điểm M, A, C, O, B cùng thuộc đường tròn đường kính MO.
K
AI BC
. Xét đường tròn đường kính MO, ta có:
AE MB, AF MC, AI BC
Suy ra E, F, I thẳng hàng (đường thng Simson).
Gọi H, J là giao điểm ca BC vi AO và OM, K là giao điểm ca
EI và AO.
Ta có:
OJH OAM ( g.g )
suy ra:
OJ.OM OH.OA=
CHUYÊN ĐỀ BỒI DƯỠNG HỌC SINH GIỎI HÌNH HỌC 9
19 | THCS.TOANMATH.com
Ta có:
OJB OBM ( g.g )
suy ra:
2
OJ.OM OB=
Do đó :
2
2
OB
OH.OA OB OH
OA
= =
không đổi
Vậy H là điểm c định.
T giác AICF ni tiếp nên
KIA FCA=
(1)
A, M, B, O, C cùng thuc một đường tròn nên
ACM AOM=
(2)
T (1), (2) suy ra :
KIA AOM=
AOM KAI=
(so le trong) nên
KIA KAI KA KI= =
AIH
vuông có
KA KI=
d dàng suy ra
AK KH=
Suy ra K là điểm c định.
5.
A thuc tia phân giác ca
xOy
nên
AC AB=
. Suy ra
IA BC
K
AH Ox, AK Oy⊥⊥
thì K, H c định K, E, H thng
hàng nên điểm E thuc HK c định.
Mà hình bình hành OBMC có
OM 2OE=
nên M c định.
Suy ra M thuc đường thng d c định song song vi
đưng thng HK và cách HK mt khoảng không đổi.
6.
Theo gi thiết ta có ADCE là hình bình hành nên
ADC AEC=
K là trc tâm ca
AEC
nên
EK AC
Mt khác
AKC AEC 180+ =
nên
AKC ADC 180+ =
Suy ra t giác ADCK ni tiếp, t đó
K (O)
EK ct AC ti I
Do đó P, Q, I thẳng hàng (đường thng Simson).
Đưng thng AH cắt đường tròn
(O)
ti M ct PQ ti N t
MN KP, KQ AB, KP BC⊥⊥
Suy ra BQKP là t giác ni tiếp nên
QPK QBK AMK==
Do đó MPKN là tứ giác ni tiếp.
CHUYÊN ĐỀ BỒI DƯỠNG HC SINH GII HÌNH HC 9
THCS.TOANMATH.com | 20
Do đó MPKN là hình thang cân nên
PMN KNM=
Mt khác
PH PM PMN PHM= =
nên
PHM KNM=
PH KN
, li có
NH KP
, suy ra HPKN là hình bình hành.
Do đó NP cắt HK tại trung điểm ca mỗi đường.
Do đó PQ đi qua trung điểm ca HK.
7.
a) K
MD BC
suy ra E, D, F thẳng hàng (đường thng Simson).
T các t giác ni tiếp MDBE, MDFC ta có:
MED MBD; MFD MCD==
EF MF
MEF MBC ( g.g )
BC MC
=
DF MF
ED DF
NC MC
= =
MDF MNC ( g.c.g )
= =MDF MNC DCF NMC
M thuc cung cha góc BAC dựng trên đoạn NC
M thuộc giao điểm ca cung cha góc BAC dng trên
đon NC và cung nh BC.
Nhn xét. Ngoài ta có th da vào ví d 3, để gii.
b) K
AH BC
, suy ra P, H, Q thẳng hàng (đường thng Simson)
PQ luôn đi qua điểm H c định.
8.
K
BE AD ( E AD)⊥
Ta có
BH AC, BK CD⊥⊥
E, H, K thng hàng.
T giác BEDK ni tiếp
EDB EKB=
T giác BHKC ni tiếp
BHK BCD 180 + =
Mt khác
BAD BCD 180+ =
BAD BHK BHK BAD ( g.g) =
CHUYÊN ĐỀ BỒI DƯỠNG HỌC SINH GIỎI HÌNH HỌC 9
21 | THCS.TOANMATH.com
MA MD, NH NK AMB HNB= =
AMB BNH BNE = =
Do đó tứ giác BEMN ni tiếp.
MEB 90 BNM 90 BN MN= =
b, Đường thng Steiner
Cho tam giác
ABC
ni tiếp đường tròn
( )
O
,
M
là điểm bt k thuộc đường tròn. Gi
,,N P Q
theo
th t là các điểm đối xng vi
M
qua
,,AB BC CA
. Chng minh rng
,,N P Q
thng hàng.
Chng minh:
Gi
,,H I K
theo th t là hình chiếu ca
M
lên
,,AB BC AC
; thế thì
,,H I K
thng hàng
(đường thng Simson). D thy
IH
đường trung bình ca tam giác
MNP
nên
//IH NP
.
Tương tự
//IK PQ
. Theo tiên đề Ơ-clit
do
,,H I K
thng hàng nên suy ra
,,N P Q
thẳng hàng. Đường thng
,,H I K
đi
qua
,,N P Q
được
gọi là đường thng Steiner của điểm
.M
Chú ý:
a, Ta có th chứng minh ba đim
,,N P Q
thng hàng bng cách dùng phép v tự: Các điểm
,,N P Q
lần lượt nh ca
M
trong phép v t tâm
M
t s 2,
,,H I K
thng hàng nên
,,N P Q
cũng
thẳng hàng. Như vy đường thng Steiner nh của đường thng Simson trong phép v t tâm
M
t s 2.
Ngoài ra liên quan đến đường thẳng Simson, Steiner ta cũng có các kết qu đẹp sau:
1. Đưng thẳng Steiner đi qua trực tâm ca tam giác
ABC
. (Olympic Nht Bn 1997)
Tht vy, gi
D
trc tâm ca tam giác
;,ABC BD CD
ct
( )
O
lần lượt
,EF
. D dàng chng
minh được
E
đối xng vi
D
qua
AC
,
F
đối xng vi
D
qua
.AB
Ta
FDMN
hình thang
cân các t giác
IBHM
MBFC
ni tiếp nên ta có:
= = =DFM DNM MBC IHM
do đó
/ / .ND IH
Tương tự ta cũng có:
CHUYÊN ĐỀ BỒI DƯỠNG HC SINH GII HÌNH HC 9
THCS.TOANMATH.com | 22
//DQ IK
,,H I K
thng hàng nên
,,N P Q
thẳng hàng. Nói cách khác: Đường thng Steiner ca
điểm
M
đi qua trực tâm ca tam giác
.ABC
Cách khác:
Gi
,,AS BJ CR
các đường cao ca tam
giác
ABC
,
D
là trc tâm.
Ta có
ANB AMB=
(tính chất đối xng).
Li có
AMB ADJ=
(cùng bù vi
SDJ
).
Suy ra
ANB ADJ=
nên
ADBN
t giác
ni tiếp, do đó
NAB NDB=
.
.NAB MAB NDB MAB= =
Chứng minh tương tự
CDQ CAM=
. Ta có
NDB CDQ MAB CAM BAC+ = + =
180 .NDQ NDB BDC CDQ BAC BDC = + + = + =
Vy
,,N D Q
thẳng hàng hay đường thẳng Steiner đi qua trực tâm ca tam giác
.ABC
2.
AB AC BC
MH MK MI
+=
(Olympic Toán M 1979)
Ta lấy điểm
T
trên cnh
BC
sao cho
MTC MBA=
suy ra
MBA MTC
(g.g)
Do
,MH MI
là các đường cao tương ứng nên
Ta có:
AB TC
MH MI
=
. T cách dựng đim
T
ta cũng
suy ra
MCA MTB
(g.g)
AC TB
MK MI
=
Cộng hai đẳng thc ta có:
.
AB AC BC
MH MK MI
+=
Ta xét mt s ví d:
Ví d 1.
Cho tam giác
ABC
3 góc nhn, ni tiếp
( )
O
. Các đường cao
,,AN BM CP
của tam giác cùng đi
qua điểm
H
. Gi
Q
điểm bt k trên cung nh
BC
(
Q
khác
,BC
). Gi
,EF
điểm đối xng
vi
Q
qua
,.AB AC
a, Chng minh:
. . .MH MA MP MN=
b, Chng minh:
,,E H F
thng hàng.
CHUYÊN ĐỀ BỒI DƯỠNG HỌC SINH GIỎI HÌNH HỌC 9
23 | THCS.TOANMATH.com
c, Gi
J
giao điểm ca
QE
AB
,
J
giao điểm ca
QF
AC
. Tìm v trí điểm
Q
để
AB AC
QJ QI
+
nh nht.
thi vào lp 10 chuyên Toán TP Hà Nội năm 2015-2016).
Gii:
Câu a) Dành cho bạn đọc.
Gi
S
chân đường vuông góc h t
Q
lên
.BC
Ta
,,I J S
thẳng hàng (Đường
thng Simson của điểm
Q
).
Ta cũng có:
,,E H F
thng hàng.
(Tính chất đường thẳng Steiner đi qua
trc tâm tam giác).
Ta cũng có:
AB AC BC
QJ QI QS
+=
nên
AB AC
QJ QI
+
nh nht khi ch khi
BC
QS
nh nht. Tc
QS
ln
nhất, điều này xy ra khi và ch khi
Q
là điểm chính gia cung nh
.BC
Bài tập tương tự: Cho tam giác nhn
ABC
ni tiếp
( )
O
các đường cao
,AE CF
ct nhau tại điểm
H
. Gi
P
là điểm thuc cung nh
BC
(
P
khác
,BC
),
,MN
lần lượt là hình chiếu vuông góc ca
P
trên các đường thng
,.AB AC
a, Chng minh:
2
,.
BH EF
OB EF
BO AC
⊥=
b, Đường thng
MN
đi qua trung điểm của đoạn
.HP
thi vào lớp 10 Trường THPT chuyên Phan Bi Châu- Ngh An 2015).
Ví d 2.
Cho t giác
ABCD
ni tiếp
( )
O
, gi
E
là điểm trên cung nh
AB
. Gi
, , ,H K P Q
lần lượt là hình
chiếu vuông góc ca
B
trên
, , ,AC CD AE DE
. Gi
,MN
lần lượt là trung điểm ca
,.AD HK
a, Chng minh:
,,AD PQ HK
đồng quy.
b, Chng minh:
.MN NB
tuyn sinh vào lp 10 chuyên toán- ĐHSP TP Hồ Chí Minh năm 2015).
Li gii:
Dng
BT AD
thì
,,T H K
thẳng hàng (Đường thng Simson của điểm
B
).
Tương tự các điểm
,,T Q P
thng hàng.
CHUYÊN ĐỀ BỒI DƯỠNG HC SINH GII HÌNH HC 9
THCS.TOANMATH.com | 24
Suy ra
,,AD PQ HK
đồng quy ti
.T
Ta có:
ADB HKB=
do
TDKB
ni tiếp.
HBK HCK ABD==
suy ra
BAD BHK
Suy ra
BAM BHN BMA BNH =
TMNB
ni tiếp
90 .MNB =
Ví d 3.
Cho tam giác
ABC
ni tiếp
( )
;OR
có trực tâm là điểm
,HD
là điểm trên cung nh
BC
, ly
E
sao
cho
//CE AD=
, gi
K
trc tâm tam giác
ACE
. Gi
,PQ
hình chiếu vuông góc ca
K
trên
,BC AB
. Chng minh:
PQ
đi qua trung điểm ca
.HK
(Trích đề thi HSG Quc gia 2004)
Gii:
Do
K
trc tâm tam giác
EAC
ADCE
hình bình hành nên
180 180AKC AEC ADC= =
suy ra t giác
ADCK
ni tiếp. Nói cách khác
K
nm trên
( )
O
.
Đưng thng
EK
ct
AC
ti
I
thì
,,P I Q
thẳng hàng (Đường thng Simson).
Do
//KP AH
nên ta nghĩ đến tính cht hai
đường chéo ca hình bình hành ct nhau ti
trung điểm mỗi đường.
Gi s
HA
ct
( )
,O PQ
lần lượt ti
,MN
do
KQBP
,
ABMK
ni tiếp nên ta có:
KPQ KBQ KMN==
suy ra
KPMN
ni tiếp, suy ra
KPMN
hình thang n. Mặt khác ta cũng
tính cht quen thuc
,HM
đối xng nhau qua
BC
nên
//MHP PMH KNH KN HP= =
suy ra
HPKN
là hình bình hành. Suy ra:
PQ
đi qua trung điểm ca
.HK
8. Đường thng Pascal
CHUYÊN ĐỀ BỒI DƯỠNG HỌC SINH GIỎI HÌNH HỌC 9
25 | THCS.TOANMATH.com
Cho 6 điểm
, , , , ,A B C D E F
cùng thuc một đường tròn (Có th hoán đổi th t). Gi
,,P Q R
ln
ợt giao điểm ca c cặp đường thng
( )
,AB DE
,
( )
,BC EF
,
( )
,CD FA
. Khi đó 3 điểm
,,P Q R
cùng nm trên một đường thng gọi là đường thng Pascal.
Chng minh:
Gi s
DE
ct
BC
ti
M
ct
AF
ti
N
,
BC
ct
AF
ti
.S
Áp dụng định Menelaus cho
SMN
cát
tuyến:
ABP
ta có:
S
. . 1
PM AN B
PN AS BM
=
hay
.
PM AS BM
PN AN BS
=
(1).
Áp dụng định lý Menelaus cho
SMN
và cát
tuyến:
CDR
ta có:
. . 1
RN CS DM
RS CM DN
=
suy ra
.
RN CM DN
RS CS DM
=
(2).
Áp dụng định Menelaus cho
SMN
cát tuyến
QEF
ta có:
. . 1
QM FS EN
QS FN EM
=
suy ra
.
QS FS EN
QM FN EM
=
(3).
Mt khác các t giác
,,ABCF BCDE AFED
ni tiếp nên:
. . ;SB SC SA SF=
..MC MB MD ME=
,
..NF NA ND NE=
(4). T (1),(2),(3),(4) ta suy ra
. . 1.
PM RN QS
PN RS QM
=
Theo định lý đảo Menelaus ta
suy ra
,,P Q R
thng hàng.
Đưng thng
PRQ
trên được gọi đường thng Pascal ng vi b điểm
, , , , , .A B C D E F
Bng
cách hoán v các điểm
, , , , ,A B C D E F
ta thu được rt nhiều các đường thng Pascal
khác nhau, c th ta tới 60 đường thng
Pascal. Chng hn v hình bên minh họa trường
hợp các điểm
.ACEBFD
Ngoài ta khi cho các điểm trùng nhau (khi đó lục giác suy biến thành tam giác, t giác, ngũ giác), ví
d
EF
thì cnh
EF
tr thành tiếp tuyến của đường tròn ti
E
, ta còn thu thêm được rt nhiu
CHUYÊN ĐỀ BỒI DƯỠNG HC SINH GII HÌNH HC 9
THCS.TOANMATH.com | 26
các đường thng Pascal khác na. Hình v dưới đây minh họa trường hợp các điểm
,.ABCDEE ABCCDD
9. Đường thng Gauss
Cho t giác
ABCD
,AB CD
ct nhau ti
,,M AD BC
ct nhau ti
.N
Khi đó trung điểm các
đoạn thng
,,AC BD MN
nm trên một đường thng gọi là đường thng Gauss ca t giác
.ABCD
Chng minh:
Gi
,,I E F
lần lượt trung điểm ca
,,BD AC MN
,,K G H
lần lượt trung đim
của các đoạn thng
,,CN CD DN
. D thy các
điểm
,,F H K
thng hàng.
,,E G K
thng hàng.
,,I G H
thng hàng.
Ta có
// , // , //FK MC IH BC EK DN
nên
,,
IG BC FH MD EK AN
IH BN FK MC EG AD
= = =
nhân 3 đẳng
thc ta có:
. . . .
IG FH EK BC MD AN
IH FK EG BN MC AD
=
. Áp dụng định lý Menelaus cho tam giác
CDN
đường thẳng đi
qua
,,B A M
ta có:
. . 1
BC MD AN
BN MC AD
=
, suy ra
. . 1.
IG FH EK
IH FK EG
=
Theo định Menelaus đảo ta suy
ra
,,I E F
thng hàng.
10. Đường thẳng Niutơn
CHUYÊN ĐỀ BỒI DƯỠNG HỌC SINH GIỎI HÌNH HỌC 9
27 | THCS.TOANMATH.com
Cho t giác
ABCD
ngoi tiếp
( )
I
, gi
,MN
lần lượt trung điểm ca
,.BD AC
Khi đó 3 điểm
,,I M N
thẳng hàng. Đường thng đi qua
,,I M N
gọi là đường thẳng Niutơn của t giác
.ABCD
Chng minh: (Ta ch xét trường hp
AB
không song song vi
CD
)
Gi các tiếp điểm ca
( )
I
vi
, , ,AB BC CD DA
ln
t là
,,,X Y Z T
thì
= = = = .IX IY IZ IT r
Gi s
,AD BC
ct nhau ti
P
, trên
PD
ly
E
sao
cho
PE AD=
, trên
PC
ly
F
sao cho
PF BC=
thế thì:
1 1 1
.
2 2 2
MAD MBC DAB DBC ABCD
S S S S S+ = + =
1 1 1
.
2 2 2
NAD NBC CAD ABC ABCD
S S S S S+ = + =
T đó suy
ra:
.
MAD MBC NAD NBC
S S S S+ = +
Theo cách xác đnh
,EF
ta có:
, , ,
MAD MPE MBC MPF NAD NPE
S S S S S S= = =
NBC NPF
SS=
suy ra:
MPE MPF NPE NPF
S S S S+ = +
hay
MEPF NEPF
SS=
(1).
Li có:
,
IAD IPE IBC IPF IAD IBC IPE IPF IEPF
S S S S S S S S S= = + = + =
nhưng
+ = + + + =
1 1 1 1 1
2 2 2 2 2
IAD IBC AXYT DZYT IZCY XIYB ABCD
S S S S S S S
suy ra
MEPF IEPF
SS=
(2). T (1), (2) ta suy
ra
MEPF NEPF IEPF
S S S==
hay
/ / , / /
MEF NEF IEF
S S S MN EF MI EF= =
suy ra
,,M N I
thng hàng.
11. Định lý Brocad
Cho t giác
ABCD
ni tiếp đường tròn
( )
O
. Gi
M
giao điểm ca
AB
CD
;
N
là giao điểm
ca
AD
BC
;
I
giao điểm ca
AC
BD
. Chng minh rng
I
trc tâm ca tam giác
.OMN
Chng minh:
Gi
E
là giao điểm th 2 của đường tròn ngoi tiếp tam giác
ABI
.CDI
Trước tiên ta chng minh:
,,E I M
thng hàng:
CHUYÊN ĐỀ BỒI DƯỠNG HC SINH GII HÌNH HC 9
THCS.TOANMATH.com | 28
Tht vy gi s
MI
cắt đường tròn ngoi tiếp
tam giác
AIB
ti
E
thì
. . .MI ME MB MA MC MD
==
suy ra 4 đim
, , ,D E I C
nm trên một đường tròn suy ra
EE
suy ra
,,M I E
thng hàng.
Ta có:
360AED DEI AEI=
180 180DEI AEI= +
ABD ACD AOD= + =
nên t giác
AEOD
ni
tiếp.
Ta cũng có:
BEC BEI IEC BAC BDC BOC= + = + =
nên t giác
BEOC
ni tiếp. T đó ta cũng
suy ra
,,N E O
thng hàng.
Ta có:
180
90
2
BOC
OEM OEC CEM OBC BDC BDC
−
= + = + = + =
suy ra
.ME ON
Chng
minh tương tự ta cũng có:
NI OM
suy ra
I
là trc tâm ca tam giác
.OMN
Định Brocard là mt công c hình hc khá mạnh để chng minh các quan h vuông góc được
s dng rt nhiu trong các k thi tuyn sinh vào lớp 10 chuyên cũng như k thi hc sinh gii tnh
thành, quc gia.
rt nhiều cách để chứng minh, nhưng cách giải trên thun túy phù hp nht vi bc THCS.
Sau đây ta sẽ xét mt s ng dng của định lý này:
Ví d 1.
Cho tam giác
ABC
không cân 3 góc nhn, ni tiếp
( )
O
, các đường cao
1 1 1
,,AA BB CC
ct nhau
tại điểm
H
, các đường thng
11
,AC AC
ct nhau ti
,D
gi
X
là giao điểm ca
BD
vi
( )
.O
a, Chng minh:
11
. . .DX DB DC DA=
b, Gi
M
là trung điểm ca
.AC
Chng minh:
.DH BM
(Trích đề tuyn sinh vào lớp 10 chuyên ĐHSP – Năm học 2013-2014)
Gii:
CHUYÊN ĐỀ BỒI DƯỠNG HỌC SINH GIỎI HÌNH HỌC 9
29 | THCS.TOANMATH.com
a, Các t giác
11
,BXAC ACA C
ni tiếp nên ta
có:
==
11
. . . .DX DB DA DC DC DA
b, Xét t giác
11
AC AC
ni tiếp đường tròn tâm
M
đường kính
AC
, hai đường chéo
11
,AA CC
ct nhau ti
.H
Theo định Brocard ta
H
là trc tâm tam giác
BMD
suy ra
.DH BM
Ví d 2.
Cho t giác
ABCD
ni tiếp đường tròn
( )
O
. Các tia
,AB DC
ct nhau ti
E
, các tia
,AD BC
ct
nhau ti
.F
Đưng tròn ngoi tiếp tam giác
BCE
cắt đường tròn ngoi tiếp tam giác
CDF
ti
M
.
Chng minh rng:
a, Ba điểm
,,E M F
thng hàng và bốn điểm
, , ,A D M E
nm trên một đường tròn.
b,
.OM EF
c, Chứng minh đường thng
OM
đi qua giao đim
I
của hai đường chéo
,AC BD
ca t giác
.ABCD
thi hc sinh gii toán TP HCM- 2013)
Phân tích định hướng gii:
a, Để chng minh
,,E M F
thng hàng ta
chng minh:
180 .EMC CMF+ =
Ta tìm
cách quy v hai góc đối nhau trong t giác
ni tiếp.
Ta có
180 .EMC CMF ABC CDA+ = + =
Ta
DMF DCF DAB==
suy ra 4 điểm
, , ,A D M E
cùng nm trên một đường tròn.
b, T đó ta dễ chứng minh được các t giác
,ABMF BDFE
ni tiếp. Xét t giác
OAMC
ta có:
180 180 180 2 180CMA CMF AME ADC ADC ADC AOC= = = =
suy ra t giác
OACM
ni tiếp. Ta
OMF OMA AMB OMA ABF OMC CME OME= + = + = + =
suy ra
90 .OME OMF= =
Theo định lý Brocard ta có:
OI EF
suy ra
,,O I M
thng hàng.
Ví d 3.
CHUYÊN ĐỀ BỒI DƯỠNG HC SINH GII HÌNH HC 9
THCS.TOANMATH.com | 30
Cho tam giác nhn
ABC
( )
AB AC
các đưng cao
,BD CE
ct nhau ti
.H
Gi
I
trung
điểm ca
,BC
đường tròn ngoi tiếp tam giác
BEI
cắt đưng tròn ngoi tiếp tam giác
CDI
ti
K
khác
.I
Đưng thng
DE
ct
BC
ti
M
. Chng minh:
,,M H K
thng hàng.
thi vào lp 10 chuyên Nguyn Trãi- Hải Dương năm 2008).
Li gii:
Áp dụng định Brocard cho t
giác
BEDC
ta
H
trc tâm
ca tam giác
AMI
nên
MH AI
.(1)
Mt khác ta cũng dễ chng minh
được:
,,A K I
thng hàng (Xem
d 1)
360 180EKD EKI DKI ABC ACB BAC= = + =
suy ra
AEKD
ni tiếp, suy ra
90AKH =
. Kết hp vi (1) ta suy ra
,,M H K
thng hàng.
12. Định lý Ptolemy.
Định Ptolemy cho t giác ni tiếp: Cho t giác
ABCD
ni tiếp trong đường tròn
( )
O
. Khi đó ta
có:
. . . .ABCD AD BC AC BD+=
Chng minh:
Trên đường chéo
BD
lấy điểm
E
sao cho
.DAE BAC=
Ta có
DAE BAC=
ADE ACB=
(cùng chn
AB
) nên
ADE ACB
(g.g)
AD DE
AC BC
=
..AD BC AC DE=
(1). Do
DAE CAB=
nên
DAC EAB=
, li
ABE ACD=
(cùng chn
AD
)
ABE ACD
(g.g)
..
AB BE
AB CD AC BE
AC CD
= =
(2).
T (1) và (2) suy ra
( )
+ = + =. . . .AB CD AD BC AC BE DE AC BD
13. Định lý Ptolemy cho t giác bt k
Cho t giác
ABCD
. Chng minh rng
. . . .ABCD AD BC AC BD+
CHUYÊN ĐỀ BỒI DƯỠNG HỌC SINH GIỎI HÌNH HỌC 9
31 | THCS.TOANMATH.com
Chng minh:
Bên trong t giác
ABCD
lấy điểm
M
sao cho
MAD CAB=
MDA ACB=
Ta có:
ADM ACB
(g.g)
..
AD DM
AD BC AC DM
AC BC
= =
(1)
Do
MAD CAB=
nên
.DAC MAB=
t tam giác
ADC
MAB
, có:
DAC MAB=
(chng minh trên)
AD AM
AC AB
=
(do
AMD ACB
) nên
ADC AMB
(c.g.c)
..
DC AC
AB CD AC MB
MB AB
= =
(2).
T (1) và (2) suy ra
( )
. . . .AB CD AD BC AC BM DM AC BD+ = +
Dấu đẳng thc xy ra khi ch khi khi
M
nằm trên đường chéo
BD
, lúc đó tứ giác
ABCD
ni
tiếp.
Bây gi ta s xét mt s ví d tiêu biu vn dụng định lý Ptolemy
Ví d 1.
Cho đường tròn tâm
O
đường kính
AB
một điểm
I
nằm trên đoạn thng
,AO I
khác điểm
,AO
. Đường thng qua
I
vuông góc vi
AB
ct
( )
O
ti
,CD
. Gi
E
điểm nm trên
( )
O
sao
cho
D
là điểm chính gia cung
.AE
Gi
K
là giao điểm ca
,.AE CD
a, Chng minh:
OK
đi qua trung điểm ca
.CE
b, Đường thng
I
song song vi
CE
ct
,AE BE
lần lượt ti
,PQ
. Chng minh:
DPEQ
hình
ch nht.
c, Tìm v trí điểm
I
trên đoạn thng
AO
sao cho
.KC KA KO=+
(Trích đề tuyn sinh vào lp 10 chuyên Toán- Tin Tp Hà Nội năm 2015).
Gii:
CHUYÊN ĐỀ BỒI DƯỠNG HC SINH GII HÌNH HC 9
THCS.TOANMATH.com | 32
a, T gi thiết ta suy ra
AC AD DC==
Suy ra t giác
CADE
hình thang cân hai
đáy
,AD CE
.
K
giao điểm của hai đường
chéo nên
K
nm trên trung trc ca
.CE
Suy
ra
.KO CE
Nói cách khác
KO
đi qua trung
điểm ca
.CE
b, Do
AB
đường kính ca
( )
O
nên
90AEB AEQ= =
T giác
IPDA
IPA EPQ AEC ADI= = =
nên
IPDA
t giác ni tiếp suy ra
90APD =
t đó dễ dàng suy ra
,,O P D
thng hàng,
IPDA
hình thang cân và
//PD CE
. T giác
DPEQ
DEQ PDE PDA DPQ= = =
nên
DPEQ
là t giác
ni tiếp, kết hp
90DPE AEQ= =
suy ra
DPEQ
là hình ch nht.
c, Xét t giác
COKA
ta có:
(
)
1
2
COA AC AC DE CKA= = + =
. T đó suy ra
COKA
t
giác ni tiếp. Áp dụng định Ptolemy ta có:
. . .KAOC AK KO CK AO+=
hay
.
AC
KA KO CK
AO
+=
như vậy để
KC KA KO=+
thì điều kin
1
AC
AC AO
AO
= =
hay
ACO
tam giác đều suy ra
I
phải là trung điểm ca
.AO
Ví d 2.
Cho tam giác
ABC
đều ni tiếp đường tròn
( )
O
bán kính
,R
điểm
M
chuyển động trên cung nh
.BC
a, Chng minh:
.MA MB MC=+
b, Tìm GTLN, GTNN ca
.P MA MB MC= + +
(Trích đề tuyn sinh vào lớp 10 Trường chuyên Đại hc Vinh 2006).
c, Tìm giá tr nh nht ca biu thc:
11
Q
MB MC
=+
(Trích đề tuyn sinh vào lớp 10 Trường chuyên Lê Quý Đôn – Đà Nẵng năm 2009)
Li gii:
CHUYÊN ĐỀ BỒI DƯỠNG HỌC SINH GIỎI HÌNH HỌC 9
33 | THCS.TOANMATH.com
a, Áp dụng định lý Ptolemy cho t giác
ABMC
ta có:
. . . .MA BC AB MC AC MB=+
Do
.AB BC CA==
Suy ra
.MA MB MC=+
b, Ta có:
2.P MA MB MC MA= + + =
P
nh nht khi và ch khi
MB
hoc
.MC
P
ln nht khi ch khi
MA
đường kính
ca
( )
.O
c, Vi mi s thực dương
,xy
ta có:
( )
1 1 1
2 .2 4.x y xy
x y xy

+ + =


Nên ta suy ra
1 1 4
.
x y x y
+
+
Áp dng vào bài toán ta có:
1 1 4 4
.
MB MC MB MC MA
+ =
+
Do
2MA R
nên ta
suy ra
1 1 4
2
2
R
MA MB R
+ =
. Du bng xy ra khi và ch khi
AM
đường kính ca
( )
O
. Hay
M
là điểm chính gia cung nh
.BC
BÀI TP T LUYN
1. Chng minh rng nếu điểm P nm trên cung nh
AB
của đường tròn ngoi tiếp hình
vuông ABCD thì
PA PC PD
PB PD PC
+
=
+
2. Cho mt t giác ni tiếp có các cnh liên tiếp bng a, b, c, d các đường chéo bng p, q.
Chng minh rng:
2 2 2 2
..pq a b c d + +
3. Cho tam giác ABC không đều. Gi I O ln t tâm đường tròn ni tiếp ngoi
tiếp tam giác. Chng minh rng
90AIO
khi và ch khi
2AB AC BC+
4. Cho t giác ABCD ni tiếp đường tròn (O).
Chng minh rng:
..
..
AC BC CD AB BD
BD BC BA DC DA
+
=
+
5. Cho hai đường tròn
( )
11
;OR
( )
22
;OR
ct nhau tại hai điểm phân bit A và B
1
(O
2
O
nm v hai phía ca AB). Mt cát tuyến d qua A ct
( )
1
,O
( )
2
O
lần lượt tại c điểm C, D
khác A (A thuộc đon CD). Tiếp tuyến ti C ca
( )
1
O
ct tiếp tuyến ti D ca
( )
2
O
M.
Tìm v trí ca d sao cho
12
MC MD
RR
+
đạt giá tr
CHUYÊN ĐỀ BỒI DƯỠNG HC SINH GII HÌNH HC 9
THCS.TOANMATH.com | 34
ln nht.
6. Cho tam giác ABC I tâm đường tròn ni tiếp, O tâm đường tròn ngoi tiếp
trng tâm G. Gi s rng
90 .OIA=
Chng minh rng IG song song vi BC.
7. Cho tam giác ABC vi
BC CA AB
ni tiếp trong đường tròn (O). Trên cnh BC ly
đim D và trên tia BA lấy điểm E sao cho
.BD BE CA==
Đưng tròn ngoi tiếp tam giác
BDE ct cnh AC tại điểm P đưng thng BP cắt đường tròn (O) tại điểm th hai Q.
a) Chng minh rng tam giác AQC đồng dng vi tam giác EPD.
b) Chng minh rng
.BP AQ CQ=+
(Tuyn sinh lp 10, THPT chuyên tỉnh Vĩnh Phúc, năm học 2011-2012)
8. Cho hình bình hành ABCD. Một đường tròn đi qua A cắt các đoạn thng AB, AC, AD ln
t tại điểm P, Q, R khác A. Chng minh rng:
. . . .AB AP AD AR AQ AC+=
9. Gi s M, N các điểm nm trong tam giác ABC sao cho
MAB NAC=
.MBA NBC=
Chng minh rng:
. . .
1
. . .
AM AN BM BN CM CN
AB AC BA BC CACB
+ + =
CHUYÊN ĐỀ BỒI DƯỠNG HỌC SINH GIỎI HÌNH HỌC 9
35 | THCS.TOANMATH.com
NG DN GII-ĐÁP SỐ
1. Đặt độ dài cnh hình vuông ABCD a thì
2.==AC BD a
Áp dụng định lí Ptôlêmê cho t giác PADC, ta có:
( ) ( )
...
. 2 1 .
=+
= +
PD AC PACD PC AD
PD a a PA PC
Áp dng định Ptôlêmê cho t giác PBCD, ta có:
...PC BD PBCD PD BC=+
( ) ( )
. 2 2 .PC a a PB PD = +
T ( 1 ) và (2), suy ra:
PB
PA PC PD
PCPD+
+
=
2. Áp dụng định lí Ptôlêmê cho t giác ni tiếp thì ta có:
.ac bd pq+=
Áp dng bất đẳng thc Bu-nhi-a-cp-xki ta có:
( )
( )( ) ( )( )
2
2 2 2 2 2 2 2 2 2 2
ac bd a b c d p q a b c d+ + + + +
Suy ra:
2 2 2 2
..pq a b c d + +
Du bng xy ra khi:
ab
cd
=
3. Kéo dài AI cắt đường tròn ngoi tiếp tam giác ABC ti D.
Ta có:
.BAD DAC DB DC= =
I là tâm đường tròn ni tiếp tam giác ABC nên:
( )
( )
1
2
1
2
BID IBA BAI B A
IBD IBC CBD B A
= + = +
= + = +
Suy ra
DIB
cân ti D nên
.DI DB=
Áp dụng định lý Ptôlêmê cho t giác ABDC ni tiếp:
( ) ( )
...
. .
AD BC ABCD AC BD
BD AB AC DI AB AC
=+
= + = +
Suy ra:
ABAD
ID BC
AC+
=
Do
AOD
cân ti O nên
90AIO
CHUYÊN ĐỀ BỒI DƯỠNG HC SINH GII HÌNH HC 9
THCS.TOANMATH.com | 36
2. 2. 2
2 .
AD AB AC
AI ID AI ID ID AD ID
ID BC
AB AC BC
+
+ =
+
4. Ly E, F thuộc đường tròn sao cho
, .CDB ADE ADB DCF==
Khi đó
, , , .AE BC FD AB EC AB BF AD= = = =
Áp dụng định Ptôlêmê cho hai t giác ni tiếp AECD
BCDF ta có:
( )
( )
. . .
. . 1
. . .
. . 2
AC ED AE CD AD EC
BC CD AD AB
BDCF BC DF BF CD
BC AB ADCD
=+
=+
=+
=+
Mt khác:
CDE CDB BDE=+
ADE BDE ADB FCD= + = =
Do đó:
FDC FDE CDE FCE FCD ECD= + = + =
suy ra
.ED FC=
T (1), (2), (3) suy ra:
. . .
. . .
BC CD AB BD AC ED AC
BC BA DC DA BD CF BD
+
==
+
5. H BH vuông góc vi MD.
Ta có:
; ;MCA CBA MDA DBA MCA MDB CBD= = + =
180CMD CBD CMD MCA MDB + = + + =
Suy ra BCMD là t giác ni tiếp.
Áp dụng định lý Ptôlêmê ta có:
( )
. . . 1MC BD MD BC MBCD+=
Do
CBD
12
O AO
nên:
1 2 1 2
BC BD CD
O A O A O O
==
Hay
1 2 1 2
BC BD CD
R R OO
==
Đặt
1 2 1 2
BC BD CD
k
R R OO
= = =
Suy ra:
1 2 1 2
. , . , BC k R BD k R CD kOO= = =
(2)
CHUYÊN ĐỀ BỒI DƯỠNG HỌC SINH GIỎI HÌNH HỌC 9
37 | THCS.TOANMATH.com
T (1) và (2) suy ra:
2 1 1 2
. . .+=MC R MD R MBOO
hay
12
1 2 1 2
.MB OO
MC MD
R R R R
+=
Do đó
12
MC MD
RR
+
đạt giá tr ln nht khi và ch khi MB ln nht
Tam giác BMH luôn t đồng dng vi chính khi d thay đổi, nên BM ln nht khi và ch
khi BH ln nht, mà
2
2. .BH BD R
Du bng xy ra khi BD đối xng qua
2
,O
khi đó
12
//d OO
6. Kéo dài AI ct BC(O) ti D; N.
Khi đó N là điểm chính gia cung BC (không cha A).
Ta có:
( )
1.BN NC=
Li có:
( )
2IBN BIN BN IN= =
Do
OI AN
suy ra
( )
1
3
2
IA IN AN==
T (1), (2), (3)
( )
4BN NC IN IA = = =
Áp dụng định lí Ptôlêmê cho t giác ni tiếp ABNC ta có:
. . .BN AC AB NC BC AN+=
T (4)
( ) ( )
2 . 2 5BN AC AB BN BC AC AB BC + = + =
Áp dng tính chất đường phân giác trong tam giác ABD, ACD và (5) ta có:
2
2
AB IA AC AB AC AB AC BC
BD ID CD BD CD BC BC
++
= = = = = =
+
Vy
2
IA
ID
=
(6).
Mt khác G là trng tâm ca tam giác suy ra
2
AG
GM
=
(7).
T (6),(7)
2 = =
IA AG
ID GM
Suy IG song song vi BC nh lý Ta-Lét đảo).
7.
CHUYÊN ĐỀ BỒI DƯỠNG HC SINH GII HÌNH HC 9
THCS.TOANMATH.com | 38
a) Do các t giác BEPD, ABCQ ni tiếp, nên
EDP EBP ABQ ACQ= = =
(1) và
( )
180
180 2
EPD EBD
ABC AQC
=
= =
T (1) và (2) suy ra
AQC
œ
.EPD
Điu phi chng minh.
b) Theo kết qu phn a, ta có
+
===
+
QA QC CA QA QC
PE PD DE PE PD
Suy ra
( ) ( ) ( )
...QA QC DE PE PD AC PE PD BD+ = + = +
(3)
Áp dụng đnh Ptôlêmê cho t giác BEPD ni tiếp, ta
đưc
( )
. . . .BP ED BE PD EP BD PD PE BD= + = +
(4)
T (3) và (4) suy ra
( )
..QA QC ED BP ED+=
hay
QA QC BP+=
(điều phi chng minh).
8.
ACB CAD RPQ==
BAC PRQ=
nên
ABC
đồng dng vi
RQP
(g.g)
AB BC AC
RQ QP RP
= =
Suy ra;
..
;
ABQP AC PQ
RQ RP
BC BC
==
Áp dụng định Ptôlêmê cho t giác APQR ta
đưc:
. . .APQR AR PQ AQ PR+=
. . .
..
. . .
AP ABQP AC QP
ARQP AQ
BC BC
AP AB ARQP AQ AC
+=
+ =
. . .AP AB AR AD AQ AC + =
(vì
)BC AD=
9
Lấy điểm K trên đường thng BN sao cho
BCK BMA=
Khi đó
BMA
BCK
Suy ra
( )
1
AB BM AM
BK BC CK
==
CHUYÊN ĐỀ BỒI DƯỠNG HỌC SINH GIỎI HÌNH HỌC 9
39 | THCS.TOANMATH.com
AB BK
MB BC
=
Mt khác, d thy
ABK MBC=
T đó
ABK
,MBC
dẫn đến
( )
2
AB BK AK
MB BC CM
==
Cũng từ
BMA
BCK
ta
,CKN MAB NAC==
suy ra A, N, C, K cùng
nm trên một đường tròn. Áp dụng định
Ptôlêmê cho t giác ni tiếp ANCK ta được:
( )
. . . 3AC NK AN CK CN AK=+
Nhưng từ (1) và (2) thì:
..
;
.
;
AM BC AB CM
CK AK B
AB BC
BM
K
BM BM
= = =
Nên t (3)
( )
. . .AC BK BN AN CK CN AK = +
. . . . .
.
AB BC AN AM BC CN ABCM
AC BN
BM BM BM


= +
. . . . . . . .AB AC BC AN AM BC CN ABCM BN BM AC= + +
. . .
1
. . .
M AN BM BN CM CN
AB AC BA BC CA B
A
C
+ =
14. Điểm Miquel- Định lý Miquel
a, Điểm Miquel ca t giác toàn phn:
Cho t giác
ABCD
có cnh
,AB CD
ct nhau ti
M
, cnh
,AD BC
ct nhau ti
.N
Các đường tròn
ngoi tiếp các tam giác
, , ,ABN BCM MAD NCD
cùng đi qua một điểm
E
(Gọi điểm Miquel ca
t giác
ABCD
).
Chng minh:
Gi s đường tròn ngoi tiếp tam giác
,MAD NCD
ct nhau tại giao điểm th 2
E
(khác
D
). Ta chng minh
E
thuộc đường tròn
ngoi tiếp các tam giác
,MBC NAB
Tc là chng minh các t giác
MBCE
,NBAE
ni tiếp. Tht vy t giác
DCNE
ni tiếp nên:
.MDE CNE=
T giác
ADEM
ni tiếp nên:
CHUYÊN ĐỀ BỒI DƯỠNG HC SINH GII HÌNH HC 9
THCS.TOANMATH.com | 40
MAE MDE=
suy ra
MAE CNE ABNE=
ni tiếp. Mặt khác ta cũng có:
ECN EDN=
EDN AME AME ECN= =
suy ra
BMEC
ni tiếp. Như vậy các đường tròn ngoi tiếp các tam
giác
, , ,ABN BCM MAD NCD
cùng đi qua một điểm
.E
b, Định lý Miquel đối vi tam giác:
Trên các cnh
,,AB BC CA
lần lượt lấy các đim
,,F D E
. Khi đó ba đường tròn ngoi tiếp các tam
giác
,,AFE BDF CED
ct nhau ti một điểm
M
(Gọi là điểm Miquel trong tam giác
ABC
).
Chng minh:
Gi s đường tròn ngoi tiếp các tam giác
AEF
BDF
ct nhau ti
.M
Ta có:
360EMD EMF DMF=
180 180 180 .FAE EMF A B C= + = + =
Vy
EMDC
là t giác ni tiếp.
Suy ra ba đường tròn ngoi tiếp các tam giác
,,AFE BDF CED
ct nhau ti một điểm
.M
Ví d 1.
Cho tam giác
ABC
,BC
góc nhn
60 .BAC =
Các đường phân giác trong
11
,AA BB
ca
tam giác
ABC
ct nhau ti
.I
a, Chng minh:
11
AB IC
là t giác ni tiếp.
b, Gi
K
là giao điểm th 2 ca
BC
với đường tròn ngoi tiếp tam giác
1
.BC I
Chng minh t giác
1
CKIB
ni tiếp.
c, Chng minh:
11
.AK B C
tuyn sinh vào lp 10- Trường THPT chuyên ĐHSP Hà Nội)
Gii:
Ta thy
(
)
1 1 1 1
1 1 1
120 180
2 2 2
C IB C IA B IA A C A B B C= + = + + + = + + =
. Nên
11
AB IC
t giác
ni tiếp. Ta cũng thấy đim
I
điểm Miquel ca tam giác
ABC
nên suy ra t giác
1
CKIB
t
giác ni tiếp.
CHUYÊN ĐỀ BỒI DƯỠNG HỌC SINH GIỎI HÌNH HỌC 9
41 | THCS.TOANMATH.com
Ta có:
1 1 1 1
AB K AB I IB K BC I ICB= + = +
180 B=
nên t giác
1
ABKB
là t giác ni tiếp.
Hoàn toàn tương t ta cũng có:
1 1 1
,BC B C ACKC
là các t giác ni tiếp.
Ta có:
(
)
1 1 1 1
11
90 .
22
AC B C AK AIB C A B C+ = + = + + =
Suy ra
11
.AK C B
Ngoài ra ta cũng có thể chng minh theo cách:
Ch ra tam giác
1
AB K
cân ti
1
B
11
BC
là đường phân giác nên cũng là đường cao.
Chuyên đề 8.ĐỊNH LÝ PTÔLÊMÊ-ĐƯNG THNG SIMSON
CH ĐỀ 1.ĐỊNH LÝ PTÔLÊMÊ
A. Kiến thc cn nh
Ptôlêmê là nhà khoa hc c Hy Lp, sng vào thế k 2. T năm 127 đến năm 151 sau công nguyên, ông sng
tại Alechxanđri (Ai Cập), nghiên cu toán học, thiên văn hc địa lý. Ông tác gi ca thuyết h trụ
địa tâm; hình cấu trúc trụ đầu tiên, khng đnh mt cách sai lm rng, các thiên th chuyển động
trên những vòng tròn có tâm là tâm trái đt nm yên, là cơ sở cho thiên văn học trong mt thòi gian dài cho
đến thế k 17, trước khi thuyết h nht tâm của Kôpecnich ra đời.
Công trình toán hc của ông khá phong phú, sau đây là một định lý mang tên ông.
Định lý. Trong mt t giác ni tiếp thì tích hai đường chéo bng tng các tích ca hai cp cạnh đối din.
Gii
Cho t giác ABCD ni tiếp đường tròn (O).
Ta cn chng minh:
. . .ABCD AD BC AC BD+=
Gi s
DBC ABD
Lấy điểm M trên đoạn
AC
sao cho
MBC ABD=
Suy ra
ABM
DBC
Suy ra
AB AM
BD CD
=
. . AB CD BD AM=
CBM
DBA
Suy ra
BC CM
BD AD
=
..AD BC BDCM=
Do đó
( )
. . . .ABCD AD BC BD AM CM AC BD+ = + =
B. Mt s ví d
d 1. Cho nửa đường tròn
( )
;,OR
đưng kính AB C
điểm chính gia. Gi M điểm bt thuc cung BC.
Chng minh rng:
. 2.AM BM CM−=
Gii
Tìm cách gii. Vi
CA CB=
ta suy ra
CA CB=
biu
diễn được qua bán kính R. M điểm bt thuc cung
BC, kết lun liên quan ti MA, MB, MC nên ta liên tưởng tới định lý Ptôlêmê.
Trình bày li gii
Ta có
, 90AC BC ACB= =
nên
ABC
vuông cân ti C
2
2
AB
AC BC R = = =
Áp dụng định lý Ptôlêmê cho t giác ABMC ta được:
. . .
2. 2 . 2. . 2
AC BM AB CM AM BC
R BM R CM R AM AM BM CM
+=
+ = =
Ví d 2. Cho t giác ABCD ni tiếp
. . .ABCD AD BC=
Gi M trung điểm ca BC. Chng minh
rng:
.MAB CAD=
Gii
Tìm ch gii.
MAB CAD=
ABM
ACD
(vì đã
)ABM ACD=
Do vy cn chng t cp cnh k góc y t l tc là
AB AC
MB DC
=
Da vào gi thiết, tt yếu ta nghĩ ti vn dụng định
Ptôlêmê.
Trình bày li gii
Áp dụng định lý Ptôlêmê cho t giác ABCD ta được:
. . . ABCD AD BC AC BD+=
..ABCD AD BC=
nên:
2. . 2 .
AB AC
AB CD AC BM
MB DC
= =
Mt khác
ABM ACD=
suy ra:
ABM
ACD
(c.g.c)
Vy
.MAB CAD=
Ví d 3. Cho đường tròn (O) và dây cung BC khác đường kính. Tìm điểm A thuc cung ln BC ca
đưng tròn sao cho
21. 10.AB AC+
đạt giá tr ln nht.
Gii
Tìm cách gii. Nếu điểm E trên cung nh BC thì ta có:
. . . .ABCE AC BE BC AE+=
Do vậy để xut
hin
21. 10.AB AC+
thì ta cần xác định điểm E sao cho
21. 10. .BE CE=
tc
10
..
21
BE C E=
Vi t l
như vậy chúng ta li nghĩ ti đưng phân giác góc BEC. Do vy bn cht ca bài dựng được
đim E.
Trình bày li gii
Gi I là điểm thuc cnh BC sao cho
10
21
IB
IC
=
Gi D là điểm chính gia cung ln BC của đường tròn (O).
Gi E là giao điểm th hai ca DI vi (O).
Khi đó EI là phân giác ca góc
BEC
Suy ra
10
21
EB IB
EC IC
= =
10
..
21
EB EC=
Áp dng định Ptoleme cho t giác ni tiếp ABEC, ta có:
. . .ABCE AC BE BC AE+=
Suy ra:
10
. . . .
21
ABCE AC EC BC AE+=
21
21 10 . .
BC
AB AC AE
CE
+=
Do đó
21 10AB AC+
đạt gtr ln nht khi AE ln nht
AE
là đường kính ca (O).
d 4. Cho tam giác ABC ni tiếp đường tròn (O)
2. .AC AB=
Các đường thng tiếp xúc vi
đưng tròn (O) ti A, C ct nhau P. Chng minh rng BP đi qua điểm chính gia ca cung BAC.
Gii
Tìm cách gii. Để chng minh
,BD CD=
ta cn chng minh
.BD CD=
Như vậy da vào kết lun
gi thiết đều liên quan ti cnh và t giác ABCD nên ta nghĩ tới vic vn dụng định lý Ptoleme.
Tuy nhiên trong bài, t giác này hai tiếp tuyến hai đỉnh đi din (A C) và đường chéo
đồng quy thì luôn có
..CD BA BC AD=
(bn nên nh tính chất này để s dng).
Trình bày li gii
Ta có:
PCD
PBC
nên
CD PC
BC PB
=
PAD
PBA
nên
DA PA
BA PB
=
.
Mt khác
PC PA=
nên
CD DA
BC BA
=
Suy ra
..CD BA BC AD=
(1)
Áp dụng định lý Ptôlêmê ta có:
. . .CD BA AD BC AC BD+=
(2)
T (1) và (2) suy ra:
2. . .CD AB AC BD=
Mt khác:
2.AB AC=
nên
.CD BD=
Vy D đim chính gia ca cung BAC.
C. Bài tp vn dng
1. Chng minh rng nếu điểm P nm trên cung nh
AB
của đường tròn ngoi tiếp hình vuông
ABCD thì
PA PC PD
PB PD PC
+
=
+
2. Cho mt t giác ni tiếp các cnh liên tiếp bng a, b, c, d các đường chéo bng p, q. Chng
minh rng:
2 2 2 2
..pq a b c d + +
3. Cho tam giác ABC không đều. Gi I O ln t tâm đường tròn ni tiếp ngoi tiếp tam
giác. Chng minh rng
90AIO
khi và ch khi
2AB AC BC+
4. Cho t giác ABCD ni tiếp đường tròn (O).
Chng minh rng:
..
..
AC BC CD AB BD
BD BC BA DC DA
+
=
+
5. Cho hai đường tròn
( )
11
;OR
( )
22
;OR
ct nhau tại hai điểm phân bit A và B
1
(O
2
O
nm v
hai phía ca AB). Mt cát tuyến d qua A ct
( )
1
,O
( )
2
O
lần lượt tại các điểm C, D khác A (A thuc
đon CD). Tiếp tuyến ti C ca
( )
1
O
ct tiếp tuyến ti D ca
( )
2
O
M. Tìm v trí ca d sao cho
12
MC MD
RR
+
đạt giá tr
ln nht.
6. Cho tam giác ABC I tâm đường tròn ni tiếp, O tâm đường tròn ngoi tiếp trng tâm
G. Gi s rng
90 .OIA=
Chng minh rng IG song song vi BC.
7. Cho tam giác ABC vi
BC CA AB
ni tiếp trong đưng tròn (O). Trên cnh BC lấy điểm D
trên tia BA lấy điểm E sao cho
.BD BE CA==
Đưng tròn ngoi tiếp tam giác BDE ct cnh AC ti
đim P đưng thng BP cắt đường tròn (O) tại điểm th hai Q.
a) Chng minh rng tam giác AQC đồng dng vi tam giác EPD.
b) Chng minh rng
.BP AQ CQ=+
(Tuyn sinh lp 10, THPT chuyên tỉnh Vĩnh Phúc, năm học 2011-2012)
8. Cho hình bình hành ABCD. Một đường tròn đi qua A cắt các đoạn thng AB, AC, AD lần lượt ti
đim P, Q, R khác A. Chng minh rng:
. . . .AB AP AD AR AQ AC+=
9. Gi s M, N các điểm nm trong tam giác ABC sao cho
MAB NAC=
.MBA NBC=
Chng
minh rng:
. . .
1
. . .
AM AN BM BN CM CN
AB AC BA BC CACB
+ + =
NG DN GII-ĐÁP SỐ
1. Đặt độ dài cnh hình vuông ABCD a thì
2.==AC BD a
Áp dụng định lí Ptôlêmê cho t giác PADC, ta có:
( ) ( )
...
. 2 1 .
=+
= +
PD AC PACD PC AD
PD a a PA PC
Áp dụng định Ptôlêmê cho t giác PBCD, ta có:
...PC BD PBCD PD BC=+
( ) ( )
. 2 2 .PC a a PB PD = +
T ( 1 ) và (2), suy ra:
PB
PA PC PD
PCPD+
+
=
2. Áp dụng định lí Ptôlêmê cho t giác ni tiếp thì ta có:
.ac bd pq+=
Áp dng bất đẳng thc Bu-nhi-a-cp-xki ta có:
( )
( )( ) ( )( )
2
2 2 2 2 2 2 2 2 2 2
ac bd a b c d p q a b c d+ + + + +
Suy ra:
2 2 2 2
..pq a b c d + +
Du bng xy ra khi:
ab
cd
=
3. Kéo dài AI cắt đường tròn ngoi tiếp tam giác ABC ti D.
Ta có:
.BAD DAC DB DC= =
I là tâm đường tròn ni tiếp tam giác ABC nên:
( )
( )
1
2
1
2
BID IBA BAI B A
IBD IBC CBD B A
= + = +
= + = +
Suy ra
DIB
cân ti D nên
.DI DB=
Áp dụng định lý Ptôlêmê cho t giác ABDC ni tiếp:
( ) ( )
...
. .
AD BC ABCD AC BD
BD AB AC DI AB AC
=+
= + = +
Suy ra:
ABAD
ID BC
AC+
=
Do
AOD
cân ti O nên
90AIO
2. 2. 2
2 .
AD AB AC
AI ID AI ID ID AD ID
ID BC
AB AC BC
+
+ =
+
4. Ly E, F thuộc đường tròn sao cho
, .CDB ADE ADB DCF==
Khi đó
, , , .AE BC FD AB EC AB BF AD= = = =
Áp dụng định Ptôlêmê cho hai t giác ni tiếp AECD
BCDF ta có:
( )
( )
. . .
. . 1
. . .
. . 2
AC ED AE CD AD EC
BC CD AD AB
BDCF BC DF BF CD
BC AB ADCD
=+
=+
=+
=+
Mt khác:
CDE CDB BDE=+
ADE BDE ADB FCD= + = =
Do đó:
FDC FDE CDE FCE FCD ECD= + = + =
suy ra
.ED FC=
T (1), (2), (3) suy ra:
. . .
. . .
BC CD AB BD AC ED AC
BC BA DC DA BDCF BD
+
==
+
5. H BH vuông góc vi MD.
Ta có:
; ;MCA CBA MDA DBA MCA MDB CBD= = + =
180CMD CBD CMD MCA MDB + = + + =
Suy ra BCMD là t giác ni tiếp.
Áp dụng định lý Ptôlêmê ta có:
( )
. . . 1MC BD MD BC MBCD+=
Do
CBD
12
O AO
nên:
1 2 1 2
BC BD CD
O A O A OO
==
Hay
1 2 1 2
BC BD CD
R R OO
==
Đặt
1 2 1 2
BC BD CD
k
R R OO
= = =
Suy ra:
1 2 1 2
. , . , BC k R BD k R CD kOO= = =
(2)
T (1) và (2) suy ra:
2 1 1 2
. . .+=MC R MD R MBOO
hay
12
1 2 1 2
.MBOO
MC MD
R R R R
+=
Do đó
12
MC MD
RR
+
đạt giá tr ln nht khi và ch khi MB ln nht
Tam giác BMH luôn t đồng dng vi chính khi d thay đổi, nên BM ln nht khi ch khi BH
ln nht, mà
2
2. .BH BD R
Du bng xy ra khi BD đối xng qua
2
,O
khi đó
12
//d OO
6. Kéo dài AI ct BC(O) ti D; N.
Khi đó N là điểm chính gia cung BC (không cha A).
Ta có:
( )
1.BN NC=
Li có:
( )
2IBN BIN BN IN= =
Do
OI AN
suy ra
( )
1
3
2
IA IN AN==
T (1), (2), (3)
( )
4BN NC IN IA = = =
Áp dụng định lí Ptôlêmê cho t giác ni tiếp ABNC ta có:
. . .BN AC AB NC BC AN+=
T (4)
( ) ( )
2 . 2 5BN AC AB BN BC AC AB BC + = + =
Áp dng tính chất đường phân giác trong tam giác ABD, ACD và (5) ta có:
2
2
AB IA AC AB AC AB AC BC
BD ID CD BD CD BC BC
++
= = = = = =
+
Vy
2
IA
ID
=
(6).
Mt khác G là trng tâm ca tam giác suy ra
2
AG
GM
=
(7).
T (6),(7)
2 = =
IA AG
ID GM
Suy IG song song vi BC nh lý Ta-Lét đảo).
7.
a) Do các t giác BEPD, ABCQ ni tiếp, nên
EDP EBP ABQ ACQ= = =
(1) và
( )
180
180 2
EPD EBD
ABC AQC
=
= =
T (1) và (2) suy ra
AQC
œ
.EPD
Điu phi chng minh.
b) Theo kết qu phn a, ta có
+
===
+
QA QC CA QA QC
PE PD DE PE PD
Suy ra
( ) ( ) ( )
...QA QC DE PE PD AC PE PD BD+ = + = +
(3)
Áp dụng định lý Ptôlêmê cho t giác BEPD ni tiếp, ta được
( )
. . . .BP ED BE PD EP BD PD PE BD= + = +
(4)
T (3) và (4) suy ra
( )
..QA QC ED BP ED+=
hay
QA QC BP+=
(điều phi chng minh).
8.
ACB CAD RPQ==
BAC PRQ=
nên
ABC
đồng dng vi
RQP
(g.g)
AB BC AC
RQ QP RP
= =
Suy ra;
..
;
ABQP AC PQ
RQ RP
BC BC
==
Áp dụng định Ptôlêmê cho t giác APQR ta
đưc:
. . .APQR AR PQ AQ PR+=
. . .
..
. . .
AP ABQP AC QP
ARQP AQ
BC BC
AP AB AR QP AQ AC
+=
+ =
. . .AP AB AR AD AQ AC + =
(vì
)BC AD=
9
Lấy điểm K trên đường thng BN sao cho
BCK BMA=
Khi đó
BMA
BCK
Suy ra
( )
1
AB BM AM
BK BC CK
==
AB BK
MB BC
=
Mt khác, d thy
ABK MBC=
T đó
ABK
,MBC
dẫn đến
( )
2
AB BK AK
MB BC CM
==
Cũng từ
BMA
BCK
ta
,CKN MAB NAC==
suy ra A, N, C, K cùng nm trên một đường
tròn. Áp dụng định lý Ptôlêmê cho t giác ni tiếp ANCK ta được:
( )
. . . 3AC NK AN CK CN AK=+
Nhưng từ (1) và (2) thì:
..
;
.
;
AM BC AB CM
CK AK B
AB BC
BM
K
BM BM
= = =
Nên t (3)
( )
. . .AC BK BN AN CK CN AK = +
. . . . .
.
AB BC AN AM BC CN ABCM
AC BN
BM BM BM


= +
. . . . . . . .AB AC BC AN AM BC CN ABCM BN BM AC= + +
. . .
1
. . .
M AN BM BN CM CN
AB AC BA BC CA B
A
C
+ =
CH ĐỀ 2. ĐƯNG THNG SIMSON
A.Kiến thc cn nh
Robert Simson mt nhà toán học người Scotland, giáo toán hc của đại hc Glasgow. Ông sinh ngày
14 tháng 10 năm 1687 tại West Kibride và mất ngày 1 tháng 10 năm 1768 ti Glasgow.
Robert Simson vn mt thy thuốc nhưng lại ưa thích hình học. Ông say mê toán hc thi c đại Hy Lp
ch ít thích t tìm nhng kết qu mi. Sau đây là một định lý mang tên ông.
Định 1. Cho tam giác ABC ni tiếp đường tròn
(O )
M các điểm bt k trên
(O )
. Gi D, E, F ln
t là hình chiếu vuông góc của M trên các đường thng AB, BC, CA. Chng minh D, E, F thng hàng.
Đưng thẳng đi qua D, E, F có tên là đường thng Simson ng vi điểm M ca
ABC
Gii
Chứng minh. Xét trường hp
ABC
nhn và
MBA MCA
(Các trưng hp
khác chứng minh tương tự)
Khi đó D thuộc tia đối của tia BA, E và F tương ứng nm trên cnh BC, CA.
Vì các t giác MDBE, ABMC và MCFE ni tiếp nên
MED MBD ACM 180 MEF= = =
MED MEF 180 DEF 180 + = =
Do đó D, E, F thẳng hàng (dpcm)
Định 2. Cho
ABC
một điểm M. Gọi D, E, F tương ng hình chiếu vuông góc ca M trên các
đường thng AB, CA, AB. Biết rằng ba điểm D, E, F thng hàng. Chng minh rng M nằm trên đường
tròn ngoi tiếp
ABC
Gii
Không mt tính tổng quát, ta xét trường hợp điểm M nm trong góc
BAC
Các t giác BEMD, CMEF là t giác ni tiếp nên:
BMD BED; CMF CEF==
Ta li có:
BED CEF=
ối đỉnh)
BMD CMF=
T giác ADMF ni tiếp nên
A DMF 180+ =
A DMB BDF 180 A CMF BDF 180 + + = + + =
Do đó tứ giác ABMC ni tiếp.
Suy ra M nằm trên đường tròn ngoi tiếp
ABC
Bây gi ta vn dụng định lí trên để gii mt s ví d sau.
B.Mt s ví d
d 1. Cho
ABC
nhn ni tiếp đường tròn
(O )
. M một điểm bt k trên
(O )
, H trc tâm
ABC
. Chng minh rằng H và các điểm đối xng ca M qua AB, BC, CA thng hàng.
Gii
Tìm các gii. Nếu gi E, I, F hình chiếu của M trên đường thng AB, BC, CA P, Q, R các
điểm đối. xng ca M qua AB, BC, CA thì E, I, F thẳng hàng (đường thng Simson). T đó suy ra
P, Q, R thng hàng. Do vy ch cn chng minh P, H, Q thng hàng.
Trình bày li gii
Cách 1. Xét điểm M thuc cung nh BC.
Gi E, I, F lần lượt là hình chiếu của M trên đường thng AB, BC, CA.
Gi P, Q, R lần lượt là điểm đối xng của M qua các đường
thng AB, BC, CA.
Ta có:
APB AMB ACB BHD= = =
APBH là t giác ni tiếp
BHP BAP=
BHP BAM=
(1)
Chứng minh tương tự ta có :
CHQ CAM=
(2)
T (1) và (2) suy ra :
PHB BHC CHQ BAM BHC CAM 180+ + = + + =
P, H, Q thng hàng.
Tương tự : H, Q, R thng hàng
H, P, Q, R thng hàng.
Cách 2.
Gi BH, CH cắt đường tròn
(O )
tại điểm G, J.
Khi đó dễ dàng chứng minh được : G, J đi xng vi H
qua AC và AB.
T đó, ta các t giác MPJH, MRGH các hình thang
cân.
Suy ra
PHJ MJH MAC; RHG MGH MAB= = = =
Do đó :
PHJ JHG RHG MAC JHG MAB 180+ + = + + =
Vậy ba điểm P, H, R thng hàng (1)
Mà E, I, F thẳng hàng (đường thng Simson)
T đó suy ra P, Q, R thẳng hàng (2)
T (1) và (2) suy ra P, H, Q, R thng hàng.
Nhn xét.
Đưng thng PR này có tên là đường thng Steiner.
Da vào bài toán trên, bn có th giải được bài toán sau:
- Cho
ABC
nhn ni tiếp đưng tròn
(O )
. M một đim bt kì trên
(O )
. Gi P, Q, R
điểm đi xng vi M qua đường thng AB, BC, CA. Chng minh rng P, Q, R thng hàng
và xác định v trí của điểm M trên cung nh BC để PR đạt giá tr ln nht.
- Cho
ABC
, Mđiểm trên đường tròn ngoi tiếp tam giác. Gi K, P, Q lần lượt các điểm
đối xng ca M qua BC, CA, AB. Chng minh rằng các điểm P, K, Q nm trên một đường
thẳng đường thẳng đó luôn đi qua một điểm c định không ph thuc vào v trí ca
điểm M thay đổi trên đường tròn ngoi tiếp
ABC
(Olympc Toán Nht Bn, năm 1996).
d 2. Cho tam giác ABC ni tiếp đường tròn
(O;R )
, M điểm thuc cung BC không cha
đỉnh A. Gi D, E, F hình chiếu ca M lần lượt trên các cnh BC, CA, AB. Chng minh rng:
BC CA AB
MD ME MH
=+
(Thi Vô Địch Mỹ, năm 1979)
Gii
Tìm cách gii. Hình v dạng đường thẳng Simson, do đó H, D,
E thng hàng. Do yêu cu ca kết lun, nên ta cn tìm cp tam giác
đồng dạng để suy ra được:
CA x AB y
;
ME MD MH MD
==
. T các góc ca các t giác ni tiếp, ta tìm được
ng chng minh.
Trình bày li gii
Theo bài toán Simson thì H, D, E thng hàng, các t giác MHBD, MDEC ni tiếp nên
MEH MCB, MBC MHE==
AMH CMD
suy ra:
AH CD
MH MD
=
AME BMD
suy ra:
AE BD
ME MD
=
Do đó:
AH AE CD BD BC AB BH AC CE BC
MH ME MD MD MD MH MH ME ME MD
+ = + = + + =
Mt khác
BHM CEM
suy ra:
BH CE
MH ME
=
t đó suy ra điều phi chng minh.
Nhn xét. Da vào bài toán trên, bn có th giải được bài toán sau:
Cho tam giác ABC ni tiếp đường tròn
(O;R )
, M điểm thuc cung BC không chứa đỉnh
A. Gi D, E, F hình chiếu ca M lần lượt trên các cnh BC, CA, AB. Chng minh rng:
1 1 1
MD ME MH
=+
Cho tam giác ABC (
AB BC CA
) ni tiếp đường tròn
(O;R )
, M điểm bt k thuc
đưng tròn
(O;R )
. Gi D, E, F nh chiếu ca M lần lượt trên các đường thng BC, CA,
AB. Tìm v trí điểm M để
BC C A AB
MD ME MH
++
đạt giá tr nh nht.
Ví d 3. Cho ABCD là mt t giác ni tiếp. Gi P, Q và R tương ứng là chân các đường vuông góc
h t D xuống các đường thng BC, CA AB. Chng minh rng
PQ QR=
khi ch khi các
đưng phân giác ca các góc
ABC
ADC
ct nhau ti một điểm nằm trên đường thng AC.
(Thi Toán Quc tế IMO ln th 44, ti Nht Bn, thi ngày 17/7/2003)
Gii
Tìm cách gii. Khi v hình bài toán, chúng ta nhn thy rng bóng dáng của bài toán đường
thng Simson, do vy chúng ta cn s dng P, Q, R thng hàng. Mt khác, chúng ta thy rng
PQ QR=
thì
1
RQ PR
2
=
. Vy phải chăng các đường phân giác ca các góc
ABC
ADC
ct nhau
ti một điểm E nằm trên đường thng AC s to ra tính cht t l đon thng? T đó ta có lời gii
sau:
Trình bày li gii
T đề bài ta P, Q, R thuc một đường thng (đường thng
Simson).
Trên tia đối ca tia DA lấy điểm M sao cho
DM DA=
Theo tính chất đường phân giác ca tam giác, ta E thuc AC khi
và ch khi
AB DA AE AB DM
BC DC EC BC DC

= = =


Mt khác
ABC MDC=
(cùng bù vi
ADC
) nên
ABC MDC
ACB MCD, CAB CMD; = =
Mà t giác AQDR ni tiếp nên
DQR DAR=
RDQ CMD( CAB)==
RQ DR DR
DQR MAC(g .g)
AC MA 2AD
= =
(4)
D thy
ADC RDP( g.g )
nên
RP RD
AC AD
=
(5)
T (4) và (5) suy ra
1
RQ RP RQ QP
2
= =
Nhn xét. Ngoài ra chúng ta có th gii trc tiếp như sau:
Ta có: P, Q, R thng hàng.
T các t giác ni tiếp CDQP, AQDR
Suy ra:
DCA DPR( g.g )
Tương tự, ta có:
DAB DQP( g.g ), DBC DRQ( g.g )
Do đó :
DC BC PQ
.
DA BA QR
=
Suy ra
DC BC
PQ QR
DA BA
= =
Điều đó tương đương vi chân đường phân giác ca các góc
ABC
và
ADC
ct nhau ti một điểm
nằm trên đường thng AC.
d 4. T đim P trên cung BC của đường tròn ngoi tiếp
ABC
k PM, PL, PK lần lượt vuông
góc vi AB, AC, AC. Gọi P’ điểm đối xng vi P qua O. K P’M’, P’L’, P’K’ lần lượt vuông góc
vi AB, AC, AC. Chng minh rng :
ML M' L'
Gii
Vn dng tính chất đường thng Simson ta : M, K, L thng hàng
và M’, K’, L’ thẳng hàng.
T giác AM’P’L’ là tứ giác ni tiếp
AM' L' AP' L'=
(1)
PP’ là đường kính
PAC AP' L' ( 90 CAP' ) = =
(2)
Mt khác
PAC PBC=
(3)
T giác PMBK là t giác ni tiếp
PMK PBC=
(4)
T (1), (2), (3), (4) suy ra :
PMK AM ' L'=
PMK AML 90 M' ML MM' L 90+ = + =
Suy ra :
ML M' L'
C.Bài tp vn dng
1. Cho
ABC
nhn ni tiếp đường tròn
(O )
. H trc tâm
ABC
, M một điểm nm trên cung
nh AC. Gi E, F lần lượt là hình chiếu vuông góc của M trên các đường thng AB, BC.
a) Chng minh rằng EF đi qua trung điểm ca MH.
b) Xác định v trí của M để EF ln nht.
2. Cho hai đường tròn
(O )
và
(O')
ct nhau tại hai điểm phân bit A B. Mt cát tuyến thay đổi
qua A ct
(O )
(O')
tương ng ti C D. Gi E F th t hình chiếu vuông góc ca B lên
tiếp tuyến ca
(O )
ti C tiếp tuyến ca
(O')
ti D. Chng minh rng EF tiếp xúc một đường
tròn c định.
3. Cho ngũ giác ABCDE nội tiếp một đường tròn. Gi M, N, P, Q, R, S, T U lần lượt hình
chiếu vuông góc của E trên các đưng thng AB, BC, CD, DA, MN, NP, PQ QM. Chng minh
R, S, T, U thng hàng.
4. Cho đường tròn
(O )
một điểm A c định nm ngoài
(O )
. M mt điểm thay đổi trên
đưng thng qua A vuông góc vi AO. Gi MB, MC các tiếp tuyến ca
(O )
(B, C các tiếp
đim). K
AE MB, AF MC ( E MB; F MC )
. Chng minh rằng EF đi qua một điểm c định.
5. Cho
xOy
, lấy điểm A c đnh thuc tia phân giác ca
xOy
. V đưng tròn
( I )
qua O A ct
Ox, Oy lần lượt ti B C v hình bình hành OBMC. Chng minh rng M thuc một đường
thng c định.
6. Cho tam giác nhn ABC ni tiếp đường tròn
(O )
trc tâm H. D là điểm trên cung nh BC.
Lấy điểm E sao cho ADCE hình bình hành K trc tâm ca tam giác ACE. Gi P, Q lần lượt
là hình chiếu ca K trên BC và AB. Chng minh rằng PQ đi qua trung điểm ca HK.
7. Cho
ABC
nhn ni tiếp đường tròn
(O )
. M điểm thay đổi trên cung nh BC. K
ME AB, MF AC (E AB,F AC )
a) Xác định v trí của M để trung điểm ca EF nằm trên đon thng BC
b) K
AP MB, AQ MC (P MB, Q MC )
. Chng minh rằng PQ đi qua một điểm c định.
8. Cho t giác ABCD ni tiếp đường tròn
(O )
. Gi H, K th thình chiếu vuông góc ca B trên
AC, CD. Gi M, N th t trung điểm ca AD, HK. Chng minh rng tam giác BMN tam giác
vuông.
NG DN GII - ĐÁP SỐ
1.
a) Xét trường hp
BAM BCM
(các trường hp khác chứng minh tương tự)
Khi đó E thuộc tia đối ca tia AB và F nm trên cnh BC
H
MI AC (I AC)⊥
Ta được EF đường thng Simson ng với điểm M ca
ABC
Hơn nữa, I nm gia E và F.
Gi P, Q theo th t điểm đối xng ca M qua các
đưng thng AB, AC
APB AMB ACB 180 AHB= = =
nên t giác AHBP ni
tiếp
AHP ABP ABMT = =
. Tương tự
CHQ CBM=
Suy ra
AHP AHC CHQ ABM AHC CBM 180+ + = + + =
Do đó P, H, Q thng hàng.
IE đường trung bình ca
MPQ
nên IE đi qua trung điểm của MH hay EF đi qua trung
đim của MH (đpcm).
Nhn xét. Có th dùng đường thẳng Steiner để chng minh.
b) Vì
MEI MAI=
MFI MCI=
nên
MEF MAC ( g.g )
Suy ra
EF EM
1
AC AM
=
Do đó EF lớn nht bng AC và xy ra khi và ch khi BM là đường kính ca
(O)
2.
Đặt
T CE DF=
Gi s
TCB TDB
H
BI CD ( I CD )⊥
CBD CBA DBA TCD TDC= + = +
nên
CBD CTD 180+ =
Do đó tứ giác TCBD ni tiếp.
Suy ra EF là đường thng Simson ng với điểm B ca
TCD
Do đó I, E, F thẳng hàng I thuộc đường tròn đường kính
AB.
Mà t giác BICE ni tiếp nên
BIE BCE BAE==
Vy EF tiếp xúc với đường tròn đường kính AB c định (đpcm).
3.
H
EI AC ( I AC )⊥
Đưng thng Simson ng vi điểm E vi tam giác ABC cho
ta ba điểm M, N, I thng hàng.
Đưng thng Simson ng vi điểm E vi tam giác ACD cho
ta các ba điểm P, Q, I thng hàng.
Ta A, I, Q, E, M cùng thuc đường tròn đường kính AE,
nên vn dụng đường thng Simson ng với đim E vi tam
giác IMQ cho ta các cặp điểm R, U, T thng hàng.
Tương tự R, S, T thng hàng, ta có đpcm.
4
Ta có
MAO MBO MCO 90= = =
Suy ra năm điểm M, A, C, O, B cùng thuộc đường tròn đường kính MO.
K
AI BC
. Xét đường tròn đường kính MO, ta có:
AE MB, AF MC, AI BC
Suy ra E, F, I thẳng hàng (đường thng Simson).
Gọi H, J là giao điểm ca BC vi AO và OM, K là giao điểm ca
EI và AO.
Ta có:
OJH OAM ( g.g )
suy ra:
OJ.OM OH.OA=
Ta có:
OJB OBM ( g.g )
suy ra:
2
OJ.OM OB=
Do đó :
2
2
OB
OH.OA OB OH
OA
= =
không đổi
Vậy H là điểm c định.
T giác AICF ni tiếp nên
KIA FCA=
(1)
A, M, B, O, C cùng thuc một đường tròn nên
ACM AOM=
(2)
T (1), (2) suy ra :
KIA AOM=
AOM KAI=
(so le trong) nên
KIA KAI KA KI= =
AIH
vuông có
KA KI=
d dàng suy ra
AK KH=
Suy ra K là điểm c định.
5.
A thuc tia phân giác ca
xOy
nên
AC AB=
. Suy ra
IA BC
K
AH Ox, AK Oy⊥⊥
thì K, H c định K, E, H thng
hàng nên điểm E thuc HK c định.
Mà hình bình hành OBMC có
OM 2OE=
nên M c định.
Suy ra M thuc đường thng d c định song song vi
đưng thng HK và cách HK mt khoảng không đổi.
6.
Theo gi thiết ta có ADCE là hình bình hành nên
ADC AEC=
K là trc tâm ca
AEC
nên
EK AC
Mt khác
AKC AEC 180+ =
nên
AKC ADC 180+ =
Suy ra t giác ADCK ni tiếp, t đó
K (O)
EK ct AC ti I
Do đó P, Q, I thẳng hàng (đường thng Simson).
Đưng thng AH cắt đường tròn
(O)
ti M và ct PQ ti N thì
MN KP, KQ AB, KP BC⊥⊥
Suy ra BQKP là t giác ni tiếp nên
QPK QBK AMK==
Do đó MPKN là tứ giác ni tiếp.
Do đó MPKN là hình thang cân nên
PMN KNM=
Mt khác
PH PM PMN PHM= =
nên
PHM KNM=
PH KN
, li có
NH KP
, suy ra HPKN là hình bình hành.
Do đó NP cắt HK tại trung điểm ca mỗi đường.
Do đó PQ đi qua trung điểm ca HK.
7.
a) K
MD BC
suy ra E, D, F thẳng hàng (đường thng Simson).
T các t giác ni tiếp MDBE, MDFC ta có:
MED MBD; MFD MCD==
EF MF
MEF MBC ( g.g )
BC MC
=
DF MF
ED DF
NC MC
= =
MDF MNC ( g.c.g )
= =MDF MNC DCF NMC
M thuc cung cha góc BAC dựng trên đoạn NC
M thuộc giao điểm ca cung cha góc BAC dng trên
đon NC và cung nh BC.
Nhn xét. Ngoài ta có th da vào ví d 3, để gii.
b) K
AH BC
, suy ra P, H, Q thẳng hàng (đường thng Simson)
PQ luôn đi qua điểm H c định.
8.
K
BE AD ( E AD)⊥
Ta có
BH AC, BK CD⊥⊥
E, H, K thng hàng.
T giác BEDK ni tiếp
EDB EKB=
T giác BHKC ni tiếp
BHK BCD 180 + =
Mt khác
BAD BCD 180+ =
BAD BHK BHK BAD ( g.g) =
MA MD, NH NK AMB HNB= =
AMB BNH BNE = =
Do đó tứ giác BEMN ni tiếp.
MEB 90 BNM 90 BN MN= =
CHUYÊN ĐỀ 9. QU TÍCH (TÌM TP HỢP ĐIỂM)
A. Kiến thc cn nh
1. Các qu tích cơ bản
Để tìm qu tích trong mt phẳng, người ta thường da vào các qu tích bản. Mt s qu tích sau đây
thường được mọi người tha nhn là qu tích cơ bản:
Qu tích 1: Qu tích những điểm cách đều hai điểm A và B c định là đường trung trc của đoạn thng AB.
Qu tích 2: Qu tích những điểm cách đều hai cnh ca một góc là đường phân giác của góc đó.
Qu tích 3: Qu tích những điểm cách đều đưng thng xy c định mt khong a cho trước hai đường
thng song song vi xy và cách xy mt khoảng a cho trước.
Qu tích 4: Qu tích nhng điểm cách đều điểm O c định mt khong R cho trước là đưng tròn tâm
O và bán kính bng R.
Qu tích 5: Qu tích những điểm nhìn đoạn thng AB c định dưới mt góc
không đổi (
0 180
) là
hai cung cha góc
dựng trên đoạn thng AB.
Đặc bit, nếu
90
=
thì ta nhận được.
Qu tích 5a: Qu tích nhng đim nhìn đoạn thng AB c định i một góc vuông đường tròn đường
kính AB.
2. Các bước gii mt bài toán qu tích
Mun chng minh qu tích (tp hợp) các điểm M tha mãn tính cht
một hình H nào đó, ta phải chng
minh hai phn:
Phn thun: Mọi điểm có tính cht
đều thuc hình H.
Gii hạn. Xem điểm M ch thuc mt phn
1
H
ca hình H hay c hình H.
Phần đảo: Mọi điểm thuc hình H hoc thuc phn
1
H
(nếu có gii hạn) đều có tính cht
.
Kết lun: Qu tích (tp hợp) các điểm M có tính cht
là hình H (hoc thuc phn
1
H
).
B. Mt s ví d
Ví d 1. Cho nửa đường tròn đường kính BC. Một điểm A di động sao cho tam giác ABC có ba góc nhn
trng tâm G ca tam giác nm trên nửa đường tròn đó. Tìm qu tích điểm A.
Gii
Tìm cách gii
Nếu gi BP, CQ đường trung tuyến, ta luôn
AP PC=
AQ QB=
. Nếu ly E đối xng vi C qua B
thì BP luôn song song vi AE, F đối xng vi B qua C thì CQ luôn song song vi AF, E, F c định. Khi
G di động thì
90EAF =
không đổi nên ta tìm được điểm A di chuyn trên nửa đường tròn đường kính EF.
G trng tâm tam giác ABC, nếu gi O trung điểm BC thì A, G, O thng hàng. Mt khác G trng
tâm nên
3.OA OG=
không đổi. T đó suy ra A di chuyển trên đường tròn
( )
;3OR
.
Trình bày li gii
Phn thun.
Cách 1. Trên đường thng BC ly hai đim E, F sao cho B trung
điểm CE, C là trung điểm BF.
Ta có:
3EF BC=
c định (1)
Gi PQ lần lượt là giao điểm ca BGAC; CGAB
CQ là đường trung bình ca
ABF
nên
//CQ AF
.
BP là đường trung bình ca
ACE
nên
//BP AE
CQ BP
nên
90AF AE EAF =
(2)
T (1) và (2), suy ra A di động trên đường tròn đường kính EF.
Cách 2. Gi O là trung điểm BC
O
c định và A, G, O thng hàng.
G là trng tâm
ABC
nên
3
2
OA OG BG==
. Suy ra A di động trên đường tròn tâm O bán kính
3
2
BG
.
Gii hn. Do
ABC
nhn nên A di động trên cung nh MN (tr hai điểm M, N).
Phần đảo.
Lấy điểm A biết bt thuc cung nh MN, gi G giao điểm ca OA vi nửa đường tròn đường kính BC
AO
là đường trung tuyến ca
ABC
.
Ta có
11
23
OG BG OA G= =
là trng tâm
ABC
.
Kết lun. Vy tp hợp điểm A là cung nh MN (tr hai điểm M, N).
d 2. Cho đường tròn tâm O đường kính AB c định, BC dây cung bất kì. Trên tia đối ca tia CB ly
điểm D sao cho
CD BC=
. Gi P là giao điểm ca ACDO. Tìm qu tích điểm P.
Gii
Tìm cách gii. Ta nghiên cu tính cht của điểm P.
Ta AC PO là hai trung tuyến ca
ABD
, do đó
1
3
CP
AC
=
; li
90ACB =
nên nếu dng
//PE CB
(vi
E AB
) thì
90APE =
1
3
BE
AB
=
, như vậy E cũng một điểm c định
90APE =
không đổi.
Như vậy qu tích của điểm P là xác định được.
Trình bày li gii.
Phn thun. Ni AD, AC DO hai trung tuyến ca
ABD
nên P là trng tâm tam giác, suy ra
1
3
CP
AC
=
.
Trên đoạn thng AB xác định điểm E sao cho
1
3
BE
AB
=
thì điểm E
điểm c định.
Ta có
1
3
CP BE
AC AB

==


nên
//PE CB
nh lý Ta-lét đảo).
90APE ACB APE = =
.
A; E là hai điểm c định nên tp hợp điểm P là đường tròn có đường kính AE.
Phần đảo. Lấy điểm P bt thuộc đường kính AE. Gi C giao điểm th hai ca tia AP với đường tròn
()O
. Gi D là giao điểm ca hai tia BCOP.
Ta có
90 ; 90ACB APE= =
(góc ni tiếp chn nửa đường tròn)
Suy ra
//
DP BE
BC EP
DO BO
=
.
1 1 2 2
3 2. 3 3 3
BE BE BE DP
AB BO BO DO
= = = =
.
ABD
DO đường trung tuyến;
2
3
DP
P
DO
=
trng tâm
ABD AC
đường trung tuyến
CD CB=
.
Kết lun. Vy qu tích đim P là đường tròn đường kính AE.
d 3. Cho đường tròn (
;RO
) đim P c định nằm trong đường tròn). Dây cung AB thay đổi luôn đi
qua P. Tiếp tuyến ti AB với đường tròn ct nhau ti M. Tìm qu tích điểm M.
Gii
Tìm cách gii. Nhn thy I giao điểm ca AB MO thì I thuộc đường tròn đường kính OP
2
.MI MO R=
. Do vy, khai thác yếu t không đổi này, ta th nhn thy nếu H nh chiếu ca M trên
đường thng OP thì
2
.OPOH R=
không đổi, suy ra H c định. T đó ta có lời gii.
Trình bày li gii.
Phn thun. Gi H là hình chiếu ca M trên đường thng OP.
Gi I là giao điểm ca ABMO.
Suy ra
AB MO
t đó ta có
OHM OIP
(g.g)
. .OP
OM OH
OM OI OH
OP OI
= =
(1)
Mt khác
OAM
vuông ti A có:
AI MO
nên
2
.OA OM OI=
(2)
T (1) và (2) suy ra
2
.OH OP OA=
2
R
OH
OP
=
không đổi
M
thuộc đường thng d vuông góc vi OP tại điểm H
cách O mt khong cách
2
R
OH
OP
=
.
Phần đảo. Trên đường thng d ly điểm
M
bt kì. T
M
k
tiếp tuyến
,
M A M B
. Đường thng
AB

ct
MO
ti
I
.
Gi s OH ct
AB

ti
P
Ta có
2
..OP OH OI OM R
==
2
R
OP P P
OH

=
.
Kết lun. Qu tích ca điểm M là đường thng d vuông góc vi OP tại điểm H tha mãn
2
R
OH
OP
=
.
d 4. Cho nửa đường tròn đường kính AB c định. C một điểm bt thuc nửa đường tròn. phía
ngoài tam giác ABC, v các hình vuông BCDEACFG. Gi Ax, By là các tiếp tuyến ca nửa đường tròn.
a) Chng minh rng khi C di chuyn trên nửa đường tròn đã cho thì đưng thng ED luôn đi qua một điểm
c định và đường thng FG luôn đi qua điểm c định khác.
b) Tìm qu tích các điểm EG khi C di chuyn trên nửa đường tròn đã cho.
c) Tìm qu tích của các điểm DF khi C di chuyn trên nửa đường tròn đã cho.
(Thi Hc sinh gii lp 9, tnh Tha Thiên Huế, năm học 2004 2005)
Gii
a) Gi K là giao điểm ca AxGF, I là giao điểm ca ByED.
Ta có:
90= = BEI BCA
=EBI CBA
(góc có các cạnh tương ứng vuông góc)
BE BC=
.
Do đó:
BEI BCA BI BA = =
By c định, suy ra điểm I c định.
Tương tự, K c định.
Vy khi C di chuyn trên nửa đường tròn
()O
thì đường thng ED
đi qua điểm I c định và đường thng GF đi qua điểm K c định.
b)
Tìm qu tích điểm E.
Phn thun. Ta B I c định (chng minh câu a)
90BEI =
(vì BCDE hình vuông) suy ra E
thuc nửa đường tròn đưng kính BI (bên phi By).
Phần đảo. Lấy điểm E bt kì thuc nửa đường tròn đường kính BI (bên phi By). Trên tia EI ly điểm D sao
cho
ED BE=
.
Dng hình vuông
BEDC BC BE=
.
Ta có
( )
90 ;ABC EBD CBI BA BI= = =
(chng minh câu a)
ABC IBE =
(c.g.c)
90ACB IEB = =
C
thuc nửa đường tròn đường kính AB.
Kết lun. Vy qu tích các điểm E là nửa đường tròn đường kính BI (bên phi By).
Tìm qu tích điểm G.
Phn thun. Ta A K c định (chng minh câu a)
90AGK =
(vì ACFG hình vuông) suy ra G
thuc nửa đường tròn đường kính AK (bên trái Ax).
Phần đảo. Lấy điểm G bt thuc nửa đường tròn đường kính AK (bên trái Ax). Trên tia GK lấy điểm F
sao cho
GA GF=
.
Dng hình vuông
AGFC AC AG=
.
Ta có
( )
90 ;BAC KAG CAK BA KA= = =
(chng minh câu a)
ABC AKG =
(c.g.c)
90ACB AGK = =
C
thuc nửa đường tròn đường kính AB.
Kết lun. Vy qu tích các điểm G là nửa đường tròn đường kính AK (bên trái Ax).
c)
Tìm qu tích điểm D.
Phn thun. Ta
90ADI =
A, I c định nên điểm D thuc nửa đường tròn đường kính AI (bên trái
AI).
Phần đảo. Lấy điểm D bt thuc nửa đường tròn đưng kính AI (bên trái AI). Dng hình vuông BCDE
(th t đỉnh theo chiều kim đồng h).
Suy ra D, I, E thng hàng (vì DI, DE cùng vuông góc vi AD).
Ta có
( )
90 ;ABC EBD CBI BA BI= = =
(chng minh câu a)
ABC IBE =
(c.g.c)
90ACB IEB = =
C
thuc nửa đường tròn đường kính AB.
Kết lun. Vy qu tích các điểm D là nửa đường tròn đường kính AI (bên trái AI).
Tìm qu tích điểm F.
Phn thun. Ta
90BFK =
B, K c định nên điểm F thuc nửa đường tròn đường kính BK (bên phi
BK).
Phần đảo. Lấy điểm F bt thuc nửa đường tròn đường kính BK (bên phi BK). Dng hình vuông AGFC
(th t đỉnh theo chiều kim đồng h).
Suy ra G, F, K thng hàng (vì GK, FK cùng vuông góc vi BK).
Ta có
( )
90 ;BAC KAG CAK BA KA= = =
(chng minh câu a)
ABC AKG =
(c.g.c)
90ACB AGK = =
C
thuc nửa đường tròn đường kính AB.
Kết lun. Vy qu tích các điểm F là nửa đường tròn đường kính BK (bên trái BK).
C. Bài tp vn dng
1. Cho ba điểm A, B, C c định nằm trên đường thng d (B nm gia A C). Một đường tròn
()O
thay đổi
luôn đi qua A B, gi DE đường kính của đường tròn
()O
vuông góc vi d. CD CE cắt đường tròn
()O
lần lượt ti MN. Khi đường tròn
()O
thay đổi thì hai điểm MN di động trên đường c định nào?
2. Cho đường tròn
( ; )OR
đoạn thng AB c định nằm bên ngoài đường tròn
()O
. Gi C một điểm
chuyển động trên đường tròn. Tìm tp hp các trng tâm G ca tam giác ABC.
3. Cho đường tròn
()O
ni tiếp hình vuông PQRS. OA OB hai bán kính thay đi vuông góc vi nhau.
Qua A k đường thng Ax song song với đưnòg thẳng PQ, qua B k đường thng By song song với đường
thng SP. Tìm qu tích giao điểm M ca AxBy.
(Tuyn sinh lp 10, THPT chuyên tỉnh Phú Yên, năm học 2009 2010)
4. Cho đường tròn
()O
dây BC c định không qua tâm O, điểm A di chuyn trên cung ln BC. Trên tia
đối ca tia AB ly điểm D sao cho
AD AC=
. Gi M trung điểm ca CD. Hi M di chuyển trên đường
nào? Nêu cách dựng đường này và gii hn ca nó.
(Thi Hc sinh gii lp 9, tnh Tha Thiên Huế, năm học 2007 2008)
5. Cho đường tròn tâm O đường kính AB. Điểm M chuyển động trên đường tròn đó. Gọi Hhình chiếu ca
điểm M trên AB. Tìm qu tích tâm I của đường tròn ni tiếp tam giác OMH.
6. Cho góc vuông xOy điểm A c định trên tia Ox, điểm B chuyển động trên tia Oy. Dng hình vuông
ABCD nm trong góc xOy. Tìm tp hợp giao điểm I hai đường chéo ca hình vuông này.
7. Cho ba điểm A, B, C theo th t đó trên đưng thng d. V các nửa đường tròn đường kính AB, AC thuc
hai na mt phẳng đối nhau b đường thng d. Một điểm H chuyển động trên đoạn AB. Đường thng
vuông góc vi d H ct c hai nửa đường tròn nói trên lần lượt D E. Gi M giao điểm hai đường
thng DBEC. Tìm qu tích điểm M.
8. Cho đường tròn
( ; )OR
và tam giác cân ABC
AB AC=
ni tiếp đường tròn
( ; )OR
. K đường kính AI.
Gi M một điểm bt trên cung nh AC. Gi Mx tia đối ca tia MC. Trên tia đối ca tia MB ly điểm
D sao cho
MD MC=
.
a) Chng minh rng MA là tia phân giác ca góc BMx.
b) Gi K là giao th hai của đường thng DC với đường tròn
()O
. T giác MIKD là hình gì? Vì sao?
c) Gi G trng tâm ca tam giác MDK. Chng minh rng khi M di động trên cung nh AC thì G luôn nm
trên một đường tròn c định.
9. Cho nửa đường tròn tâm O đường kính AB. Gi C điểm chính gia ca nửa đường tròn. M điểm
chuyển động trên cung BC. Gi N giao điểm ca AM OC. Gi I tâm đường tròn ngoi tiếp tam giác
CMB. Tìm tp hợp điểm I.
NG DN GII - ĐÁP SỐ
1. Gi H, K lần lượt là giao đim ca CA vi DE EM. Do
A, B, C c định nên H c định.
CMK
CHD
có:
90MH= =
DCH
là góc chung.
Vy:
CMK CHD
(g.g)
CK CM
CD CH
=
..CK CH CM CD=
(1)
CMB
CAD
có:
CMB CAD=
(do t giác ABMD ni tiếp);
ACD
là góc chung.
Vy:
CMB CAD
(g.g)
..
CM CB
CM CD CACB
CA CD
= =
(2)
T (1) và (2)
.
..
CACB
CK CH CACB CK
CH
= =
(không đổi)
K
là điểm c định.
Tam giác CDEK là trc tâm nên DN cũng đi qua điểm K c định.
90DME DNE= =
(góc ni tiếp chn nửa đường tròn)
90KMC KNC = =
.
Vậy: Khi đường tròn
()O
thay đổi thì hai điểm M N di động trên đường tròn c định đường kính CK, vi
.CACB
CK
CH
=
.
2. Phn thun. Gi M là trung điểm AB
M
c định.
K
//GO OC
.
O OM
.
Ta có G là trng tâm nên
1
3
MG
MC
=
.
Ta có
//GO OC
.
Suy ra
O G MO MG
OG MO MC

==
nên
1
3
O G MO
OC MO

==
.
1
3
MO MO O

=
là điểm c định.
11
33
O G OC O G R

= =
.
Vậy điểm G thuộc đường tròn tâm
O
bán kính
1
3
R
.
Phần đảo. Ly
G
thuộc đường tròn tâm
O
bán kính
1
3
R
qua O k đường thng song song vi
OG

ct
đường thng
MG
ti
C
. Ta có:
1
3. ( )
3
O G MO O G
OC O G R C O
OC MO OC
= = = =

.
Kết lun. Vy tp hp các trng tâm G ca tam giác ABC là đường tròn tâm
O
bán kính
1
3
R
.
3. Kí hiệu như hình vẽ.
Phn thun. Ta có
90AOB AMB= =
(gi thiết)
t giác AOBM luôn ni tiếp
45AMO ABO = =
(vì
AOB
vuông cân ti O)
Suy ra M luôn nằm trên đường thẳng đi qua O to với đường PQ
mt góc
45
.
Trường hp B v trí
B
thì
M
nằm trên đường thẳng đi qua O
to vi PS mt góc
45
.
Gii hn.
*) Khi
AH
thì
MQ
, khi
AK
thì
MS
*) Trường hp B v trí
B
: khi
AH
thì
MP
, khi
AK
thì
MR
.
Phần đảo. Ly M bất kì trên đường chéo SQ (hoc
M
trên PR), qua M k đường thng song song với đường
thng PQ ct
()O
ti A.
K bán kính
OB OA
.
Ta thy t giác AOBM ni tiếp (vì
45AMO ABO= =
).
Suy ra:
90AMB AOB= =
.
/ / , / /AM PQ PQ PS MB PS⊥
.
Kết lun. Qu tích giao điểm M 2 đường chéo ca hình vuông
PQRS.
4. Tam giác ACD cân ti A nên
2BAC ADC=
(Góc BAC góc
ngoài ca tam giác ACD)
Gi I trung điểm ca BC, ta
(đường trung bình
ca tam giác BCD); nên
11
2 4 4
IMC BDC BAC BOC
= = = =
(
BOC
=
không đổi).
Do đó M chy trên cung tròn nhìn IC dưới góc
4
cùng phía với điểm A đối với đường thng BC không đổi.
Cách dng. Gi I là trung điểm ca BC. Dng tia OI cắt đường tròn
()O
ti N, ta có:
1
2
NBC BAC BDC IMC= = =
Dng tia
//IN BN
, dựng đường thng qua I vuông góc vi
IN
ct trung trực đoạn IC ti
1
O
Đưng tròn tâm
1
O
đi qua C là đường cn dng. Khi A chy trên cung ln BC ti trùng vi B thì D trùng
vi
0
D
trên tiếp tuyến Bt ca
()O
0
BD BC=
. Khi đó M trùng vi
0
M
là trung điểm ca
0
CD
.
Vy M ch di chuyn trên cung ln
0
CM
của đường tròn
1
()O
.
5. Phn thun. Xét vi M thuộc đường tròn sao cho
AM MB
.
Ta có
90HMO HOM+ =
(vì
HMO
vuông ti H) mà I là tâm đường tròn ni tiếp
HMO
.
Suy ra
1 1 1
.90 45
2 2 2
IMO IOM HMO HOM+ = + = =
135MIO =
.
OIM
OIA
OM OA=
;
;MOI AOI OI=
chung
= OIM OIA
(c.g.c)
135MOI AIO = =
OA c định;
135AIO =
I nm trên
cung cha góc
135
dựng trên đoạn OA.
Tương tự vi M thuộc đường tròn sao cho
AM MB
.
Phần đảo. bạn đọc t chng minh.
Kết lun. Vy qu tích điểm I là bn cung cha góc
135
dựng trên đoạn OA; OB.
6. Phn thun. T gíc AIBO là t giác ni tiếp vì có
180AIB AOB+ =
Suy ra
IOB IAB=
(hai góc ni tiếp cùng chn cung IB)
Do đó
45IOA=
nên OI là tia phân giác ca góc AOB.
Vậy điểm I chy trên tia phân giác ca góc xOy.
Gii hn. V hình vuông
11
AOC D
nm trong góc xOy.
điểm B ch chy trên tia phân Ox nên khi B trùng vi O thì C
trùng vi
1
C
, khi đó I trùng vi
1
I
giao điểm ca
1
OD
vi
1
AC
.
Phần đảo. Lấy điểm
I
thuc tia
1
It
. Ni
AI
.
Trên na mt phng b
AI
chứa điểm O, v tia
AB
(
B
thuc
Ox) sao cho
45I AB

=
.
Gi
,CD

lần lượt các điểm đối xng ca A và
B
qua
I
. Ch cn chng minh rng
I
giao đim hai
đường chéo ca hình vuông
AB C D
.
Kết lun. Tp hợp các điểm I là tia
1
It
thuc tia phân giác Ot ca góc xOy.
7. Phn thun. Đặt
2 , 2AB R AC R
==
thì R.
R
các đ
dài không đổi.
Trong tam giác vuông ADBAEC, ta có:
22
. 2 . ; . 2 .AD AB AH R AH AE AC AH R AH
= = = =
T đó suy ra
. 2 .AD AE AH RR
=
.
T giác ADME ni tiếp đường tròn vì
180ADM AEM+ =
.
Suy ra
AMD AED=
.
T đó
DAM HAE
(g.g).
Ta có:
AD AM
AH AE
=
.
Suy ra
. . 2AM AH AD AE AH RR
==
2AM RR
=
không đổi.
T đó điểm M chạy trên đường tròn tâm A bán kính
RR
.
Gii hn.H chuyển động trên đoạn AB nên:
- Khi H trùng vi A thì D trùng vi A, khi đó M trùng vi
1
M
như hình vẽ.
Khi H trùng vi B thì M trùng vi
2
M
như hình vẽ.
Vy nên H chy trên cung
12
MM
.
Phần đảo. Lấy điểm
M
thuc cung
12
MM
. Các tia
MB
CM; ct các nửa đường tròn đường kính AB,
AC lần lượt
,DE

. Các bn có th t chng minh
DE

vuông góc vi AB.
Kết lun. Qu tích M là cung
12
MM
thuộc đường tròn tâm A bán kính
RR
.
8.
a)
=
2
AB
AMB
(góc ni tiếp
()O
chn AB).
= = = =180 180
2 2 2
ABC AC AB
AMx AMC
Vy:
AMB AMx=
hay MA là tia phân giác ca
BMx
.
b) Tam giác MCD cân
2
BMC
MCD MDC = =
(góc
ngoài ca tam giác)
Li có tam giác ABC cân
I là điểm chính gia ca cung BC
2
BMC
IMC IMB = =
.
Vy
MCD IMC=
IM
song song vi CD
MCD MDC BMI BI MK= = =
//MIK IMB IK MD =
.
Vy MIKD là hình bình hành.
c) D thuộc đường tròn
( ; )A AC
.
Gi N là điểm trên AI sao cho
1 2 2
3 3 3
NA AI NG AD AC= = =
(hng s)
G
thuộc đường tròn
2
( ; )
3
N AC
.
9. Phn thun. Ta có
= = 45
2
AC
CMN
2. 90CIN CMN = =
(góc tâm đường tròn
()I
).
ICN
; 90IC IN CIN= =
ICN
vuông cân ti I
45NCI =
.
45NCI =
(vì
OBC
cân)
Suy ra C, I, B thng hàng.
Do đó I thuộc đường thng BC.
Gii hn. Khi M tiến ti B thì I tiến ti
1
I
(
1
I
là trung điểm đoạn thng BC)
Khi M tiến ti C thì I tiến ti C.
Vy I chuyển động trên đoạn thng
1
IC
thuộc đoạn thng BC.
Phần đảo. Lấy điểm I bt thuộc đon thng
1
IC
. V đưng tròn
( ; )I IC
ct OC ti N. Gi M giao
điểm th hai của đoạn thng AN vi
()I
.
Ta có
IC IN ICN=
cân mà
45 45 90NCI CNI CIN= = =
Do đó
1
45
2
CMN CIN= =
Ta có
( )
45CMN CBA ACMB= =
là t giác ni tiếp
M
thuc nửa đường tròn
()O
.
Kết lun. Tp hợp các điểm Iđoạn thng
1
CI
(vi
1
I
là trung điểm đoạn thng BC).
D.BÀI TOÁN LUYN THÊM ( Nếu cn )
Câu 1. Cho đường tròn
O
,
A
là điểm c định nằm ngoài đường tròn
O
.
OBC
là đường kính quay
quanh
O
. Tìm tp hp tâm
I
đường ngoi tiếp tam giác
ABC
.
ng dn:
a) Phn thun:
Gi
D
là giao điểm ca
AO
với đường tròn
I
AD
.
Xét
OAB
OCD
có:
OAB OCD
(cùng chn
BD
)
(d)
D
O
C
B
A
I
ca
I
);
AOB COD
ối đỉnh). Do đó
OA OB
OAB OCD
OC OD
..OAOD OBOC
2
2
.
R
OAOD R OD
OA
2
R
OD
OA
,
2
R
OA
không đổi
D
c định. Vy
I
thuộc đường thng
d
c định là trung trc của đoạn thng
AD
.
b) Gii hn:
Khi
BOC
qua
A
thì
1
II
(
1
I
là trung điểm ca
AD
).
Khi
BOC
không qua
A
thì
I
chy xa vô tận trên đường thng
d
.
Vy
I
chuyển động trên đường thng
d
(tr điểm
1
I
là trung điểm
AD
là đường trung trc của đoạn
thng
AD
.
c) Phần đáo: Lấy điểm
I
bt k thuộc đường thng
d
1
II
. V đường tròn
;I IA
cắt đường tròn
O
ti
B
.
BO
ct
;I IA
ti
C
. Ta có:
IA ID D
thuộc đường tròn tâm
I
bán kính
IA
2
.
.
R
OA
OA OB OAOD
OA
OAB OCD OC R C
OC OD OB R
thuộc đường tròn
O
.
d) Kết lun: Tp hợp các điểm
I
là đường trung trc của đoạn thng
AD
(vi
D
thuộc tia đối ca tia
OA
2
R
OD
OA
)tr điểm
1
I
(
1
I
là trung điểm của đoạn thng
AD
).
Câu 2. Cho đường tròn
;OR
đường kính
AB
. V đường thng
d
vuông góc vi
AB
ti
I I AB
.
Gi
M
là điểm chuyển động trên đường tròn
;OR
.
MA
MB
lần lượt ct
d
ti
C
D
. Tìm tp
hp các tâm
J
của đường tròn qua ba điểm
,,A D C
.
ng dn:
a) Phn thun: Gi
E
là điểm đối xng ca
B
qua
dE
c định.
0
; 90EDC BDC AMB
(góc ni tiếp chn nửa đường tròn).
CAI BDC
(hai góc nhn có cạnh tương ứng vuông góc)
Suy ra
EDC CAI
t giác
EDCA
ni tiếp
đường tròn qua ba điểm
,,A D C
đi qua hai điểm c định
,AE
.
Vy tâm
I
của đường tròn
qua ba điểm
,,A D C
thuc
x'
d
x
O
C
M
2
M
1
I
2
M
I
J
1
J
D
B
E
A
đường thng c định là đường
trung trc
xy
của đoạn thng
AE
.
b) Gii hn:
+ Khi
1
MM
thì
1
JJ
(
1
M
là trung điểm
AB
;
1 1 1 1
,J M OM J d
+ Khi
2
MM
thì
2
JJ
(
2
M
là trung điểm
AB
;
2 2 2 2
,J M OM J d
Do đó
J
chuyển động trên hai tia
12
,J x J y
của đường trung trc của đoạn thng
AE
.
c)Phần đảo: Lấy điểm
J
bt k trên tia
1
Jx
(hoc
2
Jy
). V đường tròn
;J JA
ct
d
ti
,CD
.
AC
ct
BD
ti
M
.
Ta có:
JE JA
(
J
thuc trung trc ca
AE
)
,E J JA
.
ACI DEA
(
EDCA
ni tiếp
J
);
DBE DEA
(
,BE
đối xng qua
d
).
Suy ra
ACI DBE
t giác
ICMB
ni tiếp đường tròn.
00
90 90CIB CMB M
thuộc đường tròn
O
.
d)Kết lun: Tp hp các tâm
J
đường tròn qua ba điểm
,,A D C
là hai tia
1
Jy
của đường trung trc ca
đoạn thng
AE
.
Câu 3. Cho ba điểm c định
,,A B C
thng hàng theo th t đó. Trên đường thng
d
vuông góc
AB
ti
B
lấy điểm bt k
D
. Gi
H
là trc tâm ca tam giác
DAC
. Tìm tp hp các tâm
O
của đường tròn
ngoi tiếp tam giác
DAH
.
ng dn:
a) Phn thun:
AC
ct
O
ti
,AE
.
Xét
BAH
BDC
có:
0
90ABH DBC
;
BAH BDC
(hai góc nhn có cạnh tương ứng vuông góc).
Do đó
BAH BDC
AB BH
BD BC
. Suy ra:
..BD BH AB BC
(không đổi) (1)
M
E
A'
(m)
O
(d)
H
D
C
B
A
Xét
BAD
BHE
có:
B
chung,
BAD BHE
(t giác
ADHE
ni tiếp). Do đó:
..
BA BD
BAD BHE BABE BD BE BC BE
BH BE
(2) T (1) và (2) ta có:
. . .BD BH AB BC BABE BC BE
.
E
thuộc đường thng c định
AB
suy ra
E
c định.
OA OE
(
O
là tâm đường tròn
DAH
)
O
thuộc đường thng c định ,
m
là đường trung trc ca
đoạn thng
AE
.
b) Gii hn:
D
chuyển động trên c đường thng
d
nên
O
chuyển động trên c đưng thng
m
(loi tr
điểm
m
là giao điểm ca
AC
m
).
c) Phần đảo: Ly
O
bt k trên đường thng
m
. V đường tròn
;O OA
cắt đường thng
d
lần lượt ti
,HD
.
OA OE
nên
;E O OA
. Xét
BAD
BHE
có:
B
chung;
BAD BHE
(t giác
ADHE
ni
tiếp). Suy ra:
..
BA BD
BAD BHE BABE BD BH
BH BE
. Mà
BE BC
do đó:
..
AB BH
BD BH AB BC
BD BC
. Xét
BAH
BDC
có:
0
90ABH DBC
;
AB BH
BD BC
. Do
đó
BAH BDC BAH BDC
.
0
90DBC BCD
nên
0
90BAH BCD
0
' 90AA C AH DC
.
ADC
,DB AC AH DC H
là trc tâm ca
DAC
.
d) Kết lun: Tp hp các tâm
O
của đường tròn ngoi tiếp tam giác
DAH
là đường trung trc
m
ca
đoạn thng
AE
(tr điểm
M
là giao điểm ca
AC
vi
m
(vi
E
là điểm đối xng ca
C
qua
B
).
Câu 3. Cho tam giác cân
ABC
ni tiếp trong đường tròn
;OR
2AB AC R
.
M
là điểm
chuyển động trên cung nh
AC
đường thng
AM
cắt đường thng
BC
ti
D
. Tìm tp hợp các điểm
I
là tâm đường tròn ngoi tiếp tam giác
MCD
.
ng dn:
a) Phn thun:
2AB AC R
(gt);
,AB AC
là dây cung ca
;OR
nên
,AB AC
là các cnh ca hình
vuông ni tiếp
;OR
suy ra
ABC
vuông cân ti
A
, suy ra
BC
là đường kính ca
;OR
,
0
2 90CID CMD
Ta có:
0
45CMD CMD
nhn,
O
x
D
I
M
C
B
A
do đó
1
2
CMD CID
0
90CID
.
ICD
IC ID R ICD
cân ti
I
0
90CID
nên
ICD
vuông cân ti
I
, suy ra
0
45ICD IDC
. Ngoài ra
0
45ACB
do đó
0
90ACI
.
0
90ACI
AC
c định
Cx
vuông góc vi
AC
ti
C
.
b) Gii hn:
Khi
MC
thì
IC
.
Khi
MA
thì
I
chy xa vô tn trên tia
Cx
.
Vy
I
chuyển động trên tia
Cx
vuông góc vi
AC
ti
C
.
c) Phần đảo: Ly
I
bt k thuc tia
Cx
. V đường tròn
;I IC
, đường tròn này ct
BC
ti
B
, ct
O
ti
M
;M C D C
. T giác
BAMC
ni tiếp
00
180 135ABC AMC AMC
.
ICD
00
45 90IC ID r IDC CID
0
1
45
2
CMD CID CID
0 0 0
135 45 180 , ,AMC CMD A M D
thng hàng.
d) Kết lun: Tp hp các tâm
I
của đường tròn ngoi tiếp
MCD
là tia
Cx
vuông góc vi
AC
ti
C
.
Câu 4. Cho đường tròn
;OR
và điểm
A
c định. Đường tròn tâm
I
di động qua
A
ct
O
ti
,BC
.
Gi
M
là giao điểm ca
BC
và tiếp tuyến ti
A
của đường tròn
I
. Tìm tp hợp các điểm
M
.
ng dn:
a) Phn thun: V tiếp tuyến
MD
vi
O
DO
.
Xét
MAC
MBA
M
chung,
MAC MBA
,
(góc to bi tia
tiếp tuyến dây cung và góc ni tiếp
cùng chn cung
AC
)ca
I
.
Do đó
MAC MBA
.
2
.
MA MC
MA MB MC
MB MA
.
O
(d)
I
K
H
D
M
C
B
A
Tương tự
2
.MD MB MC
. Mt khác,
MOD
0
90D
nên theo định lý Pitago, ta có:
2 2 2 2 2
MD MO OD MO R
. Do đó
2 2 2
MA MO R
, suy ra
2 2 2
MO MA R
.
0 2 2 2
90HMA MHA MA MH AH
0 2 2 2
90HMO MHO MO MH HO
. Do đó:
2 2 2 2 2 2 2 2
MH OH MH AH R OH AH R
;
Do đó
2
2
2
1
2
R
R
OH AH
OH AH OH AH R OH OA
OA
OA
OH AH OA
(không đổi)
H
c
định.
H
c định,
OA
c định,
MH AO
ti
H
.Vy
M
thuộc đường thng
d
vuông góc vi
OA
ti
H
.
b) Gii hn:
O
chuyển động trên c đường thng
d
.
c) Phần đảo: Ly
M
bt k thuộc đường thng
d
. V cát tuyến
MBC
vi
O
,B C O
, v đường
tròn
I
qua
,,A B C
v tiếp tuyến
MD
vi
O
DO
.
Xét
MCD
MDB
M
(chung),
MDC MBD
(góc to bi tia tiếp tuyến dây cung và góc ni
tiếp cùng chn cung
CD
ca
O
).
Do đó
2
.
MC MD
MCD MDB MD MB MC
MD MB
MDO
0 2 2 2 2 2
90D MD MO OD MO R
.
Suy ra
22
.MB MC MO R
; mà
2 2 2
HO AH R
, do đó
2 2 2 2 2 2
.MB MC MO HO AH MO HO AH
2 2 2
MH AH MA
.Xét
MAC
MBA
AMC
(chung);
MA MC
MB MA
(vì
2
,MB MC MA
).Do đó
MAC MBA MAC MBA
. V
IK AC
ta có
đ
1
s
2
AIK ABC AC
suy ra:
MAC AIK
. Mt khác
AKI
0
90K
0
90AIK IAK
nên
0
90MAC IAK
0
90IAM
, do đó
MA
là tiếp tuyến ca
I
.
d) Kết lun: Tp hợp các điểm
M
là đường thng vuông góc vi
OA
ti
H
(vi
2
1
2
R
OH OA
OA
)
Câu 5. Cho đường tròn
;OR
và điểm
A
c định trong đường tròn
0A
BC
là dây cung di động
quay quanh
A
. Các tiếp tuyến ti
B
C
với đường tròn
O
ct nhau ti
D
. Tìm tp hợp các điểm
D
.
ng dn:
a) Phn thun: Gi
M
là giao điểm
ca
OD
BC
.
V
DH OA H OA
,
DB DC
nh lý tiếp tuyến),
OB OC R
suy ra
DO
là trung trc ca
BC DO BC
.
Xét
OMA
OHD
O
chung,
0
90OMA OHD
. Do đó
OMA OHD
..
OA OM
OAOH OM OD
OD OH
,
OBD
0
90 ;B BM OD
nên
22
.OM OD OB R
. Suy ra
2
2
.
R
OAOH R OH
OA
(không đổi)
H
c định. Vy
D
thuc
đường thng c định
d
vuông góc với đường thng
OA
ti
H
.
b) Gii hn:
BC
quay quanh
A
nên
D
chuyển động trên đường thng
d
.
c) Phần đảo: Ly
D
bt k trên đường thng
d
. V dây
BC
qua
A
và vuông góc vi
OD
ti
M M OD
. Xét
..
OA OM
OMA OHD OAOH OM OD
OD OH
. Mà
2
.OAOH R
nên
2
.
OM OB
OM OD R
OB OD
do đó
OMB OBD
,
suy ra
OMB OBD
MOB
(chung);
OM OB
OB OD
do đó
OMB OBD
,
suy ra
OMB OBD
; mà
0
90OMB
nên
0
90OBD DB
là tiếp tuyến ca
O
.
Tương tự
DC
là tiếp tuyến ca
O
.
d) Kết lun: Tp hp các điểm
D
là đường thng
d
vuông góc vi
OA
ti
H
(vi
2
R
OH
OA
).
Câu 6. Cho đường tròn
;OR
và điểm
A
c định nằm ngoài đường tròn. Cát tuyến
m
qua
A
ct
đường tròn
O
ti
B
C
. Tiếp tuyến ti
B
C
với đường tròn
O
ct nhau ti
D
. Tìm tp hp
các điểm
D
.
ng dn:
H
O
M
C
D
B
A
a) Phn thun: Gi
M
là giao điểm ca
OD
BC
. V đường thng
d
qua
D
vuông góc vi
OA
ti
H
H OA
.
DB DC
nh lý tiếp tuyến);
OB OC R
. Suy ra
DO
là trung trc ca
BC DO BC
.
Xét
OMA
OHD
MOA
chung;
0
90OMA OHD
do đó
OMA OHD
..
OA OM
OAOH OM OD
OD OH
OBD
0
90B
,
BM OD
nên
22
.OM OD OB R
, suy ra
2
.OAOH R
hay
2
R
OH
OA
không
đổi
H
c định. Vy
D
thuộc đường thng
d
c định vuông góc với đường thng
OA
ti
H
.
b) GIi hn:
D
nằm ngoài đường tròn
;OR
, do đó
D
chuyển động trên đường thng
d
tr đoạn thng
12
DD
(vi
12
,DD
là giao điểm ca
d
và đường tròn
;OR
.
c) Phần đảo: Lấy điểm
D
bt k trên đường thng
d
tr đoạn thng
12
DD
. V đường thng
m
qua
A
vuông góc vi
OD
cắt đường tròn
;OR
ti
,BC
ct
OD
ti
M
.
Xét
OMA
OHD
MOA
chung;
0
90OMA OHA
,
do đó
..
OA OM
OMA OHD OAOH OM OD
OD OH
.
2
.OAOH R
nên
2
.OM OD R
, suy ra
OM OB
OB OD
.
Xét
OMB
OBD
O
chung;
OM OB
OB OD
, do đó
OMB OBD
, suy ra
OMB OBD
0
90OMB
nên
0
90OBD DB
là tiếp tuyến ca
O
.
Tương tự
DC
là tiếp tuyến ca
O
.
d) Kết lun: Tp hợp các điểm
D
là đường thng
d
(tr đoạn thng
12
DD
) vuông góc vi
OA
ti
H
(vi
2
R
OH
OA
).
Câu 7. Tam giác
ABC
cân ti
A
c định ni tiếp trong đường tròn
;OR
. Điểm
M
di động trên cnh
BC
. Gi
D
là tâm đường tròn đi qua
M
và tiếp xúc vi
AB
ti
B
. Gi
E
là tâm đường tròn đi qua
M
D
2
D
1
C
O
H
M
B
A
D
và tiếp xúc vi
AC
ti
C
. Tìm tp hợp các điểm
I
là trung điểm ca
DE
.
ng dn:
a) Phn thun:
V đường kính
AF
của đường tròn
O
;
0
90ABF
(góc ni tiếp chn na
đường tròn);
0
90ABD
(
AB
tiếp xúc vi
D
ti
B
).
Suy ra
,,B D F
thng hàng.
Tương tự
,,C E F
thng hàng.
ABC
cân ti
A AF BC
;
11
;BF CF B C
.
11
;BD DM B DMB EM EC C EMC
.
Suy ra
11
B DMB EMC C
.
11
/ / ; / /B EMC BF ME C DMB MD CF
.
//
//
BF ME
DMEF
MD CF
là hình bình hành mà
I
là trung điểm ca
DE I
là trung điểm ca
MF
.
V
IK BC
.
FMH
/ / , ;IK FH IK BC FH BC
I
là trung điểm ca
MF
IK
là đường trung bình ca
1
2
FMH IK FH
(không đổi).
Vy
I
thuộc đường thng
d
song song vi
BC
cách
BC
mt khong bng
1
2
FH
.
b) Gii hn:
Khi
MB
thì
1
II
(
1
I
là trung điểm ca
BF
);
Khi
MC
thì
2
II
(
2
I
là trung điểm ca
CF
).
Do đó
I
chuyển động trên đoạn thng
12
II
.
c) Phần đảo: Lấy điểm
I
bt k thuộc đoạn thng
12
II
,
FI
ct
BC
ti
M
.
I
1
I
2
I
D
E
F
M
H
K
C
B
A
V
/ / , / /MD CF D BF ME BF E CF
DMEF
là hình bình hành mà
I
là trung điểm ca
MF
I
là trung điểm ca
DE
.
D dàng chứng minh được
DB DM
EM EC
.
Do đó
AB
tiếp xúc vi
;D AC
tiếp xúc vi
E
.
d) Kết lun: Tp hợp các điểm
I
là đường trung bình ca tam giác
FBC
(vi
F
là trung điểm ca
BC
).
Câu 8. Cho đường tròn
;OR
đường kính c định
AB
và đường kính
CD
di động.
AC
AD
ct tiếp
tuyến
a
vi
O
ti
B
lần lượt ti
M
N
. Tìm tp hp tâm
I
của đường tròn ngoi tiếp tam giác
CMN
.
ng dn:
a). Phn thun:
đ đ đ đ đ
11
s s ;s s s
22
ACD AD DNM AB BD
.
đđ
0
11
180 s s
22
BD AD
Suy ra
ACD DNM
t giác
DCMN
ni tiếp trong
đường tròn
I
.
0
90DAC
(góc ni tiếp
chn nửa đường tròn).
AMN
0
90A
,
AE
là trung tuyến suy ra
EA EM EAM AME
.
Do đó
ACF FAC ANM AMN
.
00
90 90ANM AMN ACF FAC
hay
AE DC
.
I
là tâm đường tròn qua
, , , , / /D C M N OI DC AE DC AE OI
.
, / /AO a EI a AO EI
Suy ra
AOIE
là hình bình hành
EI AO R
.
Đưng thng
a
c định.
Vy
I
thuộc đường thng c định
d
song song với đường thng
a
và cách
a
mt khong bng
R
.
I
(d)
F
O
D
C
A
N
B
E
M
b) Gii hn:
CD
quay quanh
O
nên
E
chuyển động trên c đường thng
a
do đó
I
chuyển động trên c
đường thng
, / / ,d d a d
cách
a
mt khong bng
R
.
d
nm trên na mt phng b
a
không chứa điểm
A
.
c) Phần đảo: Lấy điểm
I
tùy ý trên đường thng
d
. V
IE a E a
. V
DC OI
ti
O
.
,AC AD
lần lượt ct
a
ti
,MN
.
, / /AO a EI a AO EI
AO EI R
do đó
AOIE
là hình bình hành
//AE OI
. Mà
//OI DC
nên
AE DC
.
Tương tự như trên, ta chứng minh được t giác
DCMN
ni tiếp. Suy ra
EAM
cân ti
E EA EM
.
Suy ra
EAN
cân ti
E EA EN
. Do đó
EM EN
.
Vy
I
là tâm đường tròn ngoi tiếp tam giác
CMN
.
d) Kết lun: Tp hợp các điểm
I
là đường thng
d
, song song vi
a
,
d
cách
a
mt khong bng
R
,
d
nm trên na mt phng b
a
không chứa điểm
A
.
Câu 9. Cho nửa đường tròn đường kính
AB
tâm
O
bán kính
R
.
C
là trung điểm cung
AB
.
M
là điểm
chuyển động trên cung
BC
,
AM
ct
CO
ti
N
. Gi
I
là tâm đường tròn ngoi tiếp tam giác
CMN
.
Tìm tp hợp các điểm
I
.
ng dn:
a). Phn thun:
đ
0
1
s 45
2
CMN AC
;
CMN
nhn suy ra
1
2
CMN CIN
0
90CIN
.
ICN
cân
IC IN r
0
90CIN ICN
vuông cân ti
I
0
45NCI
.
0
45NCB
(
OBC
cân ti
O
) suy ra
,,CIB
thng hàng.
Do đó
I
thuộc đường thng
BC
.
b) Gii hn:
Khi
MB
thì
1
II
(
1
I
là trung điểm ca
BC
.
Khi
MC
thì
IC
.
Vy
I
chuyển động trên đoạn
1
IC
thuộc đoạn thng
BC
.
c) Phần đảo: Lấy điểm
I
bt k thuộc đoạn thng
1
IC
. V đường tròn
;I IC
ct
OC
ti
N
,
AN
ct
I
ti
M M N
.
I
1
I
N
M
O
C
B
A
Ta có
IC IN ICN
cân mà
0 0 0
45 45 90NCI CNI CIN
. Do đó
0
1
45
2
CMN CIN
;
0
45CMN CBA
t giác
ACMB
ni tiếp được
M
thuc nửa đường
tròn
O
.
d) Kết lun: Tp hợp các điểm
I
là đoạn thng
1
CI
(
1
I
là trung điểm của đoạn thng
BC
).
Câu 10. Cho góc nhn
xOy
.
A
là điểm c định trên tia
Ox
. Đường tròn
I
di động tiếp xúc tia
Ox
ti
A
và ct tia
Oy
ti
B
C
. Tìm tp hp tâm
K
của đường tròn ni tiếp tam giác
ABC
.
ng dn:
a) Phn thun:
1
2
BAK BAC
(tính cht tiếp tuyến).
đ
1
s
2
OAB OCA AB
.
Do đó
OAK OAB BAK
11
22
OAB OCA BAC
11
22
OCA OAC
11
22
OCA OAB BAC
00
11
90 90 .
22
AOC xOy
Ta có
OAK
không đổi,
OA
c định, do đó
K
thuc tia
Az
sao cho
0
1
90
2
OAz xOy
.
b) Gii hn:
K
nm trong
xOy
. Do đó
K
thuộc đoạn thng
'AA
(
'A
là giao điểm ca tia
Az
và tia
Oy
).
c) Phần đảo: Lấy điểm
K
bt k trên tia
Az
. V
KH Oy H Oy
, v đường tròn
;K KH
. T
A
v
các tiếp tuyến vi
K
lần lượt ct
Oy
ti
B
C
. Cn chng minh rằng đường tròn ngoi tiếp tam giác
ABC
tiếp xúc vi tia
Ox
.
Ta có
1
2
BAK BAC
(tính cht tiếp tuyến);
00
1 1 1 1
90 90
2 2 2 2
OAK OAz xOy AOC OCA OAC
11
22
OCA OAB BAC
.
11
22
OAK OCA OAB BAC
(1). Mà
1
2
OAK OAB BAK OAB BAC
(2). T (1) và (2)
suy ra
OAB OCA
. (*)
O
z
y
x
A
A'
I
K
H
C
B
V tia
Am
là tia tiếp tuyến của đường tròn ngoi tiếp
ABC
(tia
Ax
nm trên na mt phng b
AB
cha tia
OA
). Ta có:
đ
1
s
2
mAB OCA AB
(**) T (*) và (**) có
OAB mAB
suy ra hai tia
AO
Am
trùng nhau.
Vy
AO
là tiếp tuyến của đường tròn ngoi tiếp tam giác
ABC
.
d) Kết lun: Tp hợp các điểm
K
là đoạn thng
'AA
(
'A
là giao điểm ca hai tia
Az
Oy
0
1
90
2
OAz xOy
.
Câu 11. Cho
xAy
không đổi , điểm
B
c định nm trong
xAy
. Đường tròn
O
di động qua
A
B
ct
,Ax Ay
lần lượt ti
C
D
. Chng minh rng trng tâm
G
ca tam giác
ADC
thuc mt
đường c định.
ng dn:
Ta có:
,xAB CDB
0
180BAy BCDDAC DBC
Các góc
,,xAB BAy DAC
không đổi.
Do đó các góc
,,CDB BCD DBC
không đổi. Gi
M
là trung điểm ca
đoạn
CD
, ta có các góc
,BMC BMD
không đổi.
V đường tròn ngoi tiếp tam giác
MBC
ct
Ax
ti
E
, đường tròn ngoi tiếp tam giác
MBD
ct
Ay
ti
F
.
Ta có
00
180 180BEC BMC AEB BMC
không đổi
E
c định.
đ
1
s
2
BME BCE BE
,
BDF BCE
(t giác
ADBC
ni tiếp),
0
180BDF BMF
(t giác
DBMF
ni tiếp).
Do đó
0
180 , ,BME BMF E M F
thng hàng.
V
,AH EF H EF GK EF K EF
ta có
AH
không đổi; đặt
, / /AH h AH GK
.
AHM
//GK AH
suy ra
GM GK
AM AH
.
D
H
M
K
G
C
B
A
E
x
y
G
là trng tâm
ADC
,
AM
là trung tuyến ca
ADC
nên
1
3
GM
AM
.Do đó
1
3
GK
AH
, suy ra
1
3
GK h
không đổi,
EF
c định. Vy
G
thuộc đường thng song song
vi
EF
là cách
EF
mt khong bng
1
3
h
.
Câu 12. Cho đường tròn
;OR
và hai dây cung
AB
CD
song song vi nhau.
M
là điểm di động
trên đường tròn
O
. Đường thng
MD
cắt đường thng
AB
ti
Q
. Tìm tp hp tâm
J
đường tròn
ngoi tiếp tam giác
MCQ
.
ng dn:
1) Xét
M
nm trên cung ln
CD
.
Tiếp tuyến ca
O
ti
C
ct
AB
E
,
ta có
E
c định.Gi
Cx
là tia đối ca
tia
CE
.
QEC DCx
(vì
//AB DC
),
đ
1
s
2
QMC DCx DC
.
Do đó
QEC QMC
t giác
MECQ
ni tiếp.
Ta có
;JE JC
,EC
c định. Do đó
J
thuộc đường c định là đường trung trc của đoạn thng
EC
.
2) Xét
M
nm trên cung
CD
. Tương tự trường hợp 1) ta cũng có:
QEC DCx
.
0
180QMC DCx
.
Do đó
0
180QEC QMC
t giác
MCEQ
ni tiếp được. Ta có
;JE JC
,EC
c định.
Do đó
J
thuộc đường trung trc của đoạn thng
EC
.
Câu 13. Cho tam giác
ABC
cân ti
A
.
M
là điểm di động trên cnh
BC
. V
MD
song song
AC D AB
v
ME
song song
AB E AC
.
K
là tâm đường tròn ngoi tiếp tam giác
ADE
. Tìm
tp hợp điểm
K
.
ng dn:
a) Phn thun: Gi
O
là giao điểm của đường tròn
ADE
và đường cao
AH
ca tam giác
ABC
.
T giác
MDAE
là hình bình hành
(vì
//MD EA
//DA ME
),
suy ra
DM AE
.
x
J
O
E
Q
M
D
C
B
A
K
2
K
1
E
K
D
O
M
H
C
B
A
Ta có:
/ / ;DMB ACB DM AC
DBM ACB
(
ABC
cân ti
A
).
Suy ra
DMB DBM
.
Vy
DBM
cân ti
D
, suy ra
DM DB
.Do đó
AE DB DM
DAO OAE OD OE OD OE
.
Xét
OAE
OBD
,OE OD AEO ODB
(t giác
AEOD
ni tiếp),
AE DB
.
Do đó
OAE OBD
(c.g.c)
OA OB O
thuộc đường trung trc ca
AB
.
Vy
O
là điểm c định (
O
là tâm đường tròn ngoi tiếp
ABC
).
Ta có
,KA KO
OA
c định, suy ra
K
nằm trên đường trung trc
d
của đoạn thng
OA
.
b) Gii hn:
Khi
MB
thì
1
,D B K K
(
1
K
là giao điểm ca
d
và đường trung trc ca
AB
).
Khi
MC
thì
2
,E C K K
(
2
K
là giao điểm ca
d
và đường trung trc ca
AC
).
Vy
K
di động trên đoạn thng
12
KK
.
c) Phần đảo: Lấy điểm
K
bt k thuộc đoạn thng
12
KK
. V đường tròn
;K KA
ct
,AB AC
lần lượt
D
E
. V
//DM AC M AC
. Cn chng minh rng
//ME AB
.
Ta có:
KA KO O K
.
Xét
OAE
OBD
có:
;OAE OBD OAD AEO ODB
(t giác
AEOD
ni tiếp)
Do đó
1
AE OA
OAE OBD AE BD
BD OB
.
DBM ACB
(
ABC
cân ti
A
),
//DMB ACB DM AC
. Do đó
DBM DMB DBM
cân
ti
D DM BD
.
Ta có
AE DM
//AE DM
nên t giác
MDAE
là hình bình hành, suy ra
//ME AB
.
d) Kết lun: Tp hợp điểm
K
là đoạn thng
12
KK
thuộc đường trung trc của đoạn thng
AO
.
Câu 14. Cho tam giác
,ABC H
là trực tâm. Hai đương thẳng song song
d
'd
lần lượt đi qua
A
H
. Các điểm
,MN
lần lượt là hình chiếu ca
B
C
trên
d
; các điểm
,QP
lần lượt là hình chiếu ca
,BC
trên
'd
.
MP
ct
NQ
ti
I
. Tìm tập các điểm
I
khi
d
'd
di động.
ng dn:
a) Phn thun:
//
()
BM d
BM CN
CN d gt
//
/ / ( )
BM CN
MNPQ
MN QP gt
là hình bình hành.
0
90QMN
(gt) nên
MNPQ
là hình ch nht
I
là trung điểm của các đoạn thng
MP
NQ
.
Gi
D
E
lần lượt là trung điểm ca
AH
BC
, ta có
,DE
c định.
ANHQ
là hình thang,
DI
là đoạn thng nối trung điểm hai đường chéo, suy ra
//DI MN
.
MPCB
là hình thang,
IE
là đường trung bình hình thang, suy ra
//IE NC
.
/ / , / /DI MN IE NC
0
90MNC
nên
0
90DIE
.
0
90 ,DIE DE
c định. Vy
I
thuộc đường tròn đường kính
DE
.
b) Gii hn:
d
quay quanh
A
nên điểm
I
chuyển động trên đường tròn đường kính
DE
.
c) Phần đảo: Lấy điểm
I
bt k thuộc đường tròn đường kính
DE
. Ni
DI
. Qua
,AH
k các đường thng
,'dd
song song vi
DI
. Gi
,MQ
lần lượt là hình chiếu ca
B
trên
,'dd
.
MI
ct
'd
ti
P
;
QI
ct
d
ti
N
;
PQ
ct
IE
ti
K
.
/ / / / , ,MN DI QP DA DH IM IP IN IQ
,IM IP IN IQ MNPQ
là hình bình hành.
0
90M
nên
MNPQ
là hình ch nht.
PMB
, / /IM IP IK MB KB KP
;
BPC
, / /KB KP EB EC EK CP
0
90DIE
(góc ni tiếp chn nửa đường tròn),
/ / , ,DI MN EI MN EI MN PN MN
,,C P N
thng hàng.
(d')
(d)
D
P
E
Q
I
K
N
M
C
B
H
A
d) Kết lun: Tp hợp các điểm
I
là đường tròn đường kính
DE
.
Câu 15. Cho đường tròn
;,O R M
là điểm ngoài
O
, v hai tiếp tuyến
,MA MB
đến
O
(
,AB
là tiếp
điểm). đường trung trc của đường kính
BC
ct
CA
ti
D
.
1) Tìm tp hợp các điểm
M
sao cho
MAB
đều.
2) Tìm tp hợp các điểm
D
sao cho
MAB
đều.
ng dn:
1) a) Phn thun:
MAB
đều
0
60AMB
;
0
1
30
2
OMA AMB
(
,MA MB
là tiếp tuyến ca
O
)
OMA
00
90 , 30OAM OAM
suy ra
OMA
là nửa tam giác đều, do đó
1
22
2
OA OM OM OA R
2OM R
,
O
c định, suy ra
M
thuộc đường tròn c định
;2OR
.
b) Gii hn:
M
là điểm tùy ý trên
;2OR
đều v được
MAB
đều. Vy
M
chuyển động trên
;2OR
.
c) Phn đảo: Ly
M
bt k thuc
;2OR
v hai tiếp tuyến
,MA MB
đến
;OR
(
,AB
là tiếp điểm)
MA MB MAB
cân ti
M
.
Tam giác
OMA
0
1
90 ;
2
A OA OM R
, suy ra
OMA
là nửa tam giác đều nên
0
30OMA
, suy
ra
0
2. 60AMB OMA
.
MAB
cân có
0
60AMB MAB
đều.
d) Kết lun: Tp hợp các điểm
M
là đường tròn
;2OR
.
2) a) Phn thun:
MAB
đều
0
60AMB
. Mà
0
180AMB AOB
nên
0
120 ;AOB
0
1
60
2
ACB AOB
.
DOC
00
90 , 60O DCO
suy ra
DOC
là nửa tam giác đều và ta có
33DO OC R
.
D
C
B
A
M
O
3DO R
,
O
c định nên
D
thuc đường tròn
;3OR
.
b) Gii hn:
D
là điểm tùy ý trên
.
c) Phần đảo: Lấy điểm
D
bt k thuc
;3OR
. V đường kính
BC
vuông góc
,OD DC
ct
O
ti
A
.
M
là giao điểm ca hai tiếp tuyến ti
,AB
ca
O
.
DOC
0
90 ; 3 3O DO OC R DOC
là nửa tam giác đều
00
60 60DCO MAB
.
MAB
cân
MA MB
0
60MAB MAB
đều.
d) Kết lun: Tp hợp các điểm
D
là đường tròn
;3OR
.
Câu 16. Cho tam giác
ABC
có ba góc nhn. bên ngoài tam giác v hai nửa đường tròn có đường kính
,AB AC
. Một đường thng
d
quay quanh
A
ct hai nửa đường tròn trên theo th t ti
,MN
(khác
A
). Tìm tp hợp các trung điểm ca
MN
.
ng dn:
a) Phn thun:
0
90AMB
(góc ni tiếp chn nửa đường tròn),
0
90ANC
(góc ni tiếp chn nửa đường
tròn), suy ra
BCNM
là hình thang vuông.
Gi
O
là trung điểm ca
BC
ta có
O
c định; gi
K
là trung điểm ca
MN
.
OK
là đường trung bình ca
hình thang
BCNM
suy ra
//OK BM
0
0
//
90
90
OK BM
AKO
AMB
0
90AKO
,
OA
c định,
do đó
K
thuộc đường tròn
đường kính
OA
.
b) GIi hn:
Khi
1
dd
(
1
d
là tiếp tuyến của đường tròn đường kính
AB
)thì
1
KK
(
1
K
là hình chiếu ca
O
trên
1
d
).
Khi
2
dd
(
2
d
là tiếp tuyến của đường tròn đường kính
AC
)thì
12
KK
(
2
K
là hình chiếu ca
O
trên
2
d
).
K
2
K
1
K
O
C
B
A
(d)
N
M
d2
d1
Vy
K
chuyển động trên cung
12
KK
của đường tròn đường kính
OA
.
c) Phần đảo: Lấy điểm
K
bt k thuc cung
0
12
90K K OKA
.
AK
cắt các đường tròn đường kính
,AB AC
lần lượt ti
,MN
.
0
90AMB
(góc ni tiếp chn nửa đường tròn)
0
90ANC
(góc ni tiếp chn nửa đường tròn).
Suy ra
BCNM
là hình thang vuông.
OK MN
do đó
//OK BM KM KN
.
d) Kết lun: Tp hợp các điểm
K
là cung
12
KK
của đường tròn đường kính
OA
.
Câu 17. Cho đường tròn
;OR
c định
BC
là dây cung c định,
A
là điểm chuyển động trên cung ln
BC
. Trên tia đối ca tia
AB
lấy điểm
D
sao cho
AD AC
. Tìm tp hợp các điểm
D
.
ng dn:
a). Phn thun: Gi
J
là trung điểm ca cung ln
BC
,
ta có
I
c định.
xét điểm
A
thuc cung
IC
.
0
180IAC IBC
(t giác
BIAC
ni tiếp);
0
180IAD IAB
(hai góc k bù),
IBC IAB IC ID
. Suy ra
IAC IAD
.
Xét
IAC
IAD
IA
(cnh chung),
,IAC IAD AC AD
.
Do đó
IAC IAD
(c.g.c), suy ra
IC ID
.
,IC
c định
IC
không đổi. Vy
D
chuyển động trên đường tròn
;I IC
.
b) GIi hn:
Khi
AB
thì
1
DD
(
1
D
là giao điểm ca
;I IC
vi tiếp tuyến ca
O
ti
B
).
Khi
AC
thì
DC
.
O
I
D
C
B
A
D
1
Vy
D
chuyển động trên cung
1
DC
của đường tròn
;I IC
.
c) Phần đảo: Lấy điểm
D
bt k trên cung
1
DC
IC ID
.
BD
ct
O
ti
A A B
.
AIC ABC
(hai góc ni tiếp cùng chn cung
AC
ca
O
;
1
2
ABC DIC
.
Suy ra
1
2
AIC DIC
, do đó
AIC DIA
.
Xét
IAC
IAD
,,IC ID AIC DIA IA
là cnh chung.
Do đó
IAC IAD
(c.g.c), suy ra
AC AD
.
d) Kết lun: Tp hợp các điểm
D
là cung
1
DC
của đường tròn
,I IC
(vi
1
D
là giao điểm của đường tròn
,I IC
vi tiếp tuyến của đường tròn
O
ti
B
,
I
là trung điểm cung ln
BC
ca
O
).
Câu 18. Cho
AB
là dây cung c định của đường tròn
;OR
.
C
là điểm chuyển động trên cung ln
AB
. Trên tia
CA
lấy điểm
D
sao cho
CD CB
. Tìm tp hợp các điểm
D
.
ng dn:
a) Phn thun: Gi
I
là trung điểm ca
AB
.
Xét
DCI
BCI
,CD CB DCI BCI AI IB
,
CI
(cnh chung).
Do đó (c.g.c), suy ra
ID IB
(không đổi);
I
c định. vy
D
thuộc đường tròn c định
;I IB
.
b) Gii hn:
Khi
CA
thì
DE
(
E
là giao điểm ca tiếp tuyến
ti
A
vi
O
;I IB
).
Khi
CB
thì
DB
. Vy
D
chuyển động trên cung
BAE
ca
;I IB
.
E
O
I
D
B
A
C
c) Phần đảo: Lấy điểm
D
bt k trên
BAE
ca
;I IB
, ta có
ID IB
. V phân giác ca
DIB
ct
O
ti
C
.
Xét
DCI
BCI
(c.g.c), suy ra
,DCI BCI CD CB
.
đ
1
s
2
BCI BI
nên
đ
1
s
2
DCB AB
đ
1
s
2
ACB AB
. Do đó
,,A D C
thng hàng.
d) Kết lun: Tp hợp các điểm
D
BAE
ca
;I IB
(
I
là trung điểm ca
AB
.
Chú ý:
1) Xét bài toán tương t khi
C
chuyển động trên
AB
.
2) Nhn xét gì v các bài toán
Câu 19. Cho đường tròn
;OR
,
A
là điểm c định ngoài
O
. K tiếp tuyến
AB
vi
O
. Đường
thng
d
quay quanh
A
ct
O
tại hai điểm
,CD
. Tìm tp hp trng tâm
G
ca tam giác
BCD
.
ng dn:
a). Phn thun: Gi
,EF
là trung
điểm ca
,CD OA
ta có
F
c định
(vì
OA
c định);
K
là điểm trên
BF
sao cho
2
3
BK
BF
, suy ra
K
c định (vì
BF
c định).
BEF
có:
2
3
BG BK
BE BF
. Suy ra
22
//
33
GK
GK EF GK EF
EF
1
2
EF OA
, do đó
1
3
GK OA
(không đổi)
K
c định. Vy
G
thuộc đường tròn c định
K
bán kính
1
3
OA
.
b) Gii hn:
Khi
d
tiến dần đến tiếp tuyến
AB
thì
GB
.
Khi
d
tiến dần đến tiếp tuyến
1
AB
thì
1
GG
(vi
1
G
giao điểm của đường tròn
1
;
3
K OA
vi
1
BB
).
B
1
G
1
G
K
D
O
C
B
A
F
E
Vy
G
chuyển đọng trên
1
BG
của đường tròn
1
;
3
K OA
(tr hai điểm
B
1
G
).
c) Phần đảo: Lấy điểm
G
bt k trên
1
BG
( tr
B
1
G
ca
1
;
3
K OA
), suy ra
1
3
GK OA
. Trên tia
BG
lấy điểm
E
sao cho
.
AE
ct
O
ti
,DC
.
BEF
có:
2 2 3 1 1
/ / .
3 3 2 3 2
BG BK GK
GK EF GK OA OA
BE BF EF
E
thuộc đường tròn đường kính
0
90OA OAE
.
OE CD E
là trung điểm ca
CD
.
BCD
BE
là trung tuyến và
2
3
BG
BE
nên
G
là trng tâm
ca
BCD
.
d) Kết lun: Tp hợp các điểm
G
1
BG
của đường tròn
1
;
3
K OA
(vi
K
thuộc đoạn
2
,
3
BF BK BF
,
1
G
là giao điểm ca
1
BB
1
;
3
K OA
(tr
B
1
G
)).
Câu 21. Cho điểm
A
chuyển động trên cung ln
BC
c định của đường tròn
;OR
. Tìm tp hp các
tâm
I
đường tròn ni tiếp trong tam giác
ABC
.
ng dn:
Cách 1.a) Phn thun:
Cung
BC
c định,
đặt
đs BC
(không đổi)
đđ
11
ss
22
BAC BC
1
2
IBC ABC
(
BI
là phân giác ca
ABC
);
1
2
ICB ACB
(
CI
là phân giác ca
ACB
);
I
C
B
A
00
1
180 180
2
BIC IBC ICB ABC ACB
00
11
90 90 ,
22
BAC
BC
c định. Do
đó
I
thuc cung cha góc
0
1
90
2
dựng trên đoạn thng
BC
.
b) Gii hn:
Khi
AB
thì
IB
. Khi
AC
thì
IC
. Vy
I
chuyển động trên cung cha góc
0
1
90
2
dng
trên đoạn thng
BC
nm trên na mt phng b
BC
có chứa điểm
O
.
c) Phần đảo: Lấy điểm
I
bt k thuc cung
BC
ca cung cha góc
0
1
90
2
dựng trên đoạn thng
BC
.
V điểm
A
trên cung ln
BC
của đường tròn
;OR
sao cho
BI
là phân giác ca
ABC
.
0
11
90 ;
22
BIC IBC ABC
00
11
180 90
22
ICB BIC IBC BAC ABC ACB CI
là phân giác ca
ACB
.
ABC
BI
CI
là phân giác
I
là tâm đường tròn ni tiếp
ABC
.
d) Kết lun: Tp hp các tâm
I
của đường tròn ni tiếp tam giác
ABC
là cung cha góc
0
1
90
2
dng
trên đoạn thng
BC
nm trên na mt phng b
BC
có chứa điểm
O
.
Cách 2.
a) Phn thun:
AI
ct
O
ti
D
, ta có
BAD DAC
suy ra
DB DC DB DC
(không đổi).
BID ABI BAI
(
BID
là góc ngoài ca
ABI
).
;IBD IBC CBD BAI CBD DB DC
ABI IBC
(
I
là tâm đường tròn ni tiếp
ABC
)
Suy ra
IBD BID DB DI
DI DB
không đổi.
D
c định.
Vy
I
thuộc đường tròn
,D DB
.
b) Gii hn:
Khi
AB
thì
IB
, Khi
AC
thì
IC
.
Vy
I
chuyển động trên
BC
của đường tròn
,D DB
.
c) Phần đảo: Lấy điểm
I
bt k thuc
BC
của đường tròn
,D DB
, ta có
DI DB DC
.
DB DI IBD BID
,
DI
cắt đường tròn ti
A A D BAI DAC
,
CBD DAC
. Do đó
BAI CBD
;BID ABI BAI IBD IBC CBD
. Suy ra
ABI IBC
.
Vy
I
là tâm đường tròn ni tiếp
ABC
.
c) Kết lun: Tp hợp các điểm
I
BC
của đường tròn
,D DB
nm trên na mt phng b
BC
có cha
điểm
O
.
CHUYÊN ĐỀ BỒI DƯỠNG HỌC SINH GIỎI HÌNH HỌC 9
1 | THCS.TOANMATH.com
HH9-CHUYÊN ĐỀ 10. CÁC BÀI TOÁN HÌNH CHN LỌC THƯỜNG GẶP TRONG ĐỀ
HSG VÀ CHUYÊN TOÁN
Bài 1. (Trích đề TS lp 10 TP Hà Ni Năm 2006)
Cho đường tròn (O;R) đường kính AB, C trung điểm ca OA y
MN OA
ti C. Gi K
là điểm tùy ý trên cung nh BM, H là giao điểm ca AK, MN.
a. Chng minh: BCHK là t giác ni tiếp.
b. Tính AH.AK theo R.
c. Xác định v trí K để KM + KN + KB ln nhất. Tính GTLN đó.
Gii
a. Do K nm trên
; 90
2
o
AB
O AKB
Li
90
o
HCB =
(gi thiết) suy ra
180
o
AKB HCB+=
nên
t giác BCHK ni tiếp.
(Tổng hai góc đối bng
180
o
).
b. Ta có
( . )ACH AKB g g
nên
2
. . 2
2
AH AB R
AH AK AC AB R R
AC AK
= = = =
c. Đây một câu hi khá hay. Nếu bn nào biết định lý Ptolemy hoặc định Shooten thì bài toán
được gii quyết.
Cách tiếp cn th nht:
Nhn thy tam giác BMN cân ti B tam giác AMO đều (do AMO cân ti O ti M) suy ra tam
giác BMN đều nên
==
0
1
60 .
2
NKB NB
Trên y KN lấy điểm P sao cho KP = KB thì tam giác
KPB đều. Xét tam giác MKB NPB ta có:
0
, , 120= = = =KB BP MB NB MKB NPB
suy ra
( . . )MKB NPB c g c =
.KM PN KM KB KN = + =
Vy
2.KM KN KB KN+ + =
D thy
2KN R
nên
4,KM KN KB R+ +
du bng xy ra khi và ch khi K, O, N thng hàng.
T đó suy ra điểm K là giao điểm ca NO vi (O) (K khác N).
Cách 2: Áp dụng định lý Ptolemy cho t giác ni tiếp KMNB ta có:
. . .KM BN KB MN KN MB+=
chú ý rng:
==BM BN MN
suy ra
KM KB KN+=
. Phn còn lại ta làm như trên.
1.1. Định Ptolemy cho t giác ni tiếp: Cho t giác ABCD ni tiếp đường tròn (O). Khi đó ta
có:
. . .AB CD AD BC AC BD+=
CHUYÊN ĐỀ BỒI DƯỠNG HC SINH GII HÌNH HC 9
THCS.TOANMATH.com | 2
Chng minh:
Trên đường chéo BD lấy điểm E sao cho
.DAE BAC=
Ta có
DAE BAC=
ADE ACB=
(cùng chn
AB
) nên
( . ) =
AD DE
ADE ACB g g
AC BC
. . (1).AD BC AC DE=
Do
=DAE CAB
nên
,DAC EAB=
li
ABE ACD=
(cùng chn
)AD
( . ) . . (2) = =
AB BE
ABE ACD g g AB CD AC BE
AC CD
T (1) và (2) suy ra
. . ( ) .AB CD AD BC AC BE DE AC BD+ = + =
1.2. Định lý Shooten
Cho tam giác đu ABC ni tiếp đường tròn (O). Chng minh
rng vi mỗi điểm M bt k nằm trên đường tròn (O) thì mt
trong ba đoạn MA, MB, MC một đoạn độ dài bng tng
độ dài hai đoạn kia.
Chng minh
Xét điểm M nm trên cung nh BC.
Áp dụng định Ptolemy cho t giác ni tiếp ABMC, ta
. . .MABC MB AC MC AB=+
AB AC BC==
nên
MA MB MC=+
Tương tự nếu điểm M nm trên cung nh AC AB thì ta lần lượt
MB MC MA=+
MC MA MB=+
Suy ra đpcm.
Cách khác để chng minh:
MA MB MC=+
(trường hợp điểm M nm trên các cung AB, AC tương tự).
Trên MA ly điểm I sao cho
MI MB=
ta cn chng minh
.MC AI=
Tht vy, ta
60
o
BMI ACB==
MB MI=
nên tam giác BIM đều, do đó
BI BM=
60
o
IBM =
Ta li có
60
o
ABC =
nên
ABC IBM=
, suy ra
.CBM ABI=
D dàng chứng minh được
( . . )BCM BAI c g c =
nên
.MC AI=
Bài 2. (Trích đề TS lp 10 TP Hà Ni Năm 2007)
Cho đường tròn (O; R) tiếp xúc vi (d) ti A. Trên (d) ly điểm H không trùng vi A
AH R
.
Qua H k đường thng vuông góc vi (d), đường thng này ct (O) ti B, E (E nm gia BH).
a. Chng minh:
ABE EAH=
ABH EAH
CHUYÊN ĐỀ BỒI DƯỠNG HỌC SINH GIỎI HÌNH HỌC 9
3 | THCS.TOANMATH.com
b. Ly C trên (d) sao cho H trung đim ca AC, CE ct AB ti K. Chng minh: AHEK t giác
ni tiếp.
c. Tìm v trí ca H để
3AB R=
Gii
a. Do AH tiếp tuyến ca (O) ti A nên
==
1
.
2
ABE EAH AE
Xét tam giác
,ABH EAH
ta có:
AHB
chung
ABE EAH=
( . )ABH EAH g g
.
b. E nm trên trung trc ca AC nên
EAH ECH=
li có:
EAH EBA=
suy ra
ECH EBA=
. Suy ra
( . )ABH ACK g g
suy ra
90 .
o
CKA BHA==
T giác AHKE
180
o
AHE EKA+=
suy ra AHKE ni tiếp (Tổng hai góc đối
bng
180 )
o
c. Khi
3AB R=
thì sđ
120=
o
AEB
Do tam giác ABC cân ti B nên
3AB BC R==
li có
120 60 30= = = =
o o o
AEB BEC HEA HAE
3
30 .
2
o
R
HBA AH = =
Vậy điểm H nm trên (d) sao cho
3
2
R
AH =
để
3AB R=
Bài 3. (Trích đề TS lp 10 TP Hà Ni Năm 2008).
Cho đường tròn (O; R) đường kính AB đim E bt k nằm trên đường tròn (E khác A, B).
Đưng phân giác góc AEB cắt đoạn AB ti F và ct (O) tại điểm th 2 là K
a. Chng minh:
KAF KEA
b. Gi I là giao điểm ca OE vi trung trc ca EF. Chng minh (I) bán kính IE tiếp xúc vi (O) ti
E và tiếp xúc vi AB ti F.
c. Chng minh: MN // AB, trong đó M, N lần lượt là giao điểm th 2 ca AE, BE vi (I).
d. Tìm GTNN ca chu vi tam giác KPQ theo R khi E chuyển động trên (O), vi P giao điểm ca
NFAK, Q là giao điểm ca MF, BK.
Gii
CHUYÊN ĐỀ BỒI DƯỠNG HC SINH GII HÌNH HC 9
THCS.TOANMATH.com | 4
a.Vì EK phân giác ca góc
AEB
nên
AK KB AEK KAB= =
Xét tam giác
,KAF KEA
ta có:
AEK KAB=
va
AKE
chung.
Suy ra
( . )KAF KEA g g
b.
, ( )I OE E O
nên
( ; )I IE
tiếp xúc vi (O;
OE) ti E. I nm trên trung trc ca EF nên IE
= IF hay
( ; )F I IE
.
Ta có:
,IEF IFE OEK OKE==
suy ra
/ / ,IFE OKE IF OK=
OK AB
(do K điểm chính
gia cung AB) t đó suy ra
(1)IF AB
hay (I) bán kính IE tiếp xúc AB ti F.
c. Do
90
o
MEN MN=
đường kính ca (I), hay M, I, N thng hàng. Li
2 90
o
FIN FEN==
nên
(2).FI MN
T (1) và (2) suy ra MN // AB.
d. T chng minh câu b ta suy ra
90 90 .
oo
MFN PFN= =
Ta
EAB EKB=
(cùng chn cung
EB),
EAB EMN=
ng v),
EMN EFN=
(cùng chn cung EN) suy ra
/ / ,EFN EKB FN BK=
mt khác
AK KB AK NF
ti P suy ra PFQK là hình ch nht và tam giác APF vuông cân ti
P nên KQ = PF = AP.
Ta hiu chu vi tam giác KPQ c thì
.c KP KQ PQ KP PA KF KA KF= + + = + + = +
Ta
2,KA R KF KO R= =
nên
( )
12cR+
. Du bng xy ra khi ch khi
.FO
Hay E điểm
chính gia cung AB.
Bài 4. (Trích đề TS lp 10 TP Hà Ni Năm 2009)
Cho đường tròn (O; R) điểm A nm ngoài (O), k các tiếp tuyến AB, AC đến (O) (B, C các
tiếp điểm).
a. Chng minh: ABOC là t giác ni tiếp.
b. Gi E là giao điểm ca BCOA. Chng minh:
BE OA
2
.OE OA R=
c. Trên cung nh BC ca (O) lấy điểm K bt k (K khác B, C). Tiếp tuyến ti K ca (O) ct AB, AC
lần lượt ti P, Q. Chng minh: Chu vi tam giác APQ không đổi khi K chuyển động trên cung nh
BC.
d. Đường thng qua O vuông góc vi OA cắt các đường thng AB, AC lần t ti M, N. Chng
minh:
.PM QN MN+
Gii
CHUYÊN ĐỀ BỒI DƯỠNG HỌC SINH GIỎI HÌNH HỌC 9
5 | THCS.TOANMATH.com
a. AB, AC các tiếp tuyến ca (O) nên
90
o
ABO ACO==
.
T giác ABOC
180
o
ABO ACO+=
nên ABOC t
giác ni tiếp (tổng 2 góc đối bng
180 ).
o
b. Theo tính cht 2 tiếp tuyến ct nhau ta
BC AO
ti E. Áp dng h thức lượng trong tam giác
vuông ABO ta có:
22
.OE OA OB R==
.
c. Vì điểm A c định nm ngoài (O)nên AB, AC c định suy ra
AB AC+
không đổi.
Theo tính cht 2 tiếp tuyến ct nhau ta có:
,PK PB QK QC==
suy ra chu vi tam giác APQ là:
AP AQ PQ AP AQ PB QC AB AC+ + = + + + = +
không đổi.
d. Gi s BK ct PO ti I, CK ct OQ ti J thì
90
o
KIO KJO==
nên t giác KIOJ ni tiếp nên
//KOJ KIJ QKC KBC IJ BC= = =
hay
.IJ AO
T đó ta có:
90 .
o
BPO BKO IJO JAO QON= = = =
Xét
MOP
NQO
có:
PMO ONQ=
MPO QON=
suy ra
2
( . ) . . .
4
= = =
PM ON MN
MOP NQO g g PM QN OM ON
OM QN
Ta cũng có:
2 2 2
( ) ( )
..
4 4 4
++
+
PM QN PM QN MN
PM QN PM QN MN
Dấu đẳng thc xy ra khi ch khi
//PM QN PQ MN=
hay K điểm chính gia cung nh
BC.
Nhn xét: Đây là bài toán khá hay trong đề tuyn sinh vào lp 10 ca TP Hà Ni.
Mt s bài toán ôn tp thêm:
1. Cho đường tròn tâm O và điểm A nm ngoài đường tròn. K hai tiếp tuyến AB, AC vi (O) vi B,
C các tiếp điểm. Trên cung nh BC ly một đim M ri k các đường vuông góc MI, MH, MK
xung BC, CA, AB. Gi P, Q ln lượt là giao điểm ca các cặp đường thng BM IK, CMIH.
a. Chng minh: các t giác BIMK, CIMH là t giác ni tiếp.
b. Chng minh:
2
.MI MH MK=
c. Chng minh: IPMQ ni tiếp ri suy ra
.PQ MI
Tìm v trí điểm M để MI.MH.MK đạt GTLN.
Gii
CHUYÊN ĐỀ BỒI DƯỠNG HC SINH GII HÌNH HC 9
THCS.TOANMATH.com | 6
a. T gi thiết:
90
o
BKM BIM==
suy ra t giác BKMI ni tiếp.
Tương tự cho t giác CIMH, AKMH
b. t giác BKMI ni tiếp nên:
MKI MBI=
(cùng chn cung
MI). Mt khác ta có:
MBI MCH=
(tính cht góc to bi tiếp
tuyến y cung). Nhưng
MCH MIH=
(cùng chn cung MH
ca t giác ni tiếp MHCI). Suy ra
MKI MIH=
Hoàn toàn tương tự ta có:
MIK MHI=
nên
MIK MHI
(g.g).
Suy ra
2
.
MI MH
MI MH MK
MK MI
= =
c. Ta có:
180
o
PMQ PIQ BMC PIM QIM BMC MBA MCA BMC MCB MBC+ = + + = + + = + + =
Do đó tứ giác PIQM ni tiếp (Tổng hai góc đi nhau bng
180 ).
O
PIQM ni tiếp suy ra
= = =MPQ MIQ MKI MBI
suy ra
//PQ BC
hay
.MI PQ
d. T chng minh câu b) ta
23
. . .MI MH MK MI MI MH MK= =
. Suy ra
..MI MH MK
ln
nht khi và ch khi MI ln nht. Hay M là điểm chính gia cung nh BC không cha A.
2. Cho đường tròn tâm (O). T điểm A c định ngoài (O) k tiếp tuyến AB, AC ti (O) (B, C
tiếp điểm). Lấy đim M trên cung nh BC. Gi D, E, F th t hình chiếu t M đến BC, AC, AB.
Gi MB ct DF ti P, MC ct DE ti Q. Chứng minh đưng thng nối giao điểm ca hai đường tròn
ngoi tiếp tam giác MPFMQE luôn đi qua một điểm c định.
Gii
Gi s đường tròn ngoi tiếp tam giác MPF
MQE ct nhau ti M, N. Đưng thng MN ct
PQ, BC theo th t ti KI.
Ta các t giác MDCE, MDBF ni tiếp nên
MCE MDE MBC==
;
.MBF MDF MCB==
Suy ra
180
o
PMQ PDQ PMQ PDM QDM
PMQ MCB MBC
+ = + +
= + + =
Do đó tứ giác MPDQ là t giác ni tiếp.
Suy ra
,MQP MCB MEQ==
suy ra KQ tiếp tuyến của đường tròn ngoi tiếp
.MQE
Tương tự
KP tiếp tuyến của đường tròn ngoi tiếp tam giác MFP. Ta
2
.,=KM KN KQ
2
..KM KN KP=
Suy ra
.KP KQ=
CHUYÊN ĐỀ BỒI DƯỠNG HỌC SINH GIỎI HÌNH HỌC 9
7 | THCS.TOANMATH.com
Xét tam giác MBC, PQ // BC, KP = KQ. Theo định lý Thales suy ra I là trung điểm BC. Vy MN đi
qua điểm c định I là trung điểm BC.
3. Cho tam giác ABC cân đỉnh A. Gi O là trung điểm ca BC. Đường tròn (O) tiếp xúc vi AB E
tiếp xúc vi AC F. Đim H chy trên cung nh
EF
tiếp tuyến của đường tròn ti H ct ti AB, AC
lần lượt ti M, N. Xác định v trí của điểm H để din tích tam giác AMN đạt giá tr ln nht.
Gii
D thy OM, ON lần lượt là phân giác
,EOM FOH
T đó ta có:
( )
180
.)
2
= =
o
BAC
MON ABC MBO OCN g g
2
. . (1)
4
MB BO BC
BM CN OB OC const
OC CN
= = = =
Ta li
AMN ABC BMNC
S S S=−
nên
AMN
S
đạt giá tr ln
nht khi và ch khi
BMNC
S
đạt giá tr nh nht.
Gi R là bán kính của đường tròn (O), ta có:
1
()
2
BMNC BOM MON NOC
S S S S R BM MN NC= + + = + +
( )
1
2
2
R BE CF EM FN

= + + +

()MN EM FN=+
( )
R BE EM FN= + +
( ).=BE CF
( 2 ) ( ) (2)= + + = + R BE BM CN BE R BM CN BE
Áp dng bất đẳng thc Cô-si, t (1) và (2) suy ra
( )
.
2
BMNC
BC
S R BM CN BE R BE

=


Dấu “=” xy ra khi và ch khi
//BM CN MN BC=
khi ch khi H giao đim của đường
trung trc ca BC với đường tròn (O). Vy din tích tam giác AMN đạt giá tr ln nht khi Hgiao
của đường trung trc ca BC với đường tròn (O).
Bài 5. (Trích đề TS lp 10 TP Hà Ni Năm 2010)
Cho đường tròn (O;R), đường kính AB = 2R điểm C thuộc đường tròn đó
( , ).C A B
D thuc
dây BC
( , ).D B C
Tia AD ct cung nh BC ti E, tia AC ct BE ti F.
a. Chng minh t giác FCDE ni tiếp.
b. Chng minh
..DA DE DB DC=
c. Chng minh
CFD OCB=
. Gi I tâm đường tròn ngoi tiếp t giác FCDE, chng minh IC
tiếp tuyến ca (O).
CHUYÊN ĐỀ BỒI DƯỠNG HC SINH GII HÌNH HC 9
THCS.TOANMATH.com | 8
d. Cho biết
,DF R=
chng minh
tan 2.AFB =
Gii
a. Do C, E nằm trên đường tròn đường kính AB nên
90 90 .
oo
ACB AEB FCD FED= = = =
T giác FCDE
180
o
FCD FED+=
nên FCDEt giác ni
tiếp (tổng hai góc đối bng
180 ).
o
b. T giác ACED ni tiếp (O) nên
ACB AEB=
(cùng chn
cung AB). Ta có
CDA EDB=
ối đỉnh) nên
( . ) =
DA DB
ACD BED g g
DC DE
hay
..DA DE DB DC=
.
c. T giác FCDE ni tiếp nên
CFD CED=
(cùng chn cung CD). T giác ACEB ni tiếp nên
CEA CBA=
(cùng chn cung AC).
CBA OCB=
suy ra
CFD OCB=
đpcm. Do AE, BC hai
đường cao ca tam giác AFB nên D trc tâm ca tam giác AFB. Để ý rằng 4 điểm F, C, D, E
nằm trên đường tròn đường kính DF nên I trung điểm ca DF suy
ra
1
(1)
2
IE IF DF IEF IFE= = =
. Ta cũng
(2)OEB OBE=
. T (1) (2) ta suy ra
90 90
oo
IEF OEB IFE OBE IEO+ = + = =
hay IE là tiếp tuyến ca (O).
d. Ta
2. = =
CB BA
ACB DCF
CF FD
Trong tam giác vuông CFB ta có:
tan 2
CB
CFB
CF
==
đpcm.
Bài 6. (Trích đề TS lp 10 TP Hà Ni Năm 2011)
Cho đường tròn tâm O, đường kính AB = 2R. Gi
1
d
2
d
lần lượt hai tiếp tuyến của đường tròn
(O) tại hai điểm A B. Gi I trung điểm ca OA E điểm thuộc đường tròn (O) (E không
trùng vi A B). Đường thng d đi qua E vuông góc vi EI cắt hai đường thng
12
,dd
lần lượt
ti M, N.
a. Chng minh AMEI là t giác ni tiếp.
b. Chng minh
ENI EBI=
90
o
MIN =
.
c. Chng minh AM.BN = AI.BI
d. Gi F đim chính gia ca cung AB không cha E của đường tròn (O). Hãy tính din tích ca
tam giác MIN theo R khi ba điểm E, I, F thng hàng.
Gii
CHUYÊN ĐỀ BỒI DƯỠNG HỌC SINH GIỎI HÌNH HỌC 9
9 | THCS.TOANMATH.com
a. Do
90
o
MAI MEI==
suy ra
180
o
MAI MEI+=
hay MAIE
là t giác ni tiếp (Tổng hai góc đối bng
180 )
.
b. Do MAIE t giác ni tiếp nên
EMI EBI=
(cùng chn
cung EI). Chứng minh tương t câu a ta
NEIB
t giác
ni tiếp nên
ENI EBN=
(cùng chn cung NB).
T đó suy ra
90 =
o
MIN AEB MIN
c. Xét tam giác vuông MAIIBN ta có:
180 90
oo
MIA MIN NIB NIB INB= = =
Suy ra
( )
. . . = =
MA IB
MAI IBN g g AM BN IA IB
AI BN
d. Dng
.EH AB
Khi F là điểm chính gia cung AB thì EF là phân giác ca góc
AEB
suy ra:
2 2 2 2 2 2 2 2
1 2 10
9 4 .
3 5 5
= = + = + = = = =
EA IA
EA EB EA EA AB R EA R EA R
EB IB
3 10
5
EB R=
. Ta cũng có:
22
22
2
5
4
+
= = = =
MIN
AEB
S
EI IF OI OF
MIN AEB
S EH OF OF
suy ra
2
5 5 1 5 1 10 3 10 3
. . . .
4 4 2 4 2 5 5 4
MIN AEB
S S EA EB R R R= = = =
Bài 7: (Trích đề TS lp 10 TP Hà Ni Năm 2012)
Cho đường tròn (O; R) đường kính AB. Bán kính CO vuông góc vi AB, M là điểm bt k trên cung
nh AC
( ),M AC BM
ct AC ti H. Gi K là hình chiếu ca H trên AB.
a. Chng minh t giác CBKH là t giác ni tiếp.
b. Chng minh
ACM ACK=
c. Trên đoạn thng BM lấy điểm E sao cho
BE AM=
. Chng minh tam giác ECM tam giác
vuông cân ti C.
d. Gi d tiếp tuyến của đường tròn (O) tại điểm A. Cho P một điểm nm trên d sao cho hai
điểm P, C nm trong cùng mt na mt phng b AB
.
.
AP MB
R
MA
=
Chứng minh đường thng PB
đi qua trung điểm của đoạn thng HK.
Gii
CHUYÊN ĐỀ BỒI DƯỠNG HC SINH GII HÌNH HC 9
THCS.TOANMATH.com | 10
a. C nằm trên đường tròn
;
2
AB
O



nên
90
o
ACB =
, ta
90
o
HKB =
180
o
ACB HKB + =
nên CHKB t giác ni tiếp (tng 2
góc đối bng
180 )
o
b. T giác AMCB ni tiếp nên
ACM ABM=
(cùng chn cung
AM) (1). T giác CHKB ni tiếp nên
HCK HBK=
(cùng chn
cung HK) (2).
T (1) và (2) ta suy ra
ACM ACK=
c. Do C điểm chính gia cung AB nên ACB tam giác vuông cân ti C, suy ra AC = CB, ta cũng
có:
MAC EBC=
(cùng chn cung MC), AM = BE (gt) suy ra
( . . )AMC BEC c g c =
suy ra CM =
CE. Li có
90
o
MCE MCA ACE ECB ACE= + = + =
nên MCE là tam giác vuông cân ti C.
d. Gi s MB kéo dài ct tiếp tuyến ti A N. Ta d chứng minh đưc:
NAB AMB
suy ra
2.
2
= = =
MB AB R NA MB
R
MA NA NA MA
kết hp với điều kin:
.AP MB
R
MA
=
suy ra
..
2
AP MB NA MB
MA MA
=
hay
2NA AP P=
trung điểm AN. Gi s BP ct HK ti I. Do HK // NA theo định Thales ta có:
HI IK BI
NP PI BP
==
NP PI=
suy ra
HI IK=
hay là trung điểm ca HK.
Bài 8. (Trích đề TS lp 10 TP Hà Ni Năm 2013)
Cho đường tròn (O) điểm A nm bên ngoài (O). K hai tiếp tuyến AM, AN với đường tròn (O).
Một đường thng d đi qua A cắt đường tròn (O) ti hai điểm B C
(,AB AC d
không đi qua tâm
O).
a. Chng minh t giác AMON ni tiếp.
b. Chng minh
2
..AN AB AC=
Tính độ dài đoạn thng BC khi
4 , 6 .AB cm AN cm==
c. Gi I là trung điểm BC. Đường thng NI cắt đường tròn (O) tại điểm th hai T.
Chng minh MT // AC.
d. Hai tiếp tuyến của đường tròn (O) ti B C ct nhau ti K. Chng minh K thuc một đường
thng c định khi d thay đổi và thỏa mãn điều kiện đầu bài.
Gii
CHUYÊN ĐỀ BỒI DƯỠNG HỌC SINH GIỎI HÌNH HỌC 9
11 | THCS.TOANMATH.com
a. Do AM, AN các tiếp tuyến ca (O) nên
90
o
AMO ANO==
suy ra
180
o
AMO ANO+=
hay
t giác AMON ni tiếp trong đường tròn đường kính
AO.
b. Xét tam giác ANB, ACN.
Ta
ANB ACN=
(Góc to bi tiếp tuyến và y
cung),
NAC
chung.
Nên
( . )ANB ACN g g
suy ra
AN AC
AB AN
=
2
.AN AB AC=
c. Do I trung điểm ca BC nên
90
o
BI BC AIO =
kết hp với câu a ta suy ra 5 đim A, M, I,
O, N cùng nằm trên đường tròn đường kính AO suy ra
AIN AMN=
(cùng chn cung AN). Mt
khác ta cũng có:
AMN MTN=
(góc to bi tiếp tuyến và dây cung).
T đó suy ra
AIN MTN=
, hai góc này đồng v suy ra MT // AI (đpcm).
d. Gi s MN ct AO ti H. Trong tam giác vuông ANO ta
2
.AN AH AO=
kết hp vi câu b ta
suy ra
..=AH AO AB AC
nên
( . )ABH AOC g g
suy ra
AHB ACO BHOC=
ni tiếp (Góc ngoài
một đỉnh bằng góc trong đỉnh đối din). Gi s tiếp tuyến ti B, C ct nhau tại đim K, chng minh
như câu a ta KBOC ni tiếp, kết hp vi BHOC ni tiếp suy ra 5 điểm K, B, H, O, C cùng nm
trên đường tròn đường kính KO suy ra
90
o
KHO =
. Mt khác
90
o
MHO =
suy ra K, M, H thng
hàng. Hay điểm K nằm trên đường thng c định MN.
Bài 9. (Trích đề TS lp 10 TP Hà Ni Năm 2014)
Cho đường tròn (O;R) đường kính AB c định. V đường kính MN của đường tròn (O; R)
( , )M A M B
. Tiếp tuyến của đường tròn (O; R) ti B cắt các đường thng AM, AN lần lượt ti
các điểm Q, P.
a. Chng minh t giác AMBN là hình ch nht.
b. Chng minh bốn điểm M, N, P, Q cùng thuc một đường tròn.
c. Gi E trung đim ca BQ. Đường thng vuông góc vi OE ti O ct PQ tại điểm F. Chng
minh F là trung điểm ca BPME // NF.
d. Khi đường kính MN quay quanh tâm O thỏa mãn điều kiện đề bài, xác định v trí của đường
kính MN để t giác MNPQ có din tích nh nht.
Gii
CHUYÊN ĐỀ BỒI DƯỠNG HC SINH GII HÌNH HC 9
THCS.TOANMATH.com | 12
a. AB, MN các đường kính ca (O) nên AMBN hình bình
hành. Mt khác
90
o
MAN =
nên AMBN là hình ch nhật. (cũng có thể
lp lun: T giác AMBN 4 góc vuông nên AMBN hình ch
nht).
b. Xét t giác QMNP ta
MNA MBA=
(cùng chn cung MA ca t
giác ni tiếp AMBN). Mà
MBA MQB=
(cùng ph vi góc
MBQ
). T
đó suy ra
MNA MQP=
nên t giác QMNP ni tiếp (góc ngoài đỉnh N
bằng góc trong đối din với đỉnh N).
c. Do tam giác QMB vuông ti M E trung đim ca
= =BQ ME EQ EB
(tính cht trung tuyến ng vi cnh huyn ca
tam giác vuông). T đó suy ra
( . . )OME OBE c c c=
nên
90==
o
OME OBE
hay ME tiếp tuyến
ca (O). Vì ME các tiếp tuyến ca (O) nên
,OE MB
do
/ / ,MB AN OE AN⊥
mt khác
AB PQ
suy ra O trc tâm tam giác
AEP OE AP⊥
//OF OE OF AP⊥
nên OF
đường trung bình ca tam giác ABP hay F là trung điểm ca BP.
d. Dng
AH MN
thì: Ta có
( )
11
. . ( ) ( ) 2 .
22
= = = = +
MNPQ APQ MAN
S S S AB PQ MN AH R PQ AH R BP PQ AH R BP PQ AO
( )
22
23= =R AB AO R
(Vì
2
.)BP PQ AB=
.
Dấu đẳng thc xy ra khi và ch khi
, / /BP PQ AH AO MN PQ= =
hay
MN AB
Bài 10. (Trích đề TS lp 10 TP Hà Ni Năm 2015)
Cho nửa đường tròn (O;R) đường kính AB. Ly điểm C trên đoạn thng OA (C khác A, O).
Đưng thng qua C vuông góc vi AB ct na (O) ti K. Gi M điểm trên cung KB (M khác K,
B). Đường thng CK ct AM, BM lần lượt ti H, D. Đường thng BH ct nửa đường tròn tại điểm
th 2 là N.
a. Chng minh: ACMD là t giác ni tiếp.
b. Chng minh: CA.CB = CH.CD.
c. Chng minh: A, N, D thng hàng và tiếp tuyến ti N của đường tròn đi qua trung điểm ca DH.
d. Khi M di động trên cung KB, chứng minh: Đường thng MN luôn đi qua một điểm c định.
Gii
CHUYÊN ĐỀ BỒI DƯỠNG HỌC SINH GIỎI HÌNH HỌC 9
13 | THCS.TOANMATH.com
a. Do điểm M nm trên nửa đường tròn đường kính AB
nên
90 90
oo
AMB AMD= =
Ta cũng có
90
o
DCA =
nên
90
o
AMD DCA==
Suy ra 4 đim D, M, C, A nằm trên đường tròn đường
kính AD hay ACMD là t giác ni tiếp.
b. Do AM, CD là các đường cao ca tam giác ADB nên H
trc tâm ca tam giác suy ra
CDB HAC=
(cùng ph
vi
DBA
). Suy ra
( . ) . . = =
DC AC
DCB ACH g g CD CH AC BC
CB CH
(đpcm).
c. Do BH ct nửa đường tròn ti N nên
90
o
ANB =
, ta cũng
BH AD
suy ra
N AD
(qua A ch
k được 1 đường thng vuông góc vi BH). Hay A, N, D thng hàng. Gi I trung điểm ca DH.
Ta s chng minh: IN tiếp tuyến ca (O). Tht vy ta có:
1
(1).
2
ID IN IH HD IND IDN= = = =
Tương tự ta cũng có:
(2).ONA OAN=
T (1), (2) suy ra
90
o
IND ONA IDN OAN+ = + =
suy ra
90
o
INO =
hay IN là tiếp tuyến ca (O) (đpcm).
d. Gi s MN ct BC ti P. Ta chng minh P điểm c định. Tht vy do AMNB ni tiếp nên
PNA PBM=
suy ra
( . ) . . (3). = =
PN PB
PMA PBM g g PM PN PA PB
PA PM
Theo câu c) ta
IN tiếp tuyến ca (O) hoàn toàn tương tự ta cũng có: IM tiếp tuyến ca (O) nên suy ra
90= = =
o
INO ICO IMO
hay 5 đim I, N, C, O, M cùng nm trên một đường tròn đường kính IO
suy ra t giác MNCO ni tiếp nên ta cũng suy ra PM.PN = PC.PO(4). T (3) (4) ta suy ra
..PA PB PC PO=
. T P ta k tiếp tuyến
'PK
vi (O) thì
2
. ' .PA PB PK PC PO==
t đó suy ra
' ' ( . . )PCK PK O c g c
nên
' 90 ' .=
o
K CP K K
Hay P giao điểm ca tiếp tuyến ti K vi
AB. Vy P là điểm c định.
Chú ý: Ý tưởng tạo câu d) trong bài y câu d) trong đ TS năm 2013 ging nhau nên vic
phát hin P là giao điểm ca tiếp tuyến ti K vi AB là hoàn toàn t nhiên.
Ngoài ra ta cũng có thể phát biu câu d) theo mt cách khác:
“Qua điểm P ngoài đưng tròn (O) k tiếp tuyến PK cát tuyến PAB đi qua tâm O, cát tuyến KMN
bt k. Gi H là giao điểm ca AM, BN, C là hình chiếu ca H trên AB. Khi đó C, H, K thng hàng”.
CHUYÊN ĐỀ BỒI DƯỠNG HC SINH GII HÌNH HC 9
THCS.TOANMATH.com | 14
Chng minh:
T các t giác ANHC, CHMB, MNAB ni tiếp ta các
biến đổi góc sau:
ANC AHC ABM PNA= = =
suy ra
2PNC PBM POM==
(theo tính cht góc ngoài tam giác cân BOM) suy ra t giác NCOM ni tiếp.
Ta cũng có:
2
. . 90 .= = = =
o
PC PK
PK PM PN PC PO PCK PKO PCK
PK PO
KC, HC
cùng vuông góc vi AB nên K, H, C thng hàng.
Bài 11. (Trích đề TS lp 10 TP Hà Ni Năm 2016)
Cho đường tròn (O) đim A nm bên ngoài (O). K tiếp tuyến AB vi (O) (B tiếp điểm)
đường kính BC. Trên đoạn CO lấy điểm I (I khác C, O). Đường thng AI ct (O) tại hai điểm D, E
(D nm gia AE). Gi H là trung điểm DE.
a. Chứng minh: 4 điểm A, B, O, H cùng nm trên một đường tròn.
b. Chng minh:
AB BD
AE BE
=
c. Đường thng (d) qua E song song vi AO, (d) ct BC ti K. Chng minh: HK // DC.
d. Tia CD ct AO ti P, tia EO ct BP ti F. Chng minh: BECF là hình ch nht.
Gii
a. H trung điểm ca DE nên
OH DE
(liên h
đường kính và dây cung).
Suy ra
90
o
AHO ABO==
nên 4 điểm A, B, O, H cùng
nằm trên đường tròn đường kính AO.
b. AB tiếp tuyến ca (O)
ABD BEA=
(góc to
bi tiếp tuyến và dây cung).
Suy ra
( . ) =
AB BD
ABD AEB g g
AE EB
(đpcm).
c. Để chng minh: HK // DC ta chng minh:
EHK EDC=
. Ta li
EDC EBC=
nên ta quy v
chng minh:
EKH EBC=
Chng minh: BHKE ni tiếp.
Tht vy: do
//EK AO BKE AOK=
(so le trong). Ta cũng
180
o
AOK AOB=−
AOB AHB=
(cùng chn cung AB của đường tròn ngoi tiếp t giác ABOH). Suy ra
180
o
AOK AHB BHE= =
hay
BKE BHE=
suy ra t giác BHKE ni tiếp (có hai đnh liên tiếp K,
H cùng nhìn cnh BE mt góc bằng nhau) đpcm.
CHUYÊN ĐỀ BỒI DƯỠNG HỌC SINH GIỎI HÌNH HỌC 9
15 | THCS.TOANMATH.com
d. Để chng minh: BECF hình ch nht ta s chứng minh: Đim F nm trên (O) tc chng
minh:
90
o
BEP =
hay
90
o
EBC CBP+=
. Tht vy:
Xét tam giác EHB và tam giác COP ta có:
,EHB COP BED BCD==
suy ra
( . ) ( . . ) = = =
EB EH ED
EHB COP g g EDB CBP c g c EDB CBP
CP CO CB
∽∽
EDB
ph vi
góc
CDE
CDE EBC=
suy ra
90
o
EBP EBC CBP EBD CDE= + = + =
hay BECF hình ch
nht.
Cách tiếp cn khác
Ta thấy 3 bài toán trong các năm 2013, 2015, 2016 đều các tính cht cát tuyến, tiếp tuyến ca
đường tròn mà trc tiếp là liên quan đến bài toán sau:
Bài toán: Cho đường tròn (O) một điểm M nằm ngoài đường tròn (O), qua M k các tiếp tuyến
MA, MB đến đường tròn (O) (A, B các tiếp điểm) dng cát tuyến MCD sao cho MC < MD.
Gi E là trung điểm ca CD, H là trung điểm ca AB.
1. T giác CHOD ni tiếp.
2. AB chứa đường phân giác ca góc
CHD
.
3. V đường kính AQ, các đường thng QC,
QD cắt đường thng MO lần lượt ti X, Y thì O
là trung điểm ca XY.
Gii
1. Vì MA là tiếp tuyến ca (O) nên
MAC ADC=
(Tính cht góc to bi tiếp tuyến và dây cung). Xét hai tam giác MACMDA ta có:
,MAC ADC AMD=
chung nên
( . )MAC MDA g g
suy ra
2
. (1).
MA MD
MA MC MD
MC MA
= =
Áp
dng h thc ng trong tam giác vuông MAO,
AH MO
ta có
2
. (2).MA MH MO=
T (1) và (2)
suy ra
..MH MO MC MD=
hay
.
MH MD
MC MO
=
Xét tam giác MCH, MOD ta có:
MH MD
MC MO
=
OMD
chung nên
( . )MCH MOD g g
suy ra
MHC MDO=
t giác CHOD ni tiếp (góc ngoi
đỉnh H bằng góc trong đối diện đỉnh H) (3).
2. T 3) ta có
(4)MHC MDO=
(5)MDO OCD=
(do tam giác COD cân ti O). Mt khác ta có
(6)OCD OHD=
(góc ni tiếp cùng chn
OD
). T (4), (5), (6) suy ra
MHC OHD=
. Các góc
CHUYÊN ĐỀ BỒI DƯỠNG HC SINH GII HÌNH HC 9
THCS.TOANMATH.com | 16
,CHA DHA
ph vi các góc
,MHC OHD
tương ng nên suy ra
CHA DHA=
hay AH phân giác
ca góc
CHD
.
3. Do AQ đường kính ca (O) nên
90
o
ADQ ADYH=
là t giác ni tiếp. Suy ra
AYD AHD=
.
Mt khác theo 1, 2) ta có CHOD ni tiếp và AH là phân giác ca góc
CHD
suy ra
11
22
AHD CHD COD CQD AYD CQD= = = =
suy ra AY // CQ. Xét hai tam giác AOY, QOX ta
có:
,,= = =OA OQ AOY QOX YAO XQO
nên
( . . )AOY QOX g c g =
suy ra OX = OY.
Chú ý: Ta cũng có thể chứng minh theo cách khác như sau: Tứ giác AEOM ni tiếp nên
, = = = AEC AEM AOM QOY YQO DQA DCA ECA
nên
=
QY CA
OYQ EAC
QO CE
Hay
( )
..
11
22
= =
QY CA QY CA
QYA CAD c g c
QA CD
QA CD
Nên
//YAQ ADC ECA AQC AY QX= = =
nên t giác AXQY là hình bình hành suy ra OX = OY.
Cho đường tròn (O) điểm A nm ngoài (O), qua A k tiếp tuyến AB, dựng đưng kính BC. Trên
đoạn CO lấy điểm I, đưng thng AI ct (O) ti D, E (D nm gia A E). Gi H trung điểm
DE. Tia CD ct AO ti P, tia EO ct BP ti F. Chng minh: BECF là hình ch nht (Trích câu d) đề
tuyn sinh lp 10 TP Hà Ni 2016).
Gii
Kéo dài CE ct AO ti Q theo 3 ta có:
OP = OQ nên CPBQ là hình bình hành.
Suy ra CEBF cũng là hình bình hành.
Hơn nữa
90
o
CEB =
nên CEBF là hình ch nht.
Bài toán tương tự:
Cho tam giác ABC đường cao AH, đường tròn (O) đưng kính AH ct AB, AC ti D, E.
a. Chng minh: BDEC ni tiếp.
b. Đường thng ED ct BC ti S. Chng minh:
2
..SH SB SC=
c. SO ct AB, AC lần t ti M, N. HM, HN ct DE lần lượt ti P, Q. Chng minh: BP, CQ, AH
đồng quy. (Trích đề tuyn sinh lp 10 chuyên Toán ĐHSP Hà Nội 2016).
Gii
CHUYÊN ĐỀ BỒI DƯỠNG HỌC SINH GIỎI HÌNH HỌC 9
17 | THCS.TOANMATH.com
Do
AED AHD ABC==
suy ra BDEC t giác ni tiếp suy
ra SB.SC = SD. SE.
Mt khác SH tiếp tuyến ca (O) ti H nên
2
.SH SD SE=
suy ra
2
..SH SB SC=
Theo tính cht 3 ta có:
OM = ON nên AMHN là hình bình hành.
Suy ra AM // HN, AN // HM li có
MHB ACB ADE==
nên BDPH ni tiếp suy ra
,BP MH
tương
t
CQ NH
suy ra
,CQ AB BP AC⊥⊥
nên các đường thng BP, CQ, AH đồng quy ti trc tâm K
nằm trên đường cao AH.
Bài 12. (Trích đề tuyn sinh vào lớp 10 chuyên Toán năm 2011 – TP Hà Ni).
Cho đường tròn (O;R) dây cung BC c định (BC < 2R). Đim A di động trên đường tròn (O;R)
sao cho tam giác ABC là tam giác nhn. Gi ADđường cao và H là trc tâm ca tam giác ABC.
a. Đường thng cha phân giác ngoài ca góc BHC ct AB, AC lần lượt tại các điểm M, N. Chng
minh tam giác AMN là tam giác cân.
b. Gi E, F lần lượt là hình chiếu ca D trên các đường thng BH, CH. Chng minh
OA EF
c. Đường tròn ngoi tiếp tam giác AMN cắt đường phân giác trong ca góc BAC ti K. Chng minh
đường thng HK luôn đi qua một điểm c định.
Gii
a. T gi thiết ta suy ra HM phân giác ca góc
,BHQ HN
phân giác ca góc
.PHC
Ta có:
,AMN HMB MHB=+
ANM NHC NCH=+
,QHB PHC MBH NCH==
suy ra
.=AMN ANM
b. Dựng các đường cao BP, CQ ca
ABC
. K tiếp tuyến Ax
ca (O) thì:
,xAC ABC=
BQPC ni tiếp nên
ABC APQ=
suy ra
/ / .xAC APQ Ax PQ PQ OA=
Ta chng minh:
EF//PQ
Tht vy do HEDF, BQPC ni tiếp nên ta biến đổi góc
HFE HDE HBD HQP= = =
hay
//HFE HQP EF QP=
đpcm.
c. Đường tròn ngoi tiếp tam giác AMN ct phân giác trong góc A ti K thì AK chính đường kính
của đường tròn ngoi tiếp tam giác AMN (ta hiu (AMN)). Gi s KN ct HB ti T, KN ct SC
ti S. D thy HTKS hình bình hành nên TS ct HK tại trung đim ca mỗi đường. Ta chng
minh: TS // BC để t đó suy ra HK đi qua trung điểm I ca BC.
CHUYÊN ĐỀ BỒI DƯỠNG HC SINH GII HÌNH HC 9
THCS.TOANMATH.com | 18
Do TM // QH, SN //PH, HM phân giác ca góc
,QHB HN
phân giác ca góc
PHC
nên ta
biến đổi sau:
/ / .
TH MQ HQ HP NP SH TH SH
TS BC
TB MB HB HC NC SC TB SC
= = = = = =
Theo b đề hình thang
thì HK đi qua trung điểm I ca BC.
Bài 13
a. Cho tam giác MNP cân ti M góc
36 .
o
NMP =
Tính t s
NM
NP
. Cho đường tròn (O; R)
ngoi tiếp tam giác ABC. Hai đường cao AE, BF ct nhau ti trc tâm H. Gi M trung đim ca
đoạn thng AB.
b. Chng minh rng MF là tiếp tuyến của đường tròn ngoi tiếp tam giác CEF.
c. Gi N trung điểm của đoạn thng EF, CM cắt đường tròn ngoi tiếp tam giác (CEF) tại đim
K. Hãy so sánh s đo các góc
BCN
BAK
.
Gii
a. Ta chng minh b đề sau:
“Cho tam giác ABC
2ABC ACB=
.
Chng minh
22
.AC AB AB BC=+
”.
Tht vy: Dng phân giác trong BD ca tam giác ABC ta có: BDC tam giác cân t đó suy ra
2ADB DBC ABC==
suy ra
2
( . ) . (1). = =
AB AD
ABC ADB g g AB AD AC
AC AB
Theo tính
chất đường phân giác ta cũng có:
.AD AB AD AB AB AC
AD
DC BC AD DC AB BC AB BC
= = =
+ + +
thay vào
(1) ta có:
22
.AC AB AB BC=+
th biến đổi theo cách:
= =
AB AC BC
ABC ADB
AD AB DB
suy
ra
2
. , . . .AB AC AD AB BC AC DB AC DC= = =
(do DB = DC).
T đó ta có:
22
. ( )AB AB BC AC AD DC AC+ = + =
Tr li bài toán: Tam giác MNP cân ti M
36
o
NPM =
suy ra
2.N P M+=
Áp dng b đề ta có:
22
..AB BC AB BC=+
Chia hai vế cho AB
2
suy ra:
2
22
22
15
1 1 0
2
BC BC BC BC BC
AB AB AB AB AB
−+

= + + = =


b. T giác CEHF ni tiếp đường tròn tâm I đưng kính CH.
CHUYÊN ĐỀ BỒI DƯỠNG HỌC SINH GIỎI HÌNH HỌC 9
19 | THCS.TOANMATH.com
Suy ra
1
(1).
2
IE IC CH IEC ICE= = =
Ta cũng
(2).MEB MBE=
T (1), (2) ta suy ra
90 90
oo
IEF OEB IFE OBE IEM+ = + = =
hay ME là tiếp tuyến ca (I), tương tự MF cũng là tiếp
tuyến ca (I).
c. Để ý rng:
2
..MK MC ME MN MI==
nên ICKN t
giác ni tiếp. T đó ta có biến đổi góc như sau:
==MNK ICK INC
hay NE phân giác ca góc
.CNK
T đó ta có:
11
22
CNE CNK CIK CFK= = =
t đó suy ra
( . )CNE CFK g g
suy ra
(3).NCB FCK KFM==
Ta
cũng có:
CKF CEF CBA CAB= = =
suy ra KFAM ni tiếp.
T
đó suy ra
(4).KFM KAB=
T (3), (4) suy ra
BCN BAK=
Bài 14. (Trích đề tuyn sinh vào lớp 10 chuyên Toán năm 2013 – TP Hà Ni)
Cho tam giác ABC không cân. Đường tròn (O) tiếp xúc vi BC, AC, AB lần lượt ti M, N, P.
Đưng thng NP ct BO, CO lần lượt ti E, F,
a. Chng minh rng
OEN
OCA
bng nhau hoc bù nhau.
b. Chng minh rằng 4 điểm B, C, E, F cùng thuc một đường tròn.
c. Gi K là tâm đường tròn ngoi tiếp tam giác OEF. Chng minh O, M, K thng hàng.
Gii
a. Trường hp 1: E nằm trong đoạn PN
Ta cũng có:
90
2 2 2
o
B C A
EOC OBC OCB ANP= + = + = =
Nên OENC là t giác ni tiếp, suy ra
180
o
OEN OCA+=
Trường hp 2: E nằm ngoài đoạn PN. Ta có:
180
2 2 2
o
A B C
ENC ANP EOC
= = = + =
nên ONEC t
giác ni tiếp suy ra
OEN OCA=
CHUYÊN ĐỀ BỒI DƯỠNG HC SINH GII HÌNH HC 9
THCS.TOANMATH.com | 20
b. Ta xét trường hp E nằm ngoài đoạn PN.
Chứng minh tương tự câu a) ta suy ra BPFO t giác ni
tiếp.
T đó ta suy ra
90 , 90
oo
OEC ONC OFC OPB= = = =
nên
90 .
o
BFC BEC==
Hay 4 điểm B, F, E, C nằm trên đường
tròn đường kính BC.
c. Gi D giao đim ca BF, CE suy ra O là trc tâm ca tam giác DBC nên D, O, M thng hàng.
Ta cũng D, E, O, F nằm trên đường tròn đường kính DO nên điểm K chính trung điểm ca
DO. Suy ra D, K, O, M thng hàng.
Bài 15.(Trích đề thi vào lp 10 chuyên Toán Tin TP Hà Ni 2014)
Cho tam giác đều ABC ni tiếp đường tròn (O;R), H trung điểm ca BC. M đim bt thuc
đoạn thng BH (M khác B). Lấy điểm N thuộc đoạn thng CA sao cho CN = BM. Gi I là trung
điểm ca MN.
a. Chng minh bốn điểm O, M, H, I cùng thuc một đường tròn.
b. Xác định v trí của điểm M để đoạn thng MN có độ dài nh nht.
c. Khi điểm M thay đổi và thỏa mãn điều kiện đề bài, chng minh din tích tam giác IAB không đổi.
Gii
a. Xét tam giác BOM và tam giác CON ta có: BM = CN gi
thiết, OB = OC = R,
30
o
OBM OCN==
(do tam giác
ABC đều).
Suy ra
( . . )BOM CON c g c =
Suy ra OM = ON hay tam giác OMN cân ti O, do I
trung điểm ca MN suy ra
90 = =
o
OI MN OIM OHM
nên t giác OMHI ni
tiếp (có hai đỉnh liên tiếp I, H cùng nhìn OM góc bng
90 )
o
b. Ta
180
o
OMB OMC OMB ONC= + = +
suy ra t giác OMNC ni tiếp (tổng hai góc đối bng
180
o
) nên
180 120 , 30 .
o o o
MON NCM OMN OBM= = = =
Trong tam giác vuông OMI OMH
ta có:
2 2 3
3
33
OM OH R
IM = =
suy ra
23
,
3
R
MN
dấu đẳng thc xy ra khi và ch khi
.MH
CHUYÊN ĐỀ BỒI DƯỠNG HỌC SINH GIỎI HÌNH HỌC 9
21 | THCS.TOANMATH.com
c. T chng minh câu a, b suy ra
30 .
o
OMN OHI OCN= = =
Suy ra HI // AB, gi K trung
điểm ca AC thì H, I, K thng hàng. K IP, CE, KQ lần lượt vuông góc vi AB thì
1 1 1 3
. . .
2 2 4 8
IAB
S IP AB KQ AB CE AB AB
= = = =
không đổi. Suy ra đpcm.
Bài 16.(Trích đề tuyn sinh vào lớp 10 chuyên Toán năm 2014 – TP Hà Ni)
Cho tam giác ABC ni tiếp đưng tròn (O), H trung điểm ca BC. M điểm bt thuộc đoạn
thng BH (M khác B). Lấy điểm N thuộc đoạn thng CA sao cho CN = BM. Gi I là trung điểm ca
MN.
a. Chng minh bốn điểm O, M, H, I cùng thuc một đường tròn.
b. Gi P là giao điểm ca OIAB. Chng minh tam giác MNP là tam giác đều.
c. Xác định v trí của điểm M để tam giác IAB có chu vi nh nht.
Gii
a. Xét tam giác BOM tam giác CON ta có: BM = CN gi
thiết, OB = OC = R,
30
o
OBM OCN==
(do tam giác ABC
đều). Suy ra
( . . )BOM CON c g c =
suy ra OM = ON hay tam
giác OMN cân ti O, do I trung đim ca MN suy ra
OI MN OIM⊥
90
o
OHM==
nên t giác OMHI ni tiếp
(có hai đỉnh liên tiếp I, H cùng nhìn OM góc bng 90
o
).
b. Do điểm P nm trên trung trc cnh MN nên PM = PN (1).
Ta có
180
o
OMB OMC OMB ONC= + = +
suy ra t giác OMNC
ni tiếp
(tổng hai góc đối bng 180
o
) nên
180 120 , 120= = = =
o o o
MON NCM POM PON
suy ra
180
o
POM PBM+ =
t giác PBMO ni tiếp nên
30
o
OPM OBM==
. Chứng minh tương tự ta
cũng có:
30 60 (2).
oo
OPN OAN MPN= = =
T (1) và (2) suy ra tam giác PMN là tam giác đều.
c. T chng minh câu a, b suy ra
30 .
o
OMN OHI OCN= = =
Suy ra HI // AB, gi K trung điểm
ca AC thì H, I, K thng hàng.
Tam giác IAB AB không đổi nên chu vi tam giác nh nht khi IA + AB nh nhất. Đường thng
HI c định. Gi D điểm đối xng vi B qua HI thì điểm D c định, suy ra độ dài AD không đổi.
Ta
.IB ID IA IB IA ID AD= + = +
Du bng xy ra khi ch khi A, D, I thng hàng. Tc
điểm I chính giao điểm ca AD HK. Mt khác ta d chứng minh đưc AHDK hình bình
hành. Nên du bng xy ra khi I là trung điểm ca HK, khi đó điểm
.MH
Bài 17. (Trích đề thi vào lp 10 chuyên Toán Tin TP Hà Ni 2015)
CHUYÊN ĐỀ BỒI DƯỠNG HC SINH GII HÌNH HC 9
THCS.TOANMATH.com | 22
Cho đường tròn (O) đường kính AB. Gi I điểm bất trên đoạn thng AO (I khác A, I khác O).
Đưng thẳng đi qua I vuông góc vi AB cắt đường tròn (O) tại các đim C D. Gi E điểm
trên đường tròn (O) sao cho D là điểm chính gia ca cung AE. Gi K là giao điểm ca AE CD.
a. Chng minh: OK đi qua trung điểm ca CE.
b. Đường thng qua I song song vi CE ct AE, BE lần lượt ti P, Q. Chng minh: DPEQ hình
ch nht.
c. Tìm v trí điểm I trên đoạn thng AO sao cho KC = KA + KO.
Gii
a. T gi thiết ta suy ra AC = AD = DC suy ra t giác
CADE hình thang cân hai đáy AD, CE. K giao
điểm của hai đường chéo nên K nm trên trung trc ca
CE suy ra
.KO CE
Nói ch khác KO đi qua trung
điểm ca CE.
b. Do AB đường kính ca (O) nên
90
o
AEB AEQ==
.
T giác IPDA
IPA EPQ AEC ADI===
nên IPDA
t giác ni tiếp suy ra
90
o
APD =
t đó dễ dàng suy ra O,
P, D thng hàng, IPDA hình thang cân PD // CE. T
giác DPEQ
DEQ PDE PDA DPQ= = =
nên DPEQ
t giác ni tiếp,
kết hp
90
o
DPE AEQ==
suy ra DPEQ là hình ch nht.
c. Xét t giác COKA ta có:
1
( ) .
2
COA sd AC sd AC DE CKA= = + =
T đó suy ra COKA tc giác
ni tiếp. Áp dụng định lý Ptolemy ta có:
. . .KA OC AC KO CK AO+=
hay
.
AC
KA KO CK
AO
+=
như
vậy đ
KC KA KO=+
thì điều kin
1
AC
AC AO
AO
= =
hay ACO tam giác đu suy ra I phi
là trung điểm ca AO.
Bài 18.(Trích đề thi vào lp 10 chuyên Toán TP Hà Ni 2015).
Cho tam giác đều ABC ba góc nhn, ni tiếp đường tròn (O). Các đường cao AM, BN, CP ca
tam giác ABC cùng đi qua đim H. Gi Q điểm bt trên cung nh BC (Q khác B, Q khác C).
Gi E, F theo th t là điểm đối xng ca Q qua các đường thng AB AC.
a. Chng minh MH.MA = MP.MN
b. Chứng minh ba điểm E, H, F thng hàng.
CHUYÊN ĐỀ BỒI DƯỠNG HỌC SINH GIỎI HÌNH HỌC 9
23 | THCS.TOANMATH.com
c. Gi J giao điểm ca QE AB, I giao điểm ca QF AC. Tìm v trí của điểm Q trên cung
nh BC để
AB AC
QJ QI

+


nh nht.
Gii
a. Xét tam giác MHP tam giác MNA, ta có:
APMC, MHCN ni tiếp nên
,AMP ACP NMH MPH HAN= = =
nên
=
MH MP
MHP MNA
MN MA
hay
..MH MA MN MP=
b. Dng
QK BC
ta các t giác JBKQ,
QKIC, ABQC ni tiếp nên ta biến đổi góc
sau:
180
O
JKQ JBQ ACQ QKI= = =
hay
180
o
JKQ QKI+=
nên J, K, I thng hàng.
Gi s tia CH, BH ct (O) tại giao điểm th 2 là R, S thì
RAB RCB HAB==
, tương tự
RBA HBA=
nên suy ra H đối xng vi R qua AB, tương tự H đối xng vi S qua AC.
Ta t giác ERHQ hình thang cân t giác CRBQ, KBJQ ni tiếp nên
HEQ HRQ CBQ KJQ= = =
nên suy ra HE // KJ, chứng minh tương t ta có: HF // KI I, K, J
thng hàng nên suy ra E, H, F thng hàng.
c. Trên BC lấy điểm T sao cho
QCT QBA=
suy ra
QBA QCT
, QJ, QK là các đường cao tương
ng nên suy ra
(1),
AB TC
QJ QK
=
tương tự
(2) =
AC TB
BQT AQC
QI QK
. T (1) (2) ta suy ra
AB AC TC TB BC
QJ QI QK QK QK
+ = + =
suy ra
AB AC
QJ QI
+
nh nht khi ch khi
BC
QK
nh nht, hay QK ln
nht suy ra Q là điểm chính gia cung nh BC.
Bài 19. (Trích đề thi vào lp 10 chuyên Toán Tin TP Hà Ni 2016).
Cho tam giác nhn ABC AB < AC và ni tiếp (O) các đường cao BB’, CC’ ct nhau ti H. Gi M
là trung điểm ca BC tia MH ct (O) tại điểm P.
a. Chng minh: Các tam giác BPC’, CPB’ đồng dng.
b. Các đường phân giác ca
', 'BPC CPB
lần lượt ct AB, AC ti E, F. Gi O’ tâm đường tròn
ngoi tiếp tam giác AEF, K là giao điểm ca HMAO’.
+ Chng minh: PEKF ni tiếp.
+ Chng minh các tiếp tuyến ti E, F của đường tròn (O’) ct nhau ti một điểm nm trên (O).
CHUYÊN ĐỀ BỒI DƯỠNG HC SINH GII HÌNH HC 9
THCS.TOANMATH.com | 24
Gii
a. Gi A’ điểm đối xng ca A qua O, khi đó BHCL hình
bình hành
, , ',P H A M
thng hàng. Suy ra
90
o
APH =
. Vy A,
P, C’, H, B’ cùng thuộc đường tròn đường kính AH.
b. + Chng minh t giác PEKF ni tiếp. PE phân giác ca
'
''
EB PB
BPC
EC PC
=
Tương tự ta có:
''
FC PC
FB PB
=
''
PB PC
PC PB
=
(theo (1)).
T đó suy ra
(2)
''
EB FC
EC FB
=
Mt khác,
' ' (3)BHC CHB
T (2) (3) ta có, E, H, F thng hàng theo (1) ta có:
'
'
PE BC HE
PH
PF CB HF
= =
phân giác ca
EPF
Vy PH đi qua điểm chính gia ca cung EF không cha A. Li
AEF
cân ti A suy ra phân
giác AO’ đi qua điểm chính gia ca cung EF không cha A. Như vậy giao điểm K ca AO’ PH
thuộc đường tròn ngoi tiếp
.AEF
Vy PEKF là t giác ni tiếp.
+ Gi D điểm chính gia cung BC không cha A của đường tròn ngoi tiếp tam giác ABC. Q
điểm đối xng ca D qua BC nên BHQC là t giác ni tiếp.
Suy ra
.HBQ HCQ=
Gi E’ là giao điểm ca HQAB, khi đó
'.BHE QCB=
Ta li có:
, ' ', ' ' ' 'QHC QBC QHC E HC QBC QCB BHE C HE HE= = = =
phân giác ca
''BHC E E
vy
.Q EF
Ta DQ là phân giác ca
BDC
nên
180
2
o
A
QDB AEF
==
suy ra
t giác BDQE ni tiếp. Ta
180
90 ' 90 '
22
o
oo
AA
BED BQD AEO BED O E DE
= = = = + =
hay DE tiếp tuyến ca
(O’). Chứng minh tương tự. DF tiếp tuyến ca (O’). Tc các tiếp tuyến ca (O’) ti E F ct
nhau ti D nằm trên đường tròn ngoi tiếp
ABC
.
Nhn xét: Đây bài toán khó nhất trong các k thi tuyn sinh vào lp 10 chuyên Toán t trước
đến nay. Bài toán y bài tương t của đề thi VMO2016, một bài toán khác cũng tng xut hin
trên báo THTT.
Bài toán: Cho tam giác ABC 3 đường cao AX, BY, CZ ct nhau tại đim H, M trung đim
ca BC, P là một điểm thuộc đường thng HM. Đưng tròn (K) đường kính AP ct CA, AB lần lượt
ti E, F khác A. Đường tròn ngoi tiếp tam giác AYZ ct (K) tại điểm G khác A.
CHUYÊN ĐỀ BỒI DƯỠNG HỌC SINH GIỎI HÌNH HỌC 9
25 | THCS.TOANMATH.com
a. Chng minh: P, H, G thng hàng.
b. Chng minh tiếp tuyến ti E, F ca (K) cắt nhau trên đường trung trc ca BC.
Phân tích định hướng gii:
CHUYÊN ĐỀ BỒI DƯỠNG HC SINH GII HÌNH HC 9
THCS.TOANMATH.com | 26
D thấy đường tròn ngoi tiếp tam giác AXY đường kính AH nên
,HG GA
mt khác
PG GA
nên P, G, H thng hàng. Xét tam giác GYE tam giác GZF ta có:
YEG ZFG=
(cùng
chn cung
)GA
.
GYE GZF=
(cùng vi
).AYG AZG=
Do đó
.GYE GZF
Xét tam giác GYZ
và tam giác GEF ta có:
,GYZ GAZ GAF GEF= =
= =ZGY ZAY FAE FGE
suy ra
.GYZ GEF
Do
,MYB MBY YAH MZH MCZ ZAH= = = =
suy ra M
giao đim ca các tiếp tuyến ti Y,Z của đường tròn
ngoi tiếp tam giác AYZ. Tương tự ta cũng T giao
điểm ca hai tiếp tuyến ti E, F của đường tròn ngoi
tiếp tam giác AEF. Ta có:
180
180 ( ) 180
o
oo
GFT GFE EFT GAY FAE
GAY FAE GAZ
= + = +
= =
180 180
oo
GZM GZH HZM GAH HAZ GAZ= + = + =
nên
GFT GZM=
mặt khác cũng từ
MYZ TFE
GYZ GEF
ta suy ra
TF EF GF
ZM ZY GZ
==
hay
GFT GZM
suy ra
GZF GMT
suy ra
180 180
oo
GMT GZF GZA GHA AHP= = = =
hay
/ / .TM AH BC
Bài 20. T điểm M c đnh ngoài đường tròn (O) k các tiếp tuyến MA, MB cát tuyến MCD
với đường tròn MC < MD, tia MC nm gia tia MA, MO. Gi I là trung điểm ca CD.
a. Gi s CD ct AB ti N. Chng minh:
BMD IAB=
.
1.
.
NA IB
IA NB
=
b. Đường thng qua C song song vi MB ct AB, BD lần lượt ti H, K. Chng minh: H trung
điểm ca CK.
c. Gi s M c định, (O) không đổi và cát tuyến MCD thay đổi. Chng minh: Trng tâm G ca tam
giác BCD nm trên một đường tròn c định.
Gii
a. Vì MA, MB là tiếp tuyến ca (O) nên
90
o
MAO MBO==
nên các điểm M, A, O, B nm trên
đường tròn đường kính MO.
Do I trung điểm ca CD nên
OI CD
. Suy ra
điểm I nằm trên đường tròn đường kính MO. Như
vậy 5 điểm M, A, O, I, B nằm trên đường tròn đưng
kính MO do đó
=BMD IAB
(Góc ni tiếp chn cung
CHUYÊN ĐỀ BỒI DƯỠNG HỌC SINH GIỎI HÌNH HỌC 9
27 | THCS.TOANMATH.com
IB).
Theo tính cht hai tiếp tuyến ct nhau ta có:
.MA MB MA MB= =
Suy ra
AIM BIM=
(Tính cht
các góc ni tiếp một đường tròn chn cung bng nhau). T đó suy ra MI là phân giác ca góc
AIB
.
Xét tam giác IAB ta có: IN là phân giác trong ca góc
AIB
suy ra
.1
IA NA IA NB
IB NB IB NA
= =
b. CH // MB nên
BMI HCI=
ng v). Mt khác
BMI HAI=
(cùng chn cung BI ca t giác
ni tiếp MAIB). T đó suy ra
HAI HCI=
suy ra t giác ACHI ni tiếp (hai đỉnh liên tiếp A, C cùng
nhìn cnh HI góc bng nhau). T đó suy ra
CAH CIH=
(cùng chn cung CH).
CAH CDB CIH CDB= =
hay HI // BD I trung điểm ca CD suy ra H trung điểm ca
CK.
c. Qua G k các đường thng song song vi IO, MD ct OB, MB lần lượt ti Q, R.
Ta các đim B, O, M c định
2
,
3
BQ BG BP PQ
BO BI BM MO
= = = =
suy ra P, Q c định
90 .
o
PGQ =
Suy ra G thuộc đường tròn đường kính
2
.
3
PQ MO=
Bài 21. Cho đường tròn (O) đường kính AB c định đường kính CD thay đổi không trùng vi
AB. Tiếp tuyến ti A của đường tròn (O) cắt các đường thng BC BD lần lượt ti E F. Gi P
Q lần lượt là trung điểm của các đoạn thng AEAF.
1. Chng minh ACBD là hình ch nht.
2. Chng minh: CP, DQ là các tiếp tuyến ca (O)
ABE AFB
.
3. Chng minh: T giác ECDF là t giác ni tiếp.
4. Gi H là trc tâm ca tam giác BPQ. Chng minh H là trung điểm ca OA.
5. Tìm v trí điểm C trên (O) để din tích tam giác BPQ ln nht.
Gii
CHUYÊN ĐỀ BỒI DƯỠNG HC SINH GII HÌNH HC 9
THCS.TOANMATH.com | 28
1. AB, CD các đường kính ca (O) nên ACBD hình
bình hành. Mt khác
90
o
ACB =
. Suy ra ACBD hình ch
nht. (cũng th lp lun: T giác ACBD 4 góc vuông nên
ACBD là hình ch nht).
2. Do tam giác ECA vuông ti C P trung điểm ca
.EA PA PE PC = =
(tính cht trung tuyến ng vi cnh
huyn ca tam giác vuông).
T đó suy ra
( . . )PAO PCO c c c =
nên
90
o
PCO PAO==
hay
PC là tiếp tuyến ca (O).
Tương tự ta có QD là tiếp tuyến ca (O).
Xét
,ABE AFB
ta có:
AEB ABF=
(cùng ph vi góc
EBA
)
suy
ra
( . )ABE AFB g g
3. Xét t giác ECDF ta có
BCD BAD=
(cùng chn cung AD ca t giác ni tiếp ACBD)
BAD DFE=
(cùng ph vi góc
DAF
). T đó suy ra
BCD DFE=
nên t giác ECDF ni tiếp
(góc ngoài đỉnh C bằng góc trong đối din với đỉnh C).
4. PC, PA các tiếp tuyến ca (O) nên
,PO AC
do
/ / ,AC BD PO BD⊥
mt khác
BO PF
suy ra O trc tâm tam giác PBF
.FO PB⊥
Do H trc tâm ca tam giác
PBQ QH PB⊥
t đó suy ra
//QH FO H
là trung điểm ca AO.
Ta cũng thể chng minh bng cách khác: Gi H trung điểm OA, ta chng minh H trc tâm
ca tam giác PBQ.
D chứng minh được:
ABE AFB AQO∽∽
suy ra
,
22
= = =
AB AQ AB AQ AB AQ
ABP AQH QH PB
AE AO AP AH AP AH
hay H trc tâm ca
tam giác PBQ.
5. Ta
1
..
2
PBQ
S BA PQ R PQ==
suy ra
PBQ
S
nh nht khi ch khi PQ nh nht. Dng
QK CP QKCD⊥
hình ch nht nên QK = CD = 2R. Trong tam giác vuông PKQ thì
2
1
2 . . 2 .
2
PBQ
PQ QK R S BA PQ R PQ R = = =
Vy din tích tam giác PBQ nh nht bng
2
2R
khi và ch khi
//P K CD PQ CD AB
.
Cũng thể lp lun bng cách khác:
1 1 1 1
. .2 . .( ).
2 2 2 2
PBQ
S BA PQ R PQ R EF R AE AF= = = = +
Chú ý
rng:
22
. 4 .AE AF BA R==
Theo bất đẳng thc Cô-si ta có:
2 . 4 .AE AF AE AF R+ =
Suy ra
CHUYÊN ĐỀ BỒI DƯỠNG HỌC SINH GIỎI HÌNH HỌC 9
29 | THCS.TOANMATH.com
2
2.
PBQ
SR
Dấu đẳng thc xy ra khi ch khi
2AE AF R AP AQ R APCO= = = =
hình
vuông. Suy ra
.CD AB
Bài 22. Cho đường tròn tâm O đường kính BC, điểm A nm trên O
( , )A B C
. Các tiếp tuyến ti B,
A ca (O) ct nhau tại điểm M, MC ct (O) tại giao điểm th 2 D. Gi H trung điểm AB, N
trung điểm ca AM. I, K lần lượt là tâm vòng tròn ngoi tiếp, trng tâm các tam giác MAB, MNB.
a. Chng minh: BHDM là t giác ni tiếp.
b. Chng minh: OHDE là t giác ni tiếp.
c. Chng minh:
.IK BN
Gii
a. điểm D nằm trên đường tròn đường kính BC
nên
90 90 , 90
o o o
BDC BDM BHM= = =
(Tính cht 2 tiếp tuyến
ct nhau). Suy ra
90 .
o
BDM BHM==
Vy t giác BHDM ni tiếp
(có 2 đỉnh liên tiếp H, D cùng nhìn BM mt góc vuông).
b. Xét tam giác OBM và tam giác BCE.
Ta có:
90 ,
o
OBM BCE OMB CBE= = =
(cùng ph vi
ABM
). T đó
suy ra
( . )OBM BCE g g
suy ra
1
2
2
CB
OB CE CE
BM CB BM CO
= =
hay
( . . )=
CB CE
CBM OCE c g c
BM CO
,
suy ra
90
o
OCD COE OCD CMB+ = + =
hay
OE CM
tại trung đim ca CD do đó
( . . ) 90 = =
o
OCE ODE c c c OCE ODE
. Vy ED là tiếp tuyến ca (O).
90
o
OCE ODE OHE===
nên 5 điểm O, C, E, D, H nằm trên đường tròn đường kính OE. Hay
OHDE là t giác ni tiếp.
c. I tâm đường tròn ngoi tiếp tam giác cân ABM nên I nằm trên đường cao AH ca tam giác
MAB
.IN AM
Gi Q, F lần lượt trung điểm MN, MB, G trng tâm tam giác MAB thì
2
3
BG BK
BN BQ
==
theo định lý Thales đo ta suy ra
/ / .GK AM IN GK⊥
Mt khác tam giác MAB
cân ti M nên
.GI NF GI NK
T đó suy ra I là trc tâm ca tam giác GKN do đó
.IK BN
Bài 23. Cho nửa đường tròn tâm O đường kính
2.AB R EF=
dây cung di động trên nửa đường
tròn sao cho E thuc cung AF
.
2
AB
EF =
Gi H giao điểm ca AF, BE, C giao điểm ca AE,
BF, I là giao điểm ca CH, AB.
a. Chứng minh 4 điểm A, C, F, I cùng nm trên một đường tròn.
b. Chng minh:
..AE AC BF BC+
có giá tr không đổi khi EF di chuyn trên nửa đường tròn (O).
CHUYÊN ĐỀ BỒI DƯỠNG HC SINH GII HÌNH HC 9
THCS.TOANMATH.com | 30
c. Đường thng AF ct tiếp tuyến ti B N, các tiếp tuyến ti A, F ca (O) ct nhau M. Chng
minh:
.ON MB
d. Xác định v trí EF trên nửa đường tròn để t giác ABEF có din tích ln nht.
Gii
a. các điểm E, F nm trên nửa đường tròn đường kính AB
nên
90
o
AEB AFB==
(Góc ni tiếp chn nửa đường tròn).
Do C là giao điểm ca AE, BF suy ra
,BE AC AF BC⊥⊥
suy ra
BE, AF ct nhau tại đim H trc tâm tam giác CAB suy ra
.CI AB
T giác ACFI
90
o
AFC AIC==
suy ra t giác ACFI t
giác ni tiếp (Hai đỉnh liên tiếp F, I cùng nhìn AC góc
90 )
o
.
b. Xét tam giác vuông ACI và tam giác vuông ABE ta có
90 ,
o
AIC ABE CAB==
chung.
Suy ra
ACI ABE
do đó:
..
AC AB
AC AE AI AB
AI AE
= =
Tương tự ta cũng có: BC.BF = BI.AB. Cộng hai đẳng thc ta có:
22
. . ( ) 4 .AE AC BF BC AB AI BI AB R+ = + = =
c. Xét tam giác MAO và tam giác ABN. Ta có:
90 ,
o
OAM NBA OMA BAN= = =
(cùng ph vi
NAM
).
T đó suy ra
MAO ABN
(g.g) suy ra
2
1
2
= =
MA AB MA OB
AO BN BN
AB
hay
=
MA OB
MAB OBN
AB BN
(c.g.c), suy ra
90+=+=
o
NOB MBA BMA MBA
hay
ON MB
.
d. D thy: Tam giác OMN tam giác đều cnh
=MN R
. Gi K trung điểm ca EF thì
OK EF
. T đó ta tính được
22
2 2 2 2
3R 3
4 4 2
= = = =
RR
OK OE KE R OK
. Tam giác OMN
din tích
2
1
13
.
24
==
R
S OK EF
. Dng EX, FY lần lượt vuông góc vi AB ti E, F thì t giác
EFYX hình thang vuông, dng
KP AB
suy ra P trung điểm XY nên KP đường trung bình
hình thang EFYX.
Ký hiu
23
,SS
lần lượt là din tích ca các tam giác AOE, BOF thì
CHUYÊN ĐỀ BỒI DƯỠNG HỌC SINH GIỎI HÌNH HỌC 9
31 | THCS.TOANMATH.com
12
1 1 1 1
. . ; . .
2 2 2 2
= = = =S OA EX R EX S OB FY R FY
. Ta
1 2 3
= + +
AEFB
S S S S
1
S
không đổi nên
AEFB
S
ln nht khi và ch khi
23
+SS
ln nht.
Ta
( )
23
11
.2 .
22
+ = + = =S S R EX FY R KP R KP
. Trong tam giác vuông OKP ta có:
3
O.
2
=KP K R
suy ra
2
23
3
2
+S S R
.
Du bng xy ra khi ch khi
// P O PK EF EF AB
. Vy GTLN ca
AEFB
S
2
33
4
R
khi và ch khi
//EF AB
.
Bài 24. Cho đường tròn tâm O điểm A nằm ngoài đường tròn. K hai tiếp tuyến AB, AC vi (O)
vi B, C các tiếp đim. Trên cung nh BC ly một đim M ri k các đường vuông góc MI, MH,
MK xung BC, CA, AB. Gi P, Q lần lượt giao điểm ca các cặp đường thng BM IK, CM
IH.
a. Chng minh: Các t giác BIMK, CIMH là t giác ni tiếp.
b. Chng minh:
2
.MI MH MK=
c. Chng minh: IPMQ ni tiếp ri suy ra
PQ MI
.
d. Tìm v trí điểm M để MI. MH. MK đạt GTLN.
Gii
a. T gi thiết ta có:
0
90==BKM BIM
suy ra t giác BKMI ni
tiếp. Tương tự cho t giác CIMH, AKMH.
b. t giác BKMI ni tiếp nên:
MKI MBI=
(cùng chn cung
MI). Mt khác ta có:
MBI MCH=
(tính cht góc to bi tiếp
tuyến và dây cung). Nhưng
MCH MIH=
(cùng chn cung MH ca
t giác ni tiếp MHCI). Suy ra
MKI MIH=
. Hoàn toàn tương tự
ta có:
MIK MHI=
nên
( . )MIK MHI g g
Suy ra
2
.
MI MH
MI MH MK
MK MI
= =
.
c. Ta có:
0
180PMQ PIQ BMC PIM QIM BMC MBA MCA BMC MCB MBC+ = + + = + + = + + =
.
Do đó tứ giác PIQM ni tiếp (Tổng hai góc đối nhau bng
0
180
).
PIQM ni tiếp suy ra
MPQ MIQ MKI MBI= = =
suy ra
//PQ BC
hay
MI PQ
.
d. T chng minh câu b) ta
23
. . . .MI MH MK MI MI MH MK= =
Suy ra
..MI MH MK
ln
nht khi và ch khi MI ln nht. Hay M là điểm chính gia cung nh BC không cha A.
CHUYÊN ĐỀ BỒI DƯỠNG HC SINH GII HÌNH HC 9
THCS.TOANMATH.com | 32
Bài 25. Cho tam ABC có 3 góc nhn ni tiếp (O;R). Dựng đường cao AD của tam giác đường
kính AK ca (O). H BE, CF lần lượt vuông góc vi AK.
a. Chng minh: ABDE, ACFD là các t giác ni tiếp.
b. Chng minh:
ABC DEF
DF AB
c. Cho BC c định, đim A di chuyn trên cung ln BC. Chứng minh: m đường tròn ngoi tiếp
tam giác DEF là một điểm c định.
Gii
a.
0
90ABD AEB==
suy ra 4 điểm A, B, D, E nm trên
đường tròn đường kính AB có tâm là trung điểm N ca AB.
0
90ADC AFC==
nên 4 điểm A, D, F, C nằm trên đường
tròn đường kính AC có tâm là trung điểm P ca AC.
b. Do t giác ADFC ni tiếp nên:
DFA DCA=
cùng chn
cung AD.
Ta
00
90 90DEF BED BAD ABD= = =
. T đó suy
ra
( )
.ABC DEF g g
.
Ta có:
//DFA DCA BKA BK DF= =
BK AB DF AB
c. Gi M trung điểm ca BC, I là trung điểm ca BO thì
,ON AB OM BC⊥⊥
suy ra 5 điểm N, O,
E, M, B nằm trên đường tròn đường kính BO. Ta có:
1
2
MNE MBE DBE DAE DNE= = =
suy ra
MN phân giác ca góc
DNE
. Tam giác DNE cân ti N suy ra MN cũng trung trực ca DE,
tương tự ta cũng có MP trung trc ca DF. Suy ra M là tâm đường tròn ngoi tiếp tam giác DEF.
Vậy tâm đường tròn ngoi tiếp tam giác DEF là điểm M c định.
Bài 26. Cho tam giác ABC vuông ti A, D điểm trên tia đi ca tia BC, k tiếp tuyến DE vi
đường tròn tâm C bán kính CA (A, E khác phía so vi BC). Đường thng qua A vuông góc vi BC
cắt đường thng CE ti F, đưng thng BF ct DE ti M, qua B k đưng thng song song vi CM
ct DE ti N. Gi J là giao điểm th 2 của đường tròn (C; CA) vi EC.
a. Đường tròn đường kính DC ct AC ti I. Chng minh: AIFE ni tiếp.
b. Tam giác CIF cân ti C.
c. Chng minh: M là trung điểm ca NE.
Gii
CHUYÊN ĐỀ BỒI DƯỠNG HỌC SINH GIỎI HÌNH HỌC 9
33 | THCS.TOANMATH.com
a. Đường tròn đường kính DC ct AC ti I nên
0
90DIC DECI=
ni tiếp. Suy ra
0
90EIC EDC ECD CFA AIE AFE= = = =
Hay t giác AIFE ni tiếp.
b. Do
CA CE=
,CEA CIF CAE CFI AIF AFI= = =
suy ra tam
giác CIF cân ti C
//AE IF
.
c. Áp dụng định Menelaus trong tam giác BFC vi
cát tuyến DME ta có:
( )
. . 1 *
MB EF DC
MF EC DB
=
.
Mt khác, do
//AE IF
nên ta có:
EF AI
EC AC
=
, do
//AB DI
nên
DC IC
DB IA
=
thay vào (*) ta có:
. . 1 / /
MB AI IC MB AC CJ
BJ CM
MF AC IA MF IC CF
= = =
hay M là trung điểm ca NE.
Vậy tâm đường tròn ngoi tiếp tam giác DEF đim
M c định.
Bài 27. Cho tam giác ABC ngoi tiếp đường tròn (I), đường thng AI cắt đường tròn ngoi tiếp tam
giác ABC tại điểm D, E đim trên cung BDC, điểm F trên cnh BC tha mãn
1
2
BAF CAE BAC=
. Gi G trung điểm ca IF. Đường thng EI cắt đường tròn ngoi tiếp tam
giác ABC ti P, đường thng AI ct BC ti J, AF cắt đường tròn ngoi tiếp tam giác ABC ti K ct
DP ti Q.
a. Chng minh: APQI là t giác ni tiếp.
b. Chng minh:
DCJ DAC
.
c. Chng minh:
,DG EI
ct nhau ti một điểm nằm trên đường tròn ngoi tiếp tam giác ABC.
Gii
CHUYÊN ĐỀ BỒI DƯỠNG HC SINH GII HÌNH HC 9
THCS.TOANMATH.com | 34
a. Do AD phân giác trong góc A nên
,BAD CAD=
BAK CDE=
,KAD EAD=
mt khác
EAD EPD EAD EPD= =
suy ra t giác PQIA ni tiếp.
b. Xét tam giác DCJ, DAC ta có:
DCJ DAC=
,
ADC
chung
nên
( )
.DCJ DAC g g
c. Gi s PD ct FI ti
'G
. Ta chng minh
'.GG
Tht vy, áp dụng định Menelaus cho tam giác AIF cát
tuyến
'QG D
ta có
'
. . 1(*)
'
QA G F DI
QF G I DA
=
. T chng minh câu a
ta có:
AQI API AKE==
suy ra
//KE QI
. Mặt khác cũng do
BAK CDE=
suy ra
,,BK CE KAD EAD DK DE BKEC= = =
hình thang cân
//EK BC
. Như vậy
/ / / /QI BC KE
.
Theo định lý Thales ta có:
,
QA IA
QF IJ
=
mt khác I tâm đường tròn ni tiếp tam giác ABC nên theo
tính cht phân giác trong ta cũng có:
,
IA CA
IJ CJ
=
do
DCJ DAC
nên
CA DA
CJ DC
=
DI DC DB==
(Tính cht quen thuc). Suy ra
CA DA
CJ DI
=
như vậy:
QA IA CA DA DA
QF IJ CJ DC DI
= = = =
suy ra
.1
QA DI
QF DA
=
thay vào (*) ta có:
'
1'
'
GF
G
GI
=
là trung điểm ca IF hay
'GG
.
Bài 28. Cho tam giác ABC vuông ti A, đường tròn tâm B bán kính BA đường tròn tâm C bán
kính CA ct nhau ti D khác A, BC ct (B) ti E, F (F nm trong (C)) ct (C) ti M, N (M nm
trong (B)). Đường thng DM ct AE ti P, DF ct AN ti Q. Kéo dài DM ct (B) ti I, DF ct (C) ti
H.
a. Chng minh:
IB EF
.
b. Chng minh: T giác APDQ ni tiếp và
//PQ EN
.
c. Chng minh:
..
IP HF AB
IM HQ AC
=
Gii
CHUYÊN ĐỀ BỒI DƯỠNG HỌC SINH GIỎI HÌNH HỌC 9
35 | THCS.TOANMATH.com
a. Ta có:
( )
0
1
45
2
AEN ANE ABF ACM+ = + =
Li có:
,AEF ADF ANM ADM==
Suy ra
0
45IDH MDA FDA= + =
,
( )
0
2 90IBF IBA ABF ADF ADM= + = + =
Do đó
.IB EF
b. T a suy ra tam giác IBF vuông cân ti B suy ra
0
45IAE =
suy ra I, A, N thng hàng.
Tương tự, ta cũng có: E, A, H thng hàng suy ra
0
135EAN =
t giác APDQ ni tiếp suy ra
APQ ADQ AEF==
nên
//PQ EF
.
c. Áp dụng định lý Menelaus vi tam giác PEM và cát tuyến IAN ta có:
. . 1
IP NM AE
IM NE AP
=
, tương tự
vi tam giác QFN cát tuyến HAE ta cũng có:
. . 1.
HF AQ EN
HQ AN EF
=
Nhân hai đẳng thc vi chú ý:
.
AP AE
AQ AN
suy ra:
..
IP HF EF
IM HQ MN
=
Bài 29. Cho tam giác nhn ABC ni tiếp
( )
O
AB AC
. Gi I là trung điểm ca cung BC không
cha A. Trên AC lấy đim K khác C sao cho
.IK IC=
Đưng thng BK ct
( )
O
ti
( )
D D B
.
Trên DI lấy điểm M sao cho
/ / .CM AD
Đưng thng KM ct BC tại điểm N. Đường tròn ngoi tiếp
tam giác BKN ct
( )
O
ti
( )
P P B
.
a. Chng minh: K đối xng vi B qua AI.
b. Chng minh: AC là tiếp tuyến của đường tròn ngoi tiếp tam giác BKN.
c. Gi E là giao điểm ca DIAC. Chng minh: T giác EPIK ni tiếp.
d. Gi F là giao điểm ca PK
( )
O
. Chng minh: KF đi qua trung điểm ca AD.
Gii
CHUYÊN ĐỀ BỒI DƯỠNG HC SINH GII HÌNH HC 9
THCS.TOANMATH.com | 36
a. Ta có:
00
180 180AKI IKC ACI ABI= = =
T đó suy ra
( )
..BIA KIA ABI AKI c g c = =
Do đó
AB AK=
hay K đối xng vi B qua AI.
b. Ta có:
00
180 180DKI BKI DBI DCI= = =
.
Kết hp vi
BDI CDI KID CID IKD ICD= = =
Suy ra
DK DC=
nên DIđưng trung trc ca KC
Do đó
MK MC=
.
Ta có:
CKN KCM DFP KBP= = =
nên KC tiếp tuyến ca
đường tròn ngoi tiếp tam giác BKN.
c. Ta có
EKP KBP DIP EIP= =
suy ra t giác EPIK ni tiếp.
d. T câu c) ta có:
0
90KPI KEI==
PK ct
( )
O
ti F suy ra IF là đường kính ca
( )
O
.
Suy ra
,AF AI
mt khác
BD AI
suy ra
/ / / /DB AF AF KD
. Mt khác FD, EK cùng vuông
góc vi AC nên
//FD KE
suy ra t giác AFDK hình bình hành. Vy FK đi qua trung điểm ca
AD.
Bài 30. Cho tam giác ABC ni tiếp
( )
O
trc tâm H. Các đưng cao BD, CE ct
( )
O
ti giao
điểm th 2 là: F, G. Dng
( )
/ / / / ,FM GN BC M AC N AB
. Các đường thng HM, HN theo th
t ct FG ti K, L. Gi P giao điểm của đường tròn ngoi tiếp tam giác AED vi
( )
O
P khác
điểm A.
a. Chng minh: T giác HLPF ni tiếp.
b. Chứng minh: Các điểm P, L, E thng hàng.
c. Chng minh: DK, EL ct nhau ti một điểm nm trên
( )
O
.
Gii
a. Đường tròn ngoi tiếp tam giác AED đường kính AH, ta
cũng có các tính chất quen thuc là G, F lần lượt đối xng vi
H qua AB, AC. (Hc sinh t chứng minh điều này).
Theo tính cht góc ngoài ta có:
.FLH LGH LHG FBC NGH= + = +
Do
//GN BC
nên
NGH GCB HCB==
suy ra
0
0
180
90
FLH HBC HCB BHC BAC
ABF APH APF HPF
= + = =
= = =
Suy ra t giác HLPF ni tiếp.
CHUYÊN ĐỀ BỒI DƯỠNG HỌC SINH GIỎI HÌNH HỌC 9
37 | THCS.TOANMATH.com
b. Ta có:
LPH LFH GCB BAH EAH EPH= = = = =
suy ra hai tia LP, EP trùng nhau. Hay E, L, P
thng hàng.
c. Ta có
GKH KFH KHF GCB MFH HCB HBC BAC= + = + = + =
0 0 0 0 0
90 180 90 90 90GPH GPA HPA GPA GBA GBA HBA BAC= = = = = =
. Suy ra t
giác PKHG ni tiếp. T đó dễ chứng minh được: P, K, D thng hàng. Suy ra DK, EL ct nhau ti
điểm P nm trên
( )
O
.
Bài 31. Cho tam giác ABC ni tiếp
( )
O
các đường cao BK, CL ct nhau ti H. Các đường AH,
BH, CH ct
( )
O
tại giao điểm th hai D, E, F. Lấy điểm M sao cho tam giác MFH tam giác
HBC đồng dng (M, A nm cùng phía so vi KL). ML ct
( )
O
ti N (M, A, N nm cùng phía so vi
KL). K
( )
( )
//DP KL P O
.
a. Chng minh:
HM AF
.
b. Chng minh: T giác ANLK ni tiếp.
c. Gi s NH ct BC ti Q. Chng minh: Q trung điểm ca BC QK, QL tiếp tuyến ca
( )
AKL
.
d. Gi R là trung điểm ca KL. Chng minh: N, R, P thng hàng.
Gii
a. Ta các tính cht quen thuc, F đối xng vi H qua AB,
//EF LK
HBC HLK
.
= = =HBC MFH MFH HBC HLK LFE
. Suy ra
.M EF
Ta có
0
90 ,AFH FHM AHL HCB AHL HAB+ = + = + =
hay
HM AF
.
b. Do 4 điểm A, L, H, K nằm trên đường tròn đường kính AH. Gi s MH ct AB ti I thì suy ra
/ / .FI BC
Gi
'N
là giao điểm ca
( )
O
và đường tròn đường kính AH thì
HME MFH MFH=+
00
180 90 ' ' 'HBC HCB BHC BAC ABK HN A AN E HN E= + = = = = =
suy ra t giác
'HLN E
ni tiếp. Ta có:
''MN H MEH FCB BAH EAH LN H= = = = =
suy ra hai tia
', 'LN MN
trùng nhau. Hay
, , 'L M N
thng hàng suy ra
'NN
. Suy ra N nằm trên đường tròn đường kính
AH tc là t giác ANLK ni tiếp.
CHUYÊN ĐỀ BỒI DƯỠNG HC SINH GII HÌNH HC 9
THCS.TOANMATH.com | 38
c. Q trung điểm ca BC. Đây tính chất quen thuc (K đường kính
'AA
). Ngoài ra ta cũng
OA KL OA DP
suy ra
,,AD AP QK QL=
là các tiếp tuyến của đường tròn đường kính AH.
d. Ta có:
ANR ANK KNR ALK LNH ACB BAD ACD ANP= + = + = + = =
suy ra N, R, P thng
hàng. Suy ra đpcm.
Bài 32. Cho tam giác ABC đường phân giác trong BE. Đường tròn qua A, B tiếp xúc vi AC
ct BC ti D khác B. Gi K tâm đường tròn ni tiếp tam giác ADC, EK cắt đường tròn ngoi tiếp
tam giác ABE ti L. Gi s AK ct BC ti F.
a. Chng minh: T giác AEFB ni tiếp.
b. Gi s CL cắt đường tròn ngoi tiếp tam giác ABE ti M khác L. Chng minh:
, CEM CLA CBM CLF∽∽
LC là phân giác ca góc
BLE
.
c. Chứng minh: Tâm đường tròn ngoi tiếp tam giác LBC là điểm chính gia cung BC chứa điểm A.
Gii
a. Ta
1
2
FAC DAC=
mt khác do CA tiếp tuyến ca
đường tròn ngoi tiếp tam giác ABD nên
( )
2
. . .= CA CBCD CAD CBA c g c
suy ra
2DAC ABC ABE==
nên
FAE ABE=
Suy ra t giác AEFB ni tiếp.
b. Do BE phân giác ca góc
ABC
nên E điểm chính gia cung AF của đường tròn ngoi tiếp
t giác AEFB. Suy ra LE phân giác ca góc
LA KA CA
ALF
LF KF CF
= =
. Ta có:
( )
.CEM CLA g g
,
( )
.CBM CLF g g
suy ra
ME LA LF MB
ME MB
MC CA CF MC
= = = =
hay LC
là phân giác ca góc
BLE
.
c. Gi N là điểm chính gia cung BC cha A ca
( )
ABC
thì
,2NB NC BNC BAC BLC= = =
suy ra
N tâm đường tròn ngoi tiếp tam giác BLC.
Bài 33. Cho tam giác ABC ni tiếp
( )
O
, các tiếp tuyến ti B, C ca
( )
O
ct nhau ti T. Gi M, N
lần lượt các điểm thuc tia BT, CT sao cho
BM BC CN==
. Đưng thng MN ct CA, AB ln
t E, F, BE giao CT ti P, CF giao BT ti Q. Dựng đường phân giác trong AD ca tam giác ABC.
a. Chng minh:
FBM ACB=
.
b. Chng minh:
//QD BF
.
c. Chng minh:
DP DQ=
AP AQ=
.
CHUYÊN ĐỀ BỒI DƯỠNG HỌC SINH GIỎI HÌNH HỌC 9
39 | THCS.TOANMATH.com
Gii
a. Tam giác BTC, TMN cân ti T, suy ra
/ / .MN BC
Xét tam
giác ABC và tam giác MFB ta có:
ABC BFM=
ng v),
( )
.= = BAC MBC BMF ABC MFB g g
Suy ra
FBM ACB=
.
b. Do
//FM BC
ta có:
QC BC BM AC DC
QF FM FM AB DB
= = = =
/ / ,QD BF
tương tự ta có
//PD CE
c. Theo định lý Thales và tính cht phân giác ta có:
. . . . 1
DQ DQ BE CE CD AB BC
DP DQ
DP BF CE DP BC AC BD
= = = =
.
Ta li có:
1
,
2
ADQ ADB BDQ BAC ACB ABC= + = + +
tương tự ta cũng có:
1
2
ADP BAC ACB ABC= + +
suy ra
ADQ ADP AP AQ = =
.
Bài 34. Cho đường tròn tâm O ni tiếp tam giác ABC tiếp xúc vi BC, CA, AB lần lượt ti D, E, F.
Đưng thng qua A song song vi BC ct EF ti K. Gi I giao điểm ca OD, EF. Gi N giao
điểm ca OAEF.
a. Gi s AI ct OK ti H, chng minh: Bốn điểm A, N, H, K cùng nm trên mt đường tròn.
b. Chng minh:
OHD ODK
tâm đường tròn ngoi tiếp tam giác DHK nm trên một đường
thng c định.
c. Gi M là giao điểm ca AI, BC. Chng minh:
OM DK
.
Gii:
a. AE, AF các tiếp tuyến ca
( )
O
ti E, F nên
AO EF
. Vì
OI BC
suy ra
OI AK
. T đó
suy ra I là trc tâm ca tam giác
AOK AI OK⊥
ti H. Suy ra t giác ANHK ni tiếp.
CHUYÊN ĐỀ BỒI DƯỠNG HC SINH GII HÌNH HC 9
THCS.TOANMATH.com | 40
b. Vì t giác ANHK ni tiếp nên
22
..
OH OD
OH OK ON OA OF OD
OD OK
= = = =
suy ra
( . . ).OHD ODK c g c
T đó suy ra
ODH OKD=
nên OD là tiếp tuyến của đường tròn ngoi tiếp
tam giác HDK.
OD BC
suy ra tâm đường tròn ngoi tiếp tam giác HDK nằm trên đường
thng c định BC.
c. T giác ODMH ni tiếp suy ra
,= = =ODH OMH OHD ODK ODH OKD OMH OKD
OMH
ph vi
MOH
nên
OKD
ph vi
MOH OM DK⊥
.
Bài 35. Cho tam giác ABC ni tiếp
( )
O
các đường cao BD, CE ct nhau H. Gi M, N lần lượt
trung đim ca BC, DE . Gi K giao điểm th 2 ca AM với đường tròn
( )
'O
ngoi tiếp tam giác
AEF. Gi I là giao điểm th 2 ca AN vi
( )
O
.
a. Chng minh:
NAE MAC=
.
b. Chng minh:
~MCK MAC
.
c. Chng minh: T giác BHKC ni tiếp và K đối xng vi I qua BC.
d. Các tiếp tuyến ti B, C ca
( )
O
ct nhau T. Chng minh: A, I, T thng hàng.
Gii:
a. Do t giác BEDC ni tiếp nên:
ADE ABC=
Ta có:
( )
. , ,ADE ABC g g AN AM
các trung tuyến
tương ứng nên
ADE ABC
suy ra
NAE MAC=
.
b. Đường tròn
( )
'O
đường kính AH nên
0
90AKH =
,
gi F là giao điểm ca AH vi BC.
Do các t giác HFMK, HFCD ni tiếp n suy ra
. . .AH AF AK AM AD AC==
suy ra t giác MKDC ni
tiếp nên
MKC MDC MCD==
do đó
( )
.MCK MAC g g
.
c. Ta có
0
360HKC HKD DKC=
0 0 0
180 180 2 180= + = + = HAC DMC DBC DBC DBC
suy ra BHKC ni tiếp.
T chng minh câu b, a ta
KBC MAC NAB ICB= = =
. Tương tự ta cũng có:
KBC IBC=
suy
ra K, I đối xng nhau qua BC.
d. Gi s AT ct
( )
O
tại giao điểm th 2 là
'I
(khác A). Ta chng minh:
'BAI MAC=
.
CHUYÊN ĐỀ BỒI DƯỠNG HỌC SINH GIỎI HÌNH HỌC 9
41 | THCS.TOANMATH.com
Tht vy ta có:
2
'. .TI TA TB TM TO==
suy ra
'AOMI
t giác ni tiếp suy ra
' ' 'I MT OAI OI A OMA= = =
nên MB là phân giác ca góc
'AMI
. Ta có:
0 0 0 0
11
180 180 ' 180 ' 180 ' '
22
AMC AMB AMI AOI ACI ABI= = = = =
suy ra
'ABI AMC
. Do đó:
'MAC BAI=
, điều đó chứng t
', ,I A N
thng hàng. Suy ra
'II
.
Bài 36. Cho tam giác ABC có đường tròn ni tiếp
( )
I
tiếp xúc vi các cnh BC, CA, AB lần lượt ti
D, E, F. Đưng thng qua A song song vi BC ct DE, DF lần lượt ti M, N.
a. Chng minh: M, N, E, F cùng nm trên một đường tròn.
b. Chng minh: MF, NE ct nhau ti một điểm K nm trên
( )
I
.
c. Gi AK ct
( )
I
tại điểm L khác K. Chng minh: L nằm trên đường tròn ngoi tiếp tam giác
DMN.
Gii:
a. Ta có
MNF FDB DEF==
Suy ra t giác MNFE ni tiếp.
Do
ANF FDB BFD NFA = =
Nên tam giác ANF cân ti A, suy
ra
AN AF=
Tương tự
AM AE=
AE AF=
suy ra
AM AN AE AF= = =
Nói cách khác 4 đim M, N, E, F nm trên
đường tròn tâm A.
b. Suy ra
0
90==NEM NFM
suy ra t
giác FKED ni tiếp.
c. Gi s đường tròn
( )
I
cắt đường tròn
( )
J
ngoi tiếp tam giác DMN tại điểm L, ta chng minh:
X, A, L thng hàng. Tht vy k đường kính DX ca
( )
J
. Ta d chứng minh được t giác KMXN
hình bình hành suy ra X, A, K thng hàng (1)
Ta cũng có
0
90 , ,KLD XLD X K L= =
thng hàng (2).
T (1), (2) ta suy ra A, K, L thẳng hàng (đpcm).
Bài 37. Cho tam giác ABC nhọn, không cân có hai đường cao BE, CF. Gi M, N lần lượt là trung
điểm ca BE, CF.
a. Chng minh:
MAB NAC=
.
CHUYÊN ĐỀ BỒI DƯỠNG HC SINH GII HÌNH HC 9
THCS.TOANMATH.com | 42
b. Gi AM cắt đường tròn ngoi tiếp tam giác ANC tại điểm P khác A. Gi AN cắt đường tròn ngoi
tiếp tam giác AMB tại điểm Q khác A. Chng minh rằng 4 điểm M, N, P, Q nm trên một đường
tròn.
c. Gi BQ ct AM ti K, CP ct AN ti L. Chng minh:
//KL MN
.
Gii:
a. Ta có:
22
= = =
AB AC AB AC AB AC
ABE ACF
BE CF BM CN BM CN
suy ra
=ABM ACN MAB NAC
.
b. Ta có:
,APC ANC AMB AQB BAQ CAP= = = =
suy ra
=APC AQB ACP ABQ
.
Li có:
ABQ AMQ ACP ANP= = =
suy ra t giác MNQP ni tiếp.
c. T chng minh trên ta có:
= + = + =ALP LAC LCA KAB KBA AKB
suy ra
( )
1 =
AP AL
APL AQK
AQ AK
. Mt khác do
PQNM ni tiếp nên ta có:
..
AP AN
AP AM AQ AN
AQ AM
= =
(2). T (1) (2) ta có:
//
AK AL
KL MN
AM AN
=
.
Bài 38. Cho tam giác nhn ABC ni tiếp
( )
O
, các đưng cao BE, CF ct nhau ti H, đường thng
AD ct
( )
O
tại giao điểm th 2
( )
P P A
. Đường tròn ngoi tiếp tam giác AEF ct
( )
O
ti giao
điểm th 2
( )
N N A
. Gi I, M lần lượt trung đim ca EF, BC. Đường thng qua P song
song vi EF ct
( )
O
tại điểm
( )
K K D
.
CHUYÊN ĐỀ BỒI DƯỠNG HỌC SINH GIỎI HÌNH HỌC 9
43 | THCS.TOANMATH.com
a. Chng minh:
OA EF
.
b. Chng minh: M, H, N thng hàng và
NIE NFH=
.
c. Chng minh: K, I, N thng hàng.
Gii:
a. Dng tiếp tuyến Ax ca
( )
O
. Ta
xAC ABC=
(Tính cht góc to bi tiếp tuyến y cung).
Mt khác ta li
ABC AFE=
(Do BCEF ni tiếp). T đó suy ra
/ / .xAC AEF EF Ax EF OA=
b. Để chng minh bài toán ta chng minh b đề: Cho tam giác NEF ni tiếp đường tròn
( )
J
, các
tiếp tuyến ti E, F ca
( )
J
ct nhau M, MN ct
( )
O
tại giao điểm th 2 H. Gi I trung điểm
ca EF. Khi đó ta có:
NIF NEH=
.
Tht vy: Theo tính cht tiếp tuyến cát tuyến ta có:
2
.ME MH MN=
, mt khác, áp dng h thc
ng trong tam giác vuông EJM ta có
2
.ME MI MJ=
suy ra
..= MI MJ MH MN MHI MJN
MHI MJN=
suy ra HIJN là t giác ni tiếp.
T đó ta có:
HIM HNJ NIJ==
nên IE phân giác ca góc
0
1
, 180
2
NIH NIE NIH=−
0
180
1
2
NJH=
0
180 NEH NFH−==
.
c. T chng minh câu b) ta suy ra
=NIE NFH ENI HNF
. Ta có:
ANI ANE ENI AFE HNF ACB BAD ACB BCP ACP AKP ANP= + = + = + = + = = =
suy ra N, I,
K thẳng hàng. Suy ra đpcm.
CHUYÊN ĐỀ BỒI DƯỠNG HC SINH GII HÌNH HC 9
THCS.TOANMATH.com | 44
Bài 39. Cho tam giác nhn ABC ni tiếp
( )
O
, các đưng cao AD, BE, CF ct nhau ti H. Đường
thng EF ct
( )
O
ti T, S (S thuc cung nh AC). Dựng đường kính AI ca
( )
O
, SI ct BC ti K.
a. Chng minh: Tam giác AST cân.
b. Chng minh:
2
.AS AE AC=
.
c. Chng minh:
AK SH
.
Gii:
a. Ta chứng minh được tính cht quen thuc:
OA EF
. Dng tiếp tuyến Ax ca
( )
O
. Ta
xAC ABC=
(Tính cht góc to bi tiếp tuyến
y cung). Mt khác ta li
ABC AFE=
(Do BCEF ni tiếp). T đó suy ra
xAC AEF=
/ / . EF Ax EF OA
T đó suy ra OA
trung trc ca ST. Nói cách khác tam giác AST
cân ti A.
b. Ta
AST ATS ABS ACS= = =
suy ra AS
là tiếp tuyến của đường tròn ngoi tiếp tam giác
ESC.
Suy ra
( )
2
.. =ASE ACS g g AS AE AC
.
c. Gi X là giao điểm của đường tròn
( )
J
đường kính AH (đường tròn ngoi tiếp tam giác AEF) vi
đường thng AK. Suy ra
22
. . . .AH AD AX AK AE AC AS AS AX AK= = = =
.
Theo h thức lượng trong tam giác vuông ASK ta suy ra
SX AK
, do đó H, S, X thng hàng.
Hay
SH AK
.
Bài 40. T một điểm A nằm ngoài đường tròn
( )
O
ta k các tiếp tuyến AB, AC đến đường tròn (A, B
các tiếp điểm). Gi H giao đim ca AO BC. Trên đon thng BH ly một điểm M bt k,
đường thng qua M vuông góc vi MO ct AC, AB lần lượt ti P, Q.
a. Chng minh: OPQ là tam giác cân.
b. Lấy điểm R thuc CH sao cho
//PR OA
. Chng minh:
CR HM=
.
c. K đường kính CD ca
( )
O
. Gi K là điểm đối xng vi H qua B, đường thng qua B song song
vi AD ct AK tại điểm N. Chng minh: T giác ACDK ni tiếp.
d. Đường thng qua B vuông góc vi OC ct AH ti E. Chng minh: NB phân giác trong ca
góc
KNE
Gii:
CHUYÊN ĐỀ BỒI DƯỠNG HỌC SINH GIỎI HÌNH HỌC 9
45 | THCS.TOANMATH.com
a. T giác MBQO
0
90OMQ OBQ==
Suy ra MBQO t giác ni tiếp, do đó
MQO MBO=
(cùng chn cung MO).
Tương tự ta cũng tứ giác MPCO ni tiếp
nên
MPO MCO=
MCO MBO MPO MQO= =
hay tam giác
OPQ cân ti O.
b. Ta có:
=
PC CO
PCR COH
RC OH
.
,
.
=
= =
PC OH
RC MOH POC
OC
MH PC OH PC
MH
HO OC OC
T đó ta có:
.RC MH=
c. D chứng minh được:
( )
.. = =
AB AO AB AO
AHB ACO ABK AOD c g c
BH CO BK OD
∽∽
suy ra
AKB ADO=
nên t giác ACDK ni tiếp.
d. Ta d chứng minh được: ABEC là hình thoi, theo tính chất đối xng ta có:
KAB KEB=
Ta cũng
KMB KAD BAE BEA= = =
suy ra t giác BMAE ni tiếp nên
KAB MEB KEB==
hay
EB là phân giác ca góc
KEM
suy ra MB là phân giác ca góc
KME
.
Bài 41. Cho tam giác ABC ni tiếp
( )
O
, đường phân giác trong AD cắt đường tròn
( )
O
ti E, dng
đường kính EF, lấy điểm P trên đoạn thng AD (P khác A, D) . Đường thng FP cắt đường tròn
( )
O
ti Q khác F . Dựng đường thng qua P vuông góc vi AD ct AC, AB ti M, N.
a. Chng minh các t giác PQBN, PQCM ni tiếp.
b. Gi s QN, CP ct nhau trên
( )
O
. Chng minh QMPB cũng cắt nhau trên
( )
O
.
Phân tích định hướng gii:
CHUYÊN ĐỀ BỒI DƯỠNG HC SINH GII HÌNH HC 9
THCS.TOANMATH.com | 46
a. T gi thiết ta suy ra
, / /AF AE AF MN
đây
định hướng để chng minh các t giác ni tiếp.
+ Xét t giác PQBN: Ta có các biến đổi góc sau:
NPQ FPM PFA ACQ= = =
t giác ACQB ni
tiếp nên
00
180 180ACQ ABQ NPQ NBQ+ = + =
.
+ Xét t giác PQCM ta có:
FQC CAF AMP==
suy
ra PQCM ni tiếp.
b. Nếu QN, CP ct nhau tại điểm R nằm trên đường
tròn
( )
O
Ta suy ra t giác BRCQ ni tiếp, gi s BP, QM ct nhau tại điểm S. Ta cn chng minh: BQCS ni
tiếp. Tht vy ta có:
SQC MPC RPN RPB NPB PBC PCB NPB= = = = +
nhưng
PCB RQB NPB==
t đó ta suy ra
SQC PBC SBC=
hay BQCS ni tiếp.
Bài 42. Cho tam giác nhn ABC ni tiếp đường tròn
( )
O
3 đường cao AD, BE, CF đồng quy ti
điểm H. Đường thng CH ct
( )
O
tại điểm G khác C. GD ct
( )
O
tại điểm K khác G.
a. Chng minh: AK đi qua trung điểm M ca DE.
b. Gi N là trung điểm ca DF, AN ct
( )
O
tại điểm L khác A . Chứng minh 4 điểm M, L, N, K cùng
thuc một đường tròn.
Phân tích định hướng gii:
a. Vic chng minh trc tiếp AK đi qua trung
điểm ca DE tương đối khó. Để ý đến chi tiết
CH cắt đường tròn
( )
O
tại điểm G ta s thy G,
H đối xng nhau qua AB, hay F trung đim
GH.
Như vy ta cn tìm mi quan h giữa điểm F
điểm M thông qua các tam giác đồng dng.
Xét tam giác DFH tam giác DAE: Ta
thy
DFH DBH DAE==
.
Ta cũng có
0
180AED ABD FHD= =
suy ra
2 2 2
= = =
HF HD HF HD HG HD
DFH DAE
EA ED EA ED EA ED
hay
HG HD
EA EM
=
.
CHUYÊN ĐỀ BỒI DƯỠNG HỌC SINH GIỎI HÌNH HỌC 9
47 | THCS.TOANMATH.com
T đó suy ra
.= = HGD EAM EAM HGD CAK AM AK
b. Gi s BH cắt đường tròn
( )
O
tại điểm P khác B. Tương tự câu a ta có: P đối xng vi H qua
AC. Suy ra
AG AH AP==
do đó
GP OA EF⊥⊥
suy ra
/ / / /EF MN GP
, gi s AL ct GP ti Q.
Ta có:
MNA AQP AGQ QAG APG QAG AKG GKL AKL= = + = + = + =
suy ra t giác MKNL ni
tiếp.
Bài 43. Cho tam giác nhn ABC
0
, 45AB AC BAC=
. Các đưng cao AD, BE, CF ct nhau ti
H. Đường thng EF ct BC ti P. Gi I là trung điểm ca BC, đường thng IF ct PH tại điểm Q.
a. Chng minh DFEI là t giác ni tiếp và
IQH AIE=
.
b. Gi Ktrc tâm ca tam giác AEF
( )
J
là đường tròn ngoi tiếp tam giác KPD. Đường thng
CK cắt đường tròn
( )
J
tại điểm G. Đường thng IG cắt đường tròn
( )
J
ti M. Đường thng JC ct
đường tròn đường kính BC ti N. Chng minh:
IMC ICG
.
Gii:
a. T các t giác HFBD, HDCE ni tiếp ta
suy ra
FHD HDE FDE ABE ACF= = +
.
Mặt khác ta cũng có:
11
22
FBE FCE FIE FIE FIE+ = + =
suy ra
FDE FIE=
. Do đó tứ giác FDIE ni
tiếp.
+ Ta cũng có:
00
0
180 2 90
45 .
= + = =
= =
FDE ABE ACF A
ABE ACF
Gi R là giao điểm th 2 của hai đường tròn
( ) ( )
,HDI AFHE
ta thy ngay R nm trên trục đẳng phương của hai đường tròn, nên P, H, R thng
hàng, và
PR UV
(vi U là trung điểm AH , V là trung điểm ca HI) suy ra
PR AI
.
Ta có
0
90IQH AIF AIE= =
.
b. Ta có
00
180 135 .FKE A= =
00
45 180 .= + =FCE FKE FCE
Hay t giác CEKF ni tiếp. Tức là điểm K thuộc đường tròn đường kính BC.
+ Ta có:
FPD EFD FDP EFD EDC EFD EFI IFD= = = =
suy ra
IDF IFP
CHUYÊN ĐỀ BỒI DƯỠNG HC SINH GII HÌNH HC 9
THCS.TOANMATH.com | 48
2
.
ID IF
IF ID IP
IF IP
= =
2
.IF IC IC ID IP= =
. Mặt khác ta cũng có: T giác GMDP ni
tiếp nên
2
. . .ID IP IM IG IM IG IC= =
suy ra
IMC ICG
.
Bài 44. Cho t giác ABCD ni tiếp đường tròn
( )
O
. Các tia AB, DC ct nhau ti E, các tia AD, BC
ct nhau ti F. Đường tròn ngoi tiếp tam giác BCE cắt đường tròn ngoi tiếp tam giác CDF ti M.
Chng minh rng:
a. Ba điểm E, M, F thng hàng và bốn điểm A, D, M, E nm trên một đường tròn.
b.
.OM EF
c. Chứng minh đường thng OM đi qua giao điểm của hai đường chéo AC, BD ca t giác ABCD.
Phân tích định hướng gii:
a. Bài toán thc cht là s kết hp giữa hai định
hình học: Định lý Miquel Định
Brocard. Để chng minh E, M, F thng hàng ta
chng minh:
0
180EMC CMF+=
.
Ta tìm cách quy v hai góc đối nhau trong t
giác ni tiếp.
Ta
0
180EMC CMF ABC CDA+ = + =
. Ta
DMF DCF DAB==
suy ra 4 điểm A, D,
M, E cùng nm trên một đường tròn.
T đó ta dễ chứng minh được các t giác
ABMF, BDFE ni tiếp.
b. Xét t giác OACM ta có:
0 0 0 0
180 180 180 2 180CMA CME AMF ABC ABC ABC AOC= = = =
suy ra t giác
OACM ni tiếp.
Ta có
OMF OMA AMB OMA ABF OMC CME OME= + = + = + =
suy ra
0
90 .OME OMF==
Theo định lý Brocard ta có:
OI EF
suy ra O, I, M thng hàng.
Bài 45. T đim M ngoài đường tròn
( )
O
k các tiếp tuyến MA, MB đến đưng tròn
( )
O
. Gi E
trung điểm ca MB, C giao điểm ca AE
( )
O
, H giao điểm ca AB, MO, D là giao điểm
ca MC
( )
O
.
a. Chng minh HCEB là t giác ni tiếp.
b. Chng minh tam giác ABD cân.
c. Gi J là giao điểm th hai ca BO
( )
O
. K là giao điểm ca AD, MJ. Tính t s
KA
KD
.
CHUYÊN ĐỀ BỒI DƯỠNG HỌC SINH GIỎI HÌNH HỌC 9
49 | THCS.TOANMATH.com
Phân tích định hướng gii:
a. Chú ý rng: Tam giác EHB cân ti E nên
EHB EBH ADB==
. Mt khác t giác ACBD ni tiếp
nên
ECB ADB=
. T đó suy ra
ECB EHB=
hay t giác CEBH ni tiếp.
b. Để chng minh tam giác ABD cân ta chng minh:
BAD BDA=
. Nếu điều này xy ra thì
//AD MB
Vì vy ta s chng minh:
//AD MB
. Tht vy ta có:
0
180BAD CAD CAB CBD CAB= =
.
Mt khác t tính cht cát tuyến và tiếp tuyến ta chứng minh được:
11
22
CBD COD CHD CHA===
.
Suy ra
0
180BAD CHA CAB ACH ABE= = =
. Suy ra
//AD MB
(đpcm).
c. Gi N là giao điểm ca BJ, AD. Ta d chứng minh được:
2 2 2
= = =
JN AN JN AN JN AN
AJN MOB
OB MB OB MB JB MB
. Để ý rng:
NK JN
NB JB
=
suy ra
1
22
NK AN
NK AN
NB MB
= =
hay K là trung điểm ca AN. T đó tính được:
1
3
KA
KD
=
.
Bài 46. Cho tam giác nhn ABC ni tiếp
( )
O
đường cao AD, BE, CF ct nhau tại điểm H.
Dựng đường kính AG ca
( )
O
. AG ct EF, BC
ti X, Y.
a. Chng minh: H, G, X, D cùng nm trên mt
đường tròn.
b. Gi Z giao điểm ca AD tiếp tuyến ti G
ca
( )
O
. Chng minh:
//HX YZ
.
Gii:
a. Ta có tính cht quen thuc
OA EF
CHUYÊN ĐỀ BỒI DƯỠNG HC SINH GII HÌNH HC 9
THCS.TOANMATH.com | 50
Chú ý rng các t giác BFXG, BFHD ni tiếp nên:
. . .AF AB AH AD AX AG==
nên 4 điểm H, G, X, D cùng nm trên một đường tròn,
b. Chú ý t giác DZGYHZGD ni tiếp nên:
/ / .= = ZYG ZDG HXG HX YZ
Bài 47. Cho tam giác nhn ABC ni tiếp
( )
O
. Tiếp tuyến ti A ca
( )
O
ct tiếp tuyến ti B, C ca
( )
O
lần lượt ti S, T. BT ct AC ti E, CS ct AB ti F. Gi M, N, P, Q lần lượt trung đim ca
BE, CF, AB, AC. Đưng thng BQ, CP ct
( )
O
tại giao điểm th 2 là K, L.
a. Chng minh:
~ABK EBC
.
b. Chng minh t giác PQKL ni tiếp.
c. Chng minh:
BCM CBN=
.
Phân tích định hướng gii:
a. Ta d chứng minh được tính cht sau: Xem thêm phn tính cht cát tuyến, tiếp tuyến.
Tam giác ABC ni tiếp
( )
O
, các tiếp tuyến ti B C ct nhau ti T, AT ct
( )
O
ti D, OT ct BC
ti H. Khi đó
AHC ABD=
BAT HAC=
.
Tr li bài toán:
+ Áp dng kết qu bài toán ta có:
ABK EBC
.
b. T kết qu
ABK EBC
chú ý rng: KP, CM lần lượt trung tuyến ca các tam giác
, ABK EBC
nên suy ra
BCM BKP=
(1), tương tự
CBN CLQ=
(2).
c. Ta
PLK QBC PQB==
(do KLBC ni tiếp và
//PQ BC
). T đó suy ra tứ giác PQKL ni tiếp
nên ta có:
BKP CLQ=
(3). T (1), (2), (3) ta có:
BCM CBN=
.
CHUYÊN ĐỀ BỒI DƯỠNG HỌC SINH GIỎI HÌNH HỌC 9
51 | THCS.TOANMATH.com
Bài 48. Cho tam giác ABC cân ti A, I tâm đường tròn ni tiếp tam giác, M điểm trong tam
giác ABC sao cho
BMC BIC=
. Đường thng qua M song song vi BC ct AB, AC lần lượt ti P, Q.
Các đường thng qua M song song vi AB, AC ct BC ti D, E.
a. Chng minh t giác MQCD ni tiếp.
b. Gi N là giao điểm ca PD, QE. Chng minh khi M thay đổi thi N luôn chy trên một đường tròn
c định.
Phân tích định hướng gii:
a. Ta BPMD hình bình hành suy
ra
PMD ABC=
. Tam giác ABC cân ti A nên
ABC BCA=
suy ra
PMD QCD=
nên t giác
MQCD ni tiếp.
b. Gi N
1
giao đim ca PD với đường tròn
ngoi tiếp t giác
1
MQCD PN M DCM=
1
.=PN M DCM
I tâm đường tròn ni tiếp tam giác ABC cân
ti A nên:
0
180BIC B=−
.
Suy ra
0
180BMC PBD PMB QMC PBD= + =
.
PMB MBD=
(so le) nên
QMC PBM=
. Mt khác
MBE MCQ=
. Tương tự ta PMEB ni tiếp
nên gi
2
N
giao điểm ca QE với đường tròn ngoi tiếp PMEB suy ra
2
=MN Q MBE
suy
ra
2
MN Q MCQ=
12
=MN Q MN Q
, ta
222 1
== = PN PBM PN M DCM P MM N PN M
nên
12
.N N N
Khi đó ta có:
00
180 , 180= = = =BNM BPM ABC MNC MQC ACB
Suy ra
0
180BNC BAC=−
hay N thuộc đường tròn ngoi tiếp tam giác ABC.
Bài 49. Cho tam giác ABC không cân. Đường tròn tâm I ni tiếp tam giác ABC tiếp xúc vi các
cnh BC, CA, AB ti D, E, F. Đường thẳng đi qua E vuông góc vi BI ct
( )
I
tại điểm th hai
( )
K K E
, đường thng qua F vuông góc vi CI ct
( )
I
ti giao đim th hai
( )
L L F
. Gi
J trung điểm ca KL. Gi T trung đim ca BC, G giao điểm ca AI vi BC, U điểm đối
xng vi T qua G , X, Y là giao điểm ca EF vi BI, CI
a. Chng minh: Bốn điểm C, I, E, XB, C, X, Y cùng nm trên một đường tròn.
CHUYÊN ĐỀ BỒI DƯỠNG HC SINH GII HÌNH HC 9
THCS.TOANMATH.com | 52
b. Gi M, N lần lượt giao đim ca IE, IF vi
( )
I
. MN ct IB, IC ti P ,Q. Chng minh rng
đường trung trc ca PQ luôn đi qua điểm U .
Phân tích định hướng gii:
a. Trước tiên ta d chứng minh đưc D, I, J
thng hàng. Ta có
0
90
2
A
XEC AEF= =
,
0
11
90
2 2 2
A
XIC B C= + =
(Tính cht góc
ngoài). T đó suy ra IEXC ni tiếp
nên
0
90 .IXC =
Hoàn toàn tương tự ta có: IFYB t giác
ni tiếp nên
0
90BYI =
. T đó suy ra t giác
XYBC ni tiếp.
b. Suy ra điểm T nằm trên đường trung trc
ca XY.
Các cặp điểm M E, N F đối xng nhau qua I nên
//EF MN
suy ra
//EF PQ
. Tương tự X, P
đối xng nhau qua I Y, Q đối xng nhau qua I. Nên hai đường trung trc ca XY PQ đối xng
nhau qua IA suy ra đường trung trc ca PQ đi qua điểm U.
Chú ý rằng: Điểm T là c định và
GB AB
k
GC AC
==
không đổi. Suy ra điểm U là c định.
Bài 50. Cho tam giác ABC không cân đường cao AH ni tiếp đường tròn
( )
O
. Một đim P
nm trong tam giác sao cho AP đường phân giác trong ca góc
BAC
. Gi L hình chiếu vuông
góc ca P lên AH. Đường tròn đường kính AP ct
( )
O
tại giao điểm th 2 là G (khác A).
a. Chng minh AP, GL ct nhau ti một điểm nm trên
( )
O
.
b. Gi s GL chia đôi đoạn thng HP. Chng minh: P là tâm đường tròn ni tiếp tam giác ABC
Phân tích định hướng gii:
CHUYÊN ĐỀ BỒI DƯỠNG HỌC SINH GIỎI HÌNH HỌC 9
53 | THCS.TOANMATH.com
a. Gi F, D lần lượt là giao điểm ca AP vi BC
( )
O
. Ta s chng minh G, L, D thng hàng.
Tht vy ta có:
( ) ( )
11
22
AGL LPD BFD CD AB BD AB AGD= = = + = + =
. Suy ra G, L, D
thng hàng.
Cũng có thể biến đổi góc theo cách khác như sau:
.AGL LPD BFD FCD FDC FBD AGC DGC AGC AGD= = = + = + = + =
T đó suy ra G, L, D thng hàng.
b. Để chng minh: P là tâm đường tròn ni tiếp tam giác ABC ta s chng minh:
BP là phân giác trong ca góc
ABC
.
Gi E là giao điểm ca GLBC.
T gi thiết GL chia đôi HP suy ra PLHE là hình ch nht nên
//PE LH
.
Theo định lý Thales ta có:
DP PE LH PF DA DP DA DP
DA AL AL AP AP DF DA AP DP PF
= = = = =
−−
hay
2
.
DA DP
DP DF DA
DP DF
= =
.
Li có
DBF DAC DAB==
nên
DBF DAB
. Suy ra
22
.BD DF DA DP==
.
T đó suy ra
PBC PBD CBD BPD CAD BPD DAB PBA=−=−=−=
.
Vy BP là phân giác ca góc
ABC
. Suy ra điều phi chng minh.
CHUYÊN ĐỀ BỒI DƯỠNG HC SINH GII HÌNH HC 9
THCS.TOANMATH.com | 54
Bài 51. Cho tam giác ABC nhn, c
định ni tiếp trong
( )
O
. P mt
điểm chuyển động trên cnh BC. Gi
( ) ( )
,KL
lần lượt là đường tròn ngoi
tiếp các tam giác
,PAB PAC
. Ly
điểm S thuộc đường tròn
( )
K
sao
cho
//PS AB
. Lấy đim T thuc
đường tròn
( )
L
sao cho
//PT AC
.
a. Chứng minh đường tròn ngoi tiếp
tam giác AST đi qua O.
b. Gi E giao điểm ca
( )
K
AC, F giao điểm ca
( )
L
AB.
BE ct CF tại điểm G. Chng minh
đường thng PG đi qua O khi ch
khi AP đi qua trung điểm ca OH,
vi H là trc tâm ca tam giác ABC.
Phân tích định hướng gii:
a. Gi AT, AS ct BC ti Q, R. D thy các t giác ABPS, ACPT hình thang cân, do đó các tam
giác ABR, ACQ cân ti R, Q. Mt khác ta OK, OL các đường trung trc ca AB, AC do đó OK
đi qua R, OL đi qua Q.
Suy ra trong tam giác AQR thì O la tâm đường tròn ni tiếp hay AO đường phân giác ca
góc
( )
1SAT
. Ta li có t giác ACPT la hình thang cân nên OQ cũng là đường trung trc ca PT.
Do đó
OT OP=
tuơng tự ta có:
OS OP=
do đó
OT OS=
(2).
T (1), (2) ta suy ra O thuộc đường tròn ngoi tiếp tam giác AST.
b. Ta d chứng minh được t giác AEGF ni tiếp. T đó suy ra
EGC EAF EPG==
suy ra CEGP
ni tiếp kéo theo BFGP cũng nội tiếp. T đó suy ra nếu M đối xng vi A qua BC thì
===BPM APB AEB GPC
do đó PG đi qua M . Nên đường thng PG AP đối xng nhau qua
BC. Nên PG đi qua tâm O của đường tròn ngoi tiếp tam giác ABC khi và ch khi AP đi qua điểm
đối xng ca O qua BC. Hay AP đi qua tâm đường tròn Ơle của tam giác ABC.
CHUYÊN ĐỀ BỒI DƯỠNG HỌC SINH GIỎI HÌNH HỌC 9
55 | THCS.TOANMATH.com
Bài 52. Cho tam giác ABC có 3 đường cao AX, BY, CZ ct nhau tại điểm H, M trung điểm ca
BC, P là một điểm thuộc đường thng HM. Đường tròn
( )
K
đường kính AP ct CA, AB t ti E, F
khác A. Đường tròn ngoi tiếp tam giác AYZ ct
( )
K
tại điểm G khác A.
a. Chng minh: P, H, G thng hàng.
b. Chng minh tiếp tuyến ti E, F ca
( )
K
cắt nhau trên đường trung trc ca BC.
Phân tích định hướng gii:
a. D thy đường tròn ngoi tiếp tam giác
AXY đường kính AH nên
HG GA
,
mt khác
PG GA
nên P, G, H thng
hàng.
b. Xét tam giác GYE tam giác GZF ta
có:
YEG ZFG=
(cùng chn cung
GA
).
GYE GZF=
(cùng vi
AYG AZG=
).
Do đó
GYE GZF
. Xét tam giác GYZ
và tam giác GEF ta có:
,GYZ GAZ GAF GEF= =
ZGY ZAY FAE FGE= =
suy ra
GYZ GEF
.
Do
,MYB MBY YAH MZH MCZ ZAH= = = =
suy ra M giao điểm ca các tiếp tuyến ti Y, Z ca
đường tròn ngoi tiếp tam giác AYZ.
Tương tự ta cũng T giao đim ca hai tiếp tuyến ti E, F của đường tròn ngoi tiếp tam giác
AEF.
Ta có
0
180GFT GFE EFT GAY FAE= + = +
( )
00
180 180GAY FAE GAZ= =
0
180GZM GZH HZM GAH HAZ= + = +
0
180 GAZ=−
nên
GFT GZM=
mặt khác cũng t
MYZ TFE
GYZ GEF
ta suy ra
TF EF GF
ZM ZY GZ
==
hay
GFT GZM
suy ra
GZF GMT
suy ra
00
180 180GMT GZF GZA GHA AHP= = = =
hay
//TM AH BC
.
Bài 53. Cho tam giác ABC ni tiếp đường tròn
( )
O
và ngoi tiếp đường tròn
( )
I
, các đường thng
AI, BI, CI cắt đường tròn ngoi tiếp tam giác tại giao điểm th hai là: D, E, F. DE ct AC, BC ln
t ti M, N, DF ct AB, BC ti P, Q.
a. Chng minh: Tam giác CMN là tam giác cân.
CHUYÊN ĐỀ BỒI DƯỠNG HC SINH GII HÌNH HC 9
THCS.TOANMATH.com | 56
b. Chng minh:
//PM BC
t đó suy ra P, I, M thng hàng.
c. Chng minh:
DI BC
DA AB AC
=
+
.
d. Gi s EF, FD, DE lần lượt ct AI, BI, CI ti X, Y, Z. Gi J là tâm đường tròn ngoi tiếp tam giác
DYZ. Chứng minh: 4 điểm X, Y, Z, J nm trên một đường tròn.
Gii:
a. Ta có:
( )
1
,
2
DMC sđ AE DC=+
( )
1
2
CNE sđ CE DB=+
,AE CE BD DC==
DMC CNE=
hay tam giác
CMN cân ti C.
b. Ta có
IAM IEM=
suy ra t giác AIME ni tiếp.
Suy ra
//EIM EAM EBC IM BC= =
, chng minh
tương tự ta cũng có:
/ / , ,IP BC M I P
thng hàng
//MP BC
.
c. Trước hết ta cn chng minh:
DI DB DC==
.
Áp dụng định lý Ptolemy cho t giác ni tiếp: ABDC ta có:
. . .ABCD AC BD AD BC+=
suy ra
( )
.
DI BC
DI AB AC DA BC
DA AB AC
+ = =
+
.
d. Theo tính cht vòng tròn ni tiếp tam giác ta có:
0 0 0
11
90 90 180
22
YIZ YDZ BIC FDE BAC FDA ADE BAC FCA EBC+ = + = + + + = + + + =
. Suy ra
t giác XYDZ ni tiếp. Suy ra
//IYZ IDZ EBC YZ BC= =
. Tương tự ta cũng
có:
/ / , / /XY AB XZ AC
. Suy ra
YXZ BAC=
. Do J là tâm đường tròn ngoi tiếp tam giác DYZ nên
00
2 180 180YJZ YDZ ABC ACB BAC YXZ= = + = =
suy ra t giác XYJZ ni tiếp.
Bài 54. Cho tam giác ABC vuông ti A, dựng đường tròn tâm O đường kính AB. Một đường thng
qua C ct nửa đường tròn ti
( )
O
nm trong na mt phng cha điểm C b AB) ti D, E, (D
nm gia C E) sao cho
0
90 .ECA
Qua D dựng đường thng vuông góc vi CE ct AC ti F.
H
, CK EF EH AC⊥⊥
. Đường tròn ngoi tiếp tam giác EDK ct AD tại đim I khác D. Hai
đường thng AC, KI ct nhau ti M.
a. Chng minh: A, K, E, M cùng nm trên một đường tròn.
b. Chng minh:
2
.CA CF CH=
.
c. Chng minh:
..FM FA FC FH=
.
CHUYÊN ĐỀ BỒI DƯỠNG HỌC SINH GIỎI HÌNH HỌC 9
57 | THCS.TOANMATH.com
d. Chứng minh đường tròn ngoi tiếp tam giác EDM luôn tiếp xúc với đường thng c định.
Gii:
a.T giác EDIK ni tiếp nên
IDE IKF=
(cùng vi
góc
IKE
) hay
ADE IKF=
(1).
Xét đường tròn
( )
O
ta có:
ADE EAF=
(góc ni tiếp
chn cung ln AE góc to bi dây cung AE tiếp
tuyến AF) (2). T (1) và (2) suy ra
00
180 180IKF EAF IKF EAF EKM EAM= = =
suy ra AEKM là t giác ni tiếp.
b. Do CA tiếp tuyến ca
( )
O
nên
2
.CA CDCE=
(*).
Mt khác t giác FHDE ni tiếp nên
..CDCE CH CF=
suy ra
( )
2
. 1CA CF CH=
c. Do t giác AKEM ni tiếp nên
..FK FE FAFM=
. Mt
khác t giác CHKE ni tiếp nên
..FK FE FH FC=
suy ra
( )
. . 2FAFM FH FC=
d. Ta chứng minh đường tròn ngoi tiếp tam giác MDE
luôn tiếp xúc với đường thng AC c định.
T (1) và (2) suy ra
2 2 2 2
. . . .CA FAFM CF CH CF FH CF FM FA CF CA+ = + = =
Li có:
( )( ) ( )
22
.CF CA CF CA CF CA FA CF CA = + = +
Hay
FM CF CA CA CM= + =
. Kết hp vi (*) suy ra
2
.CM CDCE=
tức đường tròn ngoi
tiếp tam giác EDM tiếp xúc đường thng AC ti M.
Bài 55. Cho tam giác ABC ni tiếp
( )
O
ngoi tiếp
( )
I
. Các đường thng AI, BI, CI ct
( )
O
ln
t ti D, E, F, DE ct CF ti M, DF ct BE ti N, AD ct EF ti P. Gi J tâm đường tròn ngoi
tiếp tam giác DMN.
a. Chng minh: INDM là t giác ni tiếp.
b. Chng minh:
//MN BC
I là trc tâm
DEF
.
c. Chng minh: P, M, J, N cùng nm trên một đường tròn.
Gii:
CHUYÊN ĐỀ BỒI DƯỠNG HC SINH GII HÌNH HC 9
THCS.TOANMATH.com | 58
a. Ta có
( )
0
1
180
2
NIM ABC ACB= +
( )
00
180 180NDI MDI NDM= + =
suy ra
INDM là t giác ni tiếp.
b. Suy ra
//= = = INM IDM ABE IBC MN BC
.
Ta có:
( ) ( )
0
11
90
22
FNE EF BD AF AE BD= + = + + =
DF BE⊥
. Chứng minh tương tự ta
DA EF
suy ra I là trc tâm tam giác DEF.
c. J tâm đường tròn ngoi tiếp t giác IMDN
nên
2 2 2MJN MDN MDA NDA= = +
0
180ABC ACB BAC= + =
.
Mt khác chứng minh tương tự ta cũng có:
/ / , / /NP AB MP AC BAC MPN=
t đó suy ra
0
180MJN MPN=−
nên t giác PNJM ni tiếp.
Bài 56. Cho hai đường tròn
( )
O
( )
'O
ct nhau ti A, B. Một đường thng
( )
d
thay đổi qua A ct
( )
O
,
( )
'O
lần lượt ti C, D (A nm gia C, D). Tiếp tuyến ti C ca
( )
O
tiếp tuyến ti D ca
( )
'O
ct nhau M. Gi P, Q lần lượt là hình chiếu vuông góc ca B xung hai tiếp tuyến trên.
a. Chng minh: T giác MCBD ni tiếp.
b. H
BH CD
, chng minh: P, H, Q thng hàng.
c. Chng minh: PQ tiếp xúc với đường tròn c định.
Gii:
CHUYÊN ĐỀ BỒI DƯỠNG HỌC SINH GIỎI HÌNH HỌC 9
59 | THCS.TOANMATH.com
a. Ta có:
0
180CBD CBA ABD MCA MDA CMD= + = + =
Suy ra MCBD là t giác ni tiếp.
b. Đây thực chất đường thng Simson của điểm B
với đường tròn ngoi tiếp tam giác MCD.
c. Ta
==PHB PCB CAB
suy ra PQ tiếp tuyến
của đường tròn ngoi tiếp tam giác ABH.
AB c định
0
90AHB =
nên suy ra PQ luôn tiếp
xúc với đường tròn đường kính AB.
Bài 57. Cho hai đường tròn
( )
O
( )
'O
ct nhau ti A, B mt tiếp tuyến chung PQ
( ) ( )
.(',P O Q O
Tiếp tuyến ti PQ của đường tròn ngoi tiếp tam giác APQ ct nhau E.
Gi D là điểm đối xng vi B qua PQ. Gi s AB ct PQ tại điểm M .
a. Chng minh: APDQ là t giác ni tiếp.
b. Chng minh:
PBM APM
.
c. Chng minh: A, D, E thng hàng.
Gii:
a. Theo tính chất đối xng ta có:
PDQ PBQ PBA QBA APQ AQP= = + = +
0
180 PAQ=−
suy ra APDQ t giác ni
tiếp.
AB ct PQ ti M nên
2 2
.==MP MAMB MQ
Suy ra M trung điểm ca PQ. Tam giác
( . )PBM APM g g
suy ra
BP MB
AP MP
=
,
tương tự ta cũng có:
BQ MB BP AP
AQ MQ BQ AQ
= =
Theo gi thiết ta có:
,PB PQ QB QD==
suy ra
DP AP
DQ AQ
=
.
Theo tính cht quen thuc 2 tiếp tuyến ti P, Q của đường tròn ngoi tiếp tam giác APQ ct nhau
E ta suy ra E, D, A thng hàng.
CHUYÊN ĐỀ BỒI DƯỠNG HC SINH GII HÌNH HC 9
THCS.TOANMATH.com | 60
Bài 58. Cho tam giác ABC ni tiếp
( )
O
vi B, C c định, các tiếp tuyến ca
( )
O
ti A, B ct nhau
X, các tiếp tuyến ca
( )
O
ti A, C ct nhau Y. CX, BY ct AB, AC lần t ti F, E. Gi M, N
trung điểm ca BE, CF. Các đường trung tuyến BL, CK kéo dài ct
( )
O
lần lượt ti P, Q.
a. Chng minh: Các tam giác BFC, QAC đồng dng.
b. Chng minh: T giác PQKL ni tiếp.
c. Gi I giao điểm BN, CM. Chứng minh: Tâm đường tròn ngoi tiếp tam giác IBC nm trên
đường thng c định.
Gii:
Trước hết ta chng minh b đề sau:
Cho tam giác ABC ni tiếp
( )
O
các tiếp tuyến ti A, B ct nhau X. XC ct AB ti F
thì:
ACK FCB=
.
Tht vy gi s CX ct
( )
O
ti R thì:
2
..XR XC XB XK XO==
suy ra COKR t giác ni tiếp. Suy
ra
RKX OCR ORC OKC= = =
nên KB là phân giác ca góc
CKR
.
Ta có:
0 0 0 0
11
180 180 180 180
22
CKA CKB CKR COB CAR CBR= = = = =
suy ra
( . ) CBR CKA g g
suy ra
ACK FCB=
.
a. Tr li bài toán: T b đề ta suy ra tam giác BFC, QAC đồng dng.
b. Ta có
==PQK PBC BLK
suy ra PQKL ni tiếp.
c. Ta cn chng minh:
MBC NCB=
.
CHUYÊN ĐỀ BỒI DƯỠNG HỌC SINH GIỎI HÌNH HỌC 9
61 | THCS.TOANMATH.com
Ta
BFC QAC
BM, QL các trung tuyến tương ng
nên
=BMC QLC MBC LQK
. Tương tự ta cũng
BCN LPK=
LQK LPK=
suy ra
MBC NCB=
suy ra tam giác IBC cân ti I. Suy ra trung trc ca BC đi qua O, I hay tâm đường
tròn ngoi tiếp tam giác IBC nằm trên đường thng c định là trung trc ca BC.
Bài 59. Cho tam giác nhn ABC đường cao
AH. Đường tròn tâm O đường kính AH ct
AB, AC lần lượt ti D, E. Đường thng DE
ct BC ti S, SO ct AB, AC lần lượt ti M,
N.
a. Chng minh:
2
.SD SE SH=
.
b. H
HK SO
(K nm trên SO). Chng
minh OKDE là t giác ni tiếp.
c. Chng minh: KH là phân giác ca
DKE
.
d. Chng minh: O là trung điểm ca MN.
Gii:
a. T gi thiết ta suy ra SH là tiếp tuyến ca
( )
O
. Xét tam giác SDH và tam giác SHE ta có:
1
,
2
SHD SEH sđ DH HSE==
chung suy ra
( )
.SDH SHE g g
Suy ra
( )
2
.1
SH SE
SH SD SE
SD SH
= =
b. Tam giác SHO vuông ti H
HK SO
suy ra
( )
2
. 2SH SK SO=
. T (1) (2) suy ra
. . ( . . )= SK SO SD SE SDK SOE c g c
suy ra
SKD SEO OKDE=
là t giác ni tiếp.
c. T câu b) ta biến đổi góc:
SKD OED ODE OKE DKH HKE= = = =
hay HK phân giác
ca góc
DKE
.
d. T giác HKNE
0
180HKN NEH HKNE+ =
t giác ni tiếp. Suy ra
1
2
==HNE HKE DKE
1
//
2
DOE DAE HN AD= =
. Xét tam giác OMA tam giác ONH có:
, OA OH AOM HON==
,
OAM OHN=
(so le trong) suy ra
( )
. .OMA ONH g c g =
suy ra
OM ON=
.
CHUYÊN ĐỀ BỒI DƯỠNG HC SINH GII HÌNH HC 9
THCS.TOANMATH.com | 62
Bài 60. Cho tam giác nhn ABC ni tiếp đường tròn
( )
O
, H trc tâm ca tam giác. Gi P là điểm
đối xng ca A qua OH (gi s P nm trên na mt phng b BC không chứa điểm A), lấy các điểm
E, F lần lượt nm trên AB, AC sao cho
,BE CP CF PB==
. Gi M trung đim ca BC, K, D ln
ợt là giao điểm ca PM, AP vi OH.
a. Chng minh: BDCP là hình bình hành.
b. Chng minh:
0
90EDF =
c. Gi G giao điểm cùa đường tròn
( )
O
và đường tròn ngoi tiếp tam giác AEF. Chng minh: GP
chia đôi BC.
d. Chng minh:
0
90 .EKF =
Gii:
a. Gi R điểm đi xng vi O qua BC ta d
chứng minh được R tâm đường tròn ngoi
tiếp tam giác BHC. Do A đối xng vi P qua
OH nên
OP OA=
hay P nằm trên đường tròn
( )
O
.
Ta
,OR AH HP OC OP HR= = = =
nên
tam giác
OPR HRP =
(c.c.c) t đó suy ra
OHRP hình thang cân nên
DMO PMO =
(g.c.g) suy ra
MD MP=
. Suy
ra BDCP là hình bình hành.
b. T chng minh câu a ta có:
,BD CP BE==
BP CD CF==
nên các tam
giác BED, CFD là tam giác cân.
Ta có
0
360EDF BDC EDB FDC=
( )
0 0 0 0
0 0 0
180 180 180 180
360 360 180
2 2 2 2
EBD FCD EBD FCD
BPC BAC
= =
0
90
2 2 2 2 2
−−
= + + = + + =
ABC DBC ACB DCB B C A
BAC A
.
Gi Q là giao điểm của đường tròn ngoi tiếp tam giác AEF vi AP.
c. Ta
,GBE GCF GEB GFC==
nên
( )
. ==
G
B
GE F
BE
g
B PC
GC C
B
F
G C g
P
suy ra GP chia đôi
BC hay G, D, M, P thẳng hàng. Tương tự ta có G, N, Q thng hàng.
CHUYÊN ĐỀ BỒI DƯỠNG HỌC SINH GIỎI HÌNH HỌC 9
63 | THCS.TOANMATH.com
Ta
2
2
2
sin
..
sin
MP MC MGC
MG MP MB MC MC
MG MG
GCB


= = = =





. Tương tự
2
sin
sin
NP QGF
NG
GFE

=



,= = = =MGC PAC QGF GCB GAE GFE
suy ra
//
MP NP
MN AP
MG NG
=
.
Gi L điểm đối xng vi D qua N thì
L AP
khi đó EDFL là hình ch nht, ELKD ni tiếp nên
5 điểm E, L, F, K, D cùng nm trên một đường tròn, hay
0
90 .EKF =
Bài 61. Cho tam giác nhn ABC ni tiếp
( )
O
có trực tâm là điểm H. Một điểm D nm trên cung nh
BC, gi E là điểm đối xng vi D qua BC, đường tròn ngoi tiếp tam giác ODE ct AD ti G. Gi J
là giao điểm th 2 của đường tròn ngoi tiếp tam giác AGO vi AH.
a. Chng minh: J, O, E thng hàng.
b. Chng minh: G là tâm đường tròn ngoi tiếp tam giác JHE.
c. Chng minh: Trc tâm tam giác AGO nằm trên đường thng HE.
Gii:
a. Ta có
0
180GOE GDE GAJ GOJ= = =
Suy ra
0
180GOE GOJ+=
hay J, O, E thng
hàng.
b. T các t giác AJOG, ODEG ni tiếp
Ta có biến đổi góc:
GJO GAO GDO GEO= = =
suy ra tam giác
GJE cân ti G.
Gi F giao điểm th 2 ca AH vi
( )
O
thì F
đối xng vi H qua BC nên t giác HEDF
hình thang cân. Gi s đường tròn ngoi tiếp
tam giác JHE ct đường tròn ngoi tiếp tam
giác AGO tại giao điểm th 2 là K. Suy ra
22JGE AOD ABD AFD= = =
( )
0
2 180 2JHE JKE= =
suy ra G là tâm đường tròn ngoi tiếp tam giác JHE.
c. Ta có:
GOE GDE GAJ GKJ GJK GOK= = = = =
li
GKO GJO GEO==
suy ra
GOK GOE =
hay OG trung trc ca KE hay E điểm đối xng vi K qua OG. Do
GK GJ=
t giác AJGK ni tiếp nên AG phân giác ca
HAK
GKA GJH GHJ==
suy ra
CHUYÊN ĐỀ BỒI DƯỠNG HC SINH GII HÌNH HC 9
THCS.TOANMATH.com | 64
H, K đối xng nhau qua AG. Suy ra HE đường thng Steiner ca K trong tam giác AGO. Theo
mt tính cht quen thuc thì HK đi qua trực tâm ca tam giác AGO.
Bài 62. Cho tam giác nhn ABC ni tiếp
( )
O
trực tâm đim H, gi M một điểm
thuc cung nh BC của đường tròn
( )
I
ngoi
tiếp tam giác BHC, tiếp tuyến ti M ca
( )
I
ct
các cnh AB, AC lần t ti F, E. Gi N
giao điểm th 2 của đường tròn ngoi tiếp tam
giác BFM vi
( )
O
(N khác B), J tâm đường
tròn ngoi tiếp tam giác AEF.
a. Chng minh: 4 điểm C, N, M, E cùng nm trên một đường tròn.
b. Chng minh: NJ là phân giác ca
ENF
.
c. Chứng minh: Đường tròn ngoi tiếp tam giác EJF tiếp xúc vi
( )
O
.
Gii:
a. ABNCMNBF là t giác ni tiếp nên ta có biến đổi góc sau:
0
180MNC BNC BNM BAC AFE AEF= = =
, suy ra CNME là t giác ni tiếp.
b. Tacó:
0
180ENF BNC BNF CNE BAC BMF CME BMC BAC= = =
00
180 2 180BHC BAC BAC EJF = =
Suy ra ENFJ là t giác ni tiếp. Li có
JE JF=
nên NJ là phân giác ca
ENF
.
c. Dng tiếp tuyến Nx ca
( )
O
. Ta
FNx BNx FNB BCN FMB= + = +
BCN MCB MCN FMN= + = =
suy ra Nx cũng là tiếp tuyến của đường tròn ngoi tiếp tam giác EJF.
Bài 63. Cho tam giác nhn ABC tâm đường tròn ni tiếp điểm I, gi
( )
K
đường tròn ngoi
tiếp tam giác BIC, trên cung nh BC ly điểm D , đường tròn
( )
L
qua A tiếp xúc ngoài vi
( )
K
ti
D ct AC, AB lần lượt ti E, F . Gi J tâm đường tròn ni tiếp tam giác AEF. Gi s đường tròn
ngoi tiếp tam giác BDF ct cnh BC ti S.
a. Chng minh: SDEC ni tiếp.
b. Chng minh: EJFS ni tiếp.
CHUYÊN ĐỀ BỒI DƯỠNG HỌC SINH GIỎI HÌNH HỌC 9
65 | THCS.TOANMATH.com
c. Chng minh: BC là tiếp tuyến của đường tròn ngoi tiếp tam giác EJF.
Gii:
a. Ta có:
0
360SDE EDF SDF=
00
180 180EDF SDF BAC ABC= + = +
0
180 ACB=−
Suy ra t giác SDEC ni tiếp.
b. Ta có:
ESF ESD DSF ECD FBD= + = +
00
180 180DAC ADC DAB ADB= +
( ) ( )
0
360= + +DAB ADB DAC ADC
11
22
BDC BAC BIC BAC ABC ACB= = = +
0
180
2
BAC
=
. Li có:
( )
0
1
180
2
EJF AEF AFE= +
( )
0
00
1 180
180 180
22
BAC
EAF
+
= =
Suy ra
0
180ESF EJF+=
hay EJFS là t giác ni
tiếp.
c. K tiếp tuyến chung Dx ca
( ) ( )
,KL
Ta có:
xDB DCB=
xDF DEF=
Ta có:
FSB FDB DCB DEF DES DEF SEF= = + = + =
suy ra BC tiếp tuyến của đường tròn
ngoi tiếp tam giác EJF.
Bài 64. Cho tam giác ABC ni tiếp
( )
O
ngoi tiếp
( )
I
, các đường thng AI, BI, CI ct
( )
O
ln
t ti K, J, L.
a. Chng minh: K là tâm đường tròn ngoi tiếp tam giác IBC.
b. Đưng tròn
( )
K
ngoi tiếp IBC ct CA, AB lần lượt ti E, F khác C, B. Chng minh: EL, FJ ct
nhau trên
( )
K
c. Gọi giao điểm ca EL, FJ X, EF ct JL ti Z, ZI ct
( )
K
ti Y. Chng minh: X, Y đối xng nhau
qua AI.
Gii:
CHUYÊN ĐỀ BỒI DƯỠNG HC SINH GII HÌNH HC 9
THCS.TOANMATH.com | 66
a. Ta có:
11
22
= + = + = + =BIK BAC ABC KAC IBC KBC IBC IBK
suy ra tam giác BIK cân ti K,
hay
KB KI KC==
(đpcm).
b. Gi s JL ct BC ti S, SI ct
( )
K
ti X. Ta s chng minh: L, X, E thng hàng F, X, J thng
hàng. Tht vy ta có:
. . .SI SX SB SC SL SJ==
suy ra t giác LIXJ ni tiếp.
Ta có
0 0 0
360 180 180= = + = + = +JXE IXE JXI IXE JXI ACL CLJ BJL JBC
00
180 180LIJ LXJ= =
nên
0
180LXJ JXE+=
hay L, X, E thng hàng. Tương tự, ta cũng F,
X, J thng hàng hay EL, FJ ct nhau tại 1 điểm nm trên
( )
K
.
c. Ta I trc tâm tam giác LKJ (tính cht quen thuc), BE trung trc ca AK. Li
//FC SZ
ZFC SCF=
nên SFCZ hình thang cân, dẫn đến AK trung trc ca SZ, nên
AIZ AIS=
mt khác
ZIX SIY=
đối đỉnh, suy ra
KIX KIY=
suy ra
IX IY=
hay X, Y đối xng
nhau qua AK.
Bài 65. Cho tam giác nhn ABC đường cao AH, phân giác trong góc
BAC
ct BC ti O, qua O dng
các đường thng OM vuông góc vi AB, ON vuông góc vi AC.
a. Chứng minh: 5 điểm A, M, H, O, N cùng nm trên một đường tròn.
b. Chng minh: AH là phân giác ca
MHN
.
c. Đưng thng qua O vuông góc vi BC ct MN ti K. Chng minh:
..KN AC KM AB=
d. Gi I là trung điểm BC. Chng minh: A, K, I thng hàng.
Gii:
CHUYÊN ĐỀ BỒI DƯỠNG HỌC SINH GIỎI HÌNH HỌC 9
67 | THCS.TOANMATH.com
Do
AO MN
nên ta có:
ONK NAO=
(cùng ph vi
NOA
), ta cũng có:
0
90 .NOK NOC OCA= =
T đó suy ra
OKN COA
(g.g)
dẫn đến
KN ON
OA CA
=
tương tự ta cũng có:
KM OM
OA BA
=
Do
OM ON=
suy ra
KM AC
KN AB
=
Hay
..KM AB KN AC=
.
Dựng đường thng qua K song song vi BC
ct AB, AC lần lượt ti E, F, ta d chng minh
được:
,KEMO KNFO
ni tiếp, kết hp vi
OMK ONK=
ta có biến đổi góc:
OEK OMK ONK OFK= = =
suy ra tam giác OEF cân ti O, dn ti
KE KF=
, theo b đề hình
thang ta có A, K, I thng hàng.
Bài 66. Cho tam giác nhn ABC ni tiếp
( )
;OR
các đường cao BE, CF ct nhau tại điểm H. Tiếp
tuyến ca
( )
O
ti B, C ct nhau S, đường thng SA ct
( )
O
tại giao điểm th 2 P. Gi K là hình
chiếu vuông góc ca O lên AS, M giao điểm ca SO BC , I trung điểm AH, J giao điểm
ca MI vi EF, OK ct BC ti T, ct AH ti Q, EF ct BC ti N.
a. Chng minh:
MAC PAB=
.
b. Chng minh:
//GQ MI
.
c. Chng minh:
0
90IKN =
.
Gii:
CHUYÊN ĐỀ BỒI DƯỠNG HC SINH GII HÌNH HC 9
THCS.TOANMATH.com | 68
a, Ta
2
..SB SP SA SM SO==
.
Suy ra AOMP t giác ni tiếp.
Dẫn đến
AMO APO AOP PMS===
Hay MB là phân giác ca
AMP
.
Suy ra
11
22
AMB AMP AOP ACP= = =
0
180 ABP ABP AMC= =
( . ) AMC ABP g g
MAC PAB=
(đpcm).
b. Gi s tiếp tuyến ti A, P ca
( )
O
ct nhau
'T
, t chng minh
câu a ta suy ra 5 đim
, ', , ,A T P M O
cùng nằm trên đường
tròn đường kính
'OT
hay
0
' 90T MO =
tc
', ,T B C
thng
hàng suy ra
'TT
.
Các t giác TAKD, DQKG ni tiếp nên
KQG KDG KAT KOA= = =
suy ra
//GQ OA
MI EF
ti J
OA EF
(tính cht quen thuc), suy ra
//QG MI
.
c. Do tam giác AEF đồng dng vi tam giác ABC AJ, AM các trung tuyến tương ng nên
,,JAF MAC BAP A J P= =
thng hàng hay J thuc AS.
Ta
DIJ DQG DKG==
nên IJKD ni tiếp, ta cũng IJDN ni tiếp nên 5 đim I, J, K, D, N
cùng nằm trên đường tròn đường kính NI dẫn đến
0
90IKN =
.
Bài 67. Cho tam giác nhn ABC ni tiếp
( )
;OR
các đưng cao BE, CF ct nhau ti H. Các
đường thng BE, CE ct
( )
O
tại giao điểm th 2 là PQ. Đường thng PQ ct AC ti L, gi K là điểm
trên cnh AB sao cho
//QK BC
. Gi S giao điểm của đường tròn ngoi tiếp tam giác AEF
( )
O
.
a. Chng minh: T giác ASKL ni tiếp.
b. Gi I là giao điểm của đường tròn ngoi tiếp tam giác AEF vi AH, SI ct CF ti T. Chng minh:
KT là tiếp tuyến của đường tròn ngoi tiếp tam giác AEF.
CHUYÊN ĐỀ BỒI DƯỠNG HỌC SINH GIỎI HÌNH HỌC 9
69 | THCS.TOANMATH.com
c. Chng minh: T nm trên trung trc ca cnh BC.
Gii:
a.Ta
HFE HBC HQP==
suy ra EF
song song vi PQ dn ti
( )
1,
CE CF BF
CL CQ BK
==
li
SBF SCE
(g.g) suy ra
BF SB
CE SC
=
(2).
T (1), (2) ta suy ra
=
SB BK
SBK SCL
SC CL
(c.g.c)
Nên
SKB SLC=
suy ra
SKA SLA=
hay
ASKL là t giác ni tiếp.
b. Ta thy Q, H đối xng nhau qua AB nên
KHQ KQH BCF BAH KST= = = =
suy ra t giác SKHT ni tiếp.
Ta có
00
180 180SKx SKT SHT SHF SAK= = = =
nên KT là tiếp tuyến của đường tròn ngoi tiếp
tam giác AKL.
c. Ta biến đổi góc
,,FSH FAH KSI KIS KAS FHS IKT IAT MFH= = = = = =
suy ra
SFM SKT
(g.g), li
SKL SFE
(g.g). Gi M trung điểm ca BC thì suy ra
LKT EFM
(c.g.c) nên
TK TL=
hay TL tiếp tuyến của đường tròn ngoi tiếp tam giác AKL.
Thêm nữa ta cũng có:
22
22
,
HM MF TI TK
MS MS TS TS
==
MF TK
MS TS
=
suy ra
=
HM TI
HS TS
dn ti
//TM IH
.
CHUYÊN ĐỀ BỒI DƯỠNG HC SINH GII HÌNH HC 9
THCS.TOANMATH.com | 70
Bài 68. Cho tam giác ABC ni tiếp
( )
O
,
đường tròn
( )
K
qua B, C ct AC, AB lần lượt
ti E, F, BE, CF ct nhau ti G, AB ct BC ti
D. Gi H hình chiếu ca B trên AD. Đường
tròn ngoi tiếp tam giác DHC ct
( )
K
ti L,
gi M là trung điểm ca BC, tiếp tuyến ti B, C
ca
( )
K
ct nhau ti T, đường tròn ngoi tiếp
tam giác AEF ct BE, CF lần lượt ti P, Q.
a. Chng minh:
//AQ BT
.
b. Chng minh: A, T, G thng hàng.
c. Chng minh:
ML LC
.
Gii:
a. Ta có:
= = =QAE AFE BCE FBx
nên
//AQ BT
.
b. Ta có:
APF FEC FCT==
nên
/ / ,AP CT BQP CFE CBE==
nên
//PQ BC
do đó
APQ TCB
(g.g) suy ra
( )
..= =
GQ PQ AQ
AQG TBG c g c
GB BC TB
suy ra
,,AGQ BGT A G T=
thng hàng.
c. Ta có:
0
90THB TMB==
nên HBTM ni tiếp, có
TBL BCL DHL BHLT= =
ni tiếp hay THML
ni tiếp, dẫn đến
0
90 .MLC HLC HLM HDC HTM DMT= = = =
Bài 69. Cho tam giác nhn ABC ni tiếp
( )
O
, tiếp tuyến ti A ca
( )
O
ct CB ti K, k tiếp tuyến
KD vi
( )
O
. Gi G, E, F lần lượt là hình chiếu vuông góc ca D trên AB, BC, CA.
a. Chng minh:
2
.KA KB KC=
.
b. Chng minh:
AB DB
AC DC
=
.
c. Chng minh:
2 sinBC R BAC=
.
d. Chng minh: G là trung điểm ca EF.
Gii:
CHUYÊN ĐỀ BỒI DƯỠNG HỌC SINH GIỎI HÌNH HỌC 9
71 | THCS.TOANMATH.com
a. Hc sinh t chng minh.
b. T chng minh câu a ta suy ra:
KBA KAC
suy ra
,
KA AB
KC AC
=
tương tự
KB BD
KC BC
=
KA KB=
suy ra
AB BD
AC BC
=
.
c. K đưng kính BK ca
( )
O
ta có:
0
90 ,=BCK
li
,BAC BKC=
suy ra
sin sinBAC BKC=
.sin
BC
BC BK BAC
BK
= =
, hay
2 sin .BC R A=
d. Áp dng câu c ta có: T giác BGDE ni tiếp đường tròn đường kính BD nên
.sinGE BD GDE=
.sinBD ABC=
, tương tự ta cũng có:
.sin .sinGF CD FCG CD ACB==
T đó suy ra
.sin .sin
.sin .sin
GE BD ABC AB ABC
GF
CD ACB AC ACB
==
, dựng đường cao AH ca tam giác ABC thì ta có:
.sin .sin
1
.sin .sin
GE BD ABC AB ABC AH
GF AH
CD ACB AC ACB
= = = =
suy ra
GE GF=
.
Mt khác t các t giác BGDE, CFGD, ABCD ni tiếp ta có biến đổi góc:
00
180 180= = = + =EGD EBD ACD DGF EGD DGF
hay E, G, F thng hàng. Nói cách khác
G là trung điểm ca E, F.
Bài 70. Cho tam giác nhn ABC ni tiếp
( )
O
, các đường cao BE, CF ct nhau ti H. V y BD ct
CH ti K nm gia HC. Gi G giao điểm ca CD, BE.
a. Chng minh: BFEC là t giác ni tiếp.
b. Chng minh:
CE EG
BF KF
=
.
c. EF ct
( )
O
ti M. Chng minh: AM là tiếp tuyến của đường tròn ngoi tiếp tam giác BMF.
d. Chng minh: EF đi qua trung đim ca GK.
ng dn gii:
CHUYÊN ĐỀ BỒI DƯỠNG HC SINH GII HÌNH HC 9
THCS.TOANMATH.com | 72
a. Hc sinh t chng minh.
b. Xét
BFK
CEG
ta có:
0
90 ,BFK CEG ABD ACD= = =
(góc ni tiếp chn cùng mt cung) suy ra
( )
.BFK CEG g g
nên
CE EG
BF KF
=
đpcm.
c. Ta chứng minh được tính cht quen
thuc
OA EF
, gi s đường thng
EF ct các cung nh AB, AC lần lượt ti
M, Q suy ra tam giác AMQ cân ti A, suy
ra
APM AMP=
, li có:
APM ABM=
suy ra
AMF ABM=
hay AM tiếp
tuyến của đường tròn ngoi tiếp tam giác
BMF.
d. Dựng đường thng qua K song song vi BE ct EF ti N. Ta chng minh: EKNG hình bình
hành.
Do
//HE KN
HBF HCE
nên ta có:
KN HE CE
KF HF BF
==
li
BFK CEG
(chng minh
câu b) suy ra
CE GE
BF KF
=
. T đó suy ra
KN GE
KF KF
=
hay
KN GE=
nên EKNG hình bình hành. Suy
ra EN đi qua trung điểm I ca KG hay EF chia đôi GK.
Bài 71. T điểm M nằm ngoài đường tròn
( )
O
ta dng các tiếp tuyến MB, MC đến
( )
O
(B, C các
tiếp điểm) cát tuyến MDA sao cho tia MA nm gia 2 tia MB, MO
MD MA
. Gi H giao
điểm ca MO BC, AM ct BC ti K.
a. Chứng minh: 4 điểm M, B, O, C cùng nm trên một đường tròn và
2
.MB MA MD=
b. Chng minh:
MDH MOA
t đó suy ra
DHB DCA=
.
c. Chng minh:
CH CD
HA CA
=
.
d. Đưng tròn đường kính BC ct AB, AC ti F, E, EF ct AH ti I. Chng minh:
//IK MO
.
Gii:
a. MB, MC tiếp tuyến ca
( )
O
nên
0
90MBO MCO==
suy ra 4 điểm M, B, O, C nm trên
đường tròn đường kính MO.
CHUYÊN ĐỀ BỒI DƯỠNG HỌC SINH GIỎI HÌNH HỌC 9
73 | THCS.TOANMATH.com
Ta có
1
2
MBD MDA sđ BD==
suy ra
( . )MBD MAB g g
suy ra
( )
2
.1
MB MA
MB MA MD
MD MB
= =
b. Ta có
MO BC
ti H. Trong tam giác vuông MBO ta có:
( )
2
. 2 .MH MO MB=
T (1), (2) suy ra
..=MH MO MD MA
suy ra
( . . )MDH MOA c g c
dn ti
DHM OAM=
suy ra
DHOA là t giác ni tiếp, mà
OAM OAD ODA OHA= = =
suy ra
11
22
DHB AHB DHA DOA DCA= = = =
.
c. T câu b kết hp vi ABDC ni tiếp ta có:
00
180 180CHA BHA DCA DBA= = =
suy ra
( )
.DBA CHA g g
dn ti
=
CH BD
HA BA
(3), mặt khác cũng t
MBD MAB
ta
có:
( )
4
BD MB
BA MA
=
tương tự ta cũng có:
( )
5
CD MC
CA MA
=
( )
6 .MC MB=
T (3), (4), (5), (6) ta suy
ra
CH CD
HA CA
=
.
d. T
DBA CHA
ta suy ra
HAC BAD=
, li BFEC ni tiếp nên
AEI ABC=
suy ra
AEI ABK
suy ra
AI AE
AK AB
=
(7).
Ta cũng có:
000
180 180 180AEH HEC HCE ADB MDB MBA= = = = =
nên
AEH ABM
suy ra
AE AH
AB AM
=
(8). T (7), (8) ta suy ra
AI AH
AK AM
=
hay
//IK HM
hay
//IK MO
.
CHUYÊN ĐỀ BỒI DƯỠNG HC SINH GII HÌNH HC 9
THCS.TOANMATH.com | 74
Bài 72. Cho tam giác ABC ni tiếp
( )
;OR
có trực tâm điểm H, D là đim trên cung nh BC, dng
hình bình hành ADCE gi K là trc tâm tam giác ACE. Gi P, Q là hình chiếu vuông góc ca K trên
BC, AB. Gi s HA ct
( )
O
, PQ lần lượt ti M, N.
a. Chng minh: T giác KPMN ni tiếp.
b. Chng minh: PQ đi qua trung điểm ca HK.
c. Chng minh:
PQ AC
.
Gii:
a. Ta
00
180 180AKC AEC ADC= =
.
Suy ra
0
180AKC ADC+=
nên AKCD là t
giác ni tiếp hay
( )
KO
. Do
//MN KP
,
t giác KQBP ni tiếp nên
QPK QBK AMK==
nên t giác KPMN
ni tiếp.
b. Do t giác KPMN hình thang ni tiếp
trong một đường tròn nên KPMN hình
thang cân nên
KNM PMN=
, mt khác ta
cũng d chứng minh được H, M đối xng
nhau qua BC nên
PMN PHM=
suy ra
KNM PHM=
hay KNHP hình hình bình
hành.
c. H
KR AC
thì 3 điểm N, P, R thng
hàng. (Đường thng Simson của điểm K đối
vi tam giác ABC). T đó suy
ra
1 =
PQ KQ
PKQ CKA
AC KA
.
Bài 73. Cho tam giác nhn ABC không cân
( )
AB AC
, ni tiếp
( )
O
, lấy điểm P trên cnh AB sao
cho
BOP ABC=
, điểm Q trên cnh AC sao cho
COQ ACB=
.
a. Chng minh: O, A, P, Q nm trên một đường tròn.
b. Gi s đường tròn ngoi tiếp tam giác APQ cắt đường tròn
( )
O
ti E (khác A). Chng
minh:
//AE BC
c. Gi s BC ct PQ ti M. Chng minh: ME là tiếp tuyến của đường tròn ngoi tiếp tam giác APQ.
Gii:
CHUYÊN ĐỀ BỒI DƯỠNG HỌC SINH GIỎI HÌNH HỌC 9
75 | THCS.TOANMATH.com
Ta có:
0
360POQ POB BOC COQ=
00
360 2 180B A C BAC= =
suy ra
0
180PAQ POQ+=
hay APOQ t giác
ni tiếp. Ta AEOQ ni tiếp nên
1
2
EOQ EAQ EBC EOC= = =
suy ra
EOQ QOC ACB==
hay
//AE BC
suy ra
t giác ABCE là hình thang cân.
Do
2APE AOE ABE=−
suy ra
= = = PBE PEB B EBC B C
,
Li có :
BMP BCA CQM=−
( )
= = = = BCA AQP BCA AOP BCA AOP C AOB BOP
( )
2= = C C B B C
nên
PEB BMP=
hay BPEM t giác ni tiếp,
PB PE=
nên PM
phân giác ca
EMB
điều đó chứng t ME nằm trên đường thẳng đối xng vi BC qua PQ. Mt
khác ta cũng có:
0
180EQM EQP=−
0
180 ABC PEB= =
suy ra
2
. = =
EM PM
EQM PEM EM PM QM
QM EM
hay ME tiếp tuyến của đường tròn ngoi
tiếp tam giác APQ.
Ta cũng có thể làm theo cách khác:
Gi I tâm đường hòn ngoi tiếp tam
giác APQ thì suy ra điểm I nm trên trung
trc ca cnh BCAE. Gi s OI ct BC,
AE lần lượt ti H, N. Để chng minh ME
tiếp tuyến của đường tròn ngoi tiếp
tam giác APQ ta chng minh:
IE ME
.
Tc chng minh: T giác IHEM ni
tiếp.
Tht vy ta có:
1
2
NIE AIE AQE==
ta
ch cn ch ra t giác EQCM ni tiếp, tht
vy ta có:
EQC EAB ECM EQCM= =
ni tiếp,
CHUYÊN ĐỀ BỒI DƯỠNG HC SINH GII HÌNH HC 9
THCS.TOANMATH.com | 76
suy ra
1
2
NIE AIE AQE EMH EIHM= = =
ni tiếp dẫn đến
0
90IEM IHM==
(đpcm).
Bài 74. Cho tam giác ABC ni tiếp đường tròn
( )
O
, tiếp tuyến ti B, C ct nhau M, đường thng
AM cắt đường tròn
( )
O
tại giao điểm th 2 N. Đường thng qua C vuông góc vi NC ct
( )
O
,
BN lần lượt ti P, Q, đường thng AP ct BC ti E. Gi D giao điểm ca MO PQ, H giao
điểm ca POMQ,
a. Chng minh: T giác AMBD ni tiếp.
b. Chng minh:
NH MQ
.
c. Chng minh: Q, M, E thng hàng.
Gii:
a. Gi I giao điểm ca OM vi BC. Do
D nm trên trung trc ca BC nên
0
90BDM CDM BCP= =
. Ta cũng có:
00
90 90BAM BPN BNP BCP= = =
suy ra
BDM BAM=
hay AMBD t
giác ni tiếp.
b. Ta có:
0
90 .NCQ =
Để chng minh
NH MQ
(1) ta chng
minh:
0
90NHQ =
tc quy v chng
minh t giác NHQC ni tiếp.
Mt khác ta có:
0 0 0
90 90 90NQC QNC BPC BOM BMD= = = =
suy ra t giác BMQD ni
tiếp. Do
= = =BDM MDQ MB MQ MC
suy
ra
00
90 90MQC MCQ NCM NPC CNP= = = =
. Hay t giác NHQC ni tiếp (đpcm).
c. Gi s PN ct ME ti
1
H
, ta chng minh:
1
HH
. Tc là chng minh:
1
PH ME
.
Do MB, MNA lần lượt tiếp tuyến ti B, cát tuyến qua M ca
( )
O
MO BC
ti I nên ta có:
2
..MN MA MB MI MO==
suy ra t giác AOIN ni tiếp, mặt khác ta cũng có: MIAE ni tiếp nên:
1
MNH ONA OIA AEM= = =
suy ra
1
AEH N
ni tiếp hay
0
11
90NH E NH ME=
(2). T (1),
(2) suy ra
1
HH
hay M, Q, E thng hàng.
CHUYÊN ĐỀ BỒI DƯỠNG HỌC SINH GIỎI HÌNH HỌC 9
77 | THCS.TOANMATH.com
Bài 75. Cho đường tròn
( )
O
điểm M ngoài đường tròn, k tiếp tuyến MA, MB (A, B tiếp
điểm). Qua M k cát tuyến MCD
( )
,MC MD
gi E điểm đối xng vi C qua O. Đường thng
EA ct BC ti S, SD ct
( )
O
ti K, MO ct AE ti N.
a. Chng minh tam giác OAC và tam giác MAS đồng dng.
b. Chng minh tam giác BKC cân.
c. chng minh
AD DN
.
Gii:
a. Ta CE đường kính ca
( )
O
nên
0
90 .CAE CAS==
90SAM CAO MAC= =
. Tam giác
AOC cân ti O nên để chng minh: tam
giác OAC tam giác MAS đồng dng
ta cn ch ra tam giác AMS cân ti M. Ta
00
90 90NSB SCA AEB= =
0
90 MOB NMB= =
suy ra SMBN t
giác ni tiếp. Mt khác
ANM BNM MS MB= =
MS MB MA = =
hay tam giác AMS
cân ti M (đpcm).
b. T tam giác MSB cân ti
M MSB MBS MBC BDC BDM = = = =
t giác MSDB ni
tiếp
DBM SDM=
. Mt khác
CBK CDK MDS MBS MBC= = = =
CB CK BCK =
cân.
c. Theo chng minh trên t giác SMBN ni tiếp, t đó suy ra năm đim M, S, D, N, B nm trên mt
đường tròn
SND SBD CED = =
Ta có:
DAE DCE=
(cùng chn cung DE),
DNA DNS DBC DEC= = =
DAE DNA DCE DEC + = +
0
90 AD DN=
.
Bài 76. Cho tam giác nhn ABC 3 đường cao AD, BE, CF ct nhau ti H. Đường thng AC ct
DF ti M. Gi N là trung điểm ca AC. Đường tròn
( )
O
ngoi tiếp tam giác ABC ct BM ti K.
a. Chng minh: T giác FDEN ni tiếp.
b. Chng minh: N, H, K thng hàng.
c. Chng minh:
MH BN
.
Gii:
CHUYÊN ĐỀ BỒI DƯỠNG HC SINH GII HÌNH HC 9
THCS.TOANMATH.com | 78
a. Ta có tính cht quen thuc:
BE phân giác ca
DEF
, các t giác AFHE,
CEHD, AFDC ni tiếp. Suy ra
2DEF DAF DNF==
vy EFDN t giác ni
tiếp.
b. Dựng đường kính BP ca
( )
O
. Ta các tính
cht quen thuc: BHCP hình bình hành suy ra
H, N, P thng hàng.
Mt khác
0
90BKH BKP==
suy ra K, H, P thng hàng. T đó suy ra N, H, K thng hàng.
Chú ý: 4 điểm E, N, D, F thuộc đường tròn Ơle của tam giác ABC.
Gi J trung đim ca HN thì J tâm đưng tròn ngoi tiếp tam giác HEN. Gi I trung đim
BH thì I là tâm đường tròn ngoi tiếp t giác BFDH.
Ta có:
..MF MD ME MN=
nên điểm M nm trên trục đẳng phương của đường tròn
( )
I
( )
J
suy
ra
MH IJ
, mà IJ là đường trung bình ca tam giác BHN suy ra
MH BN
.
Chú ý: Bài toán này là một trường hợp đặc bit của định lý Brocard.
Bài 77. Cho tam giác ABC ni tiếp
( )
O
và một điểm P bt k nm trong tam giác
( )
PO
. Đường
thng AP ct
( )
O
ti D, dựng các đường kính DE, AF ca
( )
O
. Gi G, I lần lượt các giao điểm
ca EP, FP với đường tròn
( )
O
, K giao đim ca AI DG. Gi H hình chiếu vuông góc ca
K trên OP, đường thng OP ct EF ti M.
a. Chng minh: HO là phân giác ca góc
IHD
.
b. Chng minh:
KD DM
.
Gii:
CHUYÊN ĐỀ BỒI DƯỠNG HỌC SINH GIỎI HÌNH HỌC 9
79 | THCS.TOANMATH.com
a. DE, AF đường kính ca
( )
O
HK OK
nên
0
90KIP KGP KHP===
suy ra 5
điểm K, H, I, P, G cùng nm trên
đường tròn đường kính PK.
Suy ra
IHP IGP IGE IDE= = =
hay
IHO IDO=
.
Tc t giác IHOD ni tiếp
OD OI IHO DHO HO= =
phân giác ca góc
IHD
.
b. Để chng minh:
KD DM
ta phi chng minh t giác HKDM ni tiếp.
Theo câu a) ta có:
,IHO DHO=
IHO IHP IKP IGE IDE= = = =
00
90 90IED KAD= =
nên
0
90DHO KAD=−
, ta cũng
0
90KHD DHO=−
00
90 90 )(KHD KAD KAD = =
suy ra
t giác KHDA ni tiếp. Ta cũng có
, / /OA OF AP MF=
nên t giác APFM hình bình hành suy ra
DAM PFM IGE IHP MHD= = = =
nên 5 điểm A, H, D, K, M cùng nằm trên đường tròn đường
kính KM suy ra
KD DM
.
Bài 78. Cho tam giác nhn ABC ni tiếp đường tròn
( )
O
các đường cao BE, CF ct nhau ti H. Các
đường thng BE, CF lần lượt ct
( )
O
tại giao điểm th 2 P, Q
( )
,P B Q C
. Tiếp tuyến ca
( )
O
ti B, C ct EF lần lượt ti M, N.
a. Chng minh: T giác AEHF ni tiếp và
AH AP AQ==
.
b. Chng minh tam giác MEC cân ti M.
c. Gi s MP cắt đường tròn
( )
O
ti K. Chng minh:
2
.ME MK MP=
.
d. Chng minh: N, Q, K thng hàng.
Gii:
CHUYÊN ĐỀ BỒI DƯỠNG HC SINH GII HÌNH HC 9
THCS.TOANMATH.com | 80
a. BE, CF các đường cao ca tam giác
ABC nên H trc tâm ca tam giác suy ra
0
90HEA HFA AEHF= =
ni tiếp.
Ta có:
BAH BCH=
(cùng ph vi
ABC
)
==BCH BCQ BAQ
suy ra
QAB HAB=
,
tam giác QAH AF đường cao đng thi
cũng là trung tuyến nên tam giác QAH cân ti
A. Chứng minh tương tự ta PAH cân ti A
t đó suy ra
AP AH AQ==
.
b. Ta có
MEC AEF=
mà t giác BFEC ni tiếp (hs t chng minh) suy ra
AEF ABC=
Li
ABC ACM=
(do CM tiếp tuyến ca
( )
O
ti C). T đó suy ra
MEC MCE=
hay MEC
tam giác cân ti M.
c. Xét tam giác MCK MPC ta có:
KMC
chung,
= MCP MKC MCK MPC
suy ra
MC MP
MK MC
=
2
.MC MP MK=
, theo b ta có
MC ME=
suy ra
2
.ME MK MP=
.
d. T chng minh câu c) suy ra
MPE MEK
ta có:
00
180 180= = =EPK EPM MEK FEK
,
EPK BPK BAK FAK FEK FAK= = = =
hay AEFK t giác ni tiếp. Tc K giao
điểm ca
( )
AEF
vi
( )
O
. Tương tự nếu gi s NQ ct
( )
O
ti R thì R cũng giao điểm ca
( )
AEF
vi
( )
O
điều đó chứng t
KR
hay N, Q, K thng hàng.
Bài 79. Cho tam giác nhn ABC ni tiếp
( )
O
. Các đưng cao AD, BE, CF ct nhau ti H. Tiếp
tuyến ti B, C ca
( )
O
ct nhau ti G.
GD EF S=
. Gi M trung điểm cnh BC. Gi s
( )
, = =E BC T AT OFK
a. Chứng minh 5 điểm A, K, F, E, H cùng nm trên một đường tròn.
b. Chng minh M, S, H thng hàng.
Gii:
CHUYÊN ĐỀ BỒI DƯỠNG HỌC SINH GIỎI HÌNH HỌC 9
81 | THCS.TOANMATH.com
a. Hc sinh t làm.
T giác EFDM ni tiếp AD phân giác
ca
EDF
nên
MDE FDB TEM==
suy ra
EMD TME
2
.ME MD MT=
.
Thay
22
.ME MB MO MG==
suy ra
..=MO MG MD MT
EMD TME
,
OMT DMT SD TO
ti I.
Gi J giao điểm ca AO EF thì
OA EF
ti J.
Ta có:
. . . .= = = TS TJ TI TO TDTM TETF
AKSJ
t giác ni tiếp nên
0
90SKA SK TA=
.
Ta cũng có: 5 đim A, K, E, F, H nm trên một đường tròn nên
HK TA
nên
( )
, , 1K S H
. Li có:
..=TK TA TDTM AKDM
ni tiếp, suy ra
0
90ADM AKM MK KA= =
suy ra
( )
, , 2 .M H K
T (1) và (2) suy ra M, S, K thng hàng.
Bài 80. Cho tam giác ABC nhn ni tiếp
( )
O
ngoi tiếp đường tròn
( )
I
, đường thng AI kéo dài
ct
( )
O
ti K. Gi D, E, F các tiếp điểm ca
( )
I
vi BC, CA, AB. Đường thng qua A song song
vi EF cắt các đường thng DE, DF lần lượt ti P, Q.
a. Chng minh:
2
.AE AP AQ=
K là tâm đường tròn ngoi tiếp tam giác IBC.
b. Gi s
0
90AIO
. Tìm GTNN ca biu thc:
AB AC
BC
+
.
Gii:
CHUYÊN ĐỀ BỒI DƯỠNG HC SINH GII HÌNH HC 9
THCS.TOANMATH.com | 82
a. Do E, F các tiếp điểm ca
( )
I
vi AC, AB nên
, , / /EFD CED FED BFD EF PQ==
suy
ra
,EFD AQF FED APE==
. Mt khác
,PEA CED AQF BFD==
suy ra
FQA PEA
(g.g)
2
..
QA AF
AP AQ AE FA AE
EA AP
= = =
.
Ta có biến đổi góc sau:
11
22
BIK A B KAC IBC KBC IBC IBK= + = + = + =
Suy ra tam giác BKI cân ti K nên
KB KI KC==
Hay K là tâm đường tròn ngoi tiếp tam giác IBC.
b. Áp dụng định lý Ptolemy cho t giác: ABKC ta có:
( )
. . .AK BC ABCK AC BK BK AB AC= + = +
Tam giác AOD cân
0
90 2 2AOI IA IK IA IK IK AK IK +
suy ra
( )
2
BK AB AC
IK
BC
+
Hay
( )
22
BK AB AC
AB AC
BK
BC BC
+
+
. Khi tam giác ABC đều thì
2
AB AC
BC
+
=
vy GTNN
ca
AB AC
BC
+
bng 2.
Bài 81. T điểm A ngoài
( )
;OR
dng các tiếp tuyến AB, AC ca
( )
O
, gi H trung điểm BC.
Dựng đường kính BD ca
( )
O
, dng
CK BD
. Tia AO ct
( )
O
lần lượt ti M, N.
a. Chng minh: ABOC ni tiếp và
2
.AB AM AN=
.
b. Chng minh:
..MH AN AM HN=
.
c. AD ct CK ti I. Chng minh I là trung điểm ca CK.
d. Dng
BE CN
, gi F trung đim BE, NF ct
( )
O
ti P. AP ct
( )
O
ti T. Chng minh:
//BT MN
BP đi qua trung điểm K ca AH.
Gii:
CHUYÊN ĐỀ BỒI DƯỠNG HỌC SINH GIỎI HÌNH HỌC 9
83 | THCS.TOANMATH.com
a. AB, AC các tiếp tuyến ca
( )
O
nên
0
90ABO ACO==
suy ra 4 đim A, B, O, C nm
trên đường tròn đường kính AO.
Ta có:
1
2
ABM BNA BM==
suy ra
AMB ABN
AM AB
AB AN
=
2
.AB AM AN=
.
b.
ABM MCB MBC==
nên MB phân giác ca góc
ABC
. Mt khác
MB NB
nên NB
phân giác ngoài ca góc
ABC
. Áp dng tính cht phân giác ta có:
BA MA NA
BH MH NH
==
hay
..MANH NAMH=
.
c. Kéo dài AB ct CD ti S thì AO đường trung bình ca tam giác BSD suy ra A là trung điểm ca
BS. Do
//CQ BS
nên theo định lý Thales ta có:
IQ ID IC
AB DA SA
==
AB SA IQ IC= =
.
d. F trung điểm ca BE nên
, / /HF BE BF CE
. Ta
BHF BCE BPF==
nên BPHF t
giác ni tiếp suy ra
0
90BPH =
. Ta
AHP PBC PCA==
nên APHC t giác ni tiếp, suy ra
//PAH PCB PTB BT MN= =
.
BP ct AH ti K thì
2
.KH KP KB=
. Li có
= = PAH PCB PBA AKP BKA
2
.
AK BK
KA KP BK
KP KA
= =
, t đó suy ra
KH KA=
(đpcm).
Bài 82. Cho tam giác ABC có 3 góc nhn
AB AC
, các đường cao AD, BE, CF ct nhau ti H.
a. Chng minh t giác DHEC ni tiếp và xác định tâm O của đường tròn ni tiếp t giác đó.
b. Trên cung nh EC ca
( )
O
lấy điểm I sao cho
,IC IE DI
ct CE ti N. Chng minh:
. . .NI ND NE NC=
c. Gi M là giao điểm ca EF vi IC. Chng minh:
MN CH
.
d. Đường thng HM ct
( )
O
ti K, KN ct
( )
O
tại giao điểm th 2 G, MN ct BC ti T. Chng
minh: H, T, G thng hàng.
Gii:
CHUYÊN ĐỀ BỒI DƯỠNG HC SINH GII HÌNH HC 9
THCS.TOANMATH.com | 84
a. Hc sinh t chng minh.
b. Hc sinh t chng minh.
c. Để chng minh:
MN CH
ta quy v
chng minh:
//MN AB
tc chng
minh:
AFE EMN=
.
Ta có: T giác BFEC, DEIC ni tiếp
nên
AFE ACB DIE==
. Ta cn chng
minh:
DIE EMN=
. Tc quy v chng
minh: MENI là t giác ni tiếp.
Ta li có:
MEC AEF ABC DEC DIC= = = =
suy ra
t giác MENI ni tiếp (đpcm).
d. Để chng minh: H, T, G thng hàng ta s chng minh:
HGN TGN=
, ta có biến đi góc liên quan
như sau:
HGN HCK KMN==
như vậy để chng minh:
HGN TGN=
ta cn ch ra
= KMN TGN TGN KMN
. Tht vy.
+ Xét tam giác ENM, TNC ta có:
,EMN EIN NCT ENM TNC= = =
suy ra
ENM TNC
(g.g)
suy ra
( )
1
EN NM
NT NC
=
+ Xét tam giác ENK, GNC ta có:
,= = KEN CGN ENK GNC ENK GNC
(g.g) suy ra
( )
2
EN NK
GN NC
=
. T (1) và (2) ta suy ra
NK NM
NT NG
=
suy ra
TGN KMN
(c.g.c) đpcm.
Bài 83. Cho tam giác ABC ni tiếp đường tròn
( )
O
. Đường tròn K tiếp xúc vi CA, AB lần lượt ti
E, F tiếp xúc trong vi
( )
O
ti S. SE, SF lần lượt ct
( )
O
ti M, N khác S. Đường tròn ngoi tiếp
tam giác AEM, AFN ct nhau ti P khác A.
a. Chng minh t giác AMPN là hình bình hành.
b. Gi EN, FM lần lượt ct
( )
K
ti G, H khác E, F. Gi GH ct MN ti T. Chng minh tam giác
AST cân.
Gii:
CHUYÊN ĐỀ BỒI DƯỠNG HỌC SINH GIỎI HÌNH HỌC 9
85 | THCS.TOANMATH.com
a. Các t giác ANFP, AMEP, AMSN
ni tiếp nên ta biến đổi góc:
===ANP AFP AEP AMP
(1)
Ta có:
APM AEM CES EFS EAN= = = =
suy ra
//NA PM
(2).
T (1) (2) ta suy ra ANPM
hình bình hành.
b. Các tam giác SKE, SOM cân ti
K O suy ra
KES KSE OMS==
suy ra
//KE OM
.
Chng minh tương tự ta có:
//KF ON
. Theo định lý Thales ta có:
//
SF SK SE
EF MN
SN SO SM
= =
.
T đó ta có:
MNE NEF GHF MNGH= =
t giác ni tiếp. Suy ra
..TM TN TGTH=
, gi s TS
ct
( )
O
ti S
1
, ct
( )
K
ti
2
S
thì
12
. . . .TM TN TGTH TSTS TS TS= = =
suy ra
1 2 1 2
= TS TS S S S
(do 2 đường tròn
( )
O
( )
K
tiếp xúc nhau ti S). Suy ra
2
TM TN TS==
hay TS chính là tiếp tuyến chung ca
( )
O
( )
K
.
Ta s chng minh: TA cũng tiếp tuyến ca
( )
O
. Các đường thng AP MN ct nhau ti I
trung điểm ca mỗi đường.
Ta có TS là tiếp tuyến ca
( )
O
nên
TNS TSM
hay
( )
2
*
TSM
TNS
S
TM SM
TN S SN

==


Mặt khác ta cũng có:
AMI AMN ASN==
, do PEMA ni tiếp ta có:
NAS MAI MAP PES FES NST NAS= = = = = =
suy ra
.. =AIM ANS AM SN AI AS
.
Tương tự ta cũng
.. =AIN AMS AN SM AI AS
t đó suy ra
..AM SN AN SM=
suy ra
22
SM AM
SN AN
=
thay vào (*) ta suy ra
2
TM AM
TN AN

=


hay TA tiếp tuyến ca
( )
O
, vy
TA TS=
.
Bài 84. Cho tam giác ABC ni tiếp
( )
O
. P một điểm nm trong tam giác ABC. Trung trc ca
CA, AB ct PA ti E, F. Đường thng qua E song song vi AC ct tiếp tuyến ti C ca
( )
O
ti M.
Đưng thng qua F song song vi AB ct tiếp tuyến ti B ca
( )
O
ti N.
a. Chng minh MN là tiếp tuyến ca
( )
O
CHUYÊN ĐỀ BỒI DƯỠNG HC SINH GII HÌNH HC 9
THCS.TOANMATH.com | 86
b. Gi s MN cắt đường tròn ngoi tiếp các tam giác ACM, ABN lần lượt ti S, Q khác MN. Chng
minh
ABC ASQ
SB ct CQ ti một điểm nm trên
( )
O
.
Gii:
a. Gi s AP ct
( )
O
ti D , ta chng minh
DM, DN các tiếp tuyến ca
( )
O
. Tht
vy do
/ / , / /FN AB EN AC
nên
00
90 , 90OFN OEM==
. Kết hợp các điều
kin OF trung trc ca AB, tam giác
OAD cân t giác OFBN ni tiếp ta
biến đổi góc sau:
ODF OAF OBF ONF===
suy ra t giác
FNDO ni tiếp, suy ra
0
90OND DN=
tiếp tuyến ca
( )
O
.
Chứng minh tương tự ta có: DM cũng
tiếp tuyến ca
( )
O
.
b. Gi s SB, CQ ct nhau ti R, ta chng
minh: ABCR ni tiếp.
Tc chng minh:
RCA RBA=
, muốn điều này ta cn chng minh:
ABS ACQ
. Tht vy:
Ta có:
0
180 ,ASQ ASM ACM ACx ABC= = = =
0
180AQS AQN ABN ABy ACB= = = =
suy ra
ABC ASQ
(g.g). T đó suy ra đpcm.
Bài 85. Cho đường tròn
( )
;OR
. T một điểm M nm ngoài đường tròn, k 2 tiếp tuyến MA, MB
đến
( )
O
(A,B các tiếp điểm). Qua A, k đường thng song song vi MO cắt đường tròn ti E
đường thng ME cắt đường tròn ti F, đường thng AF ct MO ti N, H là giao điểm ca MOAB.
1. Chng minh: T giác MAOB ni tiếp đường tròn.
2. Chng minh:
2
.MN NF NA=
.
3. Chng minh:
0
90HFN =
MN NH=
4. Chng minh:
2
2
1.
HB EF
HF MF
−=
ng dn gii:
CHUYÊN ĐỀ BỒI DƯỠNG HỌC SINH GIỎI HÌNH HỌC 9
87 | THCS.TOANMATH.com
1. Hc sinh t làm.
2. Do
, / /MO AB AE MO
suy ra
0
90EAB =
suy ra BE đường kính ca
( )
O
hay B, O, E thng hàng.
Suy ra
0
90BFE =
, t giác MFHB
0
90MFB MHB==
nên MFHB t giác
ni tiếp.
Suy ra
FMH FBH=
(cùng chn cung
FH).
Li có:
1
2
FBA FAM FA==
t đó suy ra
( )
1 .=NAM NMF
Tam giác MNF và tam giác ANM ta có:
MNF
chung (2). T (1), (2) ta suy ra
MNF ANM
(g.g)
suy ra
MN NA
NF MN
=
hay
2
. (*)MN NF NA=
Cũng thể chng minh ngắn hơn:
NMF AEF=
(so le trong),
1
2
AEF MAF sđ AF==
suy
ra
( )
.MNF ANM g g
.
3. T chng minh câu b) ta có:
NFM NMA HMB HFB= = =
suy ra
0
90HFB BFM HFB NFM+ = + =
hay
HF AN
. Áp dng h thức lượng trong tam giác vuông
AHN ta có:
( )
2
. **NF NA NH=
. T (*), (**) ta suy ra
22
MN NH MN NH= =
.
4. Ta viết lại đẳng thc cn chng minh thành:
22
22
1
HB EF HB ME
HF MF HF MF
= + =
. Ta d chng minh
được đẳng thc quen thuc:
2
2
2
.
ME MA
ME MF MA
MF MF
= =
. Ta cn chng minh:
22
22
MA HB MA HB
MF HF MF HF
= =
thay
,MA MB HB HA==
. Ta cn chng minh:
MB HA
MF HF
=
hay chng
minh:
BFM AFH
nhưng điều này luôn đúng do
0
90BFM AFH==
1
2
MBF HAF FB==
.
Cách khác:
22
22
11
HB EF HB EF
HF MF HF MF
= + =
. Ta
2
.HF FAFN=
EF FA
MF FN
=
nh
Thales) nên vế trái bng:
2 2 2 2 2 2
2
1.
. . .
HB FA HB AF HA AF HF
FA FN FN FA FN FAFN HF
−−
= = = =
CHUYÊN ĐỀ BỒI DƯỠNG HC SINH GII HÌNH HC 9
THCS.TOANMATH.com | 88
Bài 86. T một điểm A nằm ngoài đường tròn
( )
;OR
k các tiếp tuyến AB, AC đến
( )
O
(B, Ccác
tiếp điểm). Điểm D thuc cung nh BC, CD ct OA ti E, đường thng qua E song song vi BC ct
AB ti M.
a. Chng minh: EMBD là t giác ni tiếp.
b. MD là tiếp tuyến ca
( )
O
.
ng dn gii:
a. Do
//ME BC
nên ta có:
1
2
==AME ABC BOC
, li có:
11
22
BDE DBC DCB sđCD BD= + = +
2
1
2
1
sđ BC BOC==
suy ra
AME BDE=
hay EMBD là t giác ni tiếp.
b. Ta MBOE t giác ni tiếp đường
tròn đường kính MO (hs t chng minh)
T đó suy ra 5 điểm M, B, O, D, E cùng nm trên một đường tròn. Suy ra
0
90MDO =
hay MD
tiếp tuyến ca
( )
O
.
Bài 87. Cho tam giác nhn
, ABC AB AC
, đường tròn
( )
O
đường kính BC ct AB, AC lần lượt ti
F, E. Gi K giao điểm ca EF, BC, M giao điểm ca FD đường tròn
( )
O
. H giao điểm
ca BE, CF, AH ct BC ti D.
a. Chng minh:
..AE AC AF AB=
.
b. T giác KFOM là t giác ni tiếp và
..KF DM KM DF=
.
c. Gi S giao điểm ca AK đường tròn
( )
I
ngoi tiếp tam giác AEF. Chng minh
..AS AK AF AB=
d. Chng minh: S, H, O thng hàng.
ng dn gii:
CHUYÊN ĐỀ BỒI DƯỠNG HỌC SINH GIỎI HÌNH HỌC 9
89 | THCS.TOANMATH.com
a. T giác BFEC ni tiếp
( )
O
suy ra
AEF ABC=
( )
. AEF ABC g g
Suy ra
..
AE AF
AE AC AF AB
AB AC
= =
.
b. Ta có
KFM KFB BFM=+
KCE BFM BFM BFM= + = +
2BFM BOM==
Hay
KFM KOM=
suy ra KFOM t
giác ni tiếp, mt khác
OF OM=
suy ra
FKO MKO=
hay KO phân giác ca
FKM
.
Theo tính cht phân giác ta có:
KF DF
KM DM
=
đpcm.
c. H trc tâm ca
ABC
nên
0
90AEH AFH==
suy ra 4 đim A, E, H, F nằm trên đường
tròn đường kính AH. Suy ra S thuộc đường tròn tâm I đường kính AH, t giác ASFE ni tiếp
nên
ASF FEC=
, BFEC ni tiếp nên
FEC FBK=
suy ra KSFB ni tiếp
ASF ABK
. . .AS AK AF AB=
d. Gi s đường tròn
( )
I
ct AO ti N, ta có:
ANE AFE ACO==
suy ra ENOC ni tiếp, tương tự
FNOC ni tiếp. Ta
0 0 0 0
360 180 180 180HNC HNE ENC HNE EOC HAC EOC= = + = +
0 0 0
180 180 2 180EBC EOC EBC EBC EBC= + = + =
suy ra BHNC ni tiếp. Gi s KH ct
( )
I
ti
'N
suy ra
'EFHN
ni tiếp. Suy ra
. ' . . 'KH KN KE KF KB KC BHN C= =
ni tiếp, suy
ra
'NN
. Suy ra
0
90=KNO
hay H trc tâm ca tam giác AKO suy ra
OH AK
,
SH AK
suy ra S, H, O thng hàng.
CHUYÊN ĐỀ BỒI DƯỠNG HC SINH GII HÌNH HC 9
THCS.TOANMATH.com | 90
Bài 88. Cho tam giác nhn
( )
ABC AB AC
ni tiếp
( )
O
, các
tiếp tuyến ti B, C ca
( )
O
ct nhau ti
điểm M đường thng AM ct
( )
O
ti
giao điểm th 2 D (khác A). Trên
đường thng qua O vuông góc vi AD
ta lấy điểm I (O, I nm khác phía so
vi AD) dựng đường tròn
( )
;I ID
ct
các đường thng BD, DC lần lượt ti
E, F
( )
,E D F C
.
a. Chng minh:
2
MA BA
MD BD

=


.
b. Gi K là giao điểm ca BC vi EF.
Chứng minh: 4 điểm A, C, K, F cùng nm trên một đường tròn.
c. Chng minh:
ACD AKE
suy ra K là trung điểm ca EF.
d. Dng EX, FY lần lượt vuông góc vi AI. Chng minh:
KX KY=
.
Gii:
a. Ta d chứng minh được:
( . )MDB MBA g g
suy ra
2
MDB
MBA
S
DB
S BA

=


mt khác
MDB
MBA
S
MD
S MA
=
t
đó suy ra
2
DB MD
BA MA

=


.
b. Xét t giác ACKF ta có:
ACK ACB ADB ADE AFE AFK= = = = =
suy ra ACFK t giác ni
tiếp.
c. Ta
,= = = ACD ACF AKE ADC AEK ACD AKE
suy ra
( )
*
KE CD
KA CA
=
, hoàn toàn
tương tự ta cũng có:
ABD AKF
suy ra
KF BD
KA BA
=
(**). Mặt khác ta cũng d dàng chng
minh
=
BD MB
MDB MBA
BA MA
=
CD MC
MDC MCA
CA MA
MC MB=
suy ra
CD BD
CA BA
=
kết hp vi (*), (**) suy ra
KE KF
KA KA
=
hay K là trung điểm ca EF.
d. Dựng đường cao AH ca tam giác AEF, gi N trung điểm ca AE, ta 5 đim E, N, I, Y, K
nằm trên đường tròn đường kính EI, 4 đim A, E, H, I nằm trên đường tròn đường kính AE. T đó
CHUYÊN ĐỀ BỒI DƯỠNG HỌC SINH GIỎI HÌNH HỌC 9
91 | THCS.TOANMATH.com
ta biến đổi góc:
1
2
YNK YEK YEH YAH YNH= = = =
suy ra NK phân giác ca góc
HNY
tam giác HNY cân ti N nên NK là trung trc ca HI. Chng minh tương t ta cũng có K thuc trung
trc ca HX suy ra K là tâm đường tròn ngoi tiếp tam giác HXY nên
KX KY=
.
Bài 89. Cho tam giác ABC ni tiếp
( )
O
trung tuyến
AM, điểm P nm trên cung BC không cha A ca
( )
O
,
hai điểm E, F thuc CA, AB sao cho
/ / , / /PE AB PF AC
.
Đưng thng AM cắt đường tròn ngoi tiếp tam giác AEF
ti N khác A. Gi G là giao đim AP EF, D là điểm đối
xng vi A qua M.
a. Chng minh:
PBC EAP
.
b. Chng minh:
NFE CAD
NGE AMB=
.
c. Chng minh:
2
2.AP AM AN=
.
Gii:
a. Xét tam giác PBC tam giác EAP ta có:
PBC PAE=
(cùng chn cung PC). T giác AEPF hình bình hành
nên
EPA PAF=
li
PAF PCB=
t đó suy ra
EPA PCB=
suy ra
( )
. .PBC EAP g g
b. Ta có:
,NFE NAE DAC NEF NAF ADC= = = =
suy
ra
( )
~ .NFE CAD g g
NG, CM là các trung tuyến tương ứng nên
NGE CMD AMB==
.
c. T b ta suy ra
AGN AGE NGE PMB BMA PMA= + = + =
. Suy ra NGMP là t giác ni tiếp.
Nên
2
2 . 2 .AN AM AG AP AP==
.
Bài 90. Cho tam giác ABC ni tiếp
( )
O
, P một điểm bt k trong tam giác
Q
cung nh BC
ca
( )
O
, AP ct
( )
O
tại giao điểm th 2 D, M trung đim ca AQ, QP ct
( )
O
tại giao điểm
th 2 K. Dựng đường tròn
( )
w
qua 2 điểm P, K tiếp xúc vi AP. Các đường thng AK, MP ln
t ct
( )
w
ti E, F. Gi R là giao điểm của đường tròn ngoi tiếp tam giác KPD vi MP.
a. Chng minh: AMDR là t giác ni tiếp.
b. Chng minh:
KFP KRD
suy ra F là trung điểm ca PR.
c. Chứng minh: 4 điểm A, D, E, F nm trên một đường tròn.
Gii:
CHUYÊN ĐỀ BỒI DƯỠNG HC SINH GII HÌNH HC 9
THCS.TOANMATH.com | 92
a. T các t giác KPDR, AKDQ ni tiếp ta suy
ra
DRP DKP DAQ==
hay AMDR t giác
ni tiếp.
b. Ta có: PA tiếp tuyến của đường tròn
( )
w
KPDR ni tiếp nên
KFP KPA KRD==
, li
KPR KDR KPF KDR= =
nên
( )
.KFP KRD g g
suy ra
FP KP
RD KD
=
.
Li có:
( )
.KPD APQ g g
suy ra
,=
KP AP
PRD PAM
KD AQ
nên
2
,
PR PA PA
RD AM AQ
==
t đó ta có:
2 2 2PR PA KP FP
RD AQ KD RD
= = =
suy ra
F là trung điểm ca RP.
c. Ta
00
, 180 180KRF KDP KFR KFP KPA KPD= = = =
suy ra
( )
~.KFR KPD g g
kết
hp vi
PKF DPF=
suy ra
~ ( . . )KPF PDF c g c
suy ra
0
180ADF PDF KPF AEF= = =
hay
ADEF là t giác ni tiếp.
Bài 91. Cho tam giác ABC nhn ni tiếp
( )
O
trc tâm H, M trung điểm ca BC, đường cao AF,
đường tròn đường kính AH ct
( )
O
ti Q khác A. Đường tròn đường kính HQ ct
( )
O
ti K khác Q.
Dựng đường kính AE ca
( )
O
, D là giao điểm ca AH vi
( )
O
.
a. Chng minh: Q, H, M, E thng hàng.
b. Tiếp tuyến ti H, K của đường tròn ngoi tiếp tam giác QKH ct nhau ti X. Chng minh X thuc
BC.
c. Chứng minh: Các đường tròn ngoi tiếp tam giác KQH, KFM tiếp xúc nhau.
Gii:
a. Ta có kết qu quen thuc:
+ H, M, E thng hàng.
+ H đối xng vi D qua BC.
+ Q, H, E thng hàng. Do QH, EQ cùng vuông góc vi AQ. T đó suy ra Q, H, M, E thng hàng.
b. Gi s các tiếp tuyến ti K, H của đường tròn
( )
L
ngoi tiếp tam giác QHK ct nhau ti X. Ta
chng minh:
X BC
. Tht vy:
Do
( )
0 0 0 0
180 2 180 2 2 90 2 90 ) 2 2(= = = = = =KXH XHK KQH KQH KAE EAK KDH
CHUYÊN ĐỀ BỒI DƯỠNG HỌC SINH GIỎI HÌNH HỌC 9
93 | THCS.TOANMATH.com
XK XH=
nên suy ra Xtâm vòng tròn ngoi tiếp tam giác KHD, suy ra X nm trên trung trc ca
DH hay
X BC
.
Bài 92. Cho đường tròn tâm O đường kính
2AB R=
, trên đoạn OA lấy điểm
( )
,I I A I O
. V tia
Ix AB
ct
( )
O
ti C. Lấy điểm E trên cung nh
( )
,BC E B E C
ni AE ct CI ti F, gi D
giao điểm ca BC vi tiếp tuyến ti A ca
( )
;OR
.
a. Chng minh: BEFI là t giác ni tiếp.
b. Chng minh:
..AE AF CBCD=
.
c. Tia BE ct IC ti K. Gi s I, F lần lượt là trung điểm ca OA, IC. Chng minh:
AIF KIB
t
đó tính IK theo R.
d. Khi I trung điểm ca OA E chy trên cung nh BC. Tìm v trí điểm E để
EB EC+
ln
nht.
Gii:
a. điểm E nm trên
( )
O
đường kính AB nên
0
90AEB =
, li
FI AB
ti I nên
0
90FIB =
. T
giác IFEB
0 0 0
90 90 180AEB FIB+ = + =
nên IFEB là t giác ni tiếp.
CHUYÊN ĐỀ BỒI DƯỠNG HC SINH GII HÌNH HC 9
THCS.TOANMATH.com | 94
b. Xét tam giác AIF, AEB
0
90 ,AIF AEB EAB==
chung, suy ra
(1) ( . )AIF AEB g g
dẫn đến
..
AI AE
AE AF AI AB
AF AB
= =
, điểm C nm
trên
( )
O
đường kính AB nên
0
90 ,ACB AD=
tiếp tuyến ca
( )
O
nên
0
90DAB =
áp dng h
thức lượng trong các tam giác vuông ACB, BAD
ta có:
2
..AI AB AC CDCB==
suy ra
..AE AF CDCB=
.
b. Tam giác AIFKIB
00
90 , 90AIF KIB IAF IKB KBA= = = =
nên
( )
.
.. = =
AI IF AI IB
AIF KIB g g KI
KI IB IF
Vì điểm I là trung điểm OA
CI OA
nên tam
giác OAC cân ti C, li
OC OA R==
nên tam giác OAC tam giác đu, suy ra
0
60COA=
.
0
1
30
2
CBA COA==
, t đó dễ tính được:
3 1 3
2 2 4
CI R IF CI R= = =
nên
2
.
23
1
2
IAIB IC
KI IC R
IF
IC
= = = =
.
c. Gi M là giao điểm ca tia CI vi
( )
O
( )
MC
thì tam giác CBI cân ti B.
Kết hp vi chng minh câu c
00
30 60CBA CBM= =
ta suy ra tam giác CBM đều. Lấy đim
N trên đoạn EM sao cho
EN EC=
, do
0
60CEM CBM==
nên suy ra tam giác CEN đều. Xét các
,CNM CEB
ta có:
0 0 0 0
180 120 , 180 120CNM CNE CEB CMB= = = =
,CE CN CMN CBE==
( )
. .MCN BCE CNM CEB c g c = =
suy ra
MN EB=
.
T đó ta có:
2EC EB EN MN EM R+ = + =
, dấu đẳng thc xy ra khi ch khi EM đường
kính ca
( )
O
, hay E là điểm đối xng vi M qua O.
Nhn xét: Nếu biết định lý Ptolemy thì ta kết qu:
. . .EC MB EB MC BC EM+=
tam giác
BMC đều nên
MB MC BC==
suy ra
EC EB EM+=
.
CHUYÊN ĐỀ BỒI DƯỠNG HỌC SINH GIỎI HÌNH HỌC 9
95 | THCS.TOANMATH.com
Bài 93. Cho điểm A nm ngoài
( )
;OR
. T A v các tiếp tuyến AB, AC đến
( )( )
;O R O
(B, C c
tiếp điểm) và cát tuyến AMN đến
( )
;OR
, MN không đi qua O
AM AN
).
1. Chng minh: ABOC là t giác ni tiếp.
2. Chng minh:
2
.AM AN AB=
.
3. Tiếp tuyến ti N ca
( )
;OR
cắt đường thng BC ti F. Chng minh: FM là tiếp tuyến ca
( )
;OR
.
4. Gi P giao đim ca BC dây MN, E giao đim của đường tròn ngoi tiếp tam giác MON
và đường tròn ngoi tiếp t giác ABOC (E khác O). Chng minh: P, E, O thng hàng.
Gii:
1. Gi H giao điểm ca BC, AO thì
BH AO
theo câu b) ta
2
.AM AN AB=
,
áp dng h thức lượng trong tam giác vuông
ABO ta có:
2
.AB AH AO=
, t đó suy ra
( )
. . . .= AH AO AM AN AHM ANO c g c
Dn ti
AHM ANO MHON=
là t giác ni
tiếp. Gi s tiếp tuyến ti M, N ca
( )
O
ct
nhau ti F suy ra 4 đim F, M, O, N cùng nm
trên đường tròn đường kính OF.
Ta chng minh: F, B, H thng hàng. Tht vy:
do 4 điểm M, H, O, N nm trên một đường
tròn 4 điểm F, M, O, N cùng nm trên mt
đường tròn đường kính OF ta suy ra 5 đim F,
M, H, O, N
cùng nằm trên đường tròn đường kính OF hay
0
90FHO FH AO=
, li
BH AO
(Tính
cht 2 tiếp tuyến ct nhau). T đó suy ra F, B, H thng hàng.
2. T chứng minh trên ta suy ra đường tròn ngoi tiếp tam giác OMN cũng đường tròn đường
kính OF, E giao điểm của đường tròn đường kính OF với đường tròn đường kính AO ta
0
90AEO =
0
90OEF =
suy ra
0
180+=AEO OEF
hay A,E, F thng hàng.
Gi K trung điểm MN thì
OF MN
ti K, li
FP OA
ti H suy ra P trc tâm tam giác
FOA suy ra
OP AF
do
,,EP AF E P O⊥
thng hàng.
Bài 94. Cho tam giác nhn ABC ni tiếp đường tròn
( )
O
có các đường cao AD, BE, CF ct nhau ti
điểm H, AH ct
( )
O
tại giao điểm th 2 là P (P khác A). Gi M, I lần lượt trung điểm ca BC,
CHUYÊN ĐỀ BỒI DƯỠNG HC SINH GII HÌNH HC 9
THCS.TOANMATH.com | 96
AH. Dựng đường kính AK ca
( )
O
, đường thng qua P song song vi EF ct
( )
O
tại giao điểm th
2 là Q (khác P). Gi S là giao điểm ca IMEF.
a. Chng minh: BFEC là t giác ni tiếp.
b. Chng minh:
IE EM
t đó suy ra
2
.ME MS MI=
.
c. Chng minh: Tam giác APQ cân.
d. Đưng tròn ngoi tiếp tam giác AEF ct
( )
O
tại giao đim th 2 N. Chng minh: N, S, Q thng
hàng.
Gii:
a. BE, CF các đường cao ca tam giác ABC
nên
0
90BEC BFC==
suy ra BFEC t giác
ni tiếp (Có 2 đnh liên tiếp trên cnh EF cùng
nhìn cnh BC góc bng nhau).
b. Do I trung điểm AH nên
2
AH
IA IE IH= = =
dn ti tam giác AIE cân ti
I suy ra
( )
1 .IEA IAE=
Tương t ta cũng
có:
( )
2MEC MCE=
. Ly
( ) ( )
1 2+
ta có:
0
90IEA MEC IAE MCE DAC DCA+ = + = + =
suy ra
( )
00
180 90IEM IEA MEC= + =
nên
IE EM
. Do
,IE IF ME MF==
nên IM là trung trc
ca EF hay
IM EF
ti S. Áp dng h thức lượng trong tam giác vuông IEM ta
có:
2
.ME MI MS=
.
c. Do BFECt giác ni tiếp nên
AEF ABC=
(tính cht t giác ni tiếp). Li có:
1
2
ABC AOC=
( )
00
1
180 2 90
2
= = OAC OAC
(do tam giác OAC cân ti O), t đó suy ra
0
90ABC OAC+=
hay
0
90AEF OAC OA EF+ =
,
//PQ EF PQ OA⊥
PQ 1 dây ca
( )
O
suy ra
OA PQ
tại trung điểm ca PQ hay tam giác APQ cân ti A.
d. Do
0
90 ,= = AEH AFH E F
nằm trên đường tròn tâm I đường kính AH hiu
( )
I
suy
ra
( )
0
90 3HNA =
. Li có
, / /KC AC BH AC BH KC
, tương tự
//CH KB
nên BHCK là hình
bình hành, suy ra H, M, K thẳng hàng (5). Ta cũng
0
90KNA =
(4), t (3), (4) suy ra K, H, N
thng hàng (6). T (5), (6) suy ra K, H, M, N thng hàng.
CHUYÊN ĐỀ BỒI DƯỠNG HỌC SINH GIỎI HÌNH HỌC 9
97 | THCS.TOANMATH.com
Ta
( )
.MHF MFN g g
suy ra
22
.==MF MH MN ME
(do
ME MF=
) kết hp vi chng
minh câu b ta suy ra
. . ( . . )= MH MN MI ME MHS MIN c g c
dẫn đến
HSM INM NISH=
t giác ni tiếp. Kết hp vi
//IM AK
tam giác APQ cân ti A ta có:
HNS HIS PAK QAK QNK= = = =
NQ NS
hay Q, S, N thng hàng.
Bài 95. Cho tam giác ABC nhọn các đường cao AD, BE, CF ct nhau tại điểm H. Gi P, Q ln
t là hình chiếu vuông góc ca D lên BH, CH.
a. Chứng minh: 4 điểm H, P, D, Q nm trên một đường tròn.
b. Chng minh:
//PQ EF
.
c. Đưng tròn tâm I đường kính AD ct AB, AC lần lượt ti R, S. Chng minh R, P, Q, S thng hàng.
d. Đưng tròn
( )
J
ngoi tiếp tam giác DQS ct IC ti T, DT ct AC ti K. Chng minh:
KA CA
KS CS
=
.
Gii:
a.
0
90HPD HQD==
nên 4 điểm P, H,
Q, D nằm trên đường tròn đường kính HD.
b. Do t giác HPDQ ni tiếp nên
( )
1HPQ HDQ=
(cùng chn cung HQ). Ta
cũng có:
HDQ HCD=
(2) (cùng ph
vi
QDC
), t giác BFEC
0
90BFC BEC==
nên BFEC ni tiếp
đường tròn đường kính BC suy ra
FCB FEB=
(3). T (1), (2), (3) suy
ra
//HPQ HEF PQ EF=
.
c. Đưng tròn tâm I đường kính AD ct AB,
AC lần lượt ti R, S thì
0
90ARD ASD==
suy ra
0
90BRD BSC==
nên t giác DQSC,
BRPD ni tiếp.
Ta có:
SQC SDC DAC==
(4).
==PQD PHD AHE
(5), mà
0
90AHE DAC+=
(4), (5) ta suy ra
0
180PQD DQC SQC+ + =
hay P, Q, S thng hàng, chứng minh tương tự ta có: P,
Q, R thng hàng, suy ra R, P, Q, S thng hàng.
d. Do 4 điểm D, Q, S, C cùng nằm trên đường tròn đường kính DC nên đường tròn tâm J ngoi tiếp
tam giác DQC cũng chính đường tròn đường kính DC suy ra
0
90DTC =
. Ta
CSD CDA
CHUYÊN ĐỀ BỒI DƯỠNG HC SINH GII HÌNH HC 9
THCS.TOANMATH.com | 98
nên suy ra
2
.CD CS CA=
(6). Trong tam giác vuông IDC ta cũng có:
2
.CD CT CI=
(7). T (6), (7)
ta suy ra
..= =CS CA CT CI CTS CAI CTS CAI
nên AITS t giác ni tiếp, kết hp vi
AIS
cân ti I ta có biến đổi góc:
CTS CAI ISA ITA= = =
suy ra
ATK STK=
hay TK là phân giác
ca góc
ATS
, TC vuông góc vi TK nên TC chính là phân giác ngoài ca góc
ATS
.
Theo tính cht phân giác ta có ngay
KA CA
KS CS
=
đpcm.
Bài 96. Cho đường tròn tâm O bán kính R một điểm N nm ngoài
( )
O
. T N k 2 tiếp tuyến NA,
NB đến
( )
O
(A, B là 2 tiếp điểm). Gi E là giao điểm ca AB, ON.
a. Chng minh: NAOB là t giác ni tiếp.
b. Tính AB biết
5 , 3ON cm R cm==
.
c. K tia Nx trong góc
ANO
ct
( )
O
ti C, D (C nm gia ND). Chng minh:
NEC OED=
.
d. Qua N k cát tuyến th 2 với đường tròn là NPQ (P nm gia N, Q). Gi I giao điểm CQ, DP.
Chng minh: A, B, I thng hàng.
Gii:
a. NA, NB các tiếp tuyến ca
( )
O
nên
0
90NAO NBO==
suy ra
0
180+=NAO NBO
. Hay t giác NAOB ni
tiếp (tổng 2 góc đối bng
0
180
).
b. Khi
5 , 3ON cm R cm==
, theo tính cht 2
tiếp tuyến ct nhau ta có
2AB AE=
.
AE NO
ti E, áp dng h thức lượng
trong tam giác vuông AON ta có:
2
.OA OE ON=
Suy ra
2
9 9 16
5
5 5 5
OA
OE NE NO OE
ON
= = = = =
, li có
22
16.9
.
25
AE EN EO AN= =
12 24
2.
55
AN AB AN = = =
c. Xét tam giác NCA, NAD ta có:
AND
chung và
1
2
NAC NDA sđ AC==
.
Suy ra
( )
.NCA NAD g g
dn ti
NC NA
NA ND
=
(t s đồng dng) suy ra
( )
2
. 1NA NC ND=
CHUYÊN ĐỀ BỒI DƯỠNG HỌC SINH GIỎI HÌNH HỌC 9
99 | THCS.TOANMATH.com
Áp dng h thức lượng trong tam giác vuông NAO ta có:
( )
2
. 2NA NE NO=
. T (1), (2) ta suy ra
..
NC NO
NC ND NE NO
NE ND
= =
kết hp vi
,NCE NOD
góc
OND
chung ta suy ra
NCE NOD
(c.g.c) suy ra
NEC NDO=
dn ti CEOD ni tiếp (góc ngoài một đỉnh bng góc
đối). Cũng từ CEOD ni tiếp tam giác OCD cân ti O ta biến đổi góc
NEC NDO OCD OED= = =
(đpcm).
d. Tương tự câu c ta có: PEOQ ni tiếp.
Ta có:
( ) ( )
00
11
180 180
22
= + = + = + CEP CEN NEP ODC OQP COD POQ
( ) ( ) ( )
( )
( )
0
0 0 0
360
1 1 1 1
180 180 180
2 2 2 2 2
−+
+ = + = = +
COD POQ
COD POQ COD POQ COP DOQ
( )
1 1 1
2 2 2
+ = + =COP DOQ CP QD CIP
suy ra CEIP ni tiếp, tương tự DEIQ ni tiếp.
Suy ra
= = =IEP ICP IDQ IEQ
hay EI phân giác ca
PEQ
(3). T chng minh câu c ta suy ra
CEA DEA=
hay EA phân giác ca góc
CED
, tương tự ta suy ra EB phân giác ca góc
PEQ
(4). T (3), (4) suy ra
EI EB
hay I, A, B thng hàng.
Bài 97. Cho đường tròn tâm I ni tiếp tam giác ABC tiếp xúc vi BC, CA, AB ln lut ti D, E, F.
Đưng thng qua A song song vi BC ct EF ti K, gi M trung đim BC Chng
minh:
MI DK
.
Gii:
Gi P là giao điểm ca IDEF
AE, AF là các tiếp tuyến ca
( )
I
ti E,
F nên
IA EF
.
IP BC
suy
ra
IP AK
. T đó suy ra P trc tâm
ca tam giác
AIK AP IK⊥
ti H suy
ra t giác ANHK ni tiếp.
K đường thng qua P song song vi BC
ct AB, AC ti X, Y t các t giác IPXF,
IPEY
ni tiếp ta suy ra
PXI PFI PEI PYI===
dn ti tam giác IXY cân ti I nên P trung điểm XY.
Theo định lý Thales ta d suy ra A, P, M thng hàng. Li có:
22
..IH IK IN IA IF ID= = =
IH ID
ID IK
=
suy ra
IHD IDK
(c.g.c).
CHUYÊN ĐỀ BỒI DƯỠNG HC SINH GII HÌNH HC 9
THCS.TOANMATH.com | 100
T giác IDMH ni tiếp suy ra
,IDH IMH=
~IHD IDK IDH IKD IMH IKD = =
IMH
ph vi
MIH
nên
IKD
ph vi
MIH IM DK⊥
.
Bài 98. Cho tam giác nhn ABC ni tiếp
( )
O
. Một đường tròn
( )
K
đi qua B, C ct AC, AB lần lượt
ti E, F, BE ct CF ti H, AH ct
( )
O
ti P khác A, PE ct
( )
O
ti R khác P.
Đưng tròn ngoi tiếp tam giác FHB ct AH ti D.
a. Chng minh:
KD AH
.
b. Gi s AH ct BC ti LQ đối xng vi P qua D. Chng minh: AEQF là t giác ni tiếp.
c. Chng minh: BR chia đôi EF.
Gii:
a. Ta có:
. . .AH AD AF AB AE AC==
suy ra EHDC ni tiếp. T đó ta có:
0
360BDC BDH CDH=
2BFC BEC BFC BKC= + = =
nên BDKC là t giác ni tiếp.
Ta có:
KDH CDH CDK=−
=
0
180 HEC KBC =
0
00
180
180 90
22
BKC BKC
=
.
b. Gi N là điểm đối xng vi F qua KD, do
KF KN=
nên điểm N nm trên
( )
K
.
Ta
BCP BAP BFN BCN= = =
nên
CP CN
hay C, N, P thng hàng. Do
FQPN hình thang cân ta có:
FQA FNC FBC==
nên BFQL ni tiếp.
Tương tự ta cũng có: CEQL ni tiếp.
T đó ta biến đổi góc:
0
360FQE FQL EQL=
0
180ABC ACB BAC= + =
suy ra AEQF
ni tiếp.
c. T AEQF t giác ni tiếp ta
có:
00
180 180BFE AFE AQE DQE= = =
.
Ta cũng có:
FBE FCE HDE==
nên
( . )EFB EQD g g
suy ra
FB DQ
FE QE
=
.
Gi I là trung điểm EF ta có
CHUYÊN ĐỀ BỒI DƯỠNG HỌC SINH GIỎI HÌNH HỌC 9
101 | THCS.TOANMATH.com
1
2
2
QP
FB DQ FB FB QP
FE QE FI QE FI QE
= = =
( )
. . IFB EQP c g c
nên
FBI EPQ ABR==
hay
BR BI
tc là BR chia đôi EF.
Bài 99. Cho tam giác ABC ni tiếp
( )
O
, P một điểm bt k trong tam giác
Q
cung nh BC
ca
( )
O
, AP ct
( )
O
tại giao điểm th 2 D, M trung điểm ca AQ, QP ct
( )
O
tại giao điểm
th 2 K. Dựng đường tròn
( )
w
qua 2 điểm P, K tiếp xúc vi AP. Các đường thng AK, MP ln
t ct
( )
w
ti E, F. Gi R là giao điểm của đường tròn ngoi tiếp tam giác KPD vi MP.
a. Chng minh: AMDR là t giác ni tiếp.
b. Chng minh:
~KFP KRD
suy ra F là trung điểm ca PR.
c. Chứng minh: 4 điểm A, D, E, F nm trên một đường tròn.
Gii:
a. T các t giác KPDR, AKDQ ni tiếp ta suy ra
DRP DKP DAQ==
hay AMDR là t giác ni tiếp.
b. Ta có: PA tiếp tuyến của đường tròn
( )
w
KPDR ni tiếp nên
KFP KPA KRD==
, li có
KPR KDR KPF KDR= =
nên
( . )KFP KRD g g
suy ra
FP KP
RD KD
=
Li có:
( . )KPD APQ g g
suy ra
,=
KP AP
PRD PAM
KD AQ
nên
2PR PA PA
RD AM AQ
==
, t đó ta
có:
2 2 2PR PA KP KF
RD AQ KD RD
= = =
suy ra F là trung điểm ca RP.
c. Ta
00
, 180 180KRF KDP KFR KFP KPA KPD= = = =
suy ra
( . )KFR KPD g g
kết
hp vi
PKF DPF=
suy ra
( . . )KPF PDF c g c
suy ra
0
180ADF PDF KPF AEF= = =
hay
ADEF là t giác ni tiếp.
Bài 100. Cho đường tròn tâm O đường kính BC. Một điểm A thuộc đường tròn
( )
O
sao
cho
AB AC
. Tiếp tuyến ti A ca
( )
O
ct BC ti D. Gi E là điểm đối xng vi A qua BC, AE ct
BC ti M, k đường cao AH ca tam giác ABE, AH ct BC ti F.
a. Chng minh: AFEC là hình thoi.
b. Chng minh:
..DC DB DM DO=
.
CHUYÊN ĐỀ BỒI DƯỠNG HC SINH GII HÌNH HC 9
THCS.TOANMATH.com | 102
c. Gi I là trung điểm AH, kéo dài BI ct
( )
O
tại đim th 2 K. Chng minh: AIMK là t giác ni
tiếp.
d. Đưng thng AK ct BD ti N. Chng minh: N là trung điểm ca MD.
Gii:
a. Do điểm E đối xng vi A qua đường
kính BC nên
( )
EO
dn ti
0
90BEC =
hay
BE CE
, li có:
AH BE
suy ra
( )
/ / 1 ,AH CE
ta cũng
AE BM
ti
trung điểm M ca BC nên F trc tâm
tam giác ABE dẫn đến
EF AB
, mt
khác
AC AB
Suy ra
//EF AC
(2), t (1), (2) kết hp
vi
AE FC
ta suy ra ACEF hình
thoi.
b. Do DA tiếp tuyến ca
( )
O
nên
1
2
CAD ABD sđ AC==
suy ra
( . )DAB DCA g g
suy ra
DA DB
DC DA
=
hay
( )
2
. 3 .DA DB DC=
Trong tam giác vuông OED (
0
90OAD =
ta
AM OD
nên
suy ra
( )
2
. 4 .DM DO DA=
T (3), (4) ta có:
..DB DC DM DO=
.
c. Do I trung điểm AH nên MI đường trung bình ca tam giác AHE suy ra
MI AH
( )
// MI HE AH
. Ta
IMA AEB=
ng v),
AEB AKB=
(góc ni tiếp cùng chn
cung AB) dẫn đến
IMA IKA AIMK=
t giác ni tiếp (có 2 đỉnh liên tiếp cùng nhìn cnh IA
góc bng nhau).
d. T chng minh câu c ta có:
00
180 90AKM AIM= =
. Áp dng h thức lượng trong tam giác
vuông AMN ta có:
( )
2
. 5MN NK NA=
. Kéo dài DK ct
( )
O
ti S. Ta d chứng minh được DE là tiếp
tuyến ca
( )
O
. Ta
= =KMD KAM KAE KED
suy ra KMDE t giác ni tiếp. Dn ti
0
90EKD EMD==
suy ra SE đường kính ca
( )
O
nên
SA AE
dn ti
//SA ND
. Tam giác
KND và tam giác DNA
AND
chung và
NDK DSA KAD==
suy ra
( . )DNA KND g g
suy ra
( )
2
. 6= =
DN NA
ND NA NK
KN ND
. T (5), (6) suy ra
22
ND MN=
hay
ND NM=
.
CHUYÊN ĐỀ BỒI DƯỠNG HỌC SINH GIỎI HÌNH HỌC 9
103 | THCS.TOANMATH.com
Bài 101. Cho đường tròn
( )
;OR
một điểm M nm ngoài
( )
O
, qua M k các tiếp tuyến MA, MB
đến
( )
O
, dng cát tuyến MCD sao cho
MC MD
tia MC nm gia 2 tia MB, MO. Gi I
trung điểm ca CD.
a. Chng minh: T giác AIOB ni tiếp.
b. Tia BI ct
( )
O
tại giao điểm th 2 là J (J khác B). Chng minh:
//AJ MD
.
c. Chng minh:
2
. DAAD J M=
.
Đưng thng qua I song song vi BD ct AB ti K. Tia CK ct OB ti G. Khi cát tuyến MCD thay
đổi và thỏa mãn điều kiện đề bài thì tâm đường tròn ngoi tiếp tam giác CIG thuộc đường tròn nào?
Gii:
a. I trung điểm ca dây CD nên
OI CD
(Quan h vuông góc giữa đường kính và 1 dây) suy
ra
0
90 , , OIM MA MB=
là tiếp tuyến ca
( )
O
nên
00
90 180OAM OBM OAM OBM= = + =
hay
t giác MAOB ni tiếp (tổng 2 góc đối nhau bng
0
180
). Tương t MAOI ni tiếp, suy ra 5 đim M,
A, O, I, B nm trên một đường tròn. Suy ra t giác AIOB ni tiếp.
b. 5 điểm M, A, O, I, B nm trên một đường tròn nên
MIB MOB=
(hai góc ni tiếp cùng chn
cung MB). Mặt khác ta cũng
1
2
AJB AOB=
(liên h góc ni tiếp góc tâm),
2AOB MOB=
(Tính cht 2 tiếp tuyến ct nhau), t đó suy ra
//AJB MIB AJ MD=
. Do
//AJ MD
nên
JAD ADM=
(so le trong) (1). T giác AJDC ni tiếp nên
AJD ACM=
(2), ta
MCA MAD
(g.g) (hc sinh t chng minh) suy ra
MCA MAD=
(3). T (2) (3) ta suy ra
AJD MAD=
(4). T (1), (4) ta có:
DAM AJD
(g.g) suy ra
AD DM
AJ AD
=
(cạnh tương ng) hay
2
.AD JA MD=
.
CHUYÊN ĐỀ BỒI DƯỠNG HC SINH GII HÌNH HC 9
THCS.TOANMATH.com | 104
c.
//IK BD
nên
KIC CDB=
ng v),
CDB CAK=
(cùng chn cung BC) suy ra
KIC KAC ACKI=
ni tiếp (hai đỉnh liên tiếp A, I cùng nhìn cnh CK góc bng nhau). T đó
suy ra
CKA CIA=
(5) (góc ni tiếp cùng chn cung CA). Mặt khác do 4 đim A, I, M, B nm trên
cùng một đường tròn nên
AIM ABM=
(góc ni tiếp cùng chn cung BM) (6). T (5), (6) suy ra
//CKA MBA CK MB=
MB OB CK OB
ti G. T giác CGIO
0
90CGO CIO==
nên CGOI t giác ni tiếp đường tròn đường kính CO (hai đỉnh liên tiếp G, I cùng nhìn cnh CO
góc bng
0
90
). Suy ra bán kính đường tròn ngoi tiếp tam giác CGI bng
22
CO R
=
, hay tâm đường
tròn ngoi tiếp tam giác CIG thuộc đường tròn tâm O bán kính
2
R
.
Bài 102. T đim M nằm ngoài đường tròn
( )
O
k các tiếp tuyến MA, MB đến
( )
O
(A, B là các tiếp
điểm) và cát tuyến MCD sao cho
MC MD
và tia MC nm gia 2 tia MA, MO. Gi H là giao điểm
ca ABMO, Etrung đim ca CD, đường thng CH ct
( )
O
tại giao đim th 2 là F (khác C).
Đưng thng DF ct các tia MA, MB lần lượt ti P, Q.
a. Chng minh: EM là tia phân giác ca
AEB
.
b. Chng minh:
..=MH MO MC MD
.
c. Chng minh:
//DF AB
.
d. Gi X là giao điểm ca EP vi AD, Y là giao điểm ca EQ vi BD. Chng minh: HD chia đôi XY.
Gii:
a. Chứng minh 5 đim M, A, E, O, B nm trên một đường tròn, chú ý
MA MB=
nên
AEM BEM=
suy ra đpcm.
b. Ta d chng minh:
2
.MA MC MD=
, áp dng h thức lượng trong tam giác vuông MAO,
AH MO
ta có
2
.MA MH MO=
.
T đó suy ra
..MH MO MC MD=
.
CHUYÊN ĐỀ BỒI DƯỠNG HỌC SINH GIỎI HÌNH HỌC 9
105 | THCS.TOANMATH.com
c. T b ta có:
MH MD
MC MO
=
. Xét tam giác
MCH, MOD ta có:
MH MD
MC MO
=
OMD
chung nên
( )
.MCH MOD g g
suy ra
MHC MDO=
t giác CHOD ni tiếp ta
có biến đổi góc
MHC MDO OCD ODH= = =
=MHC OHD
, các góc
,CHA DHA
ph
vi các góc
,MHC OHD
tương ng n
suy ra
CHA DHA=
hay AH phân giác
ca góc
CHD
.
Chú ý rng:
11
22
CHA CHD COD CFD===
mà hai góc
,CHA CFD
đồng v nên
//AB DF
.
d. Ta có:
===AEM MEB MAB MPQ
suy ra APDE là t giác ni tiếp. Tương tự ta cũng có BQDE
t giác ni tiếp. Ta
0
180XEY XED YED DAP DBQ ABD BAD ADB= + = + = + =
nên
EXDY là t giác ni tiếp.
T đó suy ra
DXY DEY DBQ DAB= = =
hay
//XY AB
. Theo định lý Thales ta có ngay DH đi qua
trung điểm I ca XY.
Bài 103. Cho tam giác ABC vuông ti A
( )
AB AC
, trên cnh AC ly điểm D (D khác A, C)
dng
DE BC
. Tiếp tuyến ti C của đường tròn tâm K ngoi tiếp tam giác DEC ct tia DE ti M.
Đưng thng qua C vuông góc vi MK cắt đường tròn
( )
K
ti F (F nm trong tam giác ABC).
a. Chng minh: T giác ADEB ni tiếp và
..CDCA CECB=
.
b. Chng minh:
2
.MF ME MD=
.
c. Tia CF ct cnh AB ti I. Chng minh:
BIC FEC
.
d. Chng minh: I là trung điểm ca AB.
Gii:
a. tam giác ABC vuông ti E nên
0
90 ,BAD ED BC=⊥
nên
0
90DEB =
suy ra
0
180BAD BED+=
suy ra t giác ADEB ni tiếp (tổng 2 góc đi bng
0
180
) Tam giác vuông BAC
tam giác vuông DEC
0
90BAC DEC==
ACB
chung nên
BAC DEC
(g.g) suy ra
AC BC
EC DC
=
hay
..AC DC CE BC=
.
CHUYÊN ĐỀ BỒI DƯỠNG HC SINH GII HÌNH HC 9
THCS.TOANMATH.com | 106
b. Gi s
CF KM
ti H thì H là trung điểm
ca FC (Quan h vuông góc của đường kính
đi qua trung đim 1 dây) suy ra KH phân
giác ca góc
FKC
. Xét các tam giác KFB
KCM, ta có:
KC KF=
(là bán kính ca
( )
K
), KM chung,
( )
. .FKM CKM KFB KCM c g c= =
suy ra
0
90KFM KCM==
hay MF tiếp
tuyến ca
( )
K
. T đó ta
có:
1
2
MFE EDF sđ EF==
, dẫn đến
MEF MFD
(g.g) suy ra
ME MF
MF MD
=
hay
2
.MF ME MD=
.
c. Do điểm F nằm trên đường tròn đường kính CD nên
00
90 90CFD DFI= =
suy ra t giác
ADFI ni tiếp. Ta
0
360IFE IFD DFE=
00
180 180IFD DFE= +
, chú ý rng các t giác
ADFI, DFEC ni tiếp nên ta có:
180 ,IFD BAC
−=
0
180 DFE ACB−=
t đó suy ra
IFE BAC ACB=+
0
180 ABC=−
hay
0
180IFE IBE+=
suy ra IFEB là t giác ni tiếp.
Dẫn đến
= BIC FEC BIC FEC
(g.g) (1) (vì có
ICB
chung và
BIC FEC=
).
d. Xét tam giác FEC tam giác DKM ta có:
EFC EDC MDK==
,
00
1
180 180
2
FEC FDC FKC= =
0
180= = MKC DKM FEC DKM
(2). T (1) (2)
suy ra
BIC DKM
BI BC
DK DM
=
(*), ta cũng
0
90BAC DCM==
ABC CDM=
cùng
ph vi
ACB
suy ra
BAC DCM
(g.g) suy ra
BA BC
DC DM
=
(**). T (*) và (**) suy ra
BI BA
DK DC
=
hay
2==
BA DC
BI DK
suy ra I là trung điểm ca AB.
CHUYÊN ĐỀ BỒI DƯỠNG HỌC SINH GIỎI HÌNH HỌC 9
1 | THCS.TOANMATH.com
HH9-CHUYÊN ĐỀ 11.TNG HỢP BÀI THI THƯỜNG GẶP TRONG ĐỀ HSG VÀ
CHUYÊN
Câu 1. Cho t giác
ABCD
ni tiếp
()O
. Gi
E
là giao điểm ca
,AB CD
.
F
là giao đim ca
AC
BD
. Đường tròn ngoi tiếp tam giác
BDE
cắt đường tròn ngoi tiếp tam giác
FDC
ti
điểm
K
khác
D
. Tiếp tuyến ca
()O
ti
,BC
ct nhau ti
M
.
a) Chng minh t giác
BKCM
ni tiếp
b) Chng minh
,,E M F
thng hàng.
Câu 2. Cho đường tròn
()O
đường kính
AB
. Trên tiếp tuyến ti
A
ca
()O
lấy điểm
.C
V cát
tuyến
CDE
(tia
CD
nm gia 2 tia
,CA CO
,
,D E O
,
D
nm gia
,CE
). Gi
M
là giao
điểm ca
CO
BD
,
F
là giao điểm ca
AM
()O
,
)FA
a) V tiếp tuyến
CN
ca
()O
. Chng minh
CNMD
là t giác ni tiếp
b) V
AH OC
ti
H
. Chng minh
ADMH
là t giác ni tiếp.
c) Chng minh
,,E O F
thng hàng.
Câu 3. Cho t giác
ABCD
ni tiếp
()O
()AD BC
. Gi
I
là giao điểm ca
AC
BD
. V
đường kính
,CM DN
. Gi
K
là giao điểm ca
,AN BM
. Đường tròn ngoi tiếp tam giác
IBC
ct
đường tròn ngoi tiếp tam giác
NOC
tại điểm
J
khác
C
.
a) Chng minh
KBNJ
là t giác ni tiếp
d) Chng minh
,,I K O
thng hàng.
Câu 4. Cho tam giác nhn
ABC
()AB AC
. Đường tròn
()I
đường kính
BC
ct
,AB AC
ti
,FE
.
BE
ct
CF
ti
H
.
AH
ct
BC
ti
D
. Chng minh các t giác
,BFHD IFED
ni tiếp.
Câu 5. Cho tam giác nhn
ABC
các đường cao
,,AD BE CF
ct nhau ti
H
. V
HI EF
ti
,I HK DE
ti
K
,
,IK AD M FM DE N
. Gi
S
là điểm đối xng ca
B
qua
D
.
Chng minh t giác
,FIMH HMNK
ni tiếp và
MAN DAS
Câu 6. T điểm
A
nằm ngoài đường tròn
()O
. V hai tiếp tuyến
,AB AC
,BC
là hai tiếp điểm)
và mt cát tuyến
ADE
đến
()O
sao cho (
ADE
nm gia 2 tia
,AO AB
,
,D E O
,Đường
thng qua
D
song song vi
BE
ct
,BC AB
lần lượt ti
,PQ
. Gi
K
là điểm đối xng vi
B
qua
E
. Gi
,HI
là giao điểm ca
BC
vi
,OA DE
a) Chng minh
OEDH
là t giác ni tiếp.
b) Ba điểm
,,A P K
thng hàng.
Câu 7. T điểm
A
nằm ngoài đường tròn
()O
. V hai tiếp tuyến
,AB AC
(
,BC
là hai tiếp điểm).
T điểm
K
nm trên cung
BC
(
,KA
nm cùng phía
BC
) dng tiếp tuyến ct
,AB AC
ti
,MN
.
BC
ct
,OM ON
ti
,PQ
. Gi
I
là giao điểm ca
,MQ NP
. Chng minh
,MBOQ NCOP
là các t giác ni tiếp.
Câu 8. Cho tam giác nhn
ABC
()AB AC
. Đường tròn
()O
đường kính
BC
ct
,AB AC
ti
,ED
.
BD
ct
CE
ti
H
, các tiếp tuyến ca
()O
ti
,BD
ct nhau ti
,,K AK BC M MH BK N
. V tiếp tuyến
AS
ca
()O
vi
(S
thuc cung nh
)CD
,
KD AH I
,
MH OA L
. Đường tròn ngoi tiếp tam giác
ABC
ct
AK
ti
T
.
a) Chng minh các t giác
,TKDB BELO
ni tiếp
b) Ba điểm
,,N E I
thng hàng.
CHUYÊN ĐỀ BỒI DƯỠNG HC SINH GII HÌNH HC 9
THCS.TOANMATH.com | 2
c) Ba điểm
,,M E D
thng hàng.
d) Ba điểm
,,M S H
thng hàng.
Câu 9. Cho tam giác nhn
ABC
ni tiếp đường tròn
()O
có hai đường cao
,BE CD
ct nhau ti
H
. Gi
M
là trung điểm ca
BC
. Gi s
()O
cắt đường tròn ngoi tiếp tam giác
AED
ti
N
.
a) Chng minh
,,N H M
thng hàng.
b) Gi s
AN
ct
BC
ti
K
. Chng minh
,,K E D
thng hàng.
Câu 10. Cho tam giác
ABC
ngoi tiếp
()O
. Gi
,QR
là tiếp điểm ca
()O
vi
,AB AC
.
Gi
,MN
lần lượt là trung điểm ca
,BC CA
. Đường thng
BO
ct
MN
ti
P
.
a) Chng minh
ORPC
là t giác ni tiếp
b) Ba điểm
,,P Q R
thng hàng.
Câu 11. Cho tam giác
ABC
có ba đường cao
,,AD BE CF
ct nhau ti
H
. T
A
ta dng
các tiếp tuyến
,AM AN
đến đường tròn đường kính
BC
.
a) Chng minh các t giác
,AMDN MNDO
ni tiếp
b) Chứng minh ba điểm
,,H M N
thng hàng.
Câu 12. Cho tam giác nhn
ABC
có các đường cao
,,AD BE CF
ct nhau tại điểm
H
. Gi
,MN
là trung điểm ca
,AH BC
. Các phân giác ca góc
,ABH ACH
ct nhau ti
P
.
a) Chứng minh 5 điểm
, , , ,B C E P F
nm trên một đường tròn. Điểm
P
là trung điểm cung
nh
EF
.
b) Ba điểm
,,M N P
thng hàng.
Câu 13. Cho tam giác nhn
ABC
có các đường cao
,,AD BE CF
ct nhau tại điểm
H
.Đường thng
EF
ct nhau tại điểm
M
. Gi
O
là trung điểm
BC
. Gi s các đường tròn ngoi
tiếp các tam giác
,OBF OCE
ct nhau ti giao điểm th 2 là
P
.
a) Chng minh các t giác
,EFPH
,BCHP MEPB
là t giác ni tiếp.
b) Chng minh
OPM
là tam giác vuông.
Câu 14. Cho tam giác nhn
ABC
có trc tâm là điểm
H
. Gi
,MN
là chân các đường cao
h t
,BC
ca tam giác
ABC
.Gi
D
là điểm trên cnh
BC
. Gi
( )
1
w
là đường tròn đi qua các
điểm
,,B N D
gi
( )
2
w
là đường tròn đi qua các điểm
,,C D M
.
,DP DQ
lần lượt là đường kính
ca
( ) ( )
12
,ww
. Chng minh
,,P Q H
thng hàng.
( )
2013IMO
Câu 15. Cho tam giác
ABC
BAC
là góc ln nhất. Các điểm
,PQ
thuc cnh
BC
sao
cho
,QAB BCA CAP ABC==
. Gi
,MN
lần lượt là các điếm đối xng ca
A
qua
,PQ
. Chng
minh rng:
,BN CM
cắt nhau trên đường tròn ngoi tiếp tam giác
ABC
.
( 2014)IMO
Câu 16. Cho tam giác
ABC
ni tiếp đường tròn
()O
. Ly một điểm
P
trên cung
BC
không
chứa điểm
A
ca
()O
. Gi
( )
K
là đường tròn đi qua
,AP
tiếp xúc vi
AC
.
()K
ct
PC
ti
S
khác
P
. Gi
( )
L
là đường tròn qua
,AP
đồng thi tiếp xúc vi
AB
.
()L
ct
PB
ti
T
khác
P
.Gi
D
là điểm đối xng vi
A
qua
BC
.
a) Chng minh
BD
là tiếp tuyến của đường tròn ngoi tiếp tam giác
DPC
.
b) Ba điểm
,,S D T
thng hàng.
CHUYÊN ĐỀ BỒI DƯỠNG HỌC SINH GIỎI HÌNH HỌC 9
3 | THCS.TOANMATH.com
Câu 17. Cho tam giác
ABC
, trên hai cnh
,AB AC
lần lượt lấy hai điểm
,ED
sao cho
ABD ACE=
. Đường tròn ngoi tiếp tam giác
ABD
ct tia
CE
ti
,MN
.Gi
H
là giao điểm ca
,BD CE
. Đường tròn ngoi tiếp tam giác
AEC
ct tia
BD
ti
,IK
a) Chứng minh 4 điểm
, , ,M I N K
cùng nm trên một đường tròn.
b) Gi
F
là giao điểm th 2 của các đường tròn
( )
,( )ABD AEC
. Chng minh
,,A H F
thng
hàng.
c) Chng minh : Tam giác
AMN
cân ti
A
.
Câu 18. Cho tam giác
ABC
( ),( ),( )
a
O I I
theo th t là tâm đường tròn ngoi tiếp, đường
tròn ni tiếp và đường tròn bàng tiếp đối diện đỉnh
A
ca tam giác. Gi
D
là tiếp điểm ca
()I
vi
;BC P
điểm chính gia cung
BAC
ca
()O
,
a
PI
ct
( )
O
tại điểm
K
. Gi
M
là giao điểm ca
PO
BC
a) Chng minh:
a
IBI C
là t giác ni tiếp
b) Chng minh
a
NI
là tiếp tuyến của đường tròn ngoi tiếp tam giác
a
I MP
c) Chng minh:
a
DAI KAI=
.
Câu 19. Cho đường tròn tâm
( )
O
bán kính
R
và mt dây cung
BC
c định có độ dài
3BC R=
. Điểm
A
thay đổi trên cung ln
BC
. Gi
,EF
là điểm đối xng ca
,BC
lần lượt qua
,AC AB
. Các đường tròn ngoi tiếp tam giác
,ABE ACF
ct nhau tại giao điểm th 2 là
K
.
a) Chứng minh điểm
K
luôn thuc một đường tròn c định
b) Xác định v trí điểm
K
để tam giác
KBC
có din tích ln nht và tìm giá tr ln nhất đó
theo
R
c) Gi
H
là giao điểm ca
,BE CF
. Chng minh tam giác
ABH AKC#
và đường thng
AK
luôn đi qua điểm c định.
Câu 20. T điểm
A
nằm ngoài đường tròn
()O
. V hai tiếp tuyến
,AB AC
,BC
là hai tiếp
điểm) và mt cát tuyến
ADE
đến
()O
sao cho (
ADE
nm gia 2 tia
,AO AB
,
,D E O
, Gi
F
là điểm đối xng ca
D
qua
AO
,
H
là giao điểm ca
,EF BC
. Chng minh:
,,A O H
thng
hàng.
Câu 21. T điểm
A
nằm ngoài đường tròn
()O
. V hai tiếp tuyến
,AB AC
,BC
là hai tiếp
điểm) và mt cát tuyến
AEF
đến
()O
sao cho (
AEF
nm gia 2 tia
,AO AB
,
,F E O
BAF FAC
) V đường thng qua
E
vuông góc vi
OB
ct
BC
ti
M
ct
BF
ti
N
. V
OK EF
.
a) Chng minh:
EMKC
ni tiếp
b) Chứng minh đường thng
FM
đi qua trung điểm ca
AB
Câu 22. Cho tam giác nhn
ABC
ni tiếp
()O
.Các đường cao
,,AD BE CF
ct nhau ti
H
.
Tiếp tuyến ti
,BC
ca
()O
ct nhau ti
G
.
GD EF S
. Gi
M
là trung điểm cnh
BC
. Gi
s
,EF BC T AT O K
a) Chứng minh 5 điểm
, , , ,A K F E H
cùng nm trên một đường tròn
b) Chng minh
,,M S H
thng hàng.
CHUYÊN ĐỀ BỒI DƯỠNG HC SINH GII HÌNH HC 9
THCS.TOANMATH.com | 4
Câu 23. Cho
()O
()d
không giao nhau. V
()OH d
lấy hai điểm
,AB
thuc
()d
sao
cho
HA HB
. Lấy điểm
M
thuộc đường tròn
()O
. Dng các cát tuyến qua
,,H A B
và điểm
M
cắt đường tròn
()O
lần lượt ti
,,C D E
,
DE d S
. Dựng đường thng qua
O CE
ct tiếp
tuyến ti
E
ca
()O
K
.Dng
ON DE
ti
N
.
a) Chng minh t giác
HNCS
là t giác ni tiếp
b) Ba điểm
,,S C K
thng hàng
Câu 24. Cho tam giác
ABC
có đường tròn ni tiếp là
()O
tiếp xúc vi ba cnh
,,BC AC AB
lần lượt ti
,,D E F
. Trên đoạn
OD
lấy điểm
I
và dựng đường tròn tâm
I
bán kính
ID
. Dng
,BG CH
là các tiếp tuyến ca
()I
ti
,GH
. Gi
M BG CH
,
N EF BC
a) Chng minh
EHGF
ni tiếp
b) Ba điểm
,,N G H
thng hàng.
Câu 25. Cho 3 đường tròn
12
( ),( ),( )O O O
biết
12
( ),( )OO
tiếp xúc ngoài vi nhau tại điểm
I
12
( ),( )OO
lần lượt tiếp xúc trong vi
()O
ti
12
,MM
. Tiếp tuyến ca
1
()O
ti
I
ct
()O
ln
t ti
,'AA
. Đường thng
1
AM
ct
1
()O
tại điểm
1
N
, đường thng
2
AM
ct
2
()O
tại điểm
2
N
.
a) Chng minh t giác
1 1 2 2
M N N M
ni tiếp và
21
OA N N
b) K đường kính
PQ
ca
()O
sao cho
PQ AI
( Điểm
P
nm trên cung
1
AM
không cha
điểm
2
M
). Chng minh rng nếu
12
,PM PM
không song song thì các đường thng
12
,,AI PM QM
đồng quy.
Câu 26. Cho tam giác
ABC
không cân. Đường tròn
()O
ni tiếp tam giác tiếp xúc vi các
cnh
,,BC CA AB
lần lượt ti
,,M N P
. Đường thng
NP
ct
,BO CO
lần lượt ti
,EF
a) Chng minh các góc
,OEN OCA
bng nhau hoc bù nhau.
b) Chng minh
4
điểm
, , ,B C E F
cùng nm trên một đường tròn.Chng minh
,,O M K
thng
hàng. Biết
K
là tâm đường tròn ngoi tiếp tam giác
OEF
.
Câu 27. . Cho tam giác
ABC
ni tiếp đường tròn
O
. K
AH BC H BC
BE
vuông góc với đường kiính
AD E AD
.
a) Chng minh
//HE DC
.
b) Qua trung điểm
K
của đoạn thng
AB
k đường thng song song vi
AC
ct
BC
ti
M
.
Chng minh
MHE
cân.
Câu 28. Cho tam giác nhn
ABC AB AC
. V đường cao
AD
và đường phân giác
trong
AO
ca tam giác
ABC
(
,DO
thuc
BC
). V đưng tròn tâm
O
tiếp xúc vi
,AB AC
ln
t ti
,MN
.
a) Chứng minh các đim
, , , ,M N O D A
cùng thuc một đường tròn.
b) Chng minh
BDM CDN
.
CHUYÊN ĐỀ BỒI DƯỠNG HỌC SINH GIỎI HÌNH HỌC 9
5 | THCS.TOANMATH.com
c) Qua
O
k đường thng vuông góc vi
BC
ct
MN
ti
I
. Đường thng
AI
ct
BC
ti
K
. Chng minh
K
là trung điểm cnh
BC
.
Câu 29. Cho nửa đường tròn
O
đường kính
2AB R
,CD
là hai điểm di động trên
nửa đường tròn sao cho
C
thuc cung
AD
0
60COD
(
C
khác
A
D
khác
B
). Gi
M
giao điểm ca tia
AC
BD
,
N
là giao điểm ca dây
AD
BC
.
a) Chng minh t giác
CMDN
ni tiếp đường tròn và tính khong cách t
,AB
đến đường
thng
CD
.
b) Gi
H
I
lần lượt là trung điểm
CD
MN
. Chng minh
,,H I O
thng hàng và
3
3
R
DI
.
c) Tìm giá tr ln nht ca din tích tam giác
MCD
theo
R
.
Câu 30. Cho nửa đường tròn
;OR
đường kính
AB
. Gi s
M
là điểm chuyển động trên
nửa đường tròn này, k
MH
vuông góc vi
AB
ti
H
. T
O
k đường thng song song vi
MA
ct tiếp tuyến ti
B
vi nửa đường tròn
O
K
.
a) Chng minh bốn điểm
, , ,O B K M
cùng thuc một đường tròn.
b) Gi s
,CD
là hình chiếu ca
H
trên đường thng
MA
MB
. Chứng minh ba đường
thng
,,CD MH AK
đồng quy.
c) Gi
,EF
lần lượt là trung điểm ca
AH
BH
. Xác định v trí
M
để din tích t giác
CDFE
đạt giá tr ln nht.
Câu 31. Cho hình vuông
ABCD
, trên đường chéo
BD
lấy điểm
I
sao cho
BI BA
.
Đưng thẳng đi qua
I
vuông góc vi
BD
ct
AD
ti
E
,
AI
ct
BE
ti
H
.
a) Chng minh rng
AE ID
.
b) Đưng tròn tâm
E
bán kính
EA
ct
AD
tại điểm th hai
F
. Chng minh rng:
..DF DA EH EB
.
Câu 32. Cho đường tròn
;OR
và một điểm
M
nằm ngoài đường tròn. Đường tròn đường
kính
OM
cắt đường tròn
;OR
tại hai điểm
,EF
.
a) Chứng minh giao điểm
I
của đoạn thng
OM
với đường tròn
;OR
là tâm của đường tròn
ni tiếp tam giác
MEF
.
b) Cho
A
là một điểm bt k thuc cung
EF
chứa điểm
M
của đường tròn đường kính
OM
(
A
khác
E
F
). Đoạn thng
OA
cắt đoạn thng
EF
ti
B
. Chng minh
2
.OAOB R
.
CHUYÊN ĐỀ BỒI DƯỠNG HC SINH GII HÌNH HC 9
THCS.TOANMATH.com | 6
c) Cho biết
2OM R
N
là điểm bt k thuc cung
EF
chứa điểm
I
của đường tròn
;OR
(
N
khác
E
F
). Gi
d
là đường thng qua
F
và vuông góc với đường thng
EN
ti
điểm
P
,
d
cắt đường tròn đường kính
OM
tại điểm
K
(
K
khác
F
). Hai đường thng
FN
KE
ct nhau tại điểm
Q
. Chng minh rng:
2
3
..
2
PN PK QN QK R
.
Câu 33. Cho tam giác
ABC
cân ti
A
ni tiếp đường tròn
O
. Gi
P
là điểm chính gia
ca cung nh
AC
. Hai đường thng
AP
BC
ct nhau ti
M
. Chng minh rng:
a)
ABP AMB
.
b)
..MAMP BABM
.
Câu 34. Cho hai đường tròn
;OR
'; 'OR
ct nhau ti
I
J
'RR
. K các tiếp
tuyến chung của hai đường tròn đó chúng cắt nhau
A
. Gi
B
C
là các tiếp điểm ca hai tiếp
tuyến trên vi
'; ' ,O R D
là tiếp điểm ca tiếp tuyến
AB
vi
;OR
(điểm
I
và điểm
B
cùng
na mt phng b
'OA
). Đường thng
AI
ct
'; 'OR
ti
M
(điểm
M
khác điểm
I
).
a) Gi
K
là giao điểm của đường thng
IJ
vi
BD
. Chng minh
2
.KB KI KJ
, t đó suy
ra
KB KD
.
b)
'AO
ct
BC
ti
H
. Chng minh bốn điểm
, , ',I H O M
nm trên một đường tròn.
c) Chứng minh đường thng
AM
là tiếp tuyến của đường tròn ngoi tiếp
IBD
.
Câu 35. Cho nửa đường tròn tâm
O
đường kính
AB
, trên nửa đường tròn lấy điểm
C
(cung
BC
nh hơn cung
AB
), qua
C
dng tiếp tuyến với đường tròn tâm
O
ct
AB
ti
D
. K
CH
vuông góc vi
AB
H AB
, k
BK
vuông góc vi
CD K CD
;
CH
ct
BK
ti
E
.
a) Chng minh
CB
là phân giác ca
DCE
.
b) Chng minh
BK BD EC
.
c) Chng minh
..BH AD AH BD
.
Câu 36. Cho tam giác nhn
ABC
ni tiếp đường tròn
O
. Cho
P
là điểm bt k trên đoạn
BC
sao cho đường tròn ngoi tiếp tam giác
OBP
cắt đoạn
AB
ti
N
khác
B
và đường tròn
ngoi tiếp tam giác
OCP
cắt đoạn
AC
ti
M
khác
C
.
a) Chng minh rng
OPM OAC
.
b) Chng minh rng
MPN BAC
0
90OBC BAC
.
c) Chng minh rng
O
là trc tâm tam giác
PMN
.
CHUYÊN ĐỀ BỒI DƯỠNG HỌC SINH GIỎI HÌNH HỌC 9
7 | THCS.TOANMATH.com
Câu 37. Trên nửa đường tròn
O
đường kính
2AB R
(
R
là độ dài cho trước) ly hai
điểm
,MN
(
,MN
khác
,AB
) sao cho
M
thuc
AN
và tng các khong cách t
,AB
đến đường
thng
MN
bng
3R
.
a) Tính độ dài đoạn thng
MN
theo
R
.
b) Gi
I
là giao điểm ca
AN
BM
,
K
là giao điểm ca
AM
BN
. Chng minh bn
điểm
, , ,M N I K
cùng nm trên một đường tròn. Tính bán kính của đường tròn đó theo
R
.
c) Tìm GTLN ca din tích tam giác
KAB
theo
R
khi
,MN
thay đổi trên nửa đường tròn
O
nhưng vẫn tha mãn gi thiết bài toán.
Câu 38. Cho hai đường tròn
O
'O
ct nhau tại hai điểm
A
B
. V đường thng
d
qua
A
ct
O
ti
C
và ct
'O
ti
D
sao cho
A
nm gia
C
D
. Tiếp tuyến ca
O
ti
C
và tiếp tuyến ca
'O
ti
D
ct nhau ti
E
.
a) Chng minh rng t giác
BDEC
ni tiếp
b) Chng minh rng
. . .BE DC CB ED BDCE
.
Câu 39. Cho đường tròn
;OR
có đường kính
AB
c định và đường kính
CD
thay đổi sao
cho
CD
không vuông góc cũng không trùng với
AB
. Gi
d
là tiếp tuyến ti
A
ca
;OR
. Các
đường thng
BC
BD
ct
d
tương ứng ti
E
F
.
a) Chng minh rng
CDEF
là t giác ni tiếp.
b) Gi
M
là trung điểm ca
EF
, chng minh rng
BM CD
.
c) Gi
K
là tâm đường tròn ngoi tiếp t giác
CDEF
. Chng minh rng
MK R
.
d) Gi
H
là trc tâm ca tam giác
DEF
, chng minh rng
H
luôn chy trên một đường tròn
c định.
Câu 40. Cho tam giác
ABC
vuông
A
, đường cao
AH
. V đường tròn tâm
O
, đường
kính
AH
, đường tròn này ct các cnh
,AB AC
theo th t ti
D
E
.
a) Chng minh t giác
BDEC
là t giác ni tiếp được đường tròn.
b) Chứng minh ba điểm
,,D O E
thng hàng.
c) Cho biết
3 , 5AB cm BC cm
. Tính din tích t giác
BDEC
.
Câu 41. Cho tam giác
ABC
không là tam giác cân, biết tam giác
ABC
ngoi tiếp đường
tròn
I
. Gi
,,D E F
lần lượt là các tiếp điểm ca
,,BC CA AB
với đường tròn
I
. Gi
M
CHUYÊN ĐỀ BỒI DƯỠNG HC SINH GII HÌNH HC 9
THCS.TOANMATH.com | 8
giao điểm của đường thng
EF
và đường thng
BC
, biết
AD
cắt đường tròn
I
tại điểm
N
(
N
không trùng vi
D
), gi
K
là giao điểm ca
AI
EF
.
a) Chng minh rằng các điểm
, , ,I D N K
cùng thuc một đường tròn.
b) Chng minh
MN
là tiếp tuyến của đường tròn
I
.
Câu 42. T một điểm
P
nằm ngoài đường tròn
O
k hai tiếp tuyến
,PM PN
tới đường
tròn
O
, (
,MN
là hai tiếp điểm). Gi
I
là một điểm thuc cung nh
MN
của đường tròn
O
,
(
I
khác điểm chính gia ca
MN
). Kéo dài
PI
ct
MN
tại điểm
K
, cắt đường tròn
O
tại điểm
th hai là
J
. Qua điểm
O
k đường thng vuông góc vi
PJ
tại điểm
F
và cắt đường thng
MN
tại điểm
Q
. Gi
E
là giao điểm ca
PO
MN
.
a) Chng minh rng
..PI PJ PK PF
.
b) Chứng minh năm điểm
, , ,Q I E O J
cùng thuc một đường tròn.
Câu 43. Cho đường tròn
O
có đường kính
AB
c định,
M
là một điểm thuc
O
(
M
khác
,AB
). Các tiếp tuyến ca
O
ti
A
M
ct nhau
C
. Đường tròn
I
đi qua
M
và tiếp
xúc với đường thng
AC
ti
C
.
CD
là đường kính ca
I
. Chng minh rng:
a) Ba điểm
,,O M D
thng hàng.
b) Tam giác
COD
là tam giác cân.
c) Đưng thẳng đi qua
D
và vuông góc vi
BC
luôn đi qua một điểm c định khi
M
di
động trên đường tròn
O
.
Câu 44. Cho tam giác
ABC
nhn ni tiếp đường tròn tâm
O
, đường cao
BE
CF
. Tiếp
tuyến ti
B
C
ct nhau ti
S
,
BC
OS
ct nhau ti
M
.
a) Chng minh rng
..AB MB AE BS
.
b) Hai tam giác
AEM
ABS
đồng dng.
c) Gi
AM
ct
EF
ti
N
,
AS
ct
BC
ti
P
. Chng minh rng
NP BC
.
Câu 45. Cho tam giác
ABC
vuông ti
A
AB AC
ngoi tiếp đường tròn tâm
O
. Gi
,,D E F
lần lượt là tiếp điểm ca
O
vi các cnh
,,AB AC BC
;
BO
ct
EF
ti
I
.
M
là điểm
di chuyển trên đoạn
CE
.
a) Tính
BIF
.
CHUYÊN ĐỀ BỒI DƯỠNG HỌC SINH GIỎI HÌNH HỌC 9
9 | THCS.TOANMATH.com
b) Gi
H
là giao điểm ca
BM
EF
. Chng minh rng nếu
AM AB
thì t giác
ABHI
ni tiếp.
c) Gi
N
là giao điểm ca
BM
vi cung nh
EF
ca
O
,
P
Q
lần lượt là hình chiếu
ca
N
trên các đường thng
,DE DF
. Xác định v trí của điểm
M
để
PQ
ln nht.
Câu 46. Cho tam giác nhn
ABC
ni tiếp đường tròn
O
. Gi s
M
là điểm thuộc đoạn
thng
AB
(
M
không trùng
,AB
),
N
là điểm thuc tia
CA
(
N
nằm trên đường thng
CA
sao
cho
C
nm gia
A
N
) sao cho khi
MN
ct
BC
ti
I
thì
I
là trung điểm ca
MN
. Đường
tròn ngoi tiếp tam giác
AMN
ct
O
tại điểm
P
khác
A
.
a) Chng minh rng các t giác
BMIP
CNPI
ni tiếp.
b) Gi s
PB PC
, chng minh rng tam giác
ABC
cân.
Câu 47. Cho
ABC
0
60A
. Đường tròn tâm
I
ni tiếp tam giác
ABC
tiếp xúc vi
cnh
,,BC CA AB
lần lượt ti
,,D E F
. Đường thng
ID
ct
EF
ti
K
, đường thng qua
K
song song vi
BC
ct
,AB AC
theo th t ti
,MN
.
a) Chng minh rng các t giác
IFMK
IMAN
ni tiếp.
b) Gi
J
là trung điểm cnh
BC
. Chứng minh ba điểm
,,A K J
thng hàng.
c) Gi
r
là bán kính của đường tròn
I
S
là din tích t giác
IEAF
. Tính
S
theo
r
.
Chng minh
4
IMN
S
S
(
IMN
S
là din tích
IMN
).
Câu 48. Cho hình vuông
ABCD
ni tiếp đường tròn
;OR
. Trên cung nh
AD
lấy điểm
E
(
E
không trùng vi
A
D
). Tia
EB
cắt các đường thng
,AD AC
lần lượt ti
I
K
. Tia
EC
cắt các đường thng
,DA DB
lần lượt ti
,MN
. Hai đường thng
,AN DK
ct nhau ti
P
.
a) Chng minh rng t giác
EPND
là t giác ni tiếp.
b) Chng minh rng
EKM DKM
.
c) Khi điểm
M
v trí trung điểm ca
AD
. Hãy xác định độ dài đoạn
AE
theo
R
.
Câu 49. Cho tam giác
ABC
. Trên phân giác
AD
có hai điểm
,MN
sao cho
ABN CBM
. Chng minh rng
ACN BCM
.
Câu 50. Cho hình thoi
ABCD
0
60BAD
. Một đường thng thay đổi qua
C
ct
,AB AD
lần lượt ti
,NM
. Gi
P
là giao điểm ca
BM
DN
. Chng minh rng
P
thuc mt
đường tròn c định.
CHUYÊN ĐỀ BỒI DƯỠNG HC SINH GII HÌNH HC 9
THCS.TOANMATH.com | 10
Câu 51. Cho tam giác
ABC
vuông ti
A
.
AB AC
. Gi
D
là một điểm trên cnh
BC
,
E
là một điểm trên cnh
BA
kéo dài v phía
A
sao cho
BD BE CA
. Gi
C
là một điểm
trên
AC
sao cho
, , ,E B D P
thuc cùng một đường tròn,
Q
là giao điểm th hai ca
BP
vi
đường tròn ngoi tiếp tam giác
ABC
. Chng minh rng
AQ CQ BP
.
Câu 52. Cho tam giác
ABC
A B C
ni tiếp trong đường tròn
O
, ngoi tiếp
đường tròn
I
. Cung nh
BC
M
là điểm chính gia.
N
là trung điểm cnh
BC
. Điểm
E
đối
xng vi
I
qua
N
. Đường thng
ME
cắt đường tròn
O
tại điểm th hai
Q
. Lấy điểm
K
thuc
BQ
sao cho
QK QA
. Chng minh rng:
a) Đim
Q
thuc cung nh
AC
của đường tròn
O
.
b) T giác
AIKB
ni tiếp và
BQ AQ CQ
.
Câu 53. Cho
O
là một điểm nm trong tam giác
ABC
. Gi
', ', 'A B C
lần lượt là các điểm
đối xng ca
,,A B C
qua
O
. Chng minh rằng các đường tròn ngoi tiếp ca các tam giác
' ' ', ' , ' ,A B C A BC B CA
'C AB
có điểm chung.
Câu 54. Cho tam giác
ABC
ni tiếp đường tròn
O
. Hai phân giác
BM
CN
ca góc
B
C
. Tia
MN
ct
O
ti
P
. Gi
,,X Y Z
lần lượt là hình chiếu vuông góc ca
P
xung
,,BC CA AB
. Chng minh rng:
a)
PY PX PZ
.
b)
1 1 1
PB PA PC
.
Câu 55. Cho tam giác nhn
ABC AB AC
. Đường tròn đường kính
BC
ct các cnh
,AB AC
tương ứng ti
,MN
. Gi
O
là trung điểm ca
BC
. Đường phân giác ca
BAC
MON
ct nhau ti
R
. Chng minh rằng đường tròn ngoai tiếp tam giác
BMR
CNR
cùng đi
qua một điểm nm trên cnh
BC
.
Câu 56. Cho t giác
ABCD
có đường chéo
BD
không là phân giác ca các góc
ABC
CDA
. Một điểm
P
nm trong t giác sao cho:
;PBC DBA PDC BDA
. Chng minh rng t
giác
ABCD
ni tiếp khi và ch khi
AP CP
.
Câu 57. Ba tia
,,Ix Iy Iz
chung gc
I
. Ly cặp điểm
,'AA
trên
Ix
, ly cặp điểm
,'BB
trên
Iy
, ly cặp điểm
,'CC
trên
Iz
theo th t đó kể t
I
sao cho
. ' . ' . 'IAIA IB IB IC IC
. Chng
minh rằng tâm các đường tròn ngoi tiếp các tam giác
, ' ' 'ABC A B C
I
thng hàng.
Câu 58. Cho
BC
là một dây cung khác đường kính của đường tròn
O
. Đim
A
thay đổi
trên cung ln
BC
. Đường tròn bàng tiếp góc
A
ca tam giác
ABC
tiếp xúc vi cnh
,,BC CA AB
lần lượt ti
,,M N P
.
a) Tìm v trí ca
A
để chu vi tam giác
MNP
đạt giá tr ln nht.
b) Chng minh rằng đường thẳng Ơ-le ca tam giác
MNP
luôn đi qua một điểm c định.
Câu 59. Cho hai đường tròn
11
;Or
22
;Or
tiếp xúc ngoài vi nhau. Một đường tròn
O
thay đổi tiếp xúc ngoài vi
1
O
2
O
. Gi s
AB
là một đường kính ca
O
sao cho
12
AOO B
CHUYÊN ĐỀ BỒI DƯỠNG HỌC SINH GIỎI HÌNH HỌC 9
11 | THCS.TOANMATH.com
là mt hình thang
12
//AB O O
. Gi
I
là giao điểm ca
2
AO
vi
1
BO
. Chng minh rng
I
thuc một đường thng c định.
Câu 60. Cho tam giác
ABC
I
là tâm đường tròn ni tiếp,
O
là tâm đường tròn ngoi
tiếp và trng tâm
G
. Gi s rng
0
90OIA
. Chng minh rng
IG
BC
song song.
Câu 61. Cho hình ch nht
ABCD
và bốn đường tròn
1 2 3 4
; , ; , ; , ;A R B R C R D R
sao
cho
1 3 2 4
R R R R AC
. Gi
13
,
là hai tiếp tuyến chung ngoài ca
1
;AR
3
;CR
;
13
,
là hai tiếp tuyến chung ngoài ca
2
;BR
4
;DR
. Chng minh rng tn ti một đường
tròn tiếp xúc vi c bn đường thng
1 2 3 4
, , ,
.
Câu 62. Cho t giác
ABCD
có hai đường chéo
AC
BD
vuông góc vi nhau ti
S
. Gi
, , ,M N P Q
lần lượt đối xng vi
S
qua
, , ,AB BC CD DA
. Đường tròn ngoi tiếp tam giác
SPQ
ct ti
AP
ti
S
. Chng minh rng bốn điểm
, , ,M E F Q
cùng thuc một đường tròn.
Câu 63. Cho tam giác
ABC
cân ti
A
, trên cnh
BC
ly
D
sao cho
: 2 : 1BD DC
trên đoạn
AD
ly
P
sao cho
BAC BPD
. Chng minh rng
1
2
DPC BAC
.
Câu 64. Cho t giác
ABCD
ni tiếp. Gi
,,P Q R
lần lượt là các chân đường vuông góc ca
D
xung
,,BC CA AB
. Chng t rng
PQ QR
khi và ch khi phân giác các góc
ABC
ADC
ct nhau trên
AC
.
Câu 65. Trong mt phẳng cho hai đường tròn
1
O
2
O
ct nhau hai điểm
A
B
.
Các tiếp tuyến ti
A
B
ca
1
O
ct nhau điểm
K
. Gi s
M
là một điểm nm trên
1
O
nhưng không trùng vào
A
B
. Đường thng
AM
ct
2
O
điểm th hai
P
, đường thng
KM
ct
1
O
điểm th hai
C
và đường thng
AC
ct
2
O
điểm th hai
Q
. Chng minh rng
trung điểm ca
PQ
nằm trên đường thng
MC
.
Câu 66. Cho tam giác
ABC
ni tiếp đường tròn
O
. Đường tròn
'O
nm trong
O
tiếp
xúc vi
O
ti
T
thuc cung
AC
(cung không cha
B
). K các tiếp tuyến
', ', 'AA BB CC
ti
'O
. Chng minh rng
'. '. '.BB AC AA BC CC AB
.
Câu 67. Cho hai đường tròn
1
O
2
O
cùng tiếp xúc với đường tròn
O
. Tiếp tuyến
chung ca
1
O
2
O
ct
O
ti bốn điểm. Gi
,BC
là hai trong bốn điểm đó sao cho
,BC
nm v cùng một phía đối vi
12
OO
. Chng minh rng
BC
song song vi mt tiếp tuyến chung
ngoài ca
1
O
2
O
.
Câu 68. Cho t giác
ABCD
ni tiếp đường tròn
O
. Chng minh rng
..
..
AC BC CD AB BD
BD BC BA DC DA
.
Câu 69. Cho tam giác
ABC
cân
A
. Kí hiu
,,x y z
lần lượt là khong cách
', ', 'MA MB MC
t một điểm
M
nm trong tam giác tới các đường thng
,,BC CA AB
. Gi s
2
x yz
, chng minh rng
M
thuc một đường tròn c định.
CHUYÊN ĐỀ BỒI DƯỠNG HC SINH GII HÌNH HC 9
THCS.TOANMATH.com | 12
Câu 70. Cho tam giác nhn
ABC
. Điểm
O
thay đổi trên
BC
. Đường tròn tâm
O
bán kính
OA
ct
,AB AC
lần lượt tại các điểm th hai
,MN
. Chng minh rng trc tâm ca tam giác
AMN
thuc một đường thng c định.
Câu 71. Cho t giác
ABCD
ni tiếp đường tròn tâm
O
. Gi
1 2 3 4
, , ,H H H H
lần lượt là trc
tâm ca các tam giác
, , ,BCD CDA DAB ABC
. Chng minh bốn điểm
1 2 3 4
, , ,H H H H
cùng nm
trên một đường tròn.
Câu 72. Đim
I
nm trong tam giác
ABC
và tha mãn
0
120AIB BIC CIA
.
Chng minh rằng ba đường thẳng Ơ-le ca các tam giác
,ABI BCI
CAI
đồng quy.
Câu 73. Gi
,OI
H
lần lượt là tâm đường tròn ngoi tiếp, ni tiếp và trc tâm ca tam
giác
ABC
. Chng minh rng: Nếu đường tròn ngoi tiếp tam giác
OIH
đi qua một trong các đỉnh
ca tam giác
ABC
thì phải đi qua một đỉnh khác ca tam giác
ABC
.
Câu 74. Cho tam giác
ABC
ni tiếp đường tròn
O
, trc tâm
H
, đường cao
AK
K BC
. Gi s một đường thng qua
K
vuông góc vi
OK
ct
,AB AC
lần lượt ti
,MN
.
Các tia
,MH NH
ct
,AC AB
th t ti
,PQ
. Chng minh rng t giác
APHQ
ni tiếp.
Câu 75. Tam giác
ABC
có trc tâm
H
, đường cao
BE
. Điểm
P
trên đường tròn ngoi
tiếp tam giác
ABC
. V các hình bình hành
PAQB
PARC
. Giao điểm
AQ
HR
X
.
Chng minh rng
EX
song song vi
AP
.
Câu 76. Cho tam giác
ABC
ni tiếp đường tròn tâm
O
. Một đường tròn
1
O
qua
B
C
ct các cnh
,AB AC
lần lượt ti
,DE
. Đường tròn
2
O
qua ba điểm
,,A D E
ct
O
ti
K K A
. Chng minh rng
0
1
90AKO
.
Câu 77. Cho hai đường tròn
O
'O
ct nhau ti
A
B
. Gi s
,CD EF
là hai tiếp
tuyến chung ngoài của hai đường tròn này
, ; , 'C E O D F O
, điểm
A
gn
CD
hơn
B
).
Gi
1
là đường thng qua
A
tiếp xúc với đường tròn ngoi tiếp tam giác
AEF
2
là đường
thng qua
B
tiếp xúc với đường tròn ngoi tiếp tam giác
BCD
. Chng minh rằng các đường thng
12
, , ,CD EF
đồng quy.
Câu 78. Cho hai đường tròn
O
'O
tiếp xúc trong ti
M
(
'O
cha trong
O
). Gi
s
P
N
là hai điểm bt k thuc
'O
. Qua
P
N
k các tiếp tuyến vi
'O
ct
O
ti
,AC
,BD
. Chng minh rằng tâm đường tròn ni tiếp các tam giác
,ACD BCD
nm trên
NP
.
Câu 79. Cho hai đường tròn
1
O
2
O
tiếp xúc ngoài vi nhau ti
I
và cùng tiếp xúc
trong vi
O
. K tiếp tuyến chung ngoài vi
1
O
2
O
ct
O
ti
,BC
. Qua
I
k tiếp tuyến
chung vi
1
O
2
O
ct
O
ti
A
(
A
thuc cùng na mt phng b
BC
vi
12
,OO
.
Chng minh rng
I
là tâm đường tròn ni tiếp tam giác
ABC
.
Câu 80. Cho tam giác
ABC
cân đỉnh
A
. Điểm
M
nm trong tam giác sao cho
0
1
90
2
BMC A
. Qua
M
k đường thng song song vi
BC
ct
,AB AC
lần lượt ti
,XY
.
CHUYÊN ĐỀ BỒI DƯỠNG HỌC SINH GIỎI HÌNH HỌC 9
13 | THCS.TOANMATH.com
V
,MZ MT
lần lượt song song vi
,AB AC
. Gi
N
là giao điểm ca
XZ
YT
. Chng minh
rng t giác
ABNC
là t giác ni tiếp.
Câu 81. Cho tam giác nhn
ABC
AB AC
ni tiếp đường tròn
;OR
, các đường cao
,,AD BE CF
ct nhau ti
H
.
a) Chng minh rng
..AE AC AF AB
.
b) Chng minh rng các t giác
,BFHD ABDE
ni tiếp đường tròn.
c) V tia
Ax
là tia tiếp tuyến của đường tròn
O
, tia
Ax
nm trên na mt phng b
AB
chứa điểm
C
. Chng minh rng
//Ax EF
. T đó suy ra
OA EF
.
d) Gi
K
là giao điểm của hai đường thng
EF
BC
. Đường thẳng đi qua
F
song song
vi
AC
ct
,AK AD
lần lượt ti
,MN
. Chng minh rng
MF NF
.
Câu 82. Cho đường tròn tâm
O
, đường kính
AB
. Ly
C
thuc
O
(
C
không trùng vi
,AB
),
M
là điểm chính gia ca cung nh
AC
. Các đường thng
AM
BC
ct nhau ti
I
,
các đường thng
,AC BM
ct nhau ti
K
.
a) Chng minh
ABM IBM
ABI
cân .
b) Chng minh t giác
MICK
ni tiếp.
c) Đưng thng
BM
ct tiếp tuyến ti
A
ca
O
N
. Chứng minh đường thng
NI
là tiếp
tuyến ca
,B BA
NI MO
.
d) Đưng tròn ngoi tiếp tam giác
BIK
cắt đường tròn
,B BA
ti
D
(
D
không trùng vi
I
). Chng minh
,,A C D
thng hàng.
Câu 83. Cho t giác
ABCD
ni tiếp đường tròn
O
tâm
O
, đường kính
AD
. Hai đường
chéo
AC
BD
ct nhau ti
I
. Gi
H
là hình chiếu ca
I
lên
AD
M
là trung điểm ca
ID
. Đường tròn
HMD
ct
O
ti
N
(
N
khác
D
). Gi
P
là giao điểm ca
BC
HM
.
a) Chng minh rng t giác
BCMH
ni tiếp.
b) Chng minh rằng ba điểm
,,P D N
thng hàng.
Câu 84. Cho đường tròn
O
c định. T một điểm
A
c định bên ngoài đường tròn
O
,
k các tiếp tuyến
AM
AN
với đường tròn (
,MN
là các tiếp điểm). Đường thẳng đi qua
A
ct
đường tròn
O
tại hai điểm
B
C
(
B
nm gia
A
C
). Gi
I
là trung điểm ca dây
BC
.
a) Chng minh rng
AMON
là t giác ni tiếp.
b) Gi
K
là giao điểm ca
MN
BC
. Chng minh rng
..AK AI AB AC
.
c) Khi cát tuyến
ABC
thay đổi thì điểm
I
chuyển động trên cung tròn nào? Vì sao?Xác định
v trí ca cát tuyến
ABC
để
2IM IN
.
Câu 85. Cho tam giác
ABC
nhn
AB AC
, đường cao
AH
. V đường tròn tâm
O
đường kính
AB
ct
AC
ti
N
. Gi
E
là điểm đối xng ca
H
qua
AC
,
EN
ct
AB
ti
M
cắt đường tròn
O
tại điểm th hai
D
.
a) Chng minh
AD AE
.
b) Chng minh
HA
là phân giác ca
MHN
.
CHUYÊN ĐỀ BỒI DƯỠNG HC SINH GII HÌNH HC 9
THCS.TOANMATH.com | 14
c) Chng minh rằng điểm
, , , ,A E C H M
cùng thuc một đường tròn tâm
1
O
. Và ba đường
thng
,,CM BN AH
đồng quy ti một điểm.
d)
DH
cắt đường tròn
1
O
tại điểm th hai
Q
. Gi
,IK
lần lượt là trung điểm ca
DQ
BC
. Chng minh rng
I
thuộc đường tròn ngoi tiếp tam giác
AHK
.
Câu 86. Cho t giác
ABCD
ni tiếp đường tròn đường kính
,2AC AC a
. Gi
,MN
ln
ợt là trung điểm ca
AB
AD
, tam giác
ABD
đều.
a) Tính
BC
CN
theo
a
.
b) Gi
H
là trc tâm ca tam giác
CMN
;
MH
ct
CN
ti
E
,
MN
ct
AC
ti
K
. Chng
minh năm điểm
, , , ,B M K E C
cùng thuc một đường tròn
T
.
c) Đưng tròn
T
ct
BD
ti
F F B
, tính
DF
theo
a
.
d)
KF
ct
ME
ti
I
. Chng minh
KM
tiếp xúc với đường tròn ngoi tiếp tam giác
MIF
.
Tính
IND
.
Câu 87. Cho điểm
M
nằm ngoài đường tròn
;OR
. V hai tiếp tuyến
,MA MB
và cát tuyến
MCD
(
,,,A B C D
thuộc đường tròn
O
), tia
MC
nm gia hai tia
MO
MB
. Gi
H
là giao
điểm ca
MO
AB
.
a) Chng minh rng
2
.MA MC MD
.
b) Chng minh t giác
CHOD
ni tiếp,
MHC DHO
.
c) Chng minh rng
ADH CDB
.
d)
MO
cắt đường tròn
O
ti
,EF
(
E
nm gia
,MO
). Chng minh rằng các đường thng
,DE CF
ct nhau ti một điểm trên đường thng
AB
.
Câu 88. Cho
A
ngoài đường tròn
;OR
. V các tiếp tuyến
,AB AC
vi
O
.
S
là điểm
trên tia đối ca tia
,OA OS R
. Đường thng vuông góc vi (
OA
ti
S
ct
,AB AC
lần lượt ti
,DE
; cắt đường tròn
O
ti
,FT
(
F
nm gia
,DT
).
AF
ct
O
ti
M
.
G
là điểm đối xng
ca
F
qua
D
,
L
là điểm đối xng ca
F
qua
T
. Chng minh rằng hai đường tròn
O
MGL
tiếp xúc nhau.
NG DN
Câu 1) Phân tích và định hướng gii:
a). Để chng minh t giác
BKCM
ni tiếp ta chng minh
0
180BKC BMC+=
. Điểm
K
trong bài toán có mi quan hê vi
hai đường tròn ngoi tiếp các
t giác
,EBKD KFDC
vì vy ta
I
M
K
O
F
E
D
C
B
A
CHUYÊN ĐỀ BỒI DƯỠNG HỌC SINH GIỎI HÌNH HỌC 9
15 | THCS.TOANMATH.com
tìm cách tính các góc
,BKC BMC
theo các góc có liên quan đến 2 t
giác này.
Ta có:
( )
00
360 360BKC BKE CKE BDE CKE

= + = +

( )
0 0 0
360 180 180 2BDC BDC BDC= + =
(1)
Mặt khác ta cũng có:
00
180 2 180 2BMC MBC BDC= =
(2)
T (1) và (2) ta có:
0
180BKC BMC+=
.
b). Thc nghim hình v cho ta thy
,,E K M
thng hàng. Tht vy ta có:
0
180EKB BKM EDB BCM EDB BDC+ = + = + =
. Bây gi ta chng minh:
,,F K M
thng hàng:
Tht vy ta có:
0
180MKC CKF MBC CKF BDC CKF+ = + = + =
. T đó ta suy ra điều phi chng
minh.
Câu 2)
Phân tích định hướng gii:
a). T giác
CNMD
có liên quan
đến tiếp tuyến
CN
nên ta tp trung
khai thác gi thiết v góc to bi
tiếp tuyến và mt dây.
Ta thy:
MCN MCA=
, mt khác
MCA BAN=
cùng ph vi góc
NAC
, nhưng
BAN BDN=
(góc ni tiếp) t đó ta suy ra
MDN MCN=
hay t giác
CNMD
ni tiếp.
b). D thy
0
90ADM =
. T đó suy ra
0
180ADM AHM+=
suy ra đpcm.
H
N
M
F
O
E
D
C
B
A
CHUYÊN ĐỀ BỒI DƯỠNG HC SINH GII HÌNH HC 9
THCS.TOANMATH.com | 16
c). Để chng minh
,,E O F
thng hàng: Ta chng minh:
0
180EOA AOF+=
, điều này cũng tương
đương với vic chng minh:
0
90EAF =
. Tht vy ta có:
EAF EAB BAF=+
, nhưng
EAB EDB=
(Cùng chn cung
)EB
, mt khác
EDB MNC=
do
CMND
ni tiếp, suy ra
EAB MNC MAC==
,T đó suy ra
0
90EAF MAC MAF=+=
. (đpcm).
Câu 3).
Phân tích định hướng gii:
Để chng minh t giác
BNJK
ni tiếp ta s chng minh
BJN BKN=
. Ta có:
( )
00
180 180BJN BJC NJC BIC NOC NOC BIC= = =
Mặt khác ta cũng có:
( )
1
,
2
NOC DM NC=+
( )
1
2
BIC BC AD=+
t đó suy ra:
( )
1
2
BJN DM NC BC AD= +
Ta cũng có:
( ) ( )
11
22
NKB AM NB DM AD BC NC

= =

( )
1
2
DM NC BC AD= +
. T đó suy ra đpcm.
b). Ta có t giác
BNJK
ni tiếp nên
0
180NJK NBK+ =
0 0 0
180 180 180NJK NAM NJK NCM NJK NJO+ = + = + =
hay
,,O J K
thng hàng. Mặt khác ta cũng có:
0
180KJB IJB KNB BCI KNB BNA+ = + = + =
hay
,,K I J
thng hàng. T đó suy ra
,,I K O
thng hàng.
Câu 4) Phân tích định hướng gii:
Ta có:
0
90BEC CFB==
. Suy ra
H
J
K
N
M
O
I
D
C
B
A
D
I
H
C
B
A
F
E
CHUYÊN ĐỀ BỒI DƯỠNG HỌC SINH GIỎI HÌNH HỌC 9
17 | THCS.TOANMATH.com
là trc tâm ca tam giác
ABC
.
Hay
AH BC⊥
0
90BFH HDB==
hay t giác
BFHD
ni tiếp.
Tương tự ta cũng có:
DHEC
ni tiếp.
Ta có:
2FBH FDH HCE HDE FDE FBE FIE= = = = =
tc là
FIDE
là t giác ni tiếp.
Câu 5)
+ Ta có tính cht quen thuc:
BE
là phân giác trong ca góc
FED
. (Hc sinh t chng minh
điều này da vào các t giác
ni tiếp
,,BFHD HIEK HDEC
) .
T đó suy ra
HK HI=
EI EK=
. Do đó
( )
00
1
180 90
2
KIE IEK IEH= =
. Mặt khác ta cũng
000
90 90 90MHF FAH FEH IEH= = =
. Suy ra đpcm.
+ Xét t giác
HMNK
ta có:
0
90HKN =
, mt khác ta va chng minh
FIMH
ni tiếp nên suy ra
00
90 90FMH HIF HMN= = =
. Như vậy
0
180HKN HMN+=
suy ra đpcm.
+ Ta có:
HNM HKM HIM HFM= = =
FHN
cân ti
H MF MN=
. T đó dễ dàng chng
minh được:
MAN DAS=
.
Câu 6)
Phân tích định hướng gii:
a). Áp dng h thức lượng
trong tam giác vuông
ABO
ta có:
2
.AB AH AO=
. Theo tính cht ca tiếp tuyến và cát tuyến ta có:
2
.AB AD AE=
nên
suy ra
..AH AO AD AE=
OHED
ni tiếp.
Ta có th giải thích tường minh hơn như sau:
Áp dng h thức lượng trong tam giác vuông
ABO
ta có:
2
.AB AH AO=
.
D
S
N
M
K
I
H
F
E
C
B
A
x
K
H
I
Q
P
E
D
O
C
B
A
CHUYÊN ĐỀ BỒI DƯỠNG HC SINH GII HÌNH HC 9
THCS.TOANMATH.com | 18
Xét tam giác
ABD
và tam giác
AEB
ta có:
BAD
chung,
ABD BED=
(Tính cht góc to bi tiếp
tuyến và mt dây). T đó suy ra
ABD
đồng dng vi
AEB
nên
2
.
AD AB
AD AE AB
AB AE
= =
.
b). Để gii quyết tt câu hi này ta cn nm chc tính chất liên quan đến cát tuyến và tiếp tuyến.
(Xem thêm phần: ‘’Chùm bài tập liên quan đến cát tuyến và tiếp tuyến’’) đó là:
HI
là phân giác
trong ca góc
DHE
HA
là phân giác ngoài ca góc
DHE
Tht vy ta có:
OHE ODE OED==
mặt khác ta cũng có:
AHD OED=
( Tính cht t giác ni
tiếp). Suy ra
AHD OHE DHB BHE= =
hay
HI
là phân giác ca góc
DHE
do
HA HI
nên
suy ra
HA
là phân giác ngoài ca góc
DHE
.
Quay tr li bài toán:
Ta thy rng : T vic chng minh:
HI
là phân giác trong ca góc
DHE
HA
là phân giác
ngoài ca góc
DHE
ta có:
ID HD
IE HE
=
AD HD
AE HE
=
suy ra
ID AD
IE AE
=
Mặt khác theo định lý Thales ta cũng có:
ID DP
IE BE
=
suy ra
DP AD
BE AE
=
EK BE=
nên
DP AD
EK AE
=
. Điều này chng t
D
là trung điểm ca
PQ
,,A P K
thng hàng.
Câu 7).
Ta thy rng: Nếu t giác
MBOQ
ni tiếp thì
MQB MOB=
Mt khác
MOB MKB=
do t giác
MBOK
ni tiếp suy ra
MQB MKB=
.
Như vậy ta cn quy bài toán v
chng minh
MKQB
ni tiếp.
Ta có:
ABC ACB NKQ==
(Tính cht tiếp tuyến).
Như vậy
MKQB
là t giác ni tiếp. Hoàn toàn tương tự ta cũng có:
NKPC
ni tiếp nên cũng suy
ra được:
NCOP
ni tiếp.
Câu 8).
a). Gi s đường tròn
( ')O
ngoi tiếp
tam giác
ABC
. D thy
H
là trc tâm tam giác
ABC
Q
P
N
M
K
C
B
A
O
J
F
O'
S
L
T
I
N
M
K
H
D
E
O
C
B
A
CHUYÊN ĐỀ BỒI DƯỠNG HỌC SINH GIỎI HÌNH HỌC 9
19 | THCS.TOANMATH.com
O
là trung điểm
BC
.
Những điểm đặc bit này
giúp ta ngh đến bài toán
đặc bit liên quan đến
đường thẳng, đường tròn Ơ le.
K đường kính
AF
ca
( ')O
. Ta d chứng minh được:
BHCF
là hình bình hành và
,,H O F
thng hàng. Ta có:
MTB ACB=
do
BTAC
là t giác ni tiếp.
Mt khác
KDB DBC ACB=
(Tính cht góc to bi tiếp tuyến và mt dây). T đó suy ra
KDB KTB=
tc là t giác
TKBD
ni tiếp.
Để ý rng: T giác
BKDO
ni tiếp, t đó suy ra 5 điểm
, , , ,O B K T D
cùng nm trên một đường
tròn đường kính
OK
hay
0
90OTK =
. Mt khác
0
90FTA=
suy ra
,,F O T
thẳng hàng. Do đó 4
điểm
, , ,F O H T
thng hàng. Tam giác
MAO
,AH OT
là hai đường cao nên suy ra
H
là trc
tâm, do đó
ML AO
nên 5 điểm
, , , ,A E H L D
cùng nm trên một đường tròn. Suy ra
ELA EDA EBC==
tc là t giác
BELO
ni tiếp.
b). Ta có 5 điểm
, , , ,B N E L O
cùng nằm trên đường tròn đường kính
NO
nên
0
90NEO NLO==
,
nhưng
KDB DCB BHJ IHD= = =
suy ra
I
là trung điểm ca
0
90AH IE ID IEO = =
. Như
vy:
0
180IEO NEO+=
nên
,,N E I
thng hàng.
c) Ta có
MTE ADE=
do
TADE
ni tiếp.
ADE ABC ABC MTE MTEB= =
ni tiếp.
MEB MTB=
. Mà
BED BTA=
cùng bù vi
0
180ACB MEB BED MTB BTA + = + =
hay
,,M E D
thng hàng.
d) Vì
OE IE OE⊥
là tiếp tuyến ca đường tròn đi qua các điểm
, , , , ,A E T H D L
tâm
I
. Suy ra
22
..OEL OAE OEL OAE OAOL OE OAOL OS OLS OSA= = = ##
. Mt khác
0 0 0
90 90 180 , ,OSA OLS MLO OLS M L S= = + =
thng hàng. Mà
, , ,H M N L
thng hàng
nên suy ra
,,M H S
thng hàng.
Câu 9) Phân tích định hướng gii toán:
Bài toán này làm ta liên tưởng đến đường thẳng Ơle, đường tròn Ơ le. Dựng đường kính
'AA
.Ta d
thấy 4 điểm
, , ,A E H D
cùng nằm trên đường tròn tâm
I
đường kính
AH
.
Suy ra
HN AN
. Mt
khác t tính cht quen thuc khi chng minh
'BHCA
là hình bình hành ta cũng suy ra
HIOM
hình bình hành do đó
//HM OI
. Ta li có
OI
là đường ni tâm của 2 đường tròn
( ),( )OI
nên
OI AN
(Do
OI
nằm trên đường trung trc ca
AN
). T đó suy ra
MH AN
. Hay
,,M H N
thng hàng.
I
A'
O
N
M
K
H
D
E
C
B
A
CHUYÊN ĐỀ BỒI DƯỠNG HC SINH GII HÌNH HC 9
THCS.TOANMATH.com | 20
*) Để chng minh
,,K E D
thng hàng. Ta chng minh:
0
180KEN NED+=
. Ta tìm cách quy 2
góc này v 2 góc đối nhau trong mt t giác ni tiếp.
+ Ta có:
NEA NHA=
(Cùng chn cung
NA
),
NHA NKB=
cùng ph vi góc
KAH
suy ra
NEA NKB NKBE=
ni tiếp suy ra
NEK NBK=
. Mà
NBK NAD=
(Do
NBCA
ni tiếp).
+ T đó suy ra
0
180KEN NED NAD NED+ = + =
( Điều phi chng minh).
Câu 10) Phân tích định hướng gii:
a). Ta cần dùng các góc để tn
dụng điều kin
,AR AQ
các tiếp tuyến ca
()O
Tht
vy:
0
90ORC =
,
vì vy ta cn chng minh
0
90OPC =
.
Mt khác do
NM
là đường trung bình ca tam giác
ABC
nên
ABP BPM=
nhưng
ABP PBM=
(Tính cht phân giác trong)
T đó suy ra
BMP
cân ti
M MB MP MC BPC = =
vuông ti
0
90P ORC OPC = =
hay
ORPC
là t giác ni tiếp.
b). Để chng minh
,,P Q R
thng hàng ta chng minh:
0
180PRC CRQ+=
.
Tht vy ta có:
PRC POC=
2
BC
POC OBC OCB
+
= + =
,
0
0 0 0
180
180 180 90
22
AA
CRQ ARQ

= = = +



suy ra
00
90 180
22
B C A
PRC CRQ
+
+ = + + =
(Đpcm).
11) Phân tích định hướng gii:
a). Ta có:
0
90AMO ANO ADO= = =
nên 5 điểm
, , , ,A M D O N
ng nm
trên đường tròn đường kính
AO
.
N
M
O
R
Q
P
C
B
A
O
F
E
D
H
N
M
C
B
A
CHUYÊN ĐỀ BỒI DƯỠNG HỌC SINH GIỎI HÌNH HỌC 9
21 | THCS.TOANMATH.com
Suy ra các t giác
,AMDN MNDO
là t giác ni tiếp.
b). Ta có:
BDHF
là t giác ni tiếp
nên:
..AH AD AF AB=
Mt khác
2
.AF AB AM=
nên
2
..AM AH AD AF AB==
. Hay
AM
là tiếp tuyến của đường tròn ngoi tiếp tam giác
MHD
suy ra
AMH ADM=
. Ta cũng có:
AMDN
là t giác ni tiếp nên:
AMN ANM ADM==
t đó ta suy ra
AMH AMN=
hay
,,M H N
thng
hàng.
Câu 12) Phân tích định hướng gii:
a). Ta thấy các điểm
,,,B C E F
nằm trên đường tròn đường kính
BC
. Để chng minh 5 điểm
, , , ,B C E P F
nm trên một đường tròn
Ta cn chng minh
0
90BPC =
. Tht vy ta có:
1
2
PBC PBE EBC ABE EBC= + = +
( )
( )
00
1
90 90
2
AC= +
Tương tự ta cũng có:
PCB PCF FCB=+
.
( ) ( )
00
1
90 90
2
AB= +
. T đó suy ra
( )
0 0 0
270 90 90PBC PCB A B C BPC+ = + + = =
. Vậy điểm
P
thuộc đường tròn đường kính
BC
.Mt khác
BP
là phân giác ca góc
ABH
nên
P
là trung điểm ca cung nh
EF
.
b). Để ý rng
,MN
là tâm của hai đường tròn đường kính
BC
và đường tròn đường kính
AH
Do
hai đường tròn ct nhau theo dây cung
EF
nên
MN
đi qua trung điểm ca cung
EF
. Hay
,,M N P
thng hàng.
Câu 13) Phân tích định hướng gii:
a). Điểm
P
trong bài toán
chính là điểm Miquel ca
tam giác
ABC
.
+ Ta d thấy 4 điểm
S
R
P
N
M
F
E
D
H
C
B
A
P
D
A
B
C
H
E
F
M
O
CHUYÊN ĐỀ BỒI DƯỠNG HC SINH GII HÌNH HC 9
THCS.TOANMATH.com | 22
, , ,A F H E
cùng nm
trên đường tròn
đường kính
AH
.
Bây gi ta chng minh
AFPE
là t giác ni tiếp.
Tht vy ta có:
0
360FPE FPO EPO=
( )
( )
0 0 0
360 180 180B C B C= = +
suy ra
0
180EPF A AEPF+ =
là t giác ni tiếp hay 5
điểm
, , , ,A E P F H
cùng nằm trên đường tròn đường kính
AH
EFPH
là t giác ni tiếp.
+ Xét t giác
BPHC
ta có:
0
180BPH BPE HPE BPO OPE HFE BFO C HBC= = + = +
( )
0 0 0
180 90 90B C C B= + = +
. Mặt khác ta cũng có:
00
90 180HCB B HCB BPH= + =
hay
BCHP
là t giác ni tiếp.
+ Ta có: Ta có:
0
180 , ,FPA FEA FBC FPA FPO A P O= = + =
thng hàng.
( )
1
2
FEP FAP sđ BO FP= =
,
( )
11
22
PBM PFO sđ PO FO FP= = =
, mt khác ta có:
OB OF sđOB sđOF= =
suy ra
FEP PBM=
MEPB
là t giác ni tiếp.
b). Theo câu a ta có:
MEPB
ni tiếp nên
BPM BEM BPO OPM BEC CEM= + = +
BPO OPM BEC AEF + = +
0
90AEF FBO BFO BPO OPM BEC= = = = =
hay
OPM
là tam giác vuông ti
P
Chú ý: Bài toán này có th giải theo cách như bài 1: Đó là chỉ ra
OH AM
suy ra
H
là trc tâm
tam giác
AOM
, ngoài ra ta cũng thấy
,,P H M
thng hàng.
Câu 14) Phân tích định hướng gii. Gi
S
giao điểm th 2 của hai đường tròn
( ) ( )
12
w , w
. Ta d chng
minh được
ANSM
là t giác
ni tiếp ( Đây là bài toán
rt quen thuc) t đó suy
ra 5 điểm
, , , ,A N H S M
cùng nm trên một đường tròn.
+ Trước hết ta chng minh:
,,A S D
thng hàng: Ta có:
ASN AHN=
cùng chn cung
AN
,
0
180NSD NBD NHK= =
do các t giác
,NSDB NHKB
ni
S
Q
P
K
H
N
M
D
C
B
A
CHUYÊN ĐỀ BỒI DƯỠNG HỌC SINH GIỎI HÌNH HỌC 9
23 | THCS.TOANMATH.com
tiếp . Suy ra
0
180ASN NSD AHN NHK+ = + =
do đó
,,A S D
thng hàng: +
Vì 5 điểm
, , , ,A N H S M
cùng nm trên một đường tròn nên:
0
90ASH =
.
DP
là đường kính
ca
( )
1
w
suy ra
0
90PSD =
,
DQ
là đường kính ca
( )
2
w
nên
0
90DSQ =
điều đó chứng t các tia
,,PS HS QS
trùng nhau. Hay
,,P S Q
thng hàng.
Câu 15).
Gi s
,BN CM
ct nhau ti
R
.
Ta cn chng minh
ABRC
ni tiếp.
Ta có
ABC PAC#
(C
chung,
CAP ABC=
)
suy ra
PA PC
AB AC
=
(1)
ABC QBA#
(B
chung,
BCA QAB=
) suy ra
QB QA
AB AC
=
(2). T (1), (2) ta có:
QB PA
QA PC
=
.
,PA PM=
QA QN=
suy ra
QB PM
QN PC
=
. Mt khác
,MPC PAC ACB NQB ABC QAB= + = +
(Tính cht góc ngoài tam giác). Suy ra
MPC NQB=
hay
MPC BQN BNQ PCM QCNR = #
là t giác ni tiếp. Suy ra
CRN CQN BAC ABRC= =
là t giác ni tiếp.
Câu 16). Phân tích định hướng gii:
a). Do
A
đối xng vi
D
qua
BC
nênta có
BA BD=
. Để ý rng:
AB
là tiếp tuyến ca
()L
nên
2
.BA BT BP=
2
.BD BT BP=
điều này chng t
BD
là tiếp tuyến của đường tròn ngoi tiếp tam giác
DPC
.
T
D
S
L
K
O
P
C
B
A
CHUYÊN ĐỀ BỒI DƯỠNG HC SINH GII HÌNH HC 9
THCS.TOANMATH.com | 24
b). T chng minh câu a ta có:
BDT BPD BDT BPD =#
.
Tương tự ta cũng có:
CDS CPD=
.
Ta có:
( )
SDB BDT SDC BDC BDT CPD BAC BPD CPD BPD BAC+ = + = + = +
Mt khác ta
có:
0
360CPD BPD BPC+ =
. Nhưng
0
180BPC BAC+=
do t giác
ABPC
ni tiếp. Vy
0
180SDB BDT+=
hay 3 điểm
,,S D T
thng hàng.
Câu 17)
Phân tích định hướng gii:
a). Theo gi thiết ta có:
ABD ACE=
suy raT giác
BEDC
là t giác
ni tiếp.Suy ra
..HB HD HE HC=
T giác
BNDM
ni tiếp nên:
..HB HD HM HN=
. T giác
EICK
ni tiếp nên
..HI HK HE HC=
Kết hợp các đẳng thc trên ta suy ra
..HM HN HI HK=
suy ra
NIMK
là t giác ni tiếp.
Hay bốn điểm
, , ,N I M K
cùng nm trên một đường tròn.
b). Gi s đường thng
AH
cắt đường tròn ngoi tiếp tam giác
ABD
tại điểm
F
. Ta có t giác
NFMA
ni tiếp nên:
..HF HA HM HN=
mt khác theo chng minh câu
)a
ta có:
NIMK
ni tiếp
nên:
..HM HN HI HK=
suy ra
..HF HA HI HK=
suy ra 4 điểm
, , ,I F K A
cùng nm trên mt
đường tròn. Điều đó chứng t hai đường tròn ngoi tiếp tam giác
,ABD ACE
ct nhau ti
F
,,A H F
thng hàng.
c) Ta có
AMN MAC MCA=+
( Góc ngoài ca tam giác). Mt khác
ACM ABD=
(gi thiết) suy ra
AMN ABD MAC AND MAD AND MND ANM= + = + = + =
. Suy ra tam giác
AMN
cân ti
A
.
Chú ý rng: Chứng minh tương tự ta cũng có:
AIK
cân ti
A
suy ra
A
là tâm vòng tròn ngoi tiếp
t giác
NIMK
.
Câu 18) Phân tích định hướng gii :
a). Gi
N
là giao điểm ca
PO
với đường tròn
()O
thì
N
là điểm chính gia ca cung
BC
(không cha
A
).
F
là tiếp điểm
ca
()I
vi
AB
.
Ta có các tính cht quen thuc sau:
I
M
F
E
D
K
N
H
C
B
A
P
D
F
I
a
K
N
M
O
I
C
B
A
CHUYÊN ĐỀ BỒI DƯỠNG HỌC SINH GIỎI HÌNH HỌC 9
25 | THCS.TOANMATH.com
+
, , ,
a
A I N I
thng hàng
+ Tam giác
,NIB NIC
cân ti
N
( Hay
N
là tâm
vòng tròn ngoi tiếp tam giác
IBC
)
(Xem thêm phn góc với đường tròn)
+
,
aa
BI BI CI CI⊥⊥
( Phân giác trong
và phân giác ngoài cung mt góc thì vuông góc vi nhau).
T đó suy ra tứ giác
a
IBI C
là t giác ni tiếp đường tròn tâm
N
.
b). Để chng minh
a
NI
tiếp tuyến của đường tròn ngoi tiếp tam giác
a
I MP
ta chng minh:
2
.
a
NI NM NP=
.
Mt khác
a
NI NB=
nên ta cn chng minh:
2
.NB NM NP=
. Nhưng điều này là hin nhiên do:
+
NP
là đường kính ca
()O
nên
0
90NBP =
,
M
là trung điểm ca
BC
nên
PN BC
ti
M
+
H thức lượng trong tam giác vuông
PBN
cho ta
2
.NB NM NP=
.
c). Vì
KAI KAN KPN=
(Góc ni tiếp) ,
a
KPN I PN
nhưng
a
NI
là tiếp tuyến ca ngoi tiếp
tam giác
a
I MP
nên
aa
I PN NI M=
.
Như vậy ta cn chng minh:
a
NI M DAI=
(*).Ta có:
//MN ID
nên
a
MNI DIA=
do đó ta cần
chng minh:
a
NMI IDA#
.
Điều này tương đương với:
a
NI
NM
ID IA
=
, nhưng ta có:
ID IF=
,
a
NI NB=
nên ta cn chng minh:
NM NB
FI IA
=
.
Để ý rng:
,MNB FIA
có:
0
90 ,MNB IFA==
1
2
NBM BAC IAF MNB FIA= = #
. (Bài toán
được gii quyết).
Câu 19) Phân tích định hướng:
3BC R=
. Áp dng công thc
2 sin 3BC R BAC R==
3
sin
2
BAC=
do đó
0
60A =
. Trong bài toán
Q
C'
A'
H
O
K
F
E
C
B
A
CHUYÊN ĐỀ BỒI DƯỠNG HC SINH GII HÌNH HC 9
THCS.TOANMATH.com | 26
có các yếu t c định là
,BC A
nên ta tp trung khai
thác các yếu t này.
a). Ta có:
BKC AKB AKC=+
.
00
90 30AKB AEB ABE A= = = =
,
00
90 30AKC AFC ACF A= = = =
, suy ra
0
60BKC AKB AKC= + =
. Do đó
K
luôn thuc cung
chứa góc nhìn đoạn
BC
dưới mt góc
0
60
.
b). Ta có tam giác
KBC
có độ dài cnh
BC
không đối , nên din tích ln nht khi và ch khi đường
cao h t
K
đến
BC
ln nht. Tc là
K
là trung điểm cung
BC
, khi đó
A
là trung điểm cung ln
BC
. Tam giác
KBC
đều nên độ dài đường cao tam giác đều
KBC
33
3.
22
RR=
,
2
1 3 3 3
. . 3
2 2 4
KBC
S R R R
= =
c) Để ý rng:
AB CF
tại trung điểm ca
CF
,
AC BE
tại trung điểm ca
CE
nên kéo dài
AB
cắt đường tròn
()ACF
ti
'A
thì
'AA
là đường kính của đường tròn. Kéo dài
AC
cắt đường tròn
()ABE
ti
''C AC
là đường kính của đường tròn.
D thy
', , 'A K C
thng hàng.
0
' 90 ,ACA =
0
' 90ABC =
( Góc ni tiếp chn nữa đường tròn) nên
các đường cao
', 'CA BC
ca tam giác
''AA C
ct nhau ti trc tâm
Q
.Nên đường thng
AK
đi qua
Q
. Mt khác t giác
ABQC
ni tiếp
0
90ABQ ACQ CQ= =
là đường kính của đường tròn
ngoi tiếp t giác
ABQC
. Điều đó chứng t
CQ
đi qua
O
c định.
Câu 20) Bài toán này làm ta liên
ởng đến tính cht quen thuc:
T điểm
A
ngoài đường tròn
()O
dng hai tiếp tuyến
,AB AC
và cát tuyến
ADE
.
Gi
H
là giao điểm ca
BC
AO
thì
HDEO
là t giác ni tiếp và
BH
là đường phân giác trong ca
DHE
. (Các em hc sinh t chng
minh tính cht này)
Quay tr li bài toán:
F
H
O
E
D
C
B
A
CHUYÊN ĐỀ BỒI DƯỠNG HỌC SINH GIỎI HÌNH HỌC 9
27 | THCS.TOANMATH.com
Ta có
BH
là đường phân giác trong ca
DHE
nên
DHA EHO AHF==
. Suy ra
0
180AHE AHF+=
. Nên ba điểm
,,E H F
thng hàng.
Câu 21)
Do 5 điểm
, , , ,A B K O C
cùng nm trên một đường tròn
nên ta có:
CKO OBC=
. Mà
0
90EKC CKO=−
suy ra
0
90EKC OBC BMJ EMC= = =
hay t giác
EMKC
ni tiếp.
Kéo dài
FM
ct
AB
ti
I
.
Ta chng minh
I
là trung điểm ca
AB
. Do t giác
EMKC
ni tiếp nên
EKM ECM=
ECM EFB=
suy ra
//EKM EFB MK FB=
Suy ra
M
là trung điểm ca
EN
. Áp dụng định lý Thales ta có:
ME MN FM
AI BI FI
==
ME MN AI BI= =
(đpcm).
Câu 22) Gi s
GD
ct
TO
ti
,I TF
ct
AO
ti
J
. Khi đó ta dễ dàng chứng minh được:
AO EF
ti
J
. Tht vy: Dng tiếp tuyến
Ax
ca
( )
O
thì
Ax AO
. Ta có:
xAC ABC=
//ABC AEF xAC AEF Ax EF= =
hay
AO EF
.
Ta cũng chứng minh được:
GS TO
tại điểm
I
. Tht vy ta có:
MFB MBF MFD DFB MTF TFB= + = +
DFB DCA EFA TFB= = =
suy ra
MFD MTF=
hay
MDF MTF#
suy ra
2
.MF MD MT=
. Mt khác ta
có:
2
..MO MG MB MC MF==
(Do
MB MC MF==
)
Suy ra
..MO MG MD MT=
hay
MOT MDG GI OT #
.
J
K
I
M
N
F
A
B
C
E
O
H
x
J
I
N
K
M
O
S
G
F
E
D
H
C
B
A
T
CHUYÊN ĐỀ BỒI DƯỠNG HC SINH GII HÌNH HC 9
THCS.TOANMATH.com | 28
D thấy 4 điểm
, , ,A K H E
cùng
nằm trên đường tròn đường kính
AH
.
T giác
AKBC
ni tiếp nên:
..TKTA TBTC=
T giác
EFBC
ni tiếp nên suy ra
..TK TA TF TE=
hay t giác
AKFE
ni tiếp. T đó suy ra 5
điểm
, , , ,A K F H E
cùng nm trên một đường tròn.
T giác
JSIO
ni tiếp nên
..TS TJ TI TO=
. T giác
IOMD
ni tiếp
nên
..TI TO TM TD=
. Xét t giác
MDFE
ta có:
0 0 0
180 180 180 2FDE FDB EDB A A A= = =
.Mặt khác ta cũng có
( )
( )
0 0 0 0
180 180 180 2 180 2 2( ) 180FME FMB EMC B C B C= = = +
0
180 2A=−
. Suy ra
t giác
MDFE
ni tiếp. Do đó
..TDTM TETF=
. Nhưng
..TETF TK TA=
suy ra
..TS TJ TATK=
hay t giác
AKSJ
ni tiếp
0
90SKA SJA S HK= =
. Mt khác t chứng minh trên ta cũng có:
AKMD
ni tiếp nên
0
90MKA MDA==
. Suy ra
, , ,M H S K
thng hàng.
Câu 23) Trong bài toán có gi thiết
H
là trung điểm
AB
.Mặt khác các điểm
.,A B H
có liên quan đến cát tuyến
qua
M
. Để tn dụng điều này ta s
dựng đường thng qua
D
song
song với đường thng
()d
ct
,HC BM
ti
,IF
. Khi đó ta dễ chứng minh được
I
là trung điểm ca
DF
theo định lý
Thales t đó suy ra
IN
là đường trung
bình ca tam giác
IEF
.Để chng minh
t giác
HNCS
ni tiếp ta chng minh:
NCH HSN=
. Mt khác ta có:
IDN NSH=
so le trong. Như vậy ta cn chng minh:
NCH IDN=
tc là ta cn chng minh
ICDN
ni tiếp.
+ Tht vy:
INE NEM=
( so le trong) mà
MEN MED MCD=
suy ra
F
I
O
A
B
H
(d)
M
C
D
E
S
N
K
CHUYÊN ĐỀ BỒI DƯỠNG HỌC SINH GIỎI HÌNH HỌC 9
29 | THCS.TOANMATH.com
INE MCD=
hay
ICDN
là t giác ni tiếp.
+ Ta có t giác
HNCS
nên:
SNH SCH=
. T giác
ONHS
ni tiếp nên
SNH SOH=
suy ra
SCH SOH=
. Hay t giác
SCOH
là t
giác ni tiếp. Nhưng
00
90 90OHS OCS SC= =
là tiếp tuyến ca
()O
. Mà
KC
cũng là tiếp tuyến ca
()O
nên ta suy ra
,,S K C
thng hàng.
Câu 24)
Theo tính cht tuyến
tuyến ta có:
CE CD CH==
,
BF BD BG==
,
,MH MG=
AE AF=
+ Ta có:
0
180HEF AEF HEC=
. Mt khác
0
180
2
A
AEF
=
,
0
180
2
ECH
HEC
=
suy ra
2
ECH A
HEF
+
=
(1)
+ Ta có:
EGH FGM MGH=+
nhưng
0
0 0 0
180
180 180 90
22
FBG FBG
FGM FGB
= = = +
,
( ) ( )
0
0
0
180
180
90
2 2 2
MBC MCB MBC MCB
GMH
MGH
+
= = =
suy ra
0
90
2
FBG MBC MCB
EGH
++
=+
(2) . T (1) và (2) ta có:
0 0 0
90 90 180
2 2 2
ECH A FBG MBC MCB A B C
HEF EGH
+ + + + +
+ = + + = + =
Hay t giác
EHGF
ni tiếp.
Vic chng minh trc tiếp
,,N G H
thng hàng là rất khó. Để khc phục khó khăn này ta gi s
NG
cắt đường tròn
()I
và đường tròn ngoi tiếp t giác
EHGF
lần lượt ti
12
,HH
. Ta s chng
minh
12
H H H
.
Tht vy: Theo tính cht tiếp tuyến, cát tuyến ta có:
2
1
.NG NH ND=
,
1
..NG NH NE NF=
,
2
.NE NF ND=
,
2 1 2 1 2
..NG NH NE NF NH NH NH H H H= = =
là điều phi chng minh.
O
N
M
H
G
I
F
E
D
C
B
A
CHUYÊN ĐỀ BỒI DƯỠNG HC SINH GII HÌNH HC 9
THCS.TOANMATH.com | 30
Câu 25)
Phân tích định hướng gii toán:
a). Do
AI
là tiếp tuyến chung
của các đường tròn
12
( ),( )OO
nên
2
1 1 2 2
..AN AM AI AN AM==
T đó suy ra tứ giác
1 1 2 2
N M N M
ni tiếp.
b). Để chng minh
OA
vuông góc vi
12
NN
Ta chng minh
0
1 2 1
90AN N OAM+=
Tht vy ta có: T vic chng minh
1 1 2 2
N M N M
. Ta suy ra
1 2 2 2 1
1
2
AN N N M M M OA==
Do đó
( )
0
1 2 1 1 1 1
11
90
22
AN N OAM M OA OAM OM A+ = + + =
(Do tam giác
1
M OA
cân ti
)O
. Vy
OA
vuông góc vi
12
NN
.
c) Ta có
12
//AI PQ PQ OO⊥
. Gi
S
là giao điểm ca
1
PM
2
QM
thì
22
,,O O M
thng hàng
2 2 2 2
//O I OP O IM M PO=
mt khác ta có:
2 2 2 2 2 2 2
O IM IM O M PO O IM= =
. Suy ra
2
,,P I M
thẳng hàng . Tương tự
1
,,Q I M
thng hàng. Mà
PQ
là đường kính ca
()O
nên
1 1 2 2
,QM M P QM M P⊥⊥
. Suy ra
I
là trc tâm tam giác
SPQ
. Suy ra
AI
qua
S
. Vậy ba đường
thng
12
,,AI PM QM
đồng quy ti
.I
Câu 26)
a) Có hai trường hp xy ra:
Trường hp 1:
E
nằm trong đoạn
NP
. Ta có
OEN OBP BPE=+
( Góc ngoài tam giác ).
Ta có
( )
0
0 0 0
180
, 180 180 90
2 2 2
A
BA
OBP EPB NPA
= = = = +
. T đó suy ra
00
90 180 180
2 2 2
B A C
OEN OCA= + + = =
.
Trường hp 2:
E
nằm ngoài đoạn
NP
. Ta có
0 0 0 0
180 180 90 90
2 2 2 2 2
A B A B C
OEN BPN EBP OCA

= = = + = =



(Đpcm)
Q
S
P
A'
A
O
N
2
N
1
I
M
2
M
1
O
2
O
1
K
K
M
N
P
A
B
C
O
I
E
F
F
E
I
O
C
B
A
P
N
M
CHUYÊN ĐỀ BỒI DƯỠNG HỌC SINH GIỎI HÌNH HỌC 9
31 | THCS.TOANMATH.com
b) T kết qu chng minh câu a)
Ta suy ra
ONEC
là t giác ni tiếp suy ra
0
90ONC OEC==
, Chứng minh tương tự ta cũng có:
OFPB
là t giác ni tiếp. Suy ra
0
90OFB OPB==
do đó
OFB OEC CFB BEC= =
suy ra 4
điểm
,,,B E C F
cùng nm trên một đường tròn.
c) Gi
I
là giao điểm ca
,BE CF
. T chng minh câu
b
ta suy ra
O
là trc tâm
ca tam giác
IBC
. Suy ra
,,O I M
thẳng hàng. Ta cũng có:
, , ,I F E O
cùng nằm trên đường tròn
đường kính
OI
nên
K
là trung điểm ca
OI
. T đó suy ra
,,K O M
thng hàng.
Câu 27)
a). Có
0
90BHA BEA
t giác
BHEA
ni tiếp.
HED ABC
ABC ADC
//HED EDC HE CD
b) Ni
K
vi
E
,
K
vi
H
. Ta được
1
2
KE KH AB
K
thuc trung trc ca
HE
.Có
DC CA
//HE CD HE AC
.
//KM AC
HE AC KM HE
KM
là trung trc ca
HE ME MH
hay
MHE
cân ti
M
.
Câu 28)
a). Ta có
0
90AMO ADO ANO
nên 5 điểm
, , , ,A M D O N
thuộc đường
tròn tâm
'O
đường kính
AO
.
H
D
O
M
K
E
C
B
A
H
D
O
M
K
E
C
B
A
CHUYÊN ĐỀ BỒI DƯỠNG HC SINH GII HÌNH HC 9
THCS.TOANMATH.com | 32
b) Ta có
0
90ADB ADC
(1) mà
ADM ADN
(2) (góc ni tiếp chn
hai cung bng nhau).
T (1) và (2) suy ra điều phi chng minh.
c) Qua
I
ta k đường thng song song
BC
ct
,AB AC
ti
,PQ
. Ta có các t giác
,OMPI OQNI
ni tiếp nên
;POI PMI QOI INA
PMI INA
(do
AMN
cân ti
A
). Nên
POI QOI
. Xét
POQ
OI
vừa là đường cao vừa là đường phân giác nên
IP IQ
. Áp
dng h qu định lý Talet cho hai tam giác
ABK
ACK
//PQ BC
. Ta có
BK AK CK
BK CK
IP AI IQ
(đpcm).
Câu 29)
a). Ta có
0
90ACB ADB
(góc ni tiếp chn nửa đường tròn)
Suy ra
0
90MCN MDN
0
180MCN MDN
t giác
MCDN
ni tiếp
đường tròn tâm
I
đường kính
MN
(theo định lý đảo). K
AP
AQ
vuông góc với đường thng
CD
ta có t giác
APQB
là hình thang vuông có
OH
là đường trung bình nên
2AP BQ OH
.
Trong
OCD
đều có
OH
là đường cao nên
3
2
R
OH
không đổi. Vy
3AP AQ R
không đổi. Theo gi thiết
0
60COD
nên
đ
0
s 60CD
.
đđ
0
1
s s 60
2
AMB AB CD
nên
0
60 ,CMD
ta có
0
2 120CID CMD
Trong tam giác
DIH
vuông có
0
0
3
.sin60
3
sin60
DH R
DH DI DI
(không đổi). Ta có tam
giác
,CID COD
cân ti
I
O
H
là trung điểm
CD
nên
IH
OH
cùng vuông góc vi
CD
suy ra
,,I H O
thng hàng.
c)
MCD MBA
(g.g)
nên
2
2
0
11
sin 30
44
MCD
MCD MBA
MBA
S
MD
SS
S MA
.
O
I
Q
P
H
N
M
K
D
C
B
A
CHUYÊN ĐỀ BỒI DƯỠNG HỌC SINH GIỎI HÌNH HỌC 9
33 | THCS.TOANMATH.com
MCD
S
ln nht khi
MBA
S
ln nht. Kéo dài
MN
ct
AB
ti
K
thì
MK
vuông góc vi
AB
. Ta có
MN
không đổi,
MK
ln nht khi
NK
ln nht và
N
chy trên cung
0
120
dng trên
AB
;
maxNK
khi
N
thuộc trung điểm cung này, khi đó tam giác
MAB
đều,
2
1
.3
2
MBA
S AB MH R
;
2
3
max
4
MCD
R
S
.
Cách khác: K
ME CD
thì
ME MH MI IH
. Tính được
;IH MI
theo
R
.
Câu 30)
a). Ta có
BOK OAM
(1) (đồng v);
MOK AMO
(2) (so le trong);
OMA OAM
(3) (
AOM
cân) (3).
T (1),(2),(3) ta có
BOK KOM
.
Xét
BOK
KOM
;OB OM R BOK KOM
;
OM
chung nên
BOK MOK
(c.g.c) suy ra
00
90 180OMK OBK OMK OBK
nên bốn điểm
, , ,O B K M
cùng thuc một đường tròn đường kính
OK
.
b) Ta có t giác
CHDM
là hình ch nht nên
CD
MH
ct nhau ti
I
và là trung điểm ca mi
đường. Ta chng minh
,,K I A
thng hàng. Gi
MB
ct
OK
ti
P
;
KA
ct
O
ti
N
ct
MH
ti
'I
ta có t giác
BPNK
ni tiếp (vì
0
90BPK BNK
) nên cùng bù vi
PNK
mà so le. Nên
''I NP I MP
suy ra t giác
'I MNP
ni tiếp suy ra
MNA MPI
'MNA MBA MBA MPI
v trí đồng v nên
'/ /PI AB
//PI AB
nên
'II
. Vy
AK
đi qua
I
hay ba đường thng
,,CD MH AK
đồng quy.
c) Ta có
11
22
EF AH HB AB R
(không đổi)
EHI ECI
(c.c.c);
FHI DHI
(c.c.c) nên
2.
CDFE EIF
SS
22
1 . .
.
2 4 4 4 4
EFI CDFE
R MH R MO R R
S EF IH S
2
max
4
CDFE
R
S H O
khi
M
thuc chính gia
AB
.
Câu 31)
a). Tam giác
ABI
cân ti
B
nên
BAI BIA
suy ra
EAI EIA
hay
EA EI
(1).Xét
DIE
vuông
N
F
E
C
H
A
B
D
K
M
P
I
O
E
F
H
I
D
C
B
A
CHUYÊN ĐỀ BỒI DƯỠNG HC SINH GII HÌNH HC 9
THCS.TOANMATH.com | 34
cân đỉnh
I
do đó
IE ID
(2).
T (1) và (2) suy ra
AE ID
(đpcm).
b) Do
2
.
DA ID
DADF ID
DI FD
EI BD
nên đường tròn
E
đi qua
I
và nhn
BD
làm
tiếp tuyến. T đó ta có
DAI DIF
.
Xét
AID IFD
2
.
DA ID
DADF ID
DI FD
. Mt khác
ID IE
nên
2
.DADF IE
(1). Do
ABI
cân ti
B
nên
EB AI
.
Xét
IHE BIE
2
IE EH
EH EB IE
EB IE
(2)
T (1) và (2) suy ra
..DF DA EH EB
(đpcm).
Câu 32)
a). Ta có
,ME MF
là tiếp tuyến ca
đường tròn
O
, t đó dễ dàng chng
minh được
EI FI
của đường tròn
O
. D dàng chứng minh được
,EI
,FI MI
là các đường phân giác ca
MEF
.
b) Gi
EF
ct
OM
ti
H
. D chứng minh được
2
..OAOB OH OM OE
c) Ta có
I
là tâm đường tròn ngoai tiếp
MEF
MEF
đều có cnh bng
3R
. S dng góc
ni tiếp, góc to bi tia tiếp tuyến và dây cung để chng minh
FQ EK
. Ta có
. . 2 .
KPNQ
PN PK QN QK S KN QP
. Du bng xy ra khi và ch khi
KN PQ
(*). Mà
N
là trc tâm tam giác
EKF
nên
2.KN IH R
(1). Ta có
KPQ KEF
nên
13
22
PQ KP R
PQ
EF KE
(2). Thay (1),(2) vào (*) ta có đpcm, dấu bng khi
KN PQ
hay
,NI
trùng nhau.
Câu 33)
a)
đ đ đ đ đ
1 1 1
s s s s s
2 2 2
AMB AB PC AC PC AP ABP
.
b)
PA PC CAP ABP AMB CM AC AB
.
Q
K
P
N
H
B
A
F
E
I
M
O
CHUYÊN ĐỀ BỒI DƯỠNG HỌC SINH GIỎI HÌNH HỌC 9
35 | THCS.TOANMATH.com
Xét
MAC MBP
(g.g)
MA MC
MB MP
. . .MAMP MB MC MB AB
.
Câu 34)
a) Do
AO
'AO
là hai phân giác ca
, , 'BAC A O O
thng hàng.
đ
1
s;
2
BJI IBK BI BKI
chung
KBI KJB
(g.g)
2
.
KI KB
KB KI KJ
KB KJ
(1). Tương tự
2
.
KI KD
KDI KJD KD KI KJ
KD KJ
(2). T (1) và (2)
KB KD
.
b) Xét
'ABO
vuông có:
2
.'AB AH AO
(3).
Li có
đ
1
s
2
ABI AMB BI
,
BAI
chung
ABI AMB
(g.g)
2
.
AB AI
AB AM AI
AM AB
(4). T (3) và (4)
. . '
'
AH AM
AI AM AH AO
AI AO
'AHI AMO
(vì
'
AH AM
AI AO
,
A
chung)
'AHI AMO
t giác
'MIHO
ni tiếp hay 4 điểm
, , , 'I H M O
cùng thuc một đường
tròn.
c) Do
/ / 'OD O B
(cùng
AB
)
' ' ' ' '
AO OD R OI OI
AO O B R O M O I
nhưng
OI
ct
'OI
,,A I M
thng hàng
/ / ' 'OI O M DOI BO M
. Mà
đ
11
s
22
BDI DOI DI
O
K
J
I
O'
H
M
D
C
B
A
CHUYÊN ĐỀ BỒI DƯỠNG HC SINH GII HÌNH HC 9
THCS.TOANMATH.com | 36
đ
11
's
22
BIM BO M BM
BDI BIM IM
tiếp xúc với đường tròn ngoi tiếp
BID
hay
AM
là tiếp tuyến của đường tròn ngoi tiếp
IBD
.
Câu 35)
a). Ta có
DCB CAB
(cùng chn
BC
),
BCE CAB
(góc có cạnh tương ứng vuông góc)
DCB BCE
. Do đó
CB
là tia phân giác ca
DCE
.
b) Xét
CDE
có:
EK CD BK CD
DH CE CH AB
B
là trc tâm
CDE
CB DE
ti
F
hay
CB
là đường cao ca
CDE
. Mà
CB
là tia phân giác ca
DCE
nên
CDE
cân ti
C
CED CDE
. Mt khác
11
DE
(góc cạnh tương ứng vuông góc),
22
DE
. Do đó
BDE
cân ti
B BD BE
BD BK BE BK EK
Trong tam giác
CKE
vuông ti
K
EK EC
(cnh huyn ln nht)
BK BD EC
.
c) Xét
ABC
0
90ACB
(góc ni tiếp chn nửa đường tròn)
2
.BH BA BC
(h thc v
cạnh và đường cao trong tam giác vuông) Ta li có:
BHC BFD
(g.g)
..
BH BC
BH BD BC BF
BF BD
. . . .BH BA BD BC BC BF BH AD BC CF
(1)\
Mt khác ta có:
//AC DE
(cùng vuông góc vi
CF
)
2
D CAB
(so le trong) mà
0
90AHC DFB
ACH DBE
(g.g)
..
AH AC
AH BD DF AC
DF BD
(2). Mt
khác
ABC CDF
(g.g)
..
AC CF
BC CF DF AC
BC DF
(3). T (1),(2) và (3) suy ra
..BH AD AH BD
.
Câu 36)
a)
OPM OCM
(ni tiếp chn
OM
) mà
OPM OAC
2
1
2
1
K
F
E
D
H
O
B
C
A
O
D
K
P
N
M
H
C
B
A
CHUYÊN ĐỀ BỒI DƯỠNG HỌC SINH GIỎI HÌNH HỌC 9
37 | THCS.TOANMATH.com
(
OAC
cân)
nên
OPM OAC
(1).
b) Tương tự
OPN OAB
(2)
T (1) và (2) ta có
MPN BAC
. Kéo dài
AO
ct
O
ti
D
. Ta có
;DBA DBC OBA OAB
nên
0
90 DBA DBC CBO OBA DAC COB OAB OBC BAC
0
90OBC BAC
(đpcm)
c) Gi
H
là giao điểm ca
NO
MP
,
MO
ct
NP
ti
K
. Ta có
HNP OBC
0
90OBC BAC
BAC HPN
0
90HNP HPN
hay
NH MP
(3). Tương tự
MK NP
(4)
T (3) và (4) suy ra
O
là trc tâm
MNP
(đpcm)
Câu 37)
a). Gi
', 'AB
lần lượt là hình
chiếu vuông góc ca
,AB
lên
đường thng
MN
. Gi
H
trung điểm đoạn thng
MN
thì
OH MN
.
Xét hình thang
''AA B B
OH
là đường trung bình nên
13
''
22
R
OH AA BB
;
2
2 2 2
3
42
RR
MH OM OH R
2MN MH R
.
b) Ta có
00
90 90AMB ANB KMI KNI
. Suy ra bốn điểm
, , ,M N I K
cùng nm trên
một đường tròn đường kính
KI
.
MN R
nên
OMN
đều.
00
1
30 60
2
KAN MAN MON AKN
. Gi
'O
trung điểm ca
IK
thì
'O
là tâm của đường tròn đi qua bốn điểm
, , ,M N I K
''R O M
bán kính của đường tròn này. Do đó
0
3
' 2 2 120 ' 3 '
3
R
MO N MKN AKN MN R R
. c) Gi
P
là giao điểm ca
IK
AB
, do
I
là trc tâm ca
KAB
nên
KI AB
, nên
KP
là đường cao
A
B
I
P
K
M
N
H
A'
B'
O'
O
CHUYÊN ĐỀ BỒI DƯỠNG HC SINH GII HÌNH HC 9
THCS.TOANMATH.com | 38
tam giác
KAB
h t
K
. Do
,'OO
nm trên trung trực đoạn
MN
, nên
, ',O O H
thng hàng. Xét
'MOO
0
' 90OMO
00
' 30 , ' 60MOO MO O
. Suy ra
2
' 2 '
3
R
OO MO
KAB
AB
không đổi nên nó có din tích ln nht khi
KP
ln nht. Ta có
2
' ' 3
33
RR
KP KO OO R
. Đẳng thc
xy ra khi
PO
' / /OO AB MN AB
KAB
cân ti
K KAB
đều (do
0
60AKB
). Do đó
2
1
. . 3
2
KAB
S AB KP R KP R
. Kết lun:
Din tích
KAB
ln nht bng
2
3R
khi và ch khi
//MN AB
(hay
KAB
đều).
Câu 38)
a)
đ
1
s
2
ABD ADE AD
ca
'O
,
đ
1
s
2
ABC ACE AC
ca
O
CED CBD CED ABD ABC
0
180CED ACE ADE
(tng ba góc trong
ECD
).
Vy t giác
BDEC
ni tiếp.
b) Vì t giác
BCED
ni tiếp nên
;CEB CDB EBC EDC
EDC ABD
nên
EBC ABD
.
..
EC DA
EBC DBA EC DB DAEB
EB DB
(1). Tương tự
..
ED CA
EBD CBA EDCB CAEB
EB CB
(2). T (1) và (2) ta
được:
. . . . .EC DB EDCB DAEB CAEB DA CA EB CD EB
.
Nhn xét: Kết qu câu b) thc chất là định lý Ptolemy (Xem thêm phn ‘’Các định lý hình hc
ni tiếng’’)
Câu 39)
a). Vì
CD
là đường kính
nên
0
90CBD
. Do đó
BEF ABF
(góc có
cạnh tương ứng vuông
góc cùng nhn), mà
ABF ODB
O'
O
B
A
D
C
E
OB OD R
Q
O'
H
M
O
K
F
E
D
C
B
A
CHUYÊN ĐỀ BỒI DƯỠNG HỌC SINH GIỎI HÌNH HỌC 9
39 | THCS.TOANMATH.com
OB OD R
nên
BEF CDB
. Do đó tứ giác
CDEF
ni tiếp.
b) Gi
Q BM CD
,
BEF
vuông ti
B
nên
BM ME MBE MEB
(1)
BCD
vuông ti
B
nên
0
90BCD BDC
BDC BEF
(chng minh câu a) nên
0
90BCD BEF
(2). T (1) và (2) ta có
00
90 90BCD MBE BQC
hay
BM CD
.
c)
K
là tâm đường tròn ngoi tiếp t giác
CDEF
,
M
là trung điểm ca
EF
nên
//KM EF KM AB
(vì cùng vuông góc vi
EF
). Tương tự
//KO BM
(cùng vuông góc
vi
CD
). Do đó tứ giác
KMBO
là hình bình hành nên
MK BO R
.
d)
H
là trc tâm ca tam giác
DEF
, do đó
HD EF
suy ra
//HD AB
. Tương tự
//BH AD
(cùng vuông góc vi
EF
). Do đó t giác
BHDA
là hình bình hành nên
BH AD
. Mt khác t
giác
BDAC
là hình ch nht nên
AD BC
BH BC
(3).Ly
'O
đối xng vi
O
qua
B
ta có
'BO BO
(4) vi
'O
c định vì
,BO
c định. T (3) và (4) suy ra t giác
'HO CO
là hình
bình hành nên
'O H OC R
. Vy
H
chạy trên đường tròn
';OR
c định.
Câu 40)
a) Ni
H
vi
E
.
0
90HEA
(vì
AH
là đường kính),
0
90AHC
(
AH
là đường cao)
AHE ACB
(cùng ph vi
EHC
) (1)
ADE AHE
(góc ni tiếp cùng chn
AE
) (2). T
(1) và (2)
ADE ACB
t giác
BDEC
ni tiếp đường tròn
b) Vì
0
90DAE DE
là đường kính
,,D O E
thẳng hàng (đpcm).
c) Ta có
BDEC ABC ADE
S S S
ABC
vuông có
AH
là đường cao:
2 2 2
.
46
2
ABC
AB AC
AC BC AB cm S cm
. 12
5
AB AC
DE AH cm
BC
(cùng là đường kính đường tròn
O
)
ADE
ABC
A
chung;
ADE ACB
(câu a)
O
E
D
H
C
B
A
CHUYÊN ĐỀ BỒI DƯỠNG HC SINH GII HÌNH HC 9
THCS.TOANMATH.com | 40
ADE ABC
(g.g) T s din tích bằng bình phương tỉ s đồng dng
2
2
2
.
AED ABC
AED
ABC
S S DE
DE
S
S BC
BC
22
2
2 2 2
12 2886
1 6 1
625
5 .5
BDEC ABC ADE ABC
DE
S S S S cm
BC
Câu 41)
a) Ni
N
vi
F
,
D
vi
F
.
+ Xét
ANF
AFD
AFN ADF
(vì
AF
tiếp tuyến) và
FAD
chung
ANF AFD
(g.g)
2
.
AN AF
AF AN AD
AF AD
(1) + Xét
AFI
AF IF
(vì
AF
là tiếp tuyến,
FI
là bán kính) và
FK AI
(vì
AF
AE
là tiếp
tuyến chung và
AI
ni tâm)
AFI
vuông ti
F
FK
là đường cao)
2
.AK AI AF
(2).
+ Xét
ANK
AID
IAD
chung. T (1) và (2)
..
AN AI
AN AD AK AI
AK AD
ANK AID
(c.g.c)
NKA IDN
(3). T (3) có
0
180IDN DKN NKA DKN
T giác
DIKN
ni tiếp đường tròn Các điểm
, , ,I D N K
cùng thuc đường tròn (đpcm)
b) Ta có
ID DM
(
DM
là tiếp tuyến,
DI
là bán kính) và
IK KM
(câu a) T giác
DIKM
ni tiếp đường tròn đường kính
MI
. Vì bốn điểm
, , ,D I K N
cùng thuc một đường tròn (câu a)
Hai đường tròn này ngoi tiếp
DIK
Hai đường tròn trùng nhau
N
cũng nằm trên
đường tròn đường kính
MI
0
90INM
. Vì
IN
là bán kính đường tròn
I
,
MN IN
MN
là tiếp tuyến của đường tròn
I
ti tiếp điểm
N
(đpcm).
Câu 42)
1.a)
2
.PIN PNJ PI PJ PN
(1)
Xét
PON
vuông ti
N
2
.NE PO PN PE PO
(2)
PKE POF
K
N
M
I
F
E
D
C
B
A
E
Q
I
F
J
K
O
N
M
P
CHUYÊN ĐỀ BỒI DƯỠNG HỌC SINH GIỎI HÌNH HỌC 9
41 | THCS.TOANMATH.com
..PK PF PO PE
(3)
T (1),(2) và (3)
..PI PJ PK PF
b)
PIO POJ
suy ra t giác
IEOJ
ni tiếp (4)
Ta có
00
; 90 ; 90IEP IJO IEP QEI IJO QOJ QEI QOJ
.
QOJ QOI QEI QOI
. Suy ra t giác
QIEO
ni tiếp (5)
T (4) và (5) suy ra
, , ,Q I O J
cùng thuc một đường tròn.
Câu 43) a). Ta có
MC
là tiếp tuyến của đường tròn
O MC MO
(1)
Xét đường tròn
I
:
Ta có
0
90CMD
MC MD
(2)
T (1) và (2)
//MO MD MO
MD
trùng nhau
,,O M D
thng hàng.
b)
CA
là tiếp tuyến của đường tròn
O CA AB
(3). Đường tròn
I
tiếp xúc vi
AC
ti
C
CA CD
(4). T (3) và (4)
//CD AB DCO COA
(*) (hai góc so le trong)
,CA CM
là hai tiếp tuyến ct nhau ca
O
COA COD
(**). T (*) và (**)
DOC DCO COD
cân ti
D
.
c) Gi chân đường vuông góc h t
D
ti
BC
H
0
90CHD
HI
(bài toán qu
tích)
DH
kéo dài ct
AB
K
. Gi
N
là giao điểm ca
CO
và đường tròn
I
0
90CND
COD
cân ti
D NC NO
. Ta có t giác
NHOK
ni tiếp.
Vì có
21
H O DCO
(cùng bù vi
DHN
)
0
180NHO NKO
(5) Ta có
NDH NCH
(cùng chn cung
NH
của đường tròn
I
)
CBO HND HCD DHN COB
(g.g)
HN OB
HD OC
. Tương tự ta có:
;
OB OA OA CN ON HN ON
OC OC OC CD CD HD CD
.
1
1
K
H
N
D
I
O
C
M
A
CHUYÊN ĐỀ BỒI DƯỠNG HC SINH GII HÌNH HC 9
THCS.TOANMATH.com | 42
ONH CDH
NHO DHC
(c.g.c)
0
90NHO
0
180NHO NKO
(5)
0
90 / /NKO NK AB NK AC
K
là trung điểm ca
OA
c định đpcm.
Câu 44)
a) Ta chng minh
ABE BSM
.
b) T câu a ta có
AE MB
AB BS
(1)
MB EM
(do
BEC
vuông
ti
E
M
là trung điểm ca
BC
)
nên
AE EM
AB BS
0
; 90 ;MOB BAE EBA BAE
0
90MBO MOB
nên
MBO EBA
do đó
MEB OBA MBE
. Suy ra
MEA SBA
(2)
T (1) và (2) suy ra
AME ABS
(đpcm).
c) D thy
SM
vuông góc vi
BC
nên ta chng minh
//NP SM
.
Xét
ANE
APB
: T câu b) ta có
AEM ABS
nên
NAE PAB
. Mà
AEN ABP
(do t giác
BCEF
ni tiếp). Do đó
ANE APB
nên
AN AE
AP AB
. Li có
AM AE
AEM ABS
AS AB
. Suy ra
AM AN
AS AP
nên trong tam giác
AMS
//NP SM
ịnh lý Talet đảo). Do đó bài toán được chng minh.
Câu 45)
a). Gi
K
là giao điểm ca
BO
vi
DF
IKF
vuông ti
K
0
1
45
2
DFE DOE
0
45BIF
b) Khi
AM AB
thì
ABM
P
N
M
O
F
E
S
C
B
A
H
K
A
B
C
D
E
F
O
M
I
CHUYÊN ĐỀ BỒI DƯỠNG HỌC SINH GIỎI HÌNH HỌC 9
43 | THCS.TOANMATH.com
vuông cân ti
0
45A DBH
. Có
0
45DFH
T giác
BDHF
ni tiếp 5 điểm
, , , ,B D O H F
cùng thuc một đường tròn.
0
45BAH BIH
T giác
ABHI
ni tiếp.
0
90BFO BHO OH BM
, mà
,,OA BM A O H
thng hàng.
c) Có t giác
PNQD
ni tiếp
QPN QDN EFN
.Tương tự
NQP NDP FEN NEF NQP
. Cách xác định điểm
M
: K đường kính
DN
ca
O
,
BN
ct
AC
ti
M
thì
PQ
ln nht. Dấu “=” xảy ra khi và ch khi
;P F Q E
DN
là đường kính ca
O
PQ
ln nht bng
1
PQ NQ
EF PQ EF
EF NE
.
Câu 46)
a). Ch ra
PBI PMI PAC
T giác
BMIP
ni tiếp.
Ch ra
PNI PCI PAB
T giác
CNIP
ni tiếp.
b) Vì
BP CP
(gi thiết)
BPC
cân ti
P PBI PCI
.
Kết hp câu a)
;BAP CAP PMI PNI PMN
cân
PM PN
.
PI
là đường trung trc ca
MN PI MN
Kết hp câu a)
0
90ABP ACP ABP ACP
(g.c.g)
AB AC ABC
cân.
Câu 47)
a). Ta có
//MN BC
(gt),
ID BC
(
I
tiếp xúc vi
BC
ti
D
ID MN IK MN
0
90IKM IKN
0 0 0
90 90 180IFM IKM
T giác
IFMK
ni tiếp.
M
O
N
I
P
C
B
A
0 0 0
90 90 180IFM IKM
K
J
N
M
D
F
E
I
C
B
A
CHUYÊN ĐỀ BỒI DƯỠNG HC SINH GII HÌNH HC 9
THCS.TOANMATH.com | 44
Mt khác
0
90IKN IEN
t giác
IKEN
ni tiếp. Ta có
IMF IKF
(t giác
IFMK
ni
tiếp) ;
IKF ANI
(t giác
IKEN
ni tiếp)
IMF ANI
T giác
IMAN
ni tiếp.
b) Ta có
IMK IFK
(t giác
IFMK
ni tiếp);
INK IEK
(t giác
IKEN
ni tiếp). Mt khác
IE IF r IEF
cân ti
I IMK INK
IMN
cân ti
I
IK
là đường cao
IK
là đường trung tuyến ca
IMN
K
là trung điểm ca
MN
2MN MK
2BC BJ
(
J
là trung điểm ca
BC
). Do đó
2
2
MN MK MK
BC BJ BJ
.Mt khác
ABC
//MN BC
AM MN
AB BC
(h qu của định lý Talet). Ta có
AM MK MN
AB BJ BC
. Xét
AMK
ABJ
có:
AMK ABJ
(hai góc đồng v do
//MN BC
);
AM MK
AB BJ
AMK ABJ
(c.g.c)
MAK BAJ
Hai tia
,AK AJ
trùng nhau. Vậy ba điểm
,,A K J
thng hàng.
c)
,AE AF
là các tiếp tuyến của đường tròn
I AE AF
,
AI
là tia phân giác ca
EAF
.
AEF
cân ti
A
0
60EAF
(gt)
AEF
đều
EF AE AF
,
AEF
đều có
AI
là đường phân giác
AI
là đường cao ca
AEF
1
.
2
AI EF S AI EF
.
IAE
vuông ti
.cot ; .sinE AE IE IAE IE AI IAE
0
0
.cot30 3; 2
sin 30
r
AE r r AI r
3EF AE r
. Vy
2
11
. .2 . 3. 3
22
S AI EF r r r
(đvdt). Gọi
H
là giao điểm ca
AI
EF
. Ta có
IH EF
,
H
là trung điểm ca
EF
0
60HIF
IHF
vuông ti
0
1
.cos .cos60
2
H IH IF HIF r r
. Do đó
2
1 3.
.
24
IEF
r
S IH EF
(đvdt). Xét
IMN
IEF
, ta
;IMN IFE INM IEF
. Do đó
IMN IEF
(g.g)
2
IMN
IEF
S
IM
S IF
. Mà
1
IM
IF FM IM IF
IF
. Do đó
1
IMN
IMN IEF
IEF
S
SS
S
. Ta có
2
2
3.
3 ; ;
4
IEF IMN IEF
r
S r S S S
, vy
4
IMN
S
S
.
Câu 48.
a).
*) T giác
IABN
ni tiếp do
P
N
M
K
I
E
D
C
B
A
CHUYÊN ĐỀ BỒI DƯỠNG HỌC SINH GIỎI HÌNH HỌC 9
45 | THCS.TOANMATH.com
+
đ
1
s
2
IBN ED
, +
IAN DAC NAC
DAN NCA
đđ
1
ss
2
AD AE
đ
1
s
2
ED
*) Ta có
đ2 =sPNE NAC NCA NCA EA
,
đ2sPDE PDA ADE ABE ADE ABE AE
. Suy ra t giác
PNDE
ni tiếp.
b). Ta thy t giác
AKME
ni tiếp do
0
45MEK MAK
suy ra
0
90MEA MKA
suy ra
//MK BD MKD KDB EKM
. c). Ta có
2
2
..
4
MD ME CD
MDC MEA ME MC MD MA MD
MC MA
$
Mt khác ta có:
22
2 2 2 2
5
44
CD CD
MC CD MD CD
suy ra
5
2
MC CD
5
10
ME CD
hay
5MC ME
.Ta có:
2
. 5 10
2
55
5
2
CD
EA AM AM CD
EA CD R
CD MC MC
CD
.
Câu 49). Gii:
Gi s
CN
cắt đường tròn
ngoi tiếp tam giác
ABN
tại điểm th hai
F
,
BM
cắt đường tròn ngoi tiếp
tam giác
AMC
tại điểm
th hai
E
. Ta có
BEC CAM MAB CFB
do đó
BCEF
là t giác ni tiếp.
Suy ra
EFC EBC ABN AFN
, do đó
,,A E F
thng hàng.
Khi đó
ACM AEM FCB
suy ra
BCM ACN
.
Câu 50. Gii:
Đưng thng qua
A
song song vi
BD
ct
BM
ti
Q
.
Ta có
NA NA MA AQ
AB CD MD BD
do đó
NA AQ
. Mt khác
0
60NAQ
nên tam giác
NAQ
đều.
D
F
E
M
N
C
B
A
CHUYÊN ĐỀ BỒI DƯỠNG HC SINH GII HÌNH HC 9
THCS.TOANMATH.com | 46
Vy phép quay tâm
A
góc quay
0
60
biến
D
thành
B
,
N
thành
Q
. Do đó
DN
BQ
to vi
nhau mt góc
0
60
.
Vy
0
60BPD
. Do đó
P
thuộc đường tròn ngoi tiếp tam giác
ABD
.
Câu 51. Gii:
Vì t giác
,BEPD AQCB
ni tiếp nên
CAQ CBQ DEP
.
Mt khác
0
180AQC ABC EPD
(1).Áp dụng định lý Ptô - mê
cho t giác
BEPD
ta có
. . .PE BD PD EB DE BP
(2)
T (1) và (2) suy ra
. . .AQ BD QC EB CABP
. Mt khác
BD EB CA
nên
AQ QC BP
.
Câu 52. Gii:
a) Ta có
11
22
NCE IBN B A NCM
. Do đó
E
nm trong góc
NCM
(1). Do
BC
nên
N
C
nm
v cùng một phía đối vi
AM
.
Do đó
E
C
nm v cùng
một phía đối vi
AM
(2).
T (1) và (2) suy ra
E
nm trong
góc
AMC
. Vy
Q
thuc cung nh
AC
.
b) Vì
QK QA
(3), suy ra
11
22
QAK AKQ A B
1
2
IAK CAQ B IBK CBQ
. Mt khác
CAQ CBQ
nên
CAQ IBK
hay t giác
AIKB
ni tiếp. T đó ta có
1
2
IKQ BAI A KQM
nên
//KI MQ
.
Gi
L
là giao điểm ca
KN
MQ
, khi đó
KILE
là hình bình hành. Do đó
N
là trung điểm ca
KL
.Vy
BKCL
là mt hình bình hành. Mt khác ta có
/ / ,BK CL BK CL
E
D
Q
P
C
B
A
Q
M
L
N
K
E
I
O
C
B
A
CHUYÊN ĐỀ BỒI DƯỠNG HỌC SINH GIỎI HÌNH HỌC 9
47 | THCS.TOANMATH.com
CLQ BLQ CQM
(4) nên
CL QC
(5). T (4) và (5) suy ra
BK CL
(6).
T (3) và (6) ta có
BQ QA QC
.
Câu 53. Gii:
Gi
T
là giao điểm (khác
'A
)
của đường tròn ngoi tiếp tam
giác
'''A B C
và tam giác
'A BC
. T giác
'BA CT
ni tiếp mt
đường tròn do đó
''CTB CTA A TB
0
' 180 ' ' 'CBA A C B
0
' 180CBA ACB
(do
' '/ / , / / ' 'A C AC BC B C
)
0
180 'ACB CBA
. Gi
H
giao điểm ca
'BA
AC
. Khi đó
''ACB CBA BHC CAB
do đó
/ / 'BH AB
. Suy ra
0
180 'CTB CAB
. Do đó tứ giác
'AB TC
ni tiếp hay
T
thuộc đường tròn ngoi tiếp tam
giác
'B CA
. Tương tự
T
thuộc đường tròn ngoi tiếp tam giác
'C AB
.
Câu 54. Gii:
a). Gi
,KT
lần lượt là hình
chiếu vuông góc ca
,MN
xung
BC
.
H
là hình chiếu
vuông góc ca
N
xung
AC
,
L
là hình chiếu vuông
góc ca
M
xung
AB
.
Đặt
;;
PN
MK ML a NT NH b c
PM
. D dàng tính được:
11
1 1 1 1
cc
PX NT b a
c c c c
;
;
11
cc
PZ ML a
cc
11
11
PY NH b
cc
. Do đó
PY PX PZ
.
H
T
C'
B'
A'
O
C
B
A
X
Z
H
Y
K
T
P
N
M
C
B
A
CHUYÊN ĐỀ BỒI DƯỠNG HC SINH GII HÌNH HC 9
THCS.TOANMATH.com | 48
b) D dàng chứng minh được
PZB PYC
nên
PB PZ
PC PY
;
PXB PYA
nên
PB PX
PA PY
. Do đó
1
PB PB PX PZ
PC PA PY
hay
1 1 1
PB PA PC
.
Câu 55. Gii:
Ta có
OM ON
ROM RON
nên
OMR ONR
(c.g.c).
Vy
RM RN
.Do
N
nằm trên đường trung trc ca
MN
và nm
trên phân giác
MAN
nên
R
điểm chính gia cung
MN
(cung không cha
A
) của đường
tròn ngoi tiếp tam giác
AMN
.
Vy t giác
AMRN
ni tiếp mt
đường tròn. Gi s đường tròn
ngoi tiếp tam giác
BMR
ct
BC
ti
P
khác
B
. Do đó các tứ giác
,AMRN BMRP
ni tiếp
được nên t giác
CNRP
cũng nội tiếp.
Vậy hai đường tròn ngoi tiếp các tam giác
,BMR CNR
ct nhau ti
P
thuc
BC
.
Câu 56. Gii:
S dng t giác
ABCD
ni tiếp.
Ta chng minh
PA PC
.
Gi s
BP
ct
AC
E
,
DP
ct
AC
F
. D dàng chng
minh được các cp tam giác
đồng dng
,BCE BDA
DAF DBC
suy ra
AF AD CE
BC BD BC
. Do đó
CE AF
. Mt khác
PEF PFE
nên
PE PF
. T đó ta có
PEC PFA
(c.g.c). Vy
PC PA
.
Ngược li, gi s
PA PC
. Gi
,XY
tương ứng là giao điểm ca
,CD DP
với đường tròn ngoi
tiếp tam giác
BCP
. Ta có các cặp tam giác đồng dng
,ADB PDX ADP BDX
nên
R
P
N
M
C
B
A
O
P
F
E
D
C
B
A
CHUYÊN ĐỀ BỒI DƯỠNG HỌC SINH GIỎI HÌNH HỌC 9
49 | THCS.TOANMATH.com
BX BD XD
AP AD PD
(1). Mt khác
DPC DXY
nên
XY XD
CP PD
(2). T (1) và(2) suy ra
BX XY
. Do đó
0
180DCB XYB XPY PDX PXD ADB ABD BAD
.Vy
ABCD
ni tiếp.
Câu 57. Gii:
Đưng tròn ngoi tiếp tam
giác
ABC
cắt các đường
thng
,,IA IB IC
lần lượt
tại các điểm th hai
,,M N P
.
Gi
;OR
,
'; 'OR
lần lượt
là đường tròn ngoi tiếp tam
giác
, ' ' 'ABC A B C
.
Ta có:
. ' . ' . 'IAIA IB IB IC IC
;
. . .IAIM IB IN IC IP
do đó
' ' 'IA IB IC
IM IN IP
. Suy ra
hai tam giác
'''A B C
MNP
có các cạnh tương ứng song song nên
' '/ /O C OP
. Mt khác ta có
' ' ' ' ' '
OP R NP IP
O C R C B IC
. Vậy ba điểm
, , 'O I O
thng hàng.
Câu 58.
Gii:
Khi
A
thay đổi trên cung ln
BC
thì tam giác
MNP
NMP
không đổi bng
0
90
2
A
.
Tam giác
ANP
luôn cân, có
A
không đổi nên
NP
ln nht khi
AB AC
ln nhất, khi đó
A
là điểm chính gia ca cung ln
BC
. Gi
0
A
là điểm chính gia
ca cung ln
BC
, ng vi v trí đó ta có tam giác
0 0 0
M N P
cân ti
0
M
;
0 0 0 0 0
;N P NP N M P NMP
. Vy chu vi
0 0 0
M N P
chu vi
MNP
. Do đó chu vi tam giác
MNP
ln nht khi
0
AA
.
O'
I
O
P
N
M
C'
B'
A'
C
B
A
G'
O
'
O
C'
B'
A'
P
N
M
I
C
B
A
CHUYÊN ĐỀ BỒI DƯỠNG HC SINH GII HÌNH HC 9
THCS.TOANMATH.com | 50
b) Gi
'A
là giao điểm ca
IA
vi
NP
,
'B
là giao điểm ca
IB
vi
MP
,
'C
là giao ca
IC
vi
MN
. Các điểm
', 'OG
lần lượt là tâm đường tròn ngoi tiếp, trng tâm tam giác
'''A B C
. Ta
2
. ' . ' . '
a
IAIA IB IB IC IC r
. Do đó theo câu 9 suy ra ba điểm
, ',I O O
thng hàng. Mt
khác ba điểm
, ', 'I G O
cùng nằm trên đường thẳng Ơ-le ca tam giác
MNP
luôn đi qua
O
c
định.
Câu 59) Gii:
Gi
,,M N P
lần lượt là các
tiếp điểm ca các cặp đường
tròn
1 2 1 2
, ; , ; ,O O O O O O
.
Đưng thng
MI
ct
AB
ti
Q
. Ta có ba điểm
2
,,O P O
thng hàng.
Mt khác
2
//O M OB
2 2 2
O M r O P
OB r OB
nên
22
11
MO r
QA
QB MO r
.Tương tự
22
PB PO r
PM PO r
1
r
MN
NA r
. Do đó
21
12
. . . . 1
rr
QA PB NM r
QB PM NA r r r
. Vậy các đoạn thng
,,AP MQ BN
đồng quy
nên
MQ
cũng là đường cao ca tam giác
MAB
hay
MQ AB
suy ra
12
MQ OO
. Vy
I
thuc
đường thng qua
M
vuông góc vi
12
OO
.
Câu 60. Gii:
Gi
E
là giao điểm th hai
khác
A
ca
AI
với đường
tròn
O
. Khi đó
E
là điểm
chính gia cung
BC
(cung không cha
A
).
Ta có
EB EI EC IA
.
Theo định lý Ptô--mê ta có
. . .EABC EC AB EB AC
do đó
2BC AB AC
. Theo tính
chất đường phân giác trong tam giác ta có:
2
AB AI AC AB AC AB AC
BD ID DC BD DC BC
.Vy
2
AI
AD
.Gi
M
là trung điểm cnh
BC
, khi đó
2
AG AI
GM ID
. Vy
//GI BC
.
Câu 61. Gii:
O
2
O
1
O
I
Q
P
N
M
B
A
G
D'
O
M
E
D
I
C
B
A
CHUYÊN ĐỀ BỒI DƯỠNG HỌC SINH GIỎI HÌNH HỌC 9
51 | THCS.TOANMATH.com
Gi
O
là giao điểm ca
AC
BD
. Ta có
1
/ 1 3
1
2
O
d R R
(Tính chất đường trung bình
trong mt hình thang), đó
1
/O
d
là khong cách t
O
ti
1
. Tương tự
3 2 4
/ 1 3 / 2 4 /
11
;
22
O O O
d R R d R R d
. Vy
14
/ / 3 /O O O
d d d
. Do đó
1 2 3 4
, , ,
cùng tiếp xúc vi một đường tròn tâm
O
.
Câu 62. Gii:
Đưng tròn ngoi tiếp tam giác
SMN
có tâm
B
và tiếp xúc vi
AS
ti
S
,
AM
ti
M
. Đường
tròn ngoi tiếp tam giác
SPQ
có tâm
D
và tiếp xúc vi
AQ
ti
Q
,
AS
ti
S
.
Ta có
AMQ AQM
suy ra
QME FQM AQF AME QPF MNE
(1)
Ta có
2
..AE AN AM AF AP
hay t giác
NEFP
ni tiếp. Do đó
FEM EFQ FEN NEM EFP PFQ
00
180 180FPN NEM ENP PFQ
ENP FPN NSM PSQ
ENC FPC NSM PSQ
ý rng
CP CS CN
)
ENS NSC FRS SPC NSM PSQ
ENS NSC FPS CSP NSM PSQ
NSC NSM CSP PSQ NSM FPS
00
90 90SNM SPQ ENS FPS
MSC QSC ENS FPS
(2)SPQ FPS SNM ENS QPF MNE
T (1) và (2) suy ra
QME FQM FEM EFQ
hay
QME EFQ EFM FQM
. Do đó tứ giác
MEFQ
ni
tiếp.
Câu 63. Gii:
Đưng thng qua
D
song song
F
E
P
Q
N
M
S
D
C
B
A
Q
P
N
M
D
C
B
A
CHUYÊN ĐỀ BỒI DƯỠNG HC SINH GII HÌNH HC 9
THCS.TOANMATH.com | 52
vi
AC
ct
AB
ti
M
. Đường
thng qua
M
song song vi
BC
ct
AC
ti
N
. Gi
Q
là trung
điểm ca
NC
.Ta có
1
3
AM DC
AB BC
nên
1
3
AN
AC
.
Vy
CQ NQ AN
. Do đó
1
3
QC DC
CA BC
suy ra
//DQ AB
. Do
BAC BPD
nên
BMD BPD
. Vy t giác
BMPD
là t giác ni tiếp. Do đó
MPA MDB ACB MNA
suy ra t giác
AMPN
ni tiếp.
,BMPD AMPN
là các t giác ni tiếp nên
CDPN
là t giác ni tiếp. Hơn nữa do
QD QC QN
, nên
Q
là tâm đường tròn ngoi tiếp t giác
CDPN
. Vy
11
22
DPC DQC BAC
.
Câu 64. Gii:
Theo định lý Sim-sơn, ba điểm
,,P Q R
thng hàng. T hai b
bốn điểm
, , , ; , , ,C D P Q A D Q R
cùng thuc một đường tròn ta
suy ra
DCA DPR
.
Tương tự ta được các cặp tam giác đồng dng
,DAB DQP DBC DRQ
. Do đó
.
DC BC PQ
DA BA QR
. Suy ra
DC BC
PQ QR
DA BA
. Điều đó tương đương với chân đường phân
giác góc
D
ca tam giác
ADC
và chân đường phân giác góc
B
ca tam giác
ABC
trùng nhau,
hay các phân giác góc
ABC
ADC
ct nhau trên
AC
.
Câu 65. Gii:
Gi
H
là giao điểm ca
MC
vi
PQ
. Ta cn chng minh
H
là trung điểm ca
MQ
.Ta có
KAC KMA
suy ra
MA KA
CA KC
. Tương tự
KBC KMB
suy ra
MB KB
CB KC
.
Mt khác
KA KB
nên
MA MB
CA CB
do đó
R
Q
P
O
D
C
B
A
O
2
O
1
H
K
C
B
A
Q
P
M
CHUYÊN ĐỀ BỒI DƯỠNG HỌC SINH GIỎI HÌNH HỌC 9
53 | THCS.TOANMATH.com
..AC BM BC AM
(1).
D dàng nhn thy
BMP BCQ
suy ra
BC CQ
BM MP
(2). T (1) và
(2) ta có
.1
CA MP
CQ MA
. S dụng định lý Mê--la uyt cho tam giác
QPA
vi cát tuyến
MCH
ta
có:
. . 1
MP CA HQ
MA CQ HD
. Do đó
HQ HD
.
Câu 66. Gii:
,,TA TB TC
cắt đường tròn
'O
tại các điểm th hai
111
,,A B C
.
Khi đó
111
ABC ABC
,
hơn nữa chúng có các cnh
tương ứng song song. Ta có
1 1 1 1 1 1
AC BC AB
AC BC AB
22
2 2 2
1 1 1 1 1
2
11
' . . '
'
TA TA TA TB TB
AA AA AT BB BT BB
AA
.Do đó
11
''
TA TB
AA BB
. Tương tự
11
''
TA TC
AA CC
.
Vy
1 1 1
' ' '
TA TB TC
AA BB CC
do đó
' ' '
TA TB TC
AA BB CC
(1). Theo định lý Ptô- -mê ta có
...TB AC TABC TC AB
(2). T (1) và (2) ta có:
'. '. '.BB AC AA BC CC AB
.
Câu 67. Gii:
K các tiếp tuyến chung
trong
,KH LT
ca
1
O
2
O
. Giao điểm ca
,KH LT
vi
O
lần lượt là
,BC
.K tiếp tuyến chung
C
1
B
1
A
1
C'
B'
A'
T
O
C
B
A
Q
P
X
O
2
O
1
O
T
Y
L
K
H
N
F
E
M
C
B
A
CHUYÊN ĐỀ BỒI DƯỠNG HC SINH GII HÌNH HC 9
THCS.TOANMATH.com | 54
ngoài
EF
ca
1
O
2
O
sao cho
E
B
nm v cùng một phía đối vi
12
OO
. Các điểm
,MN
lần lượt là các tiếp điểm ca
12
,OO
vi
O
.
EF
ct
O
ti
,PQ
. Ta s chng minh
//BC PQ
. Gi
A
là điểm chính gia ca cung
PQ
của đường tròn
O
, k các tiếp tuyến
,AX AY
ca
12
,OO
. D dàng chứng minh được ba điểm
,,A E M
thẳng hàng, ba điểm
,,A F N
thng hàng và t giác
MEFN
ni tiếp. Do đó
22
..AX AE AM AF AN AY
hay
AX AY
.
Áp dng câu 66 cho tam giác
ABQ
ni tiếp đường tròn
O
và đường tròn
1
O
tiếp xúc vi
O
ti
M
, ta có:
. . .AX PQ BK AC CL AB
. Tương tự
. . .AY PQ BH AC CT AB
.
Suy ra
AC BH BK AB CL CT
do đó
..AC KH ABTL
hay
AC AB
. Vy
A
trung điểm ca cung
BC
, do đó
//PQ BC
.
Câu 68. Gii:
Ly
,EF
thuộc đường tròn sao cho
,CDB ADE BDA DCF
.
Khi đó
, , ,AE BC FD AB EC AB BF AD
.
Áp dụng định lý Ptô--
cho hai t giác ni tiếp
AECD
BCDF
ta có:
. . .AC ED AE CD AD EC
..BC CD AD AB
(1)
. . .BDCF BC DF BF CD
..BC AB ADCD
(2)
Mt khác
CDE CDB BDE ADE BDE ADB FCD
Do đó
FDC FDE EDC FCE FCD ECD
suy ra
ED FC
(3)
T (1),(2),(3) ta có điều phi chng minh.
Câu 69. Gii:
T giác
''BC MA
''A MB C
ni tiếp được nên
F
E
O
D
C
B
A
CHUYÊN ĐỀ BỒI DƯỠNG HỌC SINH GIỎI HÌNH HỌC 9
55 | THCS.TOANMATH.com
0
' ' ' ' 180A MC B C A MB
' ' 'MB A MCA
. Mà
BC
suy ra
' ' ' 'A MC A MB
.
Ta li có
'
''
CM MA
MA MB
suy ra
' ' ' 'C MA A MB
(c.g.c). Do đó
' ' ' ' 'C A M A B M MCA
. Mt khác
' ' 'MC A MBA
nên
''BMC C MA
. Do đó
0
' ' 180BCM A MC B
không đổi. Vy
M
thuc một đường tròn c định.
Câu 70)
Gi
H
trc tâm ca
AMN
,
I
là trung điểm cnh
MN
. Gi
Az
là tia phân giác ca
BAC
.
Ta có
HAM OAN
nên
Az
cũng là tia phân giác của
OAH
.
Gi
'O
đối xng vi
O
qua
Az
.
Khi đó
'O
thuc
AH
. Khi
O
thay đổi trên
BC
thì
'O
thay đổi trên đường thng đối xng vi
đường thng
BC
qua
Az
. Tam giác
OIN
0
90 ,I ION BAC
không đổi nên
ON
OI
không
đổi. Mt khác
2.AH OI
nên
OA
OI
không đổi. Do đó
'AO
AH
không đổi. hơn nữa
'O
thuc c
định nên
H
thuc một đường thng
'
song song vi .
Câu 71. Gii:
Gi
, , ,M N P Q
lần lượt là
trung điểm ca
, , ,AB BC CD DA
.
Gi
G
là giao điểm ca
,MP NQ
thì
G
là trung điểm chung ca
c hai đoạn đó.
H
đối xng vi
O
qua
G
,
'
1
H
đối xng vi
A
qua
H
. Ta chng minh
'
11
HH
. Tht vy, ta có
'
1
//MH BH
(
MH
là đường trung bình ca tam giác
'
1
ABH
;
//MH OP
(
MOPH
là hình bình hành);
OP CD
(đường kính đi qua trung điểm dây cung).Suy ra
'
1
BH CD
. Tương tự
'
1
DH BC
, suy ra
'
1
H
trc tâm ca tam giác
BCD
do đó
'
1
HH
. Ly
'O
đối xng vi
O
qua
H
, thì
1
'AOH O
là hình
I
H
O'
O
z
C
B
N
M
A
Q
P
N
M
H
1
O
H
G
D
C
B
A
CHUYÊN ĐỀ BỒI DƯỠNG HC SINH GII HÌNH HC 9
THCS.TOANMATH.com | 56
bình hành. Suy ra
1
'O H OA R
(bán kính ca
O
). Tương tự
234
' ' 'O H O H O H R
.
Vậy ta có đpcm.
Câu 72. Gii:
Ta s chứng minh ba đường
thẳng Ơ-le đó cùng đi qua
trng tâm tam giác
ABC
.
Do tính tương tự, ta ch chng
minh cho tam giác
BCI
.
V phía ngoài tam giác
ABC
, dng tam giác
'A BC
đều, ni tiếp trong đường tròn
1
O
. T giác
'IBA C
ni tiếp vì
0
' 180BA C BIC
. Do
''A B A C
nên
'IA
là phân giác
CIB
, suy ra ba
điểm
, , 'A I A
thng hàng. Gi
F
là trung điểm ca
BC
,
S
1
S
lần lượt là trng tâm ca tam
giác
ABC
và tam giác
IBC
. Vì
11
1
'3
FS FO
FS
FA FI FA
nên ba điểm
11
,,S O S
thng hàng. Mt khác,
11
OS
là đường thẳng Ơ-le ca tam giác
IBC
, do đó đường thẳng Ơ-le ca
tam giác
IBC
đi qua trọng tâm tam giác
ABC
. Chng minh hoàn toàn tương tự vi các tam giác
,IAC IAB
. Ta có đpcm.
Câu 73. Gii:
Ta s dng b đề sau để chng minh.
B đề: cho tam giác nhn
ABC
O
là tâm đường tròn ngoi
tiếp,
H
là trực tâm. Khi đó nếu
AO AH
thì ta có
0
60BAC
.
Tr li bài toán: Gi s bốn điểm
, , ,O I H C
cùng thuc một đường
tròn. Vì
CI
là phân giác ca
HCO
nên
IH IO t
.
Ta chng minh: nếu
0
60BAC
thì bốn điểm
, , ,O I H A
cùng thuc một đường tròn.
Kí hiu
,MN
lần lượt là hình chiếu ca
I
trên
OA
AH
. Lấy hai điểm
12
,OO
nm trên tia
AO
sao cho
12
IO IO t
(
1
O
nm gia
A
M
,
M
nm gia
1
O
2
O
).
O
2
O
1
H
2
H
1
N
M
I
O
H
C
B
A
CHUYÊN ĐỀ BỒI DƯỠNG HỌC SINH GIỎI HÌNH HỌC 9
57 | THCS.TOANMATH.com
Lấy hai điểm
12
,HH
nm trên tia
AH
sao cho
12
IH IH t
(
1
H
nm gia
A
N
,
N
nm
gia
1
H
2
H
).
a) Nếu
1
OO
1
HH
, hoc
2
OO
2
HH
. Khi đó
AIO AIH
suy ra
AO AH
. Áp dng b đề ta được
0
60BAC
, trái với điều gi thiết.
b) Nếu
1
OO
2
HH
hoc
2
OO
1
HH
. Ta có
1 2 1 2
IOO IH H
nên
1 2 2 1
IO O IH H
2 1 1 2
IO O IH H
. Suy ra t giác
AOIH
ni tiếp.
Gi s
A
B
không nằm trên đường tròn ngoi tiếp tam giác
OIH
. Khi đó
0
60BAC ABC
nên tam giác
ABC
đều, suy ra ba điểm
,,O I H
trùng nhau, vô lý. Vy ta có
đpcm.
Câu 74. Gii:
Gi
E
là giao điểm ca
AH
O
,
(E
khác
A
). Giao
điểm ca
MN
EC
F
.
T giác
ABEC
ni tiếp đường
tròn
O
, có
OK MK
nên theo
“bài toán con bướm” ta có
KM KF
(1). Mt khác
MAK BAE HCB
HE BK
nên tam giác
HCE
cân ti
C
,
suy ra
HK KE
(2).T (1) và (2) ta có t giác
MHFE
là hình bình hành, do đó
//MH EF
.
Suy ra
MHK KEF ABC
. Chng
minh hoàn toàn tương tự ta cũng được
NHK ACB
. Ta có
0
180QHP MHN ABC ACB BAC
. Suy ra t giác
AQPH
ni tiếp được.
Câu 75. Gii:
Ta có
0
180ARC APC ABC AHC
. Do đó tứ giác
AHCR
ni tiếp.Suy ra
AHX ACR CAP
.
Tương tự ta cũng có tứ giác
AHBQ
ni tiếp.T đó suy ra
XAH QBH
QBA ABH BAP ABH
(2)
F
E
K
Q
P
N
M
H
O
C
B
A
A
B
C
O
H
Q
X
P
E
R
CHUYÊN ĐỀ BỒI DƯỠNG HC SINH GII HÌNH HC 9
THCS.TOANMATH.com | 58
T (1) và (2) suy ra
AHX XAH
0
90CAP BAP ABH CAB ABH
. Do đó
0
90AXH AEH
hay t giác
AXEH
ni tiếp được. Vy
XEA AHX CAP
(theo (1)). Suy ra
//EX AP
(đpcm).
Câu 76. Gii:
Gi
,MN
lần lượt là trung điểm ca
,BD CE
.
KN
ct
2
,,AB O O
lần lượt ti
,,S P Q
. Ta có
KQC KAC EPQ
. Suy ra
//EP CQ
, mà
N
là trung điểm ca
EC
nên
N
là trung điểm ca
PQ
.
Ta thy:
2 . . .SASM SASD SASB
;
2 . . .SK SN SK SP SK SQ
..SASD SK SP
(t giác
AKPD
ni tiếp);
..SASB SK SQ
(t
giác
AKQB
ni tiếp)
..SASM SK SN
t giác
AKNM
ni tiếp, hay
K
thuộc đường tròn ngoi tiếp tam giác
AMN
. Mt khác t giác
1
AMO N
ni tiếp vì
0
11
90AMO ANO
hay cũng có
1
O
thuộc đường tròn ngoi tiếp tam giác
AMN
. Suy ra t
giác
1
AKNO
ni tiếp
0
11
90AKO ANO
(đpcm).
Câu 77.
Gii: Gi
M
là điểm đối xng ca
B
qua
EF
. Ta có
EMF EBF
.
0
1 2 1 1
180EBF EAF EBF A A EBF E F
nên
0
180EMF EAF
. Vy
t giác
MEAF
ni tiếp đường tròn . Gi
N
là giao điểm ca các tiếp
Q
P
K
S
N
D
E
O
2
O
1
O
C
B
A
M
D
O'
B
A
F
I
M
N
O
C
E
F'
M
E
A
N
CHUYÊN ĐỀ BỒI DƯỠNG HỌC SINH GIỎI HÌNH HỌC 9
59 | THCS.TOANMATH.com
tuyến ti
A
M
ca ta
chứng minh ba điểm
,,N E F
thng hàng. Tht vy, gi
'F
là giao điểm th hai ca
NE
vi . Ta có
'NAE NF A
(g.g) . Suy ra
''
AE NA
AF NF
(1). Tương tự
'''
ME NM NA
MF NF NF
(2). T (1) và (2) ta có
''
AE ME
AF MF
nên
'
'
AE AF
ME MF
(*) Gi
I
là giao điểm ca
AB
EF
. Ta có
22
.IE IAIB IF
do đó
IE IF
. Mà
IEB IAE
(g.g) nên
EB IF IE
AE IA IA
. Tương tự
BF IF
AF IA
. Suy ra
EB BF
AE AF
. Do vy
ME MF
AE AF
hay
AE AF
ME MF
(**). T (*) và (**) ta có
'
'
AF AF
MF MF
suy ra
'FF
. Vậy ba điểm
,,N E F
thng
hàng. Ta có
N
thuộc đường tròn ngoi tiếp tam giác
ABF
. Do vy
N
thuc trung trc
AB
, suy
ra
N
thuộc đường thng
'OO
. Tương tự
2
CD
ct nhau ti một điểm
'N
thuc
'OO
. Do
tính chất đối xng,
CD
EF
ct nhau ti một điểm thuc
'OO
do đó
'NN
. Vy các
đường thng
12
, , ,CD EF
đồng quy ti
N
. (đpcm).
Câu 78. Gii(Bạn đọc có th xem thêm phần’’Các định lý hình hc ni tiếng’’Nội dung định lý
Lyness
Qua
M
k tiếp tuyến chung ca
'O
O
.
Ta có
1
N NMX
11
BM
do đó
NMD NMB
.
Vy
MN
là phân giác góc
DMB
.
Gi
Q
là giao điểm th hai ca
MN
O
. Ta có
Q
là điểm
chính gia ca cung
BD
do đó
CQ
là phân giác góc
DCB
. Gi
I
là giao điểm ca
CQ
NP
.
1
1
1
Q
x
I
O'
O
P
N
D
C
B
A
M
CHUYÊN ĐỀ BỒI DƯỠNG HC SINH GII HÌNH HC 9
THCS.TOANMATH.com | 60
Ta có
đ đ đ đ
1
11
s s s s
22
ICM DM DQ DM QB N IPM
. Suy ra t giác
IPCM
ni tiếp. Do đó
2
.QMB NPA IMC QIM QNI QI QN QM
. Mà
2
.QMD QDN DQN MQD QD QN QM QD QI
. Do đó
I
là tâm đường
tròn ni tiếp tam giác
BCD
. Tương tự tâm đường tròn ni tiếp tam giác
ACD
nm trên
NP
.
Câu 79. Gii:
Gi
'A
là giao điểm th hai ca
AI
vi
O
. Theo câu 78 ta có tâm đường tròn ni tiếp tam giác
ABC
nm trên
IM
(xét vi
1
O
IN
(xét vi
2
O
). Suy ra tâm đường tròn ni tiếp tam giác
ABC
. (đpcm).
Câu 80. Gii:
Ta có
00
1
90 180
2
BMC A B
0
180ZMY B
(vì t giác
MZCY
ni tiếp)
Do đó
BMZ YMC
nên
XBM YNC
. Suy ra
BXM MYC
do đó
KXB LYM
BXK LYM YTC BTN
. Vy t giác
BXTN
ni tiếp.
Tương tự ta cũng có t giác
YCNZ
ni tiếp. Mt khác
BNX BTX BMX
suy ra
YNC YZT YMC
XNY B
.T đó
2BNC BMX YMC B BMX XBM B B B C
do đó
0
180BNC BAC A B C
. Suy ra t giác
ABNC
ni tiếp.
Câu 81. Gii:
a)
ABE ACF
(g.g)
..
AB AE
AE AC AF AB
AC AF
.
b). Ta có
0
180BFH BDH
T giác
BFHD
ni tiếp (t giác có
hai góc đối bù nhau).
N
M
Y
L
T
Z
K
X
C
B
A
M
x
K
D
O
H
N
F
E
C
B
A
CHUYÊN ĐỀ BỒI DƯỠNG HỌC SINH GIỎI HÌNH HỌC 9
61 | THCS.TOANMATH.com
Ta có
0
90ADB AEB
T giác
ABDE
ni tiếp (t giác có hai đỉnh
,DE
cùng nhìn
AB
dưới mt góc vuông).
c). Ta có
0
90BFC BEC
T giác
BFEC
ni tiếp (t giác có hai đỉnh
,FE
cùng nhìn
BC
dưới mt góc vuông)
AEF ABC
. Mà
xAC ABC
(h quả). Do đó
xAC AEF
(hai góc v trí so le trong) nên
//Ax EF
.Li có
OA Ax
. Do đó
OA EF
.
d). Gi
I
là giao điểm ca
AD
EF
. Ta có
ADE ABE FDH DI
là tia phân giác
EDF
. Mà
AD BC
nên có
DK
là dường phân giác ngoài ca
DEF
. Xét
DEF
KF IF
KE IE
(1). Áp dng h qu Talet vào các tam giác:
IAE
/ / :
NF IF
FN AE
AE IE
(2);
KAE
/ / :
KF MF
MF AE
KE AE
(3). T (1),(2),(3) cho
NF MF
NF MF
AE AE
.
Câu 82. Gii:
a). Ta có
AM MC
(
M
là điểm chính gia ca
AC
)
ABM IBM
(h qu góc
ni tiếp).
0
90AMB ACB
(góc ni tiếp chn nửa đường
tròn)
,BM AI AC BI
.
ABI
BM
vừa là đường cao
BM AI
vừa là đường phân giác
ABM IBM
. Do đó
tam giác
ABI
cân ti
B
.
b) Ta có
0
90KMI BM AI
;
0
90KCI AC BI
0 0 0
90 90 180KMI KCI
.Vy t giác
MICK
ni tiếp.
c) Xét
ABN
IBN
AB BI
(
ABI
cân ti
B
),
ABN IBN
(chng minh trên),
BN
cạnh chung. Do đó
ABN IBN
(c.g.c)
NAB NIB
. Mà
0
90NAB
nên
0
90NIB NI BI
. Mà
I
thuộc đường tròn
,B BA
(vì
BI BA
). Vy
NI
là tiếp tuyến
của đường tròn
,B BA
. + Xét
ABC
M
là trung
điểm ca
AI
,
ABI
cân ti
B
,
BM
là đường cao,
O
là trung điểm ca
AB
MO
là đường
trung bình ca tam giác
ABI
//MO BI
. Mà
NI BI
(chng minh trên). Vy
NI MO
.
O
K
D
I
N
M
B
C
A
CHUYÊN ĐỀ BỒI DƯỠNG HC SINH GII HÌNH HC 9
THCS.TOANMATH.com | 62
d) Ta có
IKD IBM
(hai góc ni tiếp cùng chn cung
IK
của đường tròn
IBK
). Mà
1
2
IDA IBA IBM
(
IDA
IBA
là góc ni tiếp và góc tâm cùng chn cung
AI
của đường
tròn
,B BA
,
BN
là tia phân giác ca
IBA
). Do đó
IDK IDA
hai tia
,DK DA
trùng nhau.
,,D K A
thng hàng. Mà
,,C K A
thng hàng nên
, , ,D K A C
thng hàng. Vậy ba điểm
,,A C D
thng hàng.
Câu 83. Gii:a) Ta có
0
90ICD
(góc ni tiếp chn nửa đường tròn
O
).T giác
IHDC
0
90IHD ICD
. Do đó tứ giác
IHDC
ni tiếp đường tròn tâm
M
2IMH ICH
ICH IDH
. Mà
BCA ICH IDH
(hai góc cùng chn
cung
AB
ca
O
).
Do đó
BCA ICH IDH
nên
2BCH ICH
.Ta có
2BCH IMH ICH
. Vy t giác
BCMH
ni tiếp. b) Gi
T
là giao điểm ca
PD
và đường tròn
J
ngoi tiếp
tam giác
HMD T D
.
Xét
PHD
PTM
HPD
(chung),
PHD PTM
(hai góc ni tiếp cùng chn cung
MD
ca
J
).Do đó
PHD PTM
(g.g)
..
PH PD
PM PH PD PT
PT PM
. Chng
minh tương tự
..PM PH PC PB
, nên
..PD PT PC PB
.
Xét
PBD
PTC
PBD
(chung),
PD PB
PC PT
(vì
..PD PT PC PB
). Do đó
PBD PTC
(c.g.c)
PBD PTC
T giác
BCDT
ni tiếp nên
T
thuộc đường tròn
O
. Do đó
TN
. Vậy ba điểm
,,P D N
thng hàng.
Câu 84. Gii:
a)
,AM AN
là các tiếp tuyến của đường tròn
O
(gt)
0
90AMO ANO
. T giác
AMON
0 0 0
90 90 180AMO ANO
T giác
AMON
ni tiếp đường tròn đường kính
OA
.
O
N
P
J
H
M
D
C
B
A
I
CHUYÊN ĐỀ BỒI DƯỠNG HỌC SINH GIỎI HÌNH HỌC 9
63 | THCS.TOANMATH.com
b)
I
là trung điểm ca
BC
(gt)
OI BC
,
0
90AIO I
thuộc đường tròn đường kính
OA
.Ta có
AM AN
(
là các tiếp tuyến ca
O
).
+ Xét đường tròn
AMOIN
AM AN
AM AN AIM AMK
.
Xét
AIM
AMK
IAM
(chung),
AIM AMK
. Do
đó
AIM AMK
(g.g)
2
.
AI AM
AK AI AM
AM AK
.
Xét
AMB
ACM
MAB
(chung),
AMB ACM
(h qu góc to bi tiếp tuyến và dây
cung).Do đó
AMB ACM
(g.g)
2
.
AM AB
AB AC AM
AC AM
.Vy
2
..AK AI AB AC AM
.
c) Ta có
0
90AIO
;
,OA
c định. vy
I
thuộc đường tròn đường kính
OA
. Khi
BM
thì
IM
; khi
BN
thì
IN
. Do vy khi cát tuyến
ABC
thay đổi thì
I
chuyển động trên
cung tròn
MON
của đường tròn đường kính
OA
.
d) Xét đường tròn đường kính
OA
AM AN MIK NIK
IMN
IK
là đường phân giác
IM MK
IN NK
. Do đó
22
IM
IM IN
IN
2
22
3
MK
MK NK MK MN
NK
. Vy khi cát tuyến
ABC
cắt đoạn thng
MN
tại điểm
K
sao cho
2
3
MK MN
thì
2IM IN
.
Câu 85. Gii:
a) Ta có
0
90AHB AH BC
. Do đó
H
thuộc đường tròn
O
.
H
E
đối xng qua
AC
(gt) và
N AC
.
Do đó
AHN AEN
(tính chất đối xng trc)
đ
1
s
2
AHN ADN AN
.
Do đó
AEN ADN
ADE
I
C
K
O
B
N
M
A
O
1
K
I
Q
M
D
O
N
E
H
C
B
A
CHUYÊN ĐỀ BỒI DƯỠNG HC SINH GII HÌNH HC 9
THCS.TOANMATH.com | 64
cân ti
A
.Vy
AD AE
.
b)
0
90ADB
(góc ni tiếp chn
nửa đường tròn),
AH AE
(tính chất đối xng trc) và
AD AE
nên
AD AH
.
+ Xét
0
90ADB ADB
0
90AHB AHB
AD AH
,
AB
(cạnh chung). Do đó
ADB AHB
(cnh huyn- cch góc vuông)
DAB HAB
.
+ Xét
ADM
AHM
,,AD AH DAM HAM AM
(cạnh chung). Do đó
ADM AHM
(c.g.c)
ADM AHM
. Ta có
AHM AHN ADM
. Vy
HA
là tia
phân giác ca
MHN
. c)
H
E
đối xng qua
AC
(gt)
AHC AEC
(tính chất đối xng trc). Mà
0
90AHC
AH BC
nên
0
90AEC
. T
giác
AHCE
0 0 0
90 90 180AHC AEC
nên t giác
AHCE
ni tiếp
, , ,A H C E
cùng thuc một đường tròn. Mt khác
AHM AEM ADM
t giác
AEHM
ni tiếp
, , ,A E H M
cùng thuc một đường tròn
(2). T (1) và (2) ta có năm điểm
, , , ,A E C H M
cùng thuc một đường tròn.
Ta có
0
90ANB
(góc ni tiếp chn nửa đường tròn).
0
90AMC AHC
(
, , , ,A E C H M
cùng
thuc một đường tròn)
ABC
,,CM BN AH
là ba đường cao
(
00
90 , 90 ,AMC ANB AH BC
).Do đó ba đường thng
,,CM BN AH
đồng quy.
d) Xét
ADQ
ABC
ADQ ABC
(hai góc ni tiếp cùng chn cung
AH
ca
O
);
AQD ACB
(hai góc ni tiếp cùng chn
AH
ca
1
O
. Do đó
ADQ ABC
(g.g)
AD DQ
AB BC
. Mà
2
DQ
DI
(
I
là trung điểm ca
DQ
),
2
BC
BK
(
K
là trung điểm ca
BC
) nên
DI DQ
BK BC
+ Xét
ADI
ABK
,
AD DI DQ
ADI ABK
AB BK BC
. + Do đó
ADI ABK
(c.g.c)
AID AKB
T giác
AIHK
ni tiếp. Vy
I
thuộc đường tròn
ngoi tiếp tam giác
AHK
.
Câu 86). Gii:
a). Ta có
0
90ABC ADC
(góc ni tiếp chn nửa đường tròn).Xét
ABC
0
90ABC
0
90ADC ADC
AC
(cnh chung),
AB AD
(
ABD
đều) Do đó
ABC ADC
(cnh huyn cnh góc vuông)
0
30BAC DAC
CHUYÊN ĐỀ BỒI DƯỠNG HỌC SINH GIỎI HÌNH HỌC 9
65 | THCS.TOANMATH.com
0
.sin 2 sin 30CD BC AC BAC a a
;
.cos 3BD AD AB AC BAC a
,
3
22
AD a
DN
,
DNC
vuông ti
2 2 2
D CN DN CD
,
2
2
22
27
24
aa
CN a
nên
7
2
a
CN
.
b)
ABD
đều có
AC
là đường
phân giác nên là đường cao,
đường trung tuyến.
,MN
lần lượt
là trung điểm
,AB AD MN
đường trung bình ca tam giác
//ABD MN BD
.Ta có
, / /AC BD MN BD
0
90MN AC MKC
,
0 0 0
90 , 90 , 90MBC MKC MEC
(
H
là trc tâm ca
CMN
). Do đó
, , , ,B M K E C
cùng
thuc một đường tròn
T
đường kính
MC
. Ta có
KFB KCB
(xét
T
),
KCB ADB
(xét
O
)
KFB ADB
//KF AD
. T giác
KFDN
//KF ND
//KN FD
nên là hình
bình hành
DF KN
.
AMN
22
AB AD
AM AN AM AD
nên
AMN
cân
ti
A
. Mà
AK
là đường phân giác nên cũng là đường cao, đường trung
tuyến
3
2 4 4
MN BD a
KN
. Vy
3
4
a
DF
.
c)
CMN
CK
là đường cao, đường trung tuyến
CMN
cân ti
C
.
Do đó
CK
là tia phân giác
MCE MCK KCE
. Xét đường tròn
T
MCK KCE MK KE KME MFI
. V
Mx
là tiếp tuyến của đường tròn
MIF
xME MFI
Ta có
KME xME
Hai tia
,MK Mx
trùng nhau. Vy
KM
tiếp xúc với đường
tròn ngoi tiếp tam giác
MIF
.
KMI KFM
(g.g)
KM KI
KF KM
, mà
KM KN
nên
KN KI
KF KN
. Ta có
0
/ / , 90NF FM KN AC KN NF KNF
,
KIN KNF
(
,
KI KN
IKN
KN KF
chung)
0
90KIN KNF
. mà
//KF AD
. Vy
0
90IND
.
Câu 87. Gii:
x
F
I
K
E
O
H
N
M
D
C
B
A
F
D
A
M
E
B
O
K
C
H
CHUYÊN ĐỀ BỒI DƯỠNG HC SINH GII HÌNH HC 9
THCS.TOANMATH.com | 66
a)
MAC MDA
b)
MHC MDO
(c.g.c)
,MCH DOH MHC DHO
c)
22
COD CHD
CAD BHD
.
d)
DE
ct
CF
ti
K
.
đ đ đ
00
180 180 s s s
2 2 2
COD CD CE DF
OHD OCD DKF
t giác
DKHF
,
00
180 90KHF KDF KH MO
ti
H
. Mà
AB MO
ti
H
. Nên
,KH AB
trùng
nhau.
Câu 88. Gii:
V
OH MF
ti
H
. T giác
OBDH
ni tiếp,
, , , ,A B H O C
cùng thuc một đường tròn.
AHB AOB BDS
T giác
BDFH
ni tiếp
BDH BFH
ABM BDH BFH
//BM DH
,
//DH GM
(
DH
là đường trung bình ca tam giác
MGF
).
,,M B G
thẳng hàng.Tương tự
,,M C L
thng hàng
//BC GL
. Trên na
mt phng b
BM
có cha
F
v tia
Mx
là tia
tiếp tuyến của đường tròn
O
,
xMB MCD MLG
Mx
là tia tiếp tuyến của đường tròn
MGL
.
Vậy hai đường tròn
O
MGL
tiếp xúc nhau.
T
S
G
x
H
C
L
O
B
E
M
A
D
F
CHUYÊN ĐỀ BỒI DƯỠNG HỌC SINH GIỎI HÌNH HỌC 9
67 | THCS.TOANMATH.com
CHUYÊN ĐỀ BỒI DƯỠNG HỌC SINH GIỎI HÌNH HỌC 9
1
HH9-CHUYÊN ĐỀ 12. T GIÁC NI TIP
A.KIN THC CN NH
- Hai đỉnh cùng nhìn mt cnh
11
AB=
.
A và B là hai đỉnh k nhau cùng nhìn cnh CD
Đặc bit
90CBD CAD= =
A, B là 2 đỉnh k nhau cùng nhìn cnh CD.
- Hai góc đối bù nhau
180AC+ =
CHUYÊN ĐỀ BỒI DƯỠNG HỌC SINH GIỎI HÌNH HỌC 9
2
180BD+ =
Đặc bit
90AC= =
Góc trong = góc ngoài tại đỉnh đối din
A BCx=
BCx
là góc ngoài ti C
Cùng cách đều một điểm
OA=OB=OC=OD
CHUYÊN ĐỀ BỒI DƯỠNG HỌC SINH GIỎI HÌNH HỌC 9
3
Phương pháp giải
a) Phương pháp 1. Chứng minh cho bốn đnh ca t giác cách đều một điểm nào đó.
Cho một điểm I c định và t giác ABCD. Nếu chứng minh được 4 điểm A, B, C, D cách đều điểm
I, tc là
IA IB IC ID= = =
thì điểm I chính là tâm đường tròn đi qua 4 điểm A, B, C, D. Hay nói
cách khác t giác ABCD ni tiếp đường tròn tâm I bán kính IA.
b) Phương pháp 2. Chứng minh t giác có tổng 2 góc đối bng 180º
Cho t giác ABCD. Nếu chứng minh được
180AC+ =
hoc
180BD+ =
thì t giác ABCD ni
tiếp trong một đường tròn.
c) Phương pháp 3. Chứng minh t hai đỉnh cùng k mt cnh cùng nhìn mt cạnh dưới hai
góc bng nhau.
Cho t giác ABCD, nếu chứng minh được rng
DAC
DBC
bng nhau và cùng nhìn cnh DC
thì t giác ABCD ni tiếp đường tròn.
d) Phương pháp 4: Nếu mt t giác có tng s đo hai cặp góc đối din bng nhau thì t giác
đó nội tiếp được trong một đường tròn.
Cho tam giác ABCD. Nếu chứng minh được
A C B D+ = +
thì t giác ABCD cũng nội tiếp trong
một đường tròn.
e) Phương pháp 5: Tứ giác có góc ngoài ti một đỉnh bng góc trong tại đỉnh đối diện đỉnh đó
thì ni tiếp được trong một đường tròn.
Nếu cho t giác ABCD và chứng minh được góc ngoài tại đỉnh A mà bng góc trong tại đỉnh C (tc
là góc C ca t giác đó) thì ABCD cũng nội tiếp đường tròn.
f) Phương pháp 6: Chứng minh bằng phương pháp phản chng
Chú ý: Có th chng minh t giác ABCD là mt trong những hình đặc bit sau: T giác ABCD là
hình thang cân, hình ch nht, hình vuông.
Định lí Pto--mê:
Trong mt t giác ni tiếp, tích của hai đường chéo bng tng các tích ca hai cp cạnh đối din.
CHUYÊN ĐỀ BỒI DƯỠNG HỌC SINH GIỎI HÌNH HỌC 9
4
Chng minh
GT
T giác ABCD ni tiếp (O)
KL
. . .AC BD AB DC AD BC=+
Láy
E BD
sao cho
BAC EAD=
∆DAE ~ ∆CAB (g.g)
..
AD DE
AD BC AC DE
AC CB
= =
(1)
Tương tự: ∆BAE ~ ∆CAD (g.g)
BE AB
CD AC
=
..BE AC CD AB=
(2)
T (1) và (2)
. . . .AD BC ABCD AC DE EB AC + = +
. . .AD BC ABCD AC BD + =
(điều phi chng minh)
B.BÀI MINH HA
Ví d 1. Cho tam giác ABC cân tại A. Các đường cao AD, BE ct nhau ti H.
a) Chng minh t giác CEHD ni tiếp.
b) Chng minh t giác AEDB ni tiếp.
NG DN GII
CHUYÊN ĐỀ BỒI DƯỠNG HỌC SINH GIỎI HÌNH HỌC 9
5
a) Xét t giác CEHD ta có:
90CEH =
(vì BE là đường cao) và
90CDH =
(vì AD là đường cao)
180CEH CDH + =
.
,CEH CDH
là hai góc đối din ca t giác CEHD.
Do đó CEHD là tứ giác ni tiếp.
b) Theo gi thiết: BE là đường cao
90BE AC BEA =
AD là đường cao
90AD BC BDA =
.
Do đó E và D cùng nhìn AB dưới mt góc 90°, suy ra E và D cùng nằm trên đường tròn đường kính
AB. Vy bốn điểm A, E, D, B cùng nm trên một đường tròn hay t giác AEDB ni tiếp.
Ví d 2. Cho tam giác ABC nhn tha mãn
A B C
. Đường tròn ni tiếp tâm I tiếp xúc vi cnh
AB, AC ti M và N. Gi P và Q lần lượt là các giao điểm ca CI, BI với đường thng MN. Chng
minh rng:
a) T giác INQC ni tiếp. b) T giác BPQC ni tiếp.
NG DN GII
CHUYÊN ĐỀ BỒI DƯỠNG HỌC SINH GIỎI HÌNH HỌC 9
6
a) Vì đường tròn (I) tiếp xúc vi AB, AC ti M và N nên
AM AN=
.
CNQ ANM=
(hai góc đối đỉnh)
180
22
A B C
IBC ICB CIQ
+
= = = + =
T giác INQC có hai điểm liên tiếp I và N cùng nhìn cạnh QC dưới các góc bng nhau nên t giác
này ni tiếp được một đường tròn.
b) Vì INQC là t giác ni tiếp nên
INC IQC=
.
Vì AC tiếp xúc với đường tròn (I) ti N nên
IN AC
hay
90INC =
.
Suy ra
90IQC =
. (1)
Chứng minh tương tự câu a) ta có t giác IMPB ni tiếp
90IMB IPB = =
T (1) và (2) suy ra:
90BPC BQC= =
nên t giác BPQC ni tiếp đường tròn đường kính BC.
Ví d 3. Cho hình bình hành ABCD, có tâm là O. Gi M, N, P lần lượt là hình chiếu vuông góc ca
C lên BD, AD, AB. Chng minh t giác NMOP là t giác ni tiếp.
NG DN GII
CHUYÊN ĐỀ BỒI DƯỠNG HỌC SINH GIỎI HÌNH HỌC 9
7
Ta có:
90CPA CNA= =
(gt) nên t giác ANCP ni tiếp đường tròn (O) đường kính AC.
Suy ra
2PON PCN=
.
Li có
180PCN NAP+=
180PCN NAP ABC = =
(do AD // BC)
Do đó
2PON ABC=
(1)
Mt khác
( )
180PMN PMB NMD= +
Mà t giác CDNM ni tiếp đường tròn đường kính CD nên:
90 90NMD NCD CDN ABC= = =
.
Li có t giác BCMP ni tiếp đường tròn đường kính BC nên:
90PMB PCB ABC= =
( )
180 90 90 2PMN ABC ABC ABC = + =
(2)
T (1) và (2) suy ra:
PON PMN=
do đó tứ giác PMON ni tiếp.
Ví d 4. Cho hai đường tròn (O) và (O’) cắt nhau ti M, N. Tiếp tuyến ti M ca (O) cắt (O’) tại B,
tiếp tuyến ti M của (O’) cắt (O) ti A. Gọi P là điểm đối xng ca M qua N. Chng minh t giác
MAPB ni tiếp.
NG DN GII
Cách 1:
CHUYÊN ĐỀ BỒI DƯỠNG HỌC SINH GIỎI HÌNH HỌC 9
8
Ta có:
,AMN MBN MAN BMN==
Nên ∆AMN ~ ∆MBN (g.g)
Do đó
AN MN AN NP
MN BN NP BN
= =
∆ANP ~ ∆PNB (c.g.c)
NAP NPB=
(hai góc tương ứng)
T đó suy ra
MAP MAN NAP=+
180PMB MPB MBP= + =
Vy t giác AMBP là t giác ni tiếp.
Cách 2:
Gọi K là điểm đối xng ca M qua trung điểm của OO’.
CHUYÊN ĐỀ BỒI DƯỠNG HỌC SINH GIỎI HÌNH HỌC 9
9
Ta có t giác OMO’K là hình bình hành nên OM // O'K, O'M // OK.
Mt khác do
,OM MB O M MA
⊥⊥
nên
,O K MB OK MA
⊥⊥
.
Vậy OK, O’K là các đường trung trc ca MA, MB nên
KA KB KM==
(1)
Mà ta d dàng chứng minh được KN // OO’,
OO MN KN MN
.
Do
MN NP=
nên tam giác KMP cân ti K, suy ra
KM KP=
(2)
T (1) và (2) suy ra
KA KB KM KP= = =
.
Vy t giác AMBP là t giác ni tiếp.
Ví d 5. Hai đường tròn
( )
1
O
( )
2
O
ct nhau ti M và P. V dây MA của đường tròn
( )
2
O
tiếp tuyến của đường tròn
( )
1
O
. V dây MB của đường tròn
( )
1
O
là tiếp tuyến của đường tròn
( )
2
O
. Trên tia đối ca tia PM lấy điểm H sao cho
PH PM=
. Chng minh rng t giác MAHP ni
tiếp.
NG DN GII
Tìm cách gii.
- Khai thác gi thiết, ta có
PH PM=
. Do vy nếu I, K là trung điểm MB, MA mà MIPK là t giác
ni tiếp thì MAHB cũng là tứ giác ni tiếp.
- Ta biết rằng tâm đường tròn ngoi tiếp là giao điểm các đường trung trc ca các cạnh và đường
chéo. D nhn biết đường trung trc của MB đi qua
1
O
, đường trung trc của MA đi qua
2
O
. Nếu
giao điểm của hai đường trung trc này nằm trên đường trung trc ca MH thì bài toán xong.
Với hai định hướng trên ta có hai cách gii sau:
Trình bày NG DN GII
Cách 1. Gi I, K lần lượt là trung điểm ca MB, MA.
Ta có:
BMP MAP=
;
AMP MBP=
suy ra
MBP AMP
(g.g)
BP MB BP BI
MP AM MP MK
= =
BPI MPK
(c.g.c)
BPI MPK=
Xét
IMK IPK IMP PMK IPM MPK+ = + + +
180IMP MBP IPM BPI= + + + =
Suy ra t giác MIPK ni tiếp, mà IP, KP lần lượt là
CHUYÊN ĐỀ BỒI DƯỠNG HỌC SINH GIỎI HÌNH HỌC 9
10
đường trung bình ca tam giác MBH, MAH
//IP BH
,
//KP AH
180KPI AHB AMB AHB = + =
T giác MAHP ni tiếp
Cách 2. Dng hình bình hành
12
OMO O
Suy ra
12
//OO MO
,
21
//O O OM
2
MB MO
,
1
MA MO
nên:
1
OO MB
,
2
O O MA
.
Do đó
OM OB=
;
OM OA=
( )
1
Gọi giao điểm
12
OO
vi MO; MP là I, K
Ta có
12
OO MP
IM IO=
;
KM KP=
Do đó IK là đường trung bình ca tam giác MOP
Suy ra
//IK OP OP MH⊥
MP PH=
OP là đường trung trc ca
MH OM OH=
( )
2
T
( )
1
và
( )
2
OM OA OH OB = = =
T giác MAHB ni tiếp.
Ví d 6. Cho tam giác ABC vuông ti C. Trên cnh AB ly điểm M (M khác A và B). Gi O;
1
O
;
2
O
ln t là tâm của các đường tròn ngoi tiếp các tam giác ABC, AMC và BMC.
1) Chng minh bốn điểm C,
1
O
, M,
2
O
cùng nm trên một đường tròn
( )
T
.
2) Chng minh rằng đường tròn
( )
T
đi qua O.
3) Xác định v trí của M trên đoạn AB sao cho đường tròn
( )
T
có bán kính nh nht.
(Tuyển sinh 10, chuyên toán ĐHSP Hà Nội, năm học 2008 - 2009)
NG DN GII
Tìm cách gii.
1
O
,
2
O
lần lượt là tâm đường tròn ngoi tiếp nên
1
CO M
;
2
CO M
lần lượt hai góc tâm ca hai
đường tròn tròn tương ứng. Phân tích đi lên ta có bốn điểm C,
1
O
, M,
2
O
cùng nm trên một đường
tròn
12
180CO M CO M + =
90CAM CBM + =
t đó ta tìm được cách gii.
• Để chứng minh đường tròn
( )
T
đi qua điểm O, ta cn chng minh t giác
2
CO OM
ni tiếp hoc
t giác
1
CO MO
ni tiếp. C hai hướng trên đều cho NG DN GII đúng.
Trình bày NG DN GII
CHUYÊN ĐỀ BỒI DƯỠNG HỌC SINH GIỎI HÌNH HỌC 9
11
1) S dng tính cht góc tâm đường tròn, ta có:
1
2.CO M CAM=
;
2
2.CO M CBM=
.
Do tam giác ABC vuông nên:
90CAM CBM+ =
.
Suy ra
12
180CO M CO M+ =
.
Vy bốn điểm C,
1
O
, M,
2
O
cùng thuc một đường
tròn
2) Do tam giác ABC vuông ti C nên O là trung
điểm ca AB,
Gi s M thuộc đoạn OA.
Do tam giác COB cân ti O nên
2
2. .COM CBO CO M==
Vy O thuộc đường tròn
( )
T
3) Gi R là bán kính của đường tròn
( )
T
.
Do
( )
T
đi qua C và O nên
2CO R
hay
1
2
R CO
Du bằng đạt được khi M là hình chiếu ca C trên AB.
Vy bán kính của đường tròn (T) nh nht bng
1
2
CO
khi M là hình chiếu ca C trên AB.
Ví d 7. T điểm A ngoài đường tròn
( )
O
, k các tiếp tuyến AB, AC với đường tròn (B, C là các
tiếp điểm). Trên tia đối ca tia BC lấy điểm D. Gọi E là giao điểm ca DO và AC. Qua E v tiếp
tuyến th hai với đường tròn
( )
O
, tiếp tuyến này cắt đường thng AB ti K. Chng minh bốn điểm
D, B, O, K cùng thuc một đường tròn.
NG DN GII
Tìm cách gii. Da vào hình v ta có mt s định hướng sau:
Nếu gi M là tiếp điểm của đường tròn
( )
O
vi EK, d thy BOMK là t giác ni tiếp. Nên mun
chng minh D, O, K, B cùng thuc một đường tròn, ta ch cn chng minh D, O, M, B cùng thuc
một đường tròn.
Quan sát k, ta có
2
K
BKO =
. Vy ta ch cn chng minh
2
K
BDO =
.
Cũng dễ nhn thy
180
2
A
DBK ABC
−
==
.
CHUYÊN ĐỀ BỒI DƯỠNG HỌC SINH GIỎI HÌNH HỌC 9
12
Do đó ta cũng cần chng minh
180
2
A
DOK
−
=
.
Trình bày NG DN GII
Cách 1. Gi M là tiếp điểm của đường tròn
( )
O
vi EK.
Ta có EM, EC là tiếp tuyến ca
( )
O
nên:
1
2
MOE COE MOC==
1
2
MBC MOC MOE MBC= =
.
Mt khác
180MOE MOD+ =
180MBC MBD+ =
Suy ra
MOD MBD=
Vy D, O, M, B cùng thuc một đường tròn.
90KMO KBO= =
nên t giác KMOB ni tiếp.
Vậy năm điểm D, K, O, M, B cùng thuc một đường tròn
Suy ra D, O, K, B cùng thuc một đường tròn.
Cách 2. Ta có
180 180 90
22
AE
CDE DCE DEC= =
90
22
A E K
CDE
+
= =
2
K
BKO BKO BDO= =
.
Suy ra D, O, K, B cùng thuc một đường tròn.
Cách 3. Tam giác OEK có
2
KE
DOK OKE OEK
+
= + =
(tính cht góc ngoài tam giác).
Suy ra
180
2
A
DOK
−
=
Mt khác, ta có:
180
2
A
DBK ABC
−
==
Do đó:
DBK DOK=
Vy D, O, K, B cùng thuc một đường tròn.
CHUYÊN ĐỀ BỒI DƯỠNG HỌC SINH GIỎI HÌNH HỌC 9
13
Ví d 8. Cho đường tròn tâm O đường kính
2AB R=
và C là điểm chính gia cung AB. Lấy điểm
M tùy ý trên cung BC (M khác B). Gọi N là giao điểm ca hai tia OC và BM; H, I lần lượt là trung
điểm của các đoạn thẳng AO, AM; K là giao điểm các đường thng BM và HI.
a) Chng minh rng A, H, K và N cùng nm trên một đường tròn;
b) Xác định v trí của điểm M trên cung BC (M khác B) sao cho
10
2
R
AK =
(Thi hc sinh gii lp 9, TP. Hà Nội, năm học 2088 - 2019)
NG DN GII
Tìm cách gii. D dàng nhn thấy HI là đường trung bình ca
AOM
nên
//HI OM
suy ra
KHB MOB=
;
HKB OMB=
. Do vậy để chng minh A, H, K và N cùng nm trên một đường tròn
ta ch cn chng minh
NAO HKB=
hoc
ANB KHB NAO OMB= =
hoc
ANB MOB=
.
T đó ta có cách giải sau:
Trình bày NG DN GII
a) Ta có tam giác NAB cân ti N (ON là trung trc ca AB)
NAB NBA=
( )
1
Li có:
OM OB R==
.
NBA BMO=
( )
2
.
Do H, I là trung điểm của OA, AM nên HI là đường trung bình
ca tam giác AOM.
Suy ra
//HI OM BMO HKM=
( )
3
T
( )
1
,
( )
2
( )
3
, suy ra:
NAB HKB=
.
Do đó tứ giác AHKN ni tiếp hay A, H, K, N cùng thuc mt
đường tròn.
b) Ta có:
22
AO R
AH HO= = =
.
1
//
22
KM OH R
HK OM
KB OB R
= = =
Đặt
2
x
MB x MK= =
Áp dng định lí Py-ta -go trong các tam giác vuông AKM và AMB, ta có:
2 2 2 2 2
AK KM AM AB BM = =
2
2
22
10
42
24
Rx
R x x R

= =



CHUYÊN ĐỀ BỒI DƯỠNG HỌC SINH GIỎI HÌNH HỌC 9
14
10 2
sin
22
MB R
MAB
AB R
= = =
45 90MAB sdBM M C = =
.
CHUYÊN ĐỀ BỒI DƯỠNG HỌC SINH GIỎI HÌNH HỌC 9
15
C.BÀI TP RÈN LUYN
Bài 1. Cho hình hành ABCD có đỉnh D nằm trên đường tròn đường kính AB. K BN và DM cùng
vuông góc với đường chéo AC. Chng minh rng:
a) T giác CBMD là t giác ni tiếp.
b) Khi điểm D di động trên đường tròn thì
BMD BCD+
không đổi.
c)
..DB DC DN AC=
.
Bài 2. Cho hai đường tròn
( )
O
( )
O
ct nhau ti A và B. Các tiếp tuyến ti A của đường tròn
( )
O
( )
O
cắt đường tròn
( )
O
( )
O
theo th t ti C và D. Gi P và Q lần lượt là trung điểm
ca các dây AC và AD. Chng minh rng:
a) Hai tam giác ABD và CBA đồng dng.
b)
BQD APB=
.
c) T giác APBQ ni tiếp.
Bài 3. Cho tam giác ABC nhn ni tiếp đường tròn
( )
O
. Đường tròn ngoi tiếp tam giác OBC ct
AB và AC th t ti D và E. Chng minh AO vuông góc vi DE.
Bài 4. Cho hai vòng tròn
( )
1
O
( )
2
O
tiếp xúc ngoài nhau tại điểm T. Hai vòng tròn này nm
trong vòng tròn
( )
3
O
và tiếp xúc vi
( )
3
O
tương ứng ti M và N. Tiếp tuyến chung ti T ca
( )
1
O
( )
2
O
ct
( )
3
O
ti P. PM ct vòng tròn
( )
1
O
tại điểm th hai A và MN ct
( )
1
O
tại điểm th hai
B. PN ct vòng tròn
( )
2
O
tại điểm th hai D và MN ct
( )
2
O
tại điểm th hai C.
a) Chng minh rng t giác ABCD là t giác ni tiếp.
b) Chng minh rằng các đường thẳng AB, CD và PT đồng quy.
Bài 5. T điểm A nằm ngoài đường tròn tâm O k hai tiếp tuyến AB và AC (B và C là các tiếp
điểm). Gọi M là điểm bt kì trên cung nh BC của đường tròn
( )
O
(M khác B và C). Tiếp tuyến
qua M ct AB và AC tại E và F. Đường thng BC ct OE và OF P và Q. Chng minh rng:
a) T giác OBEQ, OCFP là các t giác ni tiếp.
b) T giác PQFE là t giác ni tiếp.
c) T s
PQ
FE
không đổi khi M di chuyển trên đường tròn.
Bài 6. Cho tam giác ABC, D và E là các tiếp điểm của đường tròn ni tiếp vi các cnh AB và AC.
Chứng minh đường phân giác trong của góc B, đường trung bình ca tam giác song song vi cnh
AB và đường thẳng DE đồng quy.
CHUYÊN ĐỀ BỒI DƯỠNG HỌC SINH GIỎI HÌNH HỌC 9
16
Bài 7. Cho đưòng tròn
( )
;OR
đường kính AB c định và đường kính CD quay quanh điểm O. Các
đường thng AC và AD ct tiếp tuyến ti B của đường tròn theo th t ti E và F.
1. Chng minh rng t giác CDFE ni tiếp đường tròn.
2. Gọi I là tâm đường tròn ngoi tiếp t giác CDFE. Chng minh rằng điểm I di động trên đường
thng c định khi đường kính CD quay quanh điểm O.
(Thi Hc sinh gii lp 9, tỉnh Trà Vinh, năm học 2009 - 2010)
Bài 8. Cho tam giác ABC vuông ti A và D là một điểm trên cnh AC (Khác vi A và C). V
đường tròn tâm D tiếp xúc vi BC ti E. T B k tiếp tuyến th hai BF với đường tròn
( )
D
. Gi M
là trung điểm của BC, N là giao điểm ca BF và AM. Chứng minh năm điểm A, B, E, D, F cùng
nm trên một đường tròn và
AN NF=
.
(Thi Hc sinh gii lp 9, tỉnh Vĩnh Long, năm học 2019 -2020)
Bài 9. Cho hai đường tròn
( )
;OR
( )
;OR

ct nhau tại hai điểm phân bit A và B. T một điểm
C thay đổi trên tia đối ca tia AB, v các tiếp tuyến CD, CE với đường tròn tâm O (D; E là các tiếp
điểm và E nằm trong đường tròn tâm
O
). Hai đường thng AD và AE cắt đường tròn tâm
O
ln
t ti M và N (M, N khác với điểm). Đường thng DE ct MN ti 1. Chng minh rng:
a)
..MI BE BI AE=
.
b) Khi điểm C thay đổi thì đường DE luôn đi qua một điểm c định.
(Thi Hc sinh gii lp 9, tnh Ngh An, năm học 2009 - 2010)
Bài 10. Cho tam giác ABC vuông tại A. Đường tròn tâm I ni tiếp tam giác ABC, tiếp xúc vi CA
và CB lần lượt tại M và N. Đường thng MN cắt đường thng AI ti P. Chng minh rng góc IPB
vuông.
(Thi Hc sinh gii lp 9, tỉnh Gia Lai, năm học 2009 - 2010)
Bài 11. Cho đường tròn
( )
;OR
và dây AB c định,
2AB R=
. Điểm P di động trên dây AB (P
khác A và B). Gi
( )
1
;CR
là đường tròn đi qua P và tiếp xúc với đường tròn
( )
;OR
ti A,
( )
2
;DR
là đường tròn đi qua P và tiếp xúc vi
( )
;OR
tại B. Hai đường tròn
( )
1
;CR
( )
2
;DR
ct nhau ti
điểm th hai M.
a) Trong trường hp P không trùng với trung điểm dây AB, chng minh
//OM CD
và 4 điểm C, D,
O, M cùng thuc một đường tròn;
b) Chng minh khi P di động trên dây AB thì điểm M di động trên đường tròn c định và đường
thẳng MP luôn đi qua một điểm c định N;
c) Tìm v trí ca P để tích PM.PN ln nht? Din tích tam giác AMB ln nht.
(Thi Hc sinh gii lp 9, tnh Phú Thọ, năm học 2009 - 2010)
CHUYÊN ĐỀ BỒI DƯỠNG HỌC SINH GIỎI HÌNH HỌC 9
17
Bài 12. Cho tam giác ABC
( )
AB AC
ni tiếp đường tròn
( )
O
có AD là phân giác góc
BAC
, tia
AD cắt đường tròn tại điểm E (E khác A). K đường kính EF của đường tròn
( )
O
. Gi P là mt
điểm nm gia A và D. Tia FP cắt đường tròn
( )
O
ti Q khác F. Đường thng qua P vuông góc vi
AD ct CA, AB lần lượt ti M, N.
a) Chng minh rng các t giác PQBN, PQCM là t giác ni tiếp.
b) Gi s QN và PC ct nhau ti một điểm thuộc đường tròn
( )
O
. Chng minh rng QM và PB
cũng cắt nhau ti một điểm thuộc đường tròn
( )
O
.
Bài 13. Cho tam giác ABC có 3 góc nhn ni tiếp
( )
;OR
AB AC
. V 3 đường cao AD, BE,
CF ca tam giác ABC ct nhau ti H, AD ct
( )
O
ti K và ct EF ti I.
a) Chng minh rng: BC là trung trc ca HK và
..IF IE IH IA=
;
b) Chng minh rng : Các t giác DHEC, BFIK ni tiếp được;
c) Chng minh rng:
KC BK EF
AC BA AI
+=
;
d) Đưng thng qua E song song vi AD ct BK ti M. Chng minh rằng: 3 điểm F, D, M thng
hàng;
Bài 14. Cho tam giác ABC nhn vi
AB AC
có AD là đường phân giác. Đường thng qua C
song song vi AD cắt đường trung trc ca AC tại E. Đường thng qua B song song vi AD ct
đường trung trc ca AB ti F.
a) Chng minh rằng tam giác ABF đồng dng vi tam giác ACE.
b) Chng minh rằng các đường thng BE, CF, AD đồng quy ti một điểm, gọi điểm đó là G.
c) Đưng thng qua G song song vi AE cắt đường thng BF ti Q. Đường thng QE cắt đường
tròn ngoi tiếp tam giác GEC và P khác E. Chng minh rằng các điểm A, P, G, Q, F cùng thuc mt
đường tròn.
Bài 15. Cho tam giác ABC có
BAC
là góc ln nhất. Các điểm P, Q thuc cnh BC sao cho
QAB BCA=
CAP ABC=
. Gi M, N lần lượt là các điểm đối xng ca A qua P; Q. Chng minh
rng BN và CM ct nhau ti một điểm thuộc đường tròn ngoi tiếp tam giác ABC.
(Thi vô địch Toán Quc Tế IMO, năm 2014)
Bài 16. Cho 19 điểm nm trong hay trên cnh ca mt lục giác đều cnh bng 4 cm. Chng minh
rng luôn tn ti 2 trong s 19 điểm đã cho mà khoảng cách gia chúng không vượt quá
43
3
cm.
(thi hc sinh gii lp 9, tnh Hưng Yên, năm học 2013 - 2014)
CHUYÊN ĐỀ BỒI DƯỠNG HỌC SINH GIỎI HÌNH HỌC 9
18
Bài 17. Cho hình thang ABCD vuông góc tại A và B, M là trung điểm của AB. Đường thng qua A
vuông góc vi MD cắt đường thng qua B vuông góc vi MC ti N. Chng minh
MN CD
.
(tuyn sinh lớp 10 chuyên Toán. Đại học sư phạm TP. H Chí Minh, năm học 2015 - 2016)
Bài 18. Cho tam giác ABC
( )
AB AC
có các góc nhn, ni tiếp trong đường tròn tâm O. Gi M là
trung điểm cnh BC, E là điểm chính gia ca cung nh BC, F là điểm đối xng ca E qua M.
a) Chng minh rng
2
.EB EF EO=
;
b) Gọi D là giao điểm ca AE và BC. Chứng minh các điểm A, D, O, F cùng thuc một đường tròn.
c) Gọi I là tâm đường tròn ni tiếp tam giác ABC và P là điểm thay đổi trên đường tròn ngoi tiếp
tam giác IBC sao cho P, O, F không thng hàng. Chng minh rng tiếp tuyến ti P của đường tròn
ngoi tiếp tam giác POF đi qua một điểm c định.
(tuyn sinh lớp 10, trường ph thông Năng khiếu Đại hc Quc gia TP. H Chí Minh, năm học
2015-2016)
Bài 19. Cho tam giác nhn ABC
( )
AB AC
, M là trung điểm cnh BC, O là tâm của đưòng tròn
ngoi tiếp tam giác. Các đường cao AD, BE, CF của tam giác ABC đồng quy ti H. Các tiếp tuyến
vi
( )
O
ti B và C ct nhau ti S. Gi X, Y lần lượt là giao điểm của đường thng EF vi các
đường thng BS, AO. Chng minh rng:
a)
MX BF
.
b) Hai tam giác SMX và DHF đồng dng.
c)
EF BC
FY CD
=
.
(tuyn sinh lớp 10, THPT chuyên, Đại học sư phạm Hà Nội, năm học 2015 - 2016)
Bài 20. Cho tam giác đều ABC ni tiếp đường tròn
( )
O
, H là trung điểm của BC. M là điểm bt kì
thuộc đoạn thng BH (M khác B). Lấy điểm N thuộc đoạn thng CA sao cho
CN BM=
. Gi I là
trung điểm ca MN.
1) Chng minh bốn điểm O, M, H, I cùng thuc một đường tròn.
2) Gi P là giao điểm ca OI và AB. Chứng minh tam giác MNP là tam giác đều.
3) Xác định v trí của điểm M để tam giác IAB có chu vi nh nht.
(tuyn sinh lp 10, THPT chuyên, TP. Hà Nội, năm học 2014 - 2015)
Bài 21. Cho tam giác ABC ni tiếp đường tròn
( )
O
và điểm P nm trong tam giác tha mãn
PB PC=
. D là điểm thuc cnh BC (D khác B và D khác C) sao cho P nằm trong đường tròn ngoi
tiếp tam giác DAB và đường tròn ngoi tiếp tam giác DAC. Đường thng PB cắt đường tròn ngoi
tiếp tam giác DAB tại E khác B. Đường thng PC cắt đường tròn ngoi tiếp tam giác DAC ti F
khác C.
CHUYÊN ĐỀ BỒI DƯỠNG HỌC SINH GIỎI HÌNH HỌC 9
19
1) Chng minh bn điểm A, E, P, F cùng nm trên một đường tròn .
2) Gi s đường thng AD cắt đường tròn
( )
O
ti Q khác A, đường thng AF cắt đường thng QC
ti L. Chứng minh tam giác ABE đồng dng vi tam giác CLF.
3) Gọi K là giao điểm của đường thẳng AE và đường thng QB. Chng minh:
QKL PAB QLK PAC+ = +
.
(tuyn sinh lớp 10, THPT chuyên, ĐHKHTN, Đại hc Quc Gia Hà Nội, năm học 2014 - 2015)
Bài 22. Cho đường tròn
( )
;OR
và dây cung BC không đi qua tâm. Gọi A là điểm chính gia ca
cung nh BC. Góc ni tiếp
EAF
quay quanh điểm A và có s đo bằng
không đổi sao cho E, F
khác phía với điểm A so vi BC; AFAE cắt đường thng BC lần lượt ti M N. Ly điểm D sao
cho t giác MNED là hình bình hành.
a) Chng minh MNEF là t giác ni tiếp.
b) Gi I là tâm đường tròn ngoi tiếp tam giác MDF. Chng minh rng khi góc ni tiếp
EAF
quay
quanh điểm A thì I chuyển động trên một đường thng c định.
c) Tìm độ dài nh nht của đoạn thng OI khi
60
=
BC R=
.
(thi hoc sinh gii lp 9, tnh Phú Thọ, năm học 2013 - 2014)
Bài 23. Cho 3 điểm A, B, C c định nm trên một đường thng d (B nm gia A và C). V đường
tròn tâm O thay đổi nhưng luôn đi qua B và C (O không thuộc đường thng d). K AM và AN là
các tiếp tuyến với đường tròn tâm O ti M và N. Gọi I là trung điểm ca BC, AO ct MN ti H và
cắt đường tròn tại các điểm P và Q (P nm gia A và O), BC ct MN ti K.
1. Chứng minh 4 điểm O, M, N, I cùng nm trên một đường tròn.
2. Chứng minh điểm K c định khi đường tròn tâm O thay đổi.
3. Gọi D là trung điểm ca HQ, t H k đưng thng vuông góc vi MD cắt đường thng MP ti E.
Chứng minh P là trung điểm ca ME.
(thi hc sinh gii lp 9, tinh Thanh Hóa, năm học 2017 - 2018)
Bài 24. Từ một điểm A ở ngoài đường tròn (O) ta vẽ hai tiếp tuyến AB, AC với đường tròn.
Lấy điểm D nằm giữa B và C. Qua D vẽ một đường thẳng vuông góc với OD cắt AB, AC
lần lượt tại E và F. Khi D di động trên BC, chứng minh rằng tứ giác AEOF luôn là tứ giác
nội tiếp.
Bài 25. T giác ABCD có hai đường chéo vuông góc vi nhau ti O. M, N, P, Q lần lượt là hình
chiếu ca O trên AB, BC, CD, AD. Chng minh t giác MNPQ ni tiếp.
CHUYÊN ĐỀ BỒI DƯỠNG HỌC SINH GIỎI HÌNH HỌC 9
20
Bài 26. Cho hai đường tròn (O) và (O’) ngoài nhau. Các tiếp tuyến chung ngoài AB và CD sao cho
( ) ( )
, ; ,A C O B D O

. Ni AD cắt (O), (O’) lần lượt ti E và F. Chng minh rng t giác ABCD
là t giác ni tiếp.
Bài 27. thi vào lớp 10 Chuyên Lê Quý Đôn - Bình Định năm học 2018-2019 vòng 1)
Cho tam giác ABC
( )
AB AC
có các góc đều nhn ni tiếp trong đường tròn tâm O. AD là đường
kính của đường tròn (O), H là trung điểm BC. Tiếp tuyến ti D ca (O) cắt đường thng BC ti M.
Đưng thng MO ct AB, AC lần lượt ti E và F.
a) Chng minh
2
.MD MB MC=
b) Qua B k đường thng song song vi MO cắt đường thng AD ti P. Chng minh t giác BHPD
là t giác ni tiếp.
c) Chứng minh O là trung điểm ca EF.
Bài 28. Cho tam giác ABC nhn tha mãn
AB AC
. Hai đường cao BD và CE ct nhau ti H
( )
,D AC E AB
. Gọi I là trung điểm của BC. Đường tròn ngoi tiếp ∆BEI và đường tròn ngoi
tiếp ∆CDI cắt nhau ti K
( )
KI
.
a) Chng minh rng:
BDK CEK=
b) Đường thng DE ct BC ti M. Chứng minh ba điểm M, H, K thng hàng.
c) Chng minh rng t giác BKMD ni tiếp.
Bài 29. thi vào lớp 10 Chuyên Ninh Bình năm học 2017- 2018)
Cho hai đường tròn (O) và (O’) cắt nhau tại hai điểm A, B (O, O’thuộc hai na mt phng b AB).
Tiếp tuyến chung gn B của hai đường tròn lần lượt tiếp xúc với (O) và (O’) tại C, D. Qua A k
đường thng song song vi CD lần lượt cắt (O) và (O’) tại M, N (M, N khác A). Các đường thng
CM, DN ct nhau ti E. Gi P và Q lần lượt là giao ca đường thng MN với đường thng BC và
đường thng BD. Chng minh rng:
a) Đường thng AE vuông góc với đường thng CD.
b) T giác BCED ni tiếp.
c) Tam giác EPQ là tam giác cân.
CHUYÊN ĐỀ BỒI DƯỠNG HỌC SINH GIỎI HÌNH HỌC 9
21
Bài 30. thi vào lp 10 Chuyên Bắc Giang năm học 2018-2019)
Cho tam giác ABC nhn ni tiếp đường tròn (O) vi
AB AC
. Gọi M là điểm thuc cnh BC (M
không trùng với B và C), đường thng AM cắt đường tròn (O) tại điểm D khác A. Đường tròn ngoi
tiếp tam giác MCD cắt đường thng AC tại điểm E khác C. Đường tròn ngoi tiếp tam giác MBD
cắt đường thng AB tại điểm F khác B.
a) Chng minh rng t giác BECF ni tiếp được trong một đường tròn.
b) Chng minh rằng hai tam giác ECD, FBD đồng dạng và ba điểm E, M, F thng hàng
Bài 31. thi vào lớp 10 Chuyên Sư phạm Hà Nội năm học 2017-2018 vòng 2)
Cho đường tròn (O) bán kính R và điểm M nằm bên ngoài đường tròn (O). K các tiếp tuyến MA,
MB tới đường tròn (O) (A, B là các tiếp điểm). Trên đoạn thng AB lấy điểm C (C khác A, khác B).
Gi I, K lần lượt là trung điểm ca MA, MC. Đường thng KA cắt đường tròn (O) tại điểm th hai
D.
1. Chng minh
2 2 2
KO KM R−=
2. Chng minh t giác BCDM là t giác ni tiếp.
3. Gọi E là giao điểm th hai của đường thng MD với (O) và N là trung điểm của KE. Đường
thng KE cắt đường tròn (O) tại điểm th hai F. Chng minh rng t giác ANFI là t giác ni tiếp.
CHUYÊN ĐỀ BỒI DƯỠNG HỌC SINH GIỎI HÌNH HỌC 9
22
NG DN
1.
a) AB là đường kính đường tròn
( )
90ADBO =
ADB DBC=
(so le trong)
90DBC =
.
Mt khác
90DMC =
suy ra:
90DMC DBC= =
do đó tứ giác CBMD ni tiếp đường tròn đường
kính CD.
Nhn xét. Ngoài cách gii trên, chúng ta có th gii
theo hướng sau:
Ta có:
MDB DBN DAN MCB===
.
Suy ra điều phi chng minh.
Ta có:
DMB DNB=
;
DAB DCB=
180DAB DNB+ =
.
Suy ra điều phi chng minh.
b) Khi điểm D di động trên đường tròn
( )
O
thì t
giác CBMD luôn là t giác ni tiếp
Suy ra
180BMD BCD+ =
(điều phi chng
minh).
c) Do
90ANB =
thuc
( )
O
.
Ta có:
BDN BAN=
(góc ni tiếp) mà
ACD BAN=
(so le trong)
BDN ACD=
.
Mt khác
DAC DAN DBN==
(cùng chn cung DN)
Suy ra:
ACD BDN
(g.g)
..
AC CD
AC DN BD CD
BD DN
= =
2.
a) Áp dng h qu góc to bi tia tiếp tuvến
và dây cung, ta có:
CAB ADB=
,
ACD BAD=
Suy ra:
ABD CBA
(g.g).
b) Vì
ABD CBA
, suy ra:
AD BD
CA BA
=
2
AD
DQ =
;
2
AC
AP =
CHUYÊN ĐỀ BỒI DƯỠNG HỌC SINH GIỎI HÌNH HỌC 9
23
BD DQ
BA AP
=
Li có:
QDB PAB=
Suy ra:
BQD APB
(c.g.c)
BQD APB=
.
c) Ta có:
180AQB BQD+ =
, mà
180BQD APB AQB APB= + =
Suy ra t giác APBQ ni tiếp.
3. Tia AO ct DE ti H.
Vì O là tâm đường tròn ni tiếp AABC nên
2.AOC B=
.
Suy ra
( )
1
180
2
OAC OCA AOC= =
90 B=
( )
1
T giác BDEC ni tiếp nên
( )
2
Kết hp
( )
1
( )
2
ta có:
90 90OAC HAE B AED= = =
.
Suy ra:
90 90HAE AEH AHE+ = =
Hay AO vuông góc vi BC.
4.
a) Gi
1
O
; T;
2
O
thng hàng.
Các tam giác cân
1
OMB
3
O MN
có chung góc M suy
ra
13
OMB O MN
1
3
MO
MB
MN MO
=
Tương tự suy ra
13
OMA O MP
1
3
MO
MA
MP MO
=
Vy
//
MB MA
AB PN
MN MP
=
Tương tự ta có
//CD PM
.
Gọi E là giao điểm AB và CD .
T giác AEDP là hình bình hành.
Tacó:
EBC PNM=
;
ECB PMN=
nên
EBC PNM
(g.g)
( )
1
CHUYÊN ĐỀ BỒI DƯỠNG HỌC SINH GIỎI HÌNH HỌC 9
24
EB PN
EC PM
=
Ta có:
PTA PMT=
MPT
chung, nên
PAT PTM
(g.g)
2
.
PA PT
PA PM PT
PT PM
= =
Tương tự, ta có:
2
.PD PN PT=
..PAPM PD PN=
nên
PNM PAD
(c.g.c)
( )
2
Mà APDE là hình bình hành nên
EDA PAD =
( )
3
T
( )
1
, (2),
( )
3
suy ra:
EBC EDA EBC EDA =
Do đó tứ giác ABCD ni tiếp,
b) Gọi giao điểm ca PT và AB là I. Tia IC ct
( )
2
O
ti
D
Ta có:
2
..IA IB IT IC ID
==
suy ra
IDA IIBC BC IDA

=
Do đó tứ giác
ABCD
ni tiếp mà ABCD ni tiếp nên D trùng
D
Vậy các đường thắng AB, CD và PT đồng quy.
5.
a) Ta có EB, EM là tiếp tuyến nên
1
2
EOM BOM=
;
Ta có FC, FM là tiếp tuyến nên
11
22
FOM COM EOF BOC= =
;
Mt khác
11
22
EOF BOC sdBMC

==


Suy ra
EBQ EOQ=
T đó ta có O và B là hai đỉnh liên tiếp cùng nhìn EQ dưới mt góc bng nhau
Vy OBEQ là t giác ni tiếp.
Chứng minh tương tự ta có OCFP là t giác ni tiếp.
b) OBEQ là t giác ni tiếp nên
180 90 90 .OBE OQE OQE FQE+ = = =
OCFP là t giác ni tiếp nên
180 90 90OCF OPF OPF EPF+ = = =
Suy ra
90EPF EQF= =
.
Vy t giác PQFE là t giác ni tiếp.
CHUYÊN ĐỀ BỒI DƯỠNG HỌC SINH GIỎI HÌNH HỌC 9
25
c) K OH vuông góc vi BC.
Ta có: PQFE là t giác ni tiếp
Suy ra
OPQ EFO=
Do đó
OPQ OFE
(g.g)
PQ OH
EF OM
=
Vì điểm A và
( )
O
c định nên OH và OM không đổi do đó tỉ s
PQ
FE
không đổi khi M di chuyn
trên đường tròn.
6. T giác ADOE ni tiếp
EAO EDO=
.
Gi tia BO ct tia DE ti H thì:
180 180 90
2 2 2
A B C
BHD HDB HBD= = =
Mt khác
2
C
ACO =
nên t giác EOCH ni tiếp
90OHC OEC = =
.
Hay BH vuông góc vi CH.
Gọi M là trung điểm ca BC
Suy ra
MB MC MH BHM= =
cân
HBM MHB ABH MHB = =
Suy ra BH song song vi AB.
Suy ra điều phi chng minh.
7.
1. Ta có:
ACD ABD=
;
ABD AFB=
nên
ACD AFB=
.
Do đó tứ giác CDFE ni tiếp.
2. Gi I là tâm đường tròn ngoi tiếp t giác CDFE.
Đưng tròn
( )
I
qua CD nên I thuc trung
trc ca CD.
Đưng tròn
( )
I
qua EF nên I thuc trung
trc ca EF.
Gọi H là trung điểm ca EF.
Do đó I là giao điểm hại đường trung trc
ca CD và EF
CHUYÊN ĐỀ BỒI DƯỠNG HỌC SINH GIỎI HÌNH HỌC 9
26
//AO HI
hoc trùng vi HI (cùng vuông góc vi EF)
( )
1
Tam giác AEF vuông, có AH là trung tuyến ng vi cnh huyn nên
HA HE HAE=
cân ti H
HAE HEA HAE ADC = =
90ADC ACD+ =
nên
90HAE ACD+ =
Suy ra
AH CD
.
OI CD
nên
//AH OI
( )
2
T
( )
1
( )
2
, suy ra t giác AOIH là hình bình hành. Do đó
IH OA R==
. Suy ra I cách EF mt
khoảng không đổi bng R, nên I di động trên đường thng d song song vi EF và cách EF mt
khong bng R.
5. Ta có:
90BFD BED BAD= = =
.
Do đó B, E, D, A, F cùng thuộc một đường tròn đường
kính BD.
Trong tam giác vuông ABC có AM lcà cnh huyn nên
MA MC=
MAC
cân ti M
MAC MCA=
.
Xét đường tròn đi qua năm điểm A, B, E, D, F
Ta có
DE DF=
nên
DF DE DBE DBF= =
Xét:
NAF MAC DAF MCA DBF MCA DBE BDA NFA= + = + = + = =
NAF
cân ti N
NF NA=
.
9.
a) Ta có
BDE BAE=
(cùng chn cung BE ca
đường tròn tâm O)
BDE BMN=
(cùng chn cung BN của đường
tròn tâm
O
)
BDE BMN=
hay
BDI BMN=
T giác
BDMI ni tiếp
MDI MBI=
(cùng chn cung MI)
MDI ABE=
(cùng chn cung AE ca
đường tròn tâm O)
CHUYÊN ĐỀ BỒI DƯỠNG HỌC SINH GIỎI HÌNH HỌC 9
27
ABE MBI=
Mt khác:
BMI BAE=
MBI ABE
(g.g)
..
MI BI
MI BE BI AE
AE BE
= =
b) Gọi Q là giao điểm ca CO và DE.
Ta có
OC DE
ti Q
OCD
vuông tại D , có đường cao là DQ nên
22
.OQOC OD R==
( )
1
Gọi K là giao điểm của hai đường thng
OO
và DE, H là giao điểm ca AB và
OO
Ta có:
OO AB
ti H.
KQO CHO
(
90QH= =
;
O
chung)
..
KO OQ
OC OQ KOOH
CO OH
= =
( )
2
T
( )
1
( )
2
, suy ra:
2
2
.
R
KOOH R OK
OH
= =
Vì OH c định và R không đổi nên OK không đổi. Do đó K cố định.
10. Ta có:
45PIB IAB IBA IBC= + = +
( )
1
Mt khác
( )
1
180
2
PNB CNM ACB= =
( )
1
90 90
2
ACB

= +

( )
1
90
2
ACB= +
1
45 45
2
ABC IBC= + = +
( )
2
T
( )
1
( )
2
, suy ra:
PIB PNB=
.
Do đó 4 điểm P, N, I, B cùng nm trên một đường tròn.
Mt khác
90INB =
nên IB là đường kính của đường tròn này
90IPB =
11.
a) Ni CP, PD .
Ta có A, C, O thng hàng; B, D, O thng hàng.
Ta có:
ACP
,
OAB
lần lượt cân ti C, O nên
CPA CAP OBP==
.
CHUYÊN ĐỀ BỒI DƯỠNG HỌC SINH GIỎI HÌNH HỌC 9
28
Do đó
//CP OD
( )
1
Tương tự, ta có
//OD CP
( )
2
.
T
( )
1
( )
2
suy ra t giác ODPC là hình bình hành.
Gọi H là giao điểm ca CD và MP, K là giao điểm ca CD và OP.
Do đó K là trung điểm ca OP.
Theo tính cht của hai đường tròn ct nhau thì
CD MP
H là trung điểm ca MP.
Do đó
// //HK OM CD OM
.
Gi s
AP BP
.
Vì t giác CDOM là hình bình hành nên
OC DP=
;
2
DP DM R==
nên t giác CDOM là hình
thang cân.
Do đó 4 điểm C, D, O, M cùng thuc một đường tròn.
b) Ta có:
2 2 2 2
2OA OB R AB+ = =
. Do đó
AOB
vuông cân ti O.
Vì 4 điểm C, D, O, M cùng thuc một đường tròn (K c M trùng O) nên
COB CMD=
( )
1
Ta có:
MAB MCD=
(cùng bng
1
2
sd MP
của đường tròn
( )
C
)
MBP MDC=
(cùng bng
1
2
sd MP
của đường tròn
( )
D
).
Do đó
MAB MCD
(g-g)
90AMB AOB = =
CMD COD=
(t giác CDOM ni tiếp).
Do AB c định nên điểm M thuộc đường tròn tâm I đường kính AB.
Ta có:
1
90 45
2
ACP BDP AOB AMP ACP= = = = =
(Góc ni tiếp và góc tâm ca
( )
C
)
1
45
2
BMP BCP = =
(góc ni tiếp và góc tâm ca
( )
D
)
Do đó MP là tia phân giác của
AMB
. Mà
90AMB AOB= =
nên M thuộc đường tròn
( )
I
ngoi
tiếp tam giác AOB. Gi s MP cắt đường tròn
( )
I
tại N và N là trung điểm cung AB không cha
điểm O nên N c định.
c) Ta có:
MPA BPN=
;
AMP PBN=
(góc ni tiếp cùng chn mt cung)
Do đó
MAP BNP
(g - g)
2
22
..
2 4 2
PA PM PA PB AB R
PM PN PA PB
PN PB
+

= = = =


(không đổi)
CHUYÊN ĐỀ BỒI DƯỠNG HỌC SINH GIỎI HÌNH HỌC 9
29
Vy PM.PN ln nht là
2
2
R
khi
PA PB=
hay P là trung điểm ca dây AB. Tam giác AMB vuông
ti M nên:
( )
22
22
11
.
2 4 4 2
AMB
AB R
S AM BM AM BM= + = =
Vy
ABM
S
ln nht là
2
2
R
khi
PA PB=
hay P là trung điểm ca dây AB.
12.
a) EF là đường kính nên
90EAF =
AE MN
suy ra
//AF MN QPN QFA=
.
Mà AFQB ni tiếp nên
180QFA QBA+ =
180QPN QBN + =
.
Suy ra t giác PQBN ni tiếp.
Li có
QCA QFA QPN QCM= =
Suy ra t giác PQCM ni tiếp.
b) Gi s QN và PC ct nhau ti R thuc
( )
O
.
T t giác PQBN ni tiếp suy ra
NPB NQB BCP==
.
T t giác PMCQ ni tiếp ta có:
PBC RPB PCB RPN NPB NPB RPN MPC MQC= = + = = =
T đó nếu QM ct BP tại điểm S thì SBQC ni tiếp hay S thuc
đường tròn
( )
O
.
13.
a) Ta có:
BKA ACB=
(2 góc ni tiếp cùng chn cung AB)
ACB BHK=
(cùng ph vi góc EBC)
BKA BHK=
tam giác BHK cân
BH BK=
Lp luận tương t ta có
CH CK=
BC là trung trc ca HK.
Ta có:
90AEH AFH= =
T giác AFHE ni tiếp.
Xét tam giác AIE và tam giác FIH ta có:
AIE FIH=
(2 góc đối đỉnh),
CHUYÊN ĐỀ BỒI DƯỠNG HỌC SINH GIỎI HÌNH HỌC 9
30
IAE IFH=
(T giác AFHE ni tiếp)
AIE FIH
(g.g)
..
AI FI
AI HI EI FI
EI HI
= =
b) Xét t giác DHEC ta có:
90HDC BEC= =
T giác DHEC ni tiếp .
Xét t giác BFEC ta có:
90BFC BEC= =
T giác BFEC ni tiếp
AFE ACB=
ACB AKB=
(chng minh trên)
AFE AKB =
T giác KBFI ni tiếp .
c) Theo như trên ta đã có:
BKH BHK=
BHK IHE=
(2 góc đối đỉnh)
BKH IHE=
.
Xét tam giác HEI và tam giác KAB ta có:
BKH IHE=
(cmt),
IHE BAK=
(t giác AFHE ni tiếp)
HEI KAB
(g.g)
KB HI
AB EI
=
Chứng minh tương tự ta có:
KC HI
AC FI
=
T đó suy ra
1 1 .
.
..
KB KC EI FI IH EF EF
IH IH
AB AC EI FI EI FI AI HI AI
+

+ = + = = =


(theo chng minh câu a có
..IF IE IH IA=
).
d) Ta có:
BME BKH=
(2 góc v trí đồng v do
//HK ME
)
BKH BHK=
;
BKH BME=
(2 góc v trí đồng v do
//HK ME
)
BME BEM=
Tam giác BEM là tam giác cân.
Ta có:
AD BC
//EM BC EM BC⊥
.
Trong tam giác cân BEM có BC là đường cao ca tam giác
(do
BC ME
)
BC là trung trc ca ME.
Ta có D nm trên đường trung trc ca
ME DM DE=
Tam giác DME là tam giác cân
MDC EDC=
.
Xét t giác ABDE ta có:
90ADB AEB= =
T giác ABDE ni tiếp
EDC BAC=
.
Xét t giác AFDC ta có:
90AFC ADC= =
CHUYÊN ĐỀ BỒI DƯỠNG HỌC SINH GIỎI HÌNH HỌC 9
31
T giác AFDC ni tiếp
BAC BDF=
.
T đó suy ra
MDC BDF=
Ta có:
180 BDC MDC BDM BDF BDM FDM = = + = + =
Ba điểm F, D, M thng hàng.
14.
a) Ta có
ABF
;
ACE
là các tam giác cân ti F và
E
FBA BAD DAC ECA ABF ACE= = =
.
b) Gọi G là giao điểm ca BE và CF.
Ta có:
GF BF AB DB
GC CE AC DC
= = =
//DG BF
Mt khác
//DA BF
suy ra A, D, G thng hàng.
Suy ra điều phi chng minh.
c) Ta có
BQG QGA GAE GAC= = =
GAC CAE GAB BAF GAF= + = + =
Suy ra AGQF là t giác ni tiếp.
Mt khác
QPG GCE GFQ==
nên QGPF là t giác ni tiếp. Suy
ra điều phi chng minh.
5. Gọi R là giao điểm ca BN và CM.
Ta thy
ABC PAC QBA∽∽
Do đó
BQ PA QB PM
QA PC QN PC
= =
.
Mt khác
MPC NQB=
nên
MPC BQN
BNQ PCM=
T giác QCNR ni tiếp
CRN CQN BAC = =
.
Vy t giác ABQR ni tiếp, suy ra điều phi chng minh.
16. Chia lục giác đều ABCDEF tâm O thành 6 tam giác đều
cnh 4cm (hình v).
Theo nguyên lý Điriclê có ít nhất 4 điểm trong 19 điểm nm
trong hay trên cnh một trong 6 tam giác đó. Không mất
tính tng quát gi s tam giác đó là OAB.
CHUYÊN ĐỀ BỒI DƯỠNG HỌC SINH GIỎI HÌNH HỌC 9
32
Chia tam giác đều OAB trng tâm G thành 3 t giác ni tiếp (hình v) vi
GM AB
;
GN OB
;
GP OA
.
OAB
đều cnh bằng 4 có đường cao
4 3 4 3
23
23
GA= =
Các t giác GMBN, GMAP, GPON ni tiếp trong đường tròn đường kính GB, GA, GO đều bng
43
3
Theo nguyên lý Điriclê có ít nhất 2 điểm trong 4 điểm đang xét nằm trong hay trên cnh mt trong
4 t giác nói trên, gi s t giác đó là GMBN
khong cách giữa hai điểm đó không vượt quá đường kính
43
3
GB =
ca đường tròn ngoi
tiếp t giác
điều phi chng minh.
17. Gọi H là giao điểm cúa AN và MD, K là giao điểm cua BN và MC, I là giao điểm ca MN và
CD.
AMD
vuông tại A, AH là đường cao
2
.AM MH MD=
.
BMC
vuông tại B, BK là đường cao
2
.BM MK MC=
.
AM BM=
Do đó
..
MH MK
MH MD MK MC
MC MD
= =
MKH
MDC
KMH
chung,
MH MK
MC MD
=
MKH MDC
(c.g.c)
MKH IDC=
.
T giác MKNH có
90 90 180MKN MHN+ = +
t giác
MKNH ni tiếp
MKH MNH=
, ta có
( )
MNH IDN MKH= =
t giác HNID ni
tiếp
90MIC NHD = =
. Vy
MN CD
.
18.
a) Ta có E, M, O, F thng hàng,
ME MF=
(E, F đối xng
qua M)
EF BC
BEF
cân ti B
BFE FEB=
.
Mt khác
OB OE=
suy ra
OBE
cân ti O
OBE OEB=
.
Ta có
BFE FEB OBE==
CHUYÊN ĐỀ BỒI DƯỠNG HỌC SINH GIỎI HÌNH HỌC 9
33
BEF OBE
(g.g)
2
.
EB EF
EB EF EO
OB EB
= =
b) Không gim tính tng quát xét O nm gia M và F.
D thy
FBD EAB
(g.g)
2
.
EB ED
EB ED EA
EA EB
= =
Ta có
( )
2
..ED EA EF EO E
EO ED
EF
B
AE
= ==
.
Xét
EOD
EAF
EO ED
EA EF
=
,
OED
chung
EOD EAF
(c.g.c)
EOD EAF=
, dẫn đến t giác DAFO ni tiếp. Vy các điểm A, D, O, F cùng thuc một đường
tròn.
c) Ta có
EIB ABI BAI=+
,
ABI IBC=
,
( )
BAI CBE EB EC==
EBI IBC CBE ABI BAI EIB EBI = + = + =
cân ti E
EB EI=
EB EC=
nên
EB EI EC= =
E là tâm đường tròn ni tiếp tam giác IBC.
Do đó
EP EB=
nên
2
.EP EF EO=
.
Xét
EPO
EFP
EP EO
EF EP
=
,
PEO
chung
EPO EFP
(c.g.c)
EPO EFP =
EP là tiếp tuyến của đường tròn ngoi tiếp tam giác POF.
Vy tiếp tuvến của đường tròn ngoi tiếp tam giác POF đi qua điểm E c định.
19.
a) Vì
BE CA
,
CF AB
nên BCEF là t giác ni tiếp
XFB ACB XBF XFB = =
cân ti X.
MFB
cân ti M, suy ra
MX BF
.
b) D thy
MX AB
,
HF AB
nên
//MX HF
;
MS BC
,
HD BC
nên
//MS HD
.
Mt khác, do CAFD là t giác ni tiếp và SB tiếp xúc vi
( )
O
ti B nên
SBD BAC BDF==
Suy ra
//SX FD
. Do đó
MXS HFD
(có cặp canh tươngng
song song).
c) Ta có:
180
90 90
2
AOC
OAE ABC AEF
−
= = =
, suy ra
OA EF
.
D dàng chứng minh được
AEF ABC
;
AFY ACD
.
CHUYÊN ĐỀ BỒI DƯỠNG HỌC SINH GIỎI HÌNH HỌC 9
34
Suy ra
FY AF EF EF BC
CD AC BC FY CD
= = =
20.
1) H là trung điểm ca BC nên
OH BC
.
BMO CNO =
(cgc) suy ra
OM ON=
( )
1
Tam giác OMN cân ti O, I là trung điểm ca MN nên
OI MN
( )
2
.
T
( )
1
( )
2
suy ra 4 điểm O, M, H, I cùng thuộc đường tròn đường kính OM.
2) Chng minh t giác ABON ni tiếp (vì
60 120 180PAN PON+ = + =
)
Suy ra
30OPN OAN= =
Chứng minh tương lự.
30OPM =
, suy ra
60MPN =
.
Kết hp vi
MP NP=
(P thuộc đường trung trc OI cua MN )
Suy ra tam giác MNP đều.
3) T câu 1 suy ra
60IHC IOM ABC= =
nên
//IH AB
.
Suy ra đường thng IH c định. Gọi K là trung điểm ca AC suy ra H, I, K thng hàng.
Ly điểm T đối xng vi A qua HI suy ra T c định.
Ta có
AI BI AB TI IB AB BT AB const+ + = + + + =
.
Chu vi tam giác AIB nh nht bng
BT AB+
, đạt được khi ba điểm B, I,T thng hàng.
Khi đó I là trung điểm ca BT c định (theo tính chất đường trung bình
BAT
).
Suy ra t giác BMTN là hình bình hành và
//TN BC
.
Li có
BH KT=
,
BK MT=
Suy ra t giác BHTK là hình bình hành và
//TK BC
.
T đó
NK
, suy ra
MH
.
21.
1) Chng minh bốn điểm A, E, P, F cùng nm trên một đường tròn
Ta có
AEP ADB=
(chn cung AB ca
( )
ABD
):
Ta có
AFP ADC=
(chn cung AC ca
( )
ADC
);
Nên
AEP AFP ADB ADC+ = +
nên AEFP ni tiếp
2) Gi s đường thng AD cắt đường tròn
( )
O
tại Q khác A, đường thng AF cắt đường thng QC
ti I
Chng minh tam giác ABE đồng dng vi tam giác CLF
CHUYÊN ĐỀ BỒI DƯỠNG HỌC SINH GIỎI HÌNH HỌC 9
35
Xét
ABE
;
CLF
AEB CFL=
(cùng bù
AFP
)
( )
1
ta li có
BAE BAD DAE=+
;
FCL BCL FCB=+
BAD BCL=
;
DAE FCB=
;
Nên
BAE FCL=
( )
2
t
( )
1
( )
2
suy ra
ABE CLF
(g.g)
3) Gọi K là giao điểm của đường thẳng AE và đường
thng QB chng minh
QKL PAB QLK PAC+ = +
Theo 2)
ABE CLF
nên
..LF AE BECF=
Ta li có
..KE AF BECF=
Suy ra
..KE AF LF AE=
//
LF KE
EF LK
AF AE
=
Nên
AEF AKL=
AEF APF APF AKL= =
Nên
PAC PCA EKP QKL+ = +
PCA EKP PAC QKL= =
.
Tương t
PAB QLK=
, suy ra
QKL PAB QLK PAC+ = +
.
22.
a) Ta có
( )
1
2
MNE sd AB sd BEF=+
( )
1
2
sd AC sdBEF=+
( )
1
2
AFE sd AB sdBE=+
Suy ra:
180MNE MFE+ =
Vy t giác MNEF ni tiếp.
b) Gi P là giao điểm khác A ca AO với đường tròn
( )
;OR
.
Ly G đối xng vi E qua AP
D EG
,
( )
GO
Ta có
MDG NEG=
,
AEG AFG MDG MFG= =
Suy ra t giác MDGF ni tiếp
( )
1
Gọi giao điểm ca AGBCH.
Chứng minh tương tự a) có t giác MHGF ni tiếp
( )
2
CHUYÊN ĐỀ BỒI DƯỠNG HỌC SINH GIỎI HÌNH HỌC 9
36
T
( )
1
( )
2
suy ra các điểm M, H, D, G, F nm trên một đường tròn.
Trung trc của đoạn thng FG đi qua O và cắt đường tròn
( )
O
ti J;
I OJ
,
sdJF sdJG=
sđ PG sđ PE=
nên
JOP
=
hay I nằm trên đường thng c định. Đó là đường thẳng đi qua O
to vi AO mt góc
không đổi.
c) H
( )
IT BC T BC TH TM =
.
Gi Q là giao điểm ca BC và AP.
Do
QH QN=
Suy ra
QT QH MH=−
( )
11
22
NM MH MN−=
K
1
2
IS AP IS QT MN = =
Tam giác vuông OSI
IOS
=
không đổi nên OI nh nht khi và ch khi IS nh nht
MN nh
nht.
Ta chng minh MN nh nht khi và ch khi tam giác AMN cân ti A.
Tht vy, trên BC ly
MN

sao cho
MAN

=
.
Không mt tính tng quát gi s
QM QN

Suy ra
AM AN

.
Trên đoạn
AM
lấy điểm U sao cho
AU AN
=
'AUM ANN =
(c.g.c)
AMM ANN
S S MM NN MN MN

Vi
60
=
;
BC R=
suy ra
( )
23
3
22
R
R
AQ R
= =
( ) ( ) ( )
2 3 2 3 3 2 3 3
2
.
2 3 6
3
R R R
MN OI
= = =
23.
1. I là trung điểm của BC (Dây BC không đi qua O)
90OI BC OIA =
Ta có
90AMO =
,
90ANO =
Suy ra 4 điểm O, M, N, I cùng thuộc đường tròn đường kính OA.
2. AM, AN là hai tiếp tuyến ca
( )
O
nên OA là phân giác
MON
MON
cân O nên
OA MN
ABN ANC
(Vì
ANB ACN=
,
CAN
chung)
CHUYÊN ĐỀ BỒI DƯỠNG HỌC SINH GIỎI HÌNH HỌC 9
37
2
.
AB AN
AB AC AN
AN AC
= =
ANO
vuông tại N đường cao NH nên
2
.AH AO AN=
..AB AC AH AO=
AHK AIO
(g-g) nên
AH AK
AI AO
=
..AI AK AH AO=
..AI AK AB AC=
.AB AC
AK
AI
=
Ta có A, B, C c định nên I c định
AK
c định
Mà A c định, K là giao điểm ca dây BC và dây MN nên K thuc tia AB
K c định
3) Ta có
90PMQ =
Ta có:
MHE QDM
(g-g)
ME MH
MQ DQ
=
Ta có:
PMH MQH
2
MP MH MH
MQ QH DQ
= =
1
.2
2
MP ME
ME MP
MQ MQ
= =
P là trung điểm ME.
24.
Ta có:
90EBO =
(vì AB là tiếp tuyến vi (O) ti B)
90EDO =
(gt)
hai đỉnh B và D cùng nhìn đoạn OE dưới mt góc vuông.
t giác EBOD là t giác ni tiếp đường tròn.
CHUYÊN ĐỀ BỒI DƯỠNG HỌC SINH GIỎI HÌNH HỌC 9
38
BEO BDO=
(1) (cùng chn cung OB).
Chứng minh tương tự ta có t giác ODCF ni tiếp đường tròn.
OFC BDO=
(2) (Góc trong một đỉnh bng góc ngoài tại đỉnh đối din).
T (1) và (2)
OFC BEO=
Suy ra t giác AEOF là t giác ni tiếp đường tròn (theo du hiu góc trong một đỉnh bng góc
ngoài tại đỉnh đối din).
Vy t giác AEOF là t giác ni tiếp.
25.
Xét t giác AMOQ có
180AMO AQO+ =
. Suy ra t giác AMOQ ni tiếp.
11
MA=
(cùng chn
OQ
).
Chứng minh tương tự
21
MB=
.
11
QMN A B = +
Chứng minh tương tự:
1 1 2 1 1 1
;D P P C QPN D C= = = +
1 1 1 1
180QMN QPN A B C D + = + + + =
Vy t giác MNPQ là t giác ni tiếp.
CHUYÊN ĐỀ BỒI DƯỠNG HỌC SINH GIỎI HÌNH HỌC 9
39
26.
Lấy M là trung điểm OO’. Hạ
MH AB
.
Vì AB là tiếp tuyến chung của (O), (O’) suy ra
OA AB
O B AB
OA // O’B
( )
AB
(T vuông góc đến song song).
Suy ra t giác ABO’O là hình thang.
Mà M là trung điểm OO’, MH // OA
( )
AB
H là trung điểm AB.
Li có
HM AB MA MB =
Chứng minh tương tự ta có
MC MD=
12
OO=
ịnh lí đường tiếp tuyến chung của hai đường tròn).
∆AOM = ∆COM (c.g.c)
MA MC=
(hai cạnh tương ứng)
MA MB MC MD = = =
Vy ABCD là t giác ni tiếp.
27.
CHUYÊN ĐỀ BỒI DƯỠNG HỌC SINH GIỎI HÌNH HỌC 9
40
a) Xét ∆MDB và ∆MCD có:
DMB
chung;
MDC MBD=
(góc to bi tiếp tuyến và dây cung, góc ni tiếp chn cung BD)
∆MDB ~ ∆MCD (g.g)
2
.
MC MD
MB MC MD
MD MB
= =
b) Ta có:
HB HC OH BC=
MD OD
(tính cht tiếp tuyến)
90OHM ODM = =
H, D nằm trên đường tròn đường kính OM
t giác OHDM ni tiếp
OMH ODH=
OMH HBP=
(so le trong) nên
HBP HDP=
bốn điểm B, H, D, P cùng nằm trên đường tròn đường kính OM hay t giác BHPD là t giác ni
tiếp.
c) Vì t giác BDPH ni tiếp nên
BHD BPD=
(góc ni tiếp chn cung
BD
).
Vì EF // BP
BPD EOD=
ng v) mà
EOD AOF=
ối đỉnh).
Suy ra
BHD AOF=
Li có
DBH OAF=
(góc ni tiếp chn cung
CD
)
Suy ra ∆OAF ~ ∆HBD (g.g)
OA FO
HB HD
=
(1).
Ta có:
180 180CHD BHD AOF AOE= = =
EAO HCD=
(góc ni tiếp chn cung
BD
)
CHUYÊN ĐỀ BỒI DƯỠNG HỌC SINH GIỎI HÌNH HỌC 9
41
Suy ra ∆OAE ~ ∆HCD (g.g)
OA OE
HC HD
=
(2)
Li có
BH HC=
(3)
T (1), (2), (3) suy ra
FO OE
OE FO
HD HD
= =
O là trung điểm ca FE.
Vậy O là trung điểm ca FE.
28.
a) Vì
540A B C EKD EKI IKD+ + + + + =
.
180B EKI C IKD+ = + =
. Nên
180A EKD+ =
.
Suy ra t giác AEKD ni tiếp.
Mt khác, t giác AEHD ni tiếp.
Vậy các điểm A, E, H, K, D cùng nằm trên đường tròn đường kính AH, nên
BDK CEK=
(đpcm)
b) Ta có:
,ADE ABC AKE ADE==
, suy ra
AKE ABC=
.
T đó
180AKE EKI+ =
, vậy ba điểm A, K, I thng hàng.
Li có
,,IKC IDC ICD IKC KAC ACK ICD ICK KCD= = = + = +
suy ra
KAD IKC=
KAD KED=
Vy t giác MEKC ni tiếp
T giác MEKC ni tiếp nên
MEC MKC=
.
CHUYÊN ĐỀ BỒI DƯỠNG HỌC SINH GIỎI HÌNH HỌC 9
42
; 90 ;IKC AED MEB MEC MEB MKC MKI IKC= = = + = +
90MKI =
Do A, E, H, K, D nằm trên đường tròn đường kính AH, nên
90HKA =
.
Vy K, H, M thng hàng.
c) Do t giác DEHK ni tiếp, nên
HEK HDK=
(1)
T giác MEKC ni tiếp nên
KEC KMC=
(2)
T (1) và (2) suy ra
KMB HDK=
, hay t giác MBKD ni tiếp (đpcm)
Chú ý:
S dng kiến thc của tam giác đồng dng ta thy: Nếu hai cát tuyến AB và CD ca một đường
tròn ct nhau ti M thì
..MAMB MC MD=
29.
CHUYÊN ĐỀ BỒI DƯỠNG HỌC SINH GIỎI HÌNH HỌC 9
43
a) Ta có:
ECD AMC ACD==
. Tương tự
CDE CDA=
.
T đó CD là trung trực ca AE.
b) D thy
180
BCD BAC
CBD BCD BDC
BDC BAD
=
=
=
180 180CBD CAD CED = =
BCED ni tiếp.
c) Gọi K là giao điểm ca AB và CD.
Nhn thy:
2
2
~.
~
.
BCK CAK KC KB KA
KC KD
DBK ADK
KD KB KA
=
=

=
ΔΔ
ΔΔ
CD // PQ
KD KC
AQ AP
AQ AP
= =
.
AE PQ⊥
EPQ cân ti E.
30.
a) Chng minh t giác BECF ni tiếp.
Do t giác MBFD ni tiếp nên ta d dàng chứng minh được
..AB AF AM AD=
Tương tự, t giác MECD ni tiếp nên chứng minh được
..AE AC AM AD=
.
CHUYÊN ĐỀ BỒI DƯỠNG HỌC SINH GIỎI HÌNH HỌC 9
44
Do đó
..AB AF AE AC=
. T đây ta chứng minh được BECF là t giác ni tiếp.
b) T giác ABDC ni tiếp nên ta có
FBD ACD=
(1)
Li có
AMB CMD BFD DEC= =
(2)
T (1) và (2) suy ra ∆ECD ~ ∆FBD
BDF EDC=
BDF BMF
EDC EMC
=
=
nên
BMF EMC=
.
Do vậy 3 điểm E, M, F thng hàng.
31.
a) D thy OM là trung trc ca AB nên OM vuông góc với AB. Ta có IK là đường trung bình ca
tam giác AMC nên IK // AC, t đó ta có ngay IK vuông OM.
Gọi J là giao điểm ca IK và OM. S dụng định lý Py-ta-go ta có:
( ) ( )
2 2 2 2 2 2 2 2
KO KM OJ JK JM JK OJ JM = + + =
2 2 2 2 2 2 2
OI MI OA AI MI OA R= = + = =
b) Gi s G, H là giao điểm ca KO vi (O). Do t giác ADGH ni tiếp nên
KDG AHG=
CHUYÊN ĐỀ BỒI DƯỠNG HỌC SINH GIỎI HÌNH HỌC 9
45
∆KDG ~ ∆KHA (g.g)
KD KG
KH KA
=
( )( )
2 2 2
..KAKD KG KH KO R KO R KO R KM = = + = =
2
.
KA KG
KA KD KM
KM KD
= =
∆KDM ~ ∆KMA (c.g.c)
Suy ra
DMK MAK=
.
Xét đường tròn (O) ta có:
MAK ABD=
.
Xét t giác MDCB có:
DMC DBC=
MDCB là t giác ni tiếp.
c)
MAD AED=
Theo câu b)
DMK DAM AED= =
KM // AE ( vì
DMK
AEM
là hai góc so le trong).
T đây suy ra MAKE là hình thang
INF AEN=
.
Li có
AEN IAF INF AEN IAF= = =
. Suy ra ANFI là t giác ni tiếp (đpcm).
| 1/652